You are on page 1of 653

Scanned by CamScanner

फ्री study हे तू आज ही TELIGRAM APPS download करे


(Google play फ्री study हे तू आज ही TELIGRAM install करे
(google play store ) LOGIN करे & OPEN करे SEARCH
OPTIONS मे “MEENA" type करे फिर एक link show करे गा
जजसे टच करे फिर join पर click करके ग्रप
ू मे जड
ु सकते है

ग्रप
ू मे उपलब्ध सामग्री निम्ि प्रकार है
News PAPER /EMPLOYMENT NEWS/Current affairs /Bbc
news/Hindu vocabulary /All book competition /Upsc ssc
notes/All ncert/ignou/vardman uni/bed/engineering/Medical
/computer science almost 10,000 books available in group

िये TELIGRAM INSTALL करिे के ललए यहााँ जललक करें ▶️ TELIGRAM


यदि पहले से TELIGRAM है तो निचे िीली लाईि टच करे ओर ग्रप
ू मे जड
ु े

STUDY ALL IN ONE


NEWSPAPERS
MOVIE & NOVEL
EMEMPLOYMENT NEWS
फ्री study हे तू आज ही TELIGRAM APPS download करे
(Google play फ्री study हे तू आज ही TELIGRAM install करे
(google play store ) LOGIN करे & OPEN करे SEARCH
OPTIONS मे “MEENA" type करे फिर एक link show करे गा
जजसे टच करे फिर join पर click करके ग्रप
ू मे जड
ु सकते है

ग्रप
ू मे उपलब्ध सामग्री निम्ि प्रकार है
News PAPER /EMPLOYMENT NEWS/Current affairs /Bbc
news/Hindu vocabulary /All book competition /Upsc ssc
notes/All ncert/ignou/vardman uni/bed/engineering/Medical
/computer science almost 10,000 books available in group

िये TELIGRAM INSTALL करिे के ललए यहााँ जललक करें ▶️ TELIGRAM


यदि पहले से TELIGRAM है तो निचे िीली लाईि टच करे ओर ग्रप
ू मे जड
ु े

STUDY ALL IN ONE


NEWSPAPERS
MOVIE & NOVEL
EMEMPLOYMENT NEWS
y
o
u
rs
m
a
h
b
o
o
b
.w
o
rd
p
re
s
s
.c
o
m
SBI/ IBPS
BANK PO
SOLVED PAPERS
Must for SBI/ IBPS PO/ SO/ Insurance/ RRB/ RBI Exams
y
o
u
rs
m
a
h
b
o
• Head Office : B-32, Shivalik Main Road, Malviya Nagar, New Delhi-110017

o
b
.w
• Sales Office : B-48, Shivalik Main Road, Malviya Nagar, New Delhi-110017

o
rd
Tel. : 011-26691021 / 26691713

p
re
s
s
.c
o
m
Typeset by Disha DTP Team

DISHA PUBLICATION
ALL RIGHTS RESERVED

© Copyright Publisher

No part of this publication may be reproduced in any form without prior permission of the publisher. The author and the
publisher do not take any legal responsibility for any errors or misrepresentations that might have crept in. We have
tried and made our best efforts to provide accurate up-to-date information in this book.

For further information about the books from DISHA,


Log on to www.dishapublication.com or email to info@dishapublication.com
y
o
u
rs
m
a
h
b
o
o
b
CONTENT

.w
o
rd
p
re
s
s
.c
o
m
1. SBI Rural Business PO Exam 2010 1-26

2. SBI & its Associates Bank PO Exam 2010 27-50

3. SBI PO Exam 2011 51-72

4. SBI & its Associates PO Exam 2011 73-94

5. IBPS PO/MT Exam 2011 95-124

6. IBPS PO/MT Exam 2012 125-150

7. IBPS Specialist Officer Exam 2012 151-162

8. Indian Overseas Bank PO Online Exam 2013 163-180

9. IBPS RRBs Officer Scale-I Exam 2013 181-200

10. IBPS RRBs Officer Scale-II & III CWE Exam 2013 201-220

11. SBI PO Exam 2013 221-244

12. IBPS Specialist (I.T.) Officer Exam 2013 245-266

13. IBPS Bank PO/MT CWE Exam 2013 267-290

14. IBPS Bank PO/MT CWE Exam 2014 291-316

15. SBI Management Executive Exam 2014 317-336

16. SBI Specialist Officer Online Exam 2014 337-348

17. SBI PO Exam 2014 349-376

18. IBPS Specialist (I.T.) Officer Exam 2014 377-400

19. Corporation Bank Specialist Officer Exam 2014 401-418


y
o
u
rs
m
a
h
b
o
o
20. IBPS CWE RRB Officer Scale-I Exam 2014 419-436

b
.w
o
21. IBPS Specialist (I.T.) Officer exam 2015 437-456

rd
p
re
22. IBPS CWE-RRB Officer Scale-I Exam 2015 457-474

s
s
.c
o
m
23. IBPS PO Prelim Exam 2015 475-484

24. IBPS PO Main Exam 2015 485-506

25. SBI Bank PO Prelim Exam 2015 507-518

26. SBI Bank PO Main Exam 2015 519-540

27. IBPS Specialist IT Officer Exam 2016 541-564

28. SBI Bank PO Prelim Exam 2016 565-576

29. SBI Bank PO Main Exam 2016 577-594

30. IBPS PO Prelim Exam 2016 595-606

31. IBPS PO Main Exam 2016 607-628

32. IBPS RRB Officer Scale I Exam 2016 629-644


y
o
u
rs
m
a
h
b
o
o
SBI RURAL BUSINESS PO EXAM 2010

b
.w
o
rd
p
Based on Memory

re
s
s
.c
o
m
REASONING ABILITY 8. If ‘¸’ means ‘+’; ‘–’ means ‘×’; ‘x’ means ¸ and '+' means '–
'; then –
1. In a certain code language 'how many goals scored' is written 15 – 8 × 6 ¸ 12 + 4 = ?
as '5 3 9 7'; 'many more matches' is written as '9 8 2' and 'he (a) 20 (b) 28
scored five' is written as ' 1 6 3'. How is 'goals' written in that
code language ? 4 2
(c) 8 (d) 2
(a) 5 (b) 7 7 3
(c) 5 or 7 (d) Data inadequate (e) None of these
(e) None of these 9. Town D is towards East of town F. Town B is towards North
2. In a certain code TEMPORAL is written as OLDSMBSP. of town D. Town H is towards South of town B. Towards
How is CONSIDER written in that code ? which direction is town H from town F ?
(a) RMNBSFEJ (b) BNMRSFEJ (a) East (b) South-East
(c) RMNBJEFS (d) TOPDQDCH
(c) North-East (d) Data inadequate
(e) None of these
(e) None of these
3. How many meaningful English words can be made with the
letters DLEI using each letter only once in each word ? 10. How many such pairs of letters are there in the word
(a) None (b) One SEARCHES each of which has as many letters between
(c) Two (d) Three them in the word as in the English alphabet ?
(e) More than three (a) None (b) One
4. Among A, B, C, D and E each having different weight, D is (c) Two (d) Three
heavier than only A and C is lighter than B and E. Who (e) More than three
among them is the heaviest ?
DIRECTIONS (Qs. 11 - 15) : In each of the questions below are
(a) B (b) E
given four statements followed by four conclusions numbered I.
(c) C (d) Data inadequate
II, III and IV. You have to take the given statements to be true
(e) None of these even if they seem to be at variance from commonly known facts.
5. Each odd digit in the number 5263187 is substituted by the Read all the conclusions and then decide which of the given
next higher digit and each even digit is substituted by the
previous lower digit and the digits so obtained are rearranged conclusions logically follows from the given statements
in ascending order, which of the following will be the third disregarding commonly known facts.
digit from the left end after the rearrangement? 11. Statements :
(a) 2 (b) 4 All cups are bottles.
(c) 5 (d) 6 Some bottles are jugs.
(e) None of these No jug is plate.
6. Pratap correctly remembers that his mother's birthday is Some plates are tables.
before twenty third April but after nine teenth April, whereas
his sister correctly remembers that their mother's birthday is Conclusions :
not on or after twenty second April. On which day in April is I. Some tables are bottles.
definitely their mother's birthday ? II. Some plates are cups.
(a) Twentieth III. No table is bottle.
(b) Twenty-first IV. Some jugs are cups.
(c) Twentieth or twenty-first (a) Only I follows
(d) Cannot be determined
(b) Only II follows
(e) None of these
(c) Only III follows
7. Ashok started walking towards South. After walking
50 metres he took a right turn and walked 30 metres. He then (d) Only IV follows
took a right turn and walked 100 metres. He again took a (e) Only either I or III follows
right turn and walked 30 metres and stopped. How far and in 12. Statements:
which direction was he from the starting point ? Some chairs are handles.
(a) 50 metres South (b) 150 metres North All handles are pots.
(c) 180 metres East (d) 50 metres North All pots are mats.
(e) None of these Some mats are buses.
y
o
u
rs
m
2 SBI Rural Business PO Exam 2010

a
h
b
Conclusions : DIRECTIONS (Qs. 16 - 20): Study the following information

o
I. Some buses are handles.

o
carefully and answer the questions given below :

b
II. Some mats are chairs.

.w
A, B, C, D. E. F. G and H are eight employees of an organization
III. No bus is handle. working in three departments viz. Personnel, Administration and

o
rd
IV. Some mats are handles. Marketing with not more than three of them in any department.

p
(a) Only I, II and IV follow Each of them has a different choice of sports from football, cricket,

re
volleyball, badminton, lawn tennis, basketball, hockey and table

s
(b) Only II, Ill and IV follow

s
tennis not necessarily in the same order.

.c
(c) Only either I or III and II follow D works in Administration and does not like either football or

o
(d) Only either I or III and IV follow

m
cricket. F works in Personnel with only A who likes table tennis.
(e) Only either I or III and II and IV follow E and H do not work in the same department as D. C likes hockey
13. Statements : and does not work in marketing. G does not work in administration
All birds are horses. and does not like either cricket or badminton. One of those who
work in administration likes football. The one who likes volleyball
All horses are tigers. works in Personnel. None of those who work in Administration
Some tigers are lions. likes either badminton or lawn tennis. H does not like cricket.
Some lions are monkeys. 16. Which of the following groups of employees work in
Conclusions : Administration department ?
I. Some tigers are horses. (a) EGH (b) AF
II. Some monkeys are birds. (c) BCD (d) BGD
III. Some tigers are birds. (e) Data inadequate
IV. Some monkeys are horses. 17. In which department does E work ?
(a) Only I and III follow (a) Personnel (b) Marketing
(c) Administration (d) Data inadequate
(b) Only I, II and III follow
(e) None of these
(c) Only II, III and IV follow
18. Which of the following combinations of employee-
(d) All I, II, III and IV follow department favourite sport is correct ?
(e) None of these (a) E Administration -Cricket
14. Statements : (b) F Personnel -Lawn Tennis
Some benches are walls. (c) H Marketing -Lawn Tennis
All walls are houses. (d) B Administration -Table Tennis
Some houses are jungles. (e) None of these
All jungles are roads. 19. What is E's favourite sport ?
Conclusions : (a) Cricket (b) Badminton
I. Some roads are benches. (c) Basketball (d) Lawn Tennis
II. Some jungles are walls. (e) None of these
III. Some houses are benches. 20. What is G's favourite sport ?
IV. Some roads are houses. (a) Cricket (b) Badminton
(c) Basketball (d) Lawn Tennis
(a) Only I and II follow
(e) None of these
(b) Only 1 and III follow
(c) Only III and IV follow DIRECTIONS (Qs. 21 - 25) : In the following questions, the
symbols @, $, «, # and d are used with the following meaning
(d) Only II, III and IV follow
as illustrated below :
(e) None of these
'P $ Q' means 'P is not smaller than Q'.
15. Statements :
'P @ Q' means 'P is neither smaller than nor equal to Q'.
Some sticks are lamps.
'P # Q' means 'P is neither greater than nor equal to Q'.
Some flowers are lamps. 'P d Q' means P is neither greater than nor smaller than Q'.
Some lamps are dresses. 'P « Q' means 'P is not greater than Q'.
All dresses are shirts. Now in each of the following questions assuming the given
Conclusions : statements to be true, find which of the four conclusions I, II, Ill
I. Some shirts are sticks. and IV given below them is/are definitely true and give your
II. Some shirts are flowers. answer accordingly.
III. Some flowers are sticks. 21. Statements :
H @ T, T # F, F d E, E « v
IV. Some dresses are sticks.
Conclusions : I. V $ F
(a) None follows (b) Only I follows
II. E @ T
(c) Only II follows (d) Only III follows
III. H @ V
(e) Only IV follows
IV. T # V
y
o
u
rs
m
SBI Rural Business PO Exam 2010 3

a
h
(a) Only I, II and III are true (ii) have post qualification work experience of at least ten

b
o
(b) Only I, II and IV are true years in the Advances Section of a bank.

o
b
(c) Only II, Ill and IV are true (iii) be at least 30 years and not more than 40 years as on

.w
(d) Only I, III and IV are true 01.04.2010.

o
rd
(e) All I, II, III and IV are true (iv) have secured at least 40 percent marks in the group
discussion,

p
22. Statements :

re
D # R, R« K, K @ F, F $ J (v) have secured at least 50 percent marks in interview.

s
s
In the case of a candidate who satisfies all the

.c
Conclusions : 1. J # R
conditions EXCEPT–

o
II. J # K

m
(A) at (i) above but has secured at least 50 percent marks in
III. R # F
graduation and at least 60 percent marks in post
IV. K @ D graduation in any discipline the case is to be referred
(a) Only I, II and III are true to the General Manager-Advances.
(b) Only II, III and IV are true (B) at (ii) above but has total post qualification work
(c) Only I, III and IV are true experience of at least seven years out of which at least
(d) All I, II, III and IV are true three years as Manager-Credit in a bank, the case is to
(e) None of these be referred to Executive Director.
23. Statements : In each question below details of one candidate is given.
N d B, B $ W, W # H, H « M You have to take one of the following courses of action
Conclusions : I. M @ W based on the information provided and the conditions and
sub-conditions given above and mark the number of that
II. H @ N
course of action as your answer. You are not to assume
III. W d N anything other than the information provided in each
IV. W # N question. All these cases are given to you as on 01.04.2010.
(a) Only I is true Give answer (a) if the case is to be referred to Executive
(b) Only III is true Director.
(c) Only IV is true Give answer (b) if the case is io be referred to General
(d) Only either III or IV is true Manager-Advances.
(e) Only either III or IV and I are true Give answer (c) if the data are inadequate to take a decision.
24. Statements : Give answer (d) if the candidate is not to be selected.
R « D, D $ J, J # M, M @ K Give answer (e) if the candidate is to be selected.
Conclusions : 26. Shobha Gupta has secured 50 percent marks in the interview
I. K # J and 40 percent marks in the Group Discussion. She has
II. D @ M been working for the past eight years out of which four
III. R # M years as Manager-Credit in a bank after completing her B.A.
IV. D @ K degree with 60 percent marks. She was born on 12th
September 1978.
(a) None is true (b) Only I is true
27. Rohan Maskare was born on 8th March 1974. He has been
(c) Only II is true (d) Only III is true
working in a bank for the past twelve years after completing
(e) Only IV is true his B.Com. degree with 70 percent marks. He has secured
25. Statements : 50 percent marks in both the Group Discussion and the
M $ K, K @ N, N « R, R # W interview.
Conclusions : I. W @ K 28. Prakash Gokhale was born on 4th August 1977. He has
II. M $ R secured 65 percent marks in post graduation and 58 percent
III. K @ W marks in graduation. He has been working for the past ten
IV. M @ N years in the Advances Department of a bank after completing
(a) Only I and II are true his post graduation. He has secured 45 percent marks in the
Group Discussion and 50 percent marks in the Interview.
(b) Only I, II and III are true
29. Sudha Mehrotra has been working in the Advances
(c) Only III and IV are true
department of a bank for the past twelve years after
(d) Only II, III and IV are true completing her B.Com. degree with 60 percent marks. She
(e) None of these has secured 50 percent marks in the Group Discussion and
DIRECTIONS ( Qs. 26 - 30): Study the following information 40 percent marks in the Interview. She was born on 15th
carefully and answer the questions given below : February 1972.
Following are the conditions for selecting Senior Manager-Credit 30. Amit Narayan was born on 28th May 1974. He has been
in a bank. The candidate must – working in the Advances department of a bank for the past
eleven years alter completing his B.Sc. degree with 65
(i) be a graduate in any discipline with atleast 60 percent
percent marks. He has secured 55 percent marks in the Group
marks.
discussion and 50 percent, marks in the interview.
y
o
u
rs
m
4 SBI Rural Business PO Exam 2010

a
h
Courses of action :

b
DIRECTIONS (Qs. 31 - 35) : In each question below is given a

o
(A) The Government should put up a mechanism to provide

o
statement followed by three courses of action numbered (A), (B)

b
and (C). A course of action is a step or administrative decision food grains to the poor people in these districts through

.w
to be taken for improvement, follow-up or further action in regard public distribution system to encourage the parents to

o
send their wards to school.

rd
to the problem, policy, etc. On the basis of the information given
(B) The Government should close down some of these

p
in the statement, you have to assume everything in the siatement

re
to be true, then decide which of the suggested courses of action schools in the district and deploy the teachers of these

s
logically follow(s) for pursuing. schools to nearby schools and also ask remaining

s
.c
students to join these schools.
31. Statement: A heavy unseasonal downpour during the last

o
(C) Government should issue arrest warrants for all the

m
two days has paralysed the normal life in the state in which
parents who force their children to work in fields instead
five persons were killed but this has provided a huge relief
of attending classes.
to the problem of acute water crisis in the state.
(a) Only (A) (b) Only (B)
Courses of action :
(c) Only (C) (d) Only (A) and (B)
(A) The state government should set up a committee to
(e) None of these
review the alarming situation.
35. Statement : One aspirant was killed due to stampede while
(B) The state government should immediately remove all
participating in a recruitment drive of police constables.
the restrictions on use of potable water in all the major
cities in the state. Courses of action :
(C) The state government should send relief supplies to (A) The officials in charge of the recruitment process
should immediately be suspended.
all the affected areas in the state.
(B) A team of officials should be asked to find out the
(a) None (b) Only (A)
circumstances which led to the death of the aspirant
(c) Only (B) and (C) (d) Only (C) and submit its report within a week.
(e) All (A), (B) and (C) (C) The Government should ask the home department to
32. Statement : A large private bank has decided to retrench stagger the number of aspirants over more number of
one-third of its employees in view of the huge losses days to avoid such incidents in future.
incurred by it during the past three quarters. (a) Only (A) (b) Only (B)
Courses of action :
(c) Only (C) (d) Only (B) and (C)
(A) The Government should issue a notification to general
(e) None of these
public to immediately stop all transactions with the
bank. 36. Effect: Majority of the employees of the ailing organization
opted for voluntary retirement scheme and left the
(B) The Government should direct the bank to refrain from
organization with all their retirement benefits within a
retrenching its employees.
fortnight of launching the scheme.
(C) The Government should ask the central bank of the
Which of the following can be a probable cause of the above
country to initiate an enquiry into the bank's activities
effect ?
and submit its report.
(a) None (b) Only (A) (a) The company has been making huge losses for the
past five years and is unable to pay salary to its
(c) Only (B) (d) Only (C)
employees in time.
(e) Only (A) and (C)
(b) The management of the company made huge personal
33. Statement : Many political activists have decided to stage gains through unlawful activities.
demonstrations and block traffic movement in the city during
peak hours to protest against the steep rise in prices of (c) One of the competitors of the company went bankrupt
essential commodities. last year.
Courses of action: (d) The company owns large tracts of land in the state
which will fetch huge sum to its owners.
(A) The Government should immediately ban all forms of
agitations in the country. (e) None of these
(B) The police authority of the city should deploy 37. Statement: Most of the companies in IT and ITES sectors in
additional forces all over the city to help traffic India have started hiring from engineering college campuses
movement in the city. this year and are likely to recruit much more than yearly
(C) The state administration should carry out preventive recruitment of the earlier years.
arrests of the known criminals staying in the city. Which of the following substantiates the facts stated in the
(a) Only (A) (b) Only (B) above statement ?
(c) Only (C) (d) Only (A) and (B) (a) IT and ITES are the only sectors in India which are
(e) None of these hiring from engineering college campuses.
34. Statement: The school dropout rate in many districts in the (b) Government has stepped up recruitment activities after
a gap of five years.
state has increased sharply during the last few years as the
parents of these children make them work in the fields owned (c) The IT and ITES companies have now decided to visit
by others to earn enough for them to get at least one meal a the engineering college campuses for tier II cities in
day. India as well.
y
o
u
rs
m
SBI Rural Business PO Exam 2010 5

a
h
(d) Availability of qualified engineers will substantially (a) India's fiscal deficit is negligible in comparison to other

b
o
increase in the near future. emerging economies in the world.

o
b
(e) None of these (b) Subsidy on food and fertilizers are essential for growth

.w
38. Cause : The Government has recently increased its taxes on of Indian economy.

o
petrol and diesel by about 10 percent. (c) Reform in financial sector will weaken India's position

rd
Which of the following can be a possible effect of the above in the international arena.

p
re
cause ? (d) Gradual withdrawal of subsidy is essential for

s
effectively managing fiscal deficit in India.

s
(a) The petroleum companies will reduce the prices of

.c
petrol and diesel by about 10 percent. (e) None of these

o
m
(b) The petroleum companies will increase the prices of 42. Which of the following is an assumption which is implicit in
petrol and diesel by about 10 percent. the facts stated in the above paragraph ?
(c) The petroleum companies will increase the prices of (a) People in India may not be able to pay more for
petrol and diesel by about 5 percent. petroleum products.
(d) The petrol pumps will stop selling petrol and diesel till (b) Many people in India are rich enough to buy petroleum
the taxes are rolled back by the government. products at market cost.
(e) None of these (c) Government may not be able to create more
39. Statement : The Government has decided to instruct the infrastructural facilities if the present level of subsidy
banks to open new branches in such a way that there is one continues for a longer time.
branch of any of the banks in every 'village of population (d) Government of India has sought assistance from
1000 and above or a cluster of villages with population less international financial organizations for its
than 1000 to provide banking services to all the citizens. infrastructural projects
Which of the following will weaken the step taken by the (e) None of these
Government ? DIRECTIONS (Qs. 43 - 45): Study the following information
(a) The private sector banks in India have stepped up their carefully and answer the questions given below :
branch expansion activities in rural India.
Poverty measurement is an unsettled issue, both
(b) Many Government owned banks have surplus man- conceptually and methodologically. Since poverty is a process as
power in its urban branches. well as an outcome; many come out of it while others may be
(c) All the banks including those in private sector will falling into it. The net effect of these two parallel processes is a
follow the government directive. proportion commonly identified as the 'head count ratio', but these
(d) Large number of branches of many government owned ratios hide the fundamental dynamism that characterises poverty
banks in the rural areas are making huge losses every in practice. The most recent poverty reestimates by an expert
year due to lack of adequate business activities. group has also missed the crucial dynamism. In a study conducted
(e) None of these on 13,000 households which represented the entire country in
1993-94 and again on 2004-05, it was found that in the ten-year
DIRECTIONS (Qs. 40 - 42): Study the following information
period 18.2% rural population moved out of poverty whereas
carefully and answer the questions given below :
another 22.1% fell into it over this period. This net increase of
The centre reportedly wants to continue providing subsidy about four percentage points was seen to have a considerable
to consumers for cooking gas and kerosene for five more years. variation across states and regions.
This is not good news from the point of view of reining in the 43. Which of the following is a conclusion which can be drawn
fiscal deficit. Mounting subventions for subsidies means diversion from the facts stated in the above paragraph ?
of savings by the government from investment to consumption, (a) Accurate estimates of number of people living below
raising the cost of capital in the process. The government must poverty line in India is possible to be made.
cut expenditure on subsidies to create more fiscal space for
(b) Many expert groups in India are not interested to
investments in both physical and social infrastructure. It should
measure poverty objectively.
outline a plan for comprehensive reform in major subsidies
including petroleum, food and fertilizers and set goal posts. (c) Process of poverty measurement needs to take into
account various factors to tackle its dynamic nature.
40. Which of the following is a conclusion which can be drawn
from the facts stated in the above paragraph ? (d) People living below poverty line remain in that position
for a very long time.
(a) Subsidy provided by the government under various
heads to the citizen increases the cost of capital (e) None of these
(b) Government is unable to withdraw subsidies provided 44. Which of the following is an as sumption which is implicit
to various items. in the facts stated in the above paragraph ?
(a) It may not be possible to have an accurate poverty
(c) Government subsidy on kerosene is purely a political
measurement in India.
decision.
(b) Level of poverty in India is static over the years.
(d) Govt. does not have enough resources to continue
providing subsidy on petroleum products. (c) Researchers avoid making conclusions on poverty
measurement data in India.
(e) None of these
(d) Government of India has a mechanism to measure level
41. Which of the following is an inference which can be made of poverty effectively and accurately.
from the facts stated in the above paragraph ?
(e) None of these
y
o
u
rs
m
6 SBI Rural Business PO Exam 2010

a
h
45. Which of the following is an inference which can be made 48. Problem Figures

b
o
from the facts stated in the above paragraph ?

o
b
A
(a) Poverty measurement tools in India are outdated.

.w
A OT
Z T
(b) Increase in number of persons falling into poverty varies A

o
rd
considerably across the country over a period of time.
T A

T
T

p
(c) Government of India has stopped measuring poverty

re
related studies. Answer Figures

s
s
(d) People living in rural areas are more susceptible to fall

.c
into poverty over the time

o
m
(e) None of these
DIRECTIONS (Qs. 46 - 50) : In each of the questions given below
which one of the five answer figures on the right should come after (a) (b) (c) (d) (e)
the problem figures on the left, if the sequence were continued ?
46. Problem Figures 49. Problem Figures

A K A K
K
A K K A A
Answer Figures Answer Figures

A A
K K K A K
A AK
(a) (b) (c) (d) (e) (a) (b) (c) (d) (e)
47. Problem Figures 50. Problem Figures

Z Z Z
D
D

Z
D

D
Answer Figures Answer Figures D

Z
Z Z Z Z
D

D
D

D
D

(a) (b) (c) (d) (e) (a) (b) (c) (d) (e)

DATA ANALYSIS AND INTERPRETATION


DIRECTIONS (Qs. 51 - 55) : Study the following table carefully to answer the questions that follow:
Number (N) of Candidates (In Lakhs) Appearing for An Entrance Examination From Six Different States and the Percentage (P) of
Canditates Clearing the Same Over the Years

State® A B C D E F
Year ¯ N P N P N P N P N P N P
2004 1.23 42 1.04 51 1.11 32 1.32 24 1.23 36 1.33 31
2005 1.05 43 1.12 62 1.07 47 1.15 49 1.18 55 1.24 24
2006 2.04 38 1.48 32 1.08 28 1.96 35 1.42 49 1.58 26
2007 1.98 41 2.07 43 1.19 30 1.88 46 1.36 47 1.79 29
2008 1.66 53 1.81 50 1.56 42 1.83 60 1.73 57 1.86 34
2009 1.57 39 1.73 36 1.64 52 2.01 56 1.69 55 1.95 37
y
o
u
rs
m
SBI Rural Business PO Exam 2010 7

a
h
51. What is the respective ratio of total number of candidates clearing

b
59. If four marbles are picked at random, what is the probability

o
the entrance exam from State B in the year 2004 to those clearing that one is green, two are blue and one is red?

o
b
the entrance exam from State C in the same year ?

.w
24 13
(a) 221 : 148 (b) 218 : 143 (a) (b)

o
(c) 148 : 221 (d) 143 : 218 455 35

rd
(e) None of these 11 1

p
re
52. In which year did the highest number of candidates clear (c) (d)
15 3

s
the entrance exam from State D ?

s
(e) None of these

.c
(a) 2008 (b) 2006

o
60. If two marbles are picked at random, what is the probability

m
(c) 2009 (d) 2007 that either both are green or both are yellow?
(e) None of these
5 1
53. What is the number of candidates not clearing the entrance (a) (b)
exam from State A in the year 2007 ? 91 35
(a) 186820 (b) 11682 1 4
(c) 1868200 (d) 116820 (c) (d)
3 105
(e) None of these (e) None of these
54. What is the total number of candidates clearing the entrance
exam from States E and F together in the year 2006 ? DIRECTIONS (Qs. 61- 65): Study the given pie-charts carefully
(a) 16160 (b) 110660 to answer the questions that follow :
(c) 11066 (d) 1106600 Breakup of Number of Employees working in Different
(e) None of these Departments of an Organisation, the Number of Males and the
55. What is the average number of candidates appearing for Number of Employees Who Recently Got Promoted. In Each
the entrance exam from State D in the years 2007, 2008 and Department Break-UP of Employees Working In Different
2009 together ? Departments:
Total Number of Employees = 3,600
2 1
(a) 1907 (b) 18666 Employees Working in Different Departments
3 3
1 2 Accounts
(c) 1866 (d) 190666
3 3 20%
(e) None of these
DIRECTIONS (Qs. 56 - 60) : Study the given information carefully Marketing Production
and answer the questions that follow : 18% 35%
Anurn contains 6 red, 4 blue, 2 green and 3 yellow marbles.
HR
56. If four marbles are picked at random, what is the probability 12%
that at least one is blue? IT
15%
4 69
(a) (b)
15 91
11 22
(c) (d) Break-UP of Number of Males In Each Department Total Number
15 91
(e) None of these Of Males In the Organisation = 2,040 Break-UP of Number of
57. If two marbles are picked at random, what is the probability Males Working In Each Department
that both are red?
Accounts
1 1 5%
(a) (b)
6 3
2 2
(c) (d)
15 5 Marketing
(e) None of these 15%
58. If three marbles are picked at random, what is the probability Production
HR 50%
that two are blue and one is yellow? 10%
3 1 IT
(a) (b)
31 5 20%
18 7
(c) (d)
455 15
Break-UP of Number of Employees who recently got promoted
(e) None of these
In Each Department
y
o
u
rs
m
8 SBI Rural Business PO Exam 2010

a
h
b
Total Number of Employees who got promoted = 1,200 Number Percent Rise in Profit of Two

o
of Employees Who Recently Got Promoted From Each Companies Over The Years

o
b
Department

.w
o
Compnay L

rd
Accounts

p
8% Compnay M

re
s
s
40

.c
o
35

m
Marketing
22% Production 30

Profit Percent
33%
HR 25
11% 20
IT 15
26%
10
5
0
61. If half of the number of employees who got promoted from 2004 2005 2006 2007 2008 2009
the IT department were males, what was the approximate Years
percentage of males who got promoted from the IT
department? 66. If the profit earned by Company L in the year 2005 was
(a) 61 (b) 29 ` 1.84 lakhs, what was the profit earned by the company in
the year 2006?
(c) 54 (d) 42
(a) ` 2.12 lakhs (b) ` 2.3 lakhs
(e) 38
(c) ` 2.04 lakhs (d) Cannot be determined
62. What is the total number of females working in the Production (e) None of these
and Marketing departments together ? 67. If the profit earned by Company M in the year 2008 was
(a) 468 (b) 812 ` 3. 63 lakhs, what was the amount of profit earned by it in
(c) 582 (d) 972 the year 2006 ?
(e) None of these (a) ` 2.16 lakhs (b) ` 1.98 lakhs
(c) ` 2.42 lakhs (d) Cannot be determined
63. How many females work in the Accounts department ?
(e) None of these
(a) 618 (b) 592
68. What is the average percent rise in profit of Company L
(c) 566 (d) 624 over call the years together ?
(e) None of these
1 1
64. The total number of employees who got promoted from all (a) 15 (b) 25
3 3
the departments together was what percent of the total number
of employees working in all the departments together ? 5 5
(c) 18 (d) 21
(Rounded off to the nearest integer) 6 6
(a) 56 (b) 21 (e) None of these
69. Which of the following statements is TRUE with respect to
(c) 45 (d) 33
the above graph?
(e) 51 (a) Company M made the highest profit in the year 2009
65. The number of employees who got promoted from the HR (b) Company L made least profit in the year 2008
department was what percent of the total number of (c) The respective ratio between the profits earned by
employees working in that department ? Company L and M in the year 2006 was 6 : 5
(rounded off to two digits after decimal) (d) Company L made the highest profit in the year 2005
(e) All are true
(a) 36.18 (b) 30.56
70. What is the percentage increase in percent rise in profit of
(c) 47.22 (d) 28.16 Company M in the year 2009 from the previous year ?
(e) None of these (a) 25 (b) 15
DIRECTIONS (Qs. 66 - 70): Study the graph carefully to answer (c) 50 (d) 75
the questions that follow : (e) None of these
y
o
u
rs
m
SBI Rural Business PO Exam 2010 9

a
h
b
DIRECTIONS (Qs. 71 - 75): Study the information carefully to 72. Total number of girls enrolled in Singing is approximately

o
o
answer the questions that follow: what percent of the total number of students in the school ?

b
.w
A school consisting of a total of 1560 students has boys (a) 37 (b) 19

o
and girls in the ratio of 7 :5 respectively. All the students are (c) 32 (d) 14

rd
enrolled in different types of hobby classes, viz: Singing, Dancing (e) 26

p
and Painting. One-fifth of the boys are enrolled in only Dancing

re
73. What is the total number of students enrolled in all the three
classes. Twenty percent of the girls are enrolled in only Painting

s
s
classes together ?

.c
classes. Ten percent of the boys are enrolled in only Singing

o
classes. Twenty four percent of the girls are enrolled in both Singing (a) 135 (b) 164

m
and Dancing classes together. The number of girls enrolled in (c) 187 (d) 142
only Singing classes is two hundred percent of the boys enrolled (e) None of these
in the same. One-thirteenth of the boys are enrolled in all the three
classes together. The respective ratio of boys enrolled in Dancing 74. Number of girls enrolled in only Dancing classes is what
and Painting classes together to the girls enrolled in the same is percent of the boys enrolled in the same ? (rounded off to
2 : 1 respectively. Ten percent of the girls are enrolled in only two digits after decimal)
Dancing classes whereas eight percent of the girls are enrolled in (a) 38.67 (b) 35.71
both Dancing and Painting classes together. The remaining girls (c) 41.83 (d) 28.62
are enrolled in all the three classes together. The number of boys
(e) None of these
enrolled in Singing and Dancing classes together is fifty percent
of the number of girls enrolled in the same. The remaining boys 75. What is the respective ratio of the number of girls enrolled
are enrolled in only Painting classes. in only Painting classes to the number of boys enrolled in
71. What is the total number of boys who are enrolled in Dancing ? the same?
(a) 318 (b) 364 (a) 77 : 26 (b) 21 : 73
(c) 292 (d) 434 (c) 26 : 77 (d) 73 : 21
(e) None of these (e) None of these

DIRECTIONS (Qs. 76 - 80) : Study the table carefully to answer the questions that follow:
Profit (In `'000) Made by Six Different Shopkeepers over the Months

Months ® October November December January February March


Shopkeeper ¯ 2009 2009 2009 2010 2010 2010
P 5.25 6.04 5.84 6.10 5.95 6.02
Q 4.84 4.28 4.97 4.88 5.04 5.12
R 4.99 5.82 5.48 5.45 5.68 5.36
S 5.06 5.11 5.28 5.38 5.44 5.59
T 5.28 4.96 5.31 5.69 4.93 5.72
U 5.94 6.23 5.87 6.07 6.19 6.23

76. What is the respective ratio between the profit earned by (a) R (b) Q
shopkeeper U in the months February -2010 and March -
(c) T (d) U
2010 together to that earned by shopkeeper Q in the same
months ? (e) None of these
(a) 637 : 512 (b) 621 : 508 79. What is the difference in profit earned by shopkeeper T in
(c) 512 : 637 (d) 508 : 621 January -2010 from the previous month ?
(e) None of these (a) ` 640 (b) ` 420
77. What is the percent increase in profit of shopkeeper S in the (c) ` 380 (d) ` 760
month of December -2009 over the previous month? (e) None of these
(rounded off to two digits after decimal) 80. What was the average profit earned by shopkeeper R in the
(a) 3.15 (b) 2.67 months of October -2009 and November -2009 together
(c) 2.18 (d) 3.33 (a) 5405 (b) 5040
(e) None of these (c) 4825 (d) 4950
78. Which shopkeeper's profit kept increasing continuously (e) None of these
over the given months ?
y
o
u
rs
m
10 SBI Rural Business PO Exam 2010

a
h
b
DIRECTIONS (Qs. 81- 85) : Study the given graph carefully to Number of People Staying in Five Different Localities and the

o
Percentage Breakup of Men, Women and Children in Them

o
answer the questions that follow:

b
.w
Number Of Days Taken By Three Carpenters To Finish
Locality Total N o. Pe rce ntag e

o
Making One Piece Each Of Four Different

rd
Items Of Furniture of Pe ople M e n Wo me n C hildre n

p
re
F 5640 55 35 10

s
G 4850 34 44 22

s
Carpenter X Carpenter Y

.c
H 5200 48 39 13

o
Carpenter Z

m
16 I 6020 65 25 10
14 J 4900 42 41 17
12 86. Total number of people staying in locality J forms
10 approximately what percent of the total number of people
8 staying in locality F ?
(a) 81 (b) 72
6
(c) 78 (d) 93
4 (e) 87
2 87. What is the total number of children staying in localities H
0 and I together ?
Chair Table Bed Cupboard (a) 1287 (b) 1278
(c) 1827 (d) 1728
81. If carpenter X and carpenter Y were to make a chair together (e) None of these
how many days would they take? 88. The number of women staying in which locality is the highest ?
(a) 1 day (b) 4 days (a) H (b) J
(c) F (d) G
(c) 3 days (d) 2 days
(e) None of these
(e) None of these
89. What is the total number of men and children staying in
82. If carpenters X, Y and Z were to make a table together how locality I together ?
many days would they take ? (a) 4115 (b) 4551
(a) 4 days (b) 3 days (c) 4515 (d) 4155
(c) 1 day (d) 2 days (e) None of these
90. What is the respective ratio of number of men staying in
(e) None of these
locality F to the number of men staying in locality H ?
83. What is the total number of days that carpenter Z will take (a) 517:416 (b) 403:522
to make one piece each of all the four items together ? (c) 416:517 (d) 522 :403
(a) 32 days (b) 24 days (e) None of these
1 1
(c) l days (d) 1 days 91. The compound interest earned by Suresh on a certain
59 32
(e) None of these amount at the end of two years at the rate of 8 p.c.p.a was
` 1,414.4. What was the total amount that Suresh got back
84. The radius of a circular field is equal to the side of a square at the end of two years in the form of principal plus interest
field whose perimeter is 784 feet. What is the area of the earned ?
circular field ? (a) ` 9.414.4 (b) ` 9,914.4
(a) 107914 sq.ft. (b) 120736 sq.ft. (c) ` 9,014.4 (d) ` 8,914.4
(c) 107362 sq.ft. (d) 127306 sq.ft. (e) None of these
92. The respective ratio of the present ages of a mother and
(e) None of these
daughter is 7 : 1. Four years ago the respective ratio of their
85. In how many different ways can the letters of the word ages was 19 : 1. What will be the mother's age four years
'STRESS' be arranged ? from now?
(a) 360 (b) 240 (a) 42 years (b) 38 years
(c) 720 (d) 120 (c) 46 years (d) 36 years
(e) None of these
(e) None of these
93. Three friends J, K and L jog around a circular stadium and
DIRECTIONS (Qs. 86 - 90): Study the given table carefully to complete one round in 12, 18 and 20 seconds respectively.
answer the questions that follow: In how many minutes will all the three meet again at the
y
o
u
rs
m
SBI Rural Business PO Exam 2010 11

a
h
b
starting point ? (a) 1 day (b) 3 days

o
(a) 5 (b) 8 (c) 2 days (d) 4 days

o
b
(c) 12 (d) 3 (e) None of these

.w
(e) None of these 95. The speed of a boat when travelling downstream is 32 kmph

o
rd
94. 4 men can complete a piece of work in 2 days. 4 women can whereas when travelling upstream it is 28 kmph. What is the

p
complete the same piece of work in 4 days whereas 5 children speed of the boat in still water ?

re
can complete the same piece of work in 4 days. If, 2 men, 4 (a) 27 Kmph (b) 29 Kmph

s
s
women and 10 children work together, in how many days

.c
(c) 31 Kmph (d) Cannot be determined

o
can the work be completed ? (e) None of these

m
DIRECTIONS (Qs. 96-100) : Study the following tables carefully and answer the questions given below:
Number & Percentage of Candidates Qualified in a Competitive Examination:
Number of Candidates appeared in a Competitive Examination From Five Centres Over The Years

Centre ®
Mumbai Delhi Kolkata Hyderabad Chennai
Year ¯
2001 35145 65139 45192 51124 37346
2002 17264 58248 52314 50248 48932
2003 24800 63309 56469 52368 51406
2004 28316 70316 71253 54196 52315
2005 36503 69294 69632 58360 55492
2006 29129 59216 64178 48230 57365
2007 32438 61345 56304 49178 58492

Approximate Percentages of Candidates Qualified To Appeared In the Competitive Examination From Five Centres Over the year

Centre ®
Mumbai Delhi Kolkata Hyderabad Chennai
Year ¯
2001 12 24 18 17 9
2002 10 28 12 21 12
2003 15 21 23 25 10
2004 11 27 19 24 8
2005 13 23 16 23 13
2006 14 20 21 19 11
2007 16 19 24 20 14

96. In which of the following years was the difference in number (a) 27250 (b) 25230
of candidates appeared from Mumbai over the previous year (c) 30150 (d) 28150
the minimum ? (e) 26250
(a) 2004 (b) 2006 99. Approximately how many candidates appearing from
(c) 2007 (d) 2002 Kolkata in 2004 qualified in the competitive examination ?
(e) None of these (a) 13230 (b) 13540
97. In which of the following years was the number of canditates (c) 15130 (d) 15400
qualified from Chennai, the maximum among the given years ? (e) 19240
(a) 2007 (b) 2006 100. Approximately what was the difference between the number
(c) 2005 (d) 2003 of candidates qualified from Hyderabad in 2001 and 2002 ?
(e) None of these (a) 1680 (b) 2440
98. Approximately what was the total number of canditates (c) 1450 (d) 2060
qualified from Delhi in 2002 and 2006 together ? (e) 1860
y
o
u
rs
m
12 SBI Rural Business PO Exam 2010

a
h
b
In India on the other hand, with only 40% of its farmland
ENGLISH LANGUAGE

o
irrigated, entire economic boom currently underway is held hostage

o
b
by the unpredictable monsoon. With much of India's farming areas

.w
DIRECTIONS (Qs. 101 - 115) : Read the following passage suffering from drought this year, the government will have a tough

o
carefully and answer the questions given below it. Certain words/ time meeting its economic growth targets. In a report, Goldman

rd
phrases have been printed in bold to help you locate them while Sachs predicted that if this year too receives weak rains, it could

p
re
answering some of the questions. cause agriculture to contract by 2% this fiscalyear, making the

s
s
Governments have traditionally equated economic progress government's 7% GDP-growth target look “a bit rich”. Another

.c
with steel mills and cement factories. While urban centres thrive green revolution is the need of the hour and to make it a reality,

o
m
and city dwellers get rich, hundreds of millions of farmers remain the global community still has much backbreaking farm work to do.
mired in poverty. However, fears of food shortage, a rethinking of 101. What is the author's main objective in writing the passage ?
antipoverty priorities and the crushing recession in 2008 are (a) Criticising developed countries for not bolstering
causing a dramatic shift in world economic policy in favour of economic growth in poor nations
greater support for agriculture. (b) Analysing the disadvantages of the Green Revolution
The last time when the world's farmers felt such love was in (c) Persuading experts that a strong economy depends on
the 1970s. At that time, as food prices spiked, there was real concern industrialisation and not agriculture
that the world was facing a crisis in which the planet was simply (d) Making a case for the international society to engineer
unable to produce enough grain and meat for an expanding a second Green Revolution
population. Government across the developing world and (e) Rationalising the faulty agriculture policies of emerging
international aid organisations plowed investment into agriculture countries
in the early 1970s, while technological breakthroughs, like high- 102. Which of the following is an adverse impact of the Green
yield strains of important food crops, boosted production. The Revolution ?
result was the Green Revolution and food production exploded.
(a) Unchecked crop yields resulted in large tracts of land
But the Green Revolution became a victim of its own success. becoming barren
Food prices plunged by some 60% by the late 1980s from their
(b) Withdrawal of fiscal impetus from agriculture to other
peak in the mid 1970s. Policy makers and aid workers turned their sectors
attention to the poor's other pressing needs, such as health care
(c) Farmers began soliciting government subsidies for their
and education. Farming got starved of resources and investment.
produce
By 2004, aid directed at agriculture sank to 3.5% and "Agriculture
(d) Farmers rioted as food prices fell so low that they could
lost its glitter". Also, as consumers in high growth giants such as
not make ends meet
China and India became wealthier, they began eating more meat,
so grain once used for human consumption got diverted to beef (e) None of these
up livestock. By early 2008, panicked buying by importing countries 103. What is the author trying to convey through the phrase
and restrictions slapped on grain exports by some big producers "making the government's 7% GDP growth target look "a
helped drive prices upto heights not seen for three decades. bit rich" ?
Making matters worse land and resources got reallocated to (a) Indian is unlikely to achieve the targeted growth rate
produce cash crops such as biofuels and the result was the (b) Allocation of funds to agriculture has raised India's
voluminous reserves of grain evaporated. Protests broke out across chances of having a high GDP
the emerging world and fierce food riots toppled governments. (c) Agricultural growth has artificially inflated India's GDP
This spurred global leaders into action. This made them and such growth is not real
aware thai food security is one of the fundamental issues in the (d) India is likely to have one of the highest GDP growth
world that has to be dealt with in order to maintain administrative rates
and political stability. This also spurred the U.S. which traditionally (e) A large portion of India's GDP is contributed by
provisioned food aid from American grain surpluses to help needy agriculture
nations, to move towards investing in farm sectors around the 104. Which of the following factors was/were responsible for
globe to boost productivity. This move helped countries become the neglect of the farming sector after the green revolution ?
more productive for themselves and be in a better position to feed (A) Steel and cement sectors generated more revenue for
their own people. the government as compared to agriculture.
Africa, which missed out on the first Green Revolution due (B) Large scale protests against favouring agriculture at
to poor policy and limited resources, also witnessed a 'change'. the cost of other important sectors such as education
Swayed by the success of East Asia, the primary poverty-fighting and health care.
method favoured by many policymakers in Africa was to get (C) Attention of policy makers and aid organisations was
farmers off their farms and into modern jobs in factories and urban diverted from agriculture to other sectors.
centres. But that strategy proved to be highly insufficient. Income (a) None (b) Only (C)
levels in the countryside badly trailed those in cities while the
(c) Only (B) and (C) (d) Only (A) and (B)
FAO estimated that the number of poor going hungry in 2009
(e) All (A), (B) and (C)
reached an all time high at more than one billion.
y
o
u
rs
m
SBI Rural Business PO Exam 2010 13

a
h
b
105. What prompted leaders throughout the world to take action 110. Which of the following is true about the state of agriculture

o
to boost the agriculture sector in 2008 ? in India at present ?

o
b
(a) Coercive tactics by the U.S. which restricted food aid (A) Of all the sectors, agriculture needs the highest

.w
to poor nations allocation of funds.

o
rd
(b) The realization of the link between food security and (B) Contribution of agriculture to India's GDP this year

p
political stability would depend greatly upon the monsoon rains.

re
(c) Awareness that performance in agriculture is necessary (C) As India is one of the high-growth countries, it has

s
s
in order to achieve the targeted GDP surplus food reserves to export to other nations.

.c
(d) Reports that high-growth countries like China and India

o
(a) Only (A) and (C) (b) Only (C)

m
were boosting their agriculture sectors to capture the (c) Only (B) (d) Only (B) and (C)
international markets
(e) None of these
(e) Their desire to influence developing nations to slow
down their industrial development. DIRECTIONS (Qs. 111 - 113): Choose the word/group of words
106. What motivated the U.S. to focus on investing in agriculture which is most similar in meaning to the word printed in bold as
across the globe ? used in the passage.
(a) To make developing countries become more reliant on 111. STARVED
U.S. aid (a) Deprived (b) Disadvantaged
(b) To ensure grain surpluses so that the U.S. had no need (c) Hungry (d) Fasting
to import food (e) Emaciated
(c) To make those countries more self sufficient to whom 112. SLAPPED
it previously provided food
(a) Beaten (b) Imposed
(d) To establish itself in the market before the high-growth
(c) Withdrawn (d) Avoided
giants such as India and China could establish
themselves (e) Persuaded
(e) None of these 113. PLOWED
107. What impact did the economic recession of 2008 have on (a) Cultivated (b) Bulldozed
agriculture ? (c) Recovered (d) Instilled
(a) Governments equated economic stability with (e) Withdrew
industrial development and shifted away from DIRECTIONS (Qs. 114 - 115): Choose the word/phrase which is
agriculture most opposite in meaning to the word printed in bold as used in
(b) Lack of implementation of several innovative agriculture the passage.
programmes owing to shortage of funds
114. PRESSING
(c) It prompted increased investment and interest in
agriculture (a) Unpopular (b) Undemanding
(d) The GDP as targeted by India was never achieved (c) Unobtrusive (d) Unsuitable
because of losses in agriculture (e) Unimportant
(e) None of these 115. EVAPORATED
108. What encouraged African policy makers to focus on urban (a) Absorbed (b) Accelerated
jobs ? (c) Grew (d) Plunged
(a) Misapprehension that it would alleviate poverty as it (e) Mismanaged
did in other countries
DIRECTIONS (Qs. 116 - 120): Which of the phrases (1), (2), (3)
(b) Rural development out-stripped urban development in
and (4) given below each statement should be placed in the
many parts of Africa
blank space provided so as to make a meaningful and
(c) Breaking out of protests in the country and the fear grammatically correct sentence ? If none of the sentences is
that the government would topple
appropriate, mark (5) i.e. 'None of these' as the answer.
(d) Blind imitation of western models of development
116. Refuting the rationale behind frequent agitations for
(e) None of these
formation of separate States, a recent report_______.
109. Which of the following had contributed to exorbitant food
prices in 2008 ? (a) proved that such agitations result in loss of
governmental property
(A) Hoarding of food stocks by local wholesalers which
inadvertently created a food shortage. (b) indicated that the formation of small states does not
necessarily improve the economy
(B) Export of foodgrains was reduced by large producers.
(C) Diverting resources from cultivation of foodgrains to that (c) suggested that only large scale agitations have been
of more profitable crops. effective in bringing out desired change in the past
(a) None (b) Only(C) (d) recommended dividing large States into smaller ones
to improve governance
(c) Only(B) (d) All (A), (B) and (C)
(e) None of these
(e) Only (B) and (C)
y
o
u
rs
m
14 SBI Rural Business PO Exam 2010

a
h
b
117. Overlooking the fact that water scarcity intensifies during 122. In an effort to provide ______ for higher education to all.

o
summer ________. most of the universities have been providing education

o
b
(a) the government issued guide-lines to all builders to without adequate infrastructure, thus churning out ______

.w
limit their consumption to acceptable limits graduates every year.

o
(a) chances, fresh (b) platform, capable

rd
(b) provision for rainwater harvesting has been made to

p
aid irrigation in drought prone areas (c) opportunities, unemployable

re
(c) the water table did not improve even after receiving (d) prospects, eligible (e) policy, incompetent

s
s
normal monsoon in the current year 123. The move to allow dumping of mercury ______ an outcry

.c
from residents to the area who _______ that high levels of

o
(d) Many residential areas continue to use swimming

m
mercury will affect their health and destroy ecologically
pools, wasting large quantities of water
sensitive forest area
(e) None of these
(a) resulted, insist (b) provoked, fear
118. He has lost most of the life's earning in the stock market but
(c) incited, determined (d) activated, accept
_______.
(e) angered believe
(a) he still seems to be leading his life luxuriously and
124. ______ has been taken against some wholesale drug dealers
extravagantly for dealing in surgical items without a valid license and
(b) he could not save enough to repay his enormous debts maintaining a stock of ______ drugs.
(c) stock market is not a safe option to invest money (a) Note, overwhelming (b) Step, impressive
unless done with caution (c) Execution, outdated (d) Action, expired
(d) experts have been suggesting to avoid investments in (e) Lawsuit, invalid
stock market because of its unpredictable nature 125. Even as the _______ elsewhere in the world are struggling
(e) None of these to come out of recession, Indian consumers are splurging
119. Achieving equality for women is not only a laudable goal, on consumer goods and to ______ this growth, companies
_______. are investing heavily in various sectors.
(a) political reforms are also neglected preventing women (a) economies, meet (b) countries, inhibit
from entering legislatures and positions of power (c) governments, measure (d) nations, inflict
(b) the problem is also deep rooted in the society and (e) companies, counter
supported by it DIRECTIONS (Qs. 126 - 130) : Rearrange the following sentences
(c) their empowerment is purposefully hampered by people (A), (B), (C), (D), (E) and (F) to make a meaningful paragraph
with vested interests in all sections of the society and then answer the questions which follow:
(d) it is also equally difficult to achieve and maintain for (A) While these disadvantages of biofuels are serious, they are
long term the only alternate energy source of the future and the sooner
(e) None of these we find solutions to these problems the faster we will be
120. ______ or else they would not keep electing him year after able to solve the problems we are now facing with gasoline.
year. (B) This fuel can also help to stimulate jobs locally since they
(a) The party leader gave a strong message to the mayor are also much safer to handle than gasoline and can thus
for improving his political style have the potential to turnaround a global economy.
(C) These include dependence on fossil fuels for the machinery
(b) Owing to numerous scandals against the mayor, he
required to produce biofuel which ends up polluting as much
was told to resign from the post immediately
as the burning of fossil fuels on roads and exorbitant cost
(c) The mayor threatened the residents against filing a of biofuels which makes it very difficult for the common
complaint against him man to switch to this option.
(d) The residents must really be impressed with the political (D) This turnaround can potentially help to bring world peace
style of their mayor andt end the need to depend on foreign countries for energy
(e) None of these requirements.
DIRECTIONS (Qs. 121 - 125) : Each question below has two (E) Biofuels are made from plant sources and since these sources
blanks, each blank indicating that something has been omitted. are available in abundance and can be reproduced on a
massive scale they form an energy source that is potentially
Choose the set of words for each blank that best fits the meaning
unlimited.
of the sentence as a whole.
(F) However everything is not as green with the biofuels as it
121. Drawing attention to the pitfalls of _____ solely on Uranium seems as there are numerous disadvantages involved which
as a fuel for nuclear reactors, Indian scientists warned that at times overshadow their positive impact.
Uranium will not last for long and thus research on Thorium 126. Which of the following sentences should be the FIFTH
as its ______ must be revived. after rearrangement ?
(a) using, substitute (b) believing, replacement (a) A (b) B
(c) depending, reserve (d) reckoning, option (c) C (d) E
(e) relying, alternative (e) F
y
o
u
rs
m
SBI Rural Business PO Exam 2010 15

a
h
b
127. Which of the following sentences should be the THIRD (c) who were earlier conversant

o
after rearrangement ? (d) who were earlier unaware

o
b
(a) A (b) B (e) No correction required

.w
(c) C (d) D 135. Over the last few months, while most industries are busy

o
rd
(e) E in restructuring operations, cutting costs and firing, the

p
128. Which of the following sentences should be the FIRST Indian pharmaceutical and health care industry was adding

re
after rearrangement ? manpower and giving salary hikes.

s
s
(a) as many industries are

.c
(a) A (b) B

o
(c) C (d) D (b) while most industries were

m
(e) E (c) while many industries is
129. Which of the following sentences should be the SIXTH (d) where many industries were
(LAST) after rearrangement ? (e) No correction required
(a) A (b) C DIRECTIONS (Qs. 136 - 145) : In the following passage there
(c) D (d) E are blanks, each of which has been numbered. These numbers
(e) F are printed below the passage and against each, five words/
130. Which of the following sentences should be the SECOND phrases are suggested, one of which fits the blank appropriately.
after rearrangement ? Find out the appropriate word/phrase in each case.
(a) A (b) B There is a considerable amount of research about the factors that
(c) D (d) E make a company innovate. So is it possible to create an environment
(e) F (136) to innovation ? This is a particularly pertinent (137) for
DIRECTIONS (131-135): Which of the phrases (1), (2), (3) and India today. Massive problems in health, education etc. (138) be
(4) given below each statement should replace the phrase printed solved using a conventional approach but (139) creative and
in bold in the sentence to make it grammatically correct? If the inno-vative solutions that can ensure radical change and (140).
sentence is correct as it is given and 'No correction is required', There are several factors in India's (141). Few countries have the
mark (5) as the answer. rich diversity that India or its large, young population (142). While
these (143) innovation policy interventions certain additional steps
131. Soon after the Tsunami had killed thousands of people along are also required. These include (144) investment in research and
the coasts of southern India, Parliament passess a bill that development by (145) the government and the private sector,
proposed to set up an institutional mechanism to respond easy transfer of technology from the academic world etc. To fulfill
promptly to natural disasters. its promise of being prosperous and to be at the forefront, India
(a) passed a bill that proposed must be innovative.
(b) passes a bill with purpose 136. (a) stimuli (b) conducive
(c) pass a bill proposing (c) incentive (d) facilitated
(d) passed a bill which propose (e) impetus
(e) No correction required 137. (a) objective (b) controversy
132. Denial of wages forced scientists and teachers at the (c) doubt (d) question
agriculture universities throughout the country to go on
(e) inference
strike, crippling crucial research that could help the state
of agriculture in the country. 138. (a) cannot (b) possibly
(a) from going on strike (b) which went on strike (c) should (d) never
(c) on going for a strike (d) for going to strike (e) must
(e) No correction required 139. (a) necessary (b) apply
133. In an attempt to boost their profits many edible oil producing (c) need (d) consider
companies have been engaging themselves in propaganda (e) requires
against commonly used oils and promoting exotic and 140. (a) quantity (b) advantages
expensive varieties of oil as more healthier options. (c) increase (d) chaos
(a) as most healthiest options (e) growth
(b) as less healthy option 141. (a) challenges (b) praises
(c) as a healthier option (c) favour (d) leverage
(d) as much healthiest option (e) esteem
(e) No correction required 142. (a) blessed (b) enjoys
134. Thanks to numerous government initiatives, rural masses (c) endows (d) prevails
which was earlier unaware of the luxuries of urban ways of (e) occurs
living are now connected to the same lifestyle. 143. (a) aid (b) jeoparadise
(a) who was earlier unaware (c) promotes (d) endure
(b) which were earlier aware (e) cater
y
o
u
rs
m
16 SBI Rural Business PO Exam 2010

a
h
b
144. (a) acute (b) utilising 153. Which of the following challenges will be the most crucial

o
(c) restricting (d) inspiring for human existence on earth in the days to come ?

o
b
(e) increased (a) Pollution

.w
145. (a) both (b) besides (b) Food Security

o
(c) combining (d) participating

rd
(c) Availability of natural gas
(e) also

p
(d) Need for Financial Inclusion

re
DIRECTIONS (Qs. 146-150) : In each of the following questions (e) Cyber Crimes

s
s
four words are given of which two words are most nearly the 154. The Reserve Bank of India has asked all the banks to install

.c
same or opposite in meaning. Find the two words which are "Note Sorting Machines" in the branches. How will this

o
m
most nearly the same or opposite in meaning and find the number help banks and the general public ?
of the correct letter combination, that is your answer. (A) The machines will check for counterfeit notes and drop
146. (A) consent (B) nascent these from circulation.
(C) emerging (D) insecure (B) This will help banks to count the notes quickly and
(a) A - C (b) B - D accurately.
(c) B - C (d) A - D (C) This will help in sorting out soiled notes so that they
(e) A - B are not reissued by the banks.
147. (A) elated (B) eccentric (a) Only B (b) Only A
(C) explicit (D) abnormal (c) Only C (d) Only A and B
(a) A - B (b) B - D (e) None of these
(c) A - C (d) A - D 155. A study report entitled "India Generation Gap Review" was
(e) D - C presented at the World Economic Forum in November 2009.
148. (A) abundance (B) incomparable This was presented during which of the following summits/
conferences ?
(C) projection (D) plethora
(a) G-8 Summit.
(a) A - C (b) A - B
(b) G-20 Summit
(c) C - D (d) B - D
(e) A - D (c) India Economic Summit
149. (A) purposefully (B) inaccurately (d) SAARC Finance Ministers' Conference
(C) inadvertently (D) unchangeably (e) None of these
(a) A - C (b) A - B 156. As we know the Government of India has notified certain
new rules to calculate Income Tax on various perks like
(c) B - C (d) B - D
accommodation, and conveyance etc. being given by
(e) A - D employers to their salaried employees. The new rules will
150. (A) germane (B) generate replace which of the following taxes which has been
(C) reliable (D) irrelevant abolished now ?
(a) B - D (b) B - C (a) Capital Gains Tax (b) Sales Tax
(c) A - B (d) C - D (c) Cash Transaction Tax (d) Fringe Benefit Tax
(e) A - D (e) None of these
157. Dr. K. Radhakrishnan has taken over as the Chief of -
General Awareness/ Computer (a) Electronics Commission
(b) Indian Space Research Organisation
Knowledge/Marketing Knowledge (c) Atomic Energy Commission of India
(d) Deputy Governor of Reserve Bank of India
151. As published in various newspapers, the Experian Credit (e) None of these
Information Company of India is going to launch its 158. As per the guidelines issued by the RBI, banks are preparing
operation in India soon: At present the only credit for a service which will allow customers to withdraw upto
information bureau functional in India is ` 1,000 using their debit cards from notified shops/stores
(a) AMFI (b) ICRA allover the country. All such shops/stores will have 'POS'
terminals for the same. What is full form of the 'POS'?
(c) CIBIL (d) CRISIL (a) Payment on Sale (b) Power of Sale
(e) None of these (c) Point of Sale (d) Payment Order Service
152. As per the news published in a financial population RBI is (e) None of these
redesinging its 'ECS' to function as a Automated Clearing 159. The working group set up by the RBI has suggested the
House (ACH) for bulk transactions. What is full form of launch of an Emergency Fund Facility Scheme for banks.
'ECS'? This scheme will help which of the following types of banks ?
(a) Extra Closing System (a) Public Sector Banks
(b) Electronic Cheque System (b) Small Banks
(c) Evening Cheque Sorter (c) Urban Cooperative Banks
(d) Evening Cheque System (d) Private Banks
(e) Electronic Clearing Service (e) Foreign Banks
y
o
u
rs
m
SBI Rural Business PO Exam 2010 17

a
h
b
160. As published in major newspapers public sector banks have (a) Use of Genetically Modified Crop

o
asked the Government of India to provide more funds for (b) Melting of glaciers (c) Food crisis

o
b
"Capital Infusion". How much is the need of PSBs (d) Climate change (e) None of these

.w
collectively for the purpose ? (` in crores) ? 168. An agreement between two nations or a group of nations

o
rd
(a) 10,000 (b) 15,000 which establishes unimpeded exchange and flow of goods

p
(c) 20,000 (d) 25,000 and services between/among trade partners regardless of

re
(e) 30,000 national boundaries is called -

s
s
161. Who amongst the following has purchased about 200 (a) Import Free Agreement

.c
o
tonnes of gold from International Monetary Fund (IMF) in (b) Free Trade Agreement

m
2009 ? (c) Export Free Agreement
(a) Infrastructure Finance Company Ltd. (d) Special Economic Zone Agreement
(b) Bombay Stock Exchange of India (e) None of these
(c) Gold Council of India 169. Which of the following correctly describe what "One Carbon
(d) Reserve Bank of India Credit" is ?
(e) None of these (A) Credit permit to release one ton of carbon dioxide.
162. One of the major emphasis of Basel II is that banks should (B) Providing loans to establish one new production unit
have which can produce carbon dioxide for industrial use.
(a) adequate Capital Adequacy Ratio (C) Finding out one new business which can use and
(b) only few branches in urban centres recycle green house gases.
(c) more and more branches in rural areas (a) Only C (b) Only B
(d) core banking mode of operation (c) Only A (d) All A, B and C
(e) all the above to comply with Basel II norms (e) None of these
163. As per the reports published by various agencies/ 170. As per various reports issued by the UNO and other world
organisation, it is estimated that around 3 billion extra people bodies, agricultural and allied activities consume what
will join our existing population of 6 billion by the percentage of global water resources every year ? About –
year-2050. To provide food for all of them, a second round (a) 15% (b) 30%
of which of the following initiatives is required to be started (c) 50% (d) 70%
much in advance ? (e) 60%
(a) Green Revolution (b) Planned Urbanisation 171. The Food and Agriculture Organisation (FAO) is a wing/
(c) Industrialisation (d) Climate Migration agency of the
(e) All of these (a) World Bank
164. Who amongst the following is the President of the European (b) United Nations Organisation
Union at present ? (c) Asian Development Bank
(a) Catherine Ashton (b) Mr. Jose Manuel Barroso (d) Ministry of Agriculture Government of India
(c) Mr. Fredrik Reinfeldt (d) Mr. Sergi Lavrov (e) None of these
(e) None of these 172. As a policy India does not encourage tree movement of
165. As per the reports published in various newspapers and unskilled labours from
journals almost all big nations are trying their best to resolve (a) Gulf Countries
various disputes between the nations where in either they
(b) USA and South American Nations
are a party or because of that their peace and stability may
(c) European Union Nations
be affected. Stability in which of the following two nations
is more vital to India than that of any other nations ? (d) All these nations
(a) Bangladesh & China (b) China & USA (e) None of these
(c) Iraq & Russia (d) South & North Korea 173. The Government of India has decided to disinvest its share
(e) Pakistan & Afghanistan in major listed public sector undertakings. How much of its
share will be off loaded in this method?
166. India's bilateral per annum trade with European Union at
(a) 2% (b) 4%
present is about-
(c) 8% (d) 10%
(a) 50 billion US $ (b) 75 billion US $
(e) None of these
(c) 100 billion US $ (d) 150 billion US $
174. The United Nations Development Fund for Women
(e) 200 billion US $
(UNIFEM) is working in an Indian city to make it safe for
167. Some world bodies/organisations/agencies are of the view
women. Which of the following is that city ?
that agriculture should adopt a "Cross Disciplinary
(a) Pune (b) Kolkata
Approach", and fake help of all types of sciences and studies.
This approach will help in which of the following crucial (c) Delhi (d) Mumbai
problems being faced by humanity ? (e) Lucknow
y
o
u
rs
m
18 SBI Rural Business PO Exam 2010

a
h
b
175. At present the trade between India and China is in a state of 183. Sachin Tendulkar has recently become the first cricketer in

o
"Payment Imbalance". What does this mean in real terms ? ODI history to cross the milestone of-

o
b
(A) China does not import many items from India whereas (a) 10000 runs (b) 12000 runs

.w
India imports more from China. (c) 15000 runs (d) 17000 runs

o
rd
(B) China does not pay India in time and a lot of delay is (e) 20000 runs

p
reported by the exporters. 184. The East Asia Summit took place in October 2009 in Hua

re
(C) India wants all payments to be made in US Dollars Hin town. This is a town in -

s
s
where as China pays in its own currency. (a) China (b) Myanmar

.c
o
(a) Only A (b) Only B (c) Cambodia (d) Malaysia

m
(c) Only C (d) All A, B and C (e) Thailand
(e) None of these 185. What percent of India's GDP is invested in providing
176. India has signed an agreement on climate change with which education?
of the following countries recently ? (a) 1% (b) 2.5%
(a) USA (b) Russia (c) 4% (d) 6%
(e) None of these
(c) United Kingdom (d) Australia
186. Memory, also called random access memory, or RAM,.
(e) China
(a) contains the electronic circuits that cause processing
177. Some Scientists are of the opinion that extra agricultural to occur
production should be achieved without going for the
(b) makes the information resulting from processing
cultivation of extra land and also without causing damage
available for use
to essential eco-system. This concept, in other words, is
(c) allows data, programs, commands, and user responses
known as -
to be entered into a computer
(a) Sustainable growth (b) Renewability of inputs (d) consists of electronic components that store data
(c) Global Melt down (d) Climate Migration (e) None of these
(e) None of these 187. Correcting errors in a program is referred to as –
178. Mr. Z. E. A. Ben Ali has taken over as the President of - (a) debugging (b) bugging
(a) Tunisia (b) Malaysia (c) rectifying (d) modifying
(c) Indonesia (d) Kuwait (e) None of these
(e) None of these 188. An assembler is used to translate a program written in.
179. Which of the following is NOT a Government of India (a) a low level language (b) machine language
undertaking? (c) a high level language (d) assembly language
(a) Oil India Ltd. (e) None of these
(b) Sutlej Jal Vidyut Nigam Ltd. 189. The capability of the operating system to enable two or
(c) ICICI Bank more than two programs to execute simultaneously in a
(d) National Thermal Power Corporation Ltd. single computer system by using a single processor is.
(e) Rural Electrification Corporation Ltd. (a) Multiprocessing (b) Multitasking
(c) Multiprogramming (d) Multiexecution
180. India shares its border with China along the Himalayan
range. Which of the following states touches the border of (e) None of these
China 190. The basic goal of computer process is to convert data into–
(a) Punjab (b) Bihar (a) information (b) tables
(c) Arunachal Pradesh (d) Chhattisgarh (c) files (d) graphs
(e) None of these
(e) All of these
191. A hard copy of a document is –
181. Which of the following has been the growth of India's GDP
(a) stored in the hard disk (b) stored on a floppy
during 2008-09 ? (Final data are issued in late 2009)
(c) stored on a CD (d) printed on the printer
(a) 5% (b) 5.5%
(e) None of these
(c) 6% (d) 6.5%
192. Which of the following in not an output device ?
(e) None of these
(a) Plotter (b) Printer
182. Which of the following ministries has launched a new "Skill
(c) Scanner (d) Monitor
Development Initiative Scheme" so that the employability
(e) None of these
of the young people can be increased ?
193. The sequence of a sales process is
(a) Ministry of Finance
(a) a call, a lead, presentation and sale
(b) Ministry of Agriculture
(b) a lead, a call, presentation and sale
(c) Ministry of Heavy Industry (c) presentation, sale, lead and call
(d) Ministry of Home Affairs (d) presentation, lead, sale and call
(e) Ministry of Labour and Employment (e) sale, call, lead and presentation
y
o
u
rs
m
SBI Rural Business PO Exam 2010 19

a
h
b
194. A presentation means (c) special products for each customer

o
(a) display of products (d) better relations

o
b
(b) explaining the utility of products (e) All of these

.w
(c) a gift 198. Customer Retention means

o
rd
(d) display of communication skills (a) retaining the customers at the Bank for the full day

p
(e) All of these (b) quick disposal

re
s
195. A 'lead' means (c) customers dealing with the same bank for a long time

s
.c
(a) a buyer (d) better standards

o
m
(b) a seller (e) All of these
(c) a company intending to sell its products 199. Value-added services means
(d) a prospective buyer (a) giving full value for money
(e) a disinterested buyer (b) better value for better price
196. Benchmark' means (c) costlier service
(a) products line up on bench (d) additional service
(b) salesmen sitting on a bench (e) All of these
(c) set standards 200. "POS" means (in marketing)
(d) marks on a bench (a) Preparation for Sales
(e) None of these (b) Point of Superiority
197. 'Customisation' means (c) Point of Sales
(a) customers' personal accounts (d) Primary Outlook of Salesmen
(b) customers selling goods (e) Position of Sales
y
o
u
rs
m
20 SBI Rural Business PO Exam 2010

a
h
b
o
o
b
.w
o
rd
Required distance = BE – AB

p
1. (c) How many goals scored Þ 5 3 9 7 = (100 – 50) m = 50 m

re
Direction Þ North

s
many more matches Þ9 8 2

s
8. (b) 15 – 8 × 6 ¸ 12 + 4 = ?

.c
he scored five Þ1 6 3

o
Þ ? = 15 × 8 ¸ 6 + 12 – 4

m
Codes are: Þ ? = 20 + 12 – 4 = 28
how Þ 5 or 7
many Þ 9 more Þ 8 or 2 B
goals Þ 5 or 7 matches Þ 8 or 2
he Þ 1 or 6 scored Þ 3
five Þ 1 or 6
The code for goals is either 5 or 7.
F D H
T E M P O R A L
9. (d)
–1 +1
2. (a)
O L D S M B S P H
Similarly,
So there is no sufficient data to determine the direction
G O N S I D E R of Town H.

–1 +1 19 5 1 18 3 8 5 19
10. (d) S E A R C H E S
R M N B S F E J

3. (d) Meaningful words Þ IDLE, LIED | Diel Bottles


4. (d) D > A, B, E > C
Cups Jugs Plate Tables
5 2 6 3 1 8 7 11. (e)
5. (b)
6 1 5 4 2 7 8
1<2<4<5<6<7<8 Or
6. (c) According to Pratap his mother’s birthday may be on
20th, 21st or 22nd April. Bottles
According to Pratap’s sister their mother’s birthday
may be from 1st April to 21st April. Cups Jugs
Common Dates Þ 20th and 21st Plate
7. (d)
Tables
D 30 m E
Conclusions:
N I. (False)
II. (False) Either
III. (False)
W E IV. (False)
100 m A
Mats
S
Pots Buses
50 m
Handles
12. (e) Or
C 30 m B
y
o
u
rs
m
SBI Rural Business PO Exam 2010 21

a
h
III. False

b
o
IV. False

o
Mats

b
So none of the conclusion follows.

.w
Pots (16-20):

o
rd
Handles

p
Employee Department Favourite

re
Sport

s
s
A Personnel Table Tennis

.c
Buses

o
B Administration Football

m
Conclusions: C Administration Hockey

I. (False) D Administration Basketball


II. (True) Either E Marketing Cricket
III. (False)
IV. (True) F Personnel Volleyball
G Marketing Lawn Tennis
H Marketing Badminton
Tigers
16. (c) B, C and D work in Administration Department.
Horses 17. (b) E works in Marketing Department.
18. (e) None of the Combinations is correct.
Birds 19. (a) E’s favourite sport is Cricket.
Lions Monkey
13. (a) 20. (d) G’s favourite sport is Lawn Tennis.

$ Þ³ @ Þ> # Þ<
21-25: d Þ= Þ£
Conclusions:
I. True 21. (b) H @ T Þ H > T
II. False T#F ÞT<F
III. True F dE Þ F = E
IV. False E V ÞE £V
So only conclusion I & III follows. Therefore, H > T < F = E £ V
Conclusions
Houses I. V $ F Þ V ³ F : True
Roads II. E @ T Þ E > T : True
Walls Jungles III. H @ V Þ H > V : Not True
14. (c)
Benches IV. T # V Þ T < V : True
So conclusion I , II & IV follow
22. (e) D # R Þ D < R
Conclusions:
I. False R K ÞR£K
II. False K@ F Þ K>F
III. True F$J ÞF³J
IV. True Therefore, D < R £ K > F ³ J
So only conclusion III & IV follows. Conclusions
I. J # R Þ J < R : Not True
II. J # K Þ J < K : True
III. R # F Þ R < F : Not True
Shirts Sticks IV. K @ D Þ K > D : True
So only conclusion II & IV follow
Dresses Lamps 23. (e) N d B Þ N = B
15. (a) B$W Þ B³W
W # H ÞW< H
Flowers
H MÞ H £ M
Therefore, N = B ³ W < H £ M
Conclusions: Conclusions
I. False I. M @WÞ M > W : True
II. False II. H @ N Þ H > N : Not True
y
o
u
rs
m
22 SBI Rural Business PO Exam 2010

a
h
III. W d N Þ W = N : Not True 34. (a) Course of action (A) is only feasible solution because

b
o
IV. W # N Þ W < N : Not True the government can neither close these schools nor

o
b
W is either smaller than or equal to N. Therefore either III or issue arrest warrants for all the parents.

.w
IV is true. 35. (d) Clearly course of action (B) and (C) should be followed

o
24. (a) R D Þ R £ D because terminating official in charge of recruitment

rd
D$ J Þ D ³J procers is not a solution to the problem.

p
re
J#M ÞJ<M 37. (c) The IT and ITES companies have now decided to visit

s
the engineering college campuses for tier II cities in

s
M @ KÞ M > K

.c
Therefore, R £ D ³ J < M > K India as well.

o
38. (b) The petroleum companies will increase the prices of

m
Conclusions
I. K#J ÞK<J : Not True petrol and diesel by about 10 percent.
II. D @ MÞ D > M : Not True 39. (d) Large number of branches of many government owned
banks in the rural areas are making huge losses every
III. R # M Þ R < M : Not True
year due to lack of adequate business activities.
IV. D @ K Þ D > K : Not True
40. (a) Consider the following line of the passage:
So none of the conclusion follows
“Mounting subventions for subsidies means diversion
25. (e) M $ K Þ M ³ K
of savings by the government from investment of
K@ N ÞK >N
consumption, raising the coast of Capital in the
N RÞN£R process”.
R# W ÞR<W 41. (d) Consider the following lines of the passage:
Therefore, M ³ K > N £ R < W “The government must cut expenditure on subsidies
Conclusions to create more fiscal space for investments in both
I. W @ KÞ W > K : Not True physical and social infrastructure”.
II. M $ R Þ M £ R : Not True 42. (c) The government must reduce expenditure on subsidies
III. K @ W Þ K > W : Not True so that to create more in frastructural facilities otherwise
IV. M@ N Þ M > N : True present level of subsidy cannot continue for a long
26-30: time.
43. (c) According to passage the most recent poverty
Conditions restumates by an expert group has missed the crucial
Candidate
(i) or (A) (ii) or (B) (iii) (iv) (v) dynamism so we can conclude procers of poverty
Shobha P – – P P P P measurement needs to take into account various factors
Rohan P – NG – P P P to tackle its dynamic nature.
Prakash – P P – P P P 44. (a) As per passage the net increase of about percentage
P P P P points was seen to have a considerable variation
Sudha – – ×
acrcers states and regions so it is abvious that it may
Amit P – P – P P P not be possible to have an accurate poverty
measurement in India.
26. (a) Shobha Gupta does satisfy conditions (i), (B), (iii), (iv) 45. (b) From the facts and data given in passage we can make
and (v). Therefore, her case would be referred to inference that increase in number of persons falling
Executive Director. into poverty varies considerably across the country
27. (c) It is not mentioned Rohan Maskare worked in which over a period of time.
section. 46. (d) In each subsequent figure one smaller design rotates
28. (b) Prakash Gokhale does satisfy conditions (A), (ii), (iii), through 90° clockwise or anticlockwise and it is replaced
(iv) and (v). Therefore, his case would be referred to with a new design.
General Manager - Advances.
47. (b) In the subsequent figures the design ( ) changing its
29. (d) Sudha Mehrotra does not satisfy condition (v).
position 45° and 90° anticlockwise alternately. The
30. (e) Amit Narayan does satisfy all the conditions. Therefore,
design reversing its position in each figure and after
he can be selected.
every 2 figure black circle in changing into empty circle
31. (d) Immediate course of action to avert the difficult
so (b) option is correct.
situation is to send relief rupplics in affected areas
48. (c) The design (A) changing its position on clockwise
course of action I is a long term action plan which is
direction through one half and one step alternately
time consuming and course of action II is not feasible
while the design (T) moves on half step on clockwise
practically.
direction and the central design is replaced with new
32. (a) None of course of action is feasible because the
design on each ruleseqvent figure.
government cannot interfere in bank’s internal matter
49. (a) In th e subsequent figures the square moves
but the government can componsate the huge losser
respectively one-half , one, one and a half, two ..........
incurred by bank.
step(s) in anticlockwise direction, the design (A) moves
33. (b) The course of action (A) and (C) are not feasible
along diagonal while the design (K) moves one-half
because in a democratic country the government
and one step in clockwise direction alternately. The
cannot impose ban on these things.
number of circles is the same in alternate figures.
y
o
u
rs
m
SBI Rural Business PO Exam 2010 23

a
h
50. (e) In each subsequent figure the centre design rotates Number of ways of selecting 2 marbles from urn = 15C2

b
o
through 45° anticlockwise and all other designs move = 105

o
b
one-half step in anticlockwise direction.

.w
15 1
51. (a) Total number of candidates clearing the entrance exam Required Probability = =

o
from state B in 2004 105 7

rd
58. (c) Number of ways of selecting 2 blue and one yellow

p
æ 1.04 ´ 51 ö marble = 4C2 × 3C1= 6 × 3 = 18

re
=ç ÷ lakh
è 100 ø Number of ways of selecting 3 marble from urn = 15C3

s
s
.c
Total number of candidates clearing the entrance exam = 455

o
from state C in 2004

m
18
Required Probability =
æ 1.11´ 32 ö 455
=ç ÷ lakh 59. (a) Number of ways of selecting one green, two blue and
è 100 ø
\ Required ratio one red marble = 2C1 × 4C2 × 6C1.
= 53.04:35.52 =2 × 6 × 6 = 72
= 221 : 148 Number of ways of selecting 4 marbles from urn = 15C4
52. (c) Number of candidates clearing the entrance exam from 12 ´13 ´14 ´15
state D: = = 1365
4 ´ 3 ´ 2 ´1
1.83 ´ 60
Year 2008: Þ 72 24
100 Required Probability = =
1365 455
= 1.098 lakh
60. (d) Number of ways of selecting either two green marbles
2.01´ 56 or two yellow marbles = 2C2 + 3C2 = 1 + 3 = 4
Year 2009: Þ Number of ways of selecting 2 marbles = 15C2 = 105
100
= 1.1256 lakh 4
53. (d) Number of candidates not clearing the entrance exam Required Probability =
105
from state A in 2007.
61 - 65:
= 59% of 1.98 lakh
æ 1.98 ´ 59 ö Department No. of No. of No. of employees
=ç ÷ lakh = 116820 employees males got promoted
è 100 ø
54. (b) Required number of candidates clearing the exam Production 1260 1020 396
æ 1.42 ´ 49 1.58 ´ 26 ö IT 540 408 312
=ç + ÷ lakh
è 100 100 ø HR 432 204 132
= (0.6958 + 0.4108) lakh Marketing 648 306 264
= 110660
Accounts 720 102 96
55. (d) Required average number of candidates.
æ 1.88 + 1.83 + 2.01 ö 2 61. (e) Number of males Promoted from the IT department
=ç ÷ lakh =190666 1
è 3 ø 3
= ´ 312 = 156
5.72 2
56. (b) Number of way lakh of selecting 4 marbles out of
3 156
Required % = ´ 100 » 38%
15 marbles 408
15 ´14 ´13 ´ 12 62. (c) Total number of females working in production and
= 15C4 = = 1365 marketing departments together
4 ´ 3 ´ 2 ´1 = (1260 – 1020) + (648 – 306)
Number of ways of selecting 4 marbles when no one is = 240 + 342 = 582
11 ´10 ´ 9 ´ 8 63. (a) Females working in Accounts department = (720 – 102)
blue = 11C4 = = 330 = 618
4 ´ 3´ 2 ´1
Probability of getting 4 marble (when no one is blue) 1200
64. (d) Required % = ´ 100 » 33%
330 22 3600
= =
1365 91 132
65. (b) Required % = ´ 100 » 30.56%
22 69 432
Probability that at least one is blue = 1 -= 66. (b) Profit of company L in the year 2006
91 91
57. (e) Number of ways of selecting 2 red marbles from 6 red æ 125 ö
marbles = 6C2 = 15 = ` ç 1.84 ´ ÷ lakh = ` 2.3 lakh
è 100 ø
y
o
u
rs
m
24 SBI Rural Business PO Exam 2010

a
h
67. (e) Profit of company M in the year 2006. 80. (a) Required average profit

b
o
o
æ 100 100 ö æ 4.99 + 5.82 ö

b
= ` ç 3.63 ´ ´ ÷ lakh =`2.52 l lakhakh = `ç ÷ thousand

.w
è 115 125 ø è 2 ø

o
68. (e) Average percentage rise in profit of company L over = ` 5405

rd
the years 81. (e) Part of the chair prepared by X and Y in 1 day.

p
re
20 + 15 + 25 + 30 + 35 + 30 1 1 2 +1 3

s
= + = =

s
6 =

.c
2 4 4 4

o
155 5

m
= = 25 % 4
6 6 Hence, 1 chair will be prepared in days.
3
69. (a) It is clear from the graph. 82. (c) Part of the table prepared by X, Y and Z in 1 day =
70. (d) Required percentage rise
1 1 1 2 +1+ 3
35 - 20 + + = =1
= ´ 100 = 75 3 6 2 6
20
Hence, the table will be completed in 1 day.
71-75 :
Total number of boys 83. (a) Required number of days.
= 3 + 2 + 12 + 15 = 32
1560 ´ 7 84. (b) Radius of the circular field = side of the square
= = 910
12
784
Total number of gifts = = 196 feet
=1560-910=650 4
\ Area of the circular field
Hobby B oys Girls 22
2
Pain tin g 385 130 = p´ r = ´196 ´196 =120736 sq.ft.
7
Sin g in g 91 182
85. (d) The number of letters in the word STRESS is six of
Dan cin g 182 65
which ‘S’ comes thrice.
Dan cin g an d Sin g in g 78 156
Dan cin g an d Pain tin g 104 52 6!
\ Required number of arrangements =
3!
Dan cin g , Pain tin g 70 65
an d Sin g in g 6 ´ 5 ´ 4 ´ 3 ´ 2 ´1
= = 120
1´ 2 ´ 3
71. (d) Total number of boys enrolled in dancing class. 86 - 90:
= 182 + 70 + 104 + 78 = 434
72. (e) Number of girls enrolled in singing class Locality No. of Men No. of No. of children
= 156 + 182 + 65 = 403 women
\ Required percentage F 3102 1974 564
403 G 1649 2134
= ´ 100 = 26 1067
1560
H 2496 2028 676
73. (a) Required number of students
= 70 + 65 = 135 I 3913 1505 602
74. (b) Required percentage J 2058 2009 833
65
= ´ 100 = 35.71
182 86. (c) Required percentage
75. (c) Required ratio 4900
= 130 : 385 = 26 : 77 = ´ 100 » 87%
5640
76. (b) Required ratio
= (6.19 + 6.23) : (5.04 + 5.12) 87. (b) Number of children in the localities H and I.
= 12.42 : 10.16 = 621 : 508 = 676 + 602 = 1278
77. (d) Required percentage increase 88. (d) Number of women in G is highest.
89. (c) Number of men and children in locality I.
( 5.28 - 5.11) 3913 + 602 = 4515
= ´100 = 3.33
5.11 90. (a) Required ratio
78. (e) Shopkeeper ‘S’ kept increasing his profit continuously. = 3102 : 2496
79. (c) Required difference in profit = 517 : 416
= ` (5.69 – 5.31) thousand = ` 380
y
o
u
rs
m
SBI Rural Business PO Exam 2010 25

a
h
b
100. (e) Required difference
éæ ù

o
T
R ö

o
91. (b) CI = P êç1 + ÷ - 1ú 50248 ´ 21 51124 ´17

b
100 ø -
ëêè úû =

.w
100 100

o
10551 – 8691 = 1860

rd
éæ 8 ö
2 ù 101. (d) Making a case for the international society of engineer
1414.4 = P êç1 + ÷ - 1ú

p
Þ a second Green Revolution.

re
100 ø
ëêè ûú

s
102. (b) Withdrawal of fiscal impetus from agriculture to other

s
1414.4 = p × 0.1664

.c
sectors

o
103. (c) Agricultural growth has artificially inflated India’s GDP

m
1414.4
Þ P= =` 8500 and such growth is not real.
0.1664
104. (b) It can be inferred from the fourth paragraph.
\ Amount =` (8500 + 1414.4) 105. (b) The realization of the link between food security and
= ` 9914.4 political stability.
92. (c) Let the ages of the mother and daughter be 7x and x 106. (c) To make those countries more self sufficient to whom it
years respectively. previously provided food
7x - 4 19 107. (c) It prompted increased investment and interest in
\ Four years ago, = agriculture.
x-4 1 108. (a) Misapprehension that it would alleviate poverty as it
Þ 19x – 76 = 7x – 4 did it other countries
Þ 12x = 72 = x = 6 109. (e)
\ Mother’s age after four years 110. (c)
= 7x + 4 = 7 × 6 + 4 = 46 years 111. (a) The meaning of the word Starve (Verb) as used in the
93. (d) Required time = LCM of 12, 18 and 20 seconds. passage is: keep deprived of; to not give something
180 seconds = 3 minutes that is needed.
94. (a) 4 × 2 men = 4 × 4 women = 5×4 children Look at the sentences:
Þ 2 men = 4 women = 5 children The department has been starved of resources.
\ 2 men + 4 women + 10 children I felt starved of intelligent conversation.
= 20 children Hence, the word starved and deprived are synonymous.
\ M1D1 = M2D2 112. (b) The meaning of the word Slap (Verb) as used in the
Þ 5 × 4 = 20 × D2 Þ D2 = 1 day passage is : Impose; to order especially in a sudden or
95. (e) Speed of the boat is still water an unfair way, that something must happen or
somebody must do something.
1 Look at the sentences:
= (Rate downstream + Rate upstream)
2 The company slapped a ban on using email on the staff.
Hence, the word slapped and imposed are synonymous.
1 113. (a) The meaning of the word Plow (Verb) as used in the
= (32 + 28) = 30 kmph.
2 passage is : to invest a large amount of money in a
96. (c) The difference was minimum in the year 2007. company or project; to cultivate.
In the year 2007 Hence, the words ploweel and imposed are
Difference = 32438 – 29129 synonymous.
= 3309 114. (b) The meaning of the word Pressing (Adjective) as used
97. (a) Number of candidates passed from Chennai in the passage is: urgent, serious, insistent, needing to
be dealt with immediately.
55492 ´ 13 Hence, the word pressing and undemanding are
Year 2005 Þ = 7214
100 antonymous.
115. (c) The meaning of the word Evaporate (Verb) as used in
58492 ´ 14 the passage is: to disappear, especially by gradually
Year 2007 Þ = 8189
100 becoming less and less.
98. (d) Number of candidates passed from Delhi in 2002 and Look at the sentences:
2006 His confidence had now completely evaporated.
Hence, the words evaporated and grew the antonymous.
58248 ´ 28 59216 ´ 20 116. (c) Suggested that only large scale agitations have been
= +
100 100 effective in bringing out desired change in the past.
= 16309 + 11843 117. (d) Many residential areas continue to use swimming pools,
= 28152 »28150 wasting large quantities of water.
99. (b) Required number of passed candidates. 118. (a) He still seems to be leading his life luxuriously and
extravagantly
71253 ´ 19 119. (e)
= = 13540
100
y
o
u
rs
m
26 SBI Rural Business PO Exam 2010

a
h
120. (d) The residents must really be impressed with the political

b
148. (e) The word Plethora (Noun) means: an amount that is

o
style of their mayor greater than is needed; excess.

o
b
121. (e) 122. (c) 123. (b) 124. (d) The word Abundance (Noun) means: a large quantity

.w
125. (a) 126. (c) 127. (b) 128. (e) that is more than enough.

o
rd
129. (a) 130. (c) 149. (a) The word Inadvertently (Adverb) means:

p
131. (a) The event shows past time. Hence simple past or unintentionally; without intending to; by accident.

re
‘passed a bill that proposed’ will be a correct usage. The word purposefully and inadvertently are

s
s
132. (e) No correction required

.c
antonymous.
133. (c) In comparison between two things, comparative Degree

o
150. The word Germane (Adjective) means: connected with

m
is used. Use of double comparatives is not proper.
something in an important or appropriate ways;
Hence, ‘as a healthier option’ should be used.
relevant.
134. (d) For living beings Relative pronoun ‘who’ is used.
The word germane and irrelevant are antonymous.
Hence, who were earlier unaware will be a correct usage.
151. (d) 152. (e) 153. (a) 154. (a)
135. (b) The event shows past time. Hence, ‘while most
industries were’ should be used. 155. (c) 156. (c) 157. (b) 158. (c)
136. (b) conducive 137. (d) question 159. (c) 160. (d) 161. (d) 162. (a)
138. (a) cannot 139. (c) need 163. (e) 164. (e) 165. (e) 166. (b)
140. (e) growth 141. (c) favour 167. (c) 168. (b) 169. (c) 170. (d)
142. (b) enjoys 143. (a) aid 171. (b) 172. (d) 173. (d) 174. (c)
144. (e) increased 145. (a) both 175. (d) 176. (e) 177. (a) 178. (a)
146. (c) The word Nascent (Adjective) means: beginning to 179. (c) 180. (c) 181. (d) 182. (b)
exist; not yet fully developed. 183. (d) 184. (e) 185. (a) 186. (d)
The word nascent and emerging are synonymous. 187. (a) 188. (d) 189. (b) 190. (a)
147. (a) The word Eccentric (Adjective) means: considered by 191. (d) 192. (c) 193. (b) 194. (b)
other people to be strange or unusual. 195. (d) 196. (c) 197. (c) 198. (c)
The word eccentric and abnormal are synonymous. 199. (d) 200. (c)
y
o
u
rs
m
a
h
b
o
SBI & ITS ASSOCIATES BANK PO

o
b
.w
o
EXAM 2010 Based on Memory

rd
p
re
s
s
.c
Reasoning Ability (a) %653 (b) %@63

o
m
(c) %5@3 (d) %@53
1. If 'R' denotes '–'; 'Q' denotes '×'; 'W' denotes '÷' and 'A' (e) None of these
denotes '+', then – 9. How many meaningful English words can be formed with
42 W 7 R 8 A 6 Q 4 = ? the letters LGEU using each letter only once in each word ?
(a) –22 (b) –168 (a) None (b) One
(c) 22 (d) 28
(c) Two (d) Three
(e) None of these
(e) More than three
2. Mohan walked 30 metres towards South, took a left turn
and walked 15 metres. He then took a right turn and walked 10. In a certain code THRIVES is written as SIUHRDU. How is
20 metres. He again took a right turn and walked 15 metres. SOULFUL written in that code ?
How far is he from the starting point ? (a) VPTKKTE (b) VPTKETK
(a) 95 metres (b) 50 metres (c) TPVKKTE (d) TNRKMVG
(c) 70 metres (d) cannot be determined (e) None of these
(e) None of these
DIRECTIONS (Qs. 11–15): In each of the questions below are
3. What should come next in the following letter series based given four statements followed by four conclusions numbered I,
on English alphabet ?
II, III and IV. You have to take the given statements to be true
CEA IKG OQM ? even if they seem to be at variance from commonly known facts.
(a) STW (b) WUS Read all the conclusions and then decide which of the given
(c) SWU (d) UWS conclusions logically follows from the given statements
(e) None of these disregarding commonly known facts.
4. The positions of how many digits in the number 59164823
will remain unchanged after the digits are rearranged in 11. Statements :
descending order within the number ? Some trains are cars.
(a) None (b) One All cats are branches.
(c) Two (d) Three All branches are nets.
(e) More than three Some nets are dresses.
5. What should come next in the following letter series ? Conclusions :
PQ R S TAB C D E P Q R S I. Some dresses are cars
AB C D E PQ R SABC D PQ II. Some nets are trains.
(a) R (b) T III. Some branches are trains.
(c) A (d) B IV. Some dresses are trains.
(e) None of these (a) Only I and III follow (b) Only II and III follow
6. In a certain code language, 'how can you go' is written as 'ja (c) Only I and IV follow (d) Only II, III and IV follow
da ka pa,' 'can you come here' is writter as 'na ka sa ja' and (e) None of these
'come and go' is written as 'ra pa sa'. How is 'here' written in 12. Statements:
that code language ? Some pencils are kites.
(a) ja (b) na Some kites are desks.
(c) pa (d) Data inadequate All desks are jungles.
(e) None of these All jungles are mountains.
7. How many such pairs of letters are there in the word Conclusions :
TRIBUNAL each of which has as many letters between I. Some mountains are pencils.
them in the word as in the English alphabet ? II. Some jungles are pencils.
(a) None (b) One III. Some mountains are desks.
(c) Two (d) Three IV. Some jungles are kites.
(e) More than three (a) Only I and III follow (b) Only I, II and III follow
8. In a certain code DOWN is written as '5@9#' and NAME is (c) Only III and IV follow (d) Only II, III and IV follow
written as '#6%3'. How is MODE written that code ? (e) None of these
y
o
u
rs
m
28 SBI & its Associates Bank PO Exam 2010

a
h
b
13. Statements : 18. Who is third to the right of K ?

o
All papers are clips. (a) F (b) E

o
b
Some clips are boards. (c) G (d) Data inadequate

.w
Some boards are lanes. (e) None of these

o
rd
All lanes are roads. 19. Who is to the immediate right of F ?

p
Conclusions : (a) B (b) G

re
I. Some roads are boards. (c) E (d) Data inadequate

s
s
II. Some lanes are clips. (e) None of these

.c
o
III. Some boards are papers. 20. In which of the following combinations is the third person

m
IV. Some roads are clips. sitting between the first and the second persons ?
(a) Only I and II follow (b) Only I and III follow (a) GFB (b) BGH
(c) Only I, II and III follow (d) Only II, III and IV follow (c) ADC (d) KEC
(e) None of these (e) EGF
14. Statements :
DIRECTIONS (Qs. 21–25): In the following questions, the
All pens are clocks.
symbols d, @, ©, % and * are used with the following meaning
Some clocks are tyres.
as illustrated below :
Some tyres are wheels.
All wheels are buses. 'P © Q' means 'P is not smaller than Q'.
Conclusions : 'P % Q' means 'P is neither smaller than nor equal to Q'.
I. Some buses are tyres. 'P * Q' means 'P is neither greater than nor equal to Q'.
II. Some wheels are clocks. 'P d Q' means 'P is not greater than Q'.
III. Some wheel are pens. 'P @ Q' means 'P is neither greater than nor smaller than Q'.
IV. Some buses are clocks. Now in each of the following questions assuming the given
(a) None follows (b) Only I follows statements to be true, find which of the four conclusion I, II, III
(c) Only II follows (d) Only III follows and IV given below them is/are definitely true and give your
(e) Only IV follows answer accordingly.
15. Statements : 21. Statements :
All stones are hammers. R * K, K % D, D @ V, V d M
No hammer is ring. Conclusions :
Some rings are doors. I. R * D
All doors are windows. II. V * R
Conclusions : III. D @ M
I. Some windows are stones. IV. M % D
II. Some windows are rings. (a) None is true (b) Only III is true
III. No window is stone. (c) Only IV is true (d) Only either III or IV is true
IV. Some rings are stones. (e) Only either III or IV and II are true
(a) Only I follows 22. Statements :
(b) Only II follows F % N, N © W, W d Y, Y * T
(c) Only III follows Conclusions :
(d) Only either I or III follows I. F % W
(e) Only either I or III and II follow II. T % N
III. N % Y
DIRECTIONS (Qs. 16–20): Study the following information
IV. T % W
carefully and answer the questions given below :
(a) Only I and III are true (b) Only I and IV are true
A, B, C, D, E, F, G, H and K are sitting around a circle facing the (c) Only II and III are true (d) Only I, II and IV are true
centre. F is fourth to the right of A who is third to the right of B. K (e) None of these
is fourth to the left of B and third to the right of D. C is third to the 23. Statements :
right of H. E is second to the left of G. B © T, T * R, R % F, F @ K
16. Who is fourth to the left of G ? Conclusions :
(a) C (b) A I. B % R
(c) D (d) K II. F * T
(e) Data inadequate III. R % K
17. What is E's position with respect to B ? IV. K * T
(a) Second to the left (b) Third to the right (a) None is true (b) Only I is true
(c) Fourth to the right (d) Third to the left (c) Only II is true (d) Only III is true
(e) Fifth to the right (e) Only IV is true
y
o
u
rs
m
SBI & its Associates Bank PO Exam 2010 29

a
h
b
24. Statements : (B) No, there is a huge shortage of electricity in most parts

o
J @ F, F d N, N % H, H © G of the country and hence generation of electricity

o
b
Conclusions : needs to be augmented.

.w
I. G * N (C) No, many developed countries continue to set up

o
rd
II. N © J thermal power plants in their countries.

p
III. F * J (a) None is strong

re
IV. J d G (b) Only (A) is strong

s
s
(a) Only I and II are true (c) Only (B) is strong

.c
(b) Only I, II and III are true

o
(d) Only (C) is strong

m
(c) Only II, III and IV are true (e) Only either (A) or (B) is strong
(d) All I, II, III and IV are true 28. Statement: Should there be a restriction on the construction
(e) None of these of high rise buildings in big cities in India.
25. Statements : Arguments :
D d T, T @ R, R © M, M % K (A) No, big cities in India do not have adequate open land
I. R @ D plots to accommodate the growing population.
II. R % D (B) Yes, only the builders and developers benefit from the
III. K * T construction of high rise buildings.
IV. M d T (C) Yes, the Government should first provide adequate
(a) Only either I or II is true infrastructure facilities to existing buildings before
(b) Only III and IV are true allowing the construction of new high rise buildings.
(c) Only either I or II and III are true (a) Only (B) is strong
(d) Only either I or II and IV are true (b) Only (C) is strong
(e) Only either I or II and III and IV are true (c) Only (A) and (C) are strong
DIRECTIONS (Qs. 26–30) : In making decisions about important (d) Only (A) is strong
questions, it is desirable to be able to distinguish between (e) None of these
'strong' arguments and 'weak' arguments. 'strong' arguments 29. Statement : Should road repair work in big cities be carried
are those which are both important and directly related to the out only late at night ?
question. 'Weak' arguments are those which are of minor Arguments :
importance and also may not be directly related to the question (A) No, this way the work will never get completed.
or may be related to a trivial aspect of the question. (B) No, there will be unnecessary use of electricity.
(C) Yes, the commuters will face lot of problems due to
Each question below is followed by three arguments numbered
repair work during the day.
(A), (B) and (C). You have to decide which of the arguments is a
'strong' argument and which is a 'weak' argument. (a) None is strong
26. Statement : Should there be a cap on drawing groundwater (b) Only (A) is strong
for irrigation purposes in India ? (c) Only (C) is strong
Arguments : (d) Only (B) and (C) are strong
(A) No, irrigation is of prime importance for food production (e) Only (A) and (B) are strong
in India and it is heavily dependent on groundwater in 30. Statement : Should all the deemed universities be
many parts of the country. derecognised and attached to any of the central of state
(B) Yes, water tables have gone down to alarmingly low universities in India ?
levels in some parts of the country where irrigation is Arguments :
primarily dependent on groundwater, which may lead (A) Yes, many of these deemed universities do not conform
to serious environmental consequences. to the required standards of a full – fledged university
(C) Yes, India just cannot afford to draw groundwater any and hence the level of education is compromised.
further as the international agencies have cautioned (B) No, these deemed universities have been able to
India against it. introduce innovative courses suitable to the
(a) Only (A) and (B) are strong requirement of various industries as they are free from
(b) Only (B) and (C) are strong strict Government controls.
(c) Only (A) and (C) are strong (C) Yes, many such universities are basically money
(d) All (A) , (B) and (C) are strong spinning activities and education takes a backseat in
(e) None of these these institutions
27. Statement : Should there be complete ban on setting up of (a) Only (A) and (B) are strong
thermal power plants in India ? (b) Only (B) and (C) are strong
Arguments : (c) Only (A) and (C) are strong
(A) Yes, this is the only way to arrest further addition to (d) All (A), (B) and (C) are strong
environmental pollution. (e) None of these
y
o
u
rs
m
30 SBI & its Associates Bank PO Exam 2010

a
h
b
DIRECTIONS (Qs. 31–35) : In each questions bellow is given a 34. Statement : Government has decided to auction construction

o
of highways to private entities in several blocks across the

o
statement followed by three assumptions (A), (B) and (C). An

b
assumption is something supposed or taken for granted. You country on build–operate–transfer basis.

.w
have to consider the statement and the following assumptions Which of the following assumption(s) is /are implicit in the

o
above statement ?

rd
and decide which of the assumptions is implicit in the statement.
(A) An adequate number of private entities may not

p
re
31. Statement : Police authority cordoned of the entire locality respond to the Government's auctions notification.

s
for the entire day and stopped vehicular movement for the (B) Many private entities in the country are capable of

s
.c
visit of a top functionary of the government in view the constructing highways within reasonable time.

o
threat perception and advised all the residents in the area to (C) The Government's proposal of build–operate–transfer

m
limit their movement outside their dwellings. may financially benefit the private entities.
Which of the following assumption(s) is /are implicit in the (a) Only (A) and (B) are implicit
above statement ? (b) Only (B) and (C) are implicit
(A) Police personnel may not be able to control the (c) Only (B) is implicit
vehicular movement in the locality and may seek help (d) Only (A) and (C) are implicit
from the armed forces. (e) None of these
(B) People living in the locality may move out of their 35. Statement : The airlines have requested all their bona fide
houses for the day to avoid inconvenient. passengers to check the status of flight operations before
(C) The Government functionary may request the police leaving their homes as heavy fog is causing immense
authority to lift the ban on movement of residents of problems to normal flight operations.
the locality outside their dwellings. Which of the following assumption(s) is /are implicit in the
(a) None is implicit (b) Only (A) is implicit above statement ?
(c) Only (B) is implicit (d) Only (C) is implicit (A) The majority of the air passengers may check the flight
(e) Only (B) and (C) are implicit status before starting their journey to the airport.
32. Statement : The apex body controlling universities in the (B) The Government may take serious objection to the
country has decided to revise the syllabus of all the notice issued by the airline company.
technical courses to make them focused towards the present (C) Majority of the passengers may cancel their tickets
needs of the industry thereby making the technical and postpone their journey till the situation becomes
graduates more employable than they are at present. normal.
Which of the following assumption(s) is /are implicit in the (a) None is implicit (b) Only (A) is implicit
above statement ? (c) Only (B) is implicit (d) Only (C) is implicit
(A) Technical colleges affiliated to different universities (e) Only (A) and (C) are implicit
may not welcome the apex body's decision and may
continue with the same syllabus as at present. DIRECTIONS (Qs. 36–40) : Below is given a passage followed
(B) The industry may welcome the decision of the apex by several possible inferences which can be drawn from the
body and scale up their hiring from these colleges. facts stated in the passage. You have to examine each inference
(C) The Government may not allow the apex body to separately in the context of the passage and decide upon its
implement its decision in all the colleges as it may lead degree of truth or falsity.
to chaos. Mark answer (1) if the inference is 'definitely true,' i.e. it properly
(a) None is implicit (b) Only (A) is implicit follows from the statement of facts given.
(c) Only (B) is implicit (d) Only (C) is implicit Mark answer (2) if the inference is ' probably true' though not
(e) Only (A) and (B) are implicit 'definitely true' in the light of the facts given.
33. Statement : Government has urged all the citizens to use Mark answer (3) if the 'data are inadequate, i.e. from the facts
electronic media for carrying out their daily activities, given you cannot say whether the inference is likely to be true or
whenever possible instead of using paper as the false.
manufacture of paper requires the cutting down of a large Mark answer (4) if the inference is ' probably false' though not
number of trees causing severe damage to the ecosystem. 'definitely false' in the light of the facts given.
Which of the following assumption(s) is /are implicit in the Mark answer (5) if the inference is 'definitely false, i.e. it cannot
above statement ? possibly be drawn from the facts given or it contradicts the given
(A) Most people may be capable of using electronic media facts.
to carry out various routines. The deterioration in the overall asset quality of banks– gross
(B) Most people may have access to electronic media for Non–per–forming Assets (NPAs) are reportedly 27% higher at
carrying out their daily routine activities. the end of December 2009 than at the end of December 2008– is
(C) People at large may reject the Governments appeal and not surprising. Any slowdown in growth is bound to trigger a rise
continue using paper as before. in NPAs as more and more companies default on loan repayments.
(a) Only (A) is implicit The effect would be pronounced when the slowdown coincides
(b) Only (B) is implicit with a severe global recession. But for the restructuring of loans
(c) Only (A) and (B) are implicit permitted by the Central Bank on fairly generous terms, NPAs
(d) Only (C) is implicit would have been still higher. Prudent banks that took care while
(e) None of these sanctioning loans and then monitored the post– sanction
y
o
u
rs
m
SBI & its Associates Bank PO Exam 2010 31

a
h
b
disbursement diligently should be able to weather the crisis. But 41. Kesave Vora was born on 8th November 1978. He has

o
it is one thing to have NPAs rise because of a cyclical downturn, secured 65 percent marks in Std. XII and 60 percent marks in

o
b
it is quite another to have NPAs rise because of policy errors that Graduation. He has secured 58 percent marks in M.A.

.w
are entirely within the realm of policy makers. And this is what we Economics and 60 percent marks in ICWA. He has been

o
need to guard against. Excessiyely low interest rates skew the working in a bank as generalist officer for the past two years

rd
risk–reward equation by making projects that are actually not after completing his education. He has also secured 50

p
re
viable, appear viable–till interest rates reverse and the same percent marks in the written examination and 45 percent

s
projects cease to be viable! it is now well established that tong marks in personal interview.

s
.c
periods of unduly low interest rates encourage banks to take 42. Arindam Ghosh has been working in a bank as generalist

o
more risks. A low interest rate regime driven by an easy money

m
officer for the past four years after completing his
policy rather than macroeconomic fundamentals leads to excessive postgraduate diploma in management with 60 percent marks.
expansion of credit. It incentivizes banks to take on more risk in He has secured 50 percent marks in written examination
search of higher returns and to misprice risk. and 40 percent marks in the personal interview. He has also
36. Higher NPAs indicate shortcomings in disbursement and secured 70 percent marks in Std. XII. He was born on 25th
follow-up of credit given by banks. February 1977.
37. The Central Bank always allows banks to restructure their 43. Sohan Majhi has secured 65 percent marks in B.Sc. and 70
loans in the event of rise in NPAs. percent marks in M.Sc. Statistics. He has been working in a
38. Lower interest rate cycle projects commercially unviable bank as generalist officer for the past three years after
projects as viable. completing his postgraduation. He has secured 55 percent
39. Low interest rate on credit reduces the capacity to absorb marks in the written examination and 50 percent marks in
various unaccounted risk factors. the personal interview. He was born on 8th July1982.
40. Banks' NPAs occur only due to economic factors. 44. Neha Salve has been working in a bank as generalist officer
DIRECTIONS (Qs. 41–45) : Study the following information for the past four years after completing her post–graduate
carefully and answer the questions given below : degree in Economics with 60 percent marks. She has secured
60 percent marks in both graduation and Std. XII. She was
Following are the conditions for selecting Senior Manager– born on 24th August 1979. She has secured 70 percent marks
General Banking in a bank : in the written examination and 38 percent marks in the
(i) have secured at least 60 percent marks in Std. XII. personal interview.
(ii) have secured at least 55 percent marks in Graduation in any 45. Neeta Jaiswal was born on 2nd June 1980. She has been
discipline. working in a bank as generalist officer for the past three
(iii) have secured at least 60 percent marks in Post–graduate
years after completing her post–graduate degree in
degree/diploma in Management/Economics/Statistitcs.
Economics with 60 percent marks. She has secured 68
(iv) be at least 25 years and not be more than 35 years as on
percent marks in HSC and 58 percent marks in B.Com. She
01.03.2010.
has also secured 50 percent marks in both the written
(v) have post qualification work experience of at least 2 years
examination and personal interview.
as General Banking Officer in a bank.
(vi) have secured at least 50 percent marks in written examination. DIRECTIONS (Qs. 46–50) : In each of these questions there are
(vii) have secured at least 40 percent marks in Personal Interview. two sets of figures. The figures on the left are problem figures
In the case of a candidate who satisfies all the above conditions (four figures and one question marked space) and those on the
except– right are Answer Figures indicated by numbers 1, 2, 3, 4 and 5.
(a) at (iii) above, but has secured at least 60 percent marks in A series is established if one of the five Answer Figures is placed
CA or ICWA, the case is to be referred to VP– Recruitment. at the "question–marked space". Question Figures from a series
(b) at (vii) above, but have secured at least 65 percent marks in if they change from left to right according to some rule. The
the written examination and at least 35 percent marks in the number of the Answer Figure which should be placed in the
personal interview, the case is to be referred to President– question–marked space is the answer. All the five figures i.e.
Recruitment. four problem figures and one Answer Figure placed in the
In each question below are given details of one candidate . You question–marked space should be considered as forming the
have to take one of the following courses of actions based on the series.
information provided and the conditions and sub–conditions
given above and mark the number of that course of action as your 46. Problem Figures :
answer. You are not to assume anything other than the information
X T X T
provided in each question. All these cases are given to you as on
01.03.2010. S X S T ? S
Mark answer (1) if the data provided are inadequate to take a T S X
decision. Answer Figures:
Mark answer (2) if the case is to be referred to VP–Recruitment.
Mark answer (3) if the case is to be referred to President– T S S S S
Recruitment. X X T X T X X
Mark answer (4) if the candidate is to be selected. S T T
Mark answer (5) if the candidate is not to be selected. (a) (b) (c) (d) (e)
y
o
u
rs
m
32 SBI & its Associates Bank PO Exam 2010

a
h
b
47. Problem Figures : Number (N) of Six type of Electronic Products Sold by Six

o
different stores in a month and the price per product (P)

o
b
(price in ` ’000) chargd by each Store.

.w
= U
5 Z 3 = 6

o
? = U
Store A B C D E F

rd
3 Z U 6
Product N P N P N P N P N P N P

p
re
Answer Figures: L 54 135 48 112 60 104 61 124 40 136 48 126

s
s
.c
M 71 4.5 53 3.8 57 5.6 49 4.9 57 5.5 45 4.7

o
3 3 6 6 6 N 48 12 47 18 52 15 54 12 62 11 56 11

m
Z U = 6 Z U Z = = U
= U = U O 52 53 55 48 48 50 54 49 59 47 58 51
Z
P 60 75 61 68 56 92 44 84 46 76 59 78
(a) (b) (c) (d) (e)
Q 43 16 44 15 45 15 48 16 55 18 55 15
48. Problem Figures :
51. Number of L type products sold by Store F is what percent of
the number of the same type of products sold by Store E?
(a) 76.33 (b) 124
?
(c) 83.33 (d) 115
(e) None of these
52. What is the respective ratio of total number of N and L type
Answer Figures:
products together sold by Store D and the same products
sold by store A ?
(a) 119 : 104 (b) 102 : 115
(c) 104 : 115 (d) 117 : 103
(e) None of these
(a) (b) (c) (d) (e) 53. What is the average price per product charged by all the
stores together for product Q ?
49. Problem Figures : (a) ` 14,700 (b) ` 15,700
(c) ` 15,200 (d) ` 14,800
T T T S (e) None of these
? S 54. What is the difference in the amount earned by store A
T through the sale of P type products and that earned by
store B through the sale of Q type products ?
Answer Figures: (a) ` 38.4 lakhs (b) ` 0.384 lakhs
(c) ` 3.84 lakhs (d) ` 384 lakhs
T (e) None of these
S 55. What is the total amount earned by store C through the sale
T T T T of M and O type products together ?
(a) ` 2719.2 lakhs (b) ` 271.92 lakhs
(a) (b) (c) (d) (e)
(c) ` 2.7192 lakhs (d) ` 27.192 lakhs
(e) None of these
50. Problem Figures :
DIRECTIONS (Qs. 56–60): Study the following information
carefully and answer the questions that follow :
B

B
B T T ?
T
T

An Organisation consists of 2400 employees working in different


departments, viz; HR, Marketing, IT, production and Accounts.
The ratio of male to female employees in the Organisation is 5 : 3
Answer Figures: respectively. Twelve per cent of the males work in the HR
department. Twenty four per cent of the females work in the
Accounts department. The ratio of males to females working in
the HR department is 6 : 11 respectively. One-ninth of the females
work in the IT department. Forty two percent of the males work in
the production department. Number of females working in the
(a) (c) production department is ten percent of the males working in the
(b) (d) (e)
same. The remaining females work in the Marketing department.
The total number of employees working in the IT department is
DIRECTIONS (Qs. 51–55) : Study the following table carefully 285. Twenty two percent of the males work in the Marketing
to answer the questions that follow : department and the remaining work in the Accounts department.
y
o
u
rs
m
SBI & its Associates Bank PO Exam 2010 33

a
h
b
56. The number of males working in the IT department forms 61. What is the total percentage of boys in schools R and U

o
approximately What percent of the total : number of males together (rounded off to two digits after decimal)

o
b
in the Organisation ? (a) 78.55 (b) 72.45

.w
(a) 5 (b) 12 (c) 76.28 (d) 75.83

o
(c) 21 (d) 8 (e) None of these

rd
(e) 18 62. What is the total number of boys in School T ?

p
re
57. How many males work in the Accounts department ? (a) 500 (b) 600

s
(a) 170 (b) 165 (c) 750 (d) 850

s
.c
(c) 185 (d) 160 (e) None of these

o
(e) None of these 63. The total number of students in school R, is approximately

m
58. The total number of employees working in the Accounts what percent of the total number of students in school S ?
department forms what percent of the total number of (a) 89 (b) 75
employees in the organisation ? (rounded off to two digits (c) 78 (d) 82
after decimal) (e) 94
(a) 19.34 (b) 16.29 64. What is the average number of boys in schools P and Q
(c) 11.47 (d) 23.15 together ?
(e) None of these (a) 1425 (b) 1575
59. The number of females working in the production (c) 1450 (d) 1625
department forms what percent of the total number of (e) None of these
females in the Organisation ? 65. What is the respective ratio of the number of girls in school
(a) 7 (b) 12 P to the number of girls in school Q ?
(c) 4 (d) 15 (a) 27 : 20 (b) 17 : 21
(e) None of these (c) 20 : 27 (d) 21 : 17
60. What is the total number of females working in the HR and (e) None of these
Marketing department together ?
(a) 363 (b) 433 DIRECTIONS (Qs. 66–68) : Study the given information carefully
(c) 545 (d) 521 and answer the questions that follow :
(e) None of these A basket contains 4 red, 5 blue and 3 green marbles.
DIRECTIONS (Qs. 61–65) : Study the graphs carefully to answer 66. If three marbles are picked at random, what is the probability
the questions that follow : that either all are green or all are red ?

Total number of children in 6 different schools and the percentage 7 7


(a) (b)
of girls in them 44 12
Number of Children 5 1
3500 (c) (d)
3000 12 44
2500 (e) None of these
2000 67. If two marbles are picked at random, What is the probability
1500 that both are red ?

1000 3 1
(a) (b)
7 2
500
0
2 1
P Q T U (c) (d)
R S 11 6
School (e) None of these
Percentage of Girls 68. If three marbles are picked at random, What is the probability
50
that at least one is blue ?
45
40 7 37
(a) (b)
35 12 44
30
25 5 7
(c) (d)
20 12 44
15 (e) None of these
10 DIRECTIONS (Qs. 69–70) : Study the given information carefully
5 and answer the questions that follow :

0 A committee of five members is to be formed out of 3 trainees,


P Q R S T U 4 professors and 6 research associates. In how many different
School ways can this be done if :
y
o
u
rs
m
34 SBI & its Associates Bank PO Exam 2010

a
h
b
69. The committee should have all 4 professors and 1 research 75. What is the number of candidates passing in the exam from

o
associate or all 3 trainees and 2 professors ? city E ?

o
b
(a) 12 (b) 13 (a) 13,000 (b) 11,10,000

.w
(c) 24 (d) 52 (c) 1,13,000 (d) 11,000

o
(e) None of these (e) None of these

rd
70. The committee should have 2 trainees and 3 research

p
DIRECTIONS (Qs. 76–80) : Study the following graph carefully

re
associates ?
to answer the questions that follow :

s
(a) 15 (b) 45

s
.c
(c) 60 (d) 9 Percent profit made by two companies over the years

o
m
(e) None of these
Income – Expenditure
Per cent Profit = ´ 100
DIRECTIONS (Qs. 71–75) : Study the tables carefully and Expenditure
answer the questions that follow :
Company A
Number of candidates (in lakhs) appearing in an entrance
Company B
examination from six different cities and the ratio of 60
candidates passing and failing in the same 50

Percent Profit
City Number of Candidates 40
A 1.25 30
B 3.14
20
C 1.08
D 2.27 10
E 1.85
F 2.73 0
2004 2005 2006 2007 2008 2009
Ratio of candidates passing and failing within the city Years
City Passing : Failing 76. If in the year 2004 the expenditure incurred by Company
A 7 : 3 A and B was the same, what was the respective ratio of the
B 5 : 3 income of Company A and B in that year ?
C 4 : 5 (a) 27 : 28 (b) 14 : 23
D 1 : 3 (c) 13 : 19 (d) Can not be determined
E 3 : 2 (e) None of these
F 7 : 5 77. If the amount of profit earned by Company A in the year
2007 was ` 1.5 lakhs, what was its expenditure in that year ?
71. What is the respective ratio of the number of candidates
(a) ` 1.96 lakhs (b) ` 2.64 lakhs
failing in the Exam from City D to those failing in the exam
(c) ` 1.27 lakhs (d) ` 3.75 lakhs
from City A ?
(e) None of these
(a) 289 : 42 (b) 42 : 289
78. What is the average percent profit earned by Company B
(c) 227 : 50 (d) 50 : 227
over all the years together ?
(e) None of these
72. The number of candidates appearing for the exam from City 1 1
(a) 19 (b) 24
C is what percent of the total number of candidates appearing 3 6
for the exam from City B ? (rounded off to the nearest
1 1
integer) (c) 12 (d) 37
(a) 27 (b) 34 3 6
(c) 42 (d) 21 (e) None of these
(e) 38 79. If in the year 2008, the income of both the companies A and
73. Number of candidates passing in the exam from City F is B was the same, what was the respective ratio of expenditure
what percent of the total number of candidates appearing of company A to the expenditure of company B in that year ?
form all the Cities together ? (rounded off to two digits after (a) 21 : 25 (b) 7 : 9
the decimal) (c) 13 : 15 (d) Cannot be determined
(e) None of these
(a) 12.93 (b) 14.46
80. What is the respective ratio of the amount of profit earned
(c) 10.84 (d) 11.37
by Company A and B in the year 2009 ?
(e) None of these
(a) 2 : 3 (b) 4 : 7
74. Which city has the highest number of students failing in (c) 11 : 15 (d) Cannot be determined
the entrance exam ? (e) None of these
(a) F (b) C
(c) B (d) D DIRECTIONS (Qs. 81–85) : Study the given table carefully to
(e) None of these answer the questions that follow :
y
o
u
rs
m
SBI & its Associates Bank PO Exam 2010 35

a
h
b
Percentage of marks obtained by five students in five 86. What is the average number of people using mobile service

o
different subjects in a school M for all the years together ?

o
b
.w
Subject Eng. Sci Math SST Hindi 2 1
(100) (125) (150) (75) (50) (a) 16 (b) 14444

o
Student 3 6

rd
Rahul 67 84 70 64 90

p
2 1

re
Veena 59 72 74 88 84 (c) 16666 (d) 14
3 6

s
s
Soham 66 90 84 80 76

.c
(e) None of these

o
Shreya 71 66 80 66 86 87. The total number of people using all the three mobiles

m
Varun 63 76 88 68 72 services in the year 2007 is what percent of the total number
Figured in brackets indicate maximum marked for a particular subject of people using all the three mobile services in the year
Eng. ® English 2008? (rounded off to two digits after decimal)
Sci. ® Science (a) 89.72 (b) 93.46
Math. ® Mathematics (c) 88.18 (d) 91.67
SST. ® Social Science (e) None of these
81. What are the average marks obtained by all the students
88. Number of people using mobile service N in the year 2006
together in English ?
forms approximately what percent of the total number of
(a) 64.6 (b) 60.4
(c) 66.7 (d) 68.4 people using all the three mobile services in that year ?
(e) None of these (a) 18 (b) 26
82. What is Varun's overall percentage in the examination ? (c) 11 (d) 23
(a) 81.5 (b) 78.2 (e) 29
(c) 75.4 (d) 69.8 89. What is the respective ratio of number of people using mobile
(e) None of these service L in the year 2005 to those using the same service in
83. What is the respective ratio of total marks obtained by Veena the year 2004 ?
and Sherya together in Mathematics to the marks obtained (a) 8 : 7 (b) 3 : 2
by Rahul in the same subject ? (c) 19 : 13 (d) 15 : 11
(a) 11 : 5 (b) 7 : 9
(e) None of these
(c) 5 : 11 (d) 9 : 7
(e) None of these 90. What is the total number of people using mobile service M
84. If in order to pass the exam a minimum of 95 marks are needed in the year 2008 and 2009 together ?
in Science; how many students pass in the exam ? (a) 35,000 (b) 30,000
(a) None (b) Three (c) 45,000 (d) 25,000
(c) One (d) Two (e) None of these
(e) Four
85. What are the total marks obtained by Soham all the subjects DIRECTIONS (Qs. 91–95) : Study the table carefully to answer
together ? the questions that follow :
(a) 396 (b) 408 Distance (in kms) travelled by six trucks on
(c) 402.5 (d) 398.5 six different days of the week
(e) None of these
DIRECTIONS (Qs. 86–90) : Study the given graph carefully to Truck Day P Q R S T U
answer the questions that follow :
Monday 240 250 320 325 330 300
Number of people (in thousands) using three different
types of mobile services over the years Tuesday 320 264 308 314 318 314
N M L Wednesday 324 294 330 312 310 325
70 Thursday 288 300 310 278 260 275
No. of People (in thousands)

Friday 366 302 288 292 270 268


60
Saturday 292 284 260 274 290 242
50
40 91. What is the average distance travelled by truck S in all the
30 days together ?

20 1 1
(a) 198 km (b) 296 km
3 6
10
1 1
0 (c) 199 km (d) 299 km
2004 2005 2006 2007 2008 2009 6 3
Years (e) None of these
y
o
u
rs
m
36 SBI & its Associates Bank PO Exam 2010

a
h
92. If the speed of truck P on Monday was 19.2 kmph, what was

b
97. The total number of candidates appeared in interview from

o
the time taken by it to cover the given distance ? Management and other disciplines was what percent of

o
b
(a) 10 hours (b) 11 hours number of candidates appeared from Engineering discipline?

.w
(c) 9 hours 30 minutes (d) 12 hours 30 minutes (a) 50 (b) 150

o
(e) None of these (c) 200 (d) Cannot be determined

rd
93. If to travel the given distance, the time taken by truck Q on (e) None of these

p
re
Friday was 8 hours, what was its speed on that day ? 98. Approximately what was the difference between the number

s
(a) 42.50 kmph (b) 28.25 kmph of candidates selected from Agriculture discipline and

s
.c
(c) 37.75 kmph (d) 32.25 kmph number of candidates selected from Engineering discipline?

o
(e) None of these

m
(a) 517 (b) 665
94. What is the total distance travelled by all the trucks together
(c) 346 (d) 813
on Saturday ?
(e) 296
(a) 1623 km (b) 1632 km
99. For which discipline was the difference in number of
(c) 1263 km (d) 1362 km
(e) None of these candidates selected to number of candidates appeared in
95. If on Tuesday truck R and truck T travelled at the same interview the maximum ?
speed, what was the respective ratio of time taken by truck (a) Management (b) Engineering
R and time taken by truck T to cover their respective (c) Science (d) Agriculture
distances ? (e) None of these
(a) 154 : 159 (b) 142 : 167 100. Approximately what was the total number of candidates
(c) 161 : 173 (d) Cannot be determined selected from Commerce and Agriculture discipline
(e) None of these together ?
(a) 1700 (b) 1800
DIRECTIONS (Qs. 96–100) : Study the following pie-charts (c) 2217 (d) 1996
carefully and answer the questions given below :
(e) 1550
Disciplinewise Break up of Number of candidates appeared in
Interview and Disciplinewise Break up of Number of Candidates
selected by an organisation
GENERAL AWARENESS,
Disciplinewise Break up of Number of candidates appeared in MARKETING & COMPUTER KNOWLEDGE
Interview by the organisation
Total Number of candidates Appeared In 101. What is the full form of the 'NPA' as used in banking
the Interview = 25780 percentage environment ?
(a) Not Profitable Assets (b) New Potential Accounts
1 8 ce

Others (c) Non Performing Assets(d) Net Performing Assets


m er

Ag (e) None of these


ri c 12
Com

u
1 4 l tu r 102. As per recent reports many countries are planning to
e
nt Science introduce "Tobin Tax", the idea of which was given by a
e
ag e m Nobel Prize winner economist James Tobin in 1978, Tobin
Man 1 2 28
1 6 r in g

Tax, if implemented will be levied on which of the following ?


ee
gin

(a) On the interest income of corporates earned through


En

major investments
Disciplinewise Break up of Number of candidates selected after (b) All cash transactions
Interview by the organisation (c) Services availed specifically by an elite group of people
Total Number of candidates selected After Interview (d) Foreign Exchange Transactions
= 7390 percentage (e) None of these
103. As per news reports the Indian Economy is poised to hit
1 6 ce

Others
m er

Ag high growth once again. It is estimated that the Gross


ri c 14
Com

u Domestic Product (GDP) may be somewhere in the range of


7 lture
6.25% – 7.75% in the current fiscal. But at the same time the
e nt Science economy has to face certain challenges also. What could
ag e m
Man 2 0 32
11 ring

these challenges be ?
ee

(A) Maintaining balance between price stability and


g in

exchange rate
En

96. What was the ratio between the number of candidates (B) Capital flow may not be adequate as many FIIs are still
appeared in interview from other disciplines and number of not comfortable with Indian markets.
candidates selected from Engineering discipline respectively (C) Industrial sector is still under pressure and not showing
(round off to the nearest integer) ? much improvement.
(a) 3609 : 813 (b) 3094 : 813 (a) Only (A) (b) Only (B)
(c) 3094 : 1035 (d) 4 125: 1035 (c) Only (C) (d) All (A), (B) and (C)
(e) 3981: 767 (e) None of these
y
o
u
rs
m
SBI & its Associates Bank PO Exam 2010 37

a
h
b
104. Which of the following countries has decided that from (a) Only (A) (b) Only (B)

o
October 2018 no cheques should be issued or honoured (c) Only (A) and (B) (d) Only (B) and (C)

o
b
there, and all the payments should be made and accepted in (e) All (A), (B) and (C)

.w
electronic form only. 112. The Chief Executive Officers (CEOs) of the APEC held an

o
(a) USA (b) Russia important summit in Singapore in November 2009. What is

rd
(c) United Kingdom (d) Australia the full form of APEC ?

p
re
(e) None of these (a) Asia Pacific Economic Co-operation

s
105. In commodity exchanges in India, Index Futures are not

s
(b) Afro-Pacific Economic Council

.c
permitted, as some of the provisions of the FCRA do not (c) Asia Pacific Economic Council

o
allow the same. What is the full form of FCRA ?

m
(d) Alliance for promotion of Economic Co-operation
(a) Foreign Commodities Regulation Act (e) None of these
(b) Forward Commodities Repurchasing Act 113. Which of the following Gulf countries has decided recently
(c) Forward Contracts Regulation Act
that despite the objections raised by certain countries, it
(d) Financial contracts Reformation Act
will construct 10 more Uranium enrichment plants to
(e) None of these
strengthen its position in the world ?
106. As per the findings of the expert group headed by Dr. Suresh
(a) Iraq (b) Iran
Tendulkar a substantial number of people in India still live
Below Proverty Line. What is that number in terms of (c) UAE (d) Kuwait
percentage ? (e) None of these
(a) 25 % (b) 37 % 114. Which of the following countries was the winner of Asia
(c) 41 % (d) 47 % Cup Hockey Tournament for women held in November 2009?
(e) None of these (a) India (b) Japan
107. As per the findings of the World Bank, which of the following (c) China (d) South Korea
recent development in rural areas, has doubled the income (e) None of these
of rural households, raised literacy rate by 10 % and 115. The leaders of the European Union (EU) named Mr. Herman
appreciated land prices by upto 80 % ? Van Rompuy as the first President of the E.U. Mr. Rompuy
(a) Relief packages to the farmers the present ______.
(b) Construction of all weather roads (a) Prime Minister of Britain
(c) Recharging of water bodies (b) President of Ukraine
(d) Rainwater harvesting (c) Prime Minister of Russia
(e) None of these (d) President of Italy
108. As decided by the Reserve Bank of India, all the villages (e) Prime Minister of Belgium
with a population of 2000 will have access to financial 116. Which of the following is not one of the Monitorable Targets
services by the end of _____ of the 11th Five Year Plan (2007–12) ?
(a) 2009–10 (b) 2010–11 (a) To increase the average GDP growth rate
(c) 2011–12 (d) 2012–13 (b) To increasing the literacy rate
(e) None of these (c) To increase energy efficiency
109. As reported in various newspapers the Reserve Bank of (d) To improve sex ratio
India is planning to allow more and more banks to function
(e) To maximize the effect of global warming
as "Local Area Banks". This will help RBI in the
117. Cryogenic Engines are used in which of the following areas
implementation of which of the following of its plans/drives?
of technology ?
(A) Financial Inclusion
(a) Atomic Energy (b) Food Technology
(B) Rural Banking
(c) Oceanography (d) Space Research
(C) Mobile and e-Banking
(a) Only (A) (b) Only (B) (e) None of these
(c) Only (C) (d) Only (A) and (B) 118. The National Rural Employment Guarantee Act is now
(e) None of these known by which of the following names ?
110. During his visit to Trinidad in November 2009, the Indian (a) Indira Gandhi Rural Employment Guarantee Act
Prime Minister signed a Nuclear deal between India and (b) Rajiv Gandhi Rural Employment Guarantee Act
_____. (c) Jawaharlal Nehru Rural Employment Guarantee Act
(a) Canada (b) Brazil (d) Mahatma Gandhi Rural Employment Guarantee Act
(c) Saudi Arabia (d) Tanzania (e) All (A), (B) and (C)
(e) None of these 119. Which of the following initiative is/are taken by the
111. Which of the following is/are the objective (S) of the new Government of India to boost the development of agriculture?
industrial policy of the Government of India ? (A) Accelerated Irrigation Benefit programme
(A) To transform India into a major partner and player in (B) Fertiliser Subsidy Scheme
the global trade (C) Public Distribution System (PDS)
(B) To maintain a sustained growth in productivity (a) Only (A) (b) Only (B)
(C) To become the biggest exporter of horticultural (c) Only (C) (d) Only (A) and (B)
products in South East Asia (e) All (A), (B) and (C)
y
o
u
rs
m
38 SBI & its Associates Bank PO Exam 2010

a
h
b
120. Which of the following is/are true about the Indo-US (a) compiler (b) interpreter

o
Economic and Financial Partnership deal made in November (c) converter (d) instruction

o
b
2009 ? (e) None of these

.w
(A) The objective of the deal is to strengthen bilateral 130. A program that generally has more user-friendly interface

o
engagements and understanding.

rd
than a DBMS is called a ________.
(B) Work jointly on some financial and infrastructure related

p
(a) front end (b) repository

re
projects. (c) back end (d) form

s
(C) The deal was signed during the visit of the Foreign

s
(e) None of these

.c
Minister of both the countries to South Africa for Global

o
131. When you install a new program on your computer, it is
Economic Summit 2009.

m
typically added to the ________ menu.
(a) Only (A) (b) Only (B)
(c) Only (A) and (B) (d) Only (A) and (C) (a) all programs (b) select programs
(e) None of these (c) start programs (d) desktop programs
121. Which of the following countries has signed a deal to supply (e) None of these
a fleet of 150 J-10 Fighter Jets to Pakistan ? 132. Which of the following contains information about single
(a) France (b) Canada "entity" in the database - like a person, place, event, or
(c) Russia (d) USA thing?
(e) China (a) query (b) form
122. Which of the following terms/names is not associated with (c) record (d) table
banking/financial trade in India ? (e) None of these
(a) Automated Clearing House 133. Which one of the following is a key function of a firewall ?
(b) Provision Coverage Ratio (a) Monitoring (b) Deleting
(c) Market Stabilisation Scheme (c) Copying (d) Moving
(d) Credit Default Swaps (e) None of these
(e) Double Fault
134. ________ are a type of inexpensive digital camera that
123. The standard protocol of the Internet is _______.
remains tethered to a computer and used for
(a) TCP/IP (b) Java
videoconferencing, video chatting and live Web boardcast.
(c) HTML (d) Flash
(e) None of these (a) Webcams (b) Webpics
124. Digital photos and scanned images are typically stored as (c) Browsercams (d) Browserpics
________ graphics with extension such as .bmp, .png, .jpg, (e) None of these
.tif or .gif. 135. Who amongst the following is the author of the book "The
(a) vector (b) bitmap Rediscovery of India" ?
(c) either vector or bitmap (d) neither vector nor bitmap (a) Meghnad Desai (b) Romila Thapar
(e) None of these (c) Mulkraj Anand (d) Amit Chaudhary
125. A ________ is a computer attached to the Internet that (e) None of these
runs a special Web server software and can send Web pages 136. Some banks which were not able to meet their priority sector
out to other Computers over the Internet lending targets are now allowed to do so by purchasing
(a) web client (b) web system Priority Sector Lending Certificates (PSLC). Which of the
(c) web page (d) web server following agencies/organisation is/are authorized to issue
(e) None of these this certificates ?
126. After a user has saved and deleted many files, many (A) Micro Finance Institutions
scattered areas of stored data remain that are too small to be (B) Non Banking Finance Companies
used efficiency, causing ________. (C) NABARD
(a) disorder (b) turmoil (a) Only (A) (b) Only (B)
(c) disarray (d) fragmentation (c) Only (A) and (B) (d) Only (A), (B) and (C)
(e) None of these
(e) None of these
127. Which of the following books is written by V.S. Naipaul ?
137. Market Research is useful for _______.
(a) The Last Hero (b) Bliss was in that Dawn
(a) deciding proper marketing strategies
(c) Train to Pakistan (d) A Tale of Two Cities
(e) A house for Mr. Biswas (b) deciding the selling price
128. Which of the following is the communication protocol that (c) choosing the right products
sets the standard used by every computer that accesses (d) choosing the sales persons
Web-based information ? (e) All of these
(a) XML (b) DML 138. A 'call' means ______.
(c) HTTP (d) HTML (a) shout out to somebody
(e) None of these (b) a profession or business
129. Which of the following converts all the statements in a (c) visit friends
program in a single batch and the resulting collection of (d) visiting prospective customers
instructions is placed in a new file ? (e) After-sales services
y
o
u
rs
m
SBI & its Associates Bank PO Exam 2010 39

a
h
b
139. Conversion means ______. 150. Lead generation means _____

o
(a) meeting a prospective client (a) Tips for seiling tactics

o
b
(b) interacting with a prospective client (b) Tips for better production

.w
(c) converting an employer into an employee (c) Generate leaders

o
(d) converting a seller to a buyer

rd
(d) Likely sources for prospective clilents
(e) converting a prospective client into a buyer

p
(e) All of these

re
140. Customisation means ________.

s
(a) acquiring more customers
ENGLISH LANGUAGE

s
.c
(b) regulating customers

o
(c) special products to suit each customer

m
(d) more products per customer DIRECTIONS (Qs. 151–165) : Read the following passages
(e) All of these carefully and answer the questions given below it. Certain words/
141. Modern styles to marketing include _______. phrases have been printed in bold to help you locate them while
(a) digital marketing (b) tele-marketing answering some of the questions.
(c) e-commerce (d) e-mails solicitation Despite the economic crunch worldwide that saw pulverization
(e) All of these of some of the largest banking and finance of giants, Indian
142. The acronym HTML stands for ________. banking houses have managed to show positive growth this
(a) High Transfer Machine Language quarter. Some of India's leading national banks have posted a net
(b) High Transmission Markup Language profit rise of more than 40% over the last quarter amid global
(c) Hypertext Markup Language turmoil. This would come as a big shot in the arm for the investors
(d) Hypermedia Markup Language and consumers of these banks even though apprehension is
(e) None of these
mounting on other banking and broking firms worldwide. One of
143. e-Marketing is same as _______.
the main reasons behind the success of these banks this quarter,
(a) virtual markting (b) digital marketing
would be their direct backing by the Government of India. People
(c) real time marketing (d) All of these
take solace in their in India. People take solace in their investments
(e) None of these
in public sector watching the bailout packages being cashed out
144. Value added services means _______.
(a) costlier products (b) large number of products by governments all over the world to save big business houses.
(c) additional services (d) at par services Other private banks in India have also reported a substantial
(e) None of these net profit over the last quarter given the international and domestic
145. Aggressive Marketing is necessitated due to ________. scenario one cannot put this down as a mundane achievement
(a) globalisation While others are on a cost cutting spree and firing employees,
(b) increased competition Indian Companies are actually working on boosting staffing in
(c) increased production banking and broking sectors. This can be seen as a big boon in
(d) increased job opportunities the days to come when the current recession eases and the
(e) All of these economy gradually comes back on to the fast track. The finance
146. Computers connected to a LAN can _____ minister has assured Indian public about the sound health of all
(a) run faster Indian Bank. This could also be evident from the fact that there
(b) share information and/or share peripheral equipment have been no mergers and take overs in Indian Banking sector in
(c) e-mail a contrast to world scenario where finance houses are looking for
(d) go online mergers to cut costs on operations. We definitely are not looking
(e) None of these to thrive; rather we are looking for growth. It is just that the pace
147. Efficient marketing style requires of growth is a little slow now as compared to a year or two before.
(a) proper planning These are hard times to test the hard. The weak in business and
(b) good communication skills career will be weeded out and it is sometimes very beneficial for
(c) team work business on the long run.
(d) knowledge of products
151. What, according to the author, is the reason for the success
(e) All of these
in Indian national banks in this quarter ?
148. The performance of a sales person depends on _____
(a) salary paid (a) Indian national banks do not have any commitments
(b) sales incentives paid in troubled foreign markets.
(c) size of the sales team (b) These banks can never face financial crisis because of
(d) ability and willingness of the sales person their sheer size.
(e) team leader's aggressiveness (c) These banks are ready to give loans at a very low rate
149. The sole aim of marketing is to _____ of interest.
(a) Increase sales (d) The public is ready to invest in these banks because
(b) Increase the number of employees
of the knowledge that these banks get strong support
(c) Increase profits
from the government.
(d) Increase production
(e) All of these (e) None of these
y
o
u
rs
m
40 SBI & its Associates Bank PO Exam 2010

a
h
b
152. What does the phrase 'shot in the arm' as used in the DIRECTIONS (Qs. 159-165) : Choose the word/group of words

o
passage mean ?

o
which is most opposite in meaning to the word/group of words

b
(a) Shock (b) Fear printed in bold as used in the passage.

.w
(c) Encouragement (d) Anxiety

o
159. THRIVE

rd
(e) None of these
(a) succeed (b) deteriorate

p
153. How, according to the author is the current recession

re
(c) worry (d) tremble
beneficial ?

s
(e) strive

s
(a) Worldwide, companies have realized that India is a

.c
160. MUNDANE
strong power to reckon with

o
(a) extraordinary (b) regular

m
(b) India is surging ahead of the other companies (c) severe (d) visionary
throughout the world. (e) routine
(c) After the recession is over international companies Modern bio-technology, especially the creation of genetically
will turn to India for investment. modified crops, is often presented as a magic solutions or universal
(d) Recession is bringing down the prices of essential panacea for the problems of poverty, inadequate nutrition and
commodities. even environmental degradation across the world. Coversely, there
(e) None of these are people who present the picture of tech generated monsters
154. What, according to the author, will be a big boon in the and major human health hazards being created by science. Many
days to come ? of the technological changes currently in the process of being
(a) The economy coming back on the first track utilised in agriculture can have unforeseen consequences, and
(b) The slowing down of the economy their sefety and future viability are far from secure.
The reality, as always, is far more complex than either of
(c) Increased hiring in Indian financial sector in times of
these two extremes. Even today the total food production in the
economic slowdown
world is adequate to feed the hungry of the world; the problem is
(d) The cost cutting carried out by all the companies
rather one of unequal distribution, which deprives a large part of
(e) None of these the population of even their minimal nutritional requirements.
155. Which of the following statements is definitely true in the Similarly farmers, especially in developing countries, face many
context of the passage ? problems such as lack of infrastructure, poor or unstable market
(A) India has not been affected by the economic slowdown access, volatile input and output prices etc., that biotechnology
(B) Indian Banks are showing growth in this quarter despite does not address, much less solve.
the recession It is true that transgenic plants can offer a range of benefits
(C) While banking industry in the West was severely which are above and beyond those which a emerged from more
affected by recession in the past, it is now gradually traditional innovations in cultivation. It is suggested that such
recovering and showing a positive growth new technology offers more effective pest resistance of seeds
(a) Only (A) (b) Only (B) and crops through genetic control mechanisms, which also
(c) Only (C) (d) Only (A) and (B) reduces the need for pesticide use and leads to improved yield. A
(e) Only (B) and (C) basic question, of course, is whether the new GM technology is
156. Which of the following strengthens the finance minister's safe, and whether this is absolutely crucial since the effects may
only be known much later. The jury is still very much out on this
statement about the sound health of Indian Banks with
matter, and the controversy does not appear to be resolved
respect to the passage ?
quickly.
(A) There have been no acquisitions and mergers of Indian The trouble is that most governments in developing
Banks. countries have relatively low food and beverage regulatory
(B) The Indian Banks are recording a positive growth. standards, and public systems for monitoring and surveillance of
(C) Layoffs have been observed worldwide. such items are poor or non-existent. This leaves them open for
(a) Only (A) and (B) (b) Only (A) and (C) entry and even dumping of a range of agricultural products of the
(c) Only (A) (d) Only (B) new technology, which may into pass regulatory standards in the
(e) All (A), (B) and (C) more developed countries.
161. Which of the following is true in context of the passage ?
DIRECTIONS (Qs. 157-158) : Choose the word/group of words
(a) Genetically modified crops have been universally
which is most similar in meaning to the word/group of words recognized as a solution to poverty and environmental
printed in bold as used in the passage. degradation
157. TURMOIL (b) The only way to improve the deficit in food requirement
(a) danger (b) shock and food production in the world is by adapting
(c) sadness (d) fear genetically modified crops
(e) chaos (c) Genetically modified crops produce more yield as
158. PULVERIZATION compared to yield from the traditional methods
(a) polarisation (b) mashing (d) Taking advantage of absence of regulatory standards,
(c) debasement (d) fall scientists have been dumping new products in the
(e) crushing markets without appropriate approval
(e) None is true
y
o
u
rs
m
SBI & its Associates Bank PO Exam 2010 41

a
h
b
162. Choose the word/group of words which is most similar in (c) unusual, worsened

o
meaning to open printed in bold as used in the passage. (d) insignificant, endured

o
b
(a) Vulnerable (b) Capable (e) adequate, proven

.w
(c) Threatened (d) Uncertain 168. The National Knowledge commission has said that India

o
rd
(e) Weak will have to bring _______ in education if it has to emerge

p
163. Choose the word/group of words which is most opposite in as the most _______ workforce of the world.

re
meaning to VOLATILE printed in bold as used in the (a) changes, biggest (b) reforms, talented

s
s
passage. (c) alleviation, skillful (d) quality, brighter

.c
o
(a) Never-ending (b) Meager (e) outcomes, demanded

m
(c) Valuable (d) Irreversible 169. Norway has stolen a march over other developed countries
(e) Table by _______ that it would reduce 40% of its greenhouse
164. The author of the given passage seems to be definitely gas emissions by 2020 and ______ carbon neutral by 2030.
______ (a) allowing, turn (b) posing, grew
(a) suggesting the use of traditional method of agriculture (c) estimating, exist (d) perceiving, arising
as against bio-technology by developing countries (e) declaring, become
owing to their poor regulatory standards 170. According to the language experts, children should begin
(b) in favour of utilizing bio-technology as a tool for talking in their mother tongue rather than foreign language
alleviation of poverty from the world which can _______ affect their comprehension abilities
(c) urging the policy makers to improve infrastructure leading to serious language based _______ later in their
facilities so that farmers can maximise the benefits lives.
genetically modified crops (a) significantly, abilities
(d) unconvienced of the long term effects and rationale (b) appropriately, achievements
for immediate requirement of genetically modified (c) severely, advantages
products (d) adversely, problems
(e) None of these (e) positively, issues
165. Why, according to author, is genetic modification of crops DIRECTIONS (Qs. 171-175) : In each of the following questions
not an answer to the problem of hunger in the world ? four words are given of which two words are given of which two
(A) People being highly doubtful of the long term effects words are most nearly the same or opposite in meaning. Find
of genetically modified crops, do not buy the products the two words which are most nearly the same or opposite in
grown by such methods. meaning and indicate the number of the correct letter
(B) The problem of hunger in the world is not due to combination.
inadequate production of food but due to unequal
distribution of it. 171. (A) instigate (B) enquire
(C) Many developing countries have banned genetically (C) construe (D) interpret
modified products as developed countries have been (a) A–C (b) A–B
using these countries as dumping grounds for new (c) C–D (d) B– D
genetically modified products. (e) A– D
(a) Only (A) (b) Only (B) 172. (A) superficial (B) superfluous
(c) Both (B) and (C) (d) Both (A) and (C) (C) enlightened (D) surplus
(e) None of these (a) A–C (b) A–B
(c) B–C (d) B– D
DIRECTIONS (Qs. 166-170) : Each question below has two (e) A– D
blanks, each blank indicating that something has been omitted. 173. (A) appalling (B) sinister
Choose the set of words for each blank that best fits the meaning (C) perturbed (D) threatening
of the sentence as a whole. (a) A– B (b) B– D
166. A controversial plan to build an immense dam in Brazil's (c) A–C (d) A–D
rainforest was stalled when it ______ a formidable bloc of (e) D–C
_______ in ecologists and indigenous tribes alike. 174. (A) imprison (B) torture
(a) resulted, hostilities (b) gained, supporters (C) excruciate (D) extract
(c) attracted, opponents (d) lead, protesters (a) B–D (b) B–C
(e) drew, proponents (c) A– B (d) C–D
167. According to a recent survey, sales figures of high end cars (e) A–C
have seen an ________ growth in the past year, which 175. (A) pertinent (B) impolite
shows that Indian consumers have________ the impact of (C) irrelevant (D) insecure
recession. (a) A–C (b) B– D
(a) unprecedented, negated (c) C–D (d) A–D
(b) unbelievable, suffered (e) B–C
y
o
u
rs
m
42 SBI & its Associates Bank PO Exam 2010

a
h
b
DIRECTIONS (Qs. 176-180) : Rearrange the following sentences grossly (184). There are clear (185) that farm output, particularly

o
cereals, will fall drastically. Insufficient rain is bound to shoot up

o
(A), (B), (C), (D), (E) and (F) to make a meaningful paragraph

b
and then answer the questions which follow : the (186) of agricultural commodities and that would impact the

.w
economy as a whole. The drought would also (187) a drastic

o
(A) According to it, organized retail stores are not wolves at the reduction in rural employment and consumption besides inflation

rd
doors of friendly neighbourhood grocery stores as there is in the prices of food articles.

p
re
room for expansion of both. Food prices have been (188) since the past few months,

s
(B) Many have been crying foul over the entry of organized and lower agricultural production, is likely to (189) the situation.

s
.c
retail stores expressing concern over their impact on small The government has said that food grain from the buffer stocks

o
store owners.

m
will be used to keep prices (190). Subsidised food grain is
(C) The final winner in the competition, however, is the common necessary in these times, but its effectiveness will depend a lot
man who gets to choose between the most suitable options on the distribution system.
and in turn fights with the runaway inflation in prices of 181. (a) impractical (b) ambiguous
essential commodities. (c) failing (d) working
(D) In spite of this potential for expansion, it is doubtless that
(e) weakening
the small store owners face a decline in profit in initial years
182. (a) strike (b) affected
if organized retailers set up stores in the vicinity.
(c) exposed (d) revered
(E) But a study conducted over a period of two years goes a
(e) altered
long way towards allaying these fears.
183. (a) unseasonably (b) unfavourably
(F) This impact, however, wears off once they learn to take on
(c) presently (d) meagerly
the competition which in turn enhances efficiency all
(e) later
around.
184. (a) inadequate (b) enough
176. Which of the following sentences should be the FIRST
(c) missing (d) ample
after rearrangement ?
(a) A (b) B (e) atrocious
(c) C (d) D 185. (a) contradictions (b) advices
(e) E (c) reasons (d) results
177. Which of the following sentences should be the THIRD (e) indications
after rearrangement ? 186. (a) production (b) requirement
(a) A (b) E (c) prices (d) yield
(c) D (d) F (e) labour
(e) C 187. (a) trigger (b) lead
178. Which of the following sentences should be the SIXTH (c) result (d) contribute
(LAST) after rearrangement ? (e) dampen
(a) A (b) B 188. (a) improving (b) balanced
(c) C (d) E (c) stable (d) increasing
(e) F (e) decreasing
179. Which of the following sentences should be the SECOND 189. (a) aggravate (b) amend
after rearrangement ? (c) smoothen (d) improve
(a) A (b) B (e) challenge
(c) C (d) E 190. (a) unprofitable (b) futile
(e) F (c) maximum (d) growing
180. Which of the following sentences should be the FIFTH (e) down
after rearrangement ?
DIRECTIONS (Qs. 191-195) : Which of the phrases (1), (2), (3)
(a) D (b) B
and (4) given below each statement should be placed in the
(c) C (d) E
blank space provided so as to make a meaningful and
(e) F
grammatically correct sentence ? If none of the sentences is
DIRECTIONS (Qs. 181-190) : In the following passage there are appropriate, mark (5) as the answer.
blanks, each of which has been numbered. These numbers are
191. Although information technology has entered the homes,
printed below the passage and against each, five words/phrases offices and hearts of many citizens of India, .....
are suggested, one of which best fits the blank appropriately. (a) India provides the highest number of IT experts to the
Find out the appropriate word/phrase in each case. world every year
Economic growth figures for the first quarter of this financial year (b) many people in rural areas still remain ignorant of its
seem to support the claim that the worst may be over for the immense benefits
Indian economy. The gradual revival is also an indication that the (c) government has done its best by funding research in
government's economic stimulus package is (181). What could, this field appropriately
however, upset the positive outlook is the drought which (182) (d) the face of communication in the years to come would
large parts of the country and its impact on overall growth. Even change completely from the by gone years
though the monsoon had picked up (183) the rains received were (e) None of these
y
o
u
rs
m
SBI & its Associates Bank PO Exam 2010 43

a
h
b
192. While the environment friendly – Nuclear energy could make (c) A recent report suggested that 34 percent of

o
a large addition in the energy resources, ..... government land is illegally occupied by the slum

o
b
(a) experts have a lot of expectations from this cleaner dwellers

.w
method of producing energy (d) The slum dwellers live in sub human, unhygienic

o
(b) the government is determined to extract maximum out conditions in their tiny shanties and are very often

rd
of this technology in the near future hubs for criminal activities

p
re
(c) international lobby has been pressurising the (e) None of these

s
developing nations to shift their energy production

s
197. However, since the beginning of the nineties, free market

.c
from coal to make our power forces have been allowed to play for ensuring all round

o
(d) the problem of locating adequate numbers of Uranium

m
development in Indian markets as well.
reserves to run the reactors is yet to be sorted out Which of the following sentences would immediately
(e) None of these precede the above sentence ?
193. ..........., experts proposed the idea of a common school system (a) Entry of various market forces increased the
(a) Overlooking the fundamental right of quality education competition faced by the indigenous organizations
of every child in India
manifold
(b) Since the curricular requirements of a rural child is
(b) The severe shortage of newest technology and
different from an urban child
products in the late 1980s ended with the entry of
(c) Based on the fact that difference in the quality of school
foreign players in the Indian market.
acts as a ground for discrimination
(d) Since a large percentage of Indian children are getting (c) A major roadblock faced by the foreign investors was
free education the poor infrastructure facilities which was however
(e) None of these overtaken by the prospects seen in the huge market
194. .........., the soil today is now nowhere as a rich in native (d) India had been following a very selective and
minerals as it used to be some centuries ago conservative economic policy during the late 1980s
(a) As there is a growing consent among farmers regarding (e) None of these
limiting the use of chemical fertilizers. 198. That was mainly because only two percent of the total
(b) As the chemical inputs in agriculture improved the yield farmers could actually comprehend the lengthy procedures
many folds to obtain the loan and be benefited from it.
(c) Owing to the uninhibited use of chemical inputs in Which of the following sentences would immediately
agriculture precede the above sentence ?
(d) Awareness among farmers regarding the side effects (a) Many moneylenders have been making the most out
of chemical farming grew of this situation and providing loans to the rest of the
(e) None of these farmers at inflated prices
195. As allegation of crores of rupees changing hands to permit (b) There is thus an urgent need to make the schemes and
illegal meaning began to fly thick arid fast, ...... policy as simple and farmer friendly as possible.
(a) government ordered an enquiry which exposed a nexus (c) A new loan scheme started by the government a couple
between mine operators and bureaucrats of months ago proved to be a huge failure and utterly
(b) it caused great damage to the surrounding ecosystem unpopular among the farmers
and the environment in general (d) This situation can be improved by providing mediators
(c) the officials have been irresponsible in failing to bring to carry out and follow-up of the lengthy official
it to the notice of the court in time. procedure for the farmers
(d) the powerful mining lobby had bribed the officials to (e) None of these
obtain permit for mining on ecologically sensitive land
199. The foreign embassy has put forward a demand to the
(e) None of these
government that prosecution may be carried out on a foreign
DIRECTIONS (Qs. 196-200) : In each of the given sentences land and not in the country to which the terrorists belong.
select the sentence which would either follow or precede the Which of the following sentences would immediately follow
given sentence in grammatically and conceptually appropriate the above sentence ?
manner. The instruction is given at the end of every statement. (a) Army arrested the terrorists responsible for kidnapping
four foreign tourists after a daylong operation
196. Unfortunately, however these slum dwellers are looked upon
by the society as an appendix causing ills in the urban (b) The foreign ministry has offered all possible help to
society. the government for hunting down the terrorists
Which of the following sentences would immediately involved in it
precede the above sentence. (c) Government has politely declined the request to
(a) Health officials have been warning the government handover the arrested terrorists for prosecution
against the transmission of contagious diseases from outside the country
the slum areas to other parts of the city (d) The arrested terrorists were sentenced to life
(b) Slim dwellers not only play a significant role in urban imprisonment after being prosecuted by a panel of
economy but also provide cheap labour for everyday international judges
work of the cities (e) None of these
y
o
u
rs
m
44 SBI & its Associates Bank PO Exam 2010

a
h
b
200. This is because most of the institutes for higher learning urban areas

o
lack the basic infrastructure, trained staff and equipment (c) According to a recent survey, only two out of nearly

o
b
necessary for the physically challenged. 200 universities in India have adequate number of

.w
Which of the following sentences would immediately books in Braille

o
rd
precede the above sentence ? (d) Although government has been making many

p
(a) Many physically challenged people have been provisions to improve higher education in India, no

re
performing as well as other students perform in their attention has been paid to the education of the

s
s
studies physically challenged students

.c
o
(b) The percent of illiterate, physically challenged people (e) None of these

m
in rural areas is considerably more than that in the
y
o
u
rs
m
SBI & its Associates Bank PO Exam 2010 45

a
h
b
o
o
b
.w
o
rd
1. (e) 42 W 7 R 8 A 6 Q 4 = ?

p
Þ ? = 42 ¸ 7 – 8 + 6 × 4 7. (e)

re
20 18 9 2 21 14 1 12
Þ ?= 6 - 8 + 24

s
s
Þ ? = 30 – 8 = 22

.c
T R 1 B U N A L

o
2. (b) As per conditions given

m
N D O W N N A M E
A ¯ ¯ ¯ ¯ ¯ ¯ ¯ ¯
W E 8. (d)
5 @ 9 # ¹ 6 ¸ 3
30 m So code for Mode is % @ 53
S 9. (c) Meaningful Words from LGEU are
15 m
B C Þ GLUE, LUGE
T H R I V E S
20 m
+1 –1 –1
10. (a)
E D
15 m S I U H R D U
Similarly,
Required distance = AE = AB + BE = (30 + 20) m = 50 m L
S O U F U L
+6 +6 +6
3. (d) C ¾¾® I ¾¾® O ¾¾® U +1 –1 –1
+6 +6 +6
E ¾¾® K ¾¾® Q ¾¾® W V P T K K T E
+6 +6 +6
A ¾¾® G ¾¾® M ¾¾® S
Nets
4. (c) Dresses
Branches
5 9 1 6 4 8 2 3 Cars
11. (b) Trains
9 5 3 8 4 1 2 6
5. Here 2 different series of alphabets are there.
(a) P Q R S T ABC DE
PQ RS ABC DE Conclusions:
I. False II. True
PQ RS ABCD
III. True IV. False
PQ R ABCD
Mountains
6. (b)
Jungles

how can you go ja da ka pa Desks Kites Pencils


12. (c)
can you come here na ka sa ja

Conclusions:
ra pa sa I. False II. False
come and go III. True IV. True

Codes are: Clips Roads


how Þ da come Þ sa
can Þ ja or ka here Þ na 13. (e) Papers Lanes
you Þ ja or ka and Þ ra Boards
go Þ pa
The code for here is ‘na’.
y
o
u
rs
m
46 SBI & its Associates Bank PO Exam 2010

a
h
I. R« D Þ R < D : Not True

b
Conclusions:

o
I. True II. False II. V « R Þ V < R : Not True

o
b
III. False IV. False III. D @ M Þ D = M : Not True

.w
IV. M % D Þ M > D : Not True

o
D is either smaller than or equal to M.

rd
Clocks
Therefore, either III or IV follows.

p
re
Pens 22. (b) F%NÞF>N

s
Tyres Wheels Buses NÓWÞN³W

s
14. (a)

.c
WdYÞW£Y

o
m
Y «T ÞY < T
Therefore, F > N ³ W £ Y < T
Conclusions:
Conclusions
I. False II. False
I. F % W Þ F > W : True
III. False IV. False
II. T % N Þ T > N : Not True
III. N % Y Þ N > Y : Not True
Hammers Windows IV. T % W Þ T > W : True
So only conclusion I and II follow
Stones Rings Doors 23. (d) BÓTÞB³T
15. (e) T «RÞT < R
R%FÞR>F
F@ KÞF=K
Or Therefore, B ³ T < R > F = K
Conclusions
I. B % R Þ B > R : Not True
Ring II. F « T Þ F < T : Not True
III. R % K Þ R > K : True
Door Hammer
I. False IV. K « T Þ K < T : Not True
II. True Either So only conclusion III is true.
III. False Stones
Window 24. (a) J@FÞJ=F
IV. False FdNÞF£N
N%HÞN>H
HÓGÞH³G
(16-20): Sitting arrangement Therefore, J = F £ N > H ³ G
Conclusions
F
I. G « N Þ G < N : True
G E II. N Ó J Þ N ³ J True
B III. F « J Þ F < J : Not True
IV. J d G Þ J £ G : Not True
K
So only conclusion I and II follow.
H 25. (e) D dTÞD£T
C T@ RÞ T =R
D A Rã M Þ R ³ M
M%KÞM>K
16. (a) C is fourth to the left of G. Therefore, D £ T = R ³ M > K
17. (d) E is third to the left of B. Conclusions
18. (c) G is third to the right of K. I. R @ D Þ R = D : Not True
19. (b) G is to immediate right of F. II. R % D Þ R > D : Not True
20. (e) F is sitting between G and E. R is either greater than or equal to D.
(21 - 25) : Symbols in terms of correct meaning can be written as Therefore, either I or II is true.
d Þ£ @Þ= ÓÞ³ III. K « T Þ K < T : True
IV. M d T Þ M £ T : True
% Þ > «Þ < 26. (a) Both argument (A) and (B) are strong. Which clearly
21. (d) R « K Þ R < K show the importance of irrigation nad environmental
K% DÞK>D consequences of reducing groundwater level.
D @ MÞ D = V Argument (c) is not strong.
V dMÞV £M 27. (c) Only Argument (B) is strong because thermal power
Therefore, R < K > D = V £ M plants in India are one way to increase environmental
Conclusions pollution so cannot be completly banned. Argument
(C) is based on example which is a bad argument.
y
o
u
rs
m
SBI & its Associates Bank PO Exam 2010 47

a
h
b
28. (d) Argument (A) is strong because due to shortage of 41. (b) Kesava Vora satisfies conditions (i), (ii), (a), (iv), (v),

o
space in big cities in India high rise building should be (vi) and (vii).

o
b
encouraged. Therefore, his case should be referred to Vice President

.w
29. (c) Only Argument (C) is strong because to avoid the – Recruitment.

o
42. (a) There is no information about condition (ii).

rd
inconvenience of commuters, repair work is advisable
43. (a) There is no information about condition (i).

p
in night only.

re
30. (c) Only Argument (A) and (C) are strong because 44. (c) Neha Salve satisfies conditions (i), (ii), (iii), (iv), (v), (vi)

s
and (b). Therefore, her case should be referred to

s
compromise with level of education cannot be done.

.c
President - Recruitment

o
31. (a) None of the Assumptions (A), (B) and (C) is implicit in

m
the statement. If police authority has cordoned off the 45. (d) Neeta Jaiswal satisfies all the conditions. Therefore,
entire locality, it implies that police will ably control the she can be selected.
vechicular movement in the locality. 46. (d) The following changes occur from Problem figure (a) to
(b) :
It is assumed that any advice given to the people will
be followed.
32. (c) Only Assumption (B) is implicit in the statement.
The apex body controlling universities has taken the
decision assuming that technical colleges will honour
it.
33. (c) Only Assumption (A) and (B) are implicit in the Similar changes would occur from Problem Figure (c) to (d).
statement.. Government urged all the citizens to use 47. (c) In each subsequent figure all the designs move in
electronic media assuming that most people are capable clockwise direction and the design at top position is
of using electronic media. replaced with a new design.
34. (c) Only Assumption (B) is implicit in the statement. If the 48. (e) From Problem Figure (a) to (b) the top and the bottom
Government has decided to auction construction of designs interchange positions. From Problem Figure
highways to private entities, it may be assumed that (b) to (c) the top design moves to the bottom position.
many private entities in the country are capable of These two steps are repeated alternately in the
constructing highways within reasonable time. subsequent figures.
35. (b) Only Assumption (A) is implicit in the statement. Any 49. (a) From Problem Figure (a) to (b) three designs move one
appeal/request is issued assuming that some people step in design appears at the central position. From
will pay heed to it. Problem Figure (d) to (e) the upper left and the central
36. (a) From the given data in the passage, it is clear that the designs interchange positions. Similarly, the upper right
Inference is definitely true. and lower left designs interchange positions.
37. (b) The use of term’ always’ in the inference shows that Therefore, from Problem Figure (b) to (c) similar changes
the Inference is probably true. would occur as that have been occurred from Problem
Figure (d) to (e).
38. (a) The inference is definitely true. Consider the following
50. (b) From Problem Figure (b) to (c) all the designs move in
line of the passage:
anticlockwise direction. The arrow and the design (B)
“Excessively low interest rates skew the risk reward rotate through 90° clock-wise while the T-shaped design
equation by making projects that are actually not viable, rotates through 90° anticlockwise. Similar changes
appear viable.” would occur from Problem Figure (d) to (e)
39. (e) The Inference is definitely false. Consider the following
51. (e) Number of L type products sold by : Store F = 48 Store
line of the passage:
E = 40.
“It is now well established that long periods of unduly
low interest rates encourage banks to take more risks.” 48
\ Required percentage = ´ 100 = 120
40. (a) The Inference is definitely true because rise in NPAs 40
depend upon cyclic factors. 52. (e) Required ratio = (61 + 54) : (54 + 48) = 115 : 102
(41 -45) : 53. (b) Required average = `
Con ditio ns æ 16 + 15 + 14.5 + 15.6 + 18.2 + 14.9 ö
Cand idate ç ÷ thousand
(i) (ii) (iii) or (a) (iv ) (V) (vi) (v ii) or (b ) è 6 ø
Kes ava ü ü – ü ü ü ü ü –
æ 94.2 ö
A rin dam ü NG ü – ü ü ü ü – = `ç ÷ thousands = ` 15,700
è 6 ø
Soh an NG ü ü – ü ü ü ü –
54. (a) Required difference
Neha ü ü ü – ü ü ü – ü = ` (60 × 75 – 44 × 15) thousands
Neeta ü ü ü – ü ü ü ü – = ` (4500 – 660) thousands
Note: NG ® Not given = ` 38.4 lakhs
y
o
u
rs
m
48 SBI & its Associates Bank PO Exam 2010

a
h
55. (d) Required amount earned by store C through the sale of = 3C3 + 4C3

b
o
M and O type products together =1+ 4=5

o
b
= ` (57 × 5.6 + 48 × 50) thousands \ Required probability

.w
= ` (319.2 + 2400) thousands
n ( E) 5 1

o
= ` 27.192 lakhs = =

rd
=
Calculations (56 - 60): n ( S) 220 44

p
re
Total number of employees = 2400 67. (e) Total possible outcomes

s
s
5 12 ´ 11
= n ( S) = C 2 =

.c
12
Number of male employees = ´ 2400 = 1500 = 66

o
8 1´ 2

m
3 Favourable number of cases = n(E)
Number of female employees = ´ 2400 = 900
8 4 4´3
= C2 = =6
(56 – 60) : 1´ 2
\ Required probability
Department Males Females
HR 180 330 n (E) 6 1
= n S = 66 = 11
Marketing 330 191 ( )
IT 185 100 68. (b) Total possible outcomes = n(S) = 12C3 = 220
Production 630 63 Favourable number of ways of picking 3 marbles (none
Accounts 175 216 is blue) out of 7 marbles
185 7 7 ´6´5
56. (b) Required percentage = ´ 100 = 12 = C3 = = 35
1500 1´ 2 ´ 3
57. (e) Number of males in Accounts \ Required probability
department = 175 æ 35 ö 7 37
58. (b) Required percentage = çè1 - ÷ = 1- =
220 ø 44 44
(175 + 216) 69. (a) Number of combinations
= ´ 100 = 16.29
2400
63
= ( 4 C4 ´ 6C1 + 3C3 ´ 4C2 ) = 1 × 6 + 1 × 6 = 12
59. (a) Required percentage = ´ 100 = 7 70. (c) Number of combinations
900
= Selecting 2 trainees out of 3 and selecting 3 research
60. (d) Number of females is HR and Marketing department
associates out of 6 = 3C2 × 6C3
= 330 + 191 = 521
6 ´ 5´ 4
61. (d) School Number of Girls Number of Boys = 3´ = 60
1´ 2 ´ 3
P 1000 1500 (71 - 75) :
Q 1350 1650 71. (c) Required ratio
R 550 1450
S 675 1575 City Numb er of Nu mber o f
T 500 750 Cand idates can did ates failed
U 175 825 pas s ed
Number of boys in schools R and U together A 87500 37500
= (1450 + 825) = 2275 B 196250 117750
C 48000 60000
2275
\ Required percentage = ´ 100 = 75.83 D 56750 170250
3000 E 111000 74000
62. (c) Number of boys in school T = 750 F 159250 113750
2000
63. (a) Required percentage = ´ 100 = 89 170250 : 37500 = 227 : 50
2250
1.08
1 72. (b) Required percentage = ´ 100 = 34
64. (b) Required average = (1500 + 1650 ) = 1575 3.14
2 73. (a) Total number of candidates appearing from all the cities
65. (c) Required ratio = 20 : 27 together = (1.25 + 3.14 + 1.08 + 2.27 + 1.85 + 2.73) lakh
66. (d) Total possible outcomes = Number of ways of picking = 12.32 lakh
3 marbles out of 12 marbles = n(S) \ Required percentage
12 ´ 11 ´ 10 159250
= 12c3 = 220 ´ 100 = 12.93
1´ 2 ´ 3 =
1232000
Favourable number of cases = n(E)
y
o
u
rs
m
SBI & its Associates Bank PO Exam 2010 49

a
h
74. (d) Marks obtained by Rahul in Maths

b
o
75. (e) Number of passed students from city E = 111000

o
70 + 150

b
76. (d) Let the expenditure of companies A and B in 2004 be = = 105

.w
` I lakh each. 100

o
Required tatio = 231:105 = 11:5

rd
I1 - I 84. (b) If pass percentage in Science be x, then
For company A, 35 = ´ 100

p
I

re
1255 x 95 ´ 100
Þ 135I = 10011 ....(i) = 95 Þ x = = 76

s
s
100 125

.c
I2 - I Clearly, three students will pass in science and hence
For company B, 40 = ´ 100

o
m
I in the exam.
Þ 140I = 100I2 ....(ii) 85. (c) Marks obtained by Sohan in all subjects
Dividing equation (i) by (ii), 90 ´ 125 84 ´ 150 80 ´ 75 76 ´ 50
I 1 135 27 = 66 + + + +
100 100 100 100
= =
I2 140 28 = 66 + 112.5 + 126 + 60 + 38 = 402.5
77. (d) Let the expenditure of company A in 2007 be ` x lakh. 86. (a) Average number of people using mobile service M
1.5 æ 5 + 10 + 25 + 20 + 25 + 15 ö
\ 40 = ´ 100 Þ 40 x = 150 =ç
è ÷ø thousands
40 6
150 100 2
Þ x= =`3.75 lakh = thousand = 16
40 6 3
78. (e) Required average per cent profit earned by company B 55
87. (d) Required per cent = ´ 100 = 91.67
40 + 45 + 40 + 35 + 30 + 45 60
=
6 88. (a) Required per cent
235 1 10
= = 39 % = ´ 100 » 18
6 6 55
79. (c) If the income of each company be `x lakh in 2008, then 89. (b) Required ratio = 15 : 10
for company A, =3:2
90. (e) Required number of people
x - E1 = (25 + 15) thousand = 40000
50 = ´ 100
E1 91. (d) Required average distance covered by truck S
150E1 = 100x ....(i) æ 325 + 314 + 312 + 278 + 292 + 274 ö
For company B, = çè ÷ø km
6
x - E2
30 = ´ 100 æ 1795 ö 1
E2 = çè ÷ km = 299
6 ø 6
Þ 130E2 = 100x ....(ii)
From equations (i) and (ii), 92. (d) Required time
E1 130 240
= = 13 :15 = = 12.5hours = 12 hours 30 minutes
E 2 150 19.2
93. (c) Speed of truck Q on Friday
80. (d) It cannot be determined as data for yearwise actual
profit of each company is not available. 302
= = 37.75 kmph
81. (e) Required average marks in English 8
æ 67 + 59 + 66 + 71 + 63ö 326 94. (b) `Total distance travelled by all the trucks on Saturday
= çè ÷ø = = 65.2 = (292 + 284 + 260 + 274 + 280 + 242) km = 1632 km
5 5 95. (b) Required ratio =308:318
82. (c) Varun’s total marks = 154 : 159
76 ´ 125 88 ´ 150 75 ´ 68 72 ´ 50 96. (b) Required ratio
= 63 + + + +
100 100 100 100 25780 ´ 12 7390 ´ 11
= 63 + 95 + 132 + 51 + 36 = 377 = : = 3094:813
100 100
377 24
\ Required percentage = ´ 100 = 75.4
500 97. (b) Required percentage = ×100 = 150
16
83. (a) Marks obtained by Veena and Shreya together in Maths
98. (e) Required difference
74 ´ 150 80 ´ 150
= + = 111 + 120 = 231 4 ´ 7390
100 100 = (11-7)% of 7390 = = 296
100
y
o
u
rs
m
50 SBI & its Associates Bank PO Exam 2010

a
h
99. (c) It is obvious from the Pai chart. 164. (d) Unconvinced of the long term effects and rationale for

b
o
immediate requirement of genetically modified products.

o
25780 ´ 28 7390 ´ 32

b
Science Þ - 165. (b) 166. (e) 167. (a) 168. (b)

.w
100 100 169. (e) 170. (d)

o
» 7218 – 2365 » 4853 171. (c) The word construe (verb) means : to understand the

rd
25780 ´ 16 7390 ´ 11 meaning of a word, sentence in a particular way: interpret

p
Engineering Þ -

re
Look at the sentence :
100 100

s
He considered how the remark was to be construed.

s
» 4124 – 813 » 3311

.c
Hence, the words construe an d interpret are

o
25780 ´ 18 7390 ´ 16 synonymous.

m
Commerce Þ - 172. (d) The word Superfluous (Adjective) means : more than
100 100
» 4640 – 1182 » 3458 you need or want. The word surplus means the same.
100. (a) Required number of candidates 173. (a) The word Appalling (Adjective) means : extremely bad:
shocking
23 ´ 7390 Look at the sentence:
= 23% of 7390 = » 1700
100 The prisoners were living in appalling conditions. The
101. (c) 102. (d) 103. (c) 104. (c) word sinister (Adjective) means: seeming evil or
105. (c) 106. (b) 107. (a) 108. (c) dangerous; making you think something bad will
109. (b) 110. (a) 111. (c) 112. (a) happen. Hence, the words appalling and sinister are
113. (b) 114. (c) 115. (e) 116. (e) synonymous.
117. (d) 118. (d) 119. (d) 120. (c) 174. (b) The word Torture (Verb) means: to hurt somebody
121. (e) 122. (e) 123. (a) 124. (b) physically or mentally ; make somebody feel extremely
125. (d) 126. (d) 127. (e) 128. (c) unhappy or anxious.
129. (c) 130. (c) 131. (a) 132. (c) Look at the sentence:
133. (a) 134. (a) 135. (a) 136. (b) Many of the rebels were captured and for tured by
137. (a) 138. (d) 139. (e) 140. (c) secret polic the word Exeruciate(Verb) beans the same
141. (e) 142. (c) 143. (c) 144. (c) meaning.
145. (b) 146. (b) 147. (e) 148. (d) 175. (a) The word Pertinent (Adjective) means: appropriate to
149. (a) 150. (d) a particular situation; relevant.
151. (d) The public is ready to invest in these banks because of Look at the sentence : Please keep your comments
the knowledge that these banks get strong support pertinent to the topic under discussion.
from the Government. Hence, the words pentinent and irrelevant are
152. (c) Encouragement antonymous.
153. (a) Worldwide, companies have realized that India is a 176. (b) 177. (a) 178. (c) 179. (d)
strong power to reckon with 180. (e) 181. (d) 182. (b) 183. (e)
154. (e) None of these 184. (a) 185. (e) 186. (c) 187. (a)
155. (b) Only (b) 188. (d) 189. (a) 190. (e)
156. (e) All (a), (b) and (c) 191. (b) Many people in rural areas still remain ignorant of its
157. (e) The meaning of the word Turmoil (Noun) as used in immense benefits.
the passage is : a state of great anxiety and confusion. 192. (d) The government is determined to extract maximum out
The word Chaos (Noun) means : a state of compete of this technology in the near future.
confusion and lack of order. 193. (c) Based on the fact that difference in the quality of
Hence, the words turmoil and chaos are synonymous. schools acts as a ground for discrimination.
158. (e) The meaning of the word Pulverization (Noun) as used
194. (c) Owing to the uninhibited use of chemical inputs in
in the passage is : defeat or destruction of something/
agriculture.
somebody completely: crushing.
195. (a) government ordered an enquiry which exposed a nexus
159. (b) The meaning of the word Thrive (Verb) as used in the
passage is : to flourish; to become and continue to be between mine operators and bureaucrats
successful, strong, healthy etc. 196. (b) Slum dwellers not only play a significant role in urban
The word Deteriorate (Verb) means : to become worse. economy but also provide cheap labour for everyday
Hence, the words thrive an d deteriorate are work of the cities.
antonymous. 197. (a) Entry of various market forces increased the competition
160. (a) The meaning of the word Mundane (Adjective) as used faced by the indigenous organizations manifold.
in the passage is : not interesting or exciting : dull, 198. (c) A new loan scheme started by the government a couple
ordinary. of months ago proved to be a huge failure and utterly
Look at the sentence : unpopular among the farmers.
I lead a pretty mundane existence. Hence, the words 199. (c) Government has politely declined the request to
mundane and extraordinary are antobymous. handover the arrested terrorists for prosecution outside
161. (c) Valuable. the country
162. (b) The words open and capable are synonymous. 200. (d) Although government has been making many
163. (e) The word Volatile (Adjective) means : changing easily provisions to improve higher education in India, no
from one mood to another : unstable. Its antonym attention has been paid to the education of the
should be stable. physically challenged students.
y
o
u
rs
m
a
h
b
o
o
SBI PO EXAM 2011

b
.w
o
rd
p
Based on Memory

re
s
s
.c
o
5. If ‘OD’ is related to 'GF' and 'EB is related to 'NP' in a certain
REASONING (High Level)

m
way, to which of the following is 'AL' related to, following
1. How many meaningful English words can be formed with the same pattern ?
the letters NCEO using each letter, only one in each word ? (a) KD (b) QV
(a) None (b) One (c) KL (d) KV
(c) Two (d) Three (e) DQ
(e) More than three 6. What will come in place of question mark (?) in the following
2. The positions of first and the fourth letters of the word series based on the above alphabetical series?
LIQUID are interchanged, similarly, the positions of second TE JU NK ? GH
and fifth letters and third and Sixth letters are interchanged. (a) IS (b) IR
In the new arrangement thus formed, how many letters are (c) AG (d) AR
there in the English alphabetical series between the (e) AM
alphabets which are at the extreme ends? 7. If in a certain code 'GRIM' is coded as 'RMOS' and 'DUSK' is
(a) None (b) Two coded as 'LQPI' how will 'STOP' be coded in the same code
(c) Three (d) Four language?
(e) More than four (a) MJIS (b) PJGB
(c) JPJIS (d) MJGB
DIRECTIONS (Qs. 3-4): Study the following information to
(e) PGJB
answer the given questions :
8. Among five friends, P, Q, R, S and T, each scored different
Point P is 5 m towards the South of Point M. Point Q is 3 m
marks in the examination. P scored more than Q but less
towards the East of Point P. Point O is 3 m towards the East of than R. S scored more than' only T. Who amongst the
Point M. Point N is 2 m towards the South of Point Q. following scored the second highest marks?
3. A person, facing North, takes a left turn from point M, walks (a) P (b) Q
4m and stops. He then takes another left turn, walks 5 m and (c) R (d) S
stops at point R. Which of the following points, including
(e) T
R, fall in a straight line ?
9. If each of the alphabets of the word MIRACLE is arranged
(a) M, O, R (b) N, R, P in alphabetical order from left to right and then each vowel
(c) R, O, Q (d) R, Q, N in the new word thus formed is changed to the next letter in
(e) Q, P, R the. English alphabetical series and each consonant is
changed to the previous letter in the English alphabetical
4. How far and towards which direction is Point O from Point
series which of the followingwill be fifth from the right?
N?
(a) K (b) S
(a) 5 m towards South
(c) F (d) E
(b) 7 m towards North (e) J
(c) 8 m towards West 10. How many such, pairs of letters are there in the word
(d) 7 m towards West INCLUDE, each of which has as many letters between them
(e) 5 m towards North in the word (in both forward and backward directions) as
they have between them in the English alphabetical order?
DIRECTIONS (Qs. No. 5-7): The following questions are based
(a) None (b) One
upon the alphabetical series given below:
(c) Two (d) Three
T J E N UQ AK I O G R M S PB HF D LVC (e) More than three
y
o
u
rs
m
52 SBI PO Exam 2011

a
h
b
DIRECTIONS ( Qs. 11-15) : Study the following information to 19. H is related to C and B is related to E in a certain way. To

o
o
answer the given questions: whom amongst the following is G related following the same

b
.w
In a certain code 'support the other group' is written as 'ja pe la no' pattern ?

o
'the mission gains support' is written as' ke ja zi la', 'gains other (a) F (b) H

rd
than money' is written as 'fu no ho zi' and 'more support and (c) C (d) A

p
re
money' is written as ' re qi fuja' . (e) D

s
s
11. What is the code for 'group' ?

.c
20. What will come in place of the question mark ?

o
( a ) ja (b) pe DC DB DF DA

m
(c) la (d) no (a) DG (b) DE
(e) Cannot be determined (c) DH (d) DK
12. What does 'zi' stand for ? (e) Either DK or DE
(a) mission (b) than 21. Who sits third to the left of A?
(c) other (d) the (a) H (b) G
(e) gains (c) C (d) F
13. Which of the following may represent 'more than the group'? (e) B
(a) la qi ho pe (b) re la qi ho 22. Which of the following pairs represents the immediate
(c) re no la pe (d) pe ke qi la neighbours of C?
(e) qi ho la fu (a) FH (b) AB
14. What is the code for 'mission'? (c) BD (d) EK
(a) la (b) zi (e) DF
(c) ke (d) ja DIRECTIONS (Qs. 23-27): In each of the question below is
(e) ke or la given a statement followed by two assumptions numbered I and
15. Which of the following may represent 'money matters more'? II. An assumption is something supposed or taken for granted.
You have to consider the statement and the following
(a) fu bu (b) re bu qi
assumptions and decide which of the assumptions is implicit in
(c) zi qi yo (d) yo fu no
the statement.
(e) la fu bu
Give answer (a) if only Assumption I is implicit.
DIRECTIONS (Qs. 16-22): Study the following information
carefully and answer the given questions. Give answer (b) if only'Assumption II is implicit.
Nine friends A, B, C, D, E, F, G, H and K are sitting around a circle Give answer (c) if either Assumption 1 or Assumption II is implicit.
facing the centre. A sits second to left of D. K sits third to right of Give answer (d) if neither Assumption I nor Assumption II is implicit.
F. Neither K nor F is an immediate neighbour of A Give answer (e) if both Assumptions I and II are implicit.
or D. G and H are immediate neighbours of each other. E sits third
23. Statement: The biggest private airline decided to increase
to right of H. B is not an immediate neighbor of F.
the number of flights between cities A and B to 10 flights
16. What is the position of F with respect to the position of B ?
everyday.
(a) Second to the right (b) Third to the left
Assumptions:
(c) Second to the left (d) Third to the right
I. Other private airlines may also increase the frequency
(e) Sixthtotheright
of daily flights between cities A and B.
17. Who amongst the following is an immediate neighbour of H ?
II. There may be adequate passenger load on all the
(a) C (b) B
flights of the biggest private airline even after increased
(c) K (d) F
frequency.
(e) A
24. Statement: Majority of the employees of the organisation
18. Starting from A, if all the friends are made to sit in the decided to join with their family the overnight picnic funded
alphabetical order in clockwise direction, the positons of by the organisation.
how many (except A) will remain unchanged ? Assumptions:
(a) None (b) One I. The management of the organization may not welcome
(c) Two (d) Three the employees' enthusiasm.
II. The management of the organization may provide
(e) Four
adequate funds for the picnic.
y
o
u
rs
m
SBI PO Exam 2011 53

a
h
b
25. Statement : The reputed management institute in the city (B) The prices of sugar in the retail market have stabilized

o
increased the fees by 300 percent from the next academic

o
after a considerable gap.

b
year.

.w
31. (A) Many vehicles met with accidents during the last
Assumptions :

o
fortnight at the major junction on the main arterial road

rd
I. The institute may still attract good number of students in the city.

p
for all its courses.

re
(B) The automatic signaling system at the major junction

s
II. The students may now opt for other institutes in the

s
original statement from question of the arterial road of

.c
city which charge less fees.

o
the city.

m
26. Statement : Many residents of the locality decided not to
attend the cultural function organised by the local club to 32. (A) The english medium school in the locality has decided
protest against the club's limited invitations. to admit only those students who reside in area. where
Assumptions: the school is located from the next academic session.
I. The local club may cancel the cultural function. (B) Many school in the city admit students residing
II. The local club may stop all its activities. anywhere within the city limit.
27. Statement : Government, has announced a relief package DIRECTIONS (Qs. 33-37): In each of the questions below are
for all the drought hit farmers in the country and advised three statements followed by two conclusions numbered I and
the state Governments to put in a mechanism for II. You have to take the three statements to be true even if they
disbursement. seem to be at variance from commonly known facts and then
Assumptions: decide which of the given conclusions logically follows from
I. The state Governments may be able to put in place the the three statements disregarding commonly known facts.
system for disbursement of Government relief to the
Give answer (a) if only conclusion I follows.
affected farmers.
II. Government may be able to identify all the farmers Give answer (b) if only conclusion II follows.
affected by drought in the country. Give answer (c) if either conclusion I or conclusion II follows.
DIRECTIONS (Qs. 28-32) : Below in each questions are given Give answer (d) if neither conclusion I nor conclusion II follows.
two statements (A) and (B). These statements may be either Give answer (e) if both conclusion I and conclusion II follow.
independent causes or may be effects of independent causes or a
common cause. One of these statements may be the effect of the 33. Statements:
other statement. Read both the statements and decide which of All plants are bottles.
the following answer choice correctly depicts the relationship All bottles are caps.
between these two statements.
All caps are crowns.
Mark answer (a) if statement (A) is the cause and statement (B)
is its effect. Conclusions:
Mark answer (b) if statement (B) is the cause and statement (A) I. Atleast some crowns are bottles.
is its effect.
II. All plants are caps.
Mark answer (c) if both the statements (A) and (B) are
independent causes. 34. Statements:
Mark answer (d) if both the statements (A) and (B) are effects of Some shoes are hankerchiefs.
independent causes. Some hankerchiefs are calculators.
Mark answer (e) if both the statements (A) and (B) are effects of
some common cause. All calculators are paper.
28. (A) The local traders' association urged all its members to Conclusions:
shut their establishments for two days and participate I. No calculator is a shoe.
in the protest march.
II. No shoe is a paper.
(B) Government has recently cancelled licenses of many
traders in the locality as they did not pay their taxes in 35. Statements:
time. All zebra are cows.
29. (A) Government has increased the procurement price of
All camels are cows.
kharif crops by about ten per cent for the current year.
All tigers are zebra.
(B) Inadequate monsoon rainfall has created a drought
Conclusions:
like situation in many parts of the country.
I. All tigers are cows.
30. (A) The Government has recently lifted ban on import of
sugar for selling in the retail market. II. All camels being tigers is a possibility.
y
o
u
rs
m
54 SBI PO Exam 2011

a
h
b
36. Statements: Give answer (d) If the data in both the Statements I and II are not

o
o
Some pencils are mobiles. sufficient to answer the question.

b
.w
All mobiles are grass. Give answer (e) If the data in both the Statements I and II are

o
All grass is green. together necessary to answer the question.

rd
p
Conclusions: 38. Who is the tallest of A, B, C, D and E ?

re
I. All grass being pencils is a possibility.

s
I. B is shortest amongst the five people.

s
.c
II. No Green is mobile.
II. C is taller than D.

o
m
37. Statements:
39. How is ‘stand’ coded ?
All watches are cards.
I. ‘cost of pen’ is coded as 'lake if.
Some cards are clips.
All clips are chairs. II. ‘pen stand’ is coded as ‘kehu’.
Conclusions: 40. How many marks did Sudhir score in Maths ?
I. All chairs being cards is a possibility. I. Sudhir has scored 6 5% marks overall.
II. All clips being watches is a possibility. II. The difference between Sudhir’s marks in Maths and
DIRECTIONS (Qs. 38 - 42) : Each of the following questions English i s 12 marks.
below consists of a question and two statements numbered I and
41. On which day of the week did Mansi take leave f r om her
II given below it. You have to decide whether the data provided
in the statements are sufficient to answer the question. Read office ?
both the statements and- I. Mansi correctly remembers that she took leave before
Give answer (a) If the data in Statement I alone is sufficient to Friday but after Monday.
answer the question, while the data in statement II alone are not
sufficient to answer the question. II. Mansi’s friend correctly remembers that Mansi took
Give answer (b) If the data in Statement II alone is sufficient to leave before Saturday but after Thursday.
answer the question, while the data in statement I alone are not 42. How many brothers Mary have (Mary is a girl) ?
sufficient to answer the question.
I. Mary has only one younger brother.
Give answer (c) If the data in either Statement I alone or statement
II. Mary's father has only one son.
II alone are sufficient to answer the question.
DIRECTIONS (Qs. 43-47) : In each of the question given below which one of the five answer figures on the right should come after
the problem figures on the left, if the sequence were continued?
Problem Figures Answer Figures
43.

1 2 3 4 5 (a) (b) (c) (d) (e)


44.

1 2 3 4 5 (a) (b) (c) (d) (e)


45.

1 2 3 4 5 (a) (b) (c) (d) (e)


46. Z U Z

Z Z Z Z Z Z
U U Z U U U U U Z U U

1 2 3 4 5 (a) (b) (c) (d) (e)


47.

1 2 3 4 5 (a) (b) (c) (d) (e)


y
o
u
rs
m
SBI PO Exam 2011 55

a
h
b
DIRECTIONS (Qs. 48-50) : A word arrangement machine, when Table-I

o
Yrs ®

o
given a particular input, rearranges it following a particular

b
1997 1998 1999 2000 2001 2002
rule. The following is the illustration of the input and the steps Com. ¯

.w
of arrangement : BS 3:5 8:7 1:2 4:5 6:5 5:2

o
rd
Input : and band land hand hind lack job TIS 1 : 2.5 9 : 8 13 : 15 7:6 6:7 14 : 9

p
Step I : hind and band lack land hand job SA I 13 : 15 7 : 9 1:3 11 : 7 10 : 7 62 : 71

re
Step II : hind band land job and lack hand MPI 41 : 59 17 : 8 100 : 119 53 : 42 24 : 25 7 : 9

s
s
.c
Step III : hind and lack band hand land job ES 2 : 7 12 : 19 6:5 3 : 11 10 : 13 19 : 21

o
Step IV : land band and job hand lack hind LTS 13 : 11 21 : 23 5:3 4 : 7 1 : 1.29 7 : 5

m
Step V : hand land band lack and job hind Table-II
Step VI : hand band and hind land lack job 51. The production of steel A by company MPI in 1999 is
and so on. approximately what per cent of production in 2001?
As per the rule followed in the above steps, find out the (a) 51 % (b) 53%
appropriate step for the given input or vice versa in the (c) 55% (d) 60%
following questions. (e) 64%
48. Input : do we he is it at all 52. In 2001, in how many companies production of A type steel
Which of the following steps would be is more than that of B type steel?
“all we he is do at it”? (a) No company (b) Two
(a) It is not possible to get the above step. (c) Three (d) Cannot be determined
(b) Step VI (c) Step IX (e) None of these
(d) Step X (e) None of these 53. For how many companies did the production of steel A
49. If Step IV of an input is “he is to do what her observe”. increase every year and the production of steel B decrease
Which of the following would definitely be the input? every year together from that of the previous year?
(a) to is he what observe her do (a) No company (b) One
(b) he is to what observe her do (c) Two (d) Three
(c) is he to what observe her do
(e) None of these
(d) Can’t say
54. Production of TIS increases by 10% in 2003 and production
(e) None of these
of SAI decreases by 10% in 2003 in comparison to 2002. If
50. If step III of an input is the ratio of production remains the same as in 2002, find the
“when then men can how are you”. ratio of production of A type to B type steel for both the
What would be step VII of the input? companies together.
(a) then can are when you men how (a) 8026 : 6561 (b) 7026 : 7561
(b) how are men can you then when (c) 8097 : 6061 (d) 8026 : 5061
(c) you then can men are when how (e) None of these
(d) how can then men are when you 55. If the profit ratio per unit tonne of steel A and B is 3 : 4 for
(e) None of these company BS, what is the actual profit ratio of the company
BS for the year 1998 for steel A and B?
DATA ANALYSIS & INTERPRETATION (a) 6 : 7 (b) 3 : 4
(c) 8 : 7 (d) Can’t be determined
DIRECTIONS (Qs. 51-55) : Study the following table to answer (e) None of these
the given questions : 56. On the ground 12 stones are placed. The distance between
Each company produces two types of steel. In table I the the first and the second is 1 metre, between second and 3rd
total production (in lakh tonnes) of both types of steel together 3 m, between 3rd and 4th 5 m, and so on. How far will a boy
of six companies over the years is given. In table II the ratio of have to run to touch the last stone if he starts from the first?
production of two types A and B (A : B) is given over the years. (a) 144 m (b) 121 m
(c) 132 m (d) 110 m
Yrs . ® (e) None of these
1997 1998 1999 2000 2001 2002 Total
Com. ¯ 57. A dishonest dealer prefers to sell his goods at cost price
BS 424 390 258 756 319 427 2574
TIS 339.5 663 812 598 663 782 3857.5 1
but uses less weight for a kg weight and gains 4 % .
SA I 532 576 364 936 595 665 3668 6
MPI 620 850 876 1045 1274 1296 5961 What does he use for a kg weight?
ES 612 806 627 406 874 760 4085 (a) 950 gm (b) 980 gm
LTS 840 836 776 748 384.72 816 4400.72 (c) 960 gm (d) 840 gm
Total 3367.5 4121 3713 4489 4109.72 4746 24546.22
(e) None of these
y
o
u
rs
m
56 SBI PO Exam 2011

a
h
b
58. In a town three newspapers A, B and C are published. 42% 64. The ratio of the number of boys to that of girls was 1 : 2 but

o
of the people in that town read A, 68% read B, 51% read C, when 2 boys and 2 girls left, the ratio became 1 : 3. How

o
b
30% read A and B, 28% read B and C, 36% A and C and 18% many people were at the party originally?

.w
do not read any paper. Find the % of population of town (a) 12 (b) 15

o
rd
that reads all the three. (c) 10 (d) 18

p
(a) 15% (b) 25% (e) None of these

re
s
(c) 20% (d) 35% 65. A wheel of a motorbike has radius 35 cm. How many

s
.c
(e) None of these revolutions per minute must the wheel make so that the

o
speed of the bike is 33 km/hr?

m
59. The time taken by a man to walk five times around the
boundary of a square field having 16 hectares as area, at (a) 300 (b) 250
the rate of 5 km per hour is (c) 200 (d) 220
(a) 16 minutes (b) 24 minutes (e) None of these
(c) 48 minutes (d) 96 minutes DIRECTIONS (Qs. 66 & 67) : Contribution of different sources
(e) None of these of water to fulfil the requirement in Delhi and consumption of
60. When the numerator and the denominator of a fraction are water for different uses by two major sources.
Total consumption of water = 720 million litres
2
increased by 1 and 2 respectively, the fraction becomes ,
3 Miscellaneous
Sutlej Jhelum
and when the numerator and the denominator of the same 7% 6% Building 5%
Projects
fraction are increased by 2 and 3 respectively, the fraction Under- 15%
ground Agric-
5 12% ulture
becomes . What is the original fraction? Yamuna 30%
7 40%
Industry
10%
Ganga
5 3 35% Domestic
(a) (b) 40%
6 4
Yamuna
3 6
(c) (d)
5 7
(e) None of these Miscellaneous
Agriculture
20%
61. Find the number of numbers between 300 and 3000 that can 11%
be formed with the digits 0, 1, 2, 3, 4 and 5, no digit being Building
repeated. projects
12%
(a) 120 (b) 160
Domestic
(c) 240 (d) 60 23%
(e) None of these Industry
62. A salesgirl’s terms were changed from a flat commission of 34%

5% on all her sales to a fixed salary of ` 1000 plus 2.5%


commission on all sales exceeding `4000. If her remuneration Ganga
as per the new scheme was `600 more than that by the
previous scheme, her total sales was 66. How many litres of Yamuna water is consumed for building
projects in Delhi?
(a) `10000 (b) `5000
(a) 39.9m litre (b) 43.2 m litre
(c) `2000 (d) `12000
(e) None of these (c) 47.3 m litre (d) 51.9 m litre
63. A can do a piece of work in 10 days and B is 25% more (e) None of these
efficient than A. In what time will the work be finished if A 67. What is the ratio of supply of Jehlum water and underground
and B work together? water together to consumption of Yamuna water for domestic
purposes and Ganga water for Agriculture purposes
4 5 together?
(a) 4 days (b) 5 days
9 7 (a) 17 : 23 (b) 5 : 8
2 (c) 17 : 22 (d) 18 : 23
(c) 5 days (d) 6 days
3 (e) None of these
(e) None of these
y
o
u
rs
m
SBI PO Exam 2011 57

a
h
b
DIRECTIONS (Qs. 68-73) : Study the following table carefully

o
Mark s ®

o
and answer the questions given below it : 0 -2 9 30 -5 9 60 -8 9 9 0-11 9 1 20 -1 50
S ub ¯

b
.w
Area and Population of different states Maths 22 47 74 25 12

o
rd
S tates Area (in s q k ilometres ) Population (in lak hs ) S cience 39 38 67 22 14

p
A 6230 1122

re
Hindi 19 59 47 36 19
B 2540 838

s
s
C 8135 649 Englis h 24 41 58 34 23

.c
o
D 7436 572 Geography 42 32 52 41 13

m
E 4893 711 Average of
F 3718 286 27 45 60 31 17
five s ubjects
G 4297 860
74. If for passing, the student has to obtain minimum 60% marks
68. Among the given states, in case of how many states the in the average of five subjects, how many students will
area of that state was more than 15 per cent of the total pass?
areas taken together? (a) 108 (b) 58
(a) One (b) Three (c) 48 (d) 72
(e) None of these
(c) two (d) Can’t say
75. How many students will pass in Geography if minimum
(e) None of these passing marks is 40%?
69. For which two states the density of population is (a) 74 (b) 106
approximately equal? (c) 96 (d) Can’t say
(a) No state (b) A and G (e) None of these
(c) D and F (d) C and F 76. How many students have obtained 60 or more marks in at
(e) None of these least one of the five subjects?
(a) 111 (b) 103
70. Approximately how much more is the density of population
(c) 108 (d) 106
of state B in comparison to that of state A?
(e) Data inadequate
(a) 15000 (b) 18000
77. If the criteria for distinction is minimum 75% marks in Maths,
(c) 13000 (d) 14000 how many students will get distinction?
(e) 17000 (a) 37 (b) 27
71. In case of how many states the density of population was (c) 12 (d) Can’t say
more than 12 thousand per square kilometre? (e) None of these
(a) Two (b) Five 78. The no. of students who obtained more than or equal to
(c) Three (d) Four 40% marks in Science is what per cent less than that of
those who scored less than or equal to 60% in Hindi?
(e) None of these
(a) 17.60% (b) 15.40%
72. What is the approximate ratio of the areas of state B to the (c) 19.80% (d) 24.30%
areas of state A and G together. (e) None of these
(a) 1 : 3.8 (b) 1 : 3.5
DIRECTIONS (Qs. 79-83) : Study the following data carefully
(c) 1 : 5.2 (d) 1 : 4.5 and answer accordingly.
(e) 1 : 4.1 Following chart shows the number of students in different
73. The surface area of a spherical part of a bowl with a flat universities
circular detachable cover, excluding the cover, is 616 sq cm. Delhi Uni
35%
The area of the cover is 38.5 sq cm. What is the volume of Indraprastha
Uni 13%
the bowl?
(a) 1339 cm3 (b) 1430 cm3
(c) 1570 cm3 (d) Cannot be determined Hamdard
14%
(e) None of these
DIRECTION (Qs. 74-78) : Study the following table carefully J.N.U
20%
and answer accordingly : Jamia
18%
The distribution of marks (out of 150) obtained by 180
students in each of the five subjects. Total no. of students = 120,000
y
o
u
rs
m
58 SBI PO Exam 2011

a
h
b
Percentage of listeners of different FM channels in 85. How much time will Dinesh take to walk a km distance?

o
National Capital Region I. The ratio of the speeds at which Dinesh and Ranjay

o
b
walk is 5 : 6.

.w
FM Chann els II. The average walking speed of Dinesh and Ranjay is

o
Un ivers ities ¯ Rad io Radio Red FM

rd
Rain bo w known.
M irch i City FM Go ld

p
86. Is a two-digit integer ‘x’ divisible by 12?

re
Ind rap ras tha 76% 72% 46% 54% 48% I. When ‘x’ is divided by 5, the remainder is 2.

s
s
Hamd ard 63% 64% 59% 47% 53% II. When ‘x’ is divided by 3, the remainder is 1.

.c
87. Is the average of a, b and c equal to b?

o
JNU 52% 65% 64% 51% 54%

m
DU 82% 44% 32% 35% 45% I. b - a = c - b
II. a, b and c are positive integers.
Jamia 75% 32% 36% 52% 64%
88. What is the monthly income of Rahim?
79. How many students of JNU listen to Radio city? I. Total monthly income of Rahim and Suresh is ` 27000,
(a) 15200 (b) 15600 which is 150% of their total monthly expenditure.
(c) 14400 (d) 14600 II. The ratio of their monthly expenditures is 5 : 4
(e) None of these DIRECTIONS (Qs. 89 - 92) : Answer these questions on the
80. The no. of Indraprastha students listening to Rainbow is basis of the information given below :
what per cent of the no. of Jamia students listening FM
In a survey of 1000 boys conducted in an area, it is found that
Gold?
65% play Cricket, 48% play Football and 40% play Hockey. Of the
(a) 65 (b) 56
total, 30% play both Football and Cricket, 25% play Football and
(c) 68 (d) 58
Hockey, while 24% play Cricket and Hockey. Only 5% do not play
(e) None of these
any of the three games.
81. From which of the following universities, the no. of students
liking Red FM is minimum? 89. Find the number of players who play Football but not
(a) Indraprastha (b) Jamia Hockey.
(c) JNU (d) DU (a) 180 (b) 230
(e) Hamdard (c) 350 (d) Can’t be determined
82. How many students of Indraprastha and Jamia together (e) None of these
listen to Red FM? 90. How many play all the three above-mentioned games?
(a) 12562 (b) 12872 (a) 180 (b) 240
(c) 14952 (d) 14272 (c) 230 (d) 210
(e) None of these (e) None of these
83. Which of the following channels is the most popular among 91. How many play Hockey but neither Cricket nor Football?
the students of Hamdard and JNU? (a) 140 (b) 320
(a) Radio Mirchi (b) Radio city (c) 120 (d) Can’t be determined
(c) Red FM (d) FM Gold (e) None of these
(e) Rainbow 92. Find the percentage of players who play only Football.
(a) 12 (b) 14
DIRECTIONS (Qs. 84 - 88) : Each of the questions below consists
(c) 32 (d) 18
of a question and two statements numbered I and II given below
it. You have to decide whether the data provided in the statements (e) None of these
are sufficient to answer the questions. Read both the statements DIRECTIONS (Qs. 93-97) : The following questions are
and: accompanied by three statements A, B and C. You have to
Give answer (a) if the data in statement I alone are sufficient to determine which statement(s) is/are necessary/sufficient to
answer the question, while the data in statement II alone are not answer the question.
sufficient to answer the question.
93. P, Q and R together invested an amount of `42000 in the
Give answer (b) if the data in statement II alone are sufficient to ratio of 4 : 3 : 7 for different periods of time. What was the
answer the question, while the data in statement I alone are not amount of profit earned by them individually at the end of
sufficient to answer the question. one year?
Give answer (c) if the data either in statement I alone or in A. They invested for periods in the ratios of 1 : 2 : 1.
statement II alone in sufficient to answer the question. B. R’s profit is ` 5500 less than Q’s investment.
Give answer (d) if the data even in both the statements I and II C. Total amount of profit at the end of one year is `8800.
together are not sufficient to answer the question. (a) Only A and B together
Give answer (e) if the data in both the statements I and II together (b) Only A and C together
are necessary to answer the question. (c) A and either B or C
84. What is the height of a triangle? (d) All statements are required
I. It is a right-angled triangle. (e) Question can’t be answered even after using all the
II. The area of the triangle is 5 times its base. informations
y
o
u
rs
m
SBI PO Exam 2011 59

a
h
b
94. What was the sum of the ages of the mother and the son DIRECTIONS : (Qs. 98-100) : Study the following table carefully

o
o
ten years earlier? and answer accordingly :

b
.w
A. Ratio of the mother’s present age to the son’s age after The different five-star hotel projects completed

o
10 years is 5 : 3. by different companies.

rd
B. The difference between thrice the present age of the

p
No. of Cos t Year of

re
mother and five times the present age of the son is 50 Project Company
rooms (` cr) completion

s
s
years.

.c
A 600 275 1998 P

o
C. The ratio of the age of the mother after 10 years to the B 320 210 1999 Q

m
age of the son after 16 years is 5 : 3. C 250 250 1999 R
(a) Any two of them D 400 430 1998 S
(b) Any one of them E 520 310 2000 T
F 450 400 1998 U
(c) B and either A or C
G 500 250 2000 V
(d) All statements are required
98. Which project had the minimum cost per room?
(e) Question can’t be answered even after using all the
(a) A (b) B
informations
(c) G (d) E
95. What will be the cost of fencing a rectangular plot?
(e) D
A. Cost of fencing a circular plot whose area is 616m2 is 99. What is the investment in projects to be completed in 1998
` 968. as percentage of investment in projects to be completed in
B. Perimeter of the rectangular plot is 200 m. all three years?
C. Perimeter of the square whose length is equal to the (a) 41% (b) 52%
breadth of the rectangular plot is 20 m. (c) 47% (d) 56%
(a) Only C (b) A and C together (e) None of these
100. What is the approximate average number of rooms that
(c) A and B together (d) A and either B or C
would be built per crore of rupees over the three-year period?
(e) Question can’t be answered even after using all the (a) 1.25 (b) 1.70
informations (c) 0.90 (d) 1.40
96. What is the cost price of an article? (e) 1.55
A. After allowing a discount of 10% on marked price the ENGLISH LANGUAGE
shopkeeper charges ` 810.
B. If the shopkeeper does not give the discount the DIRECTIONS (Qs. 101-109) : Read the following passage
shopkeeper gets a profit of 50%. carefully and answer the questions given below it. Certain words
are given in bold to help you locate them while answering some
C. If the shopkeeper gives only 5% discount on marked
of the questions.
price, he will have 42.50% profit.
It may be quite a while before climatologists are able to
(a) Any two of them (b) Only A and B together predict rainfall in the American Midwest by measuring snow-fall
(c) Only B and C together (d) A and either B or C in the Himalayas. But there is one prediction which they can
(e) Only A confidently make now, and that is that the earth’s ice cover —
from the polar ice caps to the Himalayas — is thawing at an
97. What is the rate of interest at which Binod has invested
alarming rate. So much so that over 50 per cent of the planet’s
money? mountain glacier mass could be history by the turn of the next
A. The compound interest at this rate of ` 2500 in 2 yrs is century. According to the latest findings of the US-based
environmental thinktank, Worldwatch institute, the Arctic Sea
2
equal to the simple interest in 3 yrs of ` 1716 at the ice has shrunk by nearly 40 per cent in the last 25 years, even as
3 Antarctica’s extensive ice fields and glaciers have been badly
same rate. ‘bleeding’ at their edges. This would indicate that the earth has
B. The total simple interest on an investment of ` 12000 entered a period of climatic change that is likely to cause
widespread environmental, economic and social disruption over
for 3 yrs and `10000 for 5 yrs at this rate is `5160.
the next century if emissions of heat-trapping gases are not
C. In 3 yrs `1500 at the same rate becomes `1725 by simple reduced. As a result of global warming, average planetwide
interest. temperatures have been going up steadily.
(a) Any of them (b) Either B or C If the levels of carbon dioxide — the bad boy of global
(c) Only C (d) Only A and either B or C warming — in the atmosphere are allowed to increase at the
present rate, more heat will be trapped in the planetary cocoon,
(e) Any two of them
y
o
u
rs
m
60 SBI PO Exam 2011

a
h
b
raising global temperatures to scorching highs. The polar ice 106. Over 50 per cent of the planet’s mountain glacier mass could

o
caps will melt and the resultant rise in sea levels will be catastrophic be history by the turn of the next century means

o
b
for low-lying island-states and countries with large coastal (a) In the next century mountain glacier will be taught as

.w
populations, such as Marshall Islands and Bangladesh. Regional a history subject.

o
flooding will threaten water supplies and dramatically alter the

rd
(b) As the next century starts, over half of the mountain
habitats of many flora and fauna. This is particularly bad news

p
glaciers will have become extinct.

re
for such regions as northern India, home to half of the total Indian (c) After the next century less than half of the planet’s

s
populace who depend wholly on the glacier-fed rivers for their

s
mountain glacier will be remaining.

.c
drinking water and irrigation needs.

o
(d) Next century will damage the glaciers heavily.

m
With the Himalayan ice caps melting like ice-cream on a hot
(e) Glacier will be a subject of interest in the next century.
summer day, these snow-fed rivers will first swell and then run
107. What efforts need to be taken to stabilise the climate?
dry, triggering off devastating floods, followed by a desolating
drought. People used to think there was time to sort out problems (a) Use of carbon should be checked and alternatives for
related to climate change, but no longer. The chilling prospect of it should be looked for to drive our industries and
an imminent global glacial melt calls for immediate damage control transport.
exercises to stabilise the climate. A good way to begin, perhaps, (b) Mountains and glaciers should be preserved at any
will be to overhaul the energy and transportation systems which cost
drive the world’s fossil fuel economy and, instead develop low- (c) The Himalayas should be given special protection as
carbon energy systems based on electronic technologies. they shape the climate.
101. Prediction of rainfall in the American Midwest depends on (d) People should not reside near natural flora and fauna.
(a) the rainfall in the region in previous years. (e) None of these
(b) the climatic conditions in the Himalayan region. 108. If the polar ice cap melts and the sea level rises
(c) the condition of glaciers. (a) it will bring more rain.
(d) the changing patterns of the season. (b) it will increase the amount of water in the world.
(e) None of these (c) it will submerge low coastal areas, thus destroying life
102. Which of the following will be the consequence(s) if there and property.
is a rapid decline in the ice cover of earth? (d) hilly areas will come under water.
(I) It will bring ecological disaster.
(e) excess water will generate diseases and epidemic.
(II) It will have negative effect on the economy.
109. Give a suitable title to the passage.
(III) It will affect the normal life mainly in the advanced
nations. (a) Glacier as a source of water
(IV) It will snatch the dreamland of our poets. (b) Glacier causing floods
(a) Only I and II (b) Only III and IV (c) Global warming and human survival
(c) Only III (d) Only I (d) Glacier and its importance
(e) None of these (e) Save water.
103. What is the prime cause behind the shrinking of ice fields? DIRECTIONS (Qs.110-112) : Choose the word which is same in
(a) human activity taking place at these places meaning as the word given in bold as used in the passage.
(b) scientific experiments being done in these area
110. THAWING
(c) rising temperature due to pollution in atmosphere
(a) diminishing (b) receding
(d) drying up of rivers which are snow-fed
(c) evaporating (d) melting
(e) None of these
104. What measure is imminent for saving our mountain glaciers (e) breaking
and ice fields? 111. SCORCHING
(a) launching a worldwide campaign to save them (a) extreme (b) mild
(b) checking the emission of carbon dioxide into the (c) uneven (d) odd
atmosphere (e) rapid
(c) Making people aware of our environment 112. IMMINENT
(d) depending less on the environment for livelihood (a) future (b) impending
(e) None of these (c) supposed (d) thought
105. Which of the following is false in the context of the passage? (e) surmise
(a) Worldwatch Institute is related to America.
(b) Ice cover of the earth is receding at a fast pace. DIRECTIONS (Qs. 113-115) : Choose the word which is opposite
in meaning of the word given in bold as used in the passage.
(c) Carbon dioxide is the major contributor to global
warming. 113. SHRUNK
(d) Snow-fed rivers will always have water in it. (a) developed (b) emerged
(e) None of these (c) built (d) widened
(e) multiplied
y
o
u
rs
m
SBI PO Exam 2011 61

a
h
b
114. CATASTROPHIC 123. (A) Unkempt (B) Unremitting

o
(a) fortunate (b) yielding (C) Slackening (D) Distasteful

o
b
(c) contributing (d) ushering (a) A-B (b) B-C

.w
(e) jovial (c) C-D (d) A-D

o
rd
115. STEADILY (e) B-D

p
(a) gradually (b) systematically 124. (A) Gregarious (B) Quixotic

re
s
(c) slowly (d) simply (C) Sociable (D) Discernible

s
.c
(e) inconstantly (a) A-B (b) B-C

o
m
DIRECTIONS (Qs. 116-120) : In each of the following sentences (c) C-D (d) A-C
there are two blank spaces. Below each sentence there are five (e) B-D
pairs of words denoted by (a), (b), (c), (d) and (e). Find out 125. (A) Apathetic (B) Wrath
which pair of words can be filled up in the blanks to make the (C) Whirl (D) Twirl
sentence meaningfully complete. (a) A-B (b) A-C
116. _________ of illiteracy from a nation that is set to become (c) A-D (d) B-C
the most populated in the world is by no ________easy. (e) C-D
(a) Countering, task (b) Driving, measure
DIRECTIONS (Qs. 126-130) : Some sentences are given below.
(c) Curbing, way (d) Eradication, means You have to arrange them in order to frame a meaningful
(e) Removal, point paragraph and then answer the following questions.
117. It is time to ________ ongoing programmes and ______ A. Invaders came to India and looted it of its wealth.
new horizons. B. Today we may be rich in wealth, but not rich at heart.
(a) value, choose (b) speculate, experiment C. Sadly, now the situation has changed.
(c) reject, consider (d) scrutinise, impound D. India has a glorious past with rich cultural heritage.
(e) assess, seek E. In fact, materialism has taken the place of spiritualism.
118. This approach would ________ the enormous illiteracy F. But they too admired the Indians.
problem to be ________ in a holistic manner. 126. Which sentence will come at FIRST place in the paragraph?
(a) enable, tackled (b) focus, viewed (a) A (b) B
(c) envision, dealt (d) combine, judged (c) C (d) D
(e) review, countered (e) E
119. The _______ of criminalisation of politics needs to be 127. Which will be the LAST sentence of the paragraph?
_________ far more seriously. (a) A (b) B
(a) lacuna, dealt (b) issue, addressed (c) C (d) D
(c) system, broken (d) continuation, suppressed (e) E
(e) process, diverted 128. Which sentence will come at SECOND place in the
paragraph?
120. It would be proper for India to judge Pakistan by its _______
(a) E (b) D
rather than ________ .
(c) F (d) A
(a) credentials, potentials (b) culture, politics
(e) B
(c) actions, words (d) promises, assurances
129. Which will be the FIFTH sentence in the paragraph?
(e) nature, behaviour (a) A (b) B
DIRECTIONS (Qs. 121-125) : In each of the following questions (c) C (d) D
four words are given, of which two are most nearly the same or (e) E
opposite in meaning. Find the two words which are most nearly 130. Which will be the THIRD sentence of the paragraph?
the same or opposite in meaning and mark the number of the (a) F (b) B
correct letter combination as your answer. (c) D (d) E
121. (A) Discomfit (B) Baffle (e) A
(C) Epicure (D) Enumerate DIRECTIONS (Qs. 131-140) : Read the following passage
(a) A-B (b) A-C carefully and answer the questions given below it. Certain words
(c) A-D (d) B-C are given in bold to help you locate them while answering some
(e) B-D of the questions.
122. (A) Testimony (B) Aura Nearly 6000 years ago, man took a giant step forward in his
(C) Augment (D) Decrease evolution on this earth when, giving up a nomadic type of life, he
(a) A-B (b) B-C took to agriculture. This memorable event happened in certain
(c) C-D (d) A-D river valleys of India, China, Mesopotamia (modern Iraq) and
(e) B-D Egypt. The reason was that the conditions in these areas were
y
o
u
rs
m
62 SBI PO Exam 2011

a
h
b
favourable for the cultivation of grains like wheat, rice etc because 131. Which step of man marks the end of his life as wanderers

o
of plenty of rich soil and water. Consequently, the farmer was on this earth?

o
b
able to produce more grains than he required for his needs. This (a) his taking up of jobs which were useful to the society

.w
helped in the development of settled cooperative living and (b) his shift towards agricultural activities

o
villages appeared where some people could make a living using

rd
(c) development of the sense of brotherhood
some specialised skills or take up other jobs useful to the society

p
re
instead of tilling the soil. (d) evolution of trade and commerce

s
(e) None of these

s
As time passed more and more families started living

.c
together for the sake of security against barbarians and wild 132. What led to the development of farming in India, China,

o
m
animals. They built big buildings and cities sprang up. Such Mesopotamia and Iraq?
developments took place in many parts of the world where (a) curiosity for producing grains among the people in
conditions were suitable for agriculture, trade and commerce. Since these areas
it was not hard to make a good living, men had spare time for the (b) early knowledge of agriculture among the people in
development of arts and crafts and engage in other activities these areas
which characterise a cultured life. (c) proper climate for agriculture
The ancient civilizations, though quite far removed from (d) vast area was available
each other, did not develop in complete isolation. There was
(e) None of these
some trade and commerce accompanied at times by sharing of
knowledge. It was not uncommon to have from time to time small 133. Development of villages became easy because
or big wars and mass exodus of people for greener pastures. By (a) people gave up agriculture and took to alternate jobs.
1000 BC, with considerable intermingling of races, several well- (b) a sense of security led people to live in units like
developed and prosperous civilisations had come into existence. villages.
Their borders of course were not well defined but kept on shifting (c) people were fed up of nomadic lifestyle.
due to wars. Here we shall be concerned essentially with some of (d) people got easy lifestyle in such a condition
the fundamental contributions of the ancient Indian and Greek
(e) None of these
civilisations to human knowledge. One may like to know why
these two could make more important and enduring contributions 134. Which of the following is true in the context of the passage?
than other civilisations. The answer briefly is that by and large (a) Agriculture, trade and commerce marked the
both in India and Greece conditions for free, sober and intelligent development of the society.
thought were an integral part of the social structure. Furthermore, (b) Living together made people brave enough to kill the
the philosophers were highly respected and they could without a wild animals.
care (c) Man’s nomadic lifestyle gave birth to agriculture
Follow knowledge like a sinking star (d) Man took up to nomadic lifestyle nearly 6000 years
Beyond the utmost bound of human thought. from now.
To begin with, the philosophical developments in the two (e) None of these
civilisations started along similar lines, with attempts to answer 135. Which of the following is false in the context of the passage.
some basic questions like : Who am I? How was the world created?
(a) Concern for security was one of the reasons which
Is there life after death? And so on. This is not surprising since
made families live together.
the forefathers of the Indo-Aryans and the Greeks lived together
in Central Asia before mass migration. But in due course, around (b) Fertile lands supported agriculture in India, China,
500 BC, the effects of different environments and some intermixing Mesopotamia and Egypt.
with other races began to have their effect and change in emphasis (c) Ancient civilisations developed without having any
in the pursuit of knowledge was noticeable. The Greeks, because support from one another.
of their rugged surrounding and contacts with other seafaring (d) Trade and commerce supported the cause of
people in trade and commerce, had to be practical. They turned development.
more and more to the study and understanding of the ‘outer (e) None of these
world’. So successful were they in their endeavour and so basic 136. Why did the established civilisations have no fixed places
were their contributions that it is now quite generally accepted or properly demarcated borders?
that the entire modern intellectual education of the West, both
(a) because several civilisations established themselves
philosophical and scientific, originates from the Greeks. The Indo-
close to each other
Aryans, on the other hand, once they had settled down
comfortably in different parts of India did not have to face many (b) because of the changing nature of the civilisations
challenges from the outside world. Consequently, they turned (c) because of the merging of smaller civilisations into
more and more towards religion and the study of man’s ‘inner bigger ones
world’. The degree of success and the mastery they achieved is (d) because of wars and migration of people to different
best summed up by Schopenhauer, a German philosopher. areas
Commenting about Upanishidic knowledge, he remarked, (e) None of these
“Thinking was finished on the banks of river Ganges.”
y
o
u
rs
m
SBI PO Exam 2011 63

a
h
b
137. What led the Greeks to expand their knowledge of the ‘outer 145. (a) grey (b) deteriorating

o
world’? (c) dry (d) rustic

o
b
(a) their conquest of various countries (e) uneven

.w
(b) their access to the other countries of the world via sea 146. (a) continued (b) drastic

o
rd
route (c) multiple (d) unjustified

p
(c) their social recognition in foreign countries

re
(e) insignificant

s
(d) their constant fighting with other countries 147. (a) unique (b) disastrous

s
.c
(e) None of these (c) umpteen (d) extensive

o
138. The Indo-Aryans with a religious bent of mind turned to

m
(e) amazing
the study of man’s inner world because 148. (a) pollution (b) cultivation
(a) they were at peace in their home and having little (c) deployment (d) demand
interference from the outside world (e) erosion
(b) they never reflected aggression even if challenged by 149. (a) urgency (b) misconception
other countries
(c) activities (d) action
(c) the study of the outer world was assigned to the
(e) misdeed
Greeks.
150. (a) major (b) demaging
(d) they had no idea of the outer world.
(c) effective (d) dastinctive
(e) None of these
(e) concerned
139. Which of the following is most similar in meaning toward
‘Exodus’ as used in the passage? GENERAL AWARENESS
(a) departure (b) trial
(c) awareness (d) sensitivity 151. Which of the following books is written by British Nobel
Prize-winning author Doris Lessing?
(e) fleet
(a) Wolf hall (b) Sea of Poppies
140. Which of the following is most opposite in meaning to word
‘Rugged’ as used in the passage? (c) Silent Spring (d) The Grass is Singing
(a) smooth (b) straight (e) None of these
(c) furnished (d) polished 152. The ‘Type-66’ agreement between China and Pakistan is
related to the development of
(e) curved
(a) Nuclear reactors (b) Industrial units
DIRECTIONS (Qs. 141-150) : In the following passage there (c) Roads/Bridges (d) Oil refinary
are blanks, each of which has been numbered. These numbers
(e) None of these
are printed below the passage and against each five words are
153. Who is the chairman of the committee constituted by RBI
suggested, one of which fits the blank appropriately. Find out
to study issues and concerns in the Micro Finance
the appropriate words.
institutions (MFI) Sector.
There has been a 141 change in people’s 142 towards and (a) Y H Malegam (b) Vijay kelkar
awareness 143 the environment. People are more 144 about the
(c) C. Rangrajan (d) M. Damodaran
145 condition of their environment. Land in all societies is
(e) Pranab Sen
subjected to 146 usage, viz. crop production, forest cover,
grassland, urbanisation and industrialisation etc. During the last 154. Which of the following awards is given in the field of Science
& Technology?
five decades 147 developmental activities have taken place. Slope
failure and 148 of soil due to such 149 are 150 environmental (a) Kalinga Prize (b) Dhyanchand Award
hazards. (c) Arjun Award (d) Moortidevi Award
141. (a) special (b) tremendous (e) Shanti Swarup Bhatnagar Award
(c) extreme (d) developmental 155. Who among the following has won the title of Swiss Open
(e) supportive Grand Prix Gold held in March 2011?
142. (a) information (b) feasibility (a) Saina Nehwal (b) Ji Hyun Sung
(c) debate (d) participation (c) Shixian Wang (d) Eriko Hirose
(e) attitude (e) None of these
143. (a) protecting (b) covering 156. RBI and the government has decided to increase the
percentage of people connected to bank to 80%. At present
(c) towards (d) surrounding
how much percent of peoples have access to banking
(e) of facilities?
144. (a) concerned (b) biased (a) 30% (b) 38%
(c) casual (d) desperate (c) 40% (d) 52%
(e) confused (e) 60%
y
o
u
rs
m
64 SBI PO Exam 2011

a
h
b
157. The government and the RBI had set a target to cover 73,000 165. At present, who is Prime Minister of Nepal?

o
villages having population in excess of 2000 to provide (a) Girija Prasad Koirala (b) Ram Baram Yadav

o
b
access to banking services, by March 2012. The name given (c) Jhalanath Khanal (d) Madhav Kumar Nepal

.w
to this scheme is (e) None of these

o
rd
(a) Swabhimaan (b) Swavalamban 166. A device which make use of pen for drawing pictures or

p
(c) Saral (d) Connecting Banks writing information to interact with a computer is called

re
___.

s
(e) None of these

s
.c
158. Who among the following has been awarded with the (a) Printer (b) Scanner

o
Russia’s highest honour ‘The Order of St. Andrew’? (c) Touch-Pen writer (d) Plotter

m
(a) Sonia Gandhi (b) Angela Markel (e) Stylus
(c) Mikhail Gorbachev (d) Nicolas Sarkozy 167. RBI appointed a committee under the chairmanship of D.
(e) None of these Mohanty to look into
159. Which of the following agencies/institutions report asked (a) 2G spectrum scam
the income tax department to ensure that a single tax payer (b) Implementation of Base Rate
is not issued multiple PAN cards because gap between PAN (c) Bank's Saving Interest rate
holders and the number of returns filed was 617.1 lakh? (d) Inflation
(a) IRDA (e) None of these
(b) SEBI 168. A mouse and printer make use of ________.
(c) RBI (a) infrared technology (b) network resources
(d) Comptroller and Auditor General (c) device drivers (d) web protocols
(e) None of these (e) None of these
160. Mitalee Jagtap Paradhar has won the best actress award at 169. A service which provides user interface to access the
58th National Film Awards ceremony for her performance in available multimedia resources over a network is called ____.
which of the following movies? (a) UseNet (b) URL
(a) Baboo Band Baaja (b) Aadukalam (c) Web (d) Internet Relay Chat (IRC)
(c) Adaminde Makan Abu (d) Do Dooni Char (e) None of these
(e) None of these 170. To communicate with each other, computers make use of
161. Who among the followings has been appointed as a director __________over a network.
on board of Bank of America – the first non-American on (a) File Server (b) URL
the board of the one of the largest financial institutions in (c) Domain (d) Website
the world? (e) Protocols
(a) Mukesh Ambani (b) Ratan Tata 171. A ________ is when you turn the computer on from an off
(c) Azim Premji (d) K V Kamath position.
(e) Chanda Kocher (a) cold boot (b) warm boot
162. According to 2011 Census figures which of the following (c) floppy boot (d) bootfailure
states is most populus? (e) None of these
(a) Maharashtra (b) Uttar Pradesh 172. A RAM in a computer provides _________ of data.
(c) Bihar (d) West Bengal (a) permanent storage (b) volatile storage
(e) Madhya Pradesh (c) virtual storage (d) distributing storage
163. Which of the following schemes is restructured as National (e) None of these
Rural Livelihood Mission (NRLM), an ambitious scheme to 173. Unsolicited or undesired electronic messages which are
eradicate poverty in rural areas? designed to trick recipients into giving up personal and
(a) SHGs financial information is called ________ .
(b) Swarnajayanti Gram Swarojgar Yojna (a) web-bug (b) spyware
(c) Mid Day Meal (c) malware (d) trojan horse
(d) Integrated Rural Development Programme (e) spam
(e) None of these 174. A common tool used to limit access to certain material on
164. The six-unit nuclear power project is being set up at Jaitapur the World Wide Web or protecting the childs from
in Maharashtra by which of the following countries? questionable sites on the Internet is called __________.
(a) France (b) USA (a) webroot (b) spybot
(c) China (d) Japan (c) web utility (d) keylogger
(e) Russia (e) filter
y
o
u
rs
m
SBI PO Exam 2011 65

a
h
b
175. The administrator in a organization consider a 183. A prospect is a

o
microcomputer as (a) business goals

o
b
(a) a simple electronic machine (b) document for marketing

.w
(c) potential buyer of a product or service

o
(b) an important component of information system

rd
(c) a powerfull tool of productivity (d) day-to-day function

p
(e) None of these

re
(d) a calculating machine

s
184. Marketing of goods is required if

s
(e) None of these

.c
(a) when goods become stale
176. The disk-caching feature improves the

o
(b) demand>supply

m
(a) performance of hard disk
(c) supply> demand
(b) speed of processor (d) supply = demand
(c) performance of monitor (e) None of these
(d) performance of CD drive 185. Selling is _____.
(e) None of these (a) different from Marketing
177. What does the term POS stands for (b) a sub-function of marketing
(a) preparation for sale (c) same as Marketing
(b) position of sales (d) more than Marketing
(c) point of sale (e) All of these
(d) point of superiority 186. Good selling skills involves _____.
(a) patience (b) perseverance
(e) primary outlook of salesman
(c) empathy (d) knowledge
178. Customer Center also known as
(e) All of these
(a) collection centre
187. Consumer information sources are.
(b) relationship center (a) personal and commercial sources
(c) customer friendly center (b) public source
(d) Loyality center (c) experiential source
(e) None of these (d) All of the above
179. Cross-selling is defined as (e) only (a) and (b)
(a) sale of bank products and services to existing 188. A target market is ____ .
customers (a) entire country
(b) a time bound seasonal function (b) entire city
(c) sharpens the mind-set of sales persons (c) entire globe
(d) control the flow of information into buying centre (d) customers who need a specific product
(e) None of these (e) None of these
189. 'Boost' is a product of which company?
180. Which is not a proper type of direct marketing
(a) Cadbury (b) Vicco
(a) boosting sales
(c) Wipro (d) Dabur
(b) having a focussed approach
(c) better customer contacts (e) None of these
(d) Advertising 190. Oracle and i2 Technologies are connected with
(e) None of these (a) Computer hardware (b) Networking
181. Marketing size means (c) Education (d) Operation systems
(a) getting new clients (e) None of these
(b) interacting with strangers 191. With which one of the following activities are Golden Globe
Awards associated?
(c) Market survey
(a) Journalism (b) Social work
(d) Area allocation
(c) Peace initiatives (d) Films
(e) None of these
(e) None of these
182. Marketing of Banking is the responsibilites of
192. Which of the following schemes of the Govt. of India is
(a) sales staff only associated with the health sector?
(b) back office staff only (a) NRDP (b) IRDP
(c) front office sales staff only (c) ASHA (d) Bharat Nirman
(d) all employees (e) All of these
(e) None of these
y
o
u
rs
m
66 SBI PO Exam 2011

a
h
b
193. Which of the following is a sort of Poverty Eradication 197. What does SAPTA stand for:

o
Programme of the Govt. of India, specially designed for

o
(a) South Asian Preferential Trade Agreement

b
rural areas ?

.w
(b) South Asian Post Trade Agreement
(a) Self Help Group

o
(c) SAARC Preferential Trade Agreement

rd
(b) Antodaya Anna Yojana
(d) SAARC Post Trade Agreement

p
(c) Social Security for Agricultural and Unorganized

re
(e) None of these
Labour

s
198. Which of the following state has registered the maximum

s
(d) Enforcement of Minimum Wages

.c
growth in Gross State Domestic Product in the year?

o
(e) None of these

m
194. Which company was the first to launch mobile phone (a) Chhattisgarh (b) Kerala
operations in India? (c) Gujarat (d) Goa
(a) Modi Telstra (b) Reliance India (e) None of these
(c) Bharati (d) Tata Indicom 199. Which of the following banks launched India’s first food
(e) None of these and agriculture private equity fund?
195. Which from the following wireless networks has a range of (a) Robo Bank (b) Citibank
50 kilometres?
(c) HSBC (d) SBI
(a) Macintosh (b) Wi-Max
(e) None of these
(c) Wi-Fi (d) Blue Tooth
200. Which of the following banks has recently entered into a
(e) None of these
joint venture agreement with Insurance Australia Group
196. Who is the author of the book Patrons of the Poor : Caste
(IAG) for launching general insurance business in India?
Politics and Policy Making in India?
(a) Allahabad Bank (b) PNB
(a) Narayan Lakshman (b) Greg Lindsay
(c) Khushwant Singh (d) Salman Rushdie (c) SBI (d) Indian Bank
(e) None of the above (e) None of these
y
o
u
rs
m
SBI PO Exam 2011 67

a
h
b
o
o
b
.w
o
rd
1. (c) Meaningful Words Þ Cone ONCE

p
7. (b) G R I M R M O S

re
2. (c) U I D L I Q +1

s
s
+1

.c
U T S R Q

o
+1

m
+1
(3 - 4) : M 3m O
D U S K L Q P I
+1
5m 5m +1
+1
3m Q +1
P
Similarly,
2m
S T O P P J G B
N +1
+1
+1
North +1
NW NE
8. (a) R > P > Q > S > T
P scored the second highest marks.
West East
9. (c) M I R A C L E
A C E I L M R
SW SE +1 –1 –1 +1 –1 –1 –1
South
B B F J K L Q

3. (e) 4m 3m 5th from the right

10. (e)
5m 9 14 3 12 21 4 5
1 N C L U D E
3m
P
(11-15 ) :
2m
N support the other group ja pe la no

Points R, P and Q are in a straight line. the mission gains support ke ja zi la


4. (b) Points O is 7 metres towards North of Point N.
gains other than money fu no ho zi
5. (a) +1 +1
O ¾¾® G E ¾¾® N re qi fu ja
more support and money
–1 -1
D ¾¾® F B ¾¾® P
11. (b) The code for ‘group’ is ‘pe’.
Similarly. 12. (e) ‘zi’ stands for ‘gains’.
+1 -1 13. (a) more Þ re or qi
A ¾¾® K L ¾¾® D
than Þ ho
+1 +2 +3
T ¾ ¾® J ¾¾® N ¾ ¾® A ¾ ¾® G
+4 the Þ la
6. (d)
+2 +3 +4 +5 group Þ pe
E ¾¾® U ¾¾® K ¾ ¾® R ¾ ¾® H
14. (c) The code for ‘mission’ is ‘ke’.
y
o
u
rs
m
68 SBI PO Exam 2011

a
h
15. (a) money Þ fu

b
of students will still join the courses. It is mentioned in

o
more Þ re or ql the statement that the institute is reputed. Therefore,

o
b
The code for ‘matters’ may we can assume that students will still prefer the institute

.w
of repute despite hike in fees.
be ‘bu’.

o
26. (d) None of the assumptions is implicit in hte statement. It

rd
(16-22) :
is mentioned in the statement that many residents of

p
A

re
the locality, not all residents, decided not to allend the
B

s
E function. This does not constitute the strong reason

s
.c
for cancellation of function. Assumption II is a long

o
D drawn conclusion.

m
K
27. (a) Only assumption I is implicit in the statement. The
Government advised the state governments to ensure
C disbursement of relief package assuming that state
G governments are capable for evolving a mechanism
for disbusrsement of relief.
F H The use of term ‘all’ in the assumption II makes it invalid.
16. (d) F is third to the right of B. 28. (b) It is clear that statement (B) is the cause and statement
17. (d) F and G are immediate neighbours of H. (A) is its effect.
29. (b) It is clear that statement (B) is the cause and statement
18. (b) F is remain in unchanged.
(A) is its effect.
A
30. (a) It is clear that statement (A) is cause and statement (B)
K A
B is its effect. If there is adequate supply in the retail
E B market, the price would stabilise.
H
31. (b) It is clear that accidents occurred due to faulty
D automatic signaling system. So that BTS couse and A
K C
is the effect.
32. (d) It is clear that both the statement (A) and (B) are effects
C of independent causes.
G GD
33. (e) All plants are bottles.

F H
F E
19. (a) H is second to the right of C. All bottles are caps.
B is second to the right of E. A + A Þ A-type of Conclusion
“All plants are caps.”
G is second to the right of F.
This is Conclusion II.
20. (c) DC, DE Þ Immediate neighbours of D. All bottles are caps.
DF, DA Þ F is second to the right of D.
A is second to the right of D.
Therefore, ? = DH All caps are crowns.
H is third to the right of D. A + A Þ A-types of Conclusion
21. (b) G sits third to the left of A. “All bottles are crowns.”
22. (e) Immediate neighbours of C are D and F. Its Converse would be :
23. (e) Clearly both the assumptions are implicit in the “Same crowns are bottles”
statement. This is Conclusion I.
If there are sufficient number of passengers, the other 34. (d) Some hankerchiefs are calculators.
private airlines may also increase the frequency of daily
flights. The airline decided to increase the frequency
of daily flights assuming that there are adequate
passenger load. All calculators are papers.
24. (b) Only assumption II is implicit in the statement. If the I + A Þ I-type of Conclusion.
management of the organisation has organised picnic “Some hankerchiefs are papers.”
for its employees, it would earmark adequate funds for 35. (e) All tigers are zebra.
picnic. Again, it is not prudent to assume that the
management of the organisation would not welcome
the employees with their family at the picnic.
25. (a) Only assumption I is implicit in the statement. The All zebra are cows.
institute increased the fees assuming that good number A + A Þ A–type of Conclusion
y
o
u
rs
m
SBI PO Exam 2011 69

a
h
I + A Þ I–type of Conclusion

b
“All tigers are cows.”

o
This is Conclusion I. “Some cards are chairs.”

o
b
“All tigers are cows” can be shown as : Its Converse would be :

.w
“Some chairs are cards.”

o
rd
If some chairs are cards, then all

p
re
or Tigers, chairs being cards is a possibility.
Tigers

s
Cows 38. (d) From both the statements
ws

s
.c
Co

o
m
I II
Now, “All camels are cows” can be shown as
C>D
39. (e) From both the statements
cost of pen ® la ke ij
or Camels,
Camels Cow Cows pen stand ® ke hu
Therefore, stand ®hu
40. (d) Data insufficient
III IV 41. (b) From statement I
Combine the figures II and IV Mansi took leave on Tuesday.
Wednesday or Thursday
Tigers, From statement II
Cows, Mansi took leave on Friday
Camels 42. (b) From statement I
Mary has only one younger brother. She may have
Therefore, “All camels being tigers is a possibility.” elder brother.
36. (a) Some pencils are mobiles. From statement II
Mary has only one brother.
43. (e) From Problem Figure (1) to (2) the shaded block moves
All mobiles are grass. one sector in anticlockwise direction and one more
I + A Þ I–type of Conclusion block gets shaded. From Problem Figure (2) to (3) the
shaded blocks move two sectors in anticlockwise
“Some pencils are grass”.
direction and one more block gets shaded. Similar
Its Converse would be :
changes ooccur from Problem Figure (4) onwards.
“Some grasses are pencils”.
44. (d) From Problem Figure (1) to (2) one design moves
If some grasses are pencils, then all grasses being
diagonally after being rotated through 90°
pencils is a possibility.
anticlockwise, the square moves one-half step and
Some pencils are grass.
shaded part of it also moves in anticlockwise direction.
Similar changes occur from Problem Figure (3) to (4)
and from Problem Figure (5) to Answer Figure.
All grass is green. 45. (e) In the subsequent figures respectively two, three, one,
I + A Þ I–type of Conclusion two three curves are added in clockwise direction to
“Some pencils are green.” complete the leaflets.
All mobiles are grass. 46. (d) The following changes occur in the subsequent figures :
(1) to (2) (2) to (3),
(3) to (4) (4) to (5)
(5) to (6)
All grass is green.
A + A Þ I–type of Conclusion
“All mobiles are green”.
37. (a) Some cards are clips.
47. (d) After Problem Figure (3) the pattern is repeated in the
reverse order and the outer line segment moves to the
opposite side.
All clips are chairs.
y
o
u
rs
m
70 SBI PO Exam 2011

a
h
b
(48-50) : 58. (a) n (A È B È C) = n(A) + n(B) + n(C) - n(A Ç B)

o
and-A, band - B, land - C, hand - D, hind - E, lack - F, job - G

o
- n(A Ç C) - n(B Ç C) + n(A Ç B Ç C)

b
.w
Chart Þ 100 – 18 = 42 + 68 + 51 – 30 – 28 – 36 + x

o
Þ x = 15

rd
Inpu t A B C D E F G

p
59. (d) Perimeter = 4 ´ 160000 = 1600 m

re
Step I: E A B F C D G

s
Distance 1600 ´ 5 ´ 60

s
Step II: E B C G A F D Time= = = 96 min

.c
Speed 5000

o
Step III: E A F B D C G

m
x +1 2
Step IV: C B A G D F E 60. (b) = Þ 3x - 2y = 1
y+2 3
Step V: D C B F A G E
x+2 5
Step VI: D B A E C F G = Þ 7x - 5y = 1
y+3 7
Step VII: D C F B G A E
x 3
Step VIII: A B C E G F D or, 3x – 2y = 7x – 5y Þ 3y = 4x Þ =
y 4
Step IX: G A B F C E D 61. (e) 3 5 4
Step X: G B C D A F E
(3 or 4 or 5)
48. (d) 49. (a) 50. (b) 3 × 5 × 4 = 60
2 5 4 3
100
´ 876
219 400 (1 or 2)
51. (e) ´100 = ´ 100 » 64%
24 624 2 × 5 × 4 × 3 = 120
´1274
49 Total = 120 + 60 = 180
52. (b) 5x 5x
62. (d) + 600 = 1000 + (x - 4000)
53. (a) From the table it can be observed. 100 2
54. (a) In 2003, TIS total production Þ x = 12000

æ 110 ö 100
= 782 ç = 860.2 63. (a) Time taken by B = 10 ´ = 8 days
è 100 ÷ø 125
8 ´ 10 4
æ 90 ö Required days = = 4 days
SAI total production = 665 çè ÷ = 598.5 18 9
100 ø
10 40 4
= 10 ¸ (1 + 1.25) = = =4
æ 860.2 ö æ 598.5 ö 2.25 9 9
Total of A-type steel = çè ´ 14÷ + ç ´ 62÷
23 ø è 133 ø 64. (a)
= (523.6) + (279) = 802.6 44
65. (b) Circumference = ´ 35 = 220 cm
Total of B-type stee 7
æ 860.2 ö æ 598.5 ö 33000
=ç ´ 9÷ + ç ´ 71÷ = (336.6) + (319.5) Distance travelled in 1 minute = = 550 m
è 23 ø è 133 ø 60
= 656.1 550 ´ 100
Required ratio = 8026 : 6561 Required no. of revolutions = = 250
220
3 8 6 66. (b) Required answer
55. (a) Required ratio = ´ = or 6 : 7
4 7 7 40 15
= 720 ´ ´ = 43.2 million litres
100 100
1 2 3 4
67. (d) Total supply from Jhelum and underground water
56. (e)
1m 3m 5m (12 + 6) = 18
= 720 ´ = 129.6 million litres
Sum of first n odd no. = n 2 100
Consumption of Yamuna water for domestic purposes
Ans = (11 – 1)2 = 100
40 40
1000 25 = 720 ´ ´ = 115.2
57. (c) 100 ´ - 100 = Þ x = 960 gm 100 100
x 6 Consumption of Ganga water for agriculture purposes
y
o
u
rs
m
SBI PO Exam 2011 71

a
h
= 67 + 22 + 14 = 103

b
35 20

o
= 720 ´ ´ = 50.4 No. of students who obtained less than 60% marks in

o
100 100

b
Hindi = 19 + 59 + 47 = 125

.w
\ Required ratio
125 - 103

o
129.6 18 Reqd % = ´ 100 = 17.60%

rd
= = = 18 : 23 125

p
(115.2 + 50.4 =)165.6 23 79. (b) No. of students (of JNU) listening to Radio City

re
[3 × 3 × 2 × 2 × 2 is common.]

s
20 65

s
68-73 : = 120000 ´ ´ = 15600

.c
Following table can be made easily : 100 100

o
m
Total area = 37249 sq kilometres 80. (e) The no. of Indraprastha students listening to FM
State Density of population Approx Rainbow = 120000 × 13% × 48%
(in thousands) % area The no. of Jamia students listening to
FM Gold = 120000 × 18% × 52%
112200 \ reqd percentage
A = 18 (approx) 16.7
6230
120000 ´ 13% ´ 48%
83800 = ´ 100 = 66.66%
B = 33 (approx) 6.8 120000 ´ 18% ´ 52%
2540 81. (a) Indraprastha University
64900 82. (c) The no. of Indraprastha students listening to
C = 8 (approx) 21.8
8135 13 46
Red FM = 120000 ´ ´ = 12 ´ 598
572 1 100 100
D = ´ 100 = 7.69 (approx) 20 The no. of Jamia students listening to Red FM
7436 13
18 36
71100 = 120000 ´ ´ = 12 ´ 648
E = 14.5 (approx) 13 100 100
4893 \ Total students = (12 × 598) + (12 × 648)
286 1 = 12 × 1246 = 14952
F = ´ 100 = 7.69 (approx) 10 83. (b) Radio City
3718 13
84. (e) From statements I and II, since it is a right-angled
86000 triangle, area
G = 20 (approx) 11.5
4297 1
[Note : Density of population is the population per square Þ ´ b ´ h = 5b \ h = 10
kilometre.] 2
68. (b) From thae above (column 3) it can be answered. 85. (e) Combining I & II, we get speed of Dinesh = 10 km/hr
States A, C and D have more than 15 per cent of total 60
areas. \ to walk 1 km he takes = 6 minutes.
10
69. (c) States D and F are some population density. 86. (b) From statement II, we find the negative answer since the
70. (a) Required answer = (33 – 18 =) 15 thousand. number is not divisible by 3 then it can’t be divisible by 12.
71. (d) For state A, B, E and G, ie for 4 states. 87. (a)
72. (e) Required answer = 2540 : (6230 + 4297 =) 88. (d) Using both the statements, we can find separate
2540 1 expenditures of Rahim and Suresh but can’t find their
= = separate incomes.
10527 4.1
(89-92) :
73. (d) n(C) = 650, n(F) = 480, n(H) = 400
n(C Ç F) = 300, n(F Ç H) = 250, n(C Ç H) = 240
and n(C È F È H) = (100 - 5)% of 1000 = 950
Since, n(C È F È H) = n(C) + n(F) + n(H) – n(C Ç F)
– n(F Ç H) – n(C Ç H) + n(C Ç F Ç H)
Þ n(C Ç F Ç H) = 210

50
The radius of the spherical part of the bowl can’t be =6 F = 480
C
determined. Hence volume cannot be calculated.
320 90 140
74. (c) 60% of 150 = 90. It means those who obtained either 90
or more than 90 marks in the average of five subjects 210
30
will be declared as passed. Therefore, 40
the required number = 31 + 17 = 48
75. (b) Reqd no. = 52 + 41 + 13 = 106 120
H = 400
76. (e) 77. (d)
78. (a) No. of students who obtained more than or equal to
89. (b) 90. (d) 91. (c) 92. (b)
40% marks in Science
y
o
u
rs
m
72 SBI PO Exam 2011

a
h
b
93. (c) The given question gives the amounts of investment 100. (d) Total no. of rooms over the three-year period

o
of P, Q and R, which is P = `12000, Q = `9000 and = 600 + 320 + 250 + 400 + 520 + 450 + 500 = 3040

o
b
S = `21000. Statement (A) combined with the reqd average no. of rooms per crore of rupees

.w
question’s information will give us the ratio of their 3040

o
= » 1.4

rd
profits, i.e. 4 : 6 : 7. Now, combine this either with B or
2125

p
C, and profit earned by them can be determined.

re
101. (e) 102. (a)
Therefore, A and either B or C is sufficient.

s
103. (c) The ice fields are melting down and shrinking due to

s
94. (e) Let the present ages of mother and son be M and S yrs

.c
global warming, which is caused by emission of carbon

o
respectively.

m
dioxide in the atmosphere.
M 5 104. (b) It is necessary to check and control the cause of global
A. Þ = or, 3M – 5S = 50
S + 10 3 warming.
B. Þ 3M – 5S = 50 105. (d) 106. (b) 107. (a)
108. (c) It will be catastrophic for low-lying island states and
M + 10 5
C. Þ = or, 3M – 5S = 50 countries with large coastal populations.
S + 16 3
109. (c) 110. (d) 111. (a) 112. (b)
Here, all the three equations are the same. Therefore 113. (d) 114. (a) 115. (e) 116. (d)
option (e) is the correct choice.
117. (e) 118. (a) 119. (b) 120. (c)
95. (c) Statement (A) gives the cost of fencing one metre of
121. (a) 122. (c) 123. (b) 124. (d)
the circular plot. Combining this with statement (B),
125. (e)
total cost of fencing can be determined.
(126-130) : DAFCBE
96. (d) A. Þ Market price of the article
126. (d) 127. (e) 128. (d) 129. (b)
100 130. (a)
= ´ 810 =` 900
90 131. (b) Man gave up his nomadic type of life after he took to
B. Þ CP of the article (with the help of A) agriculture.
132. (e) plenty of fertile land and water
900 ´ 100
= =` 600 133. (e) There was boost in agriculture production which
150
helped generate other jobs useful to the society.
Now, combining (A) with (C), 134. (a) 135. (c) 136. (d) 137. (b)
SP = 900 × 0.95 = ` 855 138. (a) 139. (a) 140. (a) 141. (b)
100 142. (e) 143. (e) 144. (a) 145. (b)
CP = 855 ´ =`600
142.50 146. (c) 147. (d) 148. (e) 149. (c)
Hence, A and either B or C are sufficient. 150. (a) 151. (d) 152. (a) 153. (a)
97. (a) Let the rate of interest be r%. 154. (e) 155. (a) 156. (c) 157. (a)
158. (c) 159. (d) 160. (a) 161. (a)
éæ r ö
2 ù 5150 ´ r ´ 3 162. (b) 163. (b) 164. (a) 165. (c)
A. Þ 2500 êçè1 + ÷ø - 1ú = 166. (e) 167. (b) 168. (c) 169. (c)
ë 100 û 100
170. (e)
12000 ´ r ´ 3 10000r ´ 5 171. (a) A warm boot is when you reset a computer that is
B. Þ + = 5160
100 100 already on.
172. (b)
1725 - 1500
C. Þ r = ´ 100 = 5% 173. (e) Actual name given to such message is 'phishing' but
3 ´ 150
phishing is a part/category of spam.
Hence, any one of them is sufficient. 174. (e) 175. (c) 176. (a) 177. (c)
98. (a) Only for project A, the value of cost per room is less 178. (b) 179. (a) 180. (d)
than half crore, whereas in other projects it is either
181. (c) Market penetration means the depth of sales of a
more than or equal to half crore.
particular product in a given market. Marketing Size
99. (b) Total investments for all the projects
involves the number of sellers and buyers in the market.
= 275 + 210 + 250 + 430 + 310 + 400 + 250 = ` 2125 cr
182. (a) 183. (c) 184. (c) 185. (b)
Investment on those projects which are completed in
186. (e) 187. (e) 188. (d) 189. (a)
1998 = 275 + 430 + 400 = `1105 cr
190. (b) 191. (d) 192. (c) 193. (b)
1105 194. (a) 195. (b) 196. (b) 197. (a)
\ reqd % = ´ 100 = 52%
2125 198. (a) 199. (a) 200. (c)
y
o
u
rs
m
a
h
b
o
SBI & ITS ASSOCIATES PO

o
b
.w
o
EXAM 2011 Based on Memory

rd
p
re
s
s
.c
REASONING (High Level)

o
DIRECTIONS (Qs. 7-12): Study the following information to

m
answer the given questions:
DIRECTIONS (Qs.1-3): Study the following information
carefully and answer the given questions. Twelve people are sitting in two parallel rows containing six
people each such that they are equidistant from each other. In
Four of the following five are alike in a certain way and
row 1: P, Q, R, S, T and V are seated and all of them are facing
hence form a group. Which is the one that does not belong to the
South. In row 2: A, B, C, D, E and F are seated and all of them are
group?
facing North. Therefore, in the given seating arrangement, each
1. (a) Break (b) Change
member seated in a row faces another member of the other row.
(c) Split (d) Divide
S sits third to the right of Q. Either S or Q sits at an extreme
(e) Separate
end of the line. The one who faces Q sits second to the right of E.
2. (a) Train (b) Instruct
Two people sit between B and F. Neither B nor F sits at an extreme
(c) Educate (d) Advocate end of the line. The immediate neighbour of B faces the person
(e) Coach who sits third to the left of P. R and T are immediate neighbours.
3. (a) Extend (b) Higher C sits second to the left of A. T does not face the immediate
(c) Upward (d) Rise neighbour of D.
(e) Ascend 7. Who amongst the following sit at the extreme ends of the
4. Which of the following symbols should replace question rows?
mark (?) in the given expression in order to make the (a) S, D (b) Q, A
expressions ‘ A > D’ and ‘F ³ C’ definitely true? (c) V, C (d) P, D
A > B ³ C ? D £ E=F (e) Q, F
(a) > (b) < 8. Who amongst the following faces S?
(c) £ (d) = (a) A (b) B
(e) Either = or ³ (c) C (d) D
5. Which of the following expressions is definitely true if the
(e) F
given expressions ‘R < P’ as well as ‘S > Q’ are definitely
9. How many persons are seated between V and R?
true?
(a) One (b) Two
(a) P > Q = R £ T < S (b) S > T ³ R > Q < P
(c) Three (d) Four
(c) Q > R £ T > P ³ S (d) S > T ³ R > Q > P
(e) None of these
(e) None of these
10. P is related to A in the same way as S is related to B based
6. Read the following information carefully and answer the
on the given arrangement. Which of the following is T
question which follows:
related to, following the same pattern?
‘A × B’ means ‘A is the father of B’.
(a) C (b) D
‘A + B’ means ‘A is the daughter of B’.
(c) E (d) F
‘A ÷ B’ means ‘A is the son of B’.
(e) Cannot be determined
‘A – B’ means ‘A is the sister of B’.
11. Which of the following is true regarding T?
What will come in place of question mark to establish that P
is the son-in-law of S in the following expression? (a) F faces T.
P×Q+R–T ?S (b) V is an immediate neighbour of T.
(a) + (b) × (c) F faces the one who is second to the right of T.
(c) – (d) ÷ (d) T sits at one of the extreme ends of the line.
(e) Either + or ÷ (e) Q sits second to the right of T.
y
o
u
rs
m
74 SBI & its Associates PO Exam 2011

a
h
b
12. Four of the following five are alike in a certain way based on 17. Is F granddaughter of B?

o
the given arrangement and so form a group. Which is the

o
I. B is the father of M. M is the sister of T. T is the

b
one that does not belong to that group? mother of F.

.w
(a) A-T (b) B-T II. S is the son of F. V is the daughter of F. R is the brother

o
rd
(c) F-P (d) C-V of T.

p
(e) E-Q

re
DIRECTIONS (Qs. 18-20): Study the following information to

s
DIRECTIONS (Qs. 13-17): Each of the questions below consists answer the given questions:

s
.c
of a question and two statements numbered I and II given below
A word and number arrangement machine when given an

o
it. You have to decide whether the data provided in the statements

m
input line of words and numbers rearranges them following a
are sufficient to answer the question. Read both the statements
particular rule in each step. The following is an illustration of
and give answer.
input and rearrangement. (All the numbers are two-digit numbers
(a) if the data in statement I alone are sufficient to answer and are arranged as per some logic based on the value of the
the question, while the data in statement II alone are number.)
not sufficient to answer the question. Input : win 56 32 93 bat for 46 him 2811 give chance
(b) if the data in statement II alone are sufficient to answer Step I : 93 56 32 bat for 46 him 28 11 give chance win
the question, while the data in statement I alone are Step II : 11 93 56 32 bat for 46 28 give chance win him
not sufficient to answer the question. Step III : 56 11 93 32 bat for 46 28 chance win him give
(c) if the data either in statement I alone or in statement II Step IV : 28 56 11 93 32 bat 46 chance win him give for
alone are sufficient to answer the question. Step V : 46 28 56 11 93 32 bat win him give for chance
(d) if the data even in both statements I and II together are Step VI : 32 46 28 56 11 93 win him give for chance bat
not sufficient to answer the question.
Step VI is the last step of the arrangement the above input.
(e) if the data in both statements I and II together are
As per the rules followed in the above steps, find out in
necessary to answer the question.
each of the following questions the appropriate steps for the
13. Which bag amongst P, Q, R, S and T is the heaviest?
given input.
I. Bag Q is heavier than R and S. Bag T is heavier than
Input for the questions:
only bag P.
Input: fun 89 at the 28 16 base camp 35 53 here 68 (All the
II. Only three bags are lighter than R. The weight of bag
numbers given in the arrangement are two-digit numbers.)
Q is 50 kg, which is 2 kg more than bag R.
18. Which of the following would be step II?
14. Are all the five friends - A, B, C, D and E - who are seated
around a circular table facing the centre? (a) 89 fun at 28 16 base camp 35 53 here 68 the
I. A sits third to the left of B. B faces the centre. D and E (b) 35 53 28 68 16 89 the here fun camp base at
are immediate neighbours of each other. C sits second (c) 16 89 at fun 28 camp base 35 53 68 the here
to the right of E. (d) 53 28 68 16 89 35 the here fun camp base at
II. D sits second to right of C. C faces the centre. Both E (e) None of these
and A are immediate neighbours of D. B sits second to 19. Which word/number would be at seventh position from the
the right of A. left in step IV?
15. In a college, five different subjects, viz Physics, Chemistry, (a) base (b) at
Botany, Zoology and Mathematics, are taught on five (c) 35 (d) the
differeIlt days of the same week, starting from Monday and (e) 53
ending on Friday. Is Chemistry taught on Wednesday? 20. Which step number would be the following output?
I. Two subjects are taught between Zoology and 53 28 68 16 89 at 35 the here fun camp base
Mathematics. Mathematics is taught before Zoology.
(a) There will be no such step.
Chemistry is taught on the day immediately next to the
(b) III
day when Physics is taught. Botany is not taught on
Friday. (c) II
II. Three lectures are scheduled between the lectures of (d) V
Botany and Zoology. Mathematics is taught (e) IV
immediately before Physics. DIRECTIONS (Qs. 21-27): Study the following information
16. Is it 9 o'clock now? carefully and answer the given questions.
I. After half an hour, the minute and the hour hands of Eight colleagues, A, B, C, D, E, F, G and H, are sitting around
the clock will make an angle of exactly 90° with each a circular table facing the centre but not necessarily in the same
other.
order. Each one of them holds a different post–Manager, Company
II. Exactly 15 minutes ago, the hour and the minute hands Secretary, Chairman, President, Vice President, Group Leader,
of the clock coincided with each other. Financial Advisor and Managing Director.
y
o
u
rs
m
SBI & its Associates PO Exam 2011 75

a
h
b
A sits third to the right of the Managing Director. Only two DIRECTIONS (Qs. 28-31): Read the following information and

o
people sit between the Managing Director and H. The Vice

o
the sentences (A), (B), (C), (D) and (E) given below it carefully

b
President and the Company Secretary are immediate neighbours.

.w
and answer the questions which follow:
Neither A nor H is a Vice President or a Company Secretary. The

o
A host of foreign companies are in talks with the Indian

rd
Vice President is not an immediate neighbour of the Managing
government for selling B 150, a tough, short-haul plane ideal for

p
Director. The Manager sits second to the left of E. E is not an

re
connectivity of smaller towns which is lacking in India at present.
immediate neighbour of H. The Manager is an immediate

s
s
(A) B 150 planes have not only low operating costs than

.c
neighbour of both the Group Leader and the Financial Advisor.
competing planes like Cezana but also a much better

o
The Financial Advisor sits third to the -right of B. B is not the Vice

m
track record in terms of safety and efficiency.
President. C sits on the immediate right of the Chairman. A is not (B) The profit margin of road transport operators in the
the Chairman. F is not an immediate neighbour of A. G is not an smaller towns connected by B 150 planes has been
immediate neighbour of the Manager. reduced substantially as a majority of people prefer air
21. Who amongst the following sits third to the left of E? transport over other means of transport.
(a) Manager (b) G (C) Smaller towns, at present, are better connected by roads
(c) A (d) Financial Advisor and railways as compared to flight services.
(e) B (D) B 150 planes are capable of operating in sectors where
large airlines cannot fly due to challenging conditions
22. Four of the following five are alike in a certain way based on
such as mist, short runways, etc. Such planes can also
the given arrangement and thus form a group. Which is the
double up as cargo planes and charter flights for the
one that does not belong to that group?
rich and the elite.
(a) F-Chairman (b) G-President (E) B 150 planes need to operate in the existing airports
(c) D-Manager (d) A-Financial Advisor which are situated in bigger cities only and are poorly
(e) Managing Director connected to the smaller cities.
23. Who among the following is the President of the company? 28. Which of the statements (A), (B), (C), (D) and (E) can be
inferred from the facts/information given in the statement?
(a) A (b) C
(An inference is something which is not directly stated but
(c) H (d) G can be inferred from the given facts.)
(e) D (a) Only A (b) Only B
24. Which of the following is true with respect to the given (c) Only C (d) Both B and D
seating arrangement? (e) Only E
(a) The Group Leader of the company is an immediate 29. Which of the statements (A), (B), (C), (D) and (E) mentioned
neighbour of the Vice President. above would weaken the offer made by the foreign
(b) G sits second to the right of D. companies for selling B 150 planes to Indian government?
(a) A (b) B
(c) The Group Leader and the Company Secretary are
(c) C (d) D
immediate neighbours.
(e) E
(d) The Chairman of the company sits to the immediate
30. Which of the statements (A), (B), (C), (D) and (E) mentioned
left of the Managing Director. above represents a possible consequence of the success
(e) The Group Leader sits second to the left of D. of B 150 planes in smaller cities?
25. Which of the following posts does B hold in the company? (a) A (b) B
(a) Chairman (b) Manager (c) C (d) D
(c) Company Secretary (d) Vice President (e) E
(e) Financial Advisor 31. Which of the statements (A), (B), (C), (D) and (E) would
favour the foreign companies’ bid to sell B 150 planes in
26. Who among the following sits exactly between the
India?
Managing Director and H? (a) Only A (b) Only B
(a) H and the Chairman (b) B and G (c) Both B and C (d) Both A and D
(c) The Chairman and C (d) F and C (e) Both E and C
(e) E and the Group Leader DIRECTIONS (Qs. 32-37): Study the following information to
27. Who among the following is the Group Leader? answer the given questions:
(a) C (b) F In a certain code, ‘always create new ideas’ is written as ‘ba
(c) G (d) H ri sha gi’, ‘ideas and new thoughts’ is written as ‘fa gi ma ri’,
(e) A ‘create thoughts and insights’ is written as ‘ma jo ba fa’, and
‘new and better solutions’ is written as ‘ki ri to fa’.
y
o
u
rs
m
76 SBI & its Associates PO Exam 2011

a
h
b
32. What is the code for ‘ideas’? (a) House owners too have demanded for laws that make

o
it easier to evict tenants who default on rent payment.

o
(a) sha (b) ba

b
(b) Such a tax law would be difficult to implement as it

.w
(c) gi (d) ma
would be difficult to record the number of vacant flats

o
(e) Cannot be determined

rd
33. What does 'fa' stand for? in the city.

p
(c) People with surplus money buy many houses and rent

re
(a) thoughts (b) insights
these out while many consumers cannot afford even

s
(c) new (d) and

s
their first house.

.c
(e) solutions

o
(d) The number of vacant flats in Mumbai is much lower

m
34. ‘fa lo ba’ could be a code for which of the following? than other metros such as Delhi and Chennai where
(a) thoughts and action (b) create and innovate vacancy tax already exists.
(c) ideas and thoughts (d) create new solutions (e) Such a tax would compel the house owners to sell!
(e) always better ideas rent properties which in turn would control hoarding
35. What is the code for ‘new’? and skyrocketing property prices.
(a) ki (b) ri DIRECTIONS (Qs. 40-45): In each question below are given
(c) to (d) fa two/three statements followed by two conclusions numbered I
(e) ba and II. You have to take the given statements to be true even if
36. Which of the following may represent ‘insights always they seem to be at variance with commonly known facts. Read
better’? all the conclusions and then decide which of the given
conclusions logically follows from the given statements,
(a) jo ki to (b) ki to ri
disregarding commonly known facts. Give answer
(c) sha jo ri (d) to sha jo
(a) if only conclusion I follows.
(e) sha to ba
(b) if only conclusion II follows.
37. What is the code for ‘throughts’? (c) if either conclusion I or conclusion II follows.
(a) ma (b) fa (d) if neither conclusion I nor conclusion II follows.
(c) ba (d) jo (e) if both conclusion I and conclusion II follow.
(e) Either jo or fa 40. Statements : All rings are circles.
DIRECTIONS (Q. 38): Read the following information carefully All squares are rings.
and answer the question which follows. No ellipse is a circle.
38. All existing and upcoming hotels within a 5 km radius of Conclusions : I. Some, rings being ellipses is a possibility.
national parks and sanctuaries in India will have to pay : II. At least some circles are squares.
30% of their annual turnover as tax to the government. 41. Statements : No house is an apartment.
Which of the following statements can be inferred from the Some bungalows are apartments.
facts/information given in the above statement? Conclusions : I. No house is a bungalow.
(a) The tax collected from the hotels will be used for the II. All bungalows are houses.
betterment of these national parks and sanctuaries. 42. Statements : Some gases are liquids.
(b) Hotels which are sponsored by the government will All liquids are water.
not have to pay any tax even if these are located within Conclusions : I. All gases being water is a possibility.
the 5 km radius of such wildlife hotspots. II. All such gases which are not water can
(c) The ecosystem of the national parks and sanctuaries never be liquids.
is adversely affected even if the hotels are located 43. Statements : All minutes are seconds.
outside the 5 km radius. All seconds are hours.
(d) Government allows the construction of hotels within No second is a day.
5km radius of national parks and sanctuaries. Conclusions : I. No day is an hour.
(e) Such a step is taken by the environment ministry to II. At least some hours are minutes.
boost eco-tourism and perk up revenue collection of (44-45): Statements : Some teachers are professors.
State governments. Some lecturers are teachers.
44. Statements : I. All teachers as well as professors being
DIRECTIONS (Q. 39):Read the following paragraph and answer lecturers is a possibility.
the question which follows. II. All those teachers who are lecturers are
39. Tenants’ associations have demanded a ‘vacancy tax’ on also professors.
all vacant and unsold flats in Mumbai. 45. Statements : I. No professor is a lecturer.
Which of the following would support the demand made II. All lecturers being professors is a
by the tenants' association? possibility.
y
o
u
rs
m
SBI & its Associates PO Exam 2011 77

a
h
b
DIRECTIONS (Qs. 46-50): In each of the questions given below which one of the five Answer Figures on the right should come after

o
o
the Problem Figures on the left, if the sequence were continued?

b
.w
PROBLEM FIGURES ANSWER FIGURES

o
rd
46.

p
S C O B E B S O L O O L B

re
E E B K E S K O S L S S K

s
s
C S K L C S L O C E E E L C

.c
O B K L L O B K C E S O L B K C E L C B K C B K C B K E S O

o
m
1 2 3 4 5 (a) (b) (c) (d) (e)

47.

1 2 3 4 5 (a) (b) (c) (d) (e)

48.

1 2 3 4 5 (a) (b) (c) (d) (e)

49. o o

c
c
o c c o c o o o
c

c o
o o c
c

1 2 3 4 5 (a) (b) (c) (d) (e)

50. E L S S S
L L
E E H H L L
=HOLE H K C C L H L L
O H L C H
K H S S H

1 2 3 4 5 (a) (b) (c) (d) (e)

QUANTITATIVE APTITUDE 53. 25.05 × 123.95 + 388.999 × 15.001 =?


(a) 900 (b) 8950
DIRECTIONS (Qs. 51-55): What will come in place of question (c) 8935 (d) 8975
mark (?) in the following questions?
(e) 8995
51. 32.05% of 259.99 =? 54. 561 ÷ 35.05 × 19.99 = ?
(a) 92 (b) 88 (a) 320 (b) 330
(c) 78 (d) 90
(c) 315 (d) 325
(e) 83
(e) 335
1 2 3
52. of of of 1715 = ? 55. (15.01)2 × 730 = ?
8 3 5
(a) 80 (b) 85 (a) 6125 (b) 6225

(c) 90 (d) 95 (c) 6200 (d) 6075


(e) 75 (e) 6250
y
o
u
rs
m
78 SBI & its Associates PO Exam 2011

a
h
b
DIRECTIONS (Qs. 56-60): In each of these questions, a number 65. Inside a square plot, a circular garden is developed which

o
o
series is given. In each series, only one number is wrong. Find exactly fits in the square plot and the diameter of the garden

b
is equal to the side of the square plot which is 28 metres.

.w
out the wrong number.
What is the area of the space left out in the square plot after

o
56. 3601 3602 1803 604 154 36 12

rd
developing the garden?

p
(a) 3602 (b) 1803

re
(a) 98 m2 (b) 146 m2
(c) 604 (d) 154

s
84 m2 (d) 168 m2

s
(c)

.c
(e) 36

o
(e) None of these
57. 4 12 42 196 1005 6066 42511

m
(a) 12 (b) 42 66. Amit and Sujit together can complete an assignment of data
(c) 1005 (d) 196 entry in five days. Sujit’s speed is 80% of Amit’s speed and
(e) 6066 the total key depressions in the assignment are 5,76,000.
58. 2 8 12 20 30 42 56 What is Amit’s speed in key depressions per hour if they
(a) 8 (b) 42 work for 8 hours a day?
(c) 30 (d) 20 (a) 4800 (b) 6400
(e) 12 (c) 8000 (d) 7200
59. 32 16 24 65 210 945 5197.5 (e) None of these
(a) 945 (b) 16 67. Out of five girls and three boys, four children are to be
(c) 24 (d) 210 randomly selected for a quiz contest. What is the probability
(e) 65 that all the selected children are girls?
60. 7 13 25 49 97 194 385
(a) 13 (b) 49 1 1
(a) (b)
(c) 97 (d) 194 14 7
(e) 25
5 2
61. Mr X invested a certain amount in Debt and Equity Funds (c) (d)
in the ratio of 4 : 5. At the end of one year, he earned a total 17 17
dividend of 30% on his investment. After one year, he (e) None of these
reinvested the amount including the dividend in the ratio of
6 : 7 in Debt and Equity Funds. If the amount reinvested in 68. Profit earned by an organisation is distributed among
Equity Funds was `94,500, what was the original amount officers and clerks in the ratio of 5 : 3. If the number of
invested in Equity Funds? officers is 45 and the number of clerks is 80 and the amount
(a) `75,000 (b) `81,000 received by each officer is `25,000, what was the total
(c) `60,000 (d) `65,000 amount of profit earned?
(e) None of these (a) ` 22 lakh (b) ` 18.25 lakh
62. The age of the father is 30 years more than the son’s age. (c) ` 18 lakh (d) ` 23.25 lakh
Ten years hence, the father’s age will become three times
(e) None of these
the son’s age that time. What is the son’s present age in
years? 69. A shopkeeper labelled the price of his articles so as to earn
(a) Eight (b) Seven a profit of 30% on the cost price. He,then sold the articles
(c) Five (d) Cannot be determined by offering a discount of 10% on the labelled price. What is
(e) None of these the actual per cent profit earned in the
63. If the length of a rectangular field is increased by 20% and deal?
the breadth is reduced by 20%, the area of the rectangle will (a) 18% (b) 15%
be 192 m2. What is the area of the original rectangle?
(c) 20% (d) Cannot be determined
(a) 184 m2 (b) 196 m2
(c) 204 m 2 (d) 225 m2 (e) None of these
(e) None of these 70. Mr Shamin’s salary increases every year by 10% in June. If
64. The product of one-third of a number and 150% of another there is no other increase or reduction in the salary and his
number is what per cent of the product of the original salary in June 2011 was `22,385, what was his salary in June
numbers? 2009?
(a) 80% (b) 50% (a) `18,650 (b) `18,000
(c) 75% (d) 120% (c) `19,250 (d) `18,500
(e) None of these (e) None of these
y
o
u
rs
m
SBI & its Associates PO Exam 2011 79

a
h
b
DIRECTIONS (Qs. 71-75): In each of these questions, one 75. What is the cost of flooring a rectangular hall?

o
o
question is given followed by data in three statements I, II and Statements :

b
.w
III. You have to study the question and the data in statements I. The length of the rectangle is 6 metres.

o
and decide the question can be answered with data in which of II. The breadth of the rectangle is two-thirds of its length.

rd
the statements and mark your answer accordingly. III. The cost of flooring the area of 100 cm2 is `45.

p
re
71. What is the rate of interest pcpa? (a) Only I and III

s
s
Statements: (b) Only II and III

.c
o
I. The difference between the compound interest and (c) All I, II and III

m
simple interest earned in two years on the amount (d) Question cannot be answered even with data in all
invested is `100. three statements.
II. The amount becomes `19,500 in three years on simple (e) None of these
interest. DIRCCTIONS (Qs. 76-83): Study the following graph carefully
III. The simple interest accrued in two years on the same to answer these questions.
amount at the same rate of interest is `3,000.
Per cent profit earned by two companies producing
(a) Only I and II (b) Only I and III
electronic goods over the years
(c) Only II and III (d) Only I and either II or III
(e) None of these Profit Earned
% Profit = ´ 100
72. What is the speed of the train in kmph? Total Investment
Statements : Profit Earned = Total Income – Total Investment in the year
I. The train crosses an ‘x’ metre-long platform in ‘n’
seconds.
100
II. The length of the train is ‘y’ metres. 90 90 90
Per cent Profit Earned

III. The train crosses a signal pole in 'm' seconds. 80 80 80


(a) Any two of the three (b) Only II and III 70 70
65 75
(c) Only I and III (d) All I, II and III 60 55 70
50 60
(e) Either I and II or II and III 55 55
40 50
45
73. How many students passed in first class?
30
Statements : 20
I. 85% of the students who appeared in examination have 10
passed either in first class or in second class or in pass 0
class. 2004 2005 2006 2007 2008 2009 2010
II. 750 students have passed in second class.
Year
III. The number of students who passed in pass class is
76. If the profit earned in 2006 by Company B was `8,12,500,
28% of those passed in second class.
what was the total income of the company in that year?
(a) All I, II and III
(a) `12,50,000 (b) `20,62,500
(b) Only I and III
(c) `16,50,000 (d) `18,25,000
(c) Only II and III
(e) None of these
(d) Question cannot be answered even with information
77. If the amount invested by the two companies in 2005 was
in all three statements.
equal, what was the ratio of the total income of the Company
(e) None of these A to that of B in 2005?
74. What is the amount invested in Scheme ‘B’? (a) 31 : 33 (b) 33 : 31
Statements : (c) 34 : 31 (d) 14 : 11
I. The amounts invested in Schemes ‘A’ and ‘B’ are in (e) None of these
the ratio of 2 : 3.
78. If the total amount invested by the two companies in 2009
II. The amount invested in Scheme ‘A’ is 40% of the total was `27 lakh, while the amount invested by Company B
amount invested. was 50% of the amount invested by Company A, what was
III. The amount invested in Scheme ‘A’ is `45,000. the total profit earned by the two companies together?
(a) Only I and II (b) Only I and III (a) `21.15 lakh (b) `20.70 lakh
(c) Only II and III (d) All I, II and III (c) `18.70 lakh (d) `20.15 lakh
(e) Only III and either I or II (e) None of these
y
o
u
rs
m
80 SBI & its Associates PO Exam 2011

a
h
b
79. If the income of Company A in 2007 and that in 2008 were 85. What is the average number of candidates (approximately)

o
equal and the amount invested in 2007 was `12 lakh, what found eligible for Post III from all states?

o
b
was the amount invested in 2008? (a) 6700 (b) 6200

.w
(a) `10,87,500 (b) `10,85,700 (c) 4200 (d) 4500

o
rd
(c) `12,45,000 (d) `12,85,000 (e) 5500

p
(e) None of these

re
86. What is the overall percentage (rounded off to one digit

s
80. If the amount of profit earned by Company A in 2006 was after decimal) of candidates short listed over the total number

s
.c
`10. 15 lakh, what was the total investment? of candidates eligible for Post I from all the States together?

o
(a) `13.8 lakh (b) `14.9 lakh (a) 9.5% (b) 12.5%

m
(c) `15.4 lakh (d) `14.2 lakh (c) 7.2% (d) 6.5%
(e) None of these (e) None of these
81. If the amount invested by Company B in 2004 is `12 lakh 87. What is the ratio of the total number of candidates shortlisted
and the income of 2004 is equal to the investment in 2005, for all the posts together from State E to that from state G?
what is the amount of profit earned in 2005 by Company B? (a) 307 : 369 (b) 73 : 79
(a) `6.6 lakh (b) `18.6 lakh (c) 6 : 5 (d) 9 : 7
(c) `10.23 lakh (d) `9.6 lakh (e) None of these
(e) None of these 88. The total number of candidates found eligible for Post I
82. If the investments of Company A in 2007 and 2008 were from all states together is approximately what per cent of
equal, what is the difference between the profits earned in the total number of candidates found eligible for Post VI
from all States together?
the two years if the income in 2008 was `24 lakh?
(a) 45% (b) 50%
(a) `2.25 lakh (b) `3.6 lakh
(c) 60% (d) 55%
(c) `1.8 lakh (d) `2.6 lakh
(e) 65%
(e) None of these
89. Which state had the lowest percentage of candidates short
83. If each of the companies A and B invested `25 lakh in 2010, listed with respect to candidate eligible for Post IV?
what was the average profit earned by the two companies? (a) G (b) F
(a) `18 lakh (b) `22.5 lakh (c) E (d) C
(c) `17.5 lakh (d) `20 lakh (e) None of these .
(e) None of these 90. What is the ratio of the total number of candidates short
DIRECTIONS (Qs. 84-90): Study the following table carefully listed for post V to that for post VI from all states together?
and answer the questions which follow. (a) 6 : 7 (b) 55 : 96
(c) 165 : 278 (d) 16 : 25
Number of Candidates found Eligible and the
(e) None of these
Number of Candidates Short listed for Interview for a
recent Recruitment Process for Six Posts from DIRECTIONS (Qs. 91-95) : These questions are based on the
following data. Study it carefully and answer the questions that
Different States follow.
E-Eligible S- Short listed
In a school having 400 students, boys and girls are in the
ratio of 3 : 5. The students speak Hindi, English or both the
Post 1 II III IV V VI
languages. 12% of the boys speak only Hindi. 22% of the girls
S t a te E S E S E S E S E S E S speak only English. 24% of the total students speak only Hindi
and the number of boys speaking both the languages is six times
A 2500 65 7200 240 5200 76 3600 200 4600 110 5400 380 the number of boys speaking only Hindi.
B 3200 220 8500 420 8400 190 6200 320 5800 180 6200 430 91. How many boys speak Hindi?
C 2800 280 4500 350 7600 160 8200 440 7300 310 3700 250 (a) 18 (b) 126
D 2400 85 4800 200 2600 55 7500 350 3900 160 4800 360 (c) 108 (d) 26
E 3000 120 5600 280 3800 75 6800 280 6100 260 7800 520 (e) None of these
F 4800 325 6400 320 4400 220 4700 180 4900 220 8800 640
92. How many girls speak only Hindi?
(a) 55 (b) 117
G 6500 550 7000 140 6000 325 5500 220 8100 410 2700 200
(c) 96 (d) 78
84. From State B, which post had the highest percentage of (e) None of these
candidates short listed? 93. How many students speak English?
(a) V (b) IV (a) 304 (b) 79
(c) VI (d) II (c) 225 (d) 117
(e) None of these (e) None of these
y
o
u
rs
m
SBI & its Associates PO Exam 2011 81

a
h
b
94. The number of girls speaking only Hindi is what per cent of DIRECTIONS (Q. 99) : Study the following pie-chart to answer

o
the total number of students speaking only Hindi?

o
the question that follow.

b
(a) 38.2% (b) 71.8%

.w
Break-up of students having preference for each subject
(c) 31.2% (d) 78%

o
rd
(e) None of these

p
re
95. What is the ratio of the number of boys to the number of

s
girls speaking both the languages? Geography

s
.c
History 28%
(a) 23 : 25 (b) 12 : 25

o
44%

m
(c) 12 : 13 (d) 25 : 13
Economics
(e) None of these 16%
DIRECTIONS (Qs. 96-98): Study the information given in eaeh Maths
of these questions and then answer the questions. 12%
99. Out of a total 550 students, how many students did not
96. The area of the circle is 616 cm2. What is the area of the
prefer Maths or Economics?
rectangle? (• or ‘dot’ or indicates centre of the circle.)
(a) 462 (b) 154
(c) 196 (d) 396
(e) None of these

100. Pers on Salary (in ` Iakh )


A 8.5
B 7.6
(a) 784 cm2 (b) 196 cm2 C 12.8
(c) 392 cm2 (d) Cannot be determined D 5.4
(e) None of these E 10.5

97. What is the difference (in ` lakh) between the average salary
Population in Million
and the lowest salary?
City Total Population Male Population (a) 4.2 (b) 2.65
A 12 6.5 (c) 3.56 (d) 4.06
B 15 7.2 (e) None of these
C 17 9.0
D 19 9.9 GENERAL AWARENESS WITH SPECIAL
E 22 10.8 REFERENCE TO BANKING INDUSTRY
What is the average female population in million? 101. A major Public Sector Bank raised interest rates on loans by
(a) 8.32 (b) 8.86 25 basis points – was news in some major financial
(c) 8.68 (d) 9.12 newspapers recently. This means the bank has raised interest
by 25 basis points of
(e) None of these
(a) Savings Bank Interest Rate
98. What is the per cent rise in production in 2007 from 2006? (b) Base Lending Rate
(Round off to two digits after decimal.) (c) Repo Rates
(d) Present Rates on Deposits
1400
(e) Discounted Rates of Interest
1300
102. The Finance Minister of India in one of his press
1200
conferences said that inflationary pressure is likely to
1100
continue following recent increase in rates of some
1000
commodities in international markets. Which of the following
900
commodities was he referring to as it gets frequent increase
800
2005 2006 2007 2008 2009 2010 at international levels and disturbs our Home Economy
substantially?
(a) 28.18% (b) 18.18% (a) Gold and Silver (b) Petroleum products
(c) 16.28% (d) 26.18% (c) Tea and Coffee (d) Sugar
(e) None of these (e) Jute and Jute products
y
o
u
rs
m
82 SBI & its Associates PO Exam 2011

a
h
103. Immediately prior to change in the measure of Food Inflation,

b
111. Which of the following international agencies/ organizations

o
which of the following indexes was being used for measuring had initiated an 'International Convention for the Protection

o
b
it? of All Persons from Enforced Disappearance', which came

.w
(a) Wholesale Price Index into effect recently?

o
(b) Consumer Price Index

rd
(a) World Health Organisation (WHO)
(c) Interest Rates offered by banks on deposits

p
(b) International Court of Justice

re
(d) Sensex and Nifty

s
(e) None of these (c) International Maritime Organisation

s
.c
104. What is the full form of ‘FINO’, a term we see frequently in (d) International Atomic Energy Agency (IAEA)

o
financial newspapers?

m
(e) United Nations Organisation (UNO)
(a) Financial Investment Network and Operations 112. ‘BRIC’ which was the organisation of 4 nations, namely
(b) Farmers’ Investment in National Organisation
Brazil, Russia, India and China, has now become 'BRICS',
(c) Farmers Inclusion News and Operations
Which is the fifth nation included in it?
(d) Financial Inclusion Network and Operations
(e) None of these (a) South Korea (b) Sri Lanka
105. Which of the following is/are commodity/ies on which (c) Singapore (d) Spain
subsidy is given to farmers/poor in India? (e) South Africa
(1) Fertilizer 113. The process of acquisition of agricultural land and its
(2) Kerosene compensation to farmers recently came up as a major issue
(3) LPG in which of the following states?(It also created law and
(a) Only (1) (b) Only (2)
order problem in the State.)
(c) Only(3) (d) Only (1) and (3)
(e) All (1), (2) and (3) (a) Haryana (b) Gujarat
106. ‘Bhagyam Oilfields’, which were recently in news, are (c) Uttar Pradesh (d) Rajasthan
located in which of the following states in India? (e) Punjab
(a) Rajasthan (b) Gujarat 114. What does the letter 'L' denote in the term ‘LAF’ as referred
(c) Assam (d) Maharashtra every now and then in relation to monetary policy of the
(e) Karnataka RBI?
107. The Finance Minister of India recently decided to review
(a) Liquidity (b) Liability
the position of ‘Bad Debts’ in priority sector. Which of the
following is/are the reason(s) owing to which FM took this (c) Leveraged (d) Longitudinal
decision? (e) Linear
(1) Bad Debt in agriculture is still rising substantially every 115. BP Plc, which was in news recently, is a major international
year despite special treatment to it. company in the field of
(2) Bad Debt in other areas of priority sector is almost nil (a) Heavy Machinery
now.
(b) Oil exploration and processing
(3) Govt is planning to disburse another instalment of
`1,60,000 crore, which will bring 'Bad Debts' almost at (c) Atomic Energy
'Nil level' and no special treatment will be needed in (d) Information Technology
next fiscal year. (e) None of these
(a) Only (1) (b) Only (2) 116. Who amongst the following has been appointed the new
(c) Only (3) (d) Both (1) and (2) coach of the Indian Cricket team for a period of two years?
(e) All (1), (2) and (3) (a) Stuart Law (b) Allan Border
108. Which amongst the following countries is NOT a member
(c) Gary Kirsten (d) Duncan Fletcher
of the TAPI pipeline project, which was in news recently?
(a) India (b) Afghanistan (e) Allan Donald
(c) Portugal (d) Pakistan 117. Why was Kanishtha Dhankhar's name in news recently?
(e) Turkmenistan (a) She was crowned Femina Miss India 2011.
109. An agreement on Arms Reduction popularly known as (b) She was adjudged Best Actress 2010 by Filmfare
'START' is one of the major issues in the way of smooth (c) She is the new captain ofIndia's woman cricket team
relations between which of the following two countries?
(d) She has become the new Dy Minister of Textile in place
(a) India - Pakistan (b) India - China
of Dayanidhi Maran
(c) China - Japan . (d) USA - Russia
(e) Russia - Iran (e) None of these
110. The International WWW Conference-2011 was organized 118. K Balachander, who was awarded prestigious Dada Saheb
in which of the following places in March 2011? Phalke Award 2010, is basically a/ an
(a) London (b) Paris (a) Actor (b) Director
(c) Vienna (d) Manila (c) Photographer (d) Music Director
(e) Hyderabad (e) Choreographer
y
o
u
rs
m
SBI & its Associates PO Exam 2011 83

a
h
b
119. Banks and other financial institutions in India are required 127. An online discussion group that allows direct "live" ,

o
to maintain a certain amount of liquid assets like cash, communication is known as

o
b
precious metals and other short-term securities as a reserve (a) Webcrawler

.w
all the time. In Banking World this is known as (b) chat group

o
rd
(a) CRR (b) Fixed Asset (c) regional service provider

p
(c) SLR (d) PLR (d) hyperlink

re
(e) e-mail

s
(e) None of these

s
128. Which of the following is a program that uses a variety of

.c
120. Which of the following space agencies recently launched

o
three satellites and placed them into Polar Sun Synchronous different approaches to identify and eliminate spam?

m
Orbit successfully? (a) Directory search
(a) NASA (b) ROSCOSMOS (b) Anti-spam program
(c) Web server
(c) CNSA (d) ISRO
(d) Web storefront creation package
(e) JAXA
(e) Virus
121. As per the decision taken by the Govt of India recently,
129. Connection or link to other documents or Web Pages that
“Bamboo is a liberated item now”. What does it really mean?
contain related information is called
(1) Bamboo, henceforth, will be treated as a minor forest
(a) dial-up (b) electronic commerce
produce only.
(c) hyperlink (d) e-cash
(2) Now villagers are free to sell bamboo in their community (e) domain name
forest areas.
130. Which of the following is a programming language for
(3) Gram Sabha has been given right to issue transport creating special programs like applets?
passes to legally transport the produce. (a) Java (b) cable
(a) Only (1) (b) Only (2) (c) domain name (d) Net
(c) Only (3) (d) All (1), (2) and (3) (e) COBOL
(e) None of these 131. The system unit
122. Union Cabinet recently passed a `7000-crore project to clean (a) coordinates input and output devices
up which of the following rivers in India? (b) is the container that houses electronic components
(a) Narmada (b) Ravi (c) is a combination of hardware and software
(c) Ganga (d) Chambal (d) controls and manipulates data
(e) Krishna (e) does the arithmetic operations
123. As per estimates of the Planning Commission of India, the 132. System software
target growth rate of 12th Five Year Plan is set at (a) allows the user to diagnose and troubleshoot the device
(a) 7 to 8% (b) 8.0% to 8.5% (b) is a programming language
(c) 9% to 9.5% (d) 10% to 10.5% (c) is part of a productivity suite
(e) 11% (d) is an optional form of software
124. Goodluck Jonathan was recently re-elected the President (e) helps the computer manage internal resources
of 133. Computer and communication technology, such as
communication links to the Internet, that provide help and
(a) South Africa (b) Nigeria
understanding to the end user is known as
(c) Kosovo (d) Haiti
(a) presentation file (b) information technology
(e) None of these (c) program (d) worksheet file
125. Which of the following schemes has NOT been launched (e) FTP
by the Ministry of Rural Development, Govt of india? 134. Which of the following is contained on chips connected to
(a) National Old Age Pension Scheme the system board and is a holding area for data instructions
(b) National Maternity Benefit Scheme and information? (processed data waiting to be output to
(c) National Family Benefit Scheme secondary storage)
(d) Mid-Day Meal Scheme (a) program (b) mouse
(e) All are launched by Ministry of Rural Development (c) Internet (d) memory
(e) modem
126. Where are you likely to find an embedded operating system?
135. Microsoft's Messenger allows users to
(a) on a desktop operating system (a) bypass a browser to surf the Web
(b) on a networked PC (b) create a blog
(c) on a network server (c) communicate via direct live communication
(d) on a PDA (d) identify and eliminate spam
(e) on a mainframe (e) make graphic presentations
y
o
u
rs
m
84 SBI & its Associates PO Exam 2011

a
h
b
136. Portable computer, also known as laptop computer, 146. Social Marketing means

o
weighing between 4 and 10 pounds is called

o
(a) Share market prices

b
(a) general-purpose application (b) Marketing by the entire society

.w
(b) Internet (c) Internet Marketing

o
rd
(c) scanner (d) Marketing for a social cause

p
(d) printer

re
(e) Society bye-laws

s
(e) notebook computer 147. Service Marketing is the same as

s
.c
137. The main circuit-board of the system unit is the (a) Internet Marketing (b) Telemarketing

o
(a) computer program (b) control unit

m
(c) Internal Marketing (d) Relationship Marketing
(c) motherboard (d) RAM (e) Transaction Marketing
(e) None of these 148. Market-driven strategies include
138. Which of the following is billionth of a second? (a) Identifying problems
(a) Gigabyte (b) Terabyte (b) Planning marketing tactics of peers
(c) Nanosecond (d) Microsecond (c) Positioning the Organisation and its brands in the
(e) Terasecond marketplace
139. Online Marketing is the function of which of the following? (d) Internal marketing
(a) Purchase Section (b) Production Department (e) Selling old products
(c) IT Department (d) Designs Section 149. Innovation in marketing is same as
(e) A collective function of all staff (a) Motivation (b) Perspiration
140. Customisation is useful for
(c) Aspiration (d) Creativity
(a) Designing customer specific products
(e) Team work
(b) Call centres
150. Personal Loans can be canvassed among
(c) Publicity
(a) Salaried persons (b) Pensioners
(d) Motivating the staff
(c) Foreign Nationals (d) NRI customers
(e) Cold calls
(e) Non-customers
141. The key challenge to market-driven strategy is
(a) Selling maximum products GENERAL ENGLISH
(b) Employing maximum DSAs
(c) Delivering superior value to customers DIRECTIONS (Qs. 151-160): Read the following passage
(d) Being rigid to changes carefully and answer the questions given below it. Certain words/
(e) Adopting short-term vision phrases have been printed in bold to help you locate them while
142. Effective selling skills depends on answering some of the questions.
(a) Size of the sales team Jagir Singh has sold red onions at a market in south Delhi
(b) Age of the sales team every day for the past half-century. Perched on an upturned crate,
(c) Peer strength wrapped tight against the chill air, he offers pyaz, a staple for
(d) Knowledge level of the sales team much Indian cooking, for 60 rupees a kilo, the most he can
(e) Educational level of the sales team remember. Business is brisk but most customers pick up only a
143. Generation of sales leads can be improved by small handful of onions. That is just as well. Wholesale supplies
(a) Being very talkative are tight, he says, and the quality is poor.
(b) Increasing personal and professional contacts As India's economy grows by some 9% a year, food prices
are soaring. In late December, the commerce ministry judged that
(c) Being passive
food inflation had reached 18.3%, with pricey vegetables mostly
(d) Engaging Recovery Agents to blame. Officials have made some attcmpts to temper the rise in
(e) Product designs the past month-scrapping import taxes for onions, banning their
144. A Market Plan is export and ordering low-priced sales at government-run shops.
(a) Performance Appraisal of marketing staff But there is no quick fix.
(b) Company Prospectus Heavy rain in the west of lndia brought a rotten harvest.
(c) Documented marketing strategies Vegetables from farther a field-including a politically sensitive
(d) Business targets delivery from a neighbouring country – are costly to move on
(e) Call centre India’s crowded, pot-holed roads. Few refrigerated lorries and
145. Marketing channels mean poor logistics mean that much of each harvest is wasted.
(a) Delivery objects (b) Sales targets Newspapers allege hoarders are cashing in.
The biggest problems are structural. Food producers,
(c) Deliavery outlets (d) Delivery boys
hampered by land restrictions, archaic retail networks and bad
(e) Sales teams
y
o
u
rs
m
SBI & its Associates PO Exam 2011 85

a
h
b
infrastructure fail to meet extra demand from consumers. It was 156. What can be said about the sale of onions at present as

o
estimated in October that a 39% rise in income per person in the given in the passage?

o
b
previous five years might have created an extra 220 million regular (a) Vegetable vendors are unwilling to sell onions.

.w
consumers of milk, eggs. meat and fish. Supplies have not kept (b) People are not buying as much as they used to.

o
rd
up with this potential demand. (c) The sale of onions has picked up and is

p
The broader inflation rate may be a less eye-watering unprecedented.

re
problem than the onions suggest. The central bank has lifted (d) People are buying more onions than they used to.

s
s
interest rates steadily in the past year and is expected to do so (e) None of these

.c
o
again later this month. Headline inflation fell to 7.5% in November, 157. The usage of the phrase ‘cashing in’ in the passage can

m
down by just over a percentage point from October. though it is possibly mean
still above the central bank's forecast of 5.5% for March. (a) Profiting (b) Running away
151. What is responsible for the increased demand of certain (c) Paying money (d) Bailing out
food items amongst consumers? (e) Buffering
(a) There has been an increase in the wholesale supplies 158. Which of the following is most similar in meaning to the
of this food stuff. word ‘tight’ as used in the passage?
(b) The vegetables in the market are very highly priced. (a) Firm (b) Loose
(c) There has been an increase in the incomes of people. (c) Limited (d) Taut
(d) There is a lack of availability of vegetables in the market. (e) Tense
(e) There has been a surge in population. 159. Which of the following is most similar in meaning to the
152. Which of the following is not true in the context of the word ‘temper’ as used in the passage?
passage? (a) Displeasure (b) Anger
(a) The overall inflation is not as bad as the food inflation (c) Rage (d) Harness
in India. (e) Control
(b) Help from other countries to counter food inflation 160. Which of the following is most opposite in meaning of the
word ‘archaic’ as. used in the passage?
has proved to be quite encouraging.
(a) Simple (b) Straightforward
(c) Government is banning the export of certain types of
(c) Modern (d) Lively
vegetables in order to check food inflation.
(e) Ancient
(d) Highly priced vegetables are mostly responsible for
the increased food inflation. DIRECTIONS (Qs. 161-165): Which of the phrases (a), (b), (c)
(e) All the above statements are true and (d) given below each sentence should replace the phrase
153. Which of the following is/are the reason/s for increase in printed in bold in the sentence to make it grammatically correct?
food/vegetable prices? If the sentence is correct as it is given and no correction is
required, mark (e) as the answer.
(1) Bad weather
(2) Land restrictions 161. Seeing that there was an ongoing sale in one of her favourite
stores, Seeta made a bee line for it immediately after entering
(3) Poor infrastructure for storage and transportation
the mall.
(a) Only (2) (b) Only (1) and (3)
(a) made a bee’s line for
(c) Only (1) and (2) (d) Only(2) and (3)
(b) make bees lined to
(e) All (1), (2) and (3)
(c) made a bee line to
154. Which of the following is/are TRUE in the context of the
(d) make bee line to
passage?
(e) No correction required
(1) Interest rates are being raised in India.
162. Sharon made it to work in the nicks of times, or else she
(2) India is witnessing a steady economic growth. would have missed the meeting.
(3) It has been proven that the food inflation is mainly (a) nick of time (b) nicked time
because of hoarding. (c) nick of timeliness (d) nick and time
(a) All (1), (2) and (3) (b) Only (1) and (2) (e) No correction required
(c) Only (3) (d) Only (2) and (3) 163. Varun was on cloud nine after having stood first in his class.
(e) None is true (a) in ninth cloud (b) on nine clouds
155. Which of the following is possibly the most appropriate (c) a cloudy nine (d) cloud on nine
title for the passage? (e) No correction required
(a) Food Inflation In India 164. Vithal had a habit of pass the buck when it came to important
(b) Onions And Vendors issues at work.
(c) Food Deficit Worldwide (a) pass to bucking (b) passing buck
(d) Food Imports In India (c) passing the buck (d) pass buck
(e) Benefits To Indian Consumers (e) No correction required
y
o
u
rs
m
86 SBI & its Associates PO Exam 2011

a
h
165. Puneet raked his brains and tried to find an answer to a

b
India's main competitors-especially China, but also

o
tricky question given in the paper but couldn't find one. Singapore, Taiwan, and South Korea-are investing in large and

o
b
(a) rake his brain (b) racked his brains differentiated higher education systems. They are providing

.w
(c) racked brains (d) raked brain access to a large number of students at the bottom of the academic

o
(e) No correction required

rd
system while at the same time building some research-based

p
DIRECTIONS (Qs. 166-170): Each question below has two universities that are able to compete with the world's best

re
blanks, each blank indicating that something has been omitted. institutions. The recent London Times Higher Education

s
s
Find out which option can be used to fill up the blank in the Supplement ranking of the world's top 200 universities included

.c
sentence in the same sequence to make it meaningfully complete. three in China, three in Hong Kong. three in South Korea, one in

o
m
166. The water transport project on the west coast is ____ to Taiwan, and one in India. These countries are positioning
get a shot in the arm with a new plan in which the Road themselves for leadership in the knowledge-based economies of
Development Corporation will build the infrastructure and the coming era.
___ a private party to operate the service. There was a time when countries could achieve economic
(a) scheduled, let (b) verge, permit success with cheap labour and low-tech manufacturing. Low
(c) set, sanctions (d) slated, allow wages still help, but contemporary large-scale development
(e) bound, task requires a sophisticated and at least partly knowledge-based
167. As the weekend finally rolled around. the city folk were economy. India has chosen that path, but will find a major
only ___ happy to settle down and laugh their cares _____. stumbling block in its university system.
(a) just, afar (b) too, away
India has significant advantages in the 21st century
(c) extremely, off (d) very, up
knowledge race. It has a large higher education sector – the third
(e) so, on
largest in the world in terms of number of students, after China
168. The flood of brilliant ideas has not only ___ us, but has
and the United States. It uses English as a primary language of
also encouraged us to ___ the last date for submission of
entries. higher education and research. It has a long academic tradition.
(a) overwhelmed, extend (b) enjoyed, stretch Academic freedom is respected. There are a small number of high-
(c) dismayed, decide (d) scared, scrap quality institutions, departments, and centres that can form the
(e) happy, boundary basis of quality sector in higher education. The fact that the
169. ___ about prolonged power cuts in urban areas, the States, rather than the Central Government, exercise major
authorities have decided to ___ over to more reliable and responsibility for higher education creates a rather cumbersome
eco-friendly systems to run its pumps. structure, but the system allows for a variety of policies and
(a) Worried, shift (b) Frantic, move approaches.
(c) Troubled, jump (d) Concerned, switch Yet the weaknesses far outweigh the strengths. India
(e) Endangered, click educates approximately 10 per cent of its young people in higher
170. The high cutoff marks this year have ___ college admission- education compared to more than half in the major industrialised
seekers to either ___ for lesser known colleges or change countries and 15 per cent in China. Almost all of the world's
their subject preferences. academic systems resemble a pyramid, with a small, high-quality
(a) cajoled, ask (b) pressured, sit tier at the top and a massive sector at the bottom. India has a tiny
(c) forced, settle (d) strained, compromise top tier. None of its universities occupies a solid position at the
(e) entrusted, wait top. A few of the best universities have some excellent departments
DIRECTIONS (Qs. 171-185): Read the following passage and centres, and there are a small number of outstanding
carefully and answer the questions given below it. Certain words/ undergraduate colleges. The University Grants Commission's
phrases have been printed in bold to help you locate them while recent major support to five universities to build on their
answering some of the questions. recognised strength is a step towards recognising a differentiated
India is rushing headlong towards economic success and academic system and fostering excellence. These universities,
modernisation, counting on high-tech industries such as combined, enrol well under one per cent of the student population.
information technology and biotechnology to propel the nation 171. Which of the following is TRUE in the context of the
to prosperity. India's recent announcement that it would no longer passage?
produce unlicensed inexpensive generic pharmaceuticals bowed (a) The top five universities in India educate more than 10
percent of the Indian student population.
to the realities of the World Trade Organisation while at the same
(b) India’s higher education sector is the largest in the
time challenging the domestic drug industry to compete with the
world.
multinational firms. Unfortunately, its weak higher education
(c) In the past, countries could progress economically
sector constitutes the Achilles’ heel of this strategy. Its systematic
through low manufacturing cost as well as low wages
disinvestment in higher education in recent years has yielded·
of labourers.
neither world-class research nor very many highly trained
(d) India has recently invested heavy sums in the higher
scholars, scientists or managers to sustain high-tech
education sector leading to world-class research.
development.
(e) All are true
y
o
u
rs
m
SBI & its Associates PO Exam 2011 87

a
h
b
172. What does the phrase ‘Achilles’ heel’ mean as used in the (a) Only (1) and (2) (b) Only (2)

o
passage?

o
(c) Only (3) (d) Only (1) and (3)

b
(a) Weakness (b) Quickness

.w
(e) All (1), (2) and (3)
(c) Low quality (d) Nimbleness

o
178. Which of the following, according to the passage, is/are

rd
(e) Advantage needed for the economic success of a country?

p
re
173. Which of the following is/are India's strength/s in terms of (1) Cheap labour

s
higher education?

s
(2) Educated employees

.c
(1) Its system of higher education allows variations.

o
(3) Research institutions to cater to the needs of

m
(2) Medium of instruction for most higher learning is
development
English.
(a) Only (1) and (2) (b) Only (2)
(3) It has the paraphernalia, albeit small in number, to build
a high-quality higher educational sector. (c) Only (3) (d) Only (2) and (3)
(a) Only (2) (b) Only (1) and (2) (e) All (1), (2) and (3)
(c) Only (3) (d) Only (2) and (3) DIRECTIONS (Qs. 179-182): Choose the word of words which
(e) AlI (1), (2) and (3) is most similar in meaning to the word of words printed in bold
174. What are the Asian countries, other than India, doing to as used in the passage.
head towards a knowledge-based economy?
179. FOSTERlNG
(1) Building competitive research-based universities
(a) Safeguarding (b) Neglecting
(2) Investing in diverse higher education systems
(c) Sidelining (d) Nurturing
(3) Providing access to higher education to select few
students (e) Ignoring
(a) Only (1) (b) Only (1) and (2) 180. PROPEL
(c) Only (2) and (3) (d) Only (2) (a) Drive (b) Jettison
(e) All (1), (2) and (3) (c) Burst (d) Acclimatise
175. Which of the following is possibly the most appropriate (e) Modify
title for the passage?
181. MASSIVE
(a) The Future of Indian Universities
(a) Lump sum (b) Strong
(b) Methods of overcoming the Educational Deficit in India
(c) India and the Hunt for a Knowledge-Based Economy (c) Little (d) Gaping
(d) Indian Economy Versus Chinese Economy (e) Huge
(e) Indian Economy and Its Features 182. STUMBLING BLOCK
176. What did India agree to do at the behest of the World Trade (a) Argument (b) Frustration
Organisation? (c) Advantage (d) Hurdle
(a) It would stop manufacturing all types of (e) Fallout
pharmaceuticals.
(b) It would ask its domestic pharmaceutical companies DIRECTIONS (Qs. 183-185): Choose the word/group of words
to compete with the international ones. which is most opposite in meaning to the word/group of words
(c) It would buy licensed drugs only from the USA. printed in bold as used in the passage.
(d) It would not manufacture cheap common medicines 183. CUMBERSOME
without a licence. (a) Handy (b) Manageable
(e) None of these (c) Breathtaking (d) Awkward
177. Which of the following is/are India's weakness/es when it
(e) Difficult
comes to higher education?
(1) Indian universities do not have the requisite teaching 184. RESEMBLE
faculty to cater to the needs of the higher education (a) Against (b) Similar to
sector. (c) Mirror (d) Differ from
(2) Only five Indian universities occupy the top position (e) Unfavourable to
very strongly in the academic pyramid when it comes
185. DIFFERENTIATED
to higher education.
(a) Similar (b) Varied
(3) India has the least percentage of young population
taking to higher education as compared to the rest of (c) Harmonized (d) Synchronized
the comparable countries. (e) Discriminated
y
o
u
rs
m
88 SBI & its Associates PO Exam 2011

a
h
b
DIRECTIONS (Qs. 186-190): Rearrange the following six Around the world, forests are being (191) at a rate of about

o
thirteen million hectares a year and deforestation accounts for an

o
sentences (1), (2), (3), (4), (5) and (6) in the proper sequence to

b
estimated 17%-20% of all global emissions. In addition, forests

.w
form a meaningful paragraph; then answer the questions given
and other terrestrial carbon sinks play a (192) role in preventing

o
below them.

rd
runaway climate change, soaking up a full 2.6 Gt of atmospheric
(1) Its prevalence reflects very badly on a society that is

p
carbon every year. The destruction of forests, therefore, not only

re
not able to stop this evil.
emits carbon–a staggering 1.6 Gt a year, which severely (193)

s
s
(2) Though elimination of child labour is an impossible

.c
forests' capacity to absorb emissions from other sources-but also
task considering the current socio-economic scenario

o
drastically (194) the amount of forested land available to act as a

m
of these poor families, the Indian government is carbon sink in the future.
committed to the task of ensuring that no child remains However, the effects of deforestation extend beyond carbon.
illiterate, hungry and without medical care.
Rainforests (195) a wide variety of ecosystems services, from
(3) Therefore, unless the socio-economic status of the regulating rainfal1 to purifying groundwater and keeping fertile
poor families is improved. India has to live with child soil from (196); deforestation in one area can seriously damage
labour. food production and (197) to clean water in an entire region. The
(4) The members of these households have to send their value of global ecosystem services has been estimated at 33
children to work, even if the future of these innocent trillion USD each year (almost half of global GDP), but these
children is ruined, as that is the only choice open for services have been taken for granted without a mechanism to
them to survive in this world. make the market reflect their value. Rainforests are also a home
(5) Child labour is, no doubt, an evil that should be done and (198) of income for a huge number of people in Africa, Asia,
away with at the earliest. and South America. (199) this, economic pressures frequently
(6) But in a society where many households may have to drive both locla communities and national government in the
suffer the pangs of hunger if the children are withdrawn developing world to (200) these forests in ways that are
from work, beggars can’t be choosers. unsustainable, clear-cutting vast areas for fuel, timber, mining, or
186. Which of the following should be the FIRST sentence after agricultural land
rearrangement? 191. (a) ended (b) destroyed
(a) 1 (b) 5 (c) extinct (d) killed
(c) 3 (d) 6 (e) wasted
(e) 4 192. (a) tough (b) important
187. Which of the following should be the THIRD sentence (c) vital (d) biggest
after rearrangement? (e) effective
(a) 2 (b) 1 193. (a) affects (b) diminishes
(c) 3 (d) 6 (c) increases (d) alternates
(e) 5 (e) impairs
188. Which of the following should be the SECOND sentence 194. (a) plagues (b) develops
after rearrangement? (c) reduces (d) shortens
(a) 1 (b) 6 (e) influences
(c) 4 (d) 2 195. (a) sell (b) offer
(e) 3 (c) give (d) provide
189. Which of the following should be the FIFTH sentence after (e) earns
rearrangement? 196. (a) transforming (b) decoding
(a) 5 (b) 1 (c) erupting (d) draining
(c) 6 (d) 3 (e) eroding
(e) 4 197. (a) handiness (b) excess
190. Which of the following should be the SIXTH (LAST) (c) availability (d) access
sentence after rearrangement? (e) supply
(a) 1 (b) 3 198. (a) beginning (b) source
(c) 5 (d) 4 (c) ways (d) reference
(e) 2 (e) measure
199. (a) Despite (b) Also
DIRECTIONS (Qs. 191-200): In the following passage, there (c) Inspite (d) Apart
are blanks, each of which has been numbered. These numbers (e) Beside
are printed below the passage and against each, five words are 200. (a) exploit (b) encompass
suggested, one of which fits the blank appropriately. Find out
(c) nurture (d) work
the appropriate word in each case.
(e) improve
y
o
u
rs
m
SBI & its Associates PO Exam 2011 89

a
h
b
o
o
b
.w
o
rd
p
re
1. (b) All others are synonyms. 15. (c) Using statement I :

s
2. (d) All others are synonyms.

s
.c
3. (a) All others imply ‘UP’. Day Subject

o
m
4. (d) In the expression A > B ³ C D £ E = F to make Monday Botany
A > D true and F ³ C true. T uesday Mathematics
5. (a) Trying option (a) Wednesday Physics
P>Q=R£T<S Thursday Chemistry
P>R Friday Zoology
R < S As Q = R so Q < S Chemistry is not taught on Wednesday.
Both the expressions are true in option (a) Using statement II :
6. (e) P × Q means P is a father of Q.
Q + P means Q is daughter of R Day Subject Subjec t
R – T means R is sister of T. Monday Botany/Zoology Botany/Zoology
It is clear that P is husband of R. If he establish that T Tuesday Mathematics Chemistry
is either son or daughter of S, then P would be son-in- Wednesday Physics Mathematics
law of S. Thursday Chemistry Physics
T + S means T is daughter of S. Friday Zoology/Botany Zoology/Botany
T ÷ S means T is son of S.
16. (c) From statement I :
(7-12) : At 9 : 30 the minute and the hour hands will make an
Row 1. ¯ P V S T R Q angle alightly more than 90°. So now, the time is not
Row 2. ­ C F A E B D 9 O’ clock.
7. (d) 8. (a) 9. (b) From Statement II : At 8 : 45, the hour and the minute
10. (b) 11. (c) hands would coincide with each other. So, time now is
not 9 O’ clock.
12. (e) There is a difference of one in all the rest.
17. (d) From statement I :
13. (c) Using statement I : B is the father of M and T.
Q > R, S > T > P B is the grandfather of F.
Q is the heaviest. The gender of F is not known.
Using statement II : From statement II :
Q > R > S, T, P There is no mention of B.
From both the statements.
Q is the heaviest.
The gender of F is not known.
14. (b) Using statement I : (18-20) : The word and number arrangement machine rearranges
A A
the input with the logic that in step I, it shifts the largest
C E C
D number to the left-most place and the last word coming
in English alphabetical series to the rightmost place.
E B D B In step II, it shifts the smallest number to the leftmost
We cannot determine if all the friends are facing the place and the next word (in reverse alphabetical order)
centre. to the rightmost. In step III 2nd largest number is shifted
Using statement II. to the leftmost place and so on.
A Input : fun 89 at the 28 16 base camp 35 53 here 68
Step I : 89 fun at 28 16 base camp 35 53 here 68 the
D C Step II : 16 89 fun at 28 base camp 35 53 68 the here
Step III : 68 16 89 at 28 base camp 35 53 the here fun
E B
Step IV : 28 68 16 89 at base 35 53 the here fun camp
A is facing outside and C is facing the centre of circle. Step V : 53 28 68 16 89 at 35 the here fun camp base
So, all friends are not facing the centre. Step VI : 35 53 28 68 16 89 the here fun camp base at
18. (e) 19. (c) 20. (d)
y
o
u
rs
m
90 SBI & its Associates PO Exam 2011

a
h
b
(21-27) :

o
(Chairman)

o
Appar- Bun-
41. (d) House

b
(Managing E tment glows

.w
Director) C F (Group

o
Leader)

rd
OR
(Company

p
re
Secretary) B H (Manager)
Bun- Appar-

s
House

s
glows tment

.c
(Vice

o
President) G D (Financial

m
A Conclusion I : False
Advisor)
(President) Conclusion II : False

21. (d) 22. (e) 23. (a)


24. (d) 25. (c) 26. (e) 42. (a) Gases Liquids
27. (b)
Water
28. (c) Even (C) can be inferred only probably.
29. (e) If the infrastructure does not exist, the deal will be a OR
wastage of money.
30. (b) Getting connected by air is bound to have an impact
on road traffic.
31. (d) Takes care of the compatibility factor and (A) states
the advantage over other competing planes. Gases Liquids
(32-37) : ‘always create new ideas’ ® ‘ba ri sha gi’ ... (1)
‘ideas and new thoughts’ ® ‘fa gi ma ri’ ...(2) Water
‘create thoughts and insights’ ® ‘ma jo ba fa’ ...(3)
‘new and better solutions’ ® ‘ki ri to fa’ ...(4) Conclusion I : True
Using (1) and (4), Conclusion II : False
new ® ri
Using (1), (2) and (4), Hours
ideas ® gi Seconds
and ® fa 43. (b) Day
thoughts ® ma
Using (1) and (3),
create ® ba
always ® sha
insights ® jo OR
better solutions ® ki to
32. (c) 33. (d) 34. (b) Hours
35. (b) 36. (d) 37. (a) Seconds
38. (a) This seems to be the only logic behind the imposition Day
of tax in such specific areas.
39. (e) This is the only choice which goes in favour of tenants.

40. (b)
Circles
Rings ellipse Conclusion I : False
Conclusion II : True
Squares (44 -45)

r s
so
Teachers

fes
o
Pr
rs
Conclusion I : False ecture
L
Conclusion II : True
y
o
u
rs
m
SBI & its Associates PO Exam 2011 91

a
h
OR

b
58. (a)

o
o
6

b
.w
Professors 2 8 12 20 30 42 56
Lecturers

o
rd
Teachers +4 +6 +8 + 10 + 12 + 14

p
8 is written in place of 6.

re
s
OR 59. (e)

s
.c
60

o
s
or

m
ess Teachers, 32 16 24 65 210 945 5197.5
of
Pr
Lecturers × 0.5 × 1.5 × 2.5 × 3.5 × 4.5 × 5.5
65 is written in place of 60.
44. (a) Conclusion I : True 60. (d)
Conclusion II : False 193
45. (b) Conclusion I : False
7 13 25 49 97 194 385
Conclusion II : True.
46. (d) The shifting of the elements takes place in such a way +6 + 12 + 24 + 48 + 96 + 192
that the change is completed in four steps. So the
194 is written in place of 193
change from figure five to six will be similar to the
change from figure one to figure two. 61. (a) Amount reinvested in Equity Funds = `94500
47. (a) In each step, two pairs of elements get changed, Amount reinvested in debt + Equity Funds
beginning from the upper left and lower right. The
13
change takes place in three successive steps. = 94500 ´ =`175500
48. (c) The main design rotates respectively through 45° 7
clockwise, 180° and 90° anticlockwise after every two Amount invested earlier in Debt + Equity Funds
figures. The shaded leaflet rotates through 45° 175500
clockwise, 90° clockwise, 135° clockwise, 45° = =`135000
1.3
anticlockwise and 90° anticlockwise.
49. (d) Watch the rotation of each element separetely. The Original amount invested in equity funds
triangel rotates by 90° ACW and 180°. The circle rotates 5
= ´ 135000 =` 75000
45°, 135°, 90° CW 45° CW ... ‘C’ rotates 45°, 90°, 135°, 9
180° CW.
62. (c) Let the son’s present age be x years. Then the father’s
50. (b) In each step elements interchange in pairs while one
element beginning from one end is replaced by a new present age is (x + 30) years.
one. The line of orientation rotates by 45° ACW. Father’s age after 10 years = (x + 40) years
32 Son’s age after 10 years = (x + 10) years
51. (e) ´ 260 = 83.2 » 83
100 (x + 40) = 3(x + 10)
1 2 3 x + 40 = 3x + 30
52. (b) ´ ´ ´1715 = 85.75 » 85 2x = 10
8 3 5
53. (c) 25 × 124 + 389 × 15 = 3100 + 5835 = 8935 \ x=5
561 63. (e) Let the length and breadth of the original rectangle be
54. (a) ´ 20 = 320.5 » 320 ‘L’ m and ‘B’ m respectively.
35
After increasing the length by 20% and decreasing
55. (d) (15)2 ´ 730 = 225 ´ 27 = 6075
the breadth by 20% area is 192.
56. (d) (1.2 L) × (0.8 B) = 192
155 or 0.96 LB = 192
3601 3602 1803 604 154 36 12
LB = 200
÷1+1 ÷2+2 ÷3+3 ÷4+4 ÷5+5 ÷6+6 64. (b) Let the original numbers be x and y and their product
154 is written in place of 155.
be xy.
57. (b)
45 1 x 3 xy
Product of rd of x and 150% of y = ´ y =
4 12 42 196 1005 6066 42511 3 3 2 2
xy
Required answer = ´ 100 = 50%
2 ´ xy
42 is written in place of 45.
y
o
u
rs
m
92 SBI & its Associates PO Exam 2011

a
h
b
From Statement III,
65. (d)

o
o
PR ´ 2

b
= 3000 ...(iii)

.w
28 100

o
Dividing equation (i) by (iii),

rd
p
The area of the shaded region area of square – Area of PR 2 1000000 20

re
= =
the circle PR 150000 3

s
s
.c
22 20
Required answer = (28)2 -

o
´14 ´ 14 ÞR= p.c.p.a

m
7 3
= 784 – 616 = 168 m2 72. (e) From Statement I and II
66. (c) Ratio of the work done by Sujit and Amit = 4 : 5
x+y æ x + y 18 ö
Total key depressions done by Amit Speed of train = m/s = ç ´ ÷ km/h
n è n 5ø
5 From statement II and III
= ´ 576000 = 3, 20,000
9
y æ y 18 ö
Amit’s speed in key depressions per hour Speed of train = m/s = ç ´ ÷ km/h
m èm 5ø
320000
= = 8000 73. (d) Using statement II and III, we can find the number of
8´ 5
students in second class and pass class only. As there
67. (a) Total number of ways of selecting 4 children out of 8
is no link given between the first class and the other
8 8´7´ 6´5 classes, we cannot find the number of students in first
= C4 = = 70
1´ 2 ´ 3 ´ 4 class.
Number of ways of selecting 4 girls out of 5 = 5C4 = 5
74. (e) Using Statement I :
5 1
Required probability = = A 2
70 14 =
B 3
68. (d) Amount received by all the officers Using Statement II :
= 45 × 25000 = 11,25,000 A is 40% of total. So B is 60% of total amount invested.
3 A 40 2
Amount received by each clerk = ´ 25000 = 15000 = =
5 B 60 3
Amount received by all the clerks Using statement III:
= 80 × 15000 = 12,00,000 A = 45000
Total amount of profit earned = 11,25,000 + 12,00,000 Putting the value of statement III in any of the
= `23.25 lakh. statements I or II, we can find the amount invested in
69. (e) Let the cost price of the articles be `100 scheme B.
Marked Price = `130 75. (c) Using statement I and II we can find the area of the
After giving a discount of 10% the selling price of the rectangle and using statement III we can find the cost.
articles = 0.9 × 130 = 117 76. (b) Profit earned by Company B in 2006 is 65% of
investment or 812500.
(117 - 100)
So, actual profit per cent = ´100 = 17% 812500
100 \ Income = ´ 165 = 2062500
70. (d) Salary in June 2011 = 22385 65
77. (c) Let the amount invested by Company A and B in the
22385
Salary in June 2009 = =`18500 year 2005 be `x each.
1.1 ´ 1.1
Income of A in 2005 = 1.70x
71. (b) From statement I, Income of B in 2005 = 1.55 x
If the investment amount be ` P and rate of interest be
A 1.70x 34
R p.c.p.a. then Ratio = = =
B 1.55x 31
PR 2 78. (b) Amount invested by Company B in 2009
Differenece = = 100 ...(i)
10000 1
From statement II, = ´ 27 ´105 = 9 lakh
3
PR1 ´ 3 Amount invested by Company A in 2009
= 19500 - P ...(ii)
100 2
= ´ 27 ´105 = 18 lakh
3
y
o
u
rs
m
SBI & its Associates PO Exam 2011 93

a
h
b
Profit earned by Company B 87. (a) Number of candidates shortlisted from state E for all

o
o
80 the posts = 120 + 280 + 75 + 280 + 260 + 520 = 1535
´ 9 ´ 105 = 72 ´104

b
.w
100 Number of candidates shortlisted from state G for all

o
75 posts = 550 + 140 + 325 + 220 + 410 + 200 = 1845

rd
Profit earned by company A = ´18 = 13.5 lakh 1535 307

p
100 Required answer = =

re
Total profit = 13.5 + 7.2 = 20.7 lakh 1845 369

s
s
88. (e) Total number of candidates eligible form all states for

.c
145
79. (a) Income of A in 2007 = ´12 ´105 = 174 ´104

o
post I = 25200

m
100
Total number of candidates eligible form all states for
174 ´104 post VI = 39400
Amount invested in 2008 = ´100 = 1087500
160 25200
80. (e) Let total investment be `x. Required answer = ´ 100 = 63.9%
39400
55% of x = 10.15 × 105 89. (b)
5
10.15 ´10 90. (c) Total candidates shortlisted for post V = 1650
x= ´100 = 1845454 » 18.45 lakh
55 Total candidates shortlisted for post VI = 2780
81. (c) Income of Company B in 2004 1650 165
Required ratio = =
= 1.55 × 12 × 105 = 18.6 lakh 2780 278
Investment in 2005 = 18.6 lakh. (91-95) : Eng Hindi Both Total
55 Boys 24 18 108 150
Profit earned in 2005 = ´ 18.6 ´ 105 = 10.23 lakh Girls 55 78 117 250
100
Total 79 96 225 400
24 ´105 91. (b) 18 + 108 = 126
82. (a) Investment of Company A in 2008 = = 15 lakh
1.60 92. (d)
Profit in 2008 = 24 – 15 = 9 lakh. 93. (a) 79 + 225 = 304
45
Profit in 2007 = ´15 ´ 105 = 6.75 lakh 94. (e)
78
´100 = 81.25%
100 96
Required answer = 9 – 6.75 = 2.25 lakh
108 12
90 70 95. (c) Ratio = =
´ 25 ´ 105 + ´ 25 ´105 117 13
83. (d) Required answer = 100 100 96. (c) Area of the circle = pr2 = 616
2 Þ r2 = 196
Þ r = 14 cm
25 ´ 105 é 90 + 70 ù Length of the rectangle = Diameter of the circle.
= êë 2 úû
100 Breadth of the rectangle = Radius of the circle
Area of rectangle = 28 × 14 = 392 cm2.
= 25 × 103 × 80 = 20 lakh
97. (a) The total population of all cities = 85 million
430 Total males in all cities = 43.4 million
84. (c) ´100 = 6.9%
6200 Total females = 85 – 43.4 = 41.6 million
85. (e) Required average 41.6
Average female population = = 8.32 million
5200 + 8400 + 7600 + 2600 + 3800 + 4400 + 6000 5
=
7 (1300 - 1100)
98. (b) ´100 = 18.18%
1100
38000
= = 5428.5 » 5500 72
7 99. (d) ´ 550 = 396
86. (d) Number of candidates eligible for post I 100
= 100 (25 + 32 + 28 + 24 + 30 + 48 + 65) 8.5 + 7.6 + 12.8 + 5.4 + 10.5
100. (e) Average salary =
= 25200 5
Number of candidates shortlisted for post I 44.8
= = 8.96
= 65 + 220 + 280 + 85 + 120 + 325 + 550 5
= 1645 Required answers = 8.96 –5.4 = 3.56 lakh
101. (b) 102. (b) 103. (a) 104. (e) 105. (e)
1645 106. (a) 107. (a) 108. (c) 109. (d) 110. (e)
Required answer = ´ 100 = 6.52%
25200
y
o
u
rs
m
94 SBI & its Associates PO Exam 2011

a
h
b
111. (e) 112. (e) 113. (c) 114. (a) 115. (b) 173. (d)

o
116. (d) 117. (a) 118. (b) 119. (c) 120. (d)

o
174. (b)

b
121. (d) 122. (c) 123. (c) 124. (b) 125. (d)

.w
126. (d) 127. (b) 128. (b) 129. (c) 130. (a) 175. (c) India And The Hunt for A Knowledge Based Economy

o
131. (c) 132. (e) 133. (e) 134. (d) 135. (c)

rd
176. (d) It would not manufacture cheap common medicines
136. (e) 137. (c) 138. (c) 139. (c) 140. (a)

p
without a license.

re
141. (c) 142. (d) 143. (b) 144. (c) 145. (c)

s
146. (d) 147. (d) 148. (c) 149. (d) 150. (a) 177. (c)

s
.c
151. (c) There has been an increase in the incomes of people. 178. (d)

o
m
152. (b) Help from other countries to counter food inflation
179. (d) The meaning of the word Foster (Verb) as used in the
has proved to be quite encouraging.
153. (e) 154. (b) 155. (a) passage is : to encourage something to develop;
156. (b) People are not buying as much as they used to. promote; nurture.
157. (a) The meaning of the phrase Cash in (on something) as 180. (a) The meaning of the word Proper (Verb) as used in the
used in the passage is : to gain an advantage for
yourself from a situation that is thought to be immoral; passage is : to move, drive or push something forward
profiting. or in a particular direction.
Look at the sentence : Look at the sentence :
The film studio is being accused of cashing in on the
singer’s death. He succeeded in propelling the ball across the line.
158. (c) The meaning of the word Tight (Adjective) as used in 181. (e) The meaning of the word Massive (Adjective) as used
the passage is : difficult to manage with because there in the passage is : extremely large or serious; huge.
is not enough; limited.
159. (e) The meaning of hte word Temper (Verb) as used in the 182. (d) The meaning of the word Stumbling block (Noun) is :
passage is : mitigate; control; to make something less something that causes problems and prevents you
severe by adding something that has the opposite from achieving your aim; hurdle; obstacle.
effect.
183. (b) The meaning of the word Cumbersome (Adjective) is:
Look at the sentence :
Justice must be tempered with mercy. large and heavy; bulky; slow and complicated.
160. (c) The meaning of the word Archaic (Adjective) as used The word manageable means : possible to deal with or
in the passage is : old and no longer used; outdated.
control.
Look at the sentence :
The system is archaic and unfair and needs changing. Look at the sentence :
161. (e) Idiom make a beeline for something means : to go The debt has been reduced to a more manageable level.
straight towards something / somebody as quickly as
184. (d) The meaning of the word Resemble (Verb) is : to look
you can.
162. (a) Idiom in the nick of time means : at the very last like or be similar to another perosn or thing.
moment; just in time before something bad happens. Its antonym should be differ from.
163. (e) Idiom on cloud nine means : extremely happy. 185. (c) The meaning of the word Differentiate (Verb) is : to
164. (c) passing the bucks
165. (b) Idiom rack your brain (s) means : to think very hard or show or recognise that two things are not the same;
for a long time about something. distinguish.
166. (e) 167. (a) 168. (a) 169. (d) 170. (c) Hence, the antonym of differentiated should be
171. (c) In the past, countries could progress economically
through low manufacturing cost as well as low wages harmonized which means : made similar.
of labourers. 186. (b) 187. (d) 188. (a) 189. (d) 190. (e)
172. (a) The word Achilles heel means : a weak point or fault in 191. (b) 192. (c) 193. (e) 194. (a) 195. (d)
somebody’s character which can be attacked by other
196. (e) 197. (d) 198. (b) 199. (a) 200. (a)
people.
y
o
u
rs
m
a
h
b
o
o
IBPS PO/MT EXAM 2011

b
.w
o
rd
p
Based on Memory

re
s
s
.c
o
Reasoning Ability 5. Which of the following words will be formed based on the

m
given conditions?
1. A school bus driver starts from the school, drives 2 km (a) SPAEL (b) PEALS
towards North, takes a left turn and drives for 5 km. He then (c) LEAPS (d) SEPAL
takes a left turn and drives for 8 km before taking a left turn (e) LAPSE
again and driving for 5 km. The driver finally takes a left DIRECTIONS (Qs. 6-10) : In each question below are two/three
turn and drives 1 km before stopping. How far and towards statements followed by two conclusions numbered I and II. You
which direction should the driver drive to reach the school
have to take the two/three given statements to be true even if
again?
they seem to be at variance from commonly known facts and
(a) 3 km towards North (b) 7 km towards East then decide which of the given conclusions logically follows
(c) 6 km towards South (d) 6 km towards West from the given statements disregarding commonly known facts.
(e) 5 km towards North
Give answer (a) if only conclusion I follows
DIRECTIONS (Qs. 2-3) : Read the following information Give answer (b) if only conclusion II follows.
carefully and answer the questions which follow: Give answer (c) if either conclusion I or conclusion II follows.
A, B, C, D, E and F live on different floors in the same building Give answer (d) if neither conclusion I nor conclusion II follows.
having six floors numbered one to six (the ground floor is numbered Give answer (e) if both conclusion I and conclusion II follow.
1, the floor above it , number 2 and so on and the topmost floor is (Qs. 6-7) :
numbered 6. Statements : All gliders are parachutes.
A lives on an even numbered floor. There are two floors No parachute is an airplane.
between the floors on which D and F live. F lives on a floor above All airplanes are helicopters.
D’s floor. D does not live on floor number 2. B does not live on an
6. Conclusions : I. No glider is an airplane.
odd numbered floor. C does not live on any of the floors below
II.All gliders being helicopters is a
F’s floor. E does not live on a floor immediately above or
possibility.
immediately below the floor on which B lives.
7. Conclusions : I. No helicopter is a glider.
2. Who amongst the following live on the floors exactly
between D and F ? II. All parachutes being helicopters is a
(a) E, B (b) C, B possibility.
(c) E, C (d) A, E 8. Statements : Some mails are chats.
(e) B, A All updates are chats.
3. On which of the following floors does B live? Conclusions : I. All mails being updates is a possibility.
(a) 6th (b) 4th II. No update is a mail.
(c) 2nd (d) 5th (Qs. 9-10) :
(e) Cannot be determined Statement : No stone is metal.
Some metals are papers.
DIRECTIONS (Qs. 4-5) : Study the following information to All papers are glass.
answer the given questions: 9. Conclusions : I. All stones being glass is a possibility.
II. No stone is a paper.
In a five letter English word (which may or may not be a
meaningful English word), there are two letters between L and P. 10. Conclusions : I. No glass is a metal.
S is not placed immediately next to L. There is only one letter II. Atleast some glass is metal.
between S and A. S is towards the right of A. S is not placed DIRECTIONS (Qs. 11-15) : Study the following information to
immediately next to E. answer the given questions:
4. Which of the following is correct with respect to the word A word and number arrangement machine when given an
thus formed?
input line of words and numbers rearranges them following a
(a) E is at one of the extreme ends of the word.
particular rule. The following is an illustration of input and
(b) P is not placed immediately next to A.
rearrangement. (All the numbers are two digits numbers.)
(c) There are two letters between A and E in the word thus
Input : sine 88 71 cos theta 14 56 gamma delta 26
formed.
Step I : cos sine 71 theta 14 56 gamma delta 26 88
(d) P is placed second to the right of E.
(e) None is correct Step II : delta cos sine theta 14 56 gamma 26 88 71
y
o
u
rs
m
96 IBPS PO/MT Exam 2011

a
h
b
Step III : gamma delta cos sine theta 14 26 88 71 56 neighbour of either B or the representative from Canara Bank.

o
Step IV : sine gamma delta cos theta 14 88 71 56 26 Representative from Bank of Maharashtra sits second to right of

o
b
Step V : theta sine gamma delta cos 88 71 56 26 14 D. D is neither the representative of Canara Bank nor Bank of

.w
and Step V is the last Step of the arrangement of India. G and the representative from UCO Bank are immediate

o
neighbours of each other. B is not the representative of UCO

rd
the above input as the intended arrangement is
Bank. Only one person sits between C and the representative

p
obtained.

re
As per the rules followed in the above steps, find out in from Oriental Bank of Commerce.

s
s
each of the following questions the appropriate steps for the H sits third to left of the representative from Dena Bank.

.c
Representative from Punjab National Bank sits second to left of

o
given input,

m
the representative from Syndicate Bank.
Input for the questions:
16. Who amongst the following sit exactly between B and the
Input : for 52 all 96 25 jam road 15 hut 73 bus stop 38 46
representative from Bank of India?
(All the numbers given in the arrangement are two digit numbers)
(a) A and the representative from UCO Bank
11. Which word/number would be at the 6th position from the (b) F and G
left in Step V ? (c) H and the representative from Bank of Maharashtra
(a) 25 (b) stop (d) H and G
(c) jam (d) all (e) Representatives from Syndicate Bank and Oriental
(e) road Bank of Commerce
12. Which of the following would be the Step III ? 17. Who amongst the following is the representative from
(a) hut for bus all 25 jam road 15 stop 38 96 73 52 46 Oriental Bank of Commerce?
(b) for bus all 25 jam road 15 hut 38 stop 96 46 73 52 (a) A (b) C
(c) hut for bus all jam road 15 stop 38 96 73 52 46 25 (c) H (d) G
(d) for bus all 25 jam road 15 hut stop 38 46 96 73 52 (e) D
(e) None of these 18. Four of the following five are alike in a certain way based on
13. Which word/number would be at 8th position from the right the given arrangement and thus form a group. Which is the
in Step IV? one that does not belong to that group?
(a) H - UCO Bank
(a) 15 (b) road
(b) A - Canara Bank
(c) hut (d) jam
(c) D - Bank of Maharashtra
(e) stop
(d) E -Syndicate Bank
14. Which of the following would be Step VII ?
(e) F - Punjab National Bank
(a) stop road jam hut for bus all 15 96 73 5246 38 25 19. Who amongst the following sits second to left of B ?
(b) road jam hut for bus all stop 15 25 38 46 52 73 96 (a) C
(c) stop road jam hut for bus all 96 73 52 46 38 25 15 (b) H
(d) jam hut for bus all 25 road stop 15 96 73 52 46 38 (c) The representative from Canara Bank
(e) There will be no such step as the arrangement gets (d) The representative from Punjab National Bank
established at Step VI (e) G
15. Which step number would be the following output ? 20. Which of the following is true with respect to the given
bus all for 52 25 jam road 15 hut stop 38 46 96 73 seating arrangement?
(a) There will be no such step (a) B is the representative from Bank of Maharashtra.
(b) III (b) C sits second to right of H.
(c) II (c) The representative from Dena Bank sits to the
(d) V immediate left of the representative from UCO Bank.
(e) VI (d) A sits second to right of the representative from Bank
of India.
DIRECTIONS (Qs. 16-20) : Study the following information
(e) The representatives from Bank of Maharashtra and
carefully and answer the given questions.
Syndicate Bank are immediate neighbours of each
Representatives from eight different Banks viz.. A, B, C, D, other.
E, F, G and H are sitting around a circular table facing the centre DIRECTIONS (Qs. 21-25) : Each of the questions below consists
but not necessarily in the same order. Each one of them is from a of a question and three statements numbered I, II and III given
different Bank viz. UCO Bank, Oriental Bank of Commerce, Bank below it. You have to decide whether the data provided in the
of Maharashtra, Canara Bank, Syndicate Bank, Punjab National statements are sufficient to answer the question.
Bank, Bank of India and Dena Bank.
21. How many daughters does W have?
F sits second to right of the representative from Canara
I. B and D are sisters of M.
Bank. Representative from Bank of India is an immediate neighbour
II. M's father T is husband of W.
of the representative from Canara Bank. Two people sit between
Ill. Out of the three children which T has, only one is a
the representative of Bank of India and B, C and E are immediate
boy.
neighbours of each other. Neither C nor E is an immediate
y
o
u
rs
m
IBPS PO/MT Exam 2011 97

a
h
(a) Only I and III

b
City, Swift D'Zire and Ford Ikon. There are three females among

o
(b) All I, II and III are required to answer the question them one in each car. There are at least two persons in each car.

o
b
(c) Only II & III are sufficient to answer the question R is not travelling with Q and W. T, a male, is travelling with

.w
(d) Question cannot be answered even with all I, II and III only Z and they are not travelling to Chennai. P is travelling in

o
(e) Only I and II

rd
Honda City to Hyderabad. S is sister of P and travels by Ford
22. Who among A, B, C, D, E and F each having a different

p
Ikon. V and R travel together. W does not travel to Chennai.

re
height, is the tallest? 26. Who is travelling with W ?

s
I. B is taller than A but shorter than E.

s
(a) Only Q (b) Only P

.c
II. Only two of them are shorter than C. (c) Both P and Q (d) Cannot be determined

o
m
III. D is taller than only F. (e) None of these
(a) Only I and II 27. Members in which of the following combinations are
(b) Only I and III travelling in Honda City ?
(c) Only II and III
(a) PRS (b) PQW
(d) All I, II and III are required to answer the question
(c) PWS (d) Data inadequate
(e) All I, II and III are not sufficient to answer the question
(e) None of these
23. How is 'go' written in a code language?
28. In which car are four members travelling?
I. 'now or never again' is written as 'tom ka na sa' in that
code language. (a) None
II. 'you come again now' is written as 'ja ka ta sa' in that (b) Honda City
code language. (c) Swift D'zire
III. 'again go now or never' is written as 'na ho ka sa tom' in (d) Ford Ikon
that code language. (e) Honda City or Ford Ikon
(a) Only I and III 29. Which of the following combinations represents the three
(b) Only II and III female members?
(c) Only I and II (a) QSZ (b) WSZ
(d) All I, II and III are required to answer the question (c) PSZ (d) Cannot be determined
(e) None of these (e) None of these
24. Towards which direction is village J from village W ? 30. Members in which car are travelling to Chennai ?
(a) Honda City
I. Village R is to the west of Village W and to the north of
(b) Swift D'Zire
Village T.
(c) Ford Ikon
II. Village Z is to the east of Village J and to the south of
(d) Either Swift D'Zire or Ford Ikon
Village T.
(e) None of these
III. Village M is to the northeast of Village J and north of
31. There has been a spate of rail accidents in India in the
Village Z. recent months killing large numbers of passengers and
(a) Only III (b) Only II and III injuring many more. This has raised serious doubts about
(c) All I, II and III are required to answer the question the railway's capability of providing safety to travellers.
(d) Question cannot be answered even with all I, II and III Which of the following statements substantiates the views
(e) None of these expressed in the above statement?
25. On which day of the week starting from Monday did Suresh (a) Indian Railways has come to be known to provide best
visit Chennai ? passenger comfort in the recent years.
l. Suresh took leave on Wednesday. (b) People have no option other than travelling by rail
over long distances.
II. Suresh visited Chennai the day after his mother's visit
(c) The railway tracks at many places have been found to
to his house be stressed due to wear and tear in the recent times.
III. Suresh's mother visited Suresh's house neither on (d) Local residents are always the first to provide a helping
Monday nor on Thursday hand to the passengers in the event of such disasters.
(a) Only II and III (e) None of these
(b) Only I and II 32. It has been reported in many leading newspapers that the
(c) Only I and III current year's monsoon may be below the expected level as
(d) All I, II and III are required to answer the question many parts of the country are still not getting adequate
(e) Question cannot be answered even with all I, II and III rainfall.
Which of the following can be a possible fall out of the
DIRECTIONS (Qs. 26-30) : Study the following information above situation?
carefully and answer the questions given below: (a) People from those affected areas with less rainfall may
migrate to urban areas.
P, Q, R, S, T, V, W and Z are travelling to three destinations
(b) Govt. may announce ex-gratia payment to all the
Delhi, Chennai and Hyderabad in three different vehicles – Honda
farmers affected in these areas.
y
o
u
rs
m
98 IBPS PO/MT Exam 2011

a
h
b
(c) Govt. may declare these areas as drought affected performed better than the students of the local college

o
o
areas. in all the previous examinations.

b
.w
(d) People may blame the govt. and agitate for not getting (d) The local management college recently retrenched

o
adequate water for cultivation. many of its regular faculty members.

rd
(e) None of these (e) None of these

p
re
33. The condition of the roads in the city has deteriorated 36. A very large number of technically qualified young Indians

s
considerably during the first two months of monsoon and are coming out of colleges every year, though there are not

s
.c
most of the roads have developed big pot holes. enough opportunities for them to get gainful employment.

o
m
Which of the following can be a possible effect of the above Which of the following contradicts the views expressed in
cause? the above statement?
(a) The municipal corporation had repaired all the roads (a) Technically qualified persons are far superior to those
in the city before onset of monsoon with good quality with standard degrees like B.A./B.Sc./B.Com. etc.
material. (b) The Govt. has not done effective -perspective planning
(b) A large number of people have developed spine related for engaging technically qualified personnel while
injuries after regularly commuting long distances by authorising the setting up of technical colleges.
road within the city. (c) A huge gap exists between the level of competence of
(c) The municipal corporation has been careful in technically qualified graduates and requirement of the
choosing the contractors for repairing roads in the industry.
past. (d) Majority of the technically qualified persons are
(d) People always complain about potholed roads during migrating from India to developed countries for better
the monsoon months. opportunities.
(e) None of these (e) None of these
34. It has been reported in recent years that a very large number 37. The govt. has appealed to all citizens to use potable water
of seats in the engineering colleges in the country remain judiciously as there is an acute shortage in supply. Excessive
vacant at the end of the admission session. use may lead to huge scarcity in future months.
Which of the following may be the probable cause of the Which of the assumptions is implicit in the above
above effect? statement?
(a) There has been a considerable decrease in hiring of (An assumption is something supposed or taken for
engineering graduates due to economic slowdown in granted)
the recent years. (a) People may ignore the appeal and continue using water
(b) Students have always preferred to complete graduation as per their consideration.
in three years time instead of four years for engineering. (b) Govt. may be able to tap those who do not respond to
(c) The Govt. has recently decided to provide post the appeal.
qualification professional training to all engineering (c) Govt. may be able to put in place alternate sources of
graduates at its own cost. water in the event of a crisis situation.
(d) There has always been a very poor success rate among (d) Large number of people may positively respond to the
the engineering students. govt.'s appeal and help tide over the crisis.
(e) None of these (e) Only poor are going to suffer from this shortage of
35. Majority of the students who appeared in the final water supply.
examination of post graduate course in management in the 38. A few travellers were severely beaten up by villagers
local college have secured first class which is comparatively recently in a remote rural part of the state as the villagers
higher than the performance of students of other found the movement of the travellers suspicious. The
management colleges in the state. district authority has sent a police team to nab the culprits.
Which of the following may indicate that the results are not Which of the following inferences can be drawn from the
in line with the general trend? above statement?
(a) The students of the local college are qualitatively better (An inference is something which is not directly stated but
than those of other colleges can be inferred from the given facts)
(b) The authorities of the other management colleges in (a) The villagers dislike presence of strangers in their
the state are stricter in their standard of evaluation for vicinity.
their students. (b) Villagers are generally suspicious in nature.
(c) The students of other management colleges in the state (c) Travellers prefer to visit countryside.
y
o
u
rs
m
IBPS PO/MT Exam 2011 99

a
h
b
(d) The Govt. generally provides protection to travellers 40. There is an alarming trend of skewed sex ratio against

o
o
across the country. women in India during the past decade and situation may

b
.w
(e) None of these get out of hand if adequate steps are not taken to stop
female foeticide.

o
39. There has been a considerable drop in sales of four wheelers

rd
during the past six months when compared to the number Which of the following can be an effective step to reverse

p
re
of four wheelers sold during this period last year. the trend?

s
Which of the following can the probable cause of the above (A) The Govt. should immediately completely ban the use

s
.c
phenomenon? of scanners / sonography on expectant mothers at all

o
m
(A) The govt. has imposed higher excise duty on four health centres.
wheelers at the beginning of this year. (B) The Govt. should announce a substantial incentive
(B) The petrol prices have risen considerably during the scheme for couples who have at least one girl child.
past eight months. (C) The Govt. should launch a nationwide campaign to
(C) The rate of interest on home and car loans have been create awareness against female foeticide.
rising for the past seven months. (a) (A) only (b) (A) and (B) Only
(a) All (A), (B) and (C) (b) (A) and (C) Only (c) (B) and (C) Only (d) All (A), (B) and (C)
(c) (B) and (C) Only (d) (B) Only (e) None of these
(e) (A) Only

DIRECTIONS (Qs. 41-45) : The first figure in the first unit of the problem figures bears a certain relationship to the second figure.
Similarly one of the figures in the answer figures bears the same relationship to the second figure in the second unit of the problem
figures. You are therefore to locate the figure which would fit in the question mark.

PROBLEM FIGURES ANSWER FIGURES

41. ?
(a) (b) (c) (d) (e)

42. ?
(a) (b) (c) (d) (e)

43. ?

(a) (b) (c) (d) (e)

44. ?
(a) (b) (c) (d) (e)

T x S C C C C S
45. C T x x
x x S ? x x T T
S C C T T x S T
(a) (b) (c) (d) (e)
y
o
u
rs
m
100 IBPS PO/MT Exam 2011

a
h
b
Directions (Qs. 46-50) : In each of the questions given below which one of the five answer figures on the right should come after the

o
problem figures on the left, if the sequence were continued?

o
b
.w
PROBLEM FIGURES ANSWER FIGURES

o
rd
x C C T S S

p
46. S S T T U T T

re
Z S Z S

s
T C T S

s
.c
(a) (b) (c) (d) (e)

o
m
S S S C C C C S
47. C S C
C C C S S S S S C
(a) (b) (c) (d) (e)
x x x C S
S
T

48. C C
C
C
x x S x x x x
(a) (b) (c) (d) (e)

49.

(a) (b) (c) (d) (e)


x C
x C C
C C CT T S T
50. S x
x
(a) (b) (c) (d) (e)

Quantitative Aptitude 55. (23.1)2 + (48.6)2 – (39.8)2 = ? + 1147.69


(a) (13.6)2 (b) 12.8
DIRECTIONS (Qs. 51-55) : What will come in place of the
(c) 163.84 (d) 12.8
question mark (?) in the following questions ?
(e) None of these
11449 ´ 6241 - ( 54 ) = ? + ( 74 )
2 2
51. DIRECTIONS (Qs. 56-60) : What approximate value should come
(a) 384 (b) 3721 in place of the question mark (?) in the following questions?
(c) 381 (d) 3638 (Note : You are not expected to calculate the exact value.)
(e) None of these
3
56. 4663 + 349 = ? ¸ 21.003
52. ë ( ) (
é 3 8 + 8 ´ 8 8 + 7 8 ù - 98 = ?
û ) (a) 7600 (b) 7650
(a) 2 8 (b) 8 8 (c) 7860 (d) 7560
(e) 7680
(c) 382 (d) 386
(e) None of these 57. 39.897% of 4331 + 58.779% of 5003 = ?
53. 3463 ´ 295 – 18611 = ? + 5883 (a) 4300 (b) 4500
(a) 997091 (b) 997071 (c) 4700 (d) 4900
(c) 997090 (d) 999070 (e) 5100
(e) None of these 58. 59.88 ¸ 12.21 ´ 6.35 = ?
28 195 39 5 (a) 10 (b) 50
54. ´ ¸ + =? (c) 30 (d) 70
65 308 44 26
(e) 90
1
(a) (b) 0.75 59. 43931.03 ¸ 2111.02 ´ 401.04 = ?
3
(a) 8800 (b) 7600
1 1
(c) 1 (d) (c) 7400 (d) 9000
2 2
(e) 8300
(e) None of these
y
o
u
rs
m
IBPS PO/MT Exam 2011 101

a
h
b
60. 6354 ´ 34.993 = ? 69. In a test, a candidate secured 468 marks out of maximum

o
o
marks 'A'. If the maximum marks 'A' were converted to 700

b
(a) 3000 (b) 2800

.w
marks, he would have secured 336 marks. What were the
(c) 2500 (d) 3300
maximum marks of the test?

o
(e) 2600

rd
(a) 775 (b) 875

p
re
DIRECTIONS (Qs. 61–65) : In the following number series only (c) 975 (d) 1075

s
one number is wrong. Find out the wrong number. (e) None of these

s
.c
70. The simple interest accrued on an amount of ` 22,500 at the

o
61. 9050 5675 3478 2147 1418 1077 950

m
(a) 3478 (b) 1418 end of four years is ` 10,800. What would be the compound
interest accrued on the same amount at the same rate at the
(c) 5675 (d) 2147
end of two years ?
(e) 1077
(a) ` 16,908 (b) ` 5,724
62. 7 12 40 222 1742 17390 208608
(c) ` 28,224 (d) ` 8,586
(a) 7 (b) 12
(e) None of these
(c) 40 (d) 1742 71. Rubina could get equal number of ` 55, ` 85 and ` 105
(e) 208608 tickets for a movie. She spents ` 2940 for all the tickets.
63. 6 91 584 2935 11756 35277 70558 How many of each did she buy?
(a) 91 (b) 70558 (a) 12 (b) 14
(c) 584 (d) 2935 (c) 16 (d) Cannot be determined
(e) 35277 (e) None of these
64. 1 4 25 256 3125 46656 823543 72. Ramola's monthly income is three times Ravina's monthly
(a) 3125 (b) 823543 income. Ravina's monthly income is fifteen percent more
(c) 46656 (d) 25 than Ruchira's monthly income. Ruchira's monthly income
is ` 32,000. What is Ramola's annual income ?
(e) 256
(a) ` 1,10,400 (b) ` 13,24,800
65. 8424 4212 2106 1051 526.5 263.25 131.625
(c) ` 36,800 (d) ` 52,200
(a) 131.625 (b) 1051
(e) None of these
(c) 4212 (d) 8424
73. In an Entrance Examination Ritu scored 56 percent marks,
(e) 263.25 Smita scored 92 percent marks and Rina scored 634 marks.
66. Six-eleventh of a number is equal to twenty-two percent of The maximum marks of the examination are 875. What are
second number. Second number is equal to the one-fourth the average marks scored by all the three girls together?
of third number. The value of the third number is 2400. What (a) 1929 (b) 815
is the 45% of first number? (c) 690 (d) 643
(a) 109.8 (b) 111.7 (e) None of these
(c) 117.6 (d) 123.4 74. The respective ratio between the present age of Manisha
(e) None of these and Deepali is 5 : X. Manisha is 9 years younger than
67. An HR Company employes 4800 people, out of which 45 Parineeta. Parineeta's age after 9 years will be 33 years. The
percent are males and 60 percent of the males are either difference between Deepali's and Manisha's age is same as
25 years or older. How many males are employed in that HR the present age of Parineeta. What will come in place of X?
Company who are younger than 25 years ? (a) 23 (b) 39
(c) 15 (d) Cannot be determined
(a) 2640 (b) 2160
(e) None of these
(c) 1296 (d) 864
75. Seema bought 20 pens, 8 packets of wax colours,
(e) None of these
6 calculators and 7 pencil boxes. The price of one pen is
68. The average marks in English subject of a class of 24
` 7, one packet of wax colour is ` 22, one calculator is
students is 56. If the marks of three students were misread ` 175 and one pencil box is ` 14 more than the combined
as 44, 45 and 61 of the actual marks 48, 59 and 67 respectively, price of one pen and one packet of wax colours. How much
then what would be the correct average? amount did Seema pay to the shopkeeper?
(a) 56.5 (b) 59 (a) ` 1,491 (b) ` 1,725
(c) 57.5 (d) 58 (c) ` 1,667 (d) ` 1,527
(e) None of these (e) None of these
y
o
u
rs
m
102 IBPS PO/MT Exam 2011

a
h
b
DIRECTIONS (Qs. 76-80) : Study the given information carefully 81. What is the respective ratio of number of women visiting

o
Super-markets A to those visiting Super-market C?

o
to answer the questions that follow.

b
(a) 35 : 37 (b) 245 : 316

.w
An urn contains 4 green, 5 blue, 2 red and 3 yellow marbles. (c) 352 : 377 (d) 1041 : 1156

o
76. If four marbles are drawn at random, what is the probability

rd
(e) None of these
that two are blue and two are red?

p
82. Number of men visiting Super-market D forms approximately

re
10 9 what percent of the total number of people visiting all the

s
(a) (b)

s
1001 14 Super-markets together?

.c
(a) 11 (b) 5.5

o
17 2

m
(c) (d) (c) 13 (d) 9
364 7 (e) 7.5
(e) None of these
83. Number of children visiting Super-market C forms what
77. If eight marbles are drawn at random, what is the probability
percent of number of children visiting super market F?
that there are equal number of marbles of each colour ?
(rounded off to two digits after decimal)
4 361 (a) 91.49 (b) 49.85
(a) (b)
7 728 (c) 121.71 (d) 109.30
60 1 (e) None of these
(c) (d) 84. What is the total number of children visiting Super-markets
1001 1
(e) None of these B and D together?
78. If two marbles are drawn at random, what is the probability (a) 18515 (b) 28479
that both are red or at least one is red ? (c) 31495 (d) 22308
26 1 (e) None of these
(a) (b) 85. What is the average number of women visiting all the Super-
91 7
markets together?
199 133 (a) 24823.5 (b) 22388.5
(c) (d)
364 191 (c) 26432.5 (d) 20988.5
(e) None of these (e) None of these
79. If three marbles are drawn at random, what is the probability
that at least one is yellow ? DIRECTIONS (Qs. 86-90) : Study the following pie-chart and
answer the questions given below:
1 199
(a) (b) Preferences of students among six beverages in terms of
3 364
degree of angle in the pie-chart
165 3
(c) (d) Total no. of students = 6800
364 11
(e) None of these
80. If three marbles are drawn at random, what is the probability
that none is green ?
Beverage E,
2 253 64.8°
(a) (b)
7 728
10 14
(c) (d)
21 91
(e) None of these
DIRECTIONS (Qs. 81-85) : Study the table carefully to answer
the questions that follow : 86. The number of students who prefer Beverage C are
approximately what percent of the number of students who
Number of people visiting six different Super-markets prefer Beverage D ?
and the percentage of men, women and children (a) 7 (b) 12 (c) 18
visiting those Super-markets. (d) 22 (e) 29
87. How many students prefer Beverage B and Beverage E
Names of the Total number Percentage of together ?
Super-markets of people Men Women Children (a) 2312 (b) 2313 (c) 2315
A 34560 35 55 10 (d) 2318 (e) None of these
B 65900 37 43 20 88. What is the difference between the total number of students
C 45640 35 45 20 who prefer Beverage A and C together and the total number
D 55500 41 26 33 of students who prefer Beverage D and F together?
E 42350 6 70 24 (a) 959 (b) 955 (c) 952
F 59650 24 62 14 (d) 954 (e) None of these
y
o
u
rs
m
IBPS PO/MT Exam 2011 103

a
h
b
89. The number of students who prefer Beverage E and F 90. What is the respective ratio between the number of students

o
together are what percent of the total number of students? who prefer Beverage F and the number of students who

o
b
(a) 18 (b) 14 (c) 26 prefer Beverage A ?

.w
(d) 24 (e) None of these (a) 3 : 11 (b) 3 : 13 (c) 6 : 11

o
(d) 5 : 11 (e) None of these

rd
p
re
DIRECTIONS (Qs. 91-95) : Study the table carefully to answer the questions that follow :

s
s
Percentage of Marks Obtained by Different Students in Different Subjects of MBA

.c
o
SUBJECTS (Maximum Marks)

m
Strategic Brand Compensation Consumer Service Training &
Students Management Management Management Behaviour Marketing Development
(150) (100) (150) (125) (75) (50)

Anushka 66 75 88 56 56 90
Archit 82 76 84 96 92 88
Arpan 76 66 78 88 72 70
Garvita 90 88 96 76 84 86
Gunit 64 70 68 72 68 74
Pranita 48 56 50 64 64 58
91. How many marks did Anushka get in all the subjects together?
(a) 369 (b) 463 (c) 558 (d) 496 (e) None of these
92. Who has scored the highest total marks in all the subjects together ?
(a) Archit (b) Gunit (c) Pranita (d) Garvita (e) Arpan
93. Marks obtained by Garvita in Brand Management are what percent of marks obtained by Archit in the same subject ? (rounded
off to two digits after decimal)
(a) 86.36 (b) 101.71 (c) 115.79 (d) 133.33 (e) None of these
94. How many students have scored the highest marks in more than one subject ?
(a) Three (b) Two (c) One (d) None (e) None of these
95. What are the average marks obtained by all students together in Compensation Management ?
(a) 116 (b) 120 (c) 123 (d) 131 (e) None of these
DIRECTIONS (Qs. 96-100) : Study the following graph and answer the questions given below :
No. of students (in thousands) who opted for three different specializations during the given
five years in a university
40
35
30
Number of Students

Hindi
(in thousands)

25
20 English

15 Mathematics
10

5
0
2005 2006 2007 2008 2009
Years
96. The total number of students who opted for Mathematics in the years 2005 and 2008 together are approximately what percent of
the total number of students who opted for all three subjects in the same years ?
(a) 38 (b) 28 (c) 42 (d) 32 (e) 48
97. Out of the total number of students who opted for the given three subjects, in the year 2009, 38% were girls. How many boys
opted for Mathematics in the same year ?
(a) 1322 (b) 1332 (c) 1312 (d) Cannot be determined (e) None of these
y
o
u
rs
m
104 IBPS PO/MT Exam 2011

a
h
b
98. What is the respective ratio between the number of students northern hemisphere oceans were not warm enough to allow

o
who opted for English in the years 2006 and 2008 together rain formation. Global dimming is also hiding the true power

o
b
and the number of students who opted for Hindi in the year of global warming. By cleaning up global dimming-causing

.w
2005 and 2009 together ? pollutants without talking greenhouse gas emissions, rapid

o
(a) 11 : 5 (b) 12 : 7 (c) 11 : 7 warming has been observed, and various human health and

rd
(d) 12 : 5 (e) None of these ecological disasters have resulted, as witnessed during the

p
re
99. If the total number of students in the university in the year European heat wave in 2003, which saw thousands of people

s
die.

s
2007 was 455030, then , the total number of students who

.c
opted for the given three subjects were approximately what (a) This though, does not bring any relief in the problems

o
associated with climate change.

m
percent of the total students ?
(a) 19 (b) 9 (c) 12 (b) This phenomenon thus is part of the climate change
(d) 5 (e) 23 problem.
(c) Scientists thus believe that this phenomenon goes
100. What is the total number of students who opted for Hindi
hand in hand with global warming
and who opted for Mathematics in the years 2006, 2007 and
(d) At first, it sounds like an ironic savior to climate change
2009 together ? problems
(a) 97000 (b) 93000 (c) 85000 (e) The answer to all our problems with respect to climate
(d) 96000 (e) None of these change is definitely here
103. Poverty is the state for the majority of the world’s people
English Language and nations. Why is this? .......... Have they been lazy, made
poor decisions, and been solely responsible for their own
DIRECTIONS (Qs. 101-105): In each of the following questions plight? What about their governments? Have they pursued
a short passage is given with one of the lines in the passage policies that actually harm successful development? Such
missing and represented by a blank. Select the best out of the causes of poverty and inequality are no doubt real. But
five answer choices given, to make the passage complete and deeper and more global causes of poverty are often less
coherent. discussed. Behind the increasing interconnectedness
promised by globalization are global decisions, policies,
101. Women’s rights around the world are an important indicator and practices. These are typically influenced, driven, or
to understand global well-being. A major global women’s formulated by the rich and powerful. These can be leaders
rights treaty was ratified by the majority of the world’s of rich countries or other global actors such as multinational
nations a few decades ago. ...... These ranges from the corporations, institutions, and influential people. In the face
cultural, political to the economic. For example, women often of such enormous external influence, the governments of
work more than men, yet are paid less; gender discrimination poor nations and their people are often powerless. As a
affects girls and women throughout their lifetime; and result, in the global context, a few get wealthy while the
women and girls are often the ones that suffer the most majority struggles.
poverty. Many may think that women’s rights are only an (a) Is it enough to blame poor people for their own
issue in countries where religion is law. Or even worse, predicament?
some may think this is no longer an issue at all. But reading (b) What is the government doing about it?
the report about the United Nation’s Women’s Treaty and (c) Are the wealthy ones in the nation even aware of this?
how an increasing number of countries are lodging (d) The government has already taken measures to
reservations will show otherwise. Gender equality furthers eradicate the same.
(e) The huge gap between the rich and the poor in the
the cause of child survival and development for all of
nation is now narrowing
society, so the importance of women’s rights and gender
104. Analysts and industry pundits forecast that notebook
equality should not be underestimated. market, which has been growing faster than the desktop
(a) This treaty tackled and solved a number of issues market for the past three years, is expected to overtake the
related to women. desktop market by the year 2011-12. A fall in prices, large
(b) Why is it then, that moment still face a number of deals from governments and institutions, and demand from
problems on the domestic front? consumers and sectors such as education are expected to
(c) Thus, the woman today is ten times more empowered help the notebook numbers. According to research agencies,
as compared to a woman say about a decade ago. the year 2010-11 saw notebook volumes rise, and for the
(d) Women’s activists across nations have implored the first time a million plus notebooks were sold in India in a
respective governments to take this seriously. single quarter. The market has grown nearly four times for
(e) Yet, despite many successes in empowering women, notebooks. The demand is driven by all sectors and a very
numerous issues still exist in all areas of life. buoyant consumer market, which prefers mobile computers.
102. Research has shown that air pollutants from fossil fuel use Entry-level notebook prices have dropped below the Rs.
make clouds reflect more of the sun’s rays back into space. 25,000 mark; this has helped break the ice with new
This leads to an effect known as global dimming whereby customers. This drop in notebook prices has been helped
less heat and energy reaches the earth. ............ However, it by the drop in the prices of the building blocks that make a
is believed that global dimming caused the droughts in notebook. It’s simple. With notebook volumes growing, the
certain parts of the world where millions died, because the prices of the components are also bound to come
down. .........
y
o
u
rs
m
IBPS PO/MT Exam 2011 105

a
h
(a) All this has resulted in a noticeable change in a number

b
106. Which of the following should be the FIRST sentence after

o
of large government tenders for notebooks; which were rearrangement?

o
b
traditionally for desktops. (a) 7 (b) 2

.w
(b) Because of this the government still prefers desktops (c) 3 (d) 4

o
to notebooks and has passed tenders for the same.

rd
(e) 5
(c) Thereby making them more expensive.

p
107. Which of the following should be the SEVENTH (LAST)

re
(d) Thus the forecast for the coming year states that
sentence after rearrangement?

s
desktops will be the preferred technology choice only

s
(a) 1 (b) 2

.c
for consumers who cannot afford the exorbitantly

o
priced notebook. (c) 3 (d) 4

m
(e) Thus notebooks will become obsolete after a decade (e) 5
or so. 108. Which of the following should be the FIFTH sentence after
105. Next to China, India is the most populated country in the rearrangement?
world. ...... Particularly, rush to technical and higher (a) 1 (b) 2
education has increased as the scope for arts and science (c) 3 (d) 6
has become lesser and lesser due to lack of reforms and up (e) 5
gradation in the course structure and materials according 109. Which of the following should be the SECOND sentence
to the developments of the world. Also, qualification in after rearrangement?
higher education gives added advantage to face
(a) 1 (b) 2
successfully competition in the job market.
(a) Keeping this in mind, the government has provided (c) 3 (d) 4
concessions in the admission fees for the arts and (e) 6
science streams in the country. 110. Which of the following should be the THIRD sentence
(b) Naturally there is too much rush and competition in after rearrangement?
every field. (a) 1 (b) 2
(c) Despite this the rush to higher education is lesser. (c) 7 (d) 4
(d) This population increase, though, has not kept pace (e) 5
with the knowledge expansion around the world.
DIRECTIONS (Qs. 111-115) : The following questions consist
(e) In the next decade it will become the most populous.
of a single sentence with one blank only. You are given six words
DIRECTIONS (Qs. 106-110) : Rearrange the following seven as answer choices and from the six choices you have to pick up
sentences (1), (2), (3), (4), (5), (6) and (7) in the proper two correct answers, either of which will make the sentence
sequence to from a meaningful paragraph; then answer the meaningfully complete.
questions given below them.
111. The ability of a woman to do well does not .......... on whether
(1) To elaborate briefly on these characteristics and dimensions it is a man’s world or not, because everyone has his/her
that the author is talking about – NRMs are general tests
own opportunities.
intended to be used to classify students by percentile for
measuring either aptitude or proficiency for admissions into (1) trust (2) depend
or placement within a program. (3) reckon (4) live
(2) Contrastingly, the CRM, such as a locally produced (5) rest (6) believe
achievement test, measures absolute performance that is (a) (4) and (5) (b) (2) and (3)
compared only with the learning objective, hence a perfect (c) (1) and (6) (d) (2) and (5)
score is theoretically obtainable by all students who have a (e) (3) and (4)
mastery of the pre-specified material, or conversely, all 112. Drugs worth ` 3 lakhs were ......... from the apartment by the
students may fail the test. police.
(3) In most of these books, the authors classify a measurement (1) manufactured (2) ruptured
strategy as either norm-referenced (NRM) or criterion- (3) seized (4) confiscated
referenced (CRM).
(5) bought (6) compared
(4) Another author points out how the type of interpretation
(a) (1) and (4) (b) (2) and (3)
that an NRM offers is the relative performance of the
students compared with that of all the others resulting in, (c) (3) and (5) (d) (5) and (6)
ideally, a bell curve distribution. (e) (3) and (4)
(5) Numerous books on constructing and using language tests 113. An organization .......... to the mission of road safety has
have been written by various authors. prepared an action plan for reducing accidents and related
(6) CRMs, on the other hand, are more specific, achievement injuries and fatalities.
or diagnostic tests intended to be used for motivating (1) specified (2) inaugurated
students by measuring to what percent they have achieved (3) committed (4) kicked off
mastery of the thought or learned material. (5) succumbed (6) dedicated
(7) One of the authors clearly delineates the differences of these (a) (3) and (6) (b) (1) and (5)
two types by focusing on the categories of “test (c) (3) and (5) (d) (4) and (6)
characteristics” and “logistical dimensions.”
(e) (1) and (3)
y
o
u
rs
m
106 IBPS PO/MT Exam 2011

a
h
b
114. A man reportedly .......... two passports with the same DIRECTIONS (Qs. 121-135): In the following passage there

o
o
photograph, but under different names was arrested by the are blanks, each of which has been numbered. These numbers

b
are printed below the passage and against each, five words are

.w
commissioner’s Task Force.
(1) possessing (2) examining suggested, one of which fits the blank appropriately. Find out

o
rd
(3) surrendering (4) mastering the appropriate word in each case.

p
re
(5) holding (6) fixating As the country embarks on planning (121) the 12th Plan (2012-

s
(a) (2) and (3) (b) (3) and (6) 17) period, a key question mark (122) hangs over the process is

s
.c
(c) (1) and (5) (d) (1) and (4) on the energy requirements.

o
Growth is energy hungry, and the aspirations of growing at 9-

m
(e) (4) and (5)
10% will (123) huge demands on the energy resources of the
115. The Hollywood star and the Bollywood heroine are being
country. In this energy Jigsaw, renewable energy will (124) like
......... as the next big onscreen couple. never before in the 12th Plan and the (125). By the rule of the
(1) labeled (2) explained thumb, India will (126) about 100 gigawatts (Gw)-100,000
(3) worshiped (4) touted megawatts-of capacity addition in the next five years. Encouraging
(5) exclaimed (6) shouted trends on energy efficiency and sustained (127) by some parts
(a) (2) and (4) (b) (1) and (3) of the government the Bureau of Energy Efficiency in particular
(c) (2) and (6) (d) (1) and (4) needs to be complimented for this-have led to substantially lesser
energy intensity of economic growth. However, even the tendered
(e) (3) and (4)
demand numbers are (128) to be below 80Gw. As against this
DIRECTIONS (Qs. 116-120) : Below is given a single word need the coal supply from domestic sources is unlikely to support
with options to its meaning in different contexts. You have to more than 25 Gw equivalent capacity. Imported coal can add some
select all those options which are synonyms of the word when more, but at a much (129) cost. Gas-based electricity generation
the context is changed. Select the correct alterative from (a), is unlikely to contribute anything substantial in view of the
(b), (c), (d) and (e) which represents all those synonyms. unprecedented gas supply challenges. Nuclear will be (130) in
116. MASK the foreseeable future. Between imported coal, gas, large hydro
(1) Cover (2) Hide and nuclear, no more than 15-20Gw equivalent can be (131) to be
added in the five-year time block. (132) (133) this, capacity
(3) Conceal (4) Disguise
addition in the renewable energy based power generation has
(a) Only (1) (b) Both (2) and (4)
touched about 3Gw a year. In the coming five years, the overall
(c) Only (2), (3) and (4) (d) Only (1), (2) and (3) capacity addition in the electricity grid (134) renewable energy is
(e) All (1), (2), (3) and (4) likely to range between 20Gw and 25Gw. Additionally, over and
117. REGULAR above the grid-based capacity, off-grid electricity applications
(1) Present (2) Common are reaching remote places and (135) lives where grid-based
(3) Indiscriminate (4) Uniform electricity supply has miserably failed.
(a) Only (4) (b) Both (2) and (4) 121. (a) On (b) Against
(c) Both (1) and (3) (d) Only (2), (3) and (4) (c) For (d) Onwards
(e) All (1), (2), (3) and (4) (e) At
118. LABOUR 122. (a) Where (b) That
(1) Expedite (2) To move faster (c) Inside (d) Always
(e) Who
(3) Controlled (4) Toil
123. (a) Replace (b) Forward
(a) Only (4) (b) Both (1) and (3)
(c) Subject (d) Place
(c) Both (2), (3) and (4) (d) Only (1), (3) and (4) (e) Demand
(e) All (1), (2), (3) and (4) 124. (a) Light (b) Pass
119. MEAN (c) Publish (d) Feature
(1) Imply (2) Understand (e) Find
(3) Average (4) Characterized by 125. (a) Earlier (b) Likewise
malice (c) Publicity (d) Next
(a) Only (3) (b) Both (1) and (4) (e) After
(c) Only (1), (3) and (4) (d) Only (1), (2) and (4) 126. (a) Consumed (b) Waste
(e) All (1), (2), (3) and (4) (c) Require (d) Highlight
120. ALONE (e) Generate
127. (a) Developmental (b) Structures
(1) Exclusively (2) Morose
(c) Efforts (d) Projections
(3) Solitary (4) Human being
(e) Practices
(a) Only (1) (b) Both (1) and (3) 128. (a) Likely (b) Sure
(c) Both (2) and (3) (d) Only (1), (3) and (4) (c) Unsure (d) Unexpected
(e) All (1), (2), (3) and (4) (e) Unlikely
y
o
u
rs
m
IBPS PO/MT Exam 2011 107

a
h
b
129. (a) Expected (b) Nominal unreformed and largely under state control, which creates

o
(c) Excelled (d) Higher difficulties. Clearances today can take three to four years and

o
b
(e) Lower many employers are keen to replace workers with machines despite

.w
130. (a) Marginal (b) Failure an abundance of labor force. This can be attributed to labor laws

o
(c) Success (d) Dangerous which are inimical to employee creation and an education system

rd
(e) Maximum that means finding quality manpower a major problem. In fact the

p
re
131. (a) Sure (b) Certain Planning Commission concluded that even achieving 9 percent

s
(c) Linked (d) Remarked growth will need marked policy action in unreformed sectors.

s
.c
(e) Expected Twenty years age it was said that yardstick against which India

o
m
132. (a) As (b) When should be measured was its potential and it is clear that there
(c) But (d) However remains much to do.
(e) If 136. Which of the following can be said about the Indian
133. (a) Against (b) For economy at present?
(c) With (d) Is (a) It can comfortably achieve double digit growth rare at
(e) Ever present.
134. (a) Capacity (b) Through (b) High food prices have led to overheating of the economy.
(c) Project (d) Versus (c) Citizens are affluent owing to laxity in regulation.
(e) Against (d) Private sector confidence in India’s growth potential is high.
135. (a) Generating (b) Lightening (e) Unreformed sectors are a drag on economic growth.
(c) Making (d) Touching 137. Why are employers reluctant to hire Indian labour force?
(e) Saving (1) India’s labour force is overqualified for the employment
DIRECTIONS (Qs. 136-143) : Read the following passage opportunities available.
carefully and answer the questions given below it. Certain words (2) High attrition rate among employees stemming from
are printed in bold to help you to locate them while answering their entrepreneurial spirit.
some of the questions. (3) Labour laws are not conducive to generating
In a reversal of the norm elsewhere, in India policymakers and emploment.
economists have become optimists while bosses do the worrying. (a) Only (3)
The country’s Central Bank has predicted that the country’s (b) All (1), (2) and (3)
economy is likely to grow at a double digit rate during the next 20- (c) Only (1) and (3)
30 years. India has the capability with its vast labour and lauded (d) Only (1) and (2)
entrepreneurial spirit. But the private sector which is supposed (e) None of these
to do the heavy lifting that turns India from the world’s tenth 138. What is the state of India’s basic input sectors at present?
largest economy to its third largest by 2030 has become fed up. (a) These sectors attract Foreign Direct Investment
Business people often carp about India’s problems but their because of their vast potential.
irritation this time has a nervous edge. In the first quarter of 2011, (b) These sectors are lagging as projects are usually
GDP grew at an annual rate of 7.8 percent; in 2005-07 it managed awarded to foreign companies.
9-10 percent. The economy may be slowing naturally as the low (c) These sectors are stagnating and badly in need of
interest rates and public spending that got India through the reforms.
global crisis are belatedly withdrawn. At the same time the surge (d) These sectors are well regulated as these are governed
in inflation caused by exorbitant food prices has spread more by the State.
widely, casting doubt over whether India can grow at 8-10 percent (e) None of these
in the medium term without overheating. 139. What is the author’s main objective in writing the passage?
In India, as in many fast growing nations, the confidence to invest (a) Showcasing the potential of India’s growth potential
depends on the conviction that the long term trajectory is intact to entice foreign investors .
and it is that which is in doubt. Big Indian firms too sometimes (b) Exhorting India to implement measures to live up to its
seem happier to invest abroad than at home, in deals that are potential.
often hailed as symbols of the country’s growing clout but (c) Recommending India’s model of development to other
sometimes speak to its weaknesses – purchases of natural developing countries
resources that India has in abundance but struggles to get out of (d) Berating the private sector for not bidding for
the ground. In fact a further dip in investment could be self infrastructure development projects.
fulfilling: if fewer roads, ports and factories are built, this will hurt (e) Criticising the measures taken by India during the global
both short term growth figures and reduce the economy’s long economic crisis.
term capacity. 140. What impact has the GDP growth of 7.8 percent had?
There is a view that because a fair amount of growth is assured (1) Indian Industry is anxious about India’s economic
the government need not try very hard. The liberalization reforms growth.
that began in 1991 freed markets for products and gave rise to (2) India has achieved status as the world’s third largest
vibrant competition, at the same time what economists call factor economy at present.
markets, those for basic inputs like land, power, labour etc remain (3) Foreign investment in India has drastically increased.
y
o
u
rs
m
108 IBPS PO/MT Exam 2011

a
h
b
(a) Only (1) causation. The general trend over the past half century may have

o
(b) All (1), (2) and (3) been towards rising living standards, a wider spread of basic

o
b
(c) Only (1) and (3) education and more democracy, but it is entirely possible that

.w
(d) Only (1) and (2) this is being by another variable. Even if the correlation were not

o
(e) None of these spurious, it would be difficult to know which way causation ran.

rd
141. Which of the following is most similar in meaning to the Does more education lead to greater democracy? Or are more

p
re
word CLOUT given in bold as used in the passage? democratic countries better at educating their citizens? A recent

s
(a) Strike NBER paper compared a group of Kenyan girls in 69 primary

s
.c
(b) Standing school whose students were randomly selected to receive a

o
m
(c) Force scholarship with similar students in schools which received no
(d) Launch such financial aid. Previous studies has shown that the
(e) Achieve scholarship programme led to higher test scores and increased
142. Which of the following is most opposite in meaning to the the likelihood that girls enrolled in secondary school. Overall, it
word MARKED given in bold as used in the passage? significantly increased the amount of education obtained. For
(a) Decreased the new study the authors tried to see how the extra schooling
(b) Ignored had affected the political and social attitudes of the women in
(c) Clear question. Findings suggested that education may make people
(d) Assessed more interested in improving their own lives but they may not
(e) Imperceptible necessarily see democracy as the way to do it. Even in established
143. What measures do experts suggest be taken to ensure democracies, more education does not always mean either more
targeted economic growth? active political participation or greater faith in democracy. Poorer
(a) Loweing of interest rates to help industries hit by and less educated people often vote in larger numbers than their
recession. more educated compatriots, who often express disdain for the
(b) Prolonged financial support for basic input industries. messiness of democracy, yearning for the kind of government
(c) Incentives to Indian companies to invest in that would deal strongly with the corrupt and build highways,
infrastucture. railway lines and bridges at a dizzying pace of authoritarian China.
(d) Formulation of policies and their implementation in 144. Which of the following most aptly describes the central
factor markets theme of the passage?
(e) Stringent implementation of licensing system.
(a) Democratic nations are richer and have a better track
DIRECTIONS (Qs. 144-150) : Read the following passage record of educating their citizens.
carefully and answer the questions given below it. Certain words/
(b) Education does not necessarily lead to greater
phrases are printed in bold to help you to locate them while
answering some of the questions. enthusiasm for a democratic form of government
(c) Educated societies with autocratic form of government
In many countries, a combustible mixture of authoritarianism,
enjoy a better quality of life than democracies.
unemployment and youth has given rise to disaffection with
strongmen rulers which has in turn spill over into uprising. Young (d) Citizens can fulfill their personal aspirations only under
people in these countries are far better educated than their parents a democratic form of government.
were. In 1990 the average Egyptian had 4.4 years of schooling; (e) Democracy makes citizens more intolerant as it does
by 2010 the figure had risen to 7.1 years. Could it be that education, not restrict personal freedoms
by making people less willing to put up with restrictions on 145. Which of the following is most similar in meaning to the
freedom and more willing to question authority, promotes
word PROMOTES given in bold as used in the passage?
democratization. Ideas about the links between education, Income
and democracy are at the heart of what social scientists have (a) Up grades (b) Prefers
long studied. Since then plenty of economists and political (c) Recommends (d) Advocates
scientists have looked for statistical evidence of a causal link (e) Publicizes
between education and democratization. Many have pointed to 146. What conclusion can be drawn from the statistics cited
the strong correlation that exists between levels of education about Egypt’s education system?
and measures like the pluralism of party politics and the existence
(a) Job prospects have been on the rise in Egypt in recent
of civil liberties. The patterns are similar when income and
democracy are considered. There are outliers, of course – until times.
recently, many Arab countries managed to combine energy-based (b) Authoritarian leaders have played a vital role in
wealth and decent education with undemocratic political systems. reforming Egypt’s education system.
But some deduce from the overall picture that as China and other (c) Egypt has one of the youngest and best educated
authoritarian states get more educated and richer, their people demographies in the world.
will agitate for greater political freedom, culminating in a shift to a (d) Egypt is likely to be successful vibrant democracy.
more democratic form of government.
(e) There has been a rise in education levels in Egypt in
This apparently reasonable intuition is shakier than it seems.
Critics of the hypothesis point out that correlation is hardly recent times.
y
o
u
rs
m
IBPS PO/MT Exam 2011 109

a
h
b
147. In the context of the passage which of the following 153. Interest payable on saving bank accounts is

o
characterize (s) democracies? (a) not regulated by RBI.

o
b
(1) Active participation of majority of educated citizens in (b) regulated by State Governments

.w
electoral process. (c) regulated by Central Government

o
(2) Fast paced economic growth and accountability of

rd
(d) regulated by RBI

p
those in power.
(e) regulated by Finance minister.

re
(3) Better standards of living and access to higher

s
154. Which of the following is the correct statement?

s
education.

.c
(a) All (1), (2) and (3) (b) Only (2) and (3) (a) State bank of India is the sole authority to issue and

o
manage currency in India.

m
(c) Only (3) (d) Only (1) and (2)
(e) None of these (b) A nationalized bank is the sole authority to Issue and
148. What according to the author has led to uprisings in manage currency in India
authoritarian countries? (c) A cooperative bank is the sole authority to issue and
(a) Lack of access to education. manage currency in India.
(b) Vast numbers of uneducated and unemployable youth. (d) RBI is the sole authority to issue and manage currency
(c) Frustration with the existing system of governance. in India.
(d) Unavailability of natural energy resources like coal and (e) None of these
oil. 155. The Home Minister of India was recently on a visit to
(e) Government’s overambitious plans for development. Bangladesh where both the countries signed a Coordinated
149. Which of the following is/are true about China in the context
Border Management Plan (CBMP). What are major points
of the passage?
of this agreement?
(1) China’s citizens are in favor of a more representative
form of government. (A) The plan will help in resolving long pending border
(2) China has made huge strides in infrastructure dispute between both the nations.
developments. (B) The head count of the people living in enclaves on the
(3) China is in the midst of a political revolution. border will be completed within next six month's time
(a) None (b) Only (1) (C) India assured that no BSF Jawan will open fire on the
(c) Only (1) and (3) (d) Only (2) people crossing the borders unless there is an attack
(e) All (1), (2) and (3) on them.
150. What does the phrase “messiness of democracy” convey (a) Only (A) (b) Only (B) (c) Only (C)
in the context of the passage? (d) All (A), (B) and (C)
(a) Democratic nations are chaotic on account of individual (e) Both (A) and (C)
freedoms. 156. Which of the following is/are commodity/ies on which
(b) Most democratic countries frequently have violent subsidy is given to farmers/ poor in India?
revolts among their citizens.
(A) Fertiliser (B) Kerosene (C) LPG
(c) The divide between the poor and educated is growing
(a) Only (A) (b) Only (B) (c) Only (C)
wider in democracies.
(d) High levels of pollution on account of frantic pace of (d) Only (A) & (C) (e) All (A), (B) & (C)
infrastructure development. 157. The President of India was recently on a visit to South
(e) Resigned acceptance of intrinsic corruption in the Korea and Mongolia. During her visit some agreements
education system. were signed by both the countries. Which of the following
was a common area on which India signed agreements with
General Awareness South Korea and Mongolia both?
(a) Peaceful use of Nuclear energy
151. Which of the following countries recently conferred its. (b) Export of edible oil to South Korea and Mongolia
Highest State Honour on Late Mrs. Indira Gandhi for her (c) Providing technical help to become self sufficient in
contribution in its 'Liberation war'? production of food grains
(a) South Sudan (b) Bangladesh (c) Bhutan (d) Willingness of both the countries to support India's
(d) Slovakin (e) Croatia bid to have permanent seat in UN Security Council
152. Which of the following statement is true? (e) To hold Olympic games in 2022 In New Delhi jointly
(a) Banks cannot accept demand and time deposite from with South Korea and Mongolia
public.
158. Iran recently threatened that it will stop supply of oil to
(b) Banks can accept only demand deposits from public.
India if the payment Issue is NOT sorted out quickly. What
(c) Banks can accept only time deposits from public.
is the problem in payment to Iran?
(d) Banks can accept both demand and time deposits from
(A) The oil supplied by Iran is of substandard quality.
public.
Hence India suspended the supply and stopped the
(e) Banks can accept demand and time deposits only from
government. payment.
y
o
u
rs
m
110 IBPS PO/MT Exam 2011

a
h
b
(B) Indian banks have stopped transfer of money to Iran (C) Some countries feel that it wan Japan who allowed

o
as UN has imposed sanctions on Iran. swapping of its currency with other nations and

o
b
(C) India is bound to purchase oil only from OPEC members. created a grim situation in world market.

.w
Iran has abandoned the membership of OPEC, which (a) Only (A) (b) Only (B) (c) Only (C)

o
has created a problem in payments. (d) All (A), (B) & (C) (e) None of these

rd
(a) Only (A) (b) Only (B) (c) Only (C)

p
166. Which of the following is NOT a Millennium Goal set by

re
(d) All (A), (B) and (C) the UNO?

s
s
(e) None of these (a) Eradicate extreme poverty and hunger

.c
159. Accounts are allowed to be operated by cheques in respect (b) Reduce child mortality

o
m
of (c) Ensure environmental sustainability
(a) Both Savings bank accounts and fixed deposit accounts (d) Combat terrorism and bring people in main stream of
(b) Savings bank accounts and current accounts
society
(c) Both Savings bank accounts and loan accounts
(e) Achieve universal primary education
(d) Both Savings bank accounts and cash accounts only
(e) Both Current accounts and fixed deposit accounts 167. Financial inclusion means provision of
160. Yingluck Shinawatra has won recently held elections to (a) financial services namely payments, remittances,
become first woman Prime Minister of savings, loans and Insurance at affordable cost to
(a) Myanmar. (b) South Korea (c) North Korea persons not yet given the same.
(d) Thailand (e) China (b) ration at affordable cost to persons not yet given the
161. Which of the following is correct statement? same.
(a) Normally no interest is paid on current deposit (c) house at affordable cost to persons not yet given the
accounts same.
(b) Interest is paid on current accounts at the same rate as (d) food at affordable cost to persons not yet given the
term deposit accounts same.
(c) The rate of interest on current account and savings (e) education at affordable cost to persons not yet given
account are the same the same.
(d) No interest is paid on any deposit by the bank 168. Which of the following Statements containing results of
(e) Savings deposits are the same as current deposits. India's recent Census is NOT correct?
162. The Lokayukta of which of the following States had (a) Kerala has about 94% literacy, the highest in India.
submitted a report on illegal mining of Iron are in the State?
(b) The Sex Ratio in India is 940
(a) Andhra Pradesh (b) Tamil Nadu (c) Karnataka
(c) Male literacy is lower than female literacy in seven
(d) Orissa (e) None of these
most developed states
163. The usual deposit accounts of banks are
(a) Current accounts, electricity accounts and insurance (d) Uttar Pradesh is the most populous State in India
premium accounts (e) The population of India has increased by 181 million
(b) Current accounts, Post office savings bank accounts during the decade 2001-2011
and term deposit accounts 169. Which of the following is known as cross selling by Banks?
(c) Loan accounts, savings bank accounts and term (A) Sale of a debit card to a credit card holder
deposit accounts (B) Sale of Insurance policy to a depositor
(d) Current accounts, savings bank accounts and term (C) Issuance of Cash against Cheque presented by a third
deposit accounts party
(e) Current bill accounts and term deposit accounts (a) Only (A) (b) Only (B) (c) Only (C)
164. Fixed deposits and recurring deposits are (d) Both (A) and (C) (e) All (A), (B) and (C)
(a) repayable after an agreed period. 170. Angela Merkel was recently in India to receive Jawaharlal
(b) repayable on demand Nehru Award for international Understanding for the year
(c) not repayable 2009. What has been her contribution in the international
(d) repayable after death of depositors politics which made her the 'Best Choice' for the award?
(e) repayable on demand or after an agreed period as per (A) She played a major role in restarting negotiations
bank's choice. between Russia and USA on 'START'.
165. The countries these days are most worried about the (B) She initiated 'Health Care Reforms' in Germany and
currency war. They are arguing about whose actions harmed
solved problems related to future energy development.'
the world economy the most. What are the views of the rich
(C) She played a crucial role in negotiation of the Treaty
countries, particularly USA, In this regard?
of Lisbon' and Berlin declaration.
(A) They feel that it was China who was unwilling to allow
the Yuan to rise. (a) Only (B) and (C) (b) Only (A) and (B)
(B) They feel that it was India who had snatched jobs in (c) Only (C)
most of the European countries and USA. This made (d) Only (A) and (C)
the situation more grim. (e) None of these
y
o
u
rs
m
IBPS PO/MT Exam 2011 111

a
h
b
171. Japan won the women's Football World Cup 2011 by conducted from the bank. No other business can be

o
defeating conducted from there

o
b
(a) England (b) Germany (c) Argentina (c) Accounts in which shares of various companies are

.w
(d) USA (e) China traded in electronic form

o
rd
172. Which of the following states became the first state in the (d) Accounts which are operated through internet banking

p
country to launch RBI's e-payment system for commercial facility

re
tax payers? (e) None of these

s
s
(a) Andhra Pradesh (b) Kerala (c) Gujarat 180. Who amongst the following won the Wimbledon Men's

.c
(d) Maharashtra (e) Karnataka Finals 2011 which was also his first Wimbledon Title?

o
m
173. When a bank returns a cheque unpaid. It is called (a) Rafael Nadal (b) Novak Djokovic
(a) payment of the cheque (c) Robert Bruce (d) Lleyton Hewitt
(b) drawing of the cheque (e) None of these
(c) cancelling of the cheque 181. Which of the following organizations/ agencies has sought
(d) dishonour of the cheque an emergency fund of Rs 1000 crore from banks to tackle
(e) taking of the cheque. acute liquidity crisis, which is coming in the way to give
174. Mortgage is a loans to micro borrowers?
(a) security on movable property for a loan given by a (a) Regional Rural & Cooperative Banks
bank. (b) RBI
(b) security on immovable property for a loan given by a (c) Micro Finance Institutions
bank. (d) NABARD
(c) concession on immovable property for a loan given (e) None of these
by a bank. 182. Upper limit prescribed for RTGS transaction is
(d) facility on immovable property for a loan given by a (a) ` 1 lac (b) ` 2 lacs (c) ` 5 lacs
bank. (d) ` 50 lacs (e) No upper limit is prescribed
(e) security on immovable property for a deposit received 183. Distribution of Insurance products and Insurance policies
by a bank. by banks as corporate agents is known as
175. The world's highest 'Rail Bridge' is being constructed in the (a) General Insurance(b) Non-life Insurance
State of Jammu & Kashmir. This bridge will be on which of (c) Bancassurance (d) Insurance banking
the following rivers? (e) Deposit Insurance
(a) Jhelum (b) Chenab (c) Indus 184. What does the letter 'L' denote in term 'LAF' as referred
(d) Ravi (e) None of these every now and then in relation to monetary policy of the
176. Which of the following films was NOT directed by Mani RBI?
Kaul, who died recently? (a) Liquidity (b) Liability (c) Leveraged
(a) 3 Idiots (b) Ghashiram Kotwal (d) Longitudinal (e) Linear
(c) Uski Roti (d) Duvidha 185. What is the full form of 'FINO' a term we see frequently in
(e) Ashadh Ka Ek Din financial newspapers?
177. The President of which of the following countries is (a) Financial Investment Network and Operations
accused of violation of “War Powers Act” of his country (b) Farmers Investment in National Organisation
when he decided to attack Libya with other NATO countries (c) Farmers Inclusion News and Operations
in June 2011? (d) Financial Inclusion Network and Operations
(a) USA (b) France (c) Germany (e) None of these
(d) Spain (e) Italy 186. Who among the followings has been appointed as a director
on board of Bank of America- the first non-American on the
178. Which of the following schemes is launched by the Govt.
board of the one of the largest financial institutions in the
of India to provide an identity to its all resident citizens and
world?
also in identifying people who are poorest often the last to
(a) Azim Premji (b) Ratan Tata
stand up to ask for their share in govt. schemes or lack
identity? (c) Mukesh Ambani (d) K.V. Kamath
(a) Public Provident Fund Scheme (e) Chanda Kocher
(b) Indira Gandhi Old Age Pension Scheme 187. When the rate of inflation increases
(c) Social Security Scheme for Gramin Dak Sevaks (a) purchasing power of money increases
(d) M. G. National Rural Employment Guarantee Act (b) purchasing power of money decreases
(e) Aadhaar Card scheme (c) value of money increases
179. Which of the following types of accounts are known as (d) purchasing power of money remains unaffected
'Demat Accounts'? (e) amount of money in circulation decreases
(a) Accounts which are Zero Balance Accounts 188. POSCO is in the process of establishing its plants in India.
What does the letter 'P'. denote in the name POSCO?
(b) Accounts which are opened to facilitate repayment of
a loan taken from the bank. No other business can be (a) Popular (b) Pallin (c) Pohang
(d) Paradeep (e) Petersburg
y
o
u
rs
m
112 IBPS PO/MT Exam 2011

a
h
b
189. Who amongst the following has become the Chief Minister (A) The paper presents the road map for general debt to

o
of a State in India third time consecutively? be raised by the Govt. of India during the period of

o
b
(a) Tarun Gogoi (b) Nitish Kumar 2011–2015 which is the period of the awards of the

.w
(c) J. Jayalalitha (d) Mamta Banerjee 13th Finance commission

o
(e) None of these (B) As per the estimates, the target debt in the terminal

rd
year 2014–15 will be lower than whatever recommended

p
190. Who amongst the following is the winner of Man Booker

re
Prize 2011? by the 13th Finance Commission.

s
s
(a) Andrea Levy (b) Nicola Barker (C) The paper has recommended that the Govt. should try

.c
(c) Tom McCarthy (d) Linda Grant to generate resources in such a way that. It does not

o
m
(e) Philip Roth have to depend much on debt. The total debt should
191. Interest on savings bank account is now calculated by banks 1
not be more than rd of the total requirements.
on 3
(a) minimum balance during the month (a) Only (A) (b) Only (B)
(b) minimum balance from 7th to last day of the month (c) Only (A) & (B) (d) Only (B) & (C)
(c) minimum balance from 10th to last day of the month (e) None of these
(d) maximum balance during the month 198. 'Gorkhaland Council' which was recently in news is a body
(e) daily product basis set up for administration of which of the following Districts?
192. The third international Conference of the Trade Unions was (a) Siliguri (b) Bagdogra (c) Malda
organized in June 2011 in (d) Darjeeling (e) Gangtok
(a) Vienna (b) Moscow (c) London 199. Banks in our country normally publicise that additional
(d) Paris (e) Berlin interest rate is allowed on retail domestic term deposits of
193. A centralized database with on line connectivity to branches, (a) Minors (b) Married women
internet as well as ATM network which has been adopted (c) Senior citizens (d) Govt. employees
by almost all major banks of our country is known as (e) Rural residents
(a) Investment banking 200. FBTR is a term/ abbreviated name related to the area of
(b) core banking (a) Nuclear power (b) Agriculture (c) Economy
(c) mobile banking (d) Medical Science (e) Sports
(d) national banking
(e) specialized banking Computer Knowledge
194. The Government of India recently decided to lift four years
201. To change selected text to all capital letters, click the change
old ban on export of wheat. What was the reason for the
case button, then click
same?
(a) Uppercase (b) Upper all (c) Capslock
(A) India had a bumper crop of wheat in last two years.
(d) Lock Upper (e) Large Size
Hence it has excess stock of wheat. 202. A person who uses his or her expertise to gain access to
(B) As per the Food Security Act India is bound to provide other people’s computers to get information illegally or do
10 million tones of wheat to World Food Grain stock damage is a
every year. India defaulted last year. This year it does (a) hacker (b) analyst
not want to be one. (c) instant messenger
(C) As advised by the Supreme Court of India, the money (d) programmer (e) spammer
received from export should be used to pay subsidy to 203. A device that connects to a network without the use of
the farmers. cables is said to be
(a) Only (B) (b) Only (A) (c) Only (C) (a) distributed (b) free (c) centralized
(d) Both (A) & (B) (e) All (A), (B) & (C) (d) open source (e) None of these
195. Which of the following is not considered a money market 204. Reusable optical storage will typically have the acronym
instrument? (a) CD (b) DVD (c) ROM
(a) Treasury bills (b) Repurchase Agreement (d) RW (e) ROS
(c) Commercial paper (d) Certificate of Deposit 205. The most common type of storage devices are
(e) Shares and bonds (a) persistent (b) optical (c) magnetic
196. With a view to facilitate payment of balance in the deposit (d) flash (e) steel
account to the person named by the depositor without any 206. Codes consisting of lines of varying widths or lengths that
hassles in the event of death of the account holder, the are computer-readable are known as
following facility was introduced for bank accounts in our (a) an ASCII code (b) a magnetic tape
country (c) a bar code (d) an OCR scanner
(a) Will (b) Registration (c) Nomination (e) None of these
(d) Indemnity (e) Guarantee 207. A Web site’s main page is called its
197. The ministry of Finance recently released a comprehensive (a) Homepage (b) Browser page
paper on Govt. debt. Which of the following is/ are true (c) Search Page (d) Bookmark
about the same? (e) None of these
y
o
u
rs
m
IBPS PO/MT Exam 2011 113

a
h
b
208. Part number, part description, and number of parts ordered 220. In Word you can force a page break

o
are examples of (a) by positioning your cursor at the appropriate place

o
b
(a) control (b) output (c) processing and pressing the F1 key

.w
(d) feedback (e) input (b) by positioning your cursor at the appropriate place

o
rd
209. To access properties of an object, the mouse technique to and pressing Ctrl + Enter

p
use is (c) by using the Insert/Section Break

re
(a) dragging (b) dropping (d) by changing the font size of your document

s
s
(c) right-clicking (d) shift-clicking (e) None of these

.c
(e) None of these

o
221. Grouping and processing all of a firm’s transactions at one

m
210. Computers use the ________ number system to store data time is called
and perform calculations. (a) a database management system
(a) binary (b) octal (c) decimal (b) batch processing
(d) hexadecimal (e) None of these (c) a real-time system
211. ________ ‘are attempts by individuals to obtain confidential (d) an on-line system
information from you by falsifying their identity. (e) None of these
(a) Phishing trips (b) Computer viruses 222. Help menu is available at which button?
(c) Spyware scams (d) Viruses (a) End (b) Start (c) Turnoff
(e) Phishing scams (d) Restart (e) Reboot
212. Why is it unethical to share copyrighted files with your 223. You can keep your personal files/folders in
friends? (a) My folder (b) My Documents (c) My Files
(a) It is not unethical, because it is legal. (d) My Text (e) None of these
(b) It is unethical because the files are being given for 224. A central computer that holds collections programs for many
free. PCs, workstations computers is a(n)
(c) Sharing copyrighted files without permission breaks (a) supercomputer (b) minicomputer
copyright laws.
(c) laptop (d) server (e) None of these
(d) It is not unethical because the files are being given for
225. When you save to this, your data will remain when the
free.
computer is turned off.
(e) It is not unethical - anyone can access a computer
(a) RAM
213. Which of the following can be used to select the entire
(b) motherboard
document?
(c) secondary storage device
(a) CTRL+A (b) ALT+ F5
(d) primary storage
(c) SHIFT+A (d) CTRL+K (e) CTRL+H
(e) None of these
214. To instruct Word to fit the width of a column to the con-
226. The____folder retains copies of messages have started but
tents of a table automatically, click the___button and then
are not yet ready to send.
point to AutoFit Contents.
(a) Drafts (b) Outbox (c) Address B
(a) Fit to Form (b) Format (c) Autosize
(d) Contents (e) AutoFit (d) Sent Items (e) Inbox
215. The simultaneous processing of two or more programs by 227. You can_____a search by providing more info the search
multiple processors is engine can use to select a smaller, more useful set of results.
(a) multiprogramming(b) multitasking (a) refine (b) expand (c) load
(c) time-sharing (d) multiprocessing (d) query (e) slowdown
(e) None of these 228. The contents of ____ are lost when the computer turns off.
216. A disk’s content that is recorded at the time of manufacture (a) storage (b) input (c) output
and that cannot be changed or erased by the user is (d) memory (e) None of these
(a) memory-only (b) write-only (c) once-only 229. The_____enables you to simultaneously keep multiple Web
(d) run-only (e) read-only pages open in one browser window.
217. What is the permanent memory built into your computer (a) tab box (b) pop-up helper
called? (c) tab row (d) address bar
(a) RAM (b) Floppy (e) Esc key
(d) CD-ROM (e) ROM 230. A DVD is an example of a(n)
218. The default view in Excel is_____view. (a) hard disk (b) optical disc
(a) Work (b) Auto (c) Normal (c) output device (d) solid-state storage device
(d) Roman (e) None of these (e) None of these
219. What displays the contents of the active cell in Excel? 231. The basic unit of a worksheet into which you enter data in
(a) Namebox (b) Row Headings Excel is called a
(c) Formulabar (d) Taskpane (a) tab (b) cell (c) box
(e) None of these (d) range (e) None of these
y
o
u
rs
m
114 IBPS PO/MT Exam 2011

a
h
b
232. _______ is the process of dividing the disk into tracks and 242. This first step in the transaction processing cycle captures

o
sectors. business data through various modes such as optical scan-

o
b
(a) Tracking (b) Formatting (c) Crashing ning or at an electronic commerce website.

.w
(d) Allotting (e) None of these (a) Document and report generation

o
(b) Database maintenance

rd
233. Which ports connect special types of music instruments
(c) Transaction processing start-up

p
to sound cards?

re
(d) Data Entry
(a) BUS (b) CPU (c) USB

s
(e) None of these

s
.c
(d) MIDI (e) MINI
243. When the pointer is positioned on a ____, it is shaped like

o
234. The process of transferring files from a computer on the

m
a hand.
Internet to your computer is called (a) Grammar error (b) Formatting error
(a) downloading (b) uploading (c) Screen Tip (d) Spelling error
(c) FTP (d) JPEG (e) downsizing (e) Hyperlink
235. In Excel ________allows users to bring together copies of 244. The computer abbreviation KB usually means
workbooks that other users have worked on independently. (a) Key Block (b) Kernel Boot (c) Key Byte
(l) Copying (b) Merging (c) Pasting (d) Kit Bit (e) Kilo Byte
(d) Compiling (e) None of these 245. Which of the following are advantages of CD-ROM as a
236. If you want to connect to your own computer through the storage media?
Internet from another location, you can use (a) CD-ROM is an inexpensive way to store large amount
(a) e-mail (b) FTP of data and information.
(c) instant message (b) CD-ROM disks retrieve data and information more
(d) Telnet (e) None of these quickly than magnetic disks do.
(c) CD-ROMs make less errors than magnetic media.
237. To reload a Web page, press the ________ button.
(d) All the above
(a) Redo (b) Reload
(e) None of these
(c) Restore (d) Ctrl (e) Refresh 246. A(n)_____ is a special visual and aduio effect applied in
238. Mobile Commerce is best described as Powerpoint to text or content.
(a) The use of kiosks in marketing (a) animation (b) flash (c) wipe
(b) Transporting products (d) dissolve (e) None of these
(c) Buying and selling goods/services through wireless 247. Which of the following is a storage device that uses rigid,
handheld devices permanently installed magnetic disks to store data/infor-
(d) Using notebook PCs in marketing mation?
(e) None of the above (a) floppy diskette (b) hard disk
239. Video processors consist of_____and_____which store (c) permanent disk (d) optical disk
and process images. (e) None of these
(a) CPU and VGA (b) CPU and memory 248. The piece of hardware that converts you computer’s digital
(c) VGA and memory (d) VGI and DVI signal to an analog signal that can travel over telephone
(e) None of these lines is called a
(a) red wire (b) blue cord (c) tower
240. _________are words that a programming language has set
(d) modem (e) None of these
aside for its own use.
249. Personal computers can be connected together to form a
(a) Control words (b) Control structures (a) server (b) supercomputer
(c) Reserved words (d) Reserved keys (c) network (d) enterprise
(e) None of these (e) None of these
241. What is the process of copying software programs from 250. A ________ is the term used when a search engine returns
secondary storage media to the hard disk called? a Web page that matches the search criteria.
(a) configuration (b) download (c) storage (a) blog (b) hit (c) link
(d) upload (e) installation (d) view (e) success
y
o
u
rs
m
IBPS PO/MT Exam 2011 115

a
h
b
o
o
b
.w
o
rd
1. (e) According to questions.

p
C 5 km B

re
AB = 2 km

s
left

s
left

.c
BC = 5 km Gliders Airplane
2 km

o
m
CD = 8 km Parachutes
AStarting
DE = 5 km point
8 km
EF = 1 km 6. (a) Hence, only conclusion I follows.
7. (b) Hence, conclusion II follows.
BC = DE = 5 km 8. (d) According to statements.
F
CD = BE = 8 km left left
1 km
BE = EF + AF + AB D 5 km E
\ AF = BE – (EF + AB) Mails Updates
= 8 – (1 + 2) = 8 – 3 = 5 km
\ Required distance = AF = 5 km and required Chats
direction is North
For questions 2-3 : or
Person Floor
B 6th
Mails
C 5th or
F 4th Mails
E 3rd Chats Chats
A 2nd
D 1st/Ground
Hence, conclusion I follows.
2. (d) A and E live on the floors exactly between D and F. For questions 9 -10 :
3. (a) B lives on floor number 6. According to statements
For questions 4 -5 :
The meaningful english word 'LEAPS' will be formed.
L E A P S
Left Right Metals
4. (d) P is placed second to the right of E.
5. (c) The word ‘LEAPS’ will be formed based on the given
conditions.
or
For questions 6 -7 :
According to statements:

Metals
Gliders Airplane

Parachutes Helicopters

or

Metals

Gliders Airplane

Parachutes 9. (a) Hence, conclusion I follows.


s
opter 10. (b) Hence, only conclusion II follows.
Helic
y
o
u
rs
m
116 IBPS PO/MT Exam 2011

a
h
b
For questions 11 -15 :

o
o
b
Input : for 52 all 96 25 jam road 15 hut 73 bus stop 38 46

.w
Step I : all for 52 25 jam road 15 hut 73 bus stop 38 46 96

o
rd
Step II : bus all for 52 25 jam road 15 hut stop 38 46 96 73

p
Step III : for bus all 25 jam road 15 hut stop 38 46 96 73 52

re
Step IV : hut for bus all 25 jam road 15 stop 38 96 73 52 46

s
s
Step V : jam hut for bus all 25 road 15 stop 96 73 52 46 38

.c
Step VI : road jam hut for bus all 15 stop 96 73 52 46 38 25

o
m
Step VII : stop road jam hut for bus all 96 73 52 46 38 25 15

and step VII is the last step of the arrangement of the above input.
11. (a) Step V : jam hut for bus all 25 road 15 stop 96 73 52 46 38
6th from left
Hence, the number '25' would be at the 6th position from the left in Step V.
12. (d) Step III : for bus all 25 jam road 15 hut stop 38 46 96 73 52

13. (b) Step IV : hut for bus all 25 jam road 15 stop 38 96 73 52 46
8th from right
Hence, the word 'road' would be at the 8th position from the rigth in step IV.
14. (c) Step VII : stop road jam hut for bus all 96 73 52 46 38 25 15
15. (c) Step II : bus all for 52 25 jam road 15 hut stop 38 46 96 73

For questions 16 -20 : A Canara Bank


According to the question
First to right
A
D Bank of Maharashtra
Bank of Maharashtra
H B
Canara Bank Syndicate Bank second to right

E Syndicate Bank

G D
second to right
Bank of India Oriental
Bank of Commerce
F Punjab National Bank

second to right
F E
Punjab National Bank Hence, 'A – Canara Bank' does not belong to that
UCO Bank C group.
Dena Bank
19. (d) Representative from Punjab National Bank (E) sits
second to left of B.
16. (c) H (representative of Canara Bank) and A 20. (e) The representative from Bank of Maharashtra (A) and
(representative of Bank of Maharashtra) sit exactly Syndicate Bank (B) are immediate neighbours of each
between B (representative of syndicate bank) and G
other.
(representative of Bank of India).
21. (c) From statement I.
17. (e) D is the representative from Oriental Bank of M
Commerce. B and D are sisters of M
sister sister
18. (b) From options + indicates Male
H UCO Bank – indicates Female
B– D–
From statement II. sister
second to right M's father T is husband of W
y
o
u
rs
m
IBPS PO/MT Exam 2011 117

a
h
now or never again Þ tom ka na sa

b
o
Husband again go now or never Þ na ho ka sa tom

o
T+ W–

b
Hence, go Þ ho

.w
So, only I and III, are required to answer the question.

o
father mother

rd
24. (e) From statement I

p
west

re
R W

s
M

s
.c
o
From statement III north

m
Out of the three children which T has, only one is a
T
boy.
From statement II
T T
son daughter
(boy) (girl) south
daughter
(girl)
1 3 J Z
2 east
From statement III
From statements II and III
M
Since W is the wife of T, Hence W has 2 daughters
north-east north

+ wife
T W
J Z
From statement I and II
girl
(daughter) R west
son W
girl
(daughter) north

T
So, II and III, are required to answer the question. south-west
22. (d) From statement I, south
E > B > A.
From statement II J east Z
•> •> • >C > •>•
Hence, village J is to the south-west of Village W.
From statement III So, only I and II are required to answer the question.
•> •> • > •> D> F 25. (e) From statement I, II, III
From statements I, II and III
Monday Suresh's mother does not visit
E > B >A> C > D > F
Tuesday
Hence E is the tallest. Wednesday Leave
So, all I, II and III are required to answer the question. Thursday Suresh's mother does not visit
23. (a) From statement I Friday
now or never again Þ tom ka na sa Saturday
From statement II
you come again now Þ ja ka ta sa From statement II, Suresh visited Chennai the day after
From statement III his mothers visit and the day of his mother's visit day
again go now or never Þ na ho ka sa tom is not given, so, we cannot answer the question even
with all I, II and III.
From statement I and III
For questions 26 -30 :
y
o
u
rs
m
118 IBPS PO/MT Exam 2011

a
h
Given information can be tabulated as follows : problems to others. Steps (B) and (C) will encourage in

b
o
favour of the girl child.

o
b
Travelling 41. (b) From 2nd figure to 1st figure, the group of four symbols

.w
Cars Destination with Gender moves one step clockwise and the symbols change

o
M/F their position as follows :

rd
HC SD FI D C H Yes No

p
P ü ü

re
Q R

s
s
R V QW

.c
o
S ü F

m
T × only M
Z
V R The larger figure gets reduced in size and vice-versa.
W × R 42. (a) From 2nd to 1st figure, lower symbol moves one step
Z × only upper side, gets enlarged and covers existing design.
T
Inside lowermost symbol change its shape as upper
most design.
From above table we can conclude the following result
43. (c) The symbols of horizontal line become vertical and
the symbols of vertical line become horizontal. Other
Group Car Destination four symbols move their position as follows:

+ –
TZ SD Delhi


S R+ V + FI Chennai

P+QW HC Hyderabad
44. (a) From 2nd figure to 1st figure, lower side symbol forms
at left, left side design forms at upperside after reversing
+ indicates male and – indicates for female
of its one end, upperside symbol forms right after
26. (c) P and Q are travelling with W. reversing of its one end and the right side symbol
27. (b) P, Q and W are travelling in Honda City. forms at lowerside after reversing of its one end.
28. (a) 29. (d) 45. (b) From 2nd figure to 1st figure, the symbol move as
30. (c) members in Ford Ikon car are travelling to Chennai. follows and new symbols form at the place of N.
31. (c) Only statement in option (c) provides reason of spate
of rail accident in India in the recent month. N
32. (e) Possible fallout may be drought affected area which is
not given in any options.
33. (b) Option (a), (c), and (d) does not explain the effect of given
cause. Only option (b) can be effect of damaged roads.
34. (a) Due to lack of job, it is possible that students don't
want to take admission in engineering colleges. Hence 46. (b) In each subsequent figure, the symbols of each row
option (a) may be probable cause of the above effect. arranged as follows :
35. (c) Only statement (c) says that in previous years, the
performance of other management colleges was better N
than the performance of local colleges.
36. (d) People are going abroad for better opportunities. N
37. (d) Assumption (d) is implicit in the given statement. If
people don't show respect then there is no meaning of
this kind of appeal. (1) to (2) (2) to (3)
38. (d) From given fact it can be inferred that the government (3) to (4) (4) to (5)
generally provides protection to travellers accross the (5) to (6)
country.
39. (a) All the three causes will effect the sales of four 47. (a) In each subsequent figure the group all symbols moves
wheelers. one side anti-clockwise and each design change their
40. (c) (A) would not be effective step as it will create a lots of position as follows:
y
o
u
rs
m
IBPS PO/MT Exam 2011 119

a
h
b
28 195 39 5 28 195 44 5

o
54. (d) ? = ´ ¸ + = ´ ´ +

o
65 308 44 26 65 308 39 26

b
.w
5 8 + 5 13 1

o
4
+ = = =

rd
=
13 26 26 26 2

p
re
55. (c) ? + 1147.69 = (23.1)2 + (48.6)2 – (39.8)2

s
(1) to (2) (2) to (3)

s
\ ? = 533.61 + 2361.96 – 1584.04 – 1147.69 = 163.84

.c
(3) to (4) (4) to (5)

o
(5) to (6)

m
56. (e) ? ¸ 21.003 = 3 4663 + 349
48. (e) In each subsequent figure, the group of five symbols Þ ? ¸ 21 = 17 + 349 = 366
arranged in a vertical line moves left to middle, middle \ ? = 366 ´ 21 = 7686 » 7680
to right and right to left. Designs slide as follows and
new symbol forms at the place of 'N'. 39.897 58.779
57. (c) ? = 4331 ´ + 5003 ´
100 100
N 40 59
N = 4330 ´ ∗ 5000´
100 100

= 1732 + 2950 = 4682 » 4700


58. (c) ? = 59.88 ¸12.21´ 6.35
(1) to (2) (2) to (3)
(3) to (4) (4) to (5) 1
(5) to (6) = 60 ¸12 ´ 6 = 60 ´ ´ 6 » 30
12

49. (b) In each step the symbol shifts to one place anti- 59. (e) ? = 43931.03 ¸ 2111.02 ´ 401.04
clockwise, rotate by 180° and a new symbol is added ; 43930 ¸ 2110 ´ 400
at the anti-clockwise end.
1
50. (c) In each subsequent figure, a group of the four symbols = 43930 ´ ´ 400 » 8300
2110
rotates 45° anti-clockwise and the symbols change
their places as follows and new symbols form at the 60. (b) ? = 6354 ´ 34.993 » 80 ´ 35 = 2800
place of 'N'.
61. (e) The given number series is based on the following pattern:
1075
9050 5675 3478 2147 1418 1077 950

N N N – (15)
3
– (13)
3
– (11)
3
– (9)
3
– (7)
3
– (5)
3

Hence, the number 1077 is wrong and it should be


(1) to (2) (2) to (3) (3) to (4) replaced by 1075.
(4) to (5) (5) to (6)
62. (d) The given number series is based on the following
pattern :
51. (b) 11449 ´ 6241 - ( 54) 2 = ? + ( 74) 2
1744
7 12 40 222 1742 17390 208608
Þ ? = 107 × 79 – 2916 – 5476
= 8453 – 2916 – 5476 = 61 × 2 – (2 × 1) × 4 – (4 × 2) × 6 – (6 × 3) × 8 – ( 8 × 4) ×10 – (10 × 5) × 12 – (12 × 6)

\ ? = (61)2 = 3721 Hence, the number 1742 is wrong and it should be


replaced by 1744.
52. (c) ? = éë( 3 8 + 8 ) ´ (8 8 + 7 8 ) ùû - 98
63. (c) The given number series is based on the following pattern:

( )
= 4 8 ´ 15 8 - 98 = ( 60 ´ 8) - 98
6 91
582
584 2935 11756 35277 70558
= 480 - 98 = 382
´ 7 + (7)2 ´ 6 + (6)2 ´ 5 + (5)2 ´ 4 + (4)2 ´ 3 + (3)2 ´ 2 + (2)2
53. (a) ? + 5883 = 3463 ´ 295 – 18611
\ ? = 1021585 – 18611 – 5883 = 997091 Hence, the number 584 is wrong and it should be
replaced by 582.
y
o
u
rs
m
120 IBPS PO/MT Exam 2011

a
h
b
64. (d) The given number series is based on the following
Interest ×100

o
pattern. 70. (b) Rate of simple interest =

o
Principal × time

b
.w
27

o
1 4 25 256 3125 46656 823543 10800 ´100

rd
= < 12%
22500 ´ 4

p
re
1 2 3 4 5 6 7
(1) (2) (3) (4) (5) (6) (7)

s
s
é time ù

.c
Compound interest = Principal êæç1+
rate ö
– 1ú

o
Hence, the number 25 is wrong and it should be ÷
êëè 100 ø

m
úû
replaced by 27.
65. (b) The given number series is based on the following
pattern : é 2 ù
= 22500 êæç1 +
12 ö
÷ - 1ú
êëè 100 ø úû
1053
8424 4212 2106 1051 526.5 263.25 131.625
éæ 3 ö2 ù
1 1 1 1 1 1 = 22500 ê ç1 + ÷ - 1ú
´ ´ ´ ´ ´ ´
2 2 2 2 2 2 ê è 25 ø ú
ë û

Hence, the number 1051 is wrong and it should be


replaced by 1053. éæ 28 ö 2 ù æ 784 ö
= 22500 êç ÷ - 1ú = 22500 ç -1
êè 25 ø ú è 625 ÷ø
66. (e) According to question ë û
Third number = 2400
159
= 22500 ´ = ` 5724
1 625
\ Second number = 2400´ < 600
4
71. (a) Value of one ticket of each kind = 55 + 85 + 105 = ` 245
Again,
\ Required number of ticket of each kind
6 22
First number ´ = Second number ´ 2940
= 12
11 100 =
245
22 11 72. (b) Ravina’s monthly income
\ First number = 600 ´ ´ < 242
100 6
100 + 15 115
= 32000 ´ = 32000 ´ = ` 36800
45 100 100
\ 45% of the first number = 242 ´ =108.09
100
= Ramola’s annual income = 36800 ´ 3 ´ 12
45 40 = ` 1324800
67. (d) Required number = 4800 ´ ´ < 864
100 100
56
68. (e) Correct Average 73. (d) Marks scored by Ritu = 875 ´ = 490
100

∋24´56( ∗ ∋48 ∗ 59 ∗ 67( ,∋44 ∗ 45 ∗ 61( 92


= Marks scored by Smita = 875 ´ = 805
24 100
\ Average marks scored by all the three together
1344 ∗ 174 ,150 1368
< < < 57 490 ∗ 805 ∗ 634 1929
24 24 = = = 643
3 3
69. (c) Suppose the maximum mark of the test be x. 74. (e) According to the question
Present age of Parineeta = 33 – 9 = 24 years
Then, 468 ´100 < 336 ´100 Present age of Manisha = 24 – 9 = 15 years
x 700 Present age of Deepali = 24 + 15 = 39 years
Q 5 : X = 15 : 39
468´100´ 700
\ x= < 975 5´ 39
336 ´100 \ X= < 13
15
y
o
u
rs
m
IBPS PO/MT Exam 2011 121

a
h
75. (c) Cost of one pencil box = 7 + 22 +14 = ` 43 80. (e) According to question,

b
o
\ Required amount = (20 ´ 7) + (8 ´ 22) + (6 ´ 175)

o
14! 14 ´ 13 ´ 12
n ( S ) = 14C3 =

b
+ (7 ´ 43) = 140 + 176 + 1050 + 301= ` 1667 = = 364
(14 - 3)!3! 3 ´ 2 ´ 1

.w
76. (a) According to question,

o
rd
10! 10 ´ 9 ´ 8
04! 14! and n ( E ) = 10 C3 = = = 120

p
n (S) =
14
C4 < < (10 - 3)!3! 3 ´ 2 ´ 1

re
∋14 , 4 (!4! 10!4!

s
s
n(E)

.c
120 30
14 ´ 13 ´ 12 ´ 11 é n! ù = =

o
\ Required probability =
= 1001 êQ nCr = n( S )

m
364 91
( n - r ) !r !úúû
=
4 ´ 3 ´ 2 ´1 êë
55 45
5 25! 2! 81. (e) Required ratio = 34560 ´ :45640 ´
and n( E ) = C2 ´ C2 = ´ 100 100
( 5 - 2)!2! ( 2 - 2)!2! = 19008 : 20538 = 1056 : 1141
82. (e) Required percentage
5´ 4 2´1
= ´ < 10
2´1 1´ 2 ´1 41
55500 ´
100 ´ 100
n∋ E( 10 =
34560 + 65900 + 45640 + 55500 + 42350 + 59650
\ Required probability = n S < 1001
∋ ( 22755
< ´100 < 7.495 ; 7.5% ∋ Approx (
77. (c) According to question, 303600
14! 14! 83. (d) Required percentage
n(S ) = 14
C8 = ´
(14 - 8)!8! 6!8! 20
45640´
14 ´ 13 ´ 12 ´ 11 ´ 10 ´ 9 100 ´100 < 9128 ´100 < 109.30%
=
= = 3003 14 8351
6 ´ 5 ´ 4 ´ 3 ´ 2 ´1 59650´
100
and n ( E ) = 4C2 ´ 5C2 ´ 2C2 ´ 3C2 84. (c) Required Number

4! 5! 2! 3! 20 33
= ´ ´ ´ = 65900 ´
+ 55500 ´ = 13180 + 18315 = 31495
( )
4 - 2 !2! ( )
5 - 2 !2! ( )
2 - 2 !2! ( 2 ) !2!
3 - 100 100
85. (a) Required Average Number
4! 5! 2! 3! 55 43 45 26 70 62
= ´ ´ ´ 34560´
100
∗ 65900´
100
∗ 45640´
100
∗ 55500 ´
100
∗ 42350´
100
∗ 59650´
100
2!2! 3!2! 0!2! 1!2! <
6

4 ´ 3 5´ 4 1 3 19008 ∗ 28337 ∗ 20538 ∗ 14430 ∗ 29645 ∗ 36983


= ´ ´ ´ < 180 =
2´1 2´1 1 1 6
n ∋ E ( 180 60 148941
\ Required probability = n S < 3003 < 1001 = < 24823.5
∋ ( 6
86. (b) Required percentage
78. (e) According to question,
14.4º
n ( S ) = 14 C2 =
14!
=
14 ´ 13
= 91 = ´100 < 11.765 ; 12% ∋ Approx (
(14 - 2)!2! 2 ´ 1 122.4º

\ Probability of at least one red ball æ 57.6º + 64.8º ö


87. (a) Required Number = çè ÷ø ´ 6800
12
C2 66 91 - 66 25 360º
= 1- 14
= 1- = =
C2 91 91 91
122.4º
= ´ 6800 = 2312
79. (2) According to question, 360º
14! 14 ´ 13 ´ 12 88. (c) Required difference
n ( S ) = 14 C3 = = = 364
(14 - 3)!3! 3 ´ 2 ´ 1
( 79.2º +14.4º ) ~ (122.4º + 21.6º ) ´ 6800
\ Required probability =
360º
11
C3 165 364 - 165 199 93.6º ~ 144.0º
= 1 - 14 = 1- = =
= ´ 6800 = 952
C3 364 364 364
360º
y
o
u
rs
m
122 IBPS PO/MT Exam 2011

a
h
b
89. (d) Required percentage

o
88
´ 100 = 115.79%

o
93. (c) Required percentage =
64.8º +21.6º

b
86.4º 76
´ 100 = ´ 100 = 24%

.w
=
360º 360º 94. (b) Two students i.e. Garvita strategic management brand

o
rd
90. (a) Required Ratio = 21.6º : 79.2º = 3 :11 management & compensation management and Archit

p
consumer behaviour service marketing.

re
91. (b) Marks scored by Anushka

s
95. (a) Required Average Marks

s
66 75 88

.c
= 150 ´ + 100 ´ + 150 ´
(88 + 84 + 78 + 96 + 68 + 50) ´ 150 = 464 ´ 150 = 116

o
100 100 100

m
=
100 ´ 6 600
56 56 90
+ 125 ´ + 75 ´ + 50 ´ 96. (d) Required percentage
100 100 100
= 99 + 75 + 132 + 70 + 42 + 45 = 463 15000 + 30000
= ´ 100
92. (d) Marks scored by Archit 5000 + 35000 + 15000 + 25000 + 30000 + 30000

82 76 84
= 150 ´ + 100 ´ + 150 ´ + 125 ´
100 100 100 45000
= ´100 < 32.14 ; 32% ∋Approx (
140000
96 92 88
´ + 75 ´ + 50 ´ 97. (e) In 2009
100 100 100
Total number of girls = (20 + 20 + 15)
= 123 + 76 + 126 + 120 + 69 + 44 = 558
38 55 ´ 38 ´ 100
Marks scored by Arpan ´ ´ 1000 = = 17100
100 100
76 66 78 Total number of boys = 45000 – 17100 = 27900
= 150 ´ + 100 ´ + 150 ´ + 125 ´
100 100 100 Total number of boys who opted for Mathematics
88 72 70 5
´ + 75 ´ + 50 ´ = 27900 ´ = 3100
100 100 100 45
= 114 + 66 + 117 + 110 + 54 + 35 = 496 98. (a) Required Ratio = ( 25 + 30) : (5 + 20) = 55 : 25 = 11 : 5
Marks scored by Garvita (15 + 10 + 15) ´ 1000 ´ 100
99. (b) Required Percentage =
90 88 96 455030
= 150 ´ + 100 ´ + 150 ´ + 125 ´
100 100 100
40´1000
= ´100 < 8.79 ; 9% ∋Approx (
76 84 86 455030
´ + 75 ´ + 50 ´
100 100 100 100. (e) Required total number of students
= 135 + 88 + 144 + 95 + 63 + 43 = 568 = (5 + 35 + 15 + 15 + 20 + 5) ´ 1000
Marks scored by Gunit. = 95 ´ 1000 = 95000
101. (a) For attempting this type of questions it is important to
64 70 68
= 150 ´ + 100 ´ + 150 ´ + 125 ´ correlate what is being stated before the blank space
100 100 100
and what is stated after the blank.
While it is also important to see in which direction the
72 68 74
´ + 75 ´ + 90 ´ passage will go if a particular choice is selected.
100 100 100
In this question, women's rights treaty being ratified
= 96 + 70 + 102 + 90 + 51 + 37 = 446 by a number of countries is discussed so in the
Marks scored by Pranita following sentence effects of these should be
discussed. Option (b) can be rejected as for solving
48 56 50 the problems treaty was ratified. (c) is a judgmental
= 150 ´ + 100 ´ + 150 ´ + 125 ´ statement and should be avoided when no certain
100 100 100
information is available. (d) can fit well before the
64 64 58 ratification of the treaty but not after it. There should
´ + 75 ´ + 50 ´ be a continuation in passage. Option (e) is incorrect
100 100 100
for the logic that first basic rights will be achieved and
= 72 + 56 + 75 + 80 + 48 + 29 = 360 then only empowering can be mentioned. This option
Hence, highest total marks scored by Garvita. is one step further and not about the women's right.
y
o
u
rs
m
IBPS PO/MT Exam 2011 123

a
h
b
102. (c) Option (b) & (d) can be rejected as climate change is 112. (e) Confiscated – means to officially take something away

o
mentioned nowhere in the passage. After the blank from somebody, especially as a punishment.

o
b
global warming is mentioned and before it global Seized definitely fits in the blank therefore option E is

.w
dimming. So blank can work as a connector for both of correct.

o
rd
these. Option (c) includes both. Hence this is the best 113. (a) Organization and mission are two words which can be

p
option. (e) is like deciding in haste without connected with committed or dedicated both the verbs

re
understanding the intent of passage. - organizations committed/ dedicated to …..Objectives.

s
s
.c
103. (a) Before blank space a question is posed. And after the Other words like kicked off/inaugurated/ succumbed

o
blank space reasons which can be associated with poor do not match for objectives of organization.

m
people themselves is discussed so the blank space 114. (c) Holding and Possessing both gives the same meaning
should have a special mention of poor people whether and sense to the sentence. Surrendering is
in form of question or a simple statement (b) & (d) inappropriate. Mastering can not be linked with
won't fit well here so they can be ruled out. passports. Fixating is also incorrect for the blank space
Rich is not even introduced in passage till this point, as it gives no logical sense to the sentence. Examining
therefore option (e) can also be rejected. Additionally can grammatically fit into the blank but not in the
Option (a) gives a logical and sensible flow to the context of passage and would give a different direction
passage. to what is being said in the sentence.
115. (d) 'Next big thing' is always a projection or a decision
104. (a) Only option which is positive for notebooks and is in
based on current situation. It is neither worshipped
accordance with their prices coming down is (a). Option
nor shouted nor explained. Rather next big thing is
(b) is against what is being said in the last line of the
touted or labeled on basis of projection.
passage. (c) is completely illogical and hard to come
116. (e) Dictionary Meaning of MASK -
by as it talks about the prices of components coming
down but prices note books soring high. (e) is also in 1. A covering worn on the face to conceal one's
different direction and is not related to spirit of passage. identity,
Option (d) is about desktops not note books. 2. A protective covering for the face or head.
117. (d
105. (b) Sentence before the blank space is about the
118. (a)
population and after it about the rush in for college
education Option (b) is related to both the statements 119. (c) Dictionary Meaning of MEAN -
(before and after the blank) (e) is repetition of what is 1. (a) To be used to convey; denote.
said in opening statement. (a) is like sudden intrusion (b) To act as a symbol of; signify or represent
of government's angle in passage with no relation to 2. To be mean – cheap, selfish or negativity attached
statements preceding and following the blank space. to it.
106-110. Use of words like 'contrastingly' and 'these' in most of 120. (b) Dictionary Meaning of ALONE -
the sentences imply that some sentences precede these 1. Being apart from others; solitary.
sentences. Only Statement which introduces the topic 2. Being without anyone or anything else; only.
of discussion is Option (e). So it is the first sentence of 3. Considered separately from all others of the same
sequence. Statement 3 extends the topic of books. So class.
follows naturally. 'these two types' of Statement 7 are 4. Being without equal; unique
two types discussed in Statement 3. Statement 6 is Synonyms: alone, lonely, lonesome, solitary
about CRMs so the just before it there should be 121. (c) We plan for future as sentence is about planning for
discussion of NRMs. Last statement is 2 as it finishes future.
the passage on summarizing note. So the correct order 122. (d)
of sentences is 5374612.
123. (d) A word like 'place' or 'put' can be used. Other words
106. (e) 107. (b) 108. (d) 109. (c) 110. (c) would give different direction to the passage.
111. (d) Option (d) is the answer and there are two reasons for 124. (d) Feature is best among the options for something to
it. First one is that from the context of situation appear in a plan.
independence of women from the man's world is 125. (d) Next suits best.
indicated so some word to show dependency should 126. (c) Need of power for India is being mentioned so 'require'
be used. Depend and Rest both show dependence so is proper word for denoting it.
these are the answers. Second one is that as blank 127. (c) An effort for fulfilling the energy need of India is being
space is followed by ON a word that connects discussed in this part of passage. Projections /
grammatically so it should be used. Both 'depend on' development or other options are not discussed here.
and 'rest on' are the correct usage, so it confirms the 128. (e) It is an unexpected line so either option (d) or (e) is to
selection on basis of the first reason. Other options be used. But the structure and place of blank space
are not logical. suits more to option (e).
y
o
u
rs
m
124 IBPS PO/MT Exam 2011

a
h
b
129. (d) Contrasting word with the supply and cost should be 150. (c) Option (a) is wrong as this is not intended from the

o
used. Supply is there but at higher cost. Supply is low

o
use of this phrase. Last part of passage has this phrase

b
but cost is low. So first case is relevant here as per the

.w
and poor-rich divide has been discussed their in voting
context of the passage.

o
pattern or why rich people even educated do not vote.

rd
130. (a) Limited nuclear power/energy is referred here up to a
Option (d) is out of the context. Option (e) can also be

p
particular time.

re
rejected as corruption is not the issue here.
131. (e) Figures are not certain and only can be expected.

s
s
151. (b) Bangladesh honoured former Indian Prime Minister

.c
132. (a) A combination of words with question no. 33 has to be

o
found out. Mrs. Indira Gandhi with its highest state award,

m
133. (a) Expectation line was crossed so for blank in 32. And ‘Bangladesh Swadhinata Sanmanona’ for her
33. As, against is to be used. outstanding contribution to the country's 1971
134. (b) 'Through' gives a right usage for the context. Other 'Liberation War'.
options are not in line with context and also not 152. (d) 153. (d) 154. (d) 155. (e) 156. (c)
grammatically correct. 157. (d) 158. (b) 159. (b) 160. (d) 161. (a)
135. (d) With lives 'touching' is proper word to use. Making,
162. (c) Rising global iron-one prices driven by Chinese
generating are incorrect with 'lives'. Saving is illogical
demand brought forces to the iron ore rich Bellary
and out of context. Lightening lives is a too strong
word for the context. region of Karnataka. This iron one is alleged to have
136. (a) Option (c) can be rejected as it is out of the context. been illegally mind after paying a miniscule royalty to
Option (d) is also not true as they like to invest abroad the government. The major regularities involve mines
than in India. Option (e) is true in parts. Only option of Obulapuram Mining Company Owned by G.
which is in sync with major portion of passage is (a). Karunakara Reddy and G. Jnardhana Reddy who were
137. (a) Other options are not mentioned in the passage. ministers in the Government of Karnataka.
138. (c) Last part of the passage chiefly describes it. Other 163. (d)
options show these sectors in positive light which is
164. (a) In fixed deposit you deposit a certain some for a definite
against passage content.
period. You can opt either for periodical (monthly/
139. (b) Option (b) is true in its completeness, encompassing
quarterly/half yearly or annually) returns or simply opt
the whole of the passage. But other options are true in
parts only. Option (e) is false in parts. to get the amount with accumulated interest at the end
140. (c) Both (1) and (3) are directly mentioned in passage. of the prescribed period. In recurring deposit, you pay
Statement 2 is just not true. a fixed amount at regular intervals (say 100 PM for 12
141. (c) Meaning of Clout is force. months). You will get back the amount with accumulated
142. (e) Marked means distinguished and different. interest at the end of the prescribed period.
Imperceptible is that cannot be distinguished or 165. (e) 166. (d) 167. (a) 168. (c) 169. (e)
percepted. 170. (d) 171. (d) 172. (e) 173. (d) 174. (b)
143. (d) Option (d) can be related to the experts as mentioned 175. (b) 176. (a) 177. (a) 178. (e) 179. (c)
in the passage. 180. (b) 181. (a) 182. (e) 183. (c) 184. (e)
144. (d) Option (d) has plenty of illustrations, references and 185. (a) 186. (e) 187. (b) 188. (c) 189. (a)
structure to support it in the passage. Other options
190. (e) 191. (c) 192. (d) 193. (b) 194. (b)
like (e) and (b) are true but not the central theme of the
195. (e) 196. (c) 197. (b) 198. (d) 199. (c)
passage. Option (a) and (c) are not true.
200. (d) 201. (a) 202. (a) 203. (e) 204. (d)
145. (d) PROMOTES means to advocate a particular cause.
205. (c) 206. (c) 207. (a) 208. (c) 209. (c)
146. (e) A rise from the 4% level to 7 % says that there is rise in
210. (a) 211. (a) 212. (c) 213. (a) 214. (c)
education in Egypt.
215. (d) 216. (e) 217. (e) 218. (c) 219. (c)
147. (a) All of the options are mentioned in one or other part of
220. (b) 221. (b) 222. (b) 223. (b) 224. (d)
the passage.
225. (c) 226. (a) 227. (a) 228. (d) 229. (e)
148. (c) First line of the passage is self explanatory. 230. (b) 231. (b) 232. (b) 233. (d) 234. (a)
149. (b) Only option which can be linked with the passage is 1. 235. (b) 236. (d) 237. (e) 238. (e) 239. (c)
Their people will agitate for greater political freedom, 240. (d) 241. (e) 242. (c) 243. (e) 244. (e)
culminating in a shift to a more democratic form of 245. (a) 246. (a) 247. (b) 248. (d) 249. (c)
government. 250. (b)
y
o
u
rs
m
a
h
b
o
o
IBPS PO/MT EXAM 2012

b
.w
o
rd
p
Based on Memory

re
s
s
.c
REASONING ABILITY • Only one person sits between A and F. A is the father of G.

o
m
H's brother D sits to the immediate left of H's mother. Only
DIRECTIONS (Qs. 1-4) : Study the following information one person sits between H's mother and E.
carefully and answer the given questions : • Only one person sits between H and G. G is the mother of C.
G is not an immediate neighbour of E.
A word and number arrangement machine when given an input 5. What is position of A with respect to his mother-in-law ?
line of words and numbers rearranges them following a particular (a) Immediate left (b) Third to the right
rule in each step. The following is an illustration of input and (c) Third to the left (d) Second to the right
rearrangement (All the numbers are two digits numbers) (e) Fourth to the left
Input : tall 48 13 rise alt 99 76 32 wise jar high 28 56 bam 6. Who amongst the following is D's daughter ?
Step I : 13 tall 48 rise 99 76 32 wise jar high 28 56 bam alt (a) B (b) C
Step II : 28 13 tall 48 rise 99 76 32 wise jar high 56 alt bam (c) E (d) G
Step III : 32 28 13 tall 48 rise 99 76 wise jar 56 alt bam high (e) H
Step IV : 48 32 28 13 tall rise 99 76 wise 56 alt bam high jar 7. What is the position of A with respect to his grandchild ?
Step V : 56 48 32 28 13 tall 99 76 wise alt bam high jar rise (a) Immediate right (b) Third to the right
Step VI : 76 56 48 32 28 13 99 wise alt bam high jar rise tall (c) Third to the left (d) Second to the left
Step VII : 99 76 56 48 32 28 13 alt bam high jar rise tall wise (e) Fourth to the left
and Step VII is the last step of the above input, as the desired 8. How many people sit between G and her uncle ?
arrangement is obtained. (a) One (b) Two
As per the rules followed in the above steps, find out in each of (c) Three (d) Four
the following questions the appropriate step for the given input. (e) More than four
Input : 84 why sit 14 32 not best ink feet 51 27 vain 68 92 9. Four of the following five are alike in a certain way based on
(All the numbers are two digits numbers) the given information and so form a group. Which is the
1. Which step number is the following output ? one that does not belong to that group ?
32 27 14 84 why sit not 51 vain 92 68 feet best ink (a) F (b) C
(a) Step V (b) Step VI (c) E (d) H
(c) Step IV (d) Step III (e) G
(e) There is no such step 10. Which of the following is true with respect to the given
2. Which word/number would be at 5th position from the right seating arrangement ?
in Step V ? (a) C is the cousin of E
(a) 14 (b) 92 (b) H and H's husband are immediate neighbours of each
(c) feet (d) best other
(e) why (c) No female is an immediate neighbour of C
3. How many elements (words or numbers) are there between (d) H sits third to left of her daughter
'feet' and '32' as they appear in the last step of the output ? (e) B is the mother of H
(a) One (b) Three 11. Who sits to the immediate left of C ?
(c) Four (d) Five (a) F's grandmother (b) G's son
(e) Seven (c) D's mother-in-law (d) A
4. Which of the following represents the position of 'why' in (e) G
the fourth step ?
DIRECTIONS (Qs. 12-18) : In each group of questions below
(a) Eighth from the left (b) Fifth from the right
(c) Sixth from the left (d) Fifth from the left are two/three statements followed by two conclusions numbered
(e) Seventh from the left I and II. You have to take the given statements to be true even if
they seem to be at variance from commonly known facts and
DIRECTIONS (Qs. 5-11) : Study the following information then decide which of the given conclusions logically follows
carefully and answer the given questions : from the two/three statements disregarding commonly known
A, B, C, D, E, F, G and H are sitting around a circle facing the facts.
centre but not necessarily in the same order. Give answer (a) if only conclusion I follows;
• B sits second to left of H's husband. No female is an Give answer (b) if only conclusion II follows;
immediate neighbour of B. Give answer (c) if either conclusion I or conclusion II follows;
• D's daughter sits second to right of F. F is the sister of G. F Give answer (d) if neither conclusion I nor conclusion II follows;
is not an immediate neighbour of H's husband. Give answer (e) if both conclusion I and conclusion II follow.
y
o
u
rs
m
126 IBPS PO/MT Exam 2012

a
h
b
12. Statements : Some exams are tests. No exam is a DIRECTIONS (Qs. 22-29) : Study the following information

o
question.

o
carefully to answer the given questions :

b
Conclusions : I. No question is a test.

.w
II. Some tests are definitely not exams. Eight persons from different banks viz. UCO bank, Syndicate

o
bank, Canara bank, PNB, Dena Bank, Oriental Bank of Commerce,

rd
13. Statements : All forces are energies. All energies are

p
powers. No power is heat. Indian bank and Bank of Maharashtra are sitting in two parallel rows

re
containing four people each, in such a way that there is an equal

s
Conclusions : I. Some forces are definitely not powers.

s
distance between adjacent persons. In row-1 A, B, C and D are seated

.c
II. No heat is force.

o
and all of them are facing south. In row-2 P, Q, R and S are seated and

m
14. Statements : All forces are energies. All energies are
all of them are facing north. Therefore, in the given seating
powers. No power is heat.
arrangement each member seated in a row faces another member of
Conclusions : I. No energy is heat.
the other row. (All the information given above does not necessarily
II. Some forces being heat is a possibility.
represent the order of seating as in the final arrangement)
15. Statements : No note is a coin. Some coins are metals.
• C sits second to right of the person from Bank of
All plastics are notes.
Maharashtra. R is an immediate neighbour of the person
Conclusions : I. No coin is plastic.
who faces the person from Bank of Maharashtra.
II. All plastics being metals is a possibility. • Only one person sits between R and the person for PNB.
16. Statements : No note is a coin. Some coins are metals. Immediate neighbour of the person from PNB faces the
All plastics are notes. person from Canara Bank.
Conclusions : I. No metal is plastic. • The person from UCO bank faces the person from Oriental
II. All notes are plastics. Bank of Commerce. R is not from Oriental Bank of Commerce.
17. Statements : Some symbols are figures. All symbols are P is not from PNB. P does not face the person from Bank of
graphics. Maharashtra.
No graphic is a picture. • Q faces the person from Dena bank. The one who faces S
Conclusions : I. Some graphics are figures. sits to the immediate left of A.
II. No symbol is a picture. • B does not sit at any of the extreme ends of the line. The
18. Statements : All vacancies are jobs. Some jobs are person from Bank of Maharashtra does not face the person
occupations. from Syndicate bank.
Conclusions : I. All vacancies are occupations. 22. Which of the following is true regarding A ?
II. All occupations being vacancies is a (a) The person from UCO bank faces A
possibility. (b) The person from Bank of Maharashtra is an immediate
DIRECTIONS (Qs. 19-21) : Study the following information neighbour of A
carefully to answer the given questions : (c) A faces the person who sits second to right of R
(d) A is from Oriental Bank of Commerce
Each of the six friends, A, B, C, D, E and F scored different
(e) A sits at one of the extreme ends of the line
marks in an examination. C scored more than only. A and E. D
23. Who is seated between R and the person from PNB ?
scored less than only B. E did not score the least. The one who
(a) The person from Oriental Bank of Commerce
scored the third highest marks scored 81 marks. E scored 62 marks.
(b) P
19. Which of the following could possibly be C's score ?
(c) Q
(a) 70 (b) 94
(d) The person from Syndicate bank
(c) 86 (d) 61
(e) S
(e) 81
24. Who amongst the following sit at extreme ends of the rows?
20. Which of the following is true with respect to the given
(a) D and the person from PNB.
information ?
(b) The person from Indian bank and UCO bank.
(a) D's score was definitely less than 60
(c) The person from Dena bank and P.
(b) F scored the maximum marks
(d) The persons from Syndicate bank and D.
(c) Only two people scored more than C
(e) C, Q
(d) There is a possibility that B scored 79 marks
25. Who amongst the following faces the person from Bank of
(e) None is true
Maharashtra ?
21. The person who scored the maximum, scored 13 marks more
(a) The person from Indian bank
than F's marks. Which of the following can be D's score ?
(b) P
(a) 94 (b) 60
(c) R
(c) 89 (d) 78
(d) The person from Syndicate bank
(e) 81
(e) The person from Canara bank
y
o
u
rs
m
IBPS PO/MT Exam 2012 127

a
h
b
26. P is related to Dena bank in the same way as B is related to 31. There are six letters W, A, R, S, N and E. Is 'ANSWER' the

o
PNB based on the given arrangement. To who amongst the word formed after performing the following operations using

o
b
following is D related to, following the same pattern ? these six letters only ?

.w
(a) Syndicate bank (b) Canara bank (I) E is placed fourth to the right of A. S is not placed

o
rd
(c) Bank of Maharashtra (d) Indian bank immediately next to either A or E.

p
(e) Oriental Bank of Commerce

re
(II) R is placed immediately next (either left or right) to E.
27. Four of the following five are alike in a certain way based on

s
W is placed immediately next (either left or right) to S.

s
.c
the given seating arrangement and thus form a group. Which
(III) Both N and W are placed immediately next to S. The

o
is the one that does not belong to that group ?

m
word does not begin with R. A is not placed immediately
(a) Canara bank (b) R
next to W.
(c) Syndicate bank (d) Q
(e) Oriental Bank of Commerce 32. Point D is in which direction with respect to Point B ?
28. Who amongst the following is from Syndicate bank ? (I) Point A is to the west of Point B. Point C is to the north
(a) C (b) R of Point B. Point D is to the south of Point C.
(c) P (d) D (II) Point G is to the south of Point D. Point G is 4 m from
(e) A Point B. Point D is 9 m from Point B.
29. C is from which of the following banks ? (III) Point A is to the west of Point B. Point B is exactly
(a) Dena bank midway between Points A and E. Point F is to the south
(b) Oriental Bank of Commerce of Point E. Point D is to the west of Point F.
(c) UCO bank
33. How is 'one' coded in the code language ?
(d) Syndicate bank
(I) 'one of its kind' is coded as 'zo pi ko fe' and 'in kind and
(e) Canara bank
cash' is coded as 'ga, to ru ko'
DIRECTIONS (Qs. 30-34) : Each of the questions below consists
(II) Its point for origin' is coded as 'ba le fe mi' and 'make a
of a question and three statements numbered I, II and III given
point clear' is coded as 'yu si mi de'
below it. You have to decide whether the data provided in the
statements are sufficient to answer the question. Read all the (III) 'make money and cash' is coded as 'to mi ru hy' and
three statements and — 'money of various kind' is coded as 'qu ko zo hy'.
Give answer (a) if the data in Statement I and II are sufficient to 34. Are all the four friends viz. A, B, C and D who are sitting
answer the question, while the data in Statement III are not around a circular table, facing the centre ?
required to answer the question (I) B sits second to right of D. D faces the centre. C sits to
Give answer (b) if the data in Statement I and III are sufficient to immediate right of both B and D.
answer the question, while the data in Statement II are not (II) A sits to immediate left of B. C is not an immediate
required to answer the question neighbour of A. C sits to immediate right of D.
Give answer (c) if the data in Statement II and III are sufficient to
(III) D is an immediate neighbour of both A and C. B sits to
answer the question, while the data in Statement I are not required
the immediate left of A. C sits to the immediate right of
to answer the question
B.
Give answer (d) if the data in either Statement I alone or
Statement II alone or Statement III alone are sufficient to answer DIRECTION (Qs. 35) : Read the following information carefully
the question. and answer the question which follows :
Give answer (e) if the data in all the Statements I, II and III
Farmers found using chemical fertilizers in the organic-farming
together are necessary to answer the question.
area of their farms would be heavily fined.
30. Among six people P, Q, R, S, T and V each lives on a different 35. Which of the following statements is an assumption implicit
floor of a six storey building having six floors numbered one in the given statement ? (An assumption is something
to six (the ground floor is numbered 1, the floor above it, supposed or taken for granted.)
number 2 and so on and the topmost floor is numbered 6). (a) Chemical fertilisers harm the crop.
Who lives on the topmost floor ?
(b) A farm's area for organic and chemical farming is
(I) There is only one floor between the floors on which R
different.
and Q live. P lives on an even numbered floor.
(II) T does not live on an even numbered floor. Q lives on (c) Farmers who do not use chemical fertilizers in the
an even numbered floor. Q does not live on the topmost chemical farming area would be penalized as well.
floor. (d) All farmers undertake both these kinds of farming
(III) S lives on an odd numbered floor. There are two floors (chemical as well as organic) in their farms.
between the floors on which S and P live. T lives on a (e) Organic fertilizers are banned in the area for chemical
floor immediately above R's floor. farming.
y
o
u
rs
m
128 IBPS PO/MT Exam 2012

a
h
b
DIRECTION (Qs. 36) : Read the following information carefully (D) Supermarkets charge the smaller brands 10% higher than

o
the amount charged to the bigger brands.

o
and answer the questions which follow:

b
(E) Being outnumbered by the bigger brands, visibility of the

.w
Small brands are now looking beyond local grocery stores smaller brands at local grocery stores is much lower as

o
and are tying up with Supermarkets such as Big Bazaar to pull compared to the supermarkets.

rd
their business out of troubled waters. (F) Smaller brands are currently making substantial losses in

p
re
36. Which of the following can be inferred from the given their businesses.

s
information ? (An inference is something that is not directly 37. Which of the statements numbered (A), (B), (C), (D), (E) and

s
.c
stated but can be inferred from the given information) (F) can be assumed from the facts/ information given in the

o
(a) Merchandise of smaller brands would not be available

m
statement ? (An assumption is something supposed or taken
at local grocery stores in the near future. for granted)
(b) Smaller brands cannot compete with bigger ones in a (a) Only (A) (b) Only (B)
supermarket set-up. (c) Both (B) and (C) (d) Both (D) and (E)
(c) There is a perception among small brands that sale in a (e) Only (F)
supermarket is higher than that of small grocery stores. 38. Which of the statements numbered (A), (B), (C), (E) and (F)
(d) Supermarkets generate more revenue by selling represents a disadvantage of the small grocery stores over
products of bigger brands as compared to the smaller the Supermarkets from the perspective of a smaller brand ?
ones. (a) Only (A) (b) Only (C)
(e) Smaller brands have always had more tie-ups with (c) Only (E) (d) Only (F)
supermarkets as compared to small grocery stores. (e) Both (B) and (C)
DIRECTIONS (Qs. 37-40) : These questions are based on the 39. Which of the statements (A), (B), (C), (D) and (E) mentioned
information given above and the sentences labeled (A), (B), above represents a reason for the shift from local grocery
(C), (D), (E) and (F) as given below. stores to supermarkets by the smaller brands ?
(a) Only(A) (b) Only (B)
(A) A smaller brand manufacturing a certain product of quality (c) Only (D) (d) Both (A) and (D)
comparable with that of a bigger brand. makes much more (e) Both (C) and (E)
profit from the local grocery stores than from the 40. Which of the statements numbered (A), (B), (C), (E) and (F)
supermarkets. mentioned above would prove that the step taken by the
(B) As the supermarkets have been set up only in bigger cities smaller brands (of moving to supermarkets) may not
at present, this step would fail to deliver results in the smaller necessarily be correct ?
cities. (a) Only (A) (b) Only (C)
(C) Supermarkets help the smaller brands to break into newer (c) Only (E) (d) Only (F)
markets without investing substantially in distribution. (e) Both (B) and (E)

DIRECTIONS (Qs. 41-45) : In each of the questions given below which one of the five answer figures on the right should come after
the problem figures on the left, if the sequence were continued?
41. PROBLEM FIGURES ANSWER FIGURES

(a) (b) (c) (d) (e)


42. PROBLEM FIGURES ANSWER FIGURES

P O A S PB P CS B PB C O A B O C AP O A BO B P A O PA B
S O A P S
C S A O
B C A B O O C A PC S PB S C SS C C S
(a) (b) (c) (d) (e)
43. PROBLEM FIGURES ANSWER FIGURES
S C N B = = B B N N = O S B = B = S OS O B S O =
O C C C C
C S S C O S S O
= B N O C O N = S O B C = NN C N B = N B N
(a) (b) (c) (d) (e)
y
o
u
rs
m
IBPS PO/MT Exam 2012 129

a
h
b
44. PROBLEM FIGURES ANSWER FIGURES

o
o
b
.w
o
rd
p
re
(a) (b) (c) (d) (e)

s
s
45. PROBLEM FIGURES ANSWER FIGURES

.c
o
m
(a) (b) (c) (d) (e)
DIRECTIONS (Qs. 46-50) : In each of the following questions
below, the figures follow a series/sequence (like Q.41-45). One
52. ( ) ( )
é 5 7 + 7 + 4 7 + 8 7 ù - (19) 2 = ?
ë û
and only one out of the five figures does not fit in the series/ (a) 143 (b) 72 7
sequence. The number of that figure is your answer.
(c) 134 (d) 70 7
46.
(e) None of these
53. (4444 ÷ 40) + (645 ÷ 25) + (3991÷ 26) = ?
(a) 280.4 (b) 290.4
(c) 295.4 (d) 285.4
(a) (b) (c) (d) (e)
(e) None of these
47. 54. 33124 ´ 2601 – (83)2 = (?)2 + (37)2
(a) 37 (b) 33
(c) 34 (d) 28
(e) None of these
(a) (b) (c) (d) (e) 17 51 1 3
55. 5 ´ 4 ´11 + 2 = ?
37 52 7 4
48.
(a) 303.75 (b) 305.75
3 1
(c) 303 (d) 305
4 4
(a) (b) (c) (d) (e) (e) None of these

49. DIRECTIONS (Qs. 56-60) : What approximate value should come


in place of the question mark (?) in the following questions?
(Note : You are not expected to calculate the exact value.)
56. 8787 ÷ 343 × 50 = ?
(a) (b) (c) (d) (e) (a) 250 (b) 140
50. (c) 180 (d) 100
(e) 280
3
57. 54821 × (303 ÷ 8) = (?)2
(a) 48 (b) 38
(a) (b) (c) (d) (e) (c) 28 (d) 18
(e) 58
QUANTITATIVE APTITUDE 5 7
58. of 4011.33 + of 3411.22 = ?
8 10
DIRECTIONS (Qs. 51-55) : What will come in place of the (a) 4810 (b) 4980
question mark (?) in the following questions ? (c) 4890 (d) 4930
(e) 4850
51. 4003 × 77 – 21015 = ? × 116
59. 23% of 6783 + 57% of 8431 = ?
(a) 2477 (b) 2478 (a) 6460 (b) 6420
(c) 2467 (d) 2476 (c) 6320 (d) 6630
(e) None of these (e) 6360
y
o
u
rs
m
130 IBPS PO/MT Exam 2012

a
h
60. 335.01 × 244.99 ÷ 55 = ?

b
66. Which of the following vehicles travelled at the same speed

o
(a) 1490 (b) 1550 on both the days ?

o
b
(c) 1420 (d) 1590 (a) Vehicle A (b) Vehicle C

.w
(e) 1400 (c) Vehicle F (d) Vehicle B

o
(e) None of these

rd
DIRECTIONS (Qs. 61-65) : In each of these questions a number

p
series is given. In each series only one number is wrong. Find 67. What was the difference between the speed of vehicle A on

re
out the wrong number. day 1 and the speed of vehicle C on the same day ?

s
s
(a) 7 km/hr. (b) 12 km/hr.

.c
61. 5531 5506 5425 5304 5135 4910 4621

o
(a) 5531 (b) 5425 (c) 11 km/hr. (d) 8 km/hr.

m
(c) 4621 (d) 5135 (e) None of these
(e) 5506 68. What was the speed of vehicle C on day 2 in terms of meters
62. 6 7 9 13 26 37 69 per second ?
(a) 7 (b) 26 (a) 15.3 (b) 12.8
(c) 69 (d) 37 (c) 11.5 (d) 13.8
(e) 9 (e) None of these
63. 1 3 10 36 152 760 4632 69. The distance travelled by vehicle F on day 2 was
(a) 3 (b) 36 approximately what percent of the distance travelled by it
(c) 4632 (d) 760 on day 1 ?
(e) 152 (a) 80 (b) 65
64. 7 4 5 9 20 51 160.5 (c) 85 (d) 95
(a) 4 (b) 5 (e) 90
(c) 9 (d) 20 70. What is the respective ratio between the speeds of vehicle
(e) 51 D and vehicle E on day 2 ?
65. 157.5 45 15 6 3 2 1 (a) 15:13 (b) 17:13
(a) 1 (b) 2 (c) 13:11 (d) 17:14
(c) 6 (d) 157.5 (e) None of these
(e) 45 71. An article was purchased for ` 78,350/-. Its price was marked
DIRECTIONS (Qs. 66-70) : Study the following graph and table up by 30%. It was sold at a discount of 20% on the marked
carefully and answer the questions given below : up price. What was the profit percent on the cost price ?
TIME TAKEN TO TRAVEL (IN HOURS) BY SIX (a) 4 (b) 7
VEHICLES ON TWO DIFFERENT DAYS (c) 5 (d) 3
20 (e) 6
72. When X is subtracted from the numbers 9,15 and 27, the
15 remainders are in continued proportion. What is the value
TIME (IN HOURS)

of X ?
(a) 8 (b) 6
10 Day 1
(c) 4 (d) 5
Day 2
(e) None of these
5
73. What is the difference between the simple and compound
interest on ? 7,300/- at the rate of 6 p.c.p.a. in 2 years ?
0 (a) ` 29.37/- (b) ` 26.28/-
A B C D E F (c) ` 31.41/- (d) ` 23.22/-
VEHICLES (e) ` 21.34/-
DISTANCE COVERED (IN KILOMETERS) BY 74. Sum of three consecutive numbers is 2262. What is 41 % of
SIX VEHICLES ON EACH DAY the highest number ?
(a) 301.51 (b) 303.14
Vehicle Day 1 Day 2
(c) 308.73 (d) 306.35
A 832 864
(e) 309.55
B 516 774 75. In how many different ways can the letters of the word
C 693 810 ‘THERAPY’ be arranged so that the vowels never come
D 552 765 together ?
E 935 546 (a) 720 (b) 1440
F 703 636 (c) 5040 (d) 3600
(e) 4800
y
o
u
rs
m
IBPS PO/MT Exam 2012 131

a
h
b
DIRECTIONS (Qs. 76-80) : Study the following pie-chart and 80. What is the total number of mobile phones sold of company

o
B during August and September together ?

o
table carefully and answer the questions given below :

b
(a) 10,000 (b) 15,000

.w
PERCENTAGEWISE DISTRIBUTION OF THE (c) 10,500 (d) 9,500

o
NUMBER OF MOBILE PHONES SOLD BY

rd
(e) None of these
A SHOPKEEPER DURING SIX MONTHS

p
re
Total number of mobile phones sold = 45,000 DIRECTIONS (Qs. 81-85) : Study the following information

s
and answer the questions that follow :

s
.c
o
December The premises of a bank are to be renovated. The renovation

m
16% is in terms of flooring. Certain areas are to be floored either with
November marble or wood. All rooms/halls and pantry are rectangular. The
July
12% area to be renovated comprises of a hall for customer transaction
17%
October measuring 23 m by 29 m, branch manager's room measuring 13 m
8% by 17 m, a pantry measuring 14 m by 13 m, a record keeping cum
server room measuring 21m by 13 m and locker area measuring 29
September August
22%
m by 21 m. The total area of the bank is 2000 square meters. The
25% cost of wooden flooring is ` 170/- per square meter and the cost
of marble flooring is ` 190/- per square meter. The locker area,
record keeping cum server room and pantry are to be floored with
The respective ratio between the number of mobile phones marble. The branch manager's room and the hall for customer
sold of company A and company B during six months transaction are to be floored with wood. No other area is to be
renovated in terms of flooring.
Month Ratio 81. What is the respective ratio of the total cost of wooden
flooring to the total cost of marble flooring ?
July 8:7
(a) 1879 : 2527 (b) 1887 : 2386
August 4:5
(c) 1887 : 2527 (d) 1829 : 2527
September 3:2 (e) 1887 : 2351
October 7:5 82. If the four walls and ceiling of the branch managers room
November 7:8 (The height of the room is 12 meters) are to be painted at the
December 7:9 cost of ` 190/- per square meter, how much will be the total
cost of renovation of the branch manager's room including
76. What is the respective ratio between the number of mobile the cost of flooring ?
phones sold of company B during July and those sold during (a) ` 1,36,800/- (b) ` 2,16,660/-
December of the same company ? (c) ` 1,78.790/- (d) ` 2,11,940/-
(a) 119:145 (b) 116:135 (e) None of these
(c) 119 :135 (d) 119:130 83. If the remaining area of the bank is to be carpeted at the rate
(e) None of these of ` 110/- per square meter, how much will be the increment
77. If 35% of the mobile phones sold by company A during in the total cost of renovation of bank premises ?
November were sold at a discount, how many mobile phones (a) ` 5,820/- (b) ` 4,848/-
of company A during that month were sold without a (c) ` 3,689/- (d) ` 6,890/-
discount? (e) None of these
(a) 882 (b) 1635 84. What is the percentage area of the bank that is not to be
renovated ?
(c) 1638 (d) 885
(a) 2.2 (b) 2.4
(e) None of these
(c) 4.2 (d) 4.4
78. If the shopkeeper earned a profit of `433/- on each mobile (e) None of these
phone sold of company B during October, what was his 85. What is the total cost of renovation of the hall for customer
total profit earned on the mobile phones of that company transaction and the locker area ?
during the same month ? (a) ` 2,29,100/- (b) ` 2,30,206/-
(a) ` 6,49,900/- (b) ` 6,45,900/- (c) ` 2,16,920/- (d) ` 2,42,440/-
(c) ` 6,49,400/- (d) ` 6,49,500/- (e) None of these
(e) None of these 86. A certain amount was to be distributed among A, B and C in
79. The number of mobile phones sold of company A during the ratio 2 :3 :4 respectively, but was erroneously distributed
July is approximately what percent of the number of mobile in the ratio 7:2:5 respectively. As a result of this, B got `40
phones sold of company A during December ? less. What is the amount ?
(a) 110 (b) 140 (a) ` 210/- (b) ` 270/-
(c) 150 (d) 105 (c) ` 230/- (d) ` 280/-
(e) 130 (e) None of these
y
o
u
rs
m
132 IBPS PO/MT Exam 2012

a
h
b
87. Rachita enters a shop to buy ice-creams, cookies and

o
59 99
pastries. She has to buy atleast 9 units of each. She buys

o
(c) (d)

b
more cookies than ice-creams and more pastries than cookies. 190 190

.w
She picks up a total of 32 items. How many cookies does

o
77

rd
she buy ? (e)
190

p
(a) Either 12 or 13 (b) Either 11 or 12

re
(c) Either 10 or 11 (d) Either 9 or 11 92. Akash scored 73 marks in subject A. He scored 56% marks

s
s
in subject B and X marks in subject C. Maximum marks in

.c
(e) Either 9 or10
each subject were 150. The overall percentage marks obtained

o
88. The fare of a bus is ` X for the first five kilometers and

m
` 13/- per kilometer thereafter. If a passenger pays ` 2402/- by Akash in all the three subjects together were 54%. How
for a journey of 187 kilometers, what is the value of X ? many marks did he score in subject C ?
(a) ` 29/- (b) ` 39/- (a) 84 (b) 86
(c) ` 36/- (d) ` 31/- (c) 79 (d) 73
(e) None of these (e) None of these
93. The area of a square is 1444 square meters. The breadth of a
89. The product of three consecutive even numbers is 4032.
The product of the first and the third number is 252. What is rectangle is 1/4th the side of the square and the length of
five times the second number ? the rectangle is thrice the breadth. What is the difference
(a) 80 (b) 100 between the area of the square and the area of the rectangle?
(c) 60 (d) 70 (a) 1152.38 sq.mtr. (b) 1169.33 sq.mtr
(c) 1181.21 sq.mtr. (d) 1173.25 sq.mtr
(e) 90
(e) None of these
90. The sum of the ages of 4 members of a family 5 years ago
94. ` 73,689/- are divided between A and B in the ratio 4 :7.
was 94 years. Today, when the daughter has been married
off and replaced by a daughter-in-law, the sum of their ages What is the difference between thrice the share of A and
is 92. Assuming that there has been no other change in the twice the share of B ?
family structure and all the people are alive, what is the (a) ` 36,699/- (b) ` 46,893/-
difference in the age of the daughter and the daughter-in- (c) ` 20.097/- (d) ` 26,796/-
law ? (e) ` 13.398/-
(a) 22 years (b) 11 years 95. A and B together can complete a task in 20 days. B and C
(c) 25 years (d) 19 years together can complete the same task in 30 days. A and C
(e) 15 years together can complete the same task in 40 days. What is the
91. A bag contains 13 white and 7 black balls. Two balls are respective ratio of the number of days taken by A when
drawn at random. What is the probability that they are of completing the same task alone to the number of days taken
the same colour ? by C when completing the same task alone ?
(a) 2 : 5 (b) 2 : 7
41 21
(a) (b) (c) 3 : 7 (d) 1 : 5
190 190 (e) 3 : 5
DIRECTIONS (Qs. 96-100) : Study the following information and answer the questions that follow :
THE GRAPH GIVEN BELOW REPRESENTS THE PRODUCTION (IN TONNES) AND SALES (IN TONNES)
OF COMPANY A FROM 2006-2011
PRODUCTION AND SALES (In tonnes)

900

800

700

600 Production
Sales
500

400

300

200
2006 2007 2008 2009 2010 2011
YEARS
y
o
u
rs
m
IBPS PO/MT Exam 2012 133

a
h
b
The table given below represents the respective ratio of the 103. Which of the following Ministries along with Planning

o
production (in tonnes) of Company A to the production (in Commission of India has decided to set up a Corpus Fund

o
b
tonnes) of Company B, and the respective ratio of the sales of ` 500 crore, so that Tribals in Naxal-hit areas can be

.w
(in tonnes) of Company A to the sales (in tonnes) of provided proper means of livelihood ?

o
(a) Ministry of Rural Development

rd
Company B.
(b) Ministry of Home Affairs

p
re
(c) Ministry of Tribal Affairs
Year Production Sales

s
(d) Ministry of Corporate Affairs

s
2006 5:4 2:3

.c
(e) Ministry of Finance

o
2007 8:7 11:12 104. Which of the following was the issue over which India

m
2008 3:4 9:14 decided to vote against Sri Lanka in the meeting of one of
2009 11:12 4:5 the UN governed bodies/agencies ?
2010 14:13 10:9 (a) Violations of human rights in Sri Lanka
2011 13:14 1:1 (b) Allowing China to establish a military base in Indian
ocean
96. What is the approximate percentage increase in the (c) Issue of subsidy on agricultural products in the meeting
production of Company A (in tonnes) from the year 2009 to of the WTO
the production of Company A (in tonnes) in the year 2010 ? (d) Allowing part of Sri Lanka to become an independent
(a) 18 (b) 38 country governed by LTTE
(c) 23 (d) 27 (e) Sri Lanka's claim to become a permanent member of
(e) 32 UN Security Council
97. The sales of Company A in the year 2009 was approximately 105. The term 'Smart Money" refers to __________ .
what percent of the production of Company A in the same (a) Foreign Currency (b) Internet Banking
year ? (c) US Dollars (d) Travelers' cheques
(a) 65 (b) 73 (e) Credit Cards
106. Which one of the following is not a 'Money Market
(c) 79 (d) 83
Instrument' ?
(e) 69
(a) Treasury Bills (b) Commercial Paper
98. What is the average production of Company B (in tonnes) (c) Certificate of Deposit (d) Equity Shares
from the year 2006 to the year 2011 ? (e) None of these
(a) 574 (b) 649 107. Which one of the following is a retail banking product ?
(c) 675 (d) 593 (a) Home Loans (b) Working capital finance
(e) 618 (c) Corporate term loans (d) Infrastructure financing
99. What is the respective ratio of the total production (in (e) Export Credit
tonnes) of Company A to the total sales (in tonnes) of 108. Which of the following Statements is TRUE about political
Company A ? situation in Mali, where a military coup burst out recently ?
(a) 81 : 64 (b) 64 : 55 (a) General elections were due there in March/April 2012
(c) 71 : 81 (d) 71 : 55 but did not take place
(e) 81 : 55 (b) The country was under the control of US army since
100. What is the respective ratio of production of Company B (in last 18 months
tonnes) in the year 2006 to production of Company B (in (c) Army of the country was not happy as Amadou Toure
tonnes) in the year 2008 ? was made President without elections for the next five
years
(a) 2 : 5 (b) 4 : 5
(d) Coup broke out in Mali when Amadou Toure. the
(c) 3 : 4 (d) 3 : 5
Military Chief got seriously injured in a bomb blast
(e) 1 : 4
(e) NATO and its associates had planned a coup there
GENERAL AWARENESS 109. In the summit of which of the following organizations/group
of Nations it was decided that all members should enforce
101. A money deposited at a bank that cannot be withdrawn for Budget Discipline ?
a preset fixed period of time is known as a __________ . (a) G-8 (b) OPEC
(a) Term deposit (b) Checking Account (c) European Union (d) SAARC
(c) Savings Bank Deposit (d) No Frills Account (e) G-20
110. As per newspaper reports, India is planning to use 'SEU' as
(e) Current Deposit
fuel in its 700 MW nuclear reactors being developed in new
102. A worldwide financial messaging network which exchanges
plants. What is full form of 'SEU' as used here ?
messages between banks and financial institutions is known (a) Safe Electrical Units
as __________ . (b) Small Electrical Unite
(a) CHAPS (b) SWIFT (c) Slightly Enriched Uranium
(c) NEFT (d) SFMS (d) Sufficiently Enriched Units
(e) CHIPS (e) Safely Enriched Uranium
y
o
u
rs
m
134 IBPS PO/MT Exam 2012

a
h
b
111. Technological Advancement in the recent times has given a 120. What was the reason owing to which Enrica Lexie, an Italian

o
new dimension to banks, mainly to which one of the ship was detained by the Port Authorities in Kerala and was

o
b
following aspects ? brought to Cochin port for inspection and search ?

.w
(a) New Age Financial Derivatives (a) It was carrying objectionable material

o
(b) It was involved in sea piracy

rd
(b) Service Delivery Mechanism
(c) It was detained as the crew killed two Indian fishermen

p
(c) Any Banking

re
(d) Any Type Banking (d) The ship started sailing without making payments of

s
iron ore it loaded

s
(e) Multilevel Marketing

.c
112. When there is a difference between all receipts and (e) It was detained as it was dumping nuclear waste in

o
m
expenditure of the Govt. of India, both capital and revenue, deep sea
it is called __________ . 121. Which of the following agencies/organizations recently gave
(a) Revenue Deficit (b) Budgetary Deficit 'go-ahead' verdict to India's one of the most controversial
(c) Zero Budgeting (d) Trade Gap project of inter linking rivers? (some of the rivers are
international rivers)
(e) Balance of payment problem
(a) UN Food and Agriculture Organisation
113. Which of the following is NOT a function of the Reserve
(b) World Meteorological Organisation
Bank of India ?
(c) International Court of Justice
(a) Fiscal Policy Functions (d) Central Water Commission
(b) Exchange Control Functions (e) Supreme Court of India
(c) Issuance, Exchange and destruction of currency notes 122. Who among the following was the Captain of the Australian
(d) Monetary Authority Functions Team which played 4 Test matches against India in January
(e) Supervisory and Control Functions 2012 and won all of them ?
114. Which of the following is NOT required for opening a bank (a) Ricky Ponting (b) Michael Clarke
account ? (c) Nathan Lyon (d) Stuart Clark
(a) Identity Proof (b) Address Proof (e) Andrew Symonds
(c) Recent Photographs (d) Domicile Certificate 123. The committee on review of National Small Saving Fund
(e) None of these (NSSF) was headed by __________ .
115. The Golden Jubilee of Afro-Asian Rural Development (a) Dr. C. Rangarajan (b) Mr. U. K. Sinha
organization was organized in which of the following places (c) Dr. Y. V. Reddy (d) Mrs.Shyamala Gopinath
in March 2012 ? (e) Dr. Usha Thorat
(a) Dhaka (b) Tokyo 124. Banking Ombudsman Scheme is applicable to the business
(c) Cairo (d) Kuala Lampur of __________ .
(e) New Delhi (a) All scheduled commercial banks excluding RRBs
116. What is the maximum deposit amount insured by DICGC ? (b) All scheduled commercial banks including RRBs
(a) ` 2,00,000 per depositor per bank (c) Only Public Sector Banks
(b) ` 2,00,000 per depositor across all banks (d) All Banking Companies
(c) ` 1,00,000 per depositor per bank (e) All scheduled banks except private banks
(d) ` 1,00,000 per depositor across all banks 125. The 5th Asian tndoor Athletics Championship was organized
(e) None of these in February 2012 in __________ .
117. The present Foreign Trade policy of India will continue till- (a) Bangladesh (b) India
(a) December 2012 (b) March 2013 (c) Qatar (d) China
(c) March 2014 (d) June 2013 (e) South Korea
(e) December 2014 126. Nationalization of banks aimed at all of the following except
118. With reference to a cheque which of the following is the __________ .
“drawee bank” ? (a) Provision of adequate credit for agriculture, SME and
(a) The bank that collects the cheque exports
(b) The payee's bank (b) Removal of control by a few capitalists
(c) The endorsee's bank (c) Provision of credit to big industries only
(d) The endorser's bank (d) Access of banking to masses
(e) The bank upon which the cheque is drawn (e) Encouragement of a new class of entrepreneurs
119. In which of the following fund transfer mechanisms, can 127. Who among the following was the Chief Guest at the three-
funds be moved from one bank to another and where the day Pravasi Bharatiya Divas function held in January 2012 ?
transaction is settled instantly without being bunched with (a) Kamla Persad - Bissessar
any other transaction ? (b) Ram Baran Yadav
(a) RTGS (b) NEFT (c) Lakshmi Mittal
(c) TT (d) EFT (d) Salman Rushdie
(e) MT (e) Benjamin Netanyahu
y
o
u
rs
m
IBPS PO/MT Exam 2012 135

a
h
b
128. Which of the following banks is headed by a woman 137. Who among the following Indian Lawn Tennis player won a

o
CEO __________ . Doubles of Australian Open 2012 ?

o
b
(a) Bank of Baroda (b) HDFC Bank (a) Mahesh Bhupati (b) Kamlesh Mehta

.w
(c) Central Bank of India (d) Punjab National Bank (c) Leander Paes (d) Sania Mirza

o
(e) ICICI Bank

rd
(e) Achanta Sarath Kamal
129. As per revised RBI Guidelines, Provision on secured portion

p
138. 'Akula-11' Class 'K-152 Nerpa' was inducted in Indian Navy

re
of loan accounts classified as Doubtful Assets for more recently. These are __________ .

s
than one year and upto 3 (three) years is to be made at the

s
(a) Aircrafts (b) Radar System

.c
rate of __________ . (c) Submarines Missiles (d) Warship

o
m
(a) 15% (b) 20% (e) Submarines
(c) 40% (d) 25%
139. Which of the following awards was conferred upon Late
(e) 30%
Mario De Miranda (Posthumously) in January 2012 ?
130. As per the provisions in the Food Security Bill - 2011, per
(a) Padma Vibhushan (b) Bharat Ratna
month how much food grain should be given to each person
(c) Kalidas Samman (d) Saraswati Samman
of the target group ?
(a) 5 kg. (b) 7 kg. (e) Padmashri
(c) 9 kg. (d) 10 kg. 140. Bad advances of a Bank are called __________ .
(e) 20 kg. (a) Bad debt (b) Book debt
131. Which of the following acts in vogue in India is against the (c) Non Performing Asset (d) Out of order accounts
thinking of raising school fee as per demand of the market (e) Overdrawn accounts
forces ? 141. Axis Bank is a __________ .
(a) Prevention of Corruption Act (a) Public Sector Bank (b) Private Sector Bank
(b) Child Labour (Prohibition & Regulation) Act (c) Co-operative Bank (d) Foreign Bank
(c) ShardaAct (e) Gramin Bank
(d) Right to Education Act 142. By increasing repo rate, the economy may observe the
(e) MG National Rural Employment Guarantee Act following effects __________.
132. Which of the following states has launched ‘Panch (a) Rate of interest on loans and advances will be costlier
Parmeshwar Yojana’ under which panchayats are allowed (b) Industrial output would be affected to an extent
to use the fund for developing infrastructure and basic (c) Banks will increase rate of interest on deposits
amenities in villages ? (d) Industry houses may borrow money from foreign
(a) Uttar Pradesh (b) Maharashtra countries
(c) Gujarat (d) Madhya Pradesh (e) All of these
(e) Tamil Nadu 143. Increased interest rates, as is existing in the economy at
133. Base Rate is the rate below which no Bank can allow their present will __________.
lending to anyone. Who sets up this ‘Base Rate’ for Banks? (a) Lead to higher GDP growth
(a) Individual Banks, Board (b) Lead to lower GDP growth
(b) Ministry of Commerce (c) Mean higher cost of raw materials
(c) Ministry of Finance (d) Mean lower cost of raw materials
(d) RBI (e) Mean higher wage bill
(e) Interest Rate Commission of India 144. Which of the following schemes is launched to provide
134. National Table Tennis Championship was organized in pension to people in unorganized sectors in India ?
January 2012 in __________ . (a) Swabhiman (b) Jeevan Dhara
(a) Mumbai (b) Delhi
(c) Jeevan Kalyan (d) ASHA
(c) Hyderabad (d) Lucknow
(e) Swavalamban
(e) Jaipur
145. The 10th Basic Ministerial Meeting on Climate Change was
135. Who among the follwing is the author of the book
organized in February 2012 in __________.
'Nirbasan'?
(a) Tokyo (b) Beijing
(a) Mahashweta Devi (b) Taslima Nasreen
(c) Sunil Gangopadhyay (d) Vikram Seth (c) Manila (d) Moscow
(e) Kiran Desai (e) New Delhi
136. What is a Debit Card' ? 146. Finance Ministry has asked the Reserve Bank of India to
(a) It is a card issued by a Rating Agency allow common ATM's that will be owned and managed by
(b) It is a card which can be used for withdrawing cash or non-banking entities hoping to cut transaction costs for
making payment even in the absence of any balance in banks. Such ATM's are known as __________.
the account (a) Black Label ATM's
(c) It is a card which can be used for withdrawing cash or (b) off site ATM's
making payment if there is balance in the account (c) on site ATM's or red ATM's
(d) It is a card which carries prepaid balance (d) third party ATM's
(e) It is a card which can be used for making STD calls (e) white label ATM's
y
o
u
rs
m
136 IBPS PO/MT Exam 2012

a
h
b
147. Which of the following schemes of the Govt. of India has 155. Fourth-generation mobile technology provides enhanced

o
provided electricity to 99000 villages and total 1.7 crore capabilities allowing the transfer of both __________ data,

o
b
households uptill now ? including full-motion video, high-speed Internet access, and

.w
(a) Kutir Jyoti videoconferencing.

o
(a) video data and information

rd
(b) Rajiv Gandhi Grameen Vidyutikaran Yojana
(b) voice and nonvoice

p
(c) Bharat Nirman

re
(c) music and video
(d) PURA

s
(d) video and audio

s
(e) SEWA

.c
(e) None of these

o
148. Ranbir Kapoor was awarded Best Actor Award in 57th 156. __________ is a form of denial of service attack in which a

m
Filmfare Award Function for his performance in ________. hostile client repeatedly sends SYN packets to every port
(a) No One Killed Jessica on the server using fake IP addresses.
(b) Stanley Ka Dabba (a) Cyborgaming crime (b) Memory shaving
(c) 7 Khoon Maaf (c) Syn flooding (d) Software piracy
(d) Rockstar (e) None of these
(e) Zindagi Na Milegi Dobara 157. Which of these is a point-and-draw device ?
149. An ECS transaction gets bounced and you are unable to (a) mouse (b) scanner
recover your money from your customer. Under which Act (c) printer (d) CD-ROM
criminal action can be initiated ? (e) Keyboard
(a) Indian Penal Code 158. The letter and number of the intersecting column and row is
the __________.
(b) Negotiable Instruments Act
(a) cell location (b) cell position
(c) Criminal Procedure Code
(c) cell address (d) cell coordinates
(d) Payment and Settlements Act
(e) cell contents
(e) Indian Contract Act 159. A set of rules for telling the computer what operations to
150. Mr. Rajendra had filed a complaint with Banking Ombudsman perform is called a __________.
but is not satisfied with the decision. What is the next option (a) procedural language (b) structures
before him for getting his matter resolved ? (c) natural language (d) command language
(a) Write to the CMD of the Bank. (e) programming language
(b) File an appeal before the Finance Minister. 160. A detailed written description of the programming cycle and
(c) File an appeal before the Banking Ombudsman again. the program, along with the test results and a printout of the
(d) File an appeal before the Dy. Governor RBI. program is called __________.
(e) Simply close the matter as going to court involves time (a) documentation (b) output
and money. (c) reporting (d) spec sheets
(e) Directory
COMPUTER KNOWLEDGE 161. Forms that are used to organize business data into rows
and columns are called __________.
151. __________ allows users to upload files to an online site (a) transaction sheets (b) registers
so they can be viewed and edited from another location. (c) business forms (d) sheet-spreads
(a) General-purpose applications (e) spreadsheets
(b) Microsoft Outlook 162. In Power Point, the Header and Footer button can be found
(c) Web-hosted technology on the Insert tab in what group ?
(d) Office Live (a) Illustrations group (b) Object group
(e) None of these (c) Text group (d) Tables group
152. What feature adjusts the top and bottom margins so that (e) None of these
the text is centered vertically on the printed page ? 163. A(n)_________is a set of programs designed to manage
(a) Vertical justifying (b) Vertical adjusting the resources of a computer, including starting the computer,
(c) Dual centering (d) Horizontal centering managing programs, managing memory, and coordinating
(e) Vertical centering tasks between input and output devices.
153. Which of these is not a means of personal communication (a) application suite (b) compiler
on the Internet ? (c) input/output system (d) interface
(a) chat (b) instant messaging (e) operating system (OS)
(c) instanotes (d) electronic mail 164. A typical slide in a slide presentation would not include
(e) None of these __________.
154. What is the overall term for creating, editing, formatting, (a) photo images charts, and graphs
storing, retrieving, and printing a text document ? (b) graphs and clip art
(a) Word processing (b) Spreadsheet design (3) clip art and audio clips
(c) Web design (d) Database management (d) full-motion video
(e) Presentation generation (e) content templates
y
o
u
rs
m
IBPS PO/MT Exam 2012 137

a
h
b
165. The PC productivity tool that manipulates data organized in 173. Different components on the motherboard of a PC unit are

o
rows and columns is called a __________. linked together by sets of parallel electrical conducting lines.

o
b
(a) spreadsheet What are these lines called ?

.w
(b) word processing document (a) Conductors (b) Buses

o
(c) presentation mechanism (c) Connectors (d) Consecutives

rd
(d) database record manager (e) None of these

p
re
(e) EDI creator 174. What is the name given to those applications that combine

s
166. In the absence of parentheses, the order of operation is text, sound, graphics, motion video, and/ or animation ?

s
.c
__________. (a) motionware (b) anigraphics

o
m
(a) Exponentiation, addition or subtraction, multiplication (c) videoscapes (d) multimedia
or division (e) maxomedia
(b) Addition or subtraction, multiplication or division, 175. A USB communication device that supports data encryption
exponentiation for secure wireless communication for notebook users is
(c) Multiplication or division, exponentiation, addition or called a __________.
subtraction (a) USB wireless network adapter
(d) Exponentiation, multiplication or division, addition or (b) wireless switch
subtraction (c) wireless hub
(e) Addition or subtraction, exponentiation, Multiplication (d) router
or division (e) None of these
167. To find the Paste Special option, you use the Clipboard 176. A(n) __________ language reflects the way people think
group on the _____ tab of PowerPoint. mathematically.
(a) Design (b) Slide Show (a) cross-platform programming
(c) Page Layout (d) Insert (b) 3GL business programming
(e) Home (c) event-driven programming
168. An _____ program is one that is ready to run and does not (d) functional
need to be altered in any way. (e) None of these
(a) interpreter (b) high-level 177. When entering text within a document, the Enter key is
(c) compiler (d) COBOL normally pressed at the end of every __________.
(e) executable (a) Line (b) Sentence
169. Usually downloaded into folders that hold temporary (c) Paragraph (d) word
Internet files, _____ are written to your computer's hard (e) file
disk by some of the Web sites you visit. 178. When a real-time telephone call between people is made
(a) anonymous files (b) behaviour files over the Internet using computers, it is called __________.
(c) banner ads (d) large files (a) a chat session (b) an e-mail
(e) cookies (c) an instant message (d) Internet telephony
170. What is the easiest way to change the phrase, revenues, (e) None of these.
profits, gross margin, to read revenues, profits, and gross 179. Which of the following is the first step in sizing a window ?
margin ? (a) Point to the title bar
(a) Use the insert mode, position the cursor before the g (b) Pull down the View menu to display the toolbar
in gross, then type the word and followed by a space (c) Point lo any corner or border
(b) Use the insert mode, position the cursor after the g in (d) Pull down the View menu and change to large icons
gross, then type the word and followed by a space (e) None of these
(c) Use the overtype mode, position the cursor before the 180. Which of the following software could assist someone who
g in gross, then type the word and followed by a space cannot use their hands for computer input ?
(d) Use the overtype mode, position the cursor after the g (a) Video conferencing (b) Speech recognition
in gross, then type the word and followed by a space (c) Audio digitizer (d) Synthesizer
(e) None of these (e) None of these
171. A program, either talk or music, that is made available in 181. ___________ a document means the file is transferred from
digital format tor automatic download over the Internet is another computer to your computer.
called a __________. (a) Uploading
(a) wiki (b) broadcast (b) Really Simple Syndication (RSS)
(c) vodcast (d) blog (c) Accessing
(e) podcast (d) Downloading
172. Which PowerPoint view displays each slide of the (e) Upgrading
presentation as a thumbnail and is useful for rearranging 182. Which computer memory is used for storing programs and
slides ? data currently being processed by the CPU ?
(a) Slide Sorter (b) Slide Show (a) Mass memory (b) Internal memory
(c) Slide Master (d) Notes Page (c) Non-volatile memory (d) PROM
(e) Slide Design (e) None of these
y
o
u
rs
m
138 IBPS PO/MT Exam 2012

a
h
b
183. Computers that control processes accept data in a (a) Internet service provider

o
continuous __________. (b) Internet access provider

o
b
(a) data traffic pattern (b) data highway (c) Application service provider

.w
(c) infinite loop (d) feedback loop (d) Application access provider

o
rd
(e) slot (e) Outsource agency

p
184. What refers to a set of characters of a particular design ? 192. A(n) ________ allows you to access your e-mail from

re
(a) keyface (b) formation anywhere.

s
s
(c) calligraph (d) stencil (a) Forum (b) Webmail interface

.c
o
(e) typeface (c) Message Board (d) Weblog

m
185. _________ is used by public and private enterprises to (e) None of these
publish and share financial information with each other and 193. Which of the following would you find on Linkedln ?
industry analysts across all computer platforms and the (a) Games (b) Connections
Internet. (c) Chat (d) Applications
(a) Extensible Markup Language (EML) (e) None of these
(b) Extensible Business Reporting Language (XBRL) 194. ________ is a technique that is used to send more than one
(c) Enterprise Application Integration (EAI) call over a single line.
(d) Sales Force Automation (SFA) software (a) Digital transmission (b) Infrared transmission
(e) None of these (c) Digitizing (d) Streaming
186. Which part of the computer is used for calculating and (e) Multiplexing
comparing ? 195. The Search Companion can __________.
(a) ALU (b) Control unit (a) Locate all files containing a specified phrase
(c) Disk unit (d) Modem (b) Restrict its search to a specified set of folders
(e) None of these (c) Locate all files containing a specified phrase and restrict
187. The method of Internet access that requires a phone line, its search to a specified set of folders
but offers faster access speeds than dial-up is the _____ (d) Cannot locate all files containing a specified phrase or
connection. restrict its search to a specified set of folders
(a) cable access (e) None of these
(b) satellite access 196. Which of the following cannot be part of an email address ?
(c) fiber-optic service (a) Period (.) (b) At sign (@)
(d) Digital Subscriber Line (DSL) (c) Space () (d) Underscore (_)
(e) modem (e) None of these
188. ___________ software creates a mirror image of the entire 197. Which of the following must be contained in a URL ?
hard disk, including the operating system, applications, files, (a) a protocol identifier
and data. (b) the letters, www.
(a) Operating system (b) Backup software (c) the unique registered domain name
(c) Utility programs (d) Driver imaging (d) www. and the unique registered domain name
(e) None of these (e) a protocol identifier, www. and the unique registered
189. What is a URL? domain name
(a) a computer software program 198. Which of the following information systems focuses on
(b) a type of programming object making manufacturing processes more efficient and of higher
(c) the address of a document or "page" on the World quality ?
Wide Web (a) Computer-aided manufacturing
(d) an acronym for Unlimited Resources for Learning (b) Computer-integrated manufacturing
(e) a piece of hardware (c) Computer-aided software engineering
190. What is the significance of a faded (dimmed) command in a (d) Computer-aided system engineering
pull-down menu ? (e) None of these
(a) The command is not currently accessible 199. A mistake in an algorithm that causes incorrect results is
(b) A dialog box appears if the command is selected called a __________.
(c) A Help window appears if the command is selected (a) logical error (b) syntax error
(d) There are no equivalent keystrokes for the particular (c) procedural error (d) compiler error
command (e) machine error
(e) None of these 200. A device for changing the connection on a connector to a
191. Your business has contracted with another company to have different configuration is-
them host and run an application for your company over (a) a converter (b) a component
the Internet. The company providing this service to your (c) an attachment (d) an adapter
business is called an __________. (e) Voltmeter
y
o
u
rs
m
IBPS PO/MT Exam 2012 139

a
h
will help Indian exporters gain market share even it global trade

b
ENGLISH LANGUAGE

o
remains depressed. More importantly, it could lead to massive

o
b
import substitution that favours domestic producers.

.w
DIRECTIONS (Qs. 201-215) : Read the following passage Let's now focus on India and start with a caveat. It is

o
carefully and answer the questions given below it. Certain words/ important not to confuse a short-run cyclical dip with a permanent

rd
phrases have been printed in bold to help you locate them while de-rating of its long-term structural potential. The arithmetic is

p
answering some of the questions. simple. Our growth rate can be in the range of 7-10 per cent

re
depending on policy action. Ten per cent if we get everything

s
s
When times are hard, doomsayers are aplenty. The problem

.c
right, 7 per cent if we get it all wrong. Which policies and reforms
is that if you listen to them too carefully, you tend to overlook the

o
are critical to taking us to our 10 per cent potential ? In judging

m
most obvious signs of change. 2011 was a bad year. Can 2012 be this, let's again be careful. Lets not go by the laundry list of reforms
any worse? Doomsday forecasts are the easiest to make these that FIIs like to wave: increase in foreign equity limits in foreign
days. So let's try a contrarian's forecast instead. shareholding, greater voting rights tor institutional shareholders
Let's start with the global economy. We have seen a steady in banks, FDI in retail, etc. These can have an impact only at the
flow of good news from the US. The employment situation seems margin. We need not bend over backwards to appease the FIIs
to be improving rapidly and consumer sentiment, reflected in retail through these reforms - they will invest in our markets when
expenditures on discretionary items like electronics and clothes, momentum picks up and will be the first to exit when the momentum
has picked up. If these trends sustain, the US might post better flags, reforms or not.
growth numbers for 2012 than the 1.5-1.8 percent being forecast The reforms that we need are the ones that can actually raise
currently. out. Sustainable long-term growth rate. These have to come in areas
Japan is likely to pull out of a recession in 2012 as post- like better targeting of subsidies, making projects in infrastructure
earthquake reconstruction efforts gather momentum and the fiscal viable so that they draw capital, raising the productivity of agriculture,
stimulus announced in 2011 begins to pay off. The consensus improving healthcare and education, bringing the parallel economy
estimate for growth in Japan is a respectable 2 per cent for 2012. under the tax net, implementing fundamental reforms in taxation like
The "hard-landing" scenario for China remains and will GST and the direct tax code and finally easing the myriad rules and
remain a myth. Growth might decelerate further from the 9 per cent regulations that make doing business in India such a nightmare. A
that it expected to clock in 2011 but is unlikely to drop below 8-8.5 number of these things do not require new legislation and can be
percent in 2012. done through executive order.
Europe is certainly in a spot of trouble. It is perhaps already
201. Which of the following is NOT TRUE according to the
in recession and for 2012 it is likely to post mildly negative growth.
passage ?
The risk of implosion has dwindled over the last few months -
(a) China's economic growth may decline in the year 2012
peripheral economies like Greece, Italy and Spain have new
as compared to the year 2011
governments in place and have made progress towards genuine
(b) The European economy is not doing very well
economic reform.
(c) Greece is on the verge of bringing about economic
Even with some of these positive factors in place, we have
to accept the fact that global growth in 2012 will be tepid. But reforms
there is a flipside to this. Softer growth means lower demand for (d) In the year 2012, Japan may post a positive growth and
commodities and this is likely to drive a correction in commodity thus pull out of recession
prices. Lower commodity inflation will enable emerging market (e) All are true
central banks to reverse their monetary stance. China, for instance, 202. Which of the following will possibly be a result of softer
has already reversed its stance and has pared its reserve ratio growth estimated for the year 2012 ?
twice. The RBI also seems poised for a reversal in its rate cycle as (A) Prices of oil will not increase.
headline inflation seems well on its way to its target of 7 per cent (B) Credit availability would be lesser.
for March 2012. (C) Commodity inflation would be lesser.
That said, oil might be an exception to the general trend in (a) Only (B) (b) Only (A) and (B)
commodities. Rising geopolitical tensions, particularly the (c) Only (A) and (C) (d) Only (C)
continuing face-off between Iran and the US, might lead to a (e) All (A), (B) and (C)
spurt in prices. It might make sense for our oil companies to hedge 203 Which of the following can be said about the present status
this risk instead of buying oil in the spot market. ol the US economy ?
As inflation fears abate and emerging market central banks (a) There is not much improvement in the economic
begin to cut rates, two things could happen Lower commodity scenario of the country from the year 2011
inflation would mean lower interest rates and better credit (b) The growth in the economy of the country, in the year
availability. This could set a floor to growth and slowly reverse 2012, would definitely be lesser than 1.8 percent
the business cycle within these economies. Second, as the fear of (c) The expenditure on clothes and electronic commodities,
untamed, runaway inflation in these economies abates, the global by consumers, is lesser than that in the year 2011
investor's comfort levels with their markets will increase. (d) There is a chance that in 2012 the economy would do
Which of the emerging markets will outperform and who better than what has been forecast
will get left behind? In an environment in which global growth is (e) The pace of change in the employment scenario of the
likely to be weak, economies like India that have a powerful country is very slow.
domestic consumption dynamic should lead; those dependent 204. Which of the following is possibly the most appropriate
on exports should, prima facie, fall behind. Specifically for India, a title for the passage ?
fall in the exchange rate could not have come at a better time. It (a) The Economic Disorder
(b) Indian Economy Versus The European Economy
y
o
u
rs
m
140 IBPS PO/MT Exam 2012

a
h
(c) Global Trade

b
212. EMERGING

o
(d) The Current Economic Scenario (a) raising (b) developing

o
b
(e) Characteristics of The Indian Economy (c) noticeable (d) conspicuous

.w
205. According to the author, which of the following would (e) uproaring

o
characterize Indian growth scenario in 2012 ?

rd
(A) Domestic producers will take a hit because of depressed DIRECTIONS (Qs. 213-215) : Choose the word/group of words

p
which is most opposite in meaning to the word/group of words

re
global trade scenario.
printed in bold as used in the passage.

s
(B) On account of its high domestic consumption, India

s
.c
will lead. 213. MYRIAD

o
(C) Indian exporters will have a hard time in gaining market

m
(a) trivial (b) difficult
share. (c) few (d) effortless
(a) Only (B) (b) Only (A) and (B) (e) countless
(c) Only (B) and (C) (d) Only (A) 214. TEPID
(e) All (A), (B) and (C) (a) moderate (b) high
206. Why does the author not recommend taking up the reforms (c) warm (d) irregular
suggested by FII's ? (e) little
(a) These will bring about only minor growth 215. MYTH
(b) The reforms suggested will have no effect on the (a) reality (b) belief
economy ol our country, whereas will benefit the FII's (c) contrast (d) idealism
significantly (e) falsehood
(c) The previous such recommendations had backfired
(d) These reforms will be the sole reason for our country's DIRECTIONS (Qs. 216-220) : Rearrange the following six
economic downfall sentences (A), (B), (C), (D), (E) and (F) in the proper sequence
(e) The reforms suggested by them are not to be trusted to form a meaningful paragraph; then answer the questions given
as they will not bring about any positive growth in below them.
India (A) If China is the world's factory, India has become the world's
207. Which of the following is TRUE as per the scenario presented outsourcing centre - keeping in line with this image.
in the passage? (B) But India's future depends crucially on its ability to compete
(a) The highest growth rate that India can expect is 7 fully in the Creative Economy - not just in tech and software,
percent but across design and entrepreneurship; arts, culture and
(b) The fall in the exchange rate will prove beneficial to entertainment; and the knowledge-based professions of
India medicine, finance and law.
(c) Increased FDI in retail as suggested by Flls would
(C) While its creative assets outstrip those of other emerging
benefit India tremendously competitors, India must address several challenges to
(d) The reforms suggested by the author require new
increase its international competitiveness as the world is in
legislation in India
the midst of a sweeping transformation.
(e) None is true
(D) This transformation is evident in the fact that the world is
208. According to the author, which ot the following reform/s is/
moving from an industrial economy to a Creative Economy
are needed to ensure long term growth in India?
that generates wealth by harnessing intellectual labour,
(A) Improving healthcare and educational facilities.
intangible goods and human creative capabilities.
(B) Bringing about reforms in taxation.
(E) Its software industry is the world's second-largest, its tech
(C) Improving agricultural productivity.
outsourcing accounts for more than hall of the $ 300 billion
(a) Only (B) (b) Only (A) and (B)
global industry, according to a technology expert.
(c) Only (B) and (C) (d) Only (A)
(F) If the meeting of world leaders at Davos is any indication,
(e) All (A), (B) and (C)
India is rapidly becoming an economic'rock star'.
DIRECTIONS (Qs. 209-212) : Choose the word/group of words 216. Which of the following should be the SIXTH (LAST)
which is most similar in meaning to the word/group of words sentence after the rearrangement ?
printed in bold as used in the passage. (a) A (b) B
209. DRAW (c) C (d) D
(a) entice (b) push (e) E
(c) decoy (d) attract 217. Which of the following should be the THIRD sentence after
(e) persuade the rearrangement ?
210. CLOCK (a) A (b) B
(a) watch (b) achieve (c) C (d) D
(c) time (d) second (e) E
(e) regulate 218. Which of the following should be the FIFTH sentence after
211. ABATE the rearrangement ?
(a) rise (b) gear (a) A (b) B
(c) hurl (d) lessen (c) C (d) F
(e) retreat (e) E
y
o
u
rs
m
IBPS PO/MT Exam 2012 141

a
h
219. Which of the following should be the FIRST sentence after

b
DIRECTIONS (Qs. 226-235) : Read each sentence to find out

o
the rearrangement ?

o
whether there is any grammatical error or idiomatic error in it.

b
(a) F (b) B The error,if any, will be in one part of the sentence. The letter of

.w
(c) C (d) A that part is the answer. If there is 'No error', the answer is ‘(e)’.

o
(e) E

rd
(ignore errors of punctuation, if any).
220. Which of the following should be the SECOND sentence

p
226. The Government has asked individuals /

re
after the rearrangement ?

s
(a) A (b) B (a)

s
.c
(c) C (d) D

o
(e) F with income of over 110 lakhs to

m
/
DIRECTIONS (Qs. 221-225) : The following questions consist (b)
of a single sentence with one blank only. You are given six words electronic file tax returns for the year 2011 -12,
denoted by A, B, C, D, E & F as answer choices and from the six /
(c)
choices you have to pick two correct answers, either of which
will make the sentence meaningfully complete. something which was optional till last year. No error
221. ___________ before the clock struck 8 on Saturday night, (d) (e)
India Gate was swamped with people wearing black tee- 227. The power tariff had already / been increased twice in /
shirts and holding candles.
(A) Minutes (B) Time (a) (b)
(C) Later (D) Quickly the last 15 months and the Electricity Board had also
(E) Since (F) Seconds /
(c)
(a) (B) and (E) (b) (A) and (C)
(c) (A) and(F) (d) (B) and (D) levied additional monthly charges to consumers.
(e) (C) and (E) (d)
222. The Stales should take steps to the process of teachers"
appointments as the Centre has already sanctioned six lakh No error
posts. (e)
(A) fasten (B) move
(C) hasten (D) speed 228. Despite of curfew / in some areas, minor /
(E) early (F) quicken (a) (b)
(a) (D) and (F) (b) (A) and (C) communal incidents were reported
(c) (C) and (F) (d) (D) and (E) /
(e) (B) and (D) (c)
223. A senior citizen's son threatened her every day and from different areas of the walled city. No error
physically harmed her, forcing her to transfer her properly /
(d) (e)
to him.
(A) superficially (B) mistakenly 229. This comes / at a time / when fund allocation /
(C) allegedly (D) miserably (a) (b) (c)
(E) doubtfully (F) purportedly
(a) (C) and (F) (b) (A) and (E) is been doubled. No error
/
(c) (C) and (E) (d) (D) and (F) (d) (e)
(e) (A) and (C)
224. Medical teachers said that the management had continued 230. As the prison will get /
to remain to their cause leading to the stretching of their (a)
strike. an official telephone facility soon, the prisoners
(A) unmoved (B) lethargic /
(C) unconcerned (D) apathetic (b)
(E) indifferent (F) bored wont have to make calls in discreet manner
(a) (B) and (C) (b) (C) and (F) (c)
(c) (A) and (E) (d) (A) and (D)
(e) (D) and (E) through smuggled mobile phones. No error
225. The parents had approached the high court to the /
(d) (e)
government order after their children, who passed UKG, were
denied admission by a school. 231. The area was plunged into / darkness mid a wave of /
(A) void (B) quash (a) (b)
(C) annual (D) stay
(E) lift (F) post cheering and shouting slogans like 'Save The Earth'.
/ /
(a) (A) and (D) (b) (B) and (C) (c) (d)
(c) (C) and (E) (d) (E) and (F)
No error
(e) (C) and (D)
(e)
y
o
u
rs
m
142 IBPS PO/MT Exam 2012

a
h
(a) first to switch of (b) the first to switch off

b
232. The poll contestants approached /

o
(c) the first of switch off (d) first in switch of

o
(a)

b
(e) No correction required

.w
the commission complaining that the hoardings 239. The campaign is significant because not just the youths are
/

o
(b) directly appealing to the world but because their efforts

rd
violated the code of conduct challenge the chimera of normalcy in the area.

p
(a) not just because (b) just not because

re
/
(c) (c) not just (d) because just

s
s
and influenced public perception. No error (e) No correction required

.c
/ 240. The doctor's association has threatened to go on indefinite

o
(d) (e)

m
strike support of their teachers.
233. The country has / adequate laws but problems / (a) on supporting to (b) to supporting
(a) (b) (c) for support (d) in support of
arise when these are not (e) No correction required
/ DIRECTIONS (Qs. 241-250) : In the following passage there
(c)
are blanks, each of which has been numbered. These numbers
implemented in letter and spirit. No error are printed below the passage and against each, five words/
/
(d) (e) phrases are suggested, one of which fits the blank appropriately.
234. The Management feels that / Find out the appropriate word/phrase in each case.
(a) Greenhouse gases are only (241) of the story when it comes to
global warming. Changes to one part of the climate system can
the employees of the organisation are
/ (242) additional changes to the way the planet absorbs or reflects
(b) energy. These secondary changes are (243)climate feedback's,
non-productive, and do not want to work hard. and they could more than double the amount of warming caused
/ / by carbon dioxide alone. The primary feedback are (244)to snow
(c) (d) and ice, water vapour, clouds, and the carbon cycle.
No error The most well (245) feedback comes from melting snow and ice in
(e) the Northern Hemisphere. Warming temperatures are already (246)
a growing percentage of Arctic sea ice, exposing dark ocean water
235. As far the issue of land encroachment / during the (247) sunlight of summer. Snow cover on land is also
(a) (248) in many areas. In the (249) of snow and ice, these areas go
in villages is concerned, people will from having bright, sunlight-reflecting surfaces that cool the planet
/ to having dark, sunlight-absorbing surfaces that (250) more
(b) energy into the Earth system and cause more warming.
have to make a start from their villages by 241. (a) whole (b) part
/ (c) material (d) issue
(c)
(e) most
sensitising and educating the villagers about this issue. 242. (a) raise (b) brings
.
(d) (c) refer (d) stop
No error (e) cause
243. (a) sensed (b) called
(e) (c) nothing (d) but
DIRECTIONS (Qs. 236-240) : Which of the phrases (a), (b), (c) (e) term
and (d) given below each sentence should replace the word/ 244. (a) due (b) results
phrase printed in bold in the sentence to make it grammatically (c) reason (d) those
correct ? If the sentence is correct as it is given and no correction (e) because
is required, mark (e) as the answer. 245. (a) done (b) known
(c) ruled (d) bestowed
236. US secretary of state made it clear that time running out for (e) said
diplomacy over Iran's nuclear programme and said that talks 246. (a) mastering (b) sending
aimed at preventing Tehran from acquiring a nuclear weapon (c) melting (d) calming
would resume in April. (e) increasing
(a) runs out (b) was running out 247. (a) make-shift (b) ceasing
(c) ran out (d) run (c) troubled (d) perpetual
(e) No correction required (e) absent
248. (a) dwindling (b) manufactured
237. While the war of the generals rage on, somewhere in small
(c) descending (d) generating
town India, wonderful things are happening, quietly and (e) supplied
minus fanfare. 249. (a) progress (b) reduced
(a) rage (b) raging (c) existence (d) midst
(c) rages on (d) raged on (e) absence
(e) No correction required 250. (a) repel (b) waft
238. According to WWF, the small island nation of Samoa was (c) monitor (d) bring
the first in switch off its lights for Earth Hour. (e) access
y
o
u
rs
m
IBPS PO/MT Exam 2012 143

a
h
b
o
o
b
.w
o
rd
(1-4) :

p
Input : 84 why s it 14 32 not bes t ink feet 51 27 vain 68 92

re
s
Step I : 14 84 why sit 32 not ink feet 51 27 vain 68 92 best

s
.c
Step II : 27 14 84 why s it 32 not ink 51 vain 68 92 bes t feet

o
Step III : 32 27 14 84 why sit not 51 vain 68 92 best feet ink

m
Step IV : 51 32 27 14 84 why s it vain 68 92 bes t feet ink not
Step V : 68 51 32 27 14 84 why vain 92 best feet ink not s it
Step VI : 84 68 51 32 27 14 why 92 bes t feet ink not s it vain
Step VII : 92 84 68 51 32 27 14 bes t feet ink not s it vain why

OR
1. (e) There is no such step.
Test Exams
2. (d) The word ‘best’ is at the fifth position from the right in
Step V.
3. (b) There are three elements - 27, 14 and best - between
‘feet’ and ‘32’ in the last step of the output.
4. (c) The word ‘why’ is sixth from the left and ninth from the
right in the Step IV.
5-11. (H's mother)
B Question
(Male)
D C (Male) Conclusion I : False
Conclusion II : False
(H's Husband) 13-14.
Power
A E (D's
Daughter)
Energies Heat

G H
Force
(Female) (Female)
F
(Female)
5. (d) A's wife is H. H's mother is B. Therefore, B is the mother-
in-law of A, A is second to the right of B.
6. (c) E is the daughter of D.
7. (b) A is the father of F and G. G is the mother of C. Therefore, 13. (b) Conclusion I : False
C is the grandchild of A. A is third to the right of C. Conclusion II : True
8. (a) D is brother-in-law of A. A is father of F and G. Therefore, 14. (a) Conclusion I : True
D is material uncle of G. There is only one person Conclusion II : False
between G and D is clockwise direction. Coin
9. (b) Except C, all others are females. 15-16. Metals
10. (e) B is the mother of H. Note
C is the nephew of E,
A is the husband of H, A is third to the left of H, Both
the neighbours of C are females,
F and G are daughters of H. Plastics
11. (a) B sits to the immediate left of C. B is grandmother of F. OR

12. (d) Note Metals Coin


Test Exams

Question Plastics
y
o
u
rs
m
144 IBPS PO/MT Exam 2012

a
h
OR 19-21. B, D, F > C > A, E

b
o
B > D > F > C > A, E

o
Note Metals

b
Win B> D> F> C >E >A

.w
Score of F = 81

o
Score of E = 62

rd
19. (a) C scored more than 62 and less than 81.

p
62 < 70 < 81

re
20. (e) D's score was more than 81.

s
s
Plastics B scored the maximum marks.

.c
Three people, B , D and F scored more than C.

o
15. (e) Conclusion I : True

m
Conclusion II: True B scored more than 81 marks.
16. (d) Conclusion I : False 21. (c) Score of F = 81
Score of B = 81 + 13 = 94
Conclusion II : False
Therefore, score of D = more than 81 and less than 94
17. (e) 81 < 89 < 94
Picture

Oriental Bank
of Commerce
22-29.

Maharashtra
R L

Canara Bank

Dena Bank
Bank of
I E
G F
H T
T
Symbols
Figures
C A B D
Graphics
P Q S R
OR
Symbols

Punjab National
Figures Syndicate Bank R

Indian Bank
L

UCO Bank
I
E
F Bank G
H
T T
22. (b) The person from Punjab National Bank, Q faces A.
Picture B is from Bank of Maharashtra and he is a immediate
neighbour of A.
A faces of person who sits second to the left of R.
A is from Dena Bank.
Graphics A sits third from the left and second from the right.
Conclusion I : True 23. (e) S is seated between R and Q, the person from Punjab
Conclusion II : True National Bank.
18. (d) Jobs 24. (d) Persons at the extreme ends : C from Canara Bank; D
from Oriental Bank of Commerce; P from Syndicate
Bank: R from UCO Bank.
25. (a) S from Indian Bank faces B from Bank of Maharashtra.
26. (d) P faces the immediate neighbour of A from Dena Bank.
B faces the immediate neighbour of Q from Punjab
National Bank. Similarly, D faces immediate neighbour
of S from Indian Bank.
27. (d) Except Q, all others are seated at the extreme ends of
Occupations
the lines.
Vacancies 28. (c) P is from Syndicate Bank.
OR 29. (e) C is from Canara Bank.
Jobs 30. (e) From statements I, II and III
Occupations
Floor No. Person
6 P
5 T
4 R
Vacancies 3 S
Conclusion I : False 2 Q
Conclusion II : False
1 V
y
o
u
rs
m
IBPS PO/MT Exam 2012 145

a
h
31. (b) From statements I and III 36. (c) There is a perception among small brands that sale in a

b
o
1 2 3 4 5 6 supermarket is higher than that of small grocery stores.

o
b
A S E 37. (e) The statement clearly implies that smaller brands are

.w
A N S W E R currently making substantial losses in their businesses.

o
32. (b) From statements I and III Consider the term ‘‘troubled waters’’.

rd
38. (b) Statement (C) represents a dlisadvantage of the small

p
North

re
grocery stores over the Supermarkets from the

s
perspective of a smaller brand. Small grocery stores do

s
C

.c
not help in distribution of any brand.
West East

o
39. (c) Statement (D) represents a reason for the shift from

m
local grocery stores of supermarkets by the smaller
A B E brands.
South 40. (a) Statement (A) will prove that step taken by the smaller
brands may not necessarily be correct.
41. (d) In the subsequent figures two and three designs are
D F inverted alternately. From problems Figure (1) to (2)
the first and the second designs from the top
33. (e) From Statements I and II interchange positions and the lowermost design moves
one of its kind zo pi ko fe to the middle position. From problem Figure (2) and (3)
the lowermost design moves to the top and the second
in kind and cash ga to ru ko design from the top moves to the lowermost position
its point for origin ba je fe mi while the third and the fourth designs interchange
positions. From problem Figure (3) to (4) the top most
make a point clear yu si mi de and the lowermost designs interchange positions and
From statements I and III. the fourth design moves to the second position. From
Problem Figure (4) to (5) the topmost design moves to
one of its kind zo pi ko fe
the fourth position, the lowermost design moves to
in kind and cash ga to ru ko the topmost position while the second and the third
make money and cash to mi ru hy designs interchange positions. Therefore, from problem
Figure (5) to Answer Figure the third and the fourth
money of various kind qu ko zo hy designs would interchange positions and would be
From statements I, II and III inverted, the lowermost design would move to the top
one of its kind zo pi ko fe position and the second design from the top would
move to the lowermost position.
in kind and cash ga to ru ko 42. (a) The following changes occur in the subsequent figures:
(1) to (2) (2) to (3)
its point for origin ba le fe mi
make a point clear yu si mi de
make money and cash to mi ru hy
money of various kind qu ko zo hy

34. (d) Statement I Statement II


B A
(3) to (4) (4) to (5)

A C B D

D C
Statement III
B (5) to (6)

A C

D
35. (b) It is clear from the statement that a farm’s areas for 43. (d) From problem Figure (1) to (2) and from problem Figure
organic and chemical farming are different. (5) to Answer Figure the smaller square with four
y
o
u
rs
m
146 IBPS PO/MT Exam 2012

a
h
designs moves two steps in anticlockwise direction

b
lower positions. In Figure (1) there should be another

o
and the designs in each row interchange positions. design at the lower side.

o
b
The other three designs move two steps in clockwise 51. (d) 4003 × 77 – 21015 = ? × 116

.w
direction. Þ 308 231 – 21015 = ? × 116 Þ 287216 = ? × 116

o
44. (c) The following changes occur in the subsequent figures:

rd
(1) to (2) (2) to (3) 287216

p
Þ? = = 2476
(5) to (6)

re
116

s
s
( ) ( )
é 5 7 + 7 ´ 4 7 + 8 7 ù - (19)2 = ?

.c
N 52. (a) ë û

o
m
Þ (6 7 ´12 7) - (361) = ?
N Þ 72 × 7 × 7 – 361 = ?
(3) to (4) (4) to (5) \ ? = 504 – 361 = 143
53. (b) (4444 ¸ 40) + (645 ¸ 25) + (3991 ¸ 26) = ?
N Þ ? = (111.1) + (25.8) + (153.5) Þ ? = 290.4
54. (e) 33124 ´ 2601 – (83) 2 = (?) 2 (37) 2

N Þ (?)2 = 33124 ´ 2601 – (83) 2 - (37) 2


Þ (?)2 = 182 × 51 – 6889 – 1369
45. (b) From problem Figure (1) to (2) the plane of designs
Þ (?)2 = 9282 – 6889 – 1369
rotates through 90° clockwise and the second and the
fifth designs interchange positions. Similarly, the third Þ (?)2 = 1024
and the fourth designs interchange positions. Similar \ ? = 1024 = 32
changes occur from Problem Figure (3) to (4) and from
problem figure (5) to Answer Figure. 17 51 1 3
55. (b) 5 ´ 4 ´11 + 2 = ?
46. (d) The following changes occur in the subsequent figures: 37 52 7 4
(1) to (2) (2) to (3)
æ 202 259 78 ö æ 11 ö
(3) to (4) (4) to (5) Þç ´ ´ ÷ + ç ÷ =?
è 37 52 7 ø è 4 ø
11
Þ 303 + =?
4
1223
\?= = 305.75
4
56. (c) 8787 ¸ 343 × 50 = ?
In Figure (4) the arrangement of designs would be as follows:) Þ 25 × 7 = ?
\ ? = 175 » 180
N 3
57. (b) 54821 ´ (303 ¸ 8) = (?) 2
S
A Þ 38 × 37.5 (?)2
D
T ? = 38 ´ 38

? = 38
47. (e) The line segment with cross rotates respectively
through 45°, 90°, 135°, 180° ... in clockwise direction. 5 7
58. (c) of 4011.33 + of 3411.22 = ?
48. (d) In each subsequent figure the designs move in 8 10
clockwise direction and the number of designs at a 5 7
particular position is fixed. Again, a new design appears Þ × 4010 + × 3410 Þ 2506 + 2387
8 10
at the lower right side. In figure (4), there should be
Þ 4893 » 4890
design ( ) in the place of circle at the lower right side.
59. (e) 23% of 6783 + 57% of 8431 = ?
49. (e) In the subsequent figures respectively one, tow, three,
four ..., curves are inverted in a set order. In Figure (5) Þ ? = 1559 + 4805
five curves have been inverted. \ ? = 6364 » 6360
50. (a) In the first step the outer part of upper design moves 60. (a) 335.01 × 244.99 ¸ 55
inside the lower design and new designs appear at the 335 ´ 245
upper and lower positions. In the second step, the Þ?=
55
outer part of lower design moves inside the upper
\ ? = 1492 » 1490
design and new designs appear at both the upper and
y
o
u
rs
m
IBPS PO/MT Exam 2012 147

a
h
61. (a) The given number series is based on the following

b
æ 5ö

o
pattern: Þ ç 45 ´ ÷ m / sec =12.5 m / sec

o
è 18 ø

b
5555

.w
5531 5506 5425 5304 5135 4910 4621 69. (e) Percentage

o
rd
Distance travelled by vehicle F on day 2
= ´ 100

p
2 2 2 2 2 2
– (7) – (9) – (11) – (13) – (15) – (17) Distance travelled by vehicle F on day 1

re
Hence, the number 5531 is wrong and it should be

s
s
replaced by 5555. 636 630

.c
= ´ 100 » ´ 100 » 90%

o
62. (b) The given number series is based on the following 703 700

m
pattern : 70. (b) Speed of vehicle D on day 2 = 51
21 Speed of vehicle E on day 2 = 39
6 7 9 13 26 37 69
51 17
Required ratio = = or17:13
× 2– 5 ×2–5 ×2–5 ×2–5 ×2–5 ×2–5 39 13
Hence, the number 26 is wrong and it should replaced 71. (a) Article purchased = ` 78350
by 21.
æ 130 ö
63. (d) The given number series is based on the following Marked price = ` çè 78350 ´ ÷ = ` 101855
pattern: 100 ø
770 After Discount Price of Article
36 152 760 4532
1 3 10 æ 80 ö÷
= ` çç101855≥ ÷ < ` 81484
çè 100 ø÷
´1+2 ´2+4 ´3+6 ´4+8 ´ 5 + 10 ´ 6 + 12
Hence, the number 760 is wrong and it should be Profit
replaced by 770. Profit Percentage = ≥100
Cost Price
64. (e) The given series is
× 0.5 + 0.5, × 1 + 1, × 1.5 + 1.5, 81484 – 78350
Þ ×100 = 3.8 » 4%
× 2 + 2, × 2.5 + 2.5, × 3 + 3 81484
65. (a) The given number series is based on the following 72. (e) 9, 15, 27
pattern : 9 – x , 15 – x, 27 – x
2
1 15 - x 27 - x
157.5 45 15 6 3 2 =
9 - x 15 - x
¸ 3.5 ¸3 ¸ 2.5 ¸2 ¸1.5 ¸1 Þ (15 – x)2 = (27 – x) (9 – x)
Hence the number 1 is wrong and it should be replaced Þ 225 + x2 – 30x = 243 – 9x – 27x +x2
by 2. Þ –30x + 9x + 27x = 243 –225
66-70. Þ 6x = 18 Þ x = 3
Day 1 Day 2 2
æ R ö
73. (b) Required difference = P ç
Vehicle Time Distance Speed in Time Distance Speed in è 100 ÷ø
in hr in km km / hr in hr in km km/hr
A 16 832 52 16 864 54 2
æ 6 ö
= 7300 ´ ç = ` 26.28
è 100 ÷ø
B 12 516 43 18 774 43
74. (e) Let the numbers are x, x + 1, x + 2
C 11 693 63 18 810 45 sum of three consecutive numbers = 2262
x + x + 1 + x + 2 = 2262
D 12 552 46 15 765 51 3x + 3 = 2262
3x = 2259
E 16 935 58.4 14 546 39 x = 753
Number are 753, 754, 755
F 19 703 37 12 636 53 \ 41% of 755 = 309.55
75. (d) No. of vowels in the word THERAPY
66. (d) Vehicle B.
= 2 i.e. E and A
67. (c) Speed of vehicle A on day 1 = 52 km/hr
Speed of vehicle C on day 1 = 63 km/hr In such cases we treat the group of two vowels as one
Difference = 63 – 52 = 11 km / hr entity or one letter because they are supposed to
68. (e) Speed of vehicle can day 2 = 45 km/hr always come together. Thus, the problem reduces to
y
o
u
rs
m
148 IBPS PO/MT Exam 2012

a
h
b
arranging 6 letters i.e. T, H, R, P, Y and EA in 6 vacant Total cost of marble flooring

o
= ` [(190 × (182 + 273 + 609)] = ` (190 × 1064)

o
places.

b
Required Ratio = 888 × 170 : 1064 × 190 = 1887 : 2527

.w
No. of ways 6 letters can be arranged in 6 places = 6! 82. (c) Area of 4 walls and ceiling of branch managers room

o
= 6 × 5 × 4 × 3 × 2 × 1 = 720

rd
= 2 (lh + bh) + lb = 2 [17 × 12 + 13 × 12] + 13 × 17
But the vowels can be arranged themselves in 2

p
= 941 sq. m

re
different ways by interchanging their position. Hence, Total cost of renovatin = `190 × 941 = `178790

s
each of the above 720 arrangements can be written in 2

s
83. (e) Total area of bank is 2000 sq. m

.c
ways. Total area of bank to be renovated = 1952 sq. m

o
\ Required no. of total arrangements when two vowels

m
Remaining Area = 2000 – 1952 = 48 sq. m
are together = 720 × 2 = 1440 Total cost Remaining Area to be carpeted at the rate of
Total no. of arrangements of THERAPY = 7! `110/sq. meter = `(48 × 110) = `5280
= 7 × 6 × 5 × 4 × 3 × 2 × 1 = 5040 84. (b) percentage area of bank not to be renovated
No. of arrangement when vowels do not come together Area bank not be renovated 48
= 5040 – 1440 = 3600 Þ Þ ´ 100 = 2.4%
Total area of bank 2000
76-80. 85. (a) Total cost of hall of customer transaction
Total number Total Number of Total Number of = `(170 × 667) = `113,390
of Mobiles Mobiles Sold of Mobiles Sold of Total cost of Locker area = `(190 × 609) = `115710
Sold Company A Company B Total cost of customer transaction hall + locker area
July 7650 4080 3570 = `(113390 + 115710) = `229100
August 9900 4400 5500 86. (a) Let amount of B = ` x
September 11250 6750 4500
October 3600 2100 1500 B’s ratio
B’s Share without error = ´ Total Amount
November 5400 2520 2880 Total ratio
December 7200 3150 4050
3
76. (c) Number of mobiles sold of company B in July = 3570 x= ´ Total Amount ...(1)
9
Number of mobiles sold of company B in December
B’s new ratio
= 4050 B’s share after error = ´ Total Amount
Total new ratio
Required Ratio = 3570 : 4050 = 119 : 135
77. (c) Total mobiles sold by companyA during November = 2520 2
x – 40 = ´ Total Amount ...(2)
Total mobiles sold by this company at discount 14
= 35% of 2520 = 882 From equation (1) and (2)
Total mobiles sold by company A without discount 3x = 7(x–40)
= 2520 – 882 = 1638 3x – 7x = –280
\ x = 70
78. (d) Mobile phones sold of company B during October = 1500
Total Amount = 7 (70 – 40) = ` 210
Total profit earned on the mobile phones
87. (c) By options
= `(433 × 1500) = ` 6,49,500
(a) Either 12 or 13
79. (e) Number of mobile phones sold of company then ice-creams should not be given atleast 9. This
A during July = 4080 can be rejected.
Number of mobile phones sold by company A during (b) Either 11 or 12
December = 3150 Ice-cream should be atleast 9. By this combination ice
cream gets less than 9.
Required percentage = 4080 ´ 100 = 129.5 » 130% (c) Either 10 or 11
3150
By giving cookies 10 or 11, we get all the possible
80. (a) Mobile phones sold of company B during August = 5500 condition fulfilled.
Mobile phones sold of company B during September (d) and (e), the ice-cream distribution can be more than
= 4500 cookies which violates our condition.
Total number of mobile phones = 5500 + 4500 = 10,000 \ option (c) is the write answer.
81. (c) Area of customer transaction room = 23m×29m = 667 sq.m 88. (c) `[(x for first 5 km) + 13 × remaining kms] = Total pay
Area of branch manager room = 13m × 17 m = 221 sq. m `x + ` 13 × 182 = ` 2402
Area of Pantry room = 14m × 13m = 182 sq. m x + 2366 = 2402
Area of Server room = 21m × 13m = 273 sq. m x = `36
Area of locker room = 29 m × 21 m = 609 sq. m 89. (a) Let the even consecutive numbers are 2n–2, 2n, 2n + 2
Total cost of wooden flooring = ` [(170 × (667 + 221)] (2n – 2) × (2n) × (2n + 2) = 4032 ...(1)
= ` (888 × 170) Product of 1st even number third even number = 252
y
o
u
rs
m
IBPS PO/MT Exam 2012 149

a
h
b
Putting this in equation ...(1) 94. (e) A and B ratio is 4 : 7

o
252 × 2n = 4032 Þ n = 8 Þ 4x + 7x = 73689

o
b
Þ 11x = 73689

.w
Numbers are 14, 16, 18
Þ x = 6699

o
Five times of 2nd number is = 5 × 16 = 80
Share of A = `26796

rd
90. (a) Let the 4 members are x1, x2, x3, daughter

p
Share of B = `46893

re
Sum of 4 members five years ago Difference = twice of share B – thrice of share A

s
s
= x1+ x2 + x3 + daughter = 94 = 2 × 46893 – 3 × 26796 = ` 13398

.c
After 5 years,

o
1

m
x1+ x2 + x3 + daughter = 114 ...(1) 95. (d) A +B 1 day’s work = ...(1)
20
daughter + daughter in law = 92
1
Daughter = 92 – daughter in law B + C 1 day's work = ...(2)
30
Put this eqn. ...(1)
1
x1 + x2 + x3+ 92 – Daughter in law = 114 C+A 1 day's work = ...(3)
40
x1 + x2 + x3 = 22 + Daughter in law
Adding eqn. (1), (2) and (3)
So, the required difference is 22 years.
1 1 1
91. (d) No. of ways of getting 2 white balls = 13C2 2 (A + B + C) = + +
20 30 40
No. of ways of getting 2 black balls = 7C2
Probability of getting 2 same colour ball 6+ 4+3
2 (A + B + C) =
120
Probability of 2 white balls or Probability of 2 Black balls
=
Total number of balls drawn 13
Þ A + B + C 1 day work together =
13! 7! 240
13 + A’ Alone 1 day’s work = (A + B + C) 12 day’s work –
C2 + 7 C2 2!´ 11! 2!´ 5!
Þ Þ (B + C) 1 day’s work
20
C2 20!
13 1 13 - 8 5
18!´ 2! A= - Þ =
240 30 240 240
13 ´ 12 ´11! 7 ´ 6 ´ 5!
+
2!´ 5! = 13 ´ 12 + 7 ´ 6 Number of days taken by A = 240 days
Þ 2!´ 11! 5
20 ´19 ´18! 20 ´19 C ’ Alone 1 day’s work = (A + B + C) 12 day’s work –
18!´ 2! (A + B) 1’ day’s work
198 99 13 1 13 - 12 1
Þ = Þ - Þ =
380 190 240 20 240 240
92. (b) Marks is subject B = 56% of 150 = 84 240
Total marks obtained = 54 % of Total marks Number of days taken by C = days
1
54 240 240
= ´ 450 [\ Maximum marks in each subject is 150] Required Ratio :
100 5 1
= 243 Þ1:5
Total marks obtained = A + B + C 2010 - 2009
96. (d) Percentage increase = ´ 100
243 = 73 + 84 + X 2009
X = 86 700 - 550
93. (d) Area of square = 1444 sq. meters = ´ 100 = 27.2 » 27%
550
Side of square = 1444 = 38m 400
97. (b) Percent of production = ´ 100 = 72.72 » 73%
1 550
Breadth of Rectangle = ´ side of square 98. (c) Year Production of B
4
2006 600
1 2007 700
Þ ´ 38 = 9.5 m
4 2008 800
Length of Rectangle Þ 3 × breadth 2009 600
Þ 3 × 9.5 Þ 28.5 m
2010 650
Area of Rectangle = 270.75 sq. m
2011 700
Difference in area = 1444 – 270.75
Þ 1173.25 sq. mtr 600 + 700 + 800 + 600 + 650 + 700
= = 675
6
y
o
u
rs
m
150 IBPS PO/MT Exam 2012

a
h
b
99. (e) Total production of company A = 4050 211. (d) The meaning of Abate (verb) as used in the passage is :

o
Total sales of company A = 2750

o
to become less.

b
Required ratio Þ 4050 : 2750 = 81 : 55

.w
Hence, the words Abate and Lessen are synonymous.
100. (c) Required ratio = production of B in the year 2006 :

o
212. (b) The meaning of emerging (verb) as used in the passage

rd
Production of B in the year 2008
Þ 600 : 800 Þ 3 : 4 is : come into sight.

p
re
101. (a) 102. (b) 103. (b) 104. (a) 105. (e) Hence, the words Emerging and Developing are

s
s
106. (d) 107. (b) 108. (c) 109. (e) 110. (c)

.c
synonymous.
111. (b) 112. (a) 113. (a) 114. (d) 115. (c)

o
213. (c) The meaning of Myraid (adjective) as used in the

m
116. (d) 117. (b) 118. (e) 119. (a) 120. (c)
121. (e) 122. (d) 123. (d) 124. (b) 125. (d) passage is : numerous.
126. (b) 127. (a) 128. (e) 129. (e) 130. (e) Hence, the words myraid and few are antonymous.
131. (d) 132. (a) 133. (d) 134. (c) 135. (b) 214. (a) The meaning of Tepid (Adjective) as used in the
136. (c) 137. (a) 138. (d) 139. (a) 140. (c)
passage is warm.
141. (b) 142. (c) 143. (a) 144. (e) 145. (d)
146. (e) 147. (b) 148. (d) 149. (c) 150. (d) Hence, the words tepid and moderate are antonymous.
151. (b) 152. (a) 153. (c) 154. (a) 155. (b) 215. (a) The meaning of Myth as used in the passage is :
156. (a) 157. (a) 158. (c) 159. (e) 160. (a) fictional.
161. (e) 162. (c) 163. (c) 164. (c) 165. (a)
216-220. FAE BC D
166. (d) 167. (e) 168. (e) 169. (e) 170. (a)
171. (d) 172. (a) 173. (b) 174. (d) 175. (a) 216. (d) D 217. (e) E
176. (d) 177. (b) 178. (d) 179. (a) 180. (b) 218. (c) C 219. (a) F
181. (d) 182. (e) 183. (c) 184. (c) 185. (b) 220. (a) A
186. (a) 187. (e) 188. (b) 189. (c) 190. (d)
221. (a) 222. (b) 223. (c)
191. (e) 192. (b) 193. (b) 194. (e) 195. (a)
196. (c) 197. (d) 198. (a) 199. (b) 200. (d) 224. (b) 225. (c) 226. (e)
201. (e) All are true. 227. (d) Here, levied additional monthly charges on consumers
202. (c) Only (A) and (C). is used.
203. (d) There is a chance that in 2012 the economy would be 228. (a) Here, Despite curfew is used.
better than what has been forecast.
229. (d) Here, Has been doubled is used.
204. (d) The current Economic Scenario.
205. (a) Only (B). 230. (a) 231. (b) 232. (c)
206. (a) These will bring about only minor growth. 233. (c) Here, arise when they are not is used.
207. (e) None is true. 234. (e) No error 235. (b)
208. (e) All (A), (B) and (C). 236. (a) runs out.
209. (a) The meaning of word Draw (verb) as used in the 237. (c) rages on.
passage is : Influence.
238. (b) the first to switch off.
Hence, the words Draw and Entice are synonymous.
210. (a) The meaning of clock (noun) as used in the passage is 239. (a) not just because.
: time keeping device. 240. (d) in support of.
Hence, the words Clock and Watch are synonymous. 241. (b) 242. (e) 243. (b) 244. (a) 245. (b)
246. (c) 247. (d) 248. (a) 249. (e) 250. (d)
y
o
u
rs
m
a
h
b
o
o
IBPS SPECIALIST OFFICER EXAM 2012

b
.w
o
rd
p
Based on Memory

re
s
s
.c
8. What should come in place of the question mark (?) in the
REASONING ABILITY

o
following series based on the above arrangement ?

m
1. In a certain code 'TEAM WORK' is written as 'NBFUJQNV' RTI $@F 416 ?
and 'SOME' is written as 'PTDL'. How is 'PERSON' written in (a) 8 « H (b) 8 H N
that code ? (c) 8 H 7 (d) D « H
(a) QDOOPT (b) QDOMNR (e) None of these
(c) SFQMNR (d) SFQOPT 9. How many such symbols are there in the above arrangement,
(e) None of these each of which is immediately preceded by a number and
2. How many such pairs of letters are there in the word immediately folled by a letter ?
'SUBSTANCE' each of which has as many letters between (a) None (b) One
them in the word (in both forward and backward directions) (c) Two (d) Three
as in the English alphabet ? (e) More than three
(a) None (b) One 10. Which of the following is the seventh to the left of the
(c) Two (d) Three twentieth from the left end of the above arrangement ?
(e) More than three
(a) V (b) @
3. How many meaningful English words, not ending with 'D'
(c) d (d) ©
can be made with the third, the fifth, the seventh and the
(e) None of these
ninth letters of the word 'STEADFAST' using each letter
only once in each word ? (All letters are counted from left to DIRECTIONS (11 - 15) : Each of the questions below consists of
right) a question and two statements numbered I and II given below it.
(a) None (b) One You have to decide whether the data provided in the statements
(c) Two (d) Three are sufficient to answer the question. Read both the statements
(e) More than three and–
4. In a certain code 'BASKET' is written as '5$3%#1' and 'TRIED'
is written as '14«#2'. How is 'SKIRT' written in that code ? Give answer (a) if the data in statement I alone are sufficient to
(a) 3%«41 (b) 3«%41 answer the question, while the data in statement II alone are not
(c) 3%#41 (d) 3#4%1 sufficient to answer the question.
(e) None of these Give answer (b) if the data in statement II alone are sufficient to
5. The positions of the first and the fifth digits of the number answer the question, while the data in statement I alone are not
81943275 are interchanged. Similarly the positions of the sufficient to answer the question.
second and the sixth digits are interchanged and so on till Give answer (c) if the data either in statement I alone or in
the fourth and the eighth digits. Which of the following will statement II alone are sufficient to answer the question.
be the third digit from the right end after the rearrangement ? Give answer (d) if the data given in both the statements I and II
(a) 1 (b) 9 together are not sufficient to answer the question, and
(c) 2 (d) 4 Give answer (e) if the data in both the statements I and II together
(e) None of these are are necessary to answer the question.
DIRECTIONS (6–10) : Study the following arrangement 11. Among M, P, T, R and W each being of a different age, who
carefully and answer the questions given bellow : is the youngest ?
I. T is younger than only P and W.
M 3 # R AT I E J $ K @ F U 5 4 V I 6 © D 8 « H N 7 d 2 W II. M is younger than T and older than R.
6. Four of the following five are alike in a certain way based on
12. How is 'gone' written in a code language?
their positions in the above arrangement and so form a group.
I. 'You will be gone' is written as 'ka pa ni sa' in that code
Which is the one that does not belong to that group ?
(a) TER (b) U4@ language.
(c) 6DV (d) J I K II. 'he will be there' is written as 'ja da ka ni' in that code
(e) Nd« language.
7. How many such numbers are there in the above arrangement, 13. On which day of the week (starting from Monday and ending
each of which is immediately preceded by a letter but not on Sunday of the same week) did Sushant visit Chennai ?
immediately folled by a letter ? I. Sushant visited Chennai two days after his brother
(a) None (b) One visited Chennai.
(c) Two (d) Three II. Sushant did not visit Chennai either on Wednesday or
(e) More than three on Friday.
y
o
u
rs
m
152 IBPS Specialist Officer Exam 2012

a
h
b
14. Towards which direction is P with respect to the starting (e) None of these

o
point ? 19. Statement :

o
b
I. P walked 20 metres, took a right turn and walked F # H, H @ M, M © E, E $ J

.w
30 metres, again took right turn and walked 20 metres Conclusions :

o
towards West. I. J © M

rd
II. P walked 30 metres, took a left turn and walked 20 II. E # H

p
re
metres; again took left turn and walked 30 metres III. M © F

s
towards East. IV. F # E

s
.c
15. How is K related to Z ? (a) Only I and II are true

o
m
I. Z and P are the only sisters of D. (b) Only II and III are true
II. D's mother is wife of K's father. (c) Only I, II and III are true
(d) Only II, III and IV are true
DIRECTIONS (16-20) : In the following questions, the symbols
(e) None of these
@, $, #, © and % are used with the following meaning as
20. Statement :
illustrated below :
D % A, A @ B, B © K, K % M
'P $ Q' means 'P is not neither greater than nor equal to Q'. Conclusions :
'P © Q' means 'P is neither greater than nor equal to Q.' I. B $ D
'P # Q' means 'P is neither smaller than nor equal to Q.' II. K # A
'P % Q' means 'P is not greater than Q.' III. M # B
'P @ Q' means 'P is neither greater than nor smaller than Q.' IV. A © M
Now in each of the following questions assuming the given (a) Only I, II and IV are true
statements to be true, find which of the four conclusions I, II, III (b) Only I, II and III are true
and IV given below them is/are definitely true and give your (c) Only II, III and IV are true
answer accordingly. (d) Only I, III and IV are true
16. Statement : (e) All I, II, III and IV are true
R # J, J $ D, D @ K, K % T
DIRECTIONS (21-25) : Study the following information carefully
Conclusions :
and answer the given questions:
I. T # D
II. R @ D A word and number arrangement machine when given an input
III. R # K line of words and numbers rearranges them following a particular
IV. J $ T rule in each step. The following is an illustration of input and
(a) Only either I or II is true rearrangement (All the numbers given in the arrangement are two
(b) Only III is true digit numbers).
(c) Only III and IV are true Input : gone over 35 69 test 72 park 27
(d) Only either I or II and III are true Step I : 27 gone over 35 69 test 72 park
(e) None of these Step II : 27 test gone over 35 69 72 park
17. Statement : Step III : 27 test 35 gone over 69 72 park
T % R, R $ M, M @ D, D © H Step IV : 27 test 35 park gone over 69 72
Conclusions : Step V : 27 test 35 park 69 gone over 72
I. D % R Step VI : 27 test 35 park 69 over gone 72
II. H # R Step VI : 27 test 35 park 69 over 72 gone
III. T Ó M and Step VII is the last step of the rearrangement of the above
IV. T % D input as the desired arrangement is obtained.
(a) Only I is true As per the rules followed in the above steps, find out in each of
(b) Only I and IV are true the following questions the appropriate step for the given input.
(c) Only II and III are true 21. Input : 86 open shut door 31 49 always 45
(d) Only II and IV are true How many steps will be required to complete the rearrangement?
(e) None of these (a) Five (b) Six
18. Statement : (c) Seven (d) Four
M @ B, B # N, N $ R, R Ó K (e) None of these
Conclusions : 22. Step III of an input 25 yes 37 enemy joy defer 52 46
I. K # B Which of the following is definitely the input ?
II. R © B (a) enemy 25 joy defeat yes 52 37 46
III. M $ R (b) 37 enemy 25 joy yes defeat 52 46
IV. N © M (c) enemy joy defeat 25 52 yes 46 37
(a) Only I and III are true (d) Cannot be determined
(b) Only I and IV are true (e) None of these
(c) Only I and IV are true 23. Step II of an Input : 18 win 71 34 now if victory 61
(d) Only II, III and IV are true
y
o
u
rs
m
IBPS Specialist Officer Exam 2012 153

a
h
b
How many more steps will be required to complete the of an organisation after completeing her post Graduation.

o
rearrangement ? She has secured 55 per cent marks in the selection process.

o
b
(a) Three (b) Four 27. Ashok Pradhan was born on 8th August 1980. He has been

.w
(c) Five (d) Six working in the personnel Department of an organisation for

o
(e) More than six the past four years after completing his post Graduate

rd
24. Input : where 47 59 12 are they going 39 Degree in personnel Management with 67 percent. Out of

p
re
Which of the following steps will be the last but one ? his entire experience, he has been working for the past two

s
(a) VII (b) IV years as Deputy Manager- HR. He has secured 62 percent

s
.c
(c) V (d) VIII marks in Graduation and 58 percent marks in the selection

o
process.

m
(e) None of these
25. Step II of an input : 33 store 81 75 full of goods 52 28. Alok Verma was born on 4th March 1976. He has been
Which of the following will be step VI ? working in the personnel Department of an organisation for
(a) 33 store 52 of 75 81 full goods the past six years after completing his post Graduate Diploma
(b) 33 store 52 of 75 full 81 goods in personnel Management with 66 percent marks. He has
(c) 33 store 52 of 75 goods 81 full secured 57 percent marks in the selection process and 63
(d) There will be no such step percent marks in Graduation.
29. Swapan Ghosh has been working in the personnel
(e) None of these
Department of an organisation for the past five years after
DIRECTIONS (26-30) : Study the following information carefully completing his post Graduate Degree in HR with 72 percent
and answer the given questions given below : marks. He has secured 56 percent marks in Graduation.He
was born on 12th May 1977. He has secured 58 percent
Following are the conditions for selecting Manager-HR in an marks in the selection process.
organisation : 30. Seema Beht has been working in the personnel Department
The candidate must – of an organisation for the past seven years after completing
(i) be at least 30 years and not more than 35 years as on her post Graduate Diploma in Personnel Management with
01.03.2012. 70 percent marks. She was born on 5th July 1979. She has
(ii) have secured at least 60 percent marks in Graduation in any secured 65 percent marks in Graduation and 50 percent marks
discipline. in the selection process.
(iii) have secured at least 65 percent marks in the Post Graduate
degree/ Diploma in personnel Management/HR. DIRECTIONS (31-35) : Study the following information carefully
(iv) have post qualification work experience of at least five years and answer the given questions given below :
in the personal/HR Department of an organisation. A, B, C, D, E, F, G and H are eight friends travelling in three
(v) have secured at least 50 percent marks in the selection different cars, viz. X, Y and Z with at least two in one car to three
process. different places, viz. Delhi, Chandigarh and Agra.
In the case of a candidate who satisfies all the above conditions There is at least one female member in each car. D is travlling with
except- G to Delhi but not in car Y. A is travelling with only H in car Z but
(a) at (ii) above, but has secured at least 55 percent marks in not to Chandigarh. C is not travelling with either D or E. F and D
Graduation in any discipline and at least 70 percent marks in are studying in the same only girls' college. H, B and G are studying
post Graduate Degree/Diploma in personnel Management/ in the same only boys' college.
HR, the case is to be referred to GM - HR. 31. Which of the following represents the group of females
(b) at (iv) above, but has post qualification work experience of among them ?
at least two years as Deputy Manager- HR. the case is to be (a) F, C, A (b) F, G, A
referred to President - HR. (c) D, C, A (d) Data inadequate
In each question below are given details of one candidate. You (e) None of these
have to take one of the following courses of actions based on the 32. Which of the following combinations is correct ?
information provided and the conditions and sub-conditions given (a) Delhi – X – C (b) Chandigarh – X – F
above and mark the number of that course of action as your (c) Agra – Z – E (d) Delhi – Y – E
answer. You are not to assume anything other than the information (e) None of these
provided in each question. All these cases are given to you as on 33. In which car are four of them travelling ?
01.03.2012. (a) X or Z (b) Y
Mark answer (a) if the candidate is not to be selected (c) X or Y (d) Z
Mark answer (b) if the data provided are not adequate to take a (e) None of these
decision. 34. In which of the following cars is C travelling ?
Mark answer (c) if the case is to be referred to president-HR (a) X (b) Y
Mark answer (d) if the case is to be referred to GM-HR (c) Z (d) Either X or Y
Mark answer (e) if the candidate is to be selected (e) Data inadequate
26. Rita Bhatt was born on 25th July 1978. She has secured 35. Passengers in which car are travelling to Chandigarh ?
62 per cent marks in Graduation and 65 per cent marks in (a) Y (b) X
post Graduate Diploma in Management. She has been (c) Either X or Y (d) Data inadequate
working for the past six years in the personnel Department (e) None of these
y
o
u
rs
m
154 IBPS Specialist Officer Exam 2012

a
h
b
DIRECTIONS (36-40) : In each of the questions below is given 39. Statement : A severe cyclonic storm hit the Eastern coastline

o
last month resulting in huge loss of life and property on the

o
a statement, followed by a question. Read the statement carefully

b
and answer the question that follows. entire east coast and the Government had to disburse a

.w
considerable amount for relief activities through the district

o
36. Statement : The State Government granted special status administration machineries.

rd
to the seminar on the importance of protecting the Which of the following may possibly be a follow up measure

p
re
environment around us organised by the local academic to be taken up by the Government?

s
body to create awareness among the citizens. (a) The Government may set up a task force to review the

s
.c
Which of the following may be a probable reason for the post relief scenario in all districts and also to confirm

o
Statement Government's decision? proper end user receipt of the relief supplies.

m
(a) The Central Government has accorded highest priority (b) The Government may set up a committee for proper
to issues related to protection of environment, over all disbursement of relief supplies in future.
other issues. (c) The Government may empower the District Magistrates
(b) In the past the State Government had granted special to make all future disbursements of relief.
(d) The Government may send relief supplies to the
status to such seminars.
affected people in future only after proper assessment
(c) The State Government has failed to implement measures
of the damage caused by such calamities.
for protecting the environment. (e) The Government may not need to activate any follow
(d) The local academic body made a fervent appeal for the up measure.
special status. 40. Statement : It has been reported in a recent study that intake
(e) None of these of moderate quantity of milk chocolate reduces the risk of
37. Statement : Most car manufacturers have marginally reduced suffering from central nervous system related illnesses.
the price of their products despite higher iput costs and Which of the following would weaken the findings of the
increased Government duties and have promised to keep study reported in the above statement?
the prices at the present level for at least the next couple of (a) People generally prefer to eat chocolate when they are
months. young.
Which of the following can be a best possible reason for (b) Majority of those not diagnosed with diseases related
the above step by car manufactures? to central nervous system have stayed away from eating
(a) Car manufactures have not been able to meet their sales chocolates in their lives.
target for the current year so far. (c) Chocolates contain certain elements which strengthen
the functions of the central nervous system.
(b) The Government is planning to review taxes on the
(d) Majority of those suffering from central nervous system
raw materials for manufacturing cars. related diseases are middle aged.
(c) The car sales market is going through a jubilant phase (e) Many of those who suffer from diabetes also suffer
as the volume of sales has picked up considerably in from other major ailments.
the recent past and profit per car has also gone up.
(d) Car buyers are still hesitant to make their purchases DIRECTIONS (41-50) : In each of the questions given below
and may postpone for another few months. which one of the five answer figures on the right should come
(e) Oil marketing companies are contemplating reviewing after the problem figures on the left. If the sequence were
the petrol and diesel prices in order to decide future continued?
market prices. 41. Problem Figures
38. Statements : The prices of vetegables and other food articles s o K
have decreased in the recent months raising hope among o o o
policy planners that the RBI's (Reserve Bank of India) tight K = = v o X c X
grip on supply of liquid money in the market for controlling Answer Figures
inflation may be eased.
o s o o
Which of the following may be a possible action of the
X X
above situation?
(a) The Reserve Bank of India may not reduce its key
X =o X o X

interest rates in near future.


42. Problem Figures
(b) The Government may step in and make certain
concessions to the common people on various issues. o os os s s =
(c) The Reserve Bank of India may consider lowering = c = = o =
s c c c oc
certain rates to bring in more liquidity in the market.
Answer Figures
(d) The RBI may wait for at least another year before taking
any step. s c = c = c= s c
(e) The RBI may collect more data from the market and o= s s s
wait for another four months to ensure they take the c o o o =o
correct step.
y
o
u
rs
m
IBPS Specialist Officer Exam 2012 155

a
h
43. Problem Figures 49. Problem Figures

b
o
s s = z = z T z

o
K P T

b
s z s K T = P s z

.w
s s T P T P s P s K s K = K =

o
rd
Answer Figures Answer Figures

p
s T s P T z T R s T

re
s s s K P K T s z P K P

s
s
s s = z = z P = K s K == z

.c
o
m
44. Problem Figures 50. Problem Figures

Answer Figures Answer Figures

45. Problem Figures

ENGLISH LANGUAGE
Answer Figures DIRECTIONS (51-60) : In the following passage there are blanks,
each of which has been numbered. These numbers are printed
below the passage and against each, five words are suggested,
one of which fits the blank appropriately. Find out the
appropriate word in each case.

46. Problem Figures (51) over the world, rights related to information technology that
are already legally recognised are daily being violated. (52) in the
= = s = s= s = name of economic advancement, political stability or for personal
s s greed and interests. Violations of these rights have (53) new
problems in human social systems, such as the digital divide,
Answer Figures cybercrime, digital security and privacy concerns, all of which
have (54) people's lives either directly or indirectly.
=s = = = = It is important that countries come up with the guidelines
s s for action to (55) the incidences of malicious attacks on the
s s confidentiality, integrity and availibility of electronic data and
systems, computer related crimes, content related offenses and
47. Problem Figures violations of intellectual property rights. (56), threats to critical
infrastructure and national interests arising from the use of the
internet for criminal and terrorist activities are of growing (57).
The harm incurred to businesses, governments and individuals
in those countries in which the internet is used (58), is gaining in
Answer Figures (59) and importance, while in other countries cybercrime threatens
the application of information and communication technology
for government services, health care, trade, and banking. As users
start losing (60) in online transactons and business, the
opportunity costs may become substantial.
51. (a) Entire (b) Lot
48. Problem Figures (c) Great (d) All
s o s s (e) Much
os K K K
K K o z z s 52. (a) scarcely (b) whether
z T z T z o
T T oT (c) and (d) for
(e) hardly
Answer Figures 53. (a) created (b) bent
K K K o T o (c) pressured (d) risen
z s z s z s z Ts z s (e) stopped
T o oT T o K K 54. (a) distanced (b) affected
(c) exaggerated (d) advanced
(e) cropped
y
o
u
rs
m
156 IBPS Specialist Officer Exam 2012

a
h
55. (a) engage (b) conflict

b
DIRECTIONS (66-70): In each of the following sentences, an

o
(c) war (d) combat

o
idiomatic expression or proverb is highlighted. Select the

b
(e) struggle alternative which best describes in use in the sentence.

.w
56. (a) But (b) More

o
(c) Addition (d) Beside 66. Facts spoke louder than words at the Company meeting

rd
(e) Further where the Director tried to paint a rosy picture of the

p
57. (a) concern (b) nature Company's financial health.

re
(c) pattern (d) important (a) Too many facts related to the good financial health of

s
s
(e) matter the Company were presented during the meeting.

.c
58. (a) really (b) figuratively (b) The Company was not doing well financially despite

o
m
(c) widely (d) never the Director saying otherwise.
(e) tandem (c) The Director was very loud while presenting the facts
59. (a) fear (b) days about the Company's financial health during the
(c) positively (d) width meeting.
(e) Scope (d) The facts stated in the meeting supported the Director's
60. (a) tracks (b) measure claims of good financial health of the Company.
(c) confidence (d) mind (e) The Company was doing exceptionally well financially,
(e) grip despite the Director saying otherwise.
67. By initiating the fight with Sakshi in office, Kajal had killed
DIRECTIONS (61-65) : Rearrange the following six sentences the goose that lay the golden egg.
(A), (B), (C), (D), (E) and (F) in the proper sequence to form a (a) By initiating the fight with Sakshi in office, Kajal had
meaningful paragraph; then answer the questions given below left her speechless.
them. (b) Kajal had ruined her chances of success by picking a
(A) It is no wonder that a majority of these excluded and low- fight with Sakshi in office.
achievers come from the most deprived sections of society. (c) Kajal had exacted her revenge by picking a fight with
(B) They are precisely those who are supposed to be empowered Sakshi in office.
through education. (d) Kajal had hurt Sakshi by picking a fight with her in
(C) With heightened political consciousness about the plight office.
of these to-be-empowered people, never in the history of (e) By initiating the fight with Sakshi in office, Kajal had
India has the demand for inclusive education been as fervent missed getting the golden egg.
as today. 68. I let the chips fall where they may and do not worry too
(D) They either never enrol or they drop out of schools at much about what I want to do next.
different stages during these eight years. (a) I take calculated risks.
(E) Of the nearly 200 million children in the age group between (b) I let others do what they want and do not interfere.
6 and 14 years, more than half do not complete eight years (c) I am clumsy.
of elementary education. (d) I do not try to control my destiny.
(F) Of those who do complete eight years of schooling, the (e) I prefer chaos to calm.
achievement levels of a large percentage, in language and 69. After trying hard to convince Narendra to change his ways,
mathematics, is unacceptably low. Raman realised that a leopard cannot change its spots.
61. Which of the following should be the THIRD sentence after (a) Raman realised that Narendra would never change his
rearrangement? ways.
(a) A (b) B (b) Raman realised that Narendra was helpless.
(c) C (d) D (c) Raman realised that he was not good at convincing
(e) F others.
62. Which of the following should be the best FIRST sentence (d) Raman realised that Narendra would change his ways
after rearrangement? soon.
(a) A (b) B (e) Raman realised that someone else was forcing Narendra
(c) C (d) D to act in a certain way.
(e) E 70. Before starting work on our new project, our mentor told us
63. Which of the following should be the SECOND sentence to not count our chickens before they hatched.
after rearrangement? (a) Our mentor warned us against being over confident
(a) F (b) E about achieving success.
(c) D (d) C (b) Our mentor asked us to meticulously count the chicken
(e) B first and then the eggs.
64. Which of the following should be the FOURTH sentence (c) Our mentor warned us against being over enthusiastic
after rearrangement? in implementing the project.
(a) A (b) B (d) Our mentor warned us about all the challenges that lay
(c) C (d) D ahead of us.
(e) E (e) Our mentor informed us about the prizes that we would
65. Which of the following should be the FIFTH sentence after get on succeeding.
rearrangement?
(a) F (b) E DIRECTIONS (71-85) : Read each sentence to find out whether
(c) D (d) B there is any grammatical error or idiomatic error in it. The error,
(e) A if any, will be in one part of the sentence. The number of that part
y
o
u
rs
m
IBPS Specialist Officer Exam 2012 157

a
h
is the answer. If there is 'No error', the answer is (5). (Ignore of articles on the subject, written for newspapers and magazines,

b
o
errors of punctuation, if any). as well as more informal platforms like online forums and blogs.

o
b
Academic engagement and Indian authorship on the subject

.w
71. The third season of (a)/ the popular television show will have also exploded in the last five years. Despite widespread
ends (b)/ on a grand note with (c)/ celebrities dancing and

o
agreement on the importance of innovation in India, there are

rd
having fun. (d)/ No error (e) wide gulfs between different conceptions of innovation and the

p
72. The website, which does not (a)/ accept advertisements path India should take towards securing benefits through

re
and is funded (b)/ entirely by donations,describes itself investments in innovation.

s
(c)/ as the fifth most popular website on the planet. (d)/

s
Many Indian conversations around innovation begin by

.c
No error (e) talking about jugaad, that uniquely Indian approach to a

o
73. As sharing crime statistics for (a)/ the year 2011, the

m
temporary fix when something complex, like an automobile or a
commissioner admitted that (b)/ there had been an undue steam engine stops working. However, many observers have
delay in (c)/ the setting up of an anti-narcotics cell. (d)/ No pointed out that while jugaad is certainly innovative, it is a
error (e) response to the lack of an innovation culture-more a survival or
74. The Moon may be the best place (a)/ to look for aliens as coping mechanism at a time of need than a systematic
their (b)/ footprints on their surface would (c)/ last far longer methodology to effectively address a wide-ranging, complex set
than radio signals. (d)/ No error (e) of problems.
75. The judge advised the government to (a)/ have metered Another specifically Indian approach to innovation that
autorickshaws across the state while (b)/ recounting his has entered into wide currency of late is so-called 'frugal
personal experience where an autorickshaw driver (c)/ made innovation' deemed by many to be the most appropriate for the
him to wait and also demanded `100. (d)/ No error (e) Indian context. In its midterm assessment of the 11th five-year
76. The Company aims (a)/ to nearly double (b)/ its revenues plan, the planning commission stressed the need for innovation
on the back (c)/ of a strongest product pipeline. (d)/ No in India in order to 'accelerate its growth and to make growth more
error (e) inclusive as well as environmentally sustainable.'The document
77. The woman that had (a)/ kidnapped a child has now (b)/ went on to say that 'India needs more frugal innovation that
been apprehended and is being (c)/ held in the city's jail. produces more frugal cost products and services that are
(d)/ No error (e) affordable by people at low levels of incomes without
78. Rose growers in (a)/ the city are waking up (b)/ to the compromising the safety, efficiency, and utility of the products.
benefits (c)/ of collective action. (d)/ No error (e) The country also needs processes of innovation that are frugal in
79. The Minister will have (a)/ a tough task on his hands (b)/ the resources required to produce the innovations. The products
where three different recommendations (c)/ for this year's and processes must also have frugal impact on the earth's
rate reach his desk. (d)/ No error (e) resources.'
80. The current economic scenario (a)/ could possibly undo Two people formulated a similar theory called the
(b)/ the growth that followed (c)/ the economic liberalisation More-from Less for More (MLM theory of innovation) theory of
of 1991. (d)/ No error (e) innovation, which advocates a focus on innovations that allow
81. In a first of its kind study, (a)/ a team of scientists have tried for more production using fewer resources but benefit more
to (b)/ "grow" new stem cells in (c)/ the ear that get damage people. Under this rubric come products that are more affordable
with age. (d)/ No error (e) versions of existing technologies. While both frugal innovation
82. If successful, the research could (a)/ pave the way towards and the MLM theory are certainly valuable in terms of bringing
affordable products and services to a greater number of people,
(b)/ the prevention in untimely deaths (c)/ due to fatal
and may even be considered a necessary first step on India's
illnesses. (d)/ No error (e)
innovation path, they barely graze the surface of what innovation
83. The Ministry has directed Banks (a)/ to do away with their
can accomplish. That is, innovation is capable of bringing about
(b)/ separate promotion policies, a move (c)/ strongly
complete paradigm shifts and redefining the way we perceive and
opposed by the officers' unions. (d)/ No error (e) interact with the world.
84. After a complaint was filed, (a)/ police teams was given the Take the cell phone, for example : it revolutionized
photograph (b)/ of the accused from the CCTV footage (c)/ communication in a previously inconceivable way, provided
recorded at the hotel. (d)/ No error (e) consumers with a product of unprecedented value and created
85. Activists opposing the rail project said (a)/ that the eleven an entirely new market. The cell phone was a result of years of
new flyovers to be built (b)/ would practically ring (c)/ the directed, international innovation efforts and large investments,
death knell for the city. (d)/ No error (e) and would not have ever been created if the people responsible
DIRECTIONS (86-100) : Read the following passage carefully simply set out to make the existing telephone cheaper and more
and answer the questions given below it. Certain words/phrases accessible to all.
have been printed in bold to help you locate them while While jugaad and frugal innovation may be indicative of
answering some of the questions: the Indian potential for innovativeness,this potential is not utilised
or given opportunity to flourish due to the lack of an enabling
In India, innovation is emerging as one of the most important culture.
rubrics in the discourse on how to bring about greater and more India's many diverse and complex needs can be met only
consistent economic and social development. One observes through systematic innovation, and major shifts have to first take
steadily growing investments in R & D accross the country, the place-in our educational institutions, government policies and
setting up of national and state innovation bodies, as well as the commercial firms in order for such an innovation enabling culture
introduction of government sponsored innovation funds. There to come about.
have also been several conferences and debates on innovation The one thing that India's innovation theorists have not
and how to best promote and accomplish it in India, and a number said is that the absence of a culture of innovation is intrinsically
y
o
u
rs
m
158 IBPS Specialist Officer Exam 2012

a
h
linked to many of the most intractable problems facing India as a (c) It is reactive and not a proactive and organised method

b
o
nation. These include poor delivery of government services, of finding solutions to problems

o
b
inadequate systems of personal identification and the absence of (d) It can provide solutions to only simple problems and

.w
widely available financial services for rural poor, health and not complex ones.

o
sanitation failures. This list can go on. Cumulatively, the inability (e) None of these

rd
of India as a nation, society and economy to adequately provide 92. Which of the following is/are true about the cell phone ?

p
for its own population no longer reflects a failure of implementation, (A) The innovation of the cell phone required investment

re
but rather of a failure of innovation, for there are not immediately– of huge capital.

s
s
available off the shelf solutions that would make it possible for (B) The cell phone, when invented was meant to be

.c
affordable to all.

o
these grand challenges facing India to be redressed. Rather, we

m
need to look at these intractable problems from the more (C) The cell phone was made available to the public in a
sophisticated and empowering lens of innovation, for them to very short time from its ideation.
being to be solved. (a) Only (A) (b) Only (A) and (B)
86. Which of the following depict/s the growing importance of (c) Only (B) and (C) (d) Only (B)
innovation in India ? (e) All (A), (B) and (C)
(A) Increased investment in research. 93. What does the Author mean by 'frugal impact on the earth's
(B) Initiation of Government backed funds for innovation. resources' as given passage ?
(C) Increase in number of conferences arranged and articles (a) The damage to the environment should be assessable.
written on innovation. (b) More consumption of natural resources as compared
(a) Only (B) (b) Only (A) and (B) to manmade ones.
(c) Only (C) (d) Only (B) and (C) (c) Minimum impact on the environment in terms of
(e) All (A), (B) and (C) pollution.
87. Which of the following best describes the MLM theory of (d) The impact on the environment should be such that it
innovation ? is reversible.
(a) Maximise output by using least number of resources (e) Minimum usage of earth's natural resources.
and benefiting a small number of people. DIRECTIONS (94–98): Choose the word /group of words which
(b) Maximise resource utilisation and cost there by benefit is most similar the meaning to the word/group of the words
maximum number of people. printed in bold as used in the passage.
(c) Minimise output and resource utilisation. Yet benefit
the maximum number of people. 94. REDRESSED
(d) Benefit most number of people through least usage of (a) addressed (b) equalised
resources and maximum output. (c) restored (d) redone
(e) Benefit most number of people through maximum usage (e) rearranged
of resources and minimising cost. 95. CURRENCY
88. Which of the following is possibly the most appropriate (a) notes (b) usage
title for the passage ? (c) money (d) cash
(a) Innovation At Its Best (e) value
(b) India And The Elixir Called Innovation 96. INTRINSICALLY
(c) Innovation Around The World vis-a-vis India And (a) internally (b) whole heartedly
Neighbouring Countries (c) fundamentally (d) virtually
(d) Worldwide Developments In Innovation (e) unavoidably
(e) Innovation The History 97. INDICATIVE
89. What tone is the author employing in the entire passage (a) causative (b) forthcoming
toget his message across ? (c) verbal (d) abstract
(a) Pessimistic (b) Sarcastic (e) suggestive
(c) Urgent (d) Informative 98. COMPROMISING
(e) Dubious (a) co-operating with
90. Why, according to the author, is India unable to adequately (b) reducing the quality
provide for its people ? (c) hampering the progress
(a) Failure to implement schemes and initiatives meant for (d) conciliating in order to
the Indian populace (e) adjusting for the better
(b) Absence of regulatory authorities to oversee the DIRECTIONS (99–100): Choose the word /group of words which
implementation process. is most opposite in meaning to the word/group of the words
(c) Failure to innovate in order to find solutions. printed in bold as used in the passage.
(d) Lack of governmental schemes and initiatives to redress
the challenges faced by India. 99. LACK
(e) Hesitance of the Indian people in trying out different (a) presence (b) sufficient
schemes provided by the Government for upliftment. (c) charisma (d) adequacy
91. Why, according to some people, is Jugaad' not the answer (e) dearth
to India's problems ? 100. INCONCEIVABLE
(a) Many a times this methodology backfires leading to (a) visible (b) truthful
further complications. (c) incredible (d) apparent
(b) 'Jugaad' provides only cheap solutions to all problems. (e) complex
y
o
u
rs
m
IBPS Specialist Officer Exam 2012 159

a
h
b
o
o
b
.w
o
rd
1. (c) T E AMW O R K N B FU J Q N V

p
7. (e) Letter Number Number or Symbol

re
+1

s
Such combinations are :

s
+1

.c
+1 M 3 # ; U 5 4 ; 16 Ó ; D 8 « ; N 7 d

o
m
+1
–1 +6 +6 +6
–1 8. (b) R ¾¾® $ ¾¾® 4 ¾¾® 8
+6 +6 +6
–1 T ¾¾® @ ¾¾® I ¾¾® H
–1 +6 +6 +6
1 ¾¾® F ¾¾® 6 ¾¾® N
S OME P T DL
9. (d) Number Symbol Letter
+1
+1 Such combinations are :
–1 3 # R ; 8« H ; 6 Ó D
–1
Similarly, 10. (e) 7th to the
left of 20th from the left endmeans 13th from
the left end, i.e., F.
P E RS O N S FQM N R 11. (e) From both the statements
P, W > T > M > R
+1 So data from both the statements are necessary to
+1 answer the question.
+1 12. (d) From both the statements
–1
–1 you will be gone ® ka pa ni sa
–1
he will be there ® ja da ka ni
2. (d)
Data from the both statements are not sufficient to
19 21 2 19 20 1 14 3 5 answer the code for ‘Gone’.
S U B S T A N C E 13. (d) From statement I
It is not given, the brother of Sushant visited Chennai
3. (b) 1 2 3 4 5 6 7 8 9 on which day.
S T E A D F A S T From statement II
Meaningful Word Þ DATE Sushant visited Chennai on Monday, Tuesday,
Thursday, Saturday or Sunday.
4. (a) B A S K E T Data from the both statements are not sufficient to
5 $ 3 % # 1 answer the code for ‘Gone’.
T R I E D 14. (c) From statement I
Starting Point
1 4 « # 2
20 m North
Therefore,
30 m

S K I R T West East
3 %« 4 1 South
5. (a) 8 1 9 4 3 2 7 5
20 m
3 2 7 5 8 1 9 4 From statement II

6. (d)
+2
T ¾¾® –4
E ¾¾® R 30 m Starting Point
+2 –4
U ¾¾® 4 ¾¾® @
20 m

–2 –4
6 ¾¾® D ¾¾® V
–2 +4
J ¾¾® 1 ¾¾® K
+2 –4
N ¾¾® d ¾¾® « 30 m
So data from either of the statement is sufficient to
answer the question.
y
o
u
rs
m
160 IBPS Specialist Officer Exam 2012

a
h
b
15. (e) From statement I Conclusions:

o
I. J © M Þ J < M : Not True

o
Z « P «D

b
( -) ( -) II. E # H Þ E > H : True

.w
From statement II III. M © F Þ M < F : True

o
rd
IV. F # E Þ F > E : Not True
? ?

p
20. (e) D % A Þ D < A

re
(–) (+)
A@ BÞ A= B

s
s
B© KÞ B< K

.c
D K

o
From both the statements K%MÞK<M

m
Therefore, D < A = B < K < M
?(–) ?(+) Conclusions:
Z P D K
I. B $ D Þ B > D : True
(–) (–) (+) (+) II. K # A Þ K > A : True
So Data from both the statements are required to III. M # B Þ M > B : True
establish relationship between K and J. IV. A © M Þ A < M : True
(21-25) : If we examine the given input and pattern of rearrangement
$Þ³ ÓÞ< #Þ> we would find that in the first step one number is rearranged and
in the next step one word is rearranged. This pattern is followed
%Þ£ @Þ= alternately till all the numbers get arranged in ascending order
and the words get arranged in reverse alphabetical order.
16. (d) R # J Þ R > J
21. (b) Input : 86 open shut door 31 49 always 45
J $DÞJ³D
Step I : 31 86 open shut door 49 always 45
D@ KÞD=K
Step II : 31 shut 86 open door 49 always 45
K%TÞK£T
Step III: 31 shut 45 86 open door 49 always
Therefore, R > J ³ D = K £ T
Step IV : 31 shut 45 open 86 door 49 always
Conclusions:
Step V : 31 shut 45 open 49 86 door always
I. T # D Þ T > D : Not True
Step VI : 31 shut 45 open 49 door 86 always
II. T @ D Þ T = D : Not True
So 6 steps are required to complete the rearrangement
Either I or II is true
22. (d) It is not possible to determine the input from any given
III. R # K Þ R > K : True
step.
IV. J $ J Þ J ³ T : Not True
23. (b) Step II: 18 win 71 34 now if victory 61
17. (a) T % R Þ T £ R
Step III: 18 win 34 71 now if victory 61
R$ M Þ R³M
Step IV : 18 win 34 victory 71 now if 61
M@DÞM=D
Step V : 18 win 34 victory 61 now 71 if
D©HÞD<H
Step VI : 18 win 34 victory 61 now 71 if
Therefore, T £ R ³ M = D < H
24. (e) Input : where 47 59 12 are they going 39
Conclusions:
Step I : 12 where 47 59 are they going 39
I. D % R Þ D £ R : True
Step II: 12 where 39 47 59 are they going
II. H # R Þ H > R : Not True
Step III: 12 where 39 they 47 59 are going
III. T © M Þ T < M : Not True
Step IV : 12 where 39 they 47 going 59 are
IV. T % D Þ T £ D : Not True
25. (c) Step II: 33 store 81 75 full of goods 52
18. (c) M @ B Þ M = B
Step III: 33 store 52 81 75 full of goods
B# NÞB> N
Step IV: 33 store 52 of 81 75 full of goods
N$ RÞN³R
Step V: 33 store 52 of 81 75 full of goods
R© KÞ R< K
Step VI : 33 store 52 of 75 goods 81 full
Therefore M = B > N ³ R < K
Conclusions: (26 - 30) :
I. K # B Þ K > B : Not True
II. R © B Þ R < B : True Candidate Conditions
III. M $ R Þ M ³ R : Not True Ans.
(i) (ii) or (a) (iii) (iv) or (b) (v)
IV. N © M Þ N < M : True
19. (b) F # H Þ F > H Rita ü ü – ü ü – ü 5
H@MÞH=M Ashok ü ü – ü – ü ü 3
M© EÞM<E Alok û ü – ü ü – ü 1
E $JÞE³J Swapan ü – ü ü ü – ü 4
Therefore, F > H = M < E ³ J Seema ü ü – ü ü – ü 5
y
o
u
rs
m
IBPS Specialist Officer Exam 2012 161

a
h
b
26. (e) Rita Bhatt satisfies all the conditions. Therefore, she

o
o
would be selected. N N

b
N

.w
27. (c) Ashok Pradhan satisfies conditions (i), (ii), (iii), (b) and

o
(v). Therefoe, his case would be referred to President –

rd
HR.

p
N N

re
28. (a) Alok Verma does not satisfy condition (i). N

s
s
29. (d) Swapan Ghosh satisfies conditions (i), (a), (iii), (iv) and (3) to (4) (4) to (5)

.c
o
(v). Therefore, his case would be referred to GM–HR.
N N

m
N
30. (e) Seema Behal satisfies conditions (i), (ii), (iii), (iv) and
(v). Therefore, she would be selected.
(31 - 35):
N N N

Friend Sex Car Destination


42. (c) The following changes occur in the subsequent figures:
A Female Z Agra (1) to (2) (2) to (3)
B Male Y Chandigarh (3) to (4) (4) to (5)
C M/F Y Chandigarh (5) to (6)
D Female X Delhi
E M/F X Delhi
F Female X/Y Delhi or Chandigarh
G Male X Delhi
H Male Z Agra
43. (a) The following changes occur in the subsequent figures:
(1) to (2) (2) to (3)
31. (d) The sex of C and E is not known
(3) to (4) (4) to (5)
32. (e) None of the combinations is true. (5) to (6)
33. (e) Four of them may be travelling in Car X.
34. (b) C is travelling in Car Y.
35. (a) The passengers of Car Y are travelling to Chandigarh.
36. (a) Option (1) may be a probable reason for the State
Government’s decision because control government
is giving top priority to environment related issues.
37. (a) Option (1) may be a best possible reason because it is
44. (c) In each figure the symbol (0) is moving in anticlockwise
necessary to meet sales target. direction through one and a half and one step
38. (c) Option (3) may be only possible action because other alternately while arrow is rotating in clockwise direction
options are not feasible for RBI and reduction in certain at an angle of 135° and 90° alternately.
rates would control inflation in market. 45. (c) In each figure 4 arrows are reversing their direction in
39. (b) Option (2) is the only follow up measure because a particular order.
committee can only assess considerable amount for 46. (e) The following changes occur in the subsequent figures:
relief activities. (1) to (2) (2) to (3)
(3) to (4) (4) to (5)
40. (e) Option (5) would weaken the findings of the study
(5) to (6)
because the person suffering from diabetes cannot
intake milk chocolate and cannot avail its benefits.
41. (e) The following changes occur in the subsequent figures:
(1) to (2)
(2) to (3)
(5) to (6)
y
o
u
rs
m
162 IBPS Specialist Officer Exam 2012

a
h
b
47. (d) From Problem Figure (1) to (2) the two designs from 69. (a) A leopard cannot change its spots means one cannot

o
change one's basic nature.

o
the left move to the right and the two designs from the

b
70. (a) To not count chicken before they hatch means to not

.w
right move to the left. From Problem Figure (2) to (3) all
plan or expect the outcome or be overconfident about

o
the four designs are inverted and the two designs from

rd
the right and left interchange positions. From Problem it before it happens.

p
re
Figure (3) to (4) the two designs from the left are inverted 71. (b) The popular television show will end.

s
72. (e) No error.

s
and the third design from the left moves to the extreme

.c
left position. These three steps are continued in the 73. (a) While sharing the crime statistics.

o
m
subsequent figures in the same order. 74. (a) Footprints on it surface.
48. (a) In each subsequent figure two pairs of designs does 75. (b) To meter the autorickshaws.
not change their positions while other designs 76. (c) Its revenues on the basis of.
interchange positions with their adjacent designs. 77. (c) Been apprehended and is.
78. (e) No error.
49. (a) The following changes occur in the subsequent figures:
79. (b) Tough task at his hands.
(1) to (2) (2) to (3)
80. (e) No error.
(3) to (4) (4) to (5)
81. (d) The ear that get damaged with age.
(5) to (6)
82. (c) Prevention of untimely deaths
83. (e) No error.
84. (b) Police teams were given.
85. (e) No error.
86. (e) It can be deciphered from the first paragraph.
87. (d) It is mentioned in the fourth paragraph.
50. (d) In the subsequent figures the («) moves one and one 88. (b) This is the most appropriate title for the passage as it
and a half step(s) in clockwise direction alternately,
describes the growing importance of innovation India
the design ( T ) rotates through 90° and 135° clockwise has realized and is taking all possible steps for it. This
alternately while the design ( ) rotates 45° and 90° passage also discusses various dimensions of
anti-clockwise alternately. implementing innovation withy respect to India.
51. (d) 52. (b) 53. (a) 54. (b) 55. (d) 56. (e) 89. (c) The tone of the passage is critical. It calls for corrective
57. (a) 58. (c) 59. (e) 60. (c) 61. (e) 62. (e) action in urgency.
63. (c) 64. (a) 65. (d) 90. (c) It is mentioned in the last paragraph.
66. (b) Facts spoke louder than the words means that what is 91. (c) It is mentioned in the second paragraph.
suggested by evidence is more credible than what is 92. (a) It is mentioned in the fifth paragraph.
spoken. 93. (c) Frugal means very less.
67. (b) Killing the goose that lay golden eggs means destroying
94. (a) 95. (b) 96. (c) 97. (e)
something that will give benefits or profitable returns.
68. (d) Let the chips fall where they may means to do whatever 98. (b) 99. (d)
one wants to without worrying too much about the 100. (d) Inconceivable means incredible, difficult to believe.
consequence(s). Apparent is opposite.
y
o
u
rs
m
a
h
b
o
INDIAN OVERSEAS BANK PO ONLINE

o
b
.w
EXAM 2013

o
rd
p
Based on Memory

re
s
s
.c
6. Who amongst the following faces P ?
REASONING ABILITY

o
m
(a) A (b) D
(c) C (d) E
DIRECTIONS (Qs. 1–3) : Study the following information (e) Cannot be determined
carefully to answer the given questions : 7. Who amongst the following sits exactly between T and R ?
There are six people – P, Q, R, S, T and U – each having different (a) V (b) Q
weight. P is heavier than U but lighter than Q. T is heavier than R (c) S (d) P
but lighter than P. Q is lighter than S. R is not the lightest. The (e) Cannot be determined
second lightest pkerson weight 52 kilogram while the third heaviest 8. Four of the following five are alike in a certain way based on
is of 64 kilogram. the given seating arrangement and thus form a group.
Which is the one that does not belong to the group ?
1. Who among the following may be of 58 kilogram in weight?
(a) F (b) Q
(a) P (b) Q (c) T (d) C
(c) T (d) R (e) E
(e) U
2. How many persons are heavier than R? DIRECTIONS (Qs. 9–13) : Study the following information
(a) None (b) One carefully and answer the given questions :
(c) Two (d) Three In a certain code language :
(e) More than three “only in serial order” is written as “ve pu na to”.
3. Which of the following may represent the weight of Q? “order in the state” is written as “li ve su pu”.
(a) 62 kg (b) 66 kg “the logical idea only” is written as “su na ri jo”.
(c) 60 kg (d) 58 kg “in idea or theory” is written as “zt jo bk pu”.
(e) 56 kg 9. Which of the following is code of ‘theory’ ?
(a) zt (b) bk
DIRECTIONS (Qs. 4-8) : Study the following information (c) jo (d) pu
carefully answer the given questions : (e) Either ‘zt’ or ‘bk’
Twelve persons are sitting in two parallel rows containing six 10. The code ‘li ri to ve’ may represent
persons each, in such a way that there is an equal distance between (a) serial order theory only
adjacent persons. In row–1, A, B, C, D, E and F are seated (but not (b) only idea state order
necessarily in the same order) and all of them are facing south. In (c) state logical serial order
row– 2, P, Q, R, S, T and V are seated (but not necessarily in the (d) serial theory state the
same order) and all of them are facing north. Therefore, in the (e) only the idea logical
11. Which of the following represent “logical idea is only order” ?
given seating arrangement each person seated in a row faces
(a) jo na ri ge ve (b) ve na ri jo pu
another person of the other row.
(c) ri ve na zt bk (d) bk to pu jo ve
A sits third to the left of E. The person facing A sits second to the (e) na ve su li pu
left of T. Two persons are sitting between T and P. C and D are 12. Which of the following is code of “logical” ?
immediate neighbours. C and D do not sit at any of the extreme (a) su (b) jo
ends of the line. Only one person sits between B and C. The (c) na (d) ri
person facing D is an immediate neighbour of Q. V is not an (e) None of these
immediate neighbour of P. S does not face A. 13. Which of the following is code of “serial” ?
4. Who amongst the following sits seconds to the right of the (a) pu (b) to
person who faces R ? (c) ve (d) su
(a) C (b) D (e) Cannot be determined
(c) B (d) E DIRECTIONS (Qs. 14–18) : In each question below is given a
(e) Cannot be determined statement followed by two courses of action numbered I and II. A
5. Which of the following statements regarding B is true ? course of action is a practicable and feasible step or
(a) B sits second to the left of C administrative decision to be taken for follow-up, improvment,
(b) A sits to immediate left of B or further action in regard to the problem, policy, etc. On the
(c) T faces B basis of the information given in the statement, you have to
(d) D is an immediate neighbour of B assume everything in the statement to be true, and decide which
(e) The person who faces B is an immediate neighbour of S of the suggested courses of action logically follow (s) far pursuing.
y
o
u
rs
m
164 Indian Overseas Bank PO Online Exam 2013

a
h
b
Give answer (a) if only I follows. 20. What will come in the place of question-mark, if it is provided

o
Give answer (b) if only II follows. that ‘J is the daughter-in-law of T’ in the expression ‘J % B

o
b
Give answer (c) if either I or I follows. ? K $ T’ ?

.w
Give answer (d) if neither I nor II follows. (a) @ (b) %

o
Give answer (e) if both I and II follow. (c) + (d) $

rd
14. Statement: The sales of ballpoint pens manufactured by (e) % or +

p
re
company Lixus have gone down considerably ever since DIRECTIONS (Qs. 21–25) : In these questions relationship

s
the same company introduced a gel-ink pen in the market.

s
between different elements is shown in the statements. The

.c
Courses of action

o
statements are followed by two conclusions.
I. Ball point pens should not be manufactured by Lixus

m
any more. Give answer (1) if only Conclusion I is true.
II. Lixus should immediately withdraw all gel-ink pens from Give answer (2) if only Conclusion II is true.
the market so as to force people to buy ballpoint pens. Give answer (3) if either Conclusion I or II is true.
15. Statement: Nearly 26 per cent of all engineering graduates Give answer (4) if neither Conclusion I nor II is true.
have been left unemployed owing to severe recession Give answer (5) if both Conclusions I and II are true.
worldwide. (21–22) :
Courses of action Statements : R > T < M = Z ; C > T > B
I. All the unemployed students should be advised to 21. Conclusions:
take up jobs in foreign countries. I. Z > C II. B < Z
II. The unemployed students should be advised to take
22. Conclusions:
up jobs only after the recession is over.
I. B < C II. R > C
16. Statement : The government airline has been making huge
(23–24) :
losses even as private airliners continue to prosper and
Statements : P > Q > T; Q < R; T > A
make substantial profit.
23. Conclusions :
Courses of action
I. A < Q II. A < P
I. All the private airlines should be banned from their
24. Conclusions :
operation in the country.
II. The government airline should be instructed to I. T < P II. R > A
increase passenger fare significantly in order to 25. Statements :
increase its profit. P > T = U < W; Q > R> W
17. Statement : Many private schools have been violating Conclusions
government directives and have been charging more fees I. T = R II. U < R
than the specified limits. DIRECTIONS (Qs. 26–30) : In each question below are two or
Courses of action three statements followed by two conclusions numbered I and II.
I. The government should identify such schools and take You have to take the two given statements to be true even if they
stringent actions against them. seem to be at variance from commonly known facts and then
II. The government should not limit school fees and keep decide which of the given conclusions logically follows from the
it flexible for the schools to decide. given statements disregarding commonly known facts.
18. Statement : A large number of patients admitted in a Give answer (a) if only conclusion I follows.
particular hospital have been diagnosed with a serious Give answer (b) if only conclusion II follows.
contagious disease. Give answer (c) if either conclusion I or conclusion II follows.
Courses of action Give answer (d) if neither conclusion I nor conclusion II follows.
I. These patients should be immediately evicted from Give answer (e) if both conclusion I and conclusion II follow.
the hospital for fear of spread of the disease. (26-27) :
II. The hospital should make appropriate quarantine Statements : No cow is a bull.
arrangements to control the spread of disease. All bulls are animals.
DIRECTIONS (Qs. 19–20) : Study the following information Some animals are mammals.
carefully to answer the given questions : 26. Conclusions :
I. At least some mammals are animals.
‘P + Q’ means ‘P is the sister of Q’. II. Some mammals being bulls is a possibility.
‘P @ Q’ means ‘P is the wife of Q’. 27. Conclusions :
‘P $ Q’ means ‘P is the son of Q’. I. At least some animals are bulls.
‘P % Q’ means ‘P is the mother of Q’. II. No animal is a cow.
19. What is the relation between ‘J and A’ in the expression ‘A 28. Statements :
@ F $ M % J + T’ ? Some pencils are pens.
(a) J is the mother-in-law of A No pen is eraser.
(b) A is the aunt of J All sharpeners are erasers.
(c) J is the sister-in-law of A Conclusions :
(d) A is the husband of J I. No eraser is a pencil.
(e) None of these II. All pencils can never be sharpeners.
y
o
u
rs
m
Indian Overseas Bank PO Online Exam 2013 165

a
h
b
(29-30) : Input : more presence required 12 42 70 for 63 37 good work 28

o
Statements : Some stars are planets. Step I: for 70 more presence required 12 42 63 37 good work 28

o
b
Some planets are moons. Step II: good 63 for 70 more presence required 12 42 37 work 28

.w
No moon is a sun. Step III: more 42 good 63 for 70 presence required 12 37 work 28

o
29. Conclusions :

rd
Step IV: presence 37 more 42 good 12 work 28
I. No star is a sun.

p
Step V: required 28 presence 37 more 42 good 63 for 70 12 work

re
II. All planets being suns is a possibility.
Step VI: work 12 required 28 presence 37 more 42 good 63 for 70

s
30. Conclusions :

s
And Step VI is the last step of the rearrangement as the desired

.c
I. All suns being stars is a possibility.

o
II. Some stars are planets. arrangement is obtained. As per rules followed in the above steps,

m
find out in each of the questions the appropriate step for the
DIRECTIONS (Qs. 31–35) : Study the following information given input.
carefully to answer the given questions : Input for the questions:
Eight persons – A, B, C, D, E, F, G and H – are sitting around a Money 48 24 18 wanted for investment 65 90 lock credit 32
circular table facing towards the centre, but not necessarily in the 36. Which step number is the following output ?
same order. All of them like different colours viz. yellow, purple, lock 32 investment 48 for 65 credit 90 money 24 18 wanted
orange, black, white, green, red and blue. B is sitting second to (a) Step II (b) Step III
the left of D. D likes blue colour. B does not like yellow colour. D (c) Step IV (d) Step V
is the immediate neighbour of that two persons who like red and (e) There is no such step
purple colours respectively. Three persons sit between B and the 37. Which word/number would be fourth from the left in step
person who likes green colour. F and A are immediate neighbours.
IV ?
E does not like black, yellow or purple colour. H is an immediate
neighbour of G. A is immediate neighbour of both who like red and (a) for (b) 48
black colours respectively. E is sitting just opposite to H who (c) 65 (d) 32
likes orange colour. (e) credit
31. How many persons are seated between H and D, if we go 38. Which word/number would be second to the right of the
clockwise from H to D ? fifth element from the left in step V?
(a) Two (b) Three (a) for (b) 65
(c) Four (d) One (c) 48 (d) investment
(e) None (e) 32
32. Which of the following pairs of persons represents the 39. How many elements (words/ numbers) are there between
immediate neighbours of G ? “for” and “18” in step III ?
(a) H and the person who likes black colour
(a) Four (b) Three
(b) F and the person who likes green colour
(c) B and H (c) Six (d) Seven
(d) The person who likes orange colour and C (e) Five
(e) None of these 40. Which of the following represents the position of “credit”
33. What is E’s position with respect to the person who likes in step V?
orange colour ? (a) Third from the right (b) Fifth from the left
(a) Third to the left (b) Fifth to the left (c) Sixth from the left (d) Fourth from the right
(c) Fifth to the right (d) Fourth to the right (e) Sixth from the right
(e) Second to the right
34. Starting from A, if all the persons are made to sit in the ENGLISH LANGUAGE
alphabetical order in clockwise direction, the positions of
how many persons (excluding A will remain unchanged) ? DIRECTIONS (Qs. 41-50) : Read the following interview and
(a) None (b) One answer the given questions based on. Some words have been
(c) Two (d) Three printed in bold to help you locate them while answering some of
(e) More than three the questions.
35. Who amongst the following is sitting exactly between the
A pioneering new book, Gender and Green Governance, explores
person who likes red colour and F?
(a) G (b) H a central question: If women had adequate representation in
(c) B (d) D forestry institutions, would it make a difference to them, their
(e) A communities, and forests as a national resource? Interview with
the author.
DIRECTIONS (Qs. 36-40) : Study the following information Why has access to forests been such a conflict-ridden issue?
carefully and answer the questions given below: This is not surprising. Forests constitute not just community and
A word and number arrangement machine when given an input national wealth, but global wealth. But for millions, forests are
line of words and numbers rearranges them following a particular also critical for livelihoods and their daily lives.
rule in each step. The following is an illustation of input and various Your first book. Cold Hearts and Barren Slopes (1986), was
steps of rearrangement. (All the numbers are two digit numbers). about forests. Is there an evolution of argument here?
y
o
u
rs
m
166 Indian Overseas Bank PO Online Exam 2013

a
h
b
Yes indeed : In Cold Hearts and Barren Slopes, I had argued that 41. What was author’s view on ‘Social Forestry Scheme’ ?

o
social forestry, with its top-down implementation and focus on (a) A great success

o
b
commercial species, was neither ‘social’ nor ‘forestry’, and would (b) Beneficial for villagers

.w
protect neither forests nor village livelihoods. The answer, I (c) Neither good nor bad

o
argued, lay in allowing forests communities to manage local

rd
(d) Should have been implemented as ‘top-down’
forests. Finally, in 1990, India launched the joint forest management

p
(e) None of these

re
programme and Nepal also started community forestry. So I
42. Which of the following is one of the reasons of forests

s
decided to see for myself how community forestry was actually

s
being a conflict-ridden issue ?

.c
doing.

o
(a) Some countries have larger forest cover

m
Between 1995 and 1999, I travelled extensively across India and
(b) There is less awareness about global warming
Nepal and found paradox : Forests were indeed becoming greener
but women’s problem of firewood shortages persisted and in many (c) High dependence of many on forests
cases had become more acute. Also, despite their high stakes in (d) Less representation of women
forests, women continued to be largely excluded from forest (e) Less representation of local women
management. I coined the term “participatory exclusions” to 43. The author is advocating inclusion of
describe this. However, the current book is less about women’s (a) More landless women
exclusion. I ask : What if women were present in forest governance (b) More landed women
? What difference would that make ? (c) More women irrespective of their financial status
But has this question not been raised before ? (d) Local people
Economists researching environmental collective action have paid (e) Younger women in the age group of 25-33 years
little attention to gender. Scholars from other disciplines focussing 44. Which of the following best describes “participatory
on gender and governance have been concerned mainly with exclusion”, as used in the interview ?
women’s near absence from governance institutions. The (a) Outside support
presumption is that once women are present all good things will
(b) Overdependence
follow. But can we assume this ? No. Rural women’s relationship
(c) Benefitting without self interest
with forests is complex.
(d) Contributing with profits
On the one hand, their everyday dependence on forests for
fire-wood, fodder, etc, creates a strong stake in conservation. On (e) None of these
the other, the same dependence can compel them to extract heavily 45. In the second question, the interviewer asked -‘Is there an
from forests. As one landless woman told me : ‘Of course, it hurts evolution of argument here ?’ Which of the following best
me to cut a green branch but what do I do if my children are describes that ?
hungry? Taking an agnostic position, I decided to test varied (a) From Barren to Greener slopes
propositions, controlling for other factors. (b) From local groups to local groups with more women
What did you find ? (c) A fine balance between conservation and commercial
First, women’s greater presence enhances their effective voice in forestry
decision-making. And there is a critical mass effect : If forests (d) Top-down approach to Community forestry
management groups have 25–33 per cent female members in their (e) Participatory exclusion to Greener slopes
executive committees it significantly increases the likelihood of 46. Why does author say, ‘Rural women’s relationship with
women attending meetings, speaking up and holding office. forests is complex’ ?
However, the inclusion of landless women makes a particular (a) Dependence forces them to extract and also have
difference. When present in sufficient numbers they are more concern for conservation
likely to attend meetings and voice their concerns than landed (b) If they protect forests, their livelihood is severely
women. So what matters is not just including more women, but affected
more poor women. (c) Poor women have been excluded from forest
Second, and unexpectedly, groups with more women typically management
make stricter forest use rules. Why is this the case ? Mainly (d) They cannot be asked to restore forests which are
because they receive poorer forests from the forest department. critical for them
To regenerate these they have to sacrifice their immediate needs.
(e) Greener forests do not meet the requirement of
Women from househ olds with some lan d have
firewood
some fallback. But remarkably even in groups with more landless
47. Landless women, when in decision making role
women, although extraction is higher, they still balance self-
interest with conservation goals, when placed in decision-making (a) extract much more from forest
positions. (b) improve their own financial status
Third, groups with more women outperform other groups in (c) do not care for forest
improving forest conditions, despite getting poorer forests. (d) are able to meet conservation objectives as well as
Involving women substantially improves protection and conflict their own interest
resolution, helps the use of their knowledge of local biodiversity, (e) fulfill their own interest at the cost of conservation
and raises children’s awareness about conservation. goals
y
o
u
rs
m
Indian Overseas Bank PO Online Exam 2013 167

a
h
b
DIRECTIONS (Qs. 48-50) : Choose the word/group of words 59. The pilot knew she would be able to see the ____ lights of

o
the city from her cockpit window, but she would not see the

o
which is most nearly the same in meaning to the word/group of

b
words printed in bold. fireworks explode to welcome the new year as she would

.w
have ____ to cruising altitude.

o
48. CONTROLLING (a) few, soared (b) divine, escalate

rd
(a) holding in check (b) increasing (c) glistening, jumped (d) shining, reached

p
re
(c) decreasing (d) passing (e) glittering, climbed

s
(e) ignoring 60. The New Year has ____ in good news for city hotels as

s
.c
49. PARADOX most properties are ____ for the whole month.

o
(a) similarity (b) position (a) brought, deserted (b) ushered, packed

m
(c) anomaly (d) difference (c) pushed, full (d) steered, renovating
(e) excuse (e) escorted, vacant
50. ACUTE
(a) accurate (b) severe DIRECTIONS (Qs. 61-65) : Rearrange the following seven
(c) dull (d) focused sentences (A), (B), (C), (D), (E), (F) and (G) in the proper
(e) refined sequence to form a meaningful paragraph; then answer the
questions given below them.
DIRECTIONS (Qs. 51–55) : Read each sentence to find out
whether there is any grammatical error or idiomatic error in it. (A) But seriously, how much would you pay to know what
The error, if any, will be in one part of the sentence. The number thoughts are swimming around in someone else’s head ?
of that part the answer. If there is no error, the answer is (e). (B) In most fictional movies thus, the idea of reading minds - of
(Ignore errors of punctuation, if any). seeing the private intentions of another, and the possibility
of intervening in those plans has always been highly
51. The dense fog on Friday morning (a)/ forced the Airport attractive
authority of India to divert (b) eight flights, includes two (C) Such fantastical questions have long been the bread and
international flights, (c)/ out of the International Airport (d) butter of fiction.
No error (e). (D) Today, more than four centuries since the phrase,
52. The civic authorities had scrapped the trade license (a)/ of “A penny for your thoughts ?”, was first recorded,
a well known restaurant after investigation proved that (b)/ inflationary accounting makes that ancient penny worth
it had chopped down full grown tree simple because (c) / more than $40.
the tree was blocking the view of the restaurant. (d)/ No (E) The going rate for a “thought” a probe into the thinking of
error (e). another was once quite a bargain.
53. Investigating officers have found (a)/ a lot of gaps between (F) And if you could really know their truthfulness how much
(b)/ the information they gather on their own (c)/ and the more would you pay?
version of events narated by the accused. (d)/ No error (e). (G) Even with the sliding value of the dollar, this still seems
54. The death of the man has brought (a)/ to the fore flight of quite a bargain.
thousands of grants (b)/ languishing behind bars in India 61. Which of the following should be the SECOND sentence
after (c)/ being apprehended in the border. (d) No error (e). after rearrangement?
55. Only recently (a)/ did the manager ager realized that (b)/ the (a) F (b) D
information provided to him (c)/ by his employees is (c) C (d) G
fabricated. (d)/ No error (e). (e) A
DIRECTIONS (Qs. 56-60) : Each question below has two blanks, 62. Which of the following should be the FOURTH sentence
each blank indicating that something has been omitted. Choose after rearrangement?
(a) A (b) B
the set of words for each blank which best fits, the meaning of
the sentence as a whole. (c) G (d) D
(e) F
56. Much of the ____ that cricket has is due to the fact it is a 63. Which of the following should be the SIXTH sentence after
____ sport. rearrangement?
(a) allure, lucrative (b) criticism, controversial (a) E (b) G
(c) attraction, unpopular (d) flak, great (c) C (d) B
(e) comments, unusual (e) A
57. Since foggy weather ____ visibility by several metres, the 64. Which of the following should be the SEVENTH (Last)
railways has either partially/or diverted some of the trains. sentence after rearrangement?
(a) improves, started (b) impairs, called off (a) A (b) B
(c) hampers, withdrawn (d) decrease, stopped (c) F (d) D
(e) reduces, cancelled (e) E
58. The once ____ district is gradually being ____ of its green 65. Which of the following should be the FIRST sentence after
cover. rearrangement?
(a) remote, eroded (b) arid, replenished (a) A (b) B
(c) beautiful, devoid (d) picturesque, depleted (c) C (d) D
(e) lush, rob (e) E
y
o
u
rs
m
168 Indian Overseas Bank PO Online Exam 2013

a
h
b
DIRECTIONS (Qs. 66–75) : In the following passage, there are 76. A nation that had been once tried to kill him was the first to

o
embrace him.

o
blanks, each of which has been numbered. These numbers are

b
printed below the passage and against each, five words are (a) had once been trying to

.w
suggested, one of which fits the blanks appropriately. Find out (b) had once tried to

o
(c) once had been tried to

rd
the appropriate word in each case.
(d) had to once try and

p
With the U.S. military tied down on two fronts and the rest of the

re
(e) No correction required

s
world growing (66) to American power, the challenges for Rice are
77. Had he known more about the policies of the company, he

s
.c
as (67) as they have been for any Secretary of State in the past
might not have accepted the offer.

o
three decades. After six years of tussling with others on Bush’s

m
(a) He had known more
national-security team, Rice has seen off her rivals and (68) as the
(b) Did he know more
principal spokesperson for Bush’s foreign policy. Her reward has
been to (69) responsibility for selling a failed policy in Iraq and (c) Since not more was known
framing a legacy for Bush at a time when (70) in the world are in the (d) If he would know more
mood to help her. “Bush is severely (71) and has very little credibility (e) No correction required
or support at home or abroad,” says Leslie Gelb, former president 78. His life is an example of how the human will can flourish
of the Council on Foreign Relations. “That is (72) true for his even in harsh conditions.
Secretary of State. So they are basically flailing around.” (a) what the human will can
That’s a grim assessment, since the (73) to international order are (b) how the human can and will
bigger today than at any other time since the end of the cold war. (c) when the human will can
The most immediate source of (74) emanates from Iraq, where the (d) where the human will can
country’s civil war risks (75) a region wide conflict. (e) No correction required
66. (a) resistant (b) subservient 79. The social worker passionately stroked the annoyed
(c) immune (d) cordial passenger in order that be pacified.
(e) indifference (a) to pacify him (b) that to pacify
67. (a) obvious (b) trivial (c) to be pacified for (d) that to be pacified with
(c) superfluous (d) daunting (e) No correction required
(e) rewarding 80. They were no longer able to provide the help their children
68. (a) renamed (b) emerged need.
(c) appointed (d) entrusted (a) helped their children need
(e) visited (b) help their children needed
69. (a) shirk (b) avoid (c) help that their children need
(c) transfer (d) visualize (d) help that their children would need
(e) inherit (e) No correction required
70. (a) people (b) few
(c) diplomats (d) autocrats QUANTITATIVE APTITUDE
(e) most
71. (a) intensified (b) master-minded 81. In how many ways the letters of the word SACRED can be
(b) weakened (d) projected arranged so that vowels come together ?
(e) supported (a) 240 (b) 120
72. (a) not (b) uniformly (c) 320 (d) 720
(c) remotely (d) partially (e) None of these
(e) also 82. How many natural numbers are there lying between 134
73. (a) admirations (b) threats and 467 which are divisible by 7 ?
(c) pleasantries (d) demands (a) 46 (b) 47
(e) accolades (c) 49 (d) 51
74. (a) instability (b) fuel (e) None of these
(c) energy (d) peace 83. A sum of ` 3200 becomes ` 3456 in two years at a certain
(e) atrocity rate of simple interest. What is the rate of interest per annum ?
75. (a) defusing (b) demolishing (a) 5.5% (b) 6%
(c) terminating (d) igniting (c) 4% (d) 4.5%
(e) extinguishing (e) None of these
DIRECTIONS (Qs. 76–80) : In each question below, a sentence 84. A discount of 15% is given the marked price of an article
is given with a part of it printed in bold type. That part may The shopkeeper charges sales tax of 6% on the discounted
contain a grammatical error. Each sentence is followed by price. If the selling price be ` 1081.20, what is the marked
phrases (a), (b), (c) and (d). Find out which phrase should price of the article ?
replace the phrase given in bold to correct the error, if there is (a) ` 1185.20 (b) ` 1250.20
any, and to make the sentence grammatically meaningful and (c) ` 302 (d) ` 1200
correct. If the sentence is correct as it is and ‘No correction is (e) None of these
required.’ mark (e) as the answer.
y
o
u
rs
m
Indian Overseas Bank PO Online Exam 2013 169

a
h
b
DIRECTIONS (Qs. 85–89) : What will come in place of the 93. In which school the number of students is the highest ?

o
(a) A (b) B

o
question mark : (?) in the following number series ?

b
(c) C (d) F

.w
85. 21 10.5 ? 15. 75 31.5 78.75 (e) D

o
(a) 10.5 (b) 11.5 94. In which two other schools the number of students is same

rd
(c) 12.5 (d) 10.25 as that of studying in schools A and C ?

p
re
(e) None of these (a) A and B (b) D and E

s
86. 6 19 58 ? 214 331 (c) E and F (d) D and F

s
.c
(a) 113 (b) 123 (e) B and D

o
(c) 133 (d) 143

m
95. What is the average number of students studying in schools
(e) None of these B, D and E ?
87. ? 16 28 58 114 204 (a) 11200 (b) 10600
(a) 7 (b) 9 (c) 10000 (d) 10500
(c) 14 (d) 6 (e) None of these
(e) 10 96. Ratio between the number of students studying in schools
88. 13.76 14.91 17.21 20.66 ? 31.01 B and E is
(a) 25.66 (b) 24.36 (a) 7 : 3 (b) 5 : 3
(c) 24.26 (d) 25.26 (c) 3 : 5 (d) 6 : 7
(e) 25.36 (e) 7 : 6
89. 15 ? 24 33 97 122 97. By what percent the number of students studying in school
(a) 20 (b) 19 D is less than that of studying in school C ?
(c) 17 (d) 18
(e) 16 2 1
(a) 16 % (b) 33 %
90. A sum of ` 221 is divided among X, Y and Z such that X 3 3
gets ` 52 more than Y. Y gets ` 26 more than Z. The ratio of
1 2
the shares of X, Y and Z respectively is: (c) 13 % (d) 26 %
(a) 9 : 5 : 3 (b) 9 : 3 : 5 3 3
(c) 5 : 9 : 3 (d) 10 : 6 : 5 (e) 30%
(e) None of these DIRECTIONS (Qs. 98–102): In the following table the marks
91. A boat covers 20 km in 4 hours along the current and obtained by 6 students in 8 different subjects are given. Study
9 km in 3 hours against the current. What is the speed of the the table carefully to answer the questions. Maximum marks of
current ? each subject is 100.
(a) 2 kmph (b) 1 kmph
(c) 1.5 kmph (d) 1.75 kmph Subjects
Sunil Meeta Farhan Shweta Davis Kirti
(e) None of these Students
92. A sum of ` 2200 is invested at two different rates of interest. English 45 42 53 81 57 70
The difference between the interests got after Chemistry 63 35 65 50 51 52
4 years is ` 202.40. What is the difference between the rates Hindi 78 46 56 82 42 55
of interest ? Algebra 54 54 43 81 62 42
(a) 3.3% (b) 2.3% Physics 81 29 67 85 66 62
(c) 3.5% (d) 2.5% Geography 82 59 45 36 41 38
(e) None of these Biology 63 48 69 88 53 76
DIRECTIONS (Qs. 93–97) : In the following pie-chart, the History 72 62 58 73 72 49
percentage of students studying in 6 different schools has been 98. What is the average of marks obtained by all students in
shown. Study the pie-chart carefully to answer the qestions. Algebra ?
(a) 52 (b) 54
Percentage of students studying 6 different schools A, B, C, D, E
(c) 56 (d) 66
and F. Total number of students = 75000
(e) 57
A
F 99. What is the differnce between the total marks obtained by
Sunil and Shweta ?
11% (a) 45 (b) 42
23%
(c) 35 (d) 38
B (e) 36
E 12% 14%
100. By what percent approximately are the total marks obtained
by Sunil more than that of Meeta ?
16% (a) 43% (b) 45%
24% (c) 46% (d) 40%
(e) 42%
D C
y
o
u
rs
m
170 Indian Overseas Bank PO Online Exam 2013

a
h
b
101. What is the percentage of marks obtained by Farhan ? 1 2 3

o
(a) 52% (b) 57% 112. 3 + 6 + ? = 13

o
4 7 28

b
(c) 54% (d) 58%

.w
2 4
(e) 60% (a) 3 (b) 3

o
102. What is the average of marks obtained by Davis ? 7 7

rd
(a) 53.5 (b) 56.5 3 5

p
(c) 3 (d) 3

re
(c) 55.5 (d) 58 7 7

s
(e) 57

s
6

.c
(e) 3

o
DIRECTIONS (Qs. 103-112): What approximate value will come 7

m
in place of the question mark (?) in the following questions ? 113. The circumference of a circular playground is 308 metre.
(You are not required to find the exact value). There is 7 metre wide path around the ground. The area of
the path is
103. 2371 ÷ 6 + (43 × 4.35) = ? (a) 2130 sq. metre (b) 2410 sq. metre
(a) 582 (b) 590 (c) 2510 sq. metre (d) 2310 sq. metre
(c) 600 (d) 570 (e) None of these
(e) 595 114. The average of the 9 consecutive positive integers is 63.
104. 3
3380 + 1300 = ? The product of the largest and smallest integers is
(a) 56 (b) 51 (a) 3935 (b) 3953
(c) 53 (d) 54 (c) 3853 (d) 3835
(e) 55 (e) 3635
115. In two vessels A and B, there is mixture of milk and water.
105. (4.989)2 + (21.012)3 + 1090 = ? The ratio of milk and water in these vessels is 5 : 2 and 8 : 5
(a) 9219 (b) 9391 respectively. In what ratio these mixtures be mixed together
(c) 9319 (d) 9129 so that the ratio of milk and water in the new mixture becomes
(e) None of these 9:4?
13 (a) 7 : 2 (b) 2 : 7
106. 7020 ¸ 2.99 × =? (c) 3 : 5 (d) 5 : 3
29
(e) 7 : 9
(a) 1040 (b) 1100
(c) 1060 (d) 1050 DIRECTIONS (Qs. 116-120) : In the following bar diagram, the
(e) None of these number of mobile phones and laptops (in thousands) sold by 6
107. 24.99% of 5001 – 65.01% of 2999 = ? different companies in a certain month has been given. Study
(a) 840 (b) 500 the bar diagram carefully to answer the questions.
(c) 700 (d) –500 Number of mobile phones and laptops (in thousands) sold by 6
(e) –700 different companies in a month.
1 2 Mobile phone
Number of mobile phones

- -
108. (81) 2 - (64) 3 =? Laptops
3 1 40 33
(a) (b) 17 30
19 16 30 22 27
15 2017
and laptops

20 13 18
7 1 13 12
(c) (d) 10
144 9
(e) None of these 0 Q R S
P T U
109. 331.8 ¸ 23.7 + (–21)2 – 94 = (?)2
(a) 15 (b) 16 Company ®
(c) 18 (d) 19 116. What is the average number of mobile phones sold by all
(e) 17 companies taken together in a month ?
110. 34% of 576 + 18% of 842 = ?% of 400 + 83.4 (a) 18 thousands (b) 20 thousands
(a) 75 (b) 72 (c) 17 thousands (d) 19 thousands
(c) 62 (d) 65 (e) None of these
(e) 66 117. By what percent the number of mobile phones sold by
company U is more than that of company T ?
29241 2 1
111. ´5 =? (a) 33 % (b) 22%
361 9 3
(a) 47 (b) 49 2
(c) 46 (d) 45 (c) 20% (d) 23 %
3
(e) 61 (e) None of these
y
o
u
rs
m
Indian Overseas Bank PO Online Exam 2013 171

a
h
b
118. What is the average of the number of laptops sold by (a) ` 5 lakh (b) ` 10 lakh

o
companies P, R and T ? (c) ` 25 lakh (d) ` 50 lakh

o
b
(a) 17 thousands (b) 17.3 thousands (e) ` 1 crore

.w
(c) 18 thousands (d) 16 thousands 128. Public Provident Fund in India is open for a period of

o
(e) None of these

rd
(a) 1 year (b) 5 years
119. What is the respective ratio between the number of mobile

p
(c) 10 years (d) 15 years

re
phones sold by company T and that of laptops sold by (e) 20 years

s
company Q ?

s
129. Which of the following bank customer relationship does

.c
(a) 3 : 5 (b) 6 : 5

o
not come under the Code of Fair Banking Practice?

m
(c) 5 : 3 (d) 5 : 6 (a) To describe standards of good practice and service
(e) None of these (b) To force the customers comply with the Know Your
120. What is the respective ratio of the numbers of laptops sold Customer (KYC) norms
by company Q and company R?
(c) To promote disclosure of information relevant and
(a) 2 : 5 (b) 4 : 3
useful to Customers
(c) 3 : 4 (d) 3 : 2
(d) To promote informed and effective relationships
(e) 2 : 3
between Banks and Customers
GENERAL AWARENESS (e) To require Banks to have procedures for resolution of
disputes between Banks and Customers.
121. Savings Bank interest is calculated on the basis of 130. Which among the following constitutes the largest
(a) Minimum available balance percentage of retail loans in India?
(b) Bank’s liquidity (a) Personal Loans (b) Auto Loans
(c) Profitability preferences of a bank (c) Home Loans (d) Consumer Loans
(d) Daily balance method (e) Education Loans
(e) None of these 131. The head of Hindu Undivided Family is called
122. A customer deposits a sealed box in a bank in Safe Custody.
(a) Karta (b) Dharta
What will be the relation between the customer and the
(c) Co-parcener (d) Mukhya
bank in this case?
(e) Grihastha
(a) Creditor-Debtor (b) Bailor-Bailee
132. What is the Reverse Repo Rate at present?
(c) Trustee-Beneficiary (d) Principal-Agent
(e) Licensor-Licensee (a) 7.50 per cent (b) 7 per cent
123. Which term is used for an account which has been (c) 6.5 percent (d) 6.25 per cent
inoperative since 2 years due to change of residence of the (e) 6 per cent
customer? 133. Which among the following is not one of the main purposes
(a) Dormant account (b) Inactive account behind the start of Basic Saving Account by banks in India?
(c) Static account (d) Unclaimed account (a) To provide normal banking service available to all
(e) Anonymous account (b) To provide saving account facility without requirement
124. Which policy of the Reserve Bank of India does not deal in of any minimum balance
fixing Interest rates? (c) To promote ‘no-frills’ account
(a) Monetary Policy (b) Expansionary policy (d) To provide facility of ATM card or ATM-cum-Debit
(c) Contractionary policy (d) Fiscal Policy Card
(e) Credit Policy (e) None of these
125. Which of the following bodies/ organizations is responsible 134. Under the Prevention of Money laundering Act, every
for the inspection, supervision and surveillance of Regional banking company is needed to furnish details of cash
Rural Banks (RRBs)? transactions of the value of more than or its equivalent in
(a) NABARD (b) SEBI foreign currency.
(c) RBI (d) IRDA (a) ` 1 lakh (b) ` 5 lakh
(e) Union Ministry of Rural Development (c) ` 10 lakh (d) ` 25 lakh
126. The “Three Pillar Approach” of Risk Management does not (e) ` 50 lakh
include 135. At present, Demand Drafts in India are valid for a period of
(a) Minimum capital requirements (a) 1 month (b) 2 months
(b) Supervisory review (c) 3 months (d) 6 months
(c) Market discipline (e) 1 year
(d) Thrift 136. As per the guidelines issued by the Reserve Bank of India
(e) All of the above in May 2013, debt service coverage ratio should be greater
127. According to the Micro, Small and Medium Enterprises than ____ within the 5 years period.
Development (MSMED) Act, 2006 micro enterprise is an (a) 0 per cent (b) 0.50 per cent
enterprise where investment in plant and machinery does (c) 1 per cent (d) 1.25 per cent
not exceed (e) 1.50 per cent
y
o
u
rs
m
172 Indian Overseas Bank PO Online Exam 2013

a
h
b
137. Which of the following is not correct regarding the Cheque (a) Pound Sterling (b) Euro

o
Truncation System (CTS) in India? (c) Franc (d) Krona

o
b
(a) It speeds up the process of collection of cheques (e) Forint

.w
(b) It reduces the scope for cheque clearing-related frauds 145. Under which Act can criminal prosecution of the defaulter

o
(c) Truncation of cheques is yet to be legalized

rd
be initiated for dishonoring of an electronic fund transfer
(d) It lowers the cost of collection of cheques

p
instruction due to insufficiency of funds in the account?

re
(e) It removes reconciliation related and logistics-related (a) Negotiable Instruments Act, 1881

s
s
problems (b) Indian Penal Code, 1860

.c
138. What is the liability of the Banker in Case of Wrongful (c) Payment and Settlement Systems Act, 2007

o
m
Dishonor of Cheques? (d) Both 1 and 2
(a) He is liable to compensate the customer for the loss (e) Both 2 and 3
suffered by him 146. Which of the following statements correctly describe Brown
(b) He is not liable to compensate the customer for the Label ATMs in India?
loss (a) They are those ATMs where hardware and the lease
(c) He can transfer the matter to the Customer Grievance of the ATM machine is owned by a service provider
Redressal Department (b) They are those ATMs where cash management and
(d) Both the customer and the banker have to split the connectivity to banking networks is provided by a
compensation in the ratio of 50:50 sponsor bank
(e) None of these (c) They are alternative between bank-owned ATMs and
139. In the context of financial inclusion, what does USB refer ‘white label’ ATMs
to? (d) These ATMs are marked by an incentive to ensure
(a) Ultra Small Banking efficiency in terms of usage
(b) Unified Services Branch (e) All of the above
(c) Ultra Simplified Banking 147. A bank customer (a company) has a cash credit limit of ` 1,
(d) Union Saving Banking 00,000. After making a payment on a scheduled date, its
(e) Ultra Small Branch credit balance gets reduced to ` 80,205. In this context,
140. In the field of banking sector, what is the full form of ECS? under which facility can one entity / company make payments
(a) E-Commerce Service from its bank account to a number of recipients by direct
(b) Entity component system credit to their bank account?
(c) Embodied cognitive science (a) NEFT
(d) Electronic Clearing Service (b) RTGS
(e) Extended Core Storage (c) ECS
141. If a person had income from salary, pension and interest
(d) Standing Instructions Payment
income, then which Income Tax Form will be filled by him?
(e) Only 1 and 2
(a) ITR-l (b) ITR-2
148. A letter of credit where in the credit available to the customer
(c) ITR-4 (d) ITR-8
gets reinstated after the bill is paid is known as
(e) ITR-12
(a) Back to back LC (b) Red clause LC
142. Which of the following is not correct regarding Financial
(c) Back to front LC (d) Revolving LC
Inclusion?
(e) Recourse LC
(a) It is the delivery of financial services at affordable costs
(b) It is targeted at sections of disadvantaged and low- 149. Banks are required to maintain secrecy and confidentiality
income segments of society of customers’ accounts under
(c) It promotes financial and institutional sustainability (a) Indian Contract Act 1872
(d) The Reserve Bank of India (RBI) set up the Jalan (b) Banker’s Book Evidence 1891
Commission in 2004 to look into financial inclusion (c) Consumer Protection Act 1986
(e) Opening of no-frills accounts is a step towards financial (d) Banking Regulation Act 1949
inclusion (e) Income Tax Act 1961
143. Which of the following is a fully-owned subsidiary of the 150. Which of the following authorities instructs a bank that
Reserve Bank of India? funds held on behalf of a debt or should not be released
(a) Reserve Bank of India Staff College until directed by it?
(b) Institute for Development and Research in Banking (a) Reserve Bank of India
Technology (b) Banking Ombudsman
(c) Indira Gandhi Institute for Development Research (c) Court
(d) Deposit Insurance and Credit Guarantee Corporation (d) Union Ministry of Finance
of India (e) Cabinet Committee on Economic Affairs
(e) National Institute for Bank Management 151. Which Act has empowered the Reserve Bank of India to
144. Which of the following is the currency of the European conduct the inspection of banks and regulate them in the
Union? interest of banking system?
y
o
u
rs
m
Indian Overseas Bank PO Online Exam 2013 173

a
h
b
(a) Reserve Bank of India Act, 1934
COMPUTER KNOWLEDGE

o
(b) Banking Regulation Act, 1949

o
b
(c) Negotiable Instruments Act, 1881

.w
161. Full form of FTP is
(d) Banking Supervision Act, 1956 (a) File Transfer Protocol

o
(e) Payment and Settlement Systems Act, 2007

rd
(b) File Transit Protocol

p
152. Major portion of share capital in the case of Regional Rural
(c) Folder Transfer Protocol

re
Banks in India is held by

s
(d) File Transfer Procedure

s
(a) Indian Banking Association

.c
(e) None of these
(b) Central Government

o
162. Which area of microprocessor is used to temporarily store

m
(c) Reserve Bank of India
instructions and data ?
(d) State government
(a) Register (b) ALU
(e) Sponsor Bank
(c) Accumulator (d) Cache memory
153. Which of the following statements sum up the features of a
(e) Interrupt control
Debit Card?
(a) It is a plastic payment card 163. ____ are usually real technology buffs which modify
computer hardware or software in a way that alters the
(b) Payments using a debit card are immediately
creator’s original content.
transferred from the cardholder’s designated bank
account (a) Crackers (b) Hackers
(c) It is known as a bank card or check card (c) Virus (d) Software engineer
(d) The debit card transactions are routed through the (e) None of these
Visa or Master Card networks 164. In computing ____ is a term that means replacing of
(e) All of the above hardware, software or firmware with a new or better version,
154. What does the letter ‘S’ stands for in BRICS? in order to bring the system up to date or to improve its
(a) Sustainable (b) Sweden characteristics.
(c) Service (d) South Africa (a) Upgrading (b) Installing
(e) Sales (c) New version (d) Replacement
155. Which of the following bodies is responsible for maintaining (e) None of these
the Cash Reserve Ratio? 165. What is the blinking symbol on computer screen ?
(a) All scheduled commercial banks (a) Mouse (b) Cursor
(b) All Private Banks (c) Cathode Ray tube (d) Logo
(c) Regional Rural Banks (e) Palm cursor
(d) Non Banking Financial Companies 166. Which hardware device enables a computer to send and
(e) All of the above receive information over telephone lines by converting the
156. Foreign Exchange Reserves in India are maintained and digital data used by computer into analog signal used on
managed by phone lines and then converting it back once received on
(a) SEBI (b) RBI the other end.
(c) IRDA (d) Planning Commission (a) Demodulator (b) Modulator
(e) Union Ministry of Commerce (c) Modem (d) Ethernet
157. Which of the following is not a Credit rating agency of (e) All of the above
India? 167. ____ are descended from early text formatting tools, it is
(a) CIBIL (b) ICRA Limited the earliest applications for the personal computer in office
(c) Fitch (d) Onicra productivity.
(e) Experian (a) Word processor (b) Diskettes
158. With which institution is the Trans Union Score associated? (c) Printer (d) Plotter
(a) Educational institutions (b) Credit Institutions (e) None of these
(c) Legal Institutions (d) Forex Institutions 168. ____ is a concept to abstractly represent all instances of a
(e) None of these group of similar things.
159. A cheque which bears the word “Not Negotiable” between (a) Revoke
the two parallel lines is considered as a (b) Data Manipulation language
(a) Uncrossed cheque (b) Truncated cheque (c) Data Definition language
(c) Valid cheque (d) Crossed cheque (d) Data Control language
(e) Post-dated cheque (e) Entity
160. To which of the following clients of the Reserve Bank of 169. How many Gigabytes is equal to 1 petabyte?
India does its rates (Bank Rate and Repo Rate) apply? (a) 104857 (b) 1048576
(a) Union Government (b) State Government (c) 1 (d) 10485
(c) NABARD (d) NBFCs (e) None of these
(e) All of the above
y
o
u
rs
m
174 Indian Overseas Bank PO Online Exam 2013

a
h
b
170. .xls, .doc, .ppt are extensions of ____ respectively. 181. A (n) ____ allows you to access your e-mail from any where

o
(a) Word document, Presentation, Excel (a) Forum (b) Webmail interface

o
b
(b) Presentation, Excel, Word document (c) Message Board (d) Weblog

.w
(c) Excel, Word document, Presentation (e) None of these

o
(d) Excel, Presentation, Word document

rd
182. When you enter text in a cell in Excel, it also appears in the
(e) None of these

p
(a) status bar (b) formula bar

re
171. PIN stands for ____ (c) row heading (d) name box

s
(a) Password Indentification Number

s
(e) None of these

.c
(b) Public Indentification Number

o
183. Where is newly received email stored ?

m
(c) Private Indentification Number
(a) Your website (b) Address-box
(d) Public Indentification Number
(c) Inbox (d) Your personal laptop
(e) None of the above
(e) None of these
172 ____ refers to the copying and archiving of computer data,
184. When the mouse is moved, it causes a picture to move on
where primaiy purpose is to recover data after its loss (due
the screen which is referred as a
to deletion or corruption).
(a) Reverse data (b) Extraction of data (a) menu (b) icon
(c) Back up of data (d) Version change (c) pointer (d) tab
(e) All of the above (e) None of these
173. An email attachment is a computer file, documents and 185. To save a document for first time, ____ option is used.
images sent along with an _______. (a) Save as (b) Save first
(a) email message (b) email extension (c) Save on (d) Copy
(c) Inbox (d) Insert file (e) Paste
(e) All of the above 186. Two basic types of graphics used in Word 2000 are____.
174. TSO stands for (a) Autoshapes and Clip Art
(a) Time sharing operation (b) Header and Footer
(b) Time sharing option (c) Drawing Objects and Pictures
(c) Time support option (d) Spelling and Grammar
(d) Time synchronous option (e) Word Count
(e) None of these 187. Which of the following terms is not related to Internet ?
175. Which of the following is known as Network of (a) Link (b) Function key
Networks? (c) Browser (d) Search Engine
(a) LAN (b) WAN (e) Hyperlink
(c) MAN (d) Internet 188. To move the text form its original position to another
(e) None of these position without deleting it is called –
176. What refers to how system can quickly transform to support
(a) Scrolling (b) Searching
environmental changes?
(c) Moving (d) Copying
(a) Reliability (b) Scalability
(e) Halting
(c) Availability (d) Maintainability
189. Which of the following is an active cell in Excel?
(e) None of these
177. Booting up of PC depends on factors like so that files (a) Recycle Bin (b) Task Bar
function without erros. (c) Tool Bar (d) My Computer
(a) the hardware (b) BIOS (e) None of these
(c) Opertaing system (d) All of the above 190. All the deleted files go to .....
(e) None of these (a) Recycle Bin (b) Task Bar
178. Which amongst the following is/are pointing devices? (c) Tool Bar (d) My Computer
(a) Mouse (b) Trackball (e) None of these
(c) Joystick (d) Stylus 191. You organize files by storing them in .....
(e) All of the above (a) archives (b) folders
179. What feature adjusts the top and bottom margins so that (c) indexes (d) lists
the text is centred vertically on the printed page? (e) None of these
(a) Vertical justifying (b) Vertical adjusting 192. Letters, numbers, and symbols found on a keyboard are
(c) Dual centring (d) Horizontal centring (a) Icon (b) Screen
(e) Vertical centring (c) Keys (d) Menu
180. What is the overall term for creating, editing, formatting, (e) None of these
storing, retrieving, and printing a text document ? 193. ____ is the process of carrying out commands.
(a) Word processing (b) Spreadsheet design (a) Fetching (b) Storing
(c) Web design (d) Database management (c) Decoding (d) Executing
(e) Presentation generation
(e) None of these
y
o
u
rs
m
Indian Overseas Bank PO Online Exam 2013 175

a
h
b
194. The microprocessor contains a special purpose storage 197. State what is true about XML:

o
areas called (a) XML is a markup language very different from HTML

o
b
(a) Cache (b) RAM (b) XML is not a W3C Recommendation

.w
(c) ROM (d) Internal memory (c) XML does not do anything

o
(e) Registers

rd
(d) XML stands for Extra Markup Language
195. Six aspects of e-mail system in the right order are

p
(e) XML tags are predefined

re
(a) Composition, transfer, conversion, reporting, 198 _____ be used to insert a page break in word at the current

s
formatting, disposition

s
position

.c
(b) Composition, conversion, reporting, transfer, (a) Enter (b) Shift + Enter

o
m
formatting, disposition (c) CTRL + Break (d) CTRL + Enter
(c) Composition, tr ansfer, reporting, conversion, (e) CTRL + B
formatting, disposition 199. System proposal is prepared in _____ phase of SDLC
(d) Composition, formatting, transfer, conversion, (a) Conception (b) Initiation
reporting, disposition (c) Analysis (d) Design
(e) Composition, transfer, conversion, reporting, (e) Construction
disposition, formatting 200. _____ menu provides an option to check spellings
196. COMMIT, ROLLBACK statements are parts of (a) View (b) Edit
(a) TCL (b) DCL (c) Format (d) Tools
(c) DDL (d) DML
(e) Review
(e) DQL
y
o
u
rs
m
176 Indian Overseas Bank PO Online Exam 2013

a
h
b
o
o
b
.w
o
rd
(1-3): 11. (a) logical Þ ri

p
Q>P>U idea Þ jo

re
P>T>R

s
only Þ na

s
S>Q

.c
Now, order Þ ve

o
m
S Q P T R U The code for ‘is’ may be ‘ge’
12. (d) logical Þ ri
64 kg 52 kg 13. (b) serial Þ to
1. P
(c) > T > R 14. (d) Both of the course of action are not suitable because
¯ ¯ the company lixus does not a manufacture ball point
64 kg 52 kg pens it would be an opportunity to some other
Therefore, T may be of 58 kg. company. Similarly, close competitor would introduce
2. (e) Four persons S, Q, P and T are heavier than R. gel-into pens in the market.
3. (b) Q is heavier than P and P’s weight is 64 kg. Therefore 15. (e) Both the course of action should be followed
Q’s weight may be 66 kg. because recession in whole world would affect the
(4–8) : job market of foreign countries also adversely.
Facing Row - 1 16. (d) Both of the course of action are not feasible because
LEFT
RIGHT

South private curlines; if banned, would create further


problems and government curlines cannot increase
E D C A B F passengert fare beyond certain limits.
RIGHT

Facing Q S P R V T 17. (a) II course of action is not feasible because


LEFT

North government cannot limit school fees.


18. (b) Only II course of action should be followed because
Row - 2
appropriate arrangements can only control spread of
4. (b) A faces R. D sits second to the right of A. disease.
5. (a) B sits second to the left of C.
A sits to the immediate right of B. 19. (c) A @ F Þ A is wife of F.
V faces B. F $ M Þ F is son of M.
A and F are immediate neighbours of B. M % J Þ M is mother of J.
V faces B. R and T are immediate neighbours of V.
J + T Þ J is the sister of T.
6. (c) C faces P.
7. (a) V sits exactly between T and R. M(–)
8. (d) Except C, all others are seated at extreme ends of the
lines. A F J T
(9 – 13) : (–) (–) (–)

only in serial order ® ve pu na to Sister-in-Law


order in the state ® li ve su pu 20. (d) J % B Þ J is the mother of B
B $ K Þ B is the son of K.
the logical idea only ® su na ri jo K $ T Þ K is the son of T.
in idea or theory ® zt jo bk pu Therefore J is daughter-in-law of T.
(21–22) :
Codes are : R³T<M=Z
only Þ na the Þ su or Þ zt or bk C>T³B
in Þ pu state Þ li theory Þ zt or bk
R³T<C
serial Þ to logical Þ ri
R³T³B
order Þ ve idea Þ jo
Z=M>T³B
9. (e) The code of ‘theory’ is either ‘zt’ or ‘bk’,
C>T<M=Z
10. (c) li Þ state 21. (b) Conclusions :
ri Þ logical I. Z > C : Not True
to Þ serial II. B < Z : True
ve Þ order So only conclusions II follows.
y
o
u
rs
m
Indian Overseas Bank PO Online Exam 2013 177

a
h
Erasers

b
22. (a) Conclusions :
Pens

o
I. B < C : True Pencils

o
b
II. R ³ C : Not True

.w
So only conclusions I follows.

o
(23–24):

rd
P³Q>T

p
re
Q£R Sharpeners

s
T³A

s
Conclusions:

.c
P³Q>T³A

o
I. False

m
P³Q £R II. False
R³Q>T³A
So neither conclusion I nor conclusion II follow.
23. (a) Conclusions :
I. A < Q : True
(29–30)

Planet
II. A £ P : Not True Stars Moons
So only conclusions I follows.
24. (e) Conclusions :
I. T < P : True Sun
II. R > A : True
So both the conclusions follow.
OR
25. (d) P > T = U £ W
Q³R³W

Planet
P>T=U£W£R£Q Stars Moons Sun
Conclusions :
I. T = R : Not True 29. (d) Conclusions:
II. U < R : Not True
I. False
So None of the conclusion follows.
II. False
(26–27): None of the conclusions follows.
Animals
30. (b) Conclusions:
Mammals
Bulls I. False
II. True
Cow
(31–35):
OR White
Purple B Orange
Animals C H

Bulls Mammals Blue D G Yellow


w

F Black
Co

E
26. (e) Conclusions: Red A
Green
I. True 31. (c) There are four persons G, F, A and E between H and
Conversion of the third premise. D,
II. True if we go clockwise from H to D.
So both of the conclusions I and II follow.
32. (a) H and F are immediate neighbours of G.
27. (a) Conclusions:
F likes black colour.
I. True
33. (d) H likes orange colour. E is fourth to the left or right
Conversion of the second premise. of H.
II. False
E
28. (d) Pencils Pens 34. (b) D B F
Erasers C H
Sharpeners
CD GG

E F H
OR
B A
A
y
o
u
rs
m
178 Indian Overseas Bank PO Online Exam 2013

a
h
b
35. (e) E likes red colour. right flights including two international flights will

o
A sits exactly between E and F. be a correct usage.

o
b
(36–40) : Look at the sentence :

.w
If we analyse the given input and various steps of rearrangement Ten people were killed in the riot, including a

o
rd
we would find that in each step there is an arrangement of one policeman.

p
word and one number. In the first step, word which comes first 52. (c) Here, adverbial form of ‘simple’ should be used.

re
in alphabetical order and the highest number moves to the Hence, it had chopped down a full grown tree simply

s
s
extreme left position in the second step, there is similar because .... should be used.

.c
rearrangement of words and numbers in the same pattern. Same

o
53. (c) Here, Simple Past (V2) i.e., the information they

m
procedure is followed til all the words are arranged in the gathered .... should be used,
reverse alphabetical order from left to right and all the numbers 54. (d) Here, being apprehended across the border ....
are arranged in ascending order.
should be used.
Input : money 48 24 18 wanted for investment 65 90 lock
55. (b) Here, did the manager realize that/the manager
credit 32
realized that (Past Simple) should be used.
Step I: credit 90 money 48 24 18 wanted for investment 65
lock 32 56. (b) Allure means to attract, charm or tempt . Lucrative
Step II: for 65 credit 90 money 48 24 18 wanted investment means producing a great deal of profit.
lock 32 57. (b) Call off means temporraily suspend.
Step III: investment 48 for 65 credit 90 money 24 18 wanted 58. (d) 59. (b) 60. (b) 61. (b)
lock 32 62. (a) 63. (c) 64. (b) 65. (e)
Step IV: lock 32 investment 48 for 65 credit 90 money 24 18 66. (a) 67. (d) 68. (c) 69. (e)
wanted 70. (e) 71. (d) 72. (e) 73. (d)
Step V : money 24 lock 32 investment 48 for 65 credit 90 18 74. (a) 75. (d)
wanted 76. (b) Here, had once tried to …… should be used.
Step VI: wanted 18 money 24 lock 32 investment 48 for 65 77. (e) 78. (c)
credit 90 79. (a) Here, infinitive, i.e., to pacity him …… should be used.
And, Step VI is the last step for the given input. 80. (b) The Principal clause is in Past tense. Hence. help
36. (c) This is step IV. their children needed ... should be used
37. (b) Fourth from left in Step IV Þ 48 Look at the sentence :
38. (a) Fifth element from the left in the Step V Þ I gave him the money he needed.
investment.
¯ ¯
Second to the right of ‘investment’ Þ for Simple Past Simple Past
39. (e) There are five elements (65, credit, 90, money, 24) I will give him the money he needs.
between ‘for’ and ‘18’ in the Step III.
¯ ¯
40. (d) Fourth from the right.
Simple Future Simple Present
41. (e)
81. (a) The word SACRED consists of 4 consonants
42. (c) This is the answer to the first querstion.
(SCRD) and two vowels (AE). On keeping vowels
43. (a) This can be inferred from the last paragrpah of the
together we get SCRD (AE).
answer to the second question. It can alos be
\ Number of arrangements = 5! × 2!
inferred from the answer to the third question.
44. (e) = 5 × 4 × 3 × 2 × 1× 1× 2 = 240
45. (d) It is mentioned in the answer to the second qusetion 82. (b) Smallest number divisible by 7 = 140
where social forestry has been criticised and Largest number = 462
community forestry has been offered as solution. By an = a + (n –1)d
46. (a) It is mentioned in the answer to the third question. 462 = 140 + ( n – 1) × 7
47. (d) It is mentioned in the second last paragrpah. Þ (n – 1) × 7 = 462 – 140 = 322
48. (a) The word Control (Verb) means: to have power over Þ n – 1 = 322 ÷ 7 = 46
a person etc. to limit something: to manage to make Þ n = 46 + 1 = 47
yourself remain calm; to stop something from getting 83. (c) Interest = 3456 – 3200 = ` 256
worst or spreading. S.I. ´100
49. (c) The word Paradox (Noun) means : a person, thing or \ Rate = Principal ´ Time
situation that has two opposite features and therefore
256 ´ 100
seems strange; a statement containing two opposite = = 4% per annum
ideas. 3200 ´ 2
84. (d) If the marked price of the article be ` x then
50. (b) The word Acute (Adjective) means : very serious or
Price after discount of 15%
severe.
51. (c) Here, preposition including should be used. Hence, 85 x 17 x
=` =`
100 20
y
o
u
rs
m
Indian Overseas Bank PO Online Exam 2013 179

a
h
b
æ 17 x 106 ö 12 x + 12 y = 60

o
Actual S.P. = ` ç ´ ÷

o
è 20 100 ø 12 x - 12 y = 36

b
.w
17 x 106 – + –
\ ´ = 1081.20

o
20 100 24 y = 24

rd
1081.20 ´ 20 ´ 100

p
y = 1 km/h
Þ x= = ` 1200

re
17 ´ 106 The speed of current is 1 kmph.

s
s
85. (a) Identifying the pattern of number series 92. (b) Let R1 and R2 be the two different rate of interest,

.c
where R1 > R2

o
×0.5 ×1 ×1.5 ×2.0
21 10.5 10.5 15.75 31.50

m
×2.5 2200 ´ R1 ´ 4 2200 ´ R2 ´ 4
– = 202.40
78.75 100 100
86. (b) Identifying the pattern of number series 2200 ´ 4
[R1– R2] = 202.40
+(1×13) +(3×13) +(5×13) +(7×13) 100
6 19 58 123 214
202.40 ´ 100 50.6
+(9×13) R1– R2 = =
2200 ´ 4 22
331 R1– R2 = 2.3%
87. (c) Identifying the pattern of number series 93. (c) Clearly, the percentage of students in school C in
+(1×2) +(3×4) +(5×6) pichart is highest i.e. 24%.
14 16 28 58 94. (c) Percentage of students in schools A and C
+(7×8) = 24 + 11= 35%
+(9×10) Percentage of students in schools E and F
204 114
= 23 + 12 = 35%
88. (d) Identifying the pattern of number series 95. (d) Number of students in schools B, D and E
+(1×1.15) +(2×1.15) +(3×1.15)
13.76 14.91 17.21 20.66 (14 + 16 + 12)
= 75000 ×
+(4×1.15) 100
+(5×1.15)
31.01 25.26 42
= 75000 × = 31500
89. (e) The pattern of the number series is : 100
15 + 12 = 16 31500
\ Required average = = 10500
16 + 23 = 16 + 8 = 24 3
96. (e) Required ratio = 14 : 12 = 7 : 6
24 + 32 = 24 + 9 = 33
33 + 43 = 33 + 64 = 97 24 - 16
97. (b) Percentage decrease = ´ 100
97 + 52 = 97 + 25 = 122 24
90. (a) We have, 100 1
= = 33 %
x = y + 52 3 3
y = z + 26 or z = y – 26 98. (c) Total marks obtained by all students in Algebra
and x + y + z = 221 = 54 + 54 + 43 + 81 + 62 + 42 = 336
(y + 52) + y + (y – 26) = 221 336
3y + 26 = 221 Required average = = 56
6
3y = 221 – 26 = 195 99. (d) Total marks obtained by Sunil = 538
195 Total marks obtained by Shweta = 576
y=m = 65 Required difference = 576 – 538 = 38
3
x = y + 52 = 65 + 52 = 117 100. (a) Total marks obtained by Sunil = 538
z = y – 26 = 65 – 26 = 39 Total marks obtained by Meeta = 375
x : y : z = 117 : 65 : 39 = 9 : 5 : 3 163
Required percentage = ´ 100 = 43%
91. (b) Let speed of boat in steady water and speed of 375
current be x km/h and y km/h respectively. 101. (b) Total marks obtained by Farhan = 456
Speed of upstream = (x – y) km/h 456
Speed of down stream = (x + y) km/h Required percentage = ´ 100 = 57
800
20 102. (c) Total marks obtained by Davis = 444
x+ y
= 4 or 4x + 4y = 20 … (1)
444
Required average = = 55.5
9 8
= 3 or 3x – 3y = 9 … (2) 103. (a) ? » 395 + 187 = 582
x- y
Multiplying equation (1) by 3 and equation (2) by 4, 104. (b) ? = 3 3380 + 1300 » 3 3375 + 1296
then subtract equation (2) from (1). » 15 + 36 » 51
y
o
u
rs
m
180 Indian Overseas Bank PO Online Exam 2013

a
h
b
105. (c) ? » (5)2 + (21)3 + 1089 22

o
= (56 + 49) (56 – 49)

o
» 25 + 9261 + 33 » 9319 7

b
= 22 × 105 = 2310 sq.m.

.w
7020 13 2340 ´ 13
106. (d) ? » ´ » = 1049 114. (b) Let positive consecutive integers are

o
3 29 29

rd
x, x + 1, x + 2 ..., x + 8.
5000 ´ 25 3000 ´ 65

p
107. (e) ? » - x + ( x + 1) + .... + ( x + 8)

re
100 100 Average = = 63

s
9

s
» 1250 – 1950 » – 700

.c
9 x + 36
108. (c) ? = (81)-1/2 - (64) -2/3

o
Þ = 63

m
9
1 2 Þ x + 4 = 63 Þ x = 59
æ 1 ö2 æ 1 ö3 1 1 Largest number, x + 8 = 59 + 8 = 67
= ç ÷ -ç ÷ = -
è8ø è 64 ø 9 16 Product of largest and smallest = 59 × 67 = 3953
16 - 9 7 115. (a) Let C.P. of milk per litre be ` 1
= =
144 144 5
109. (d) (?)2 = 331.8 ÷ 23.7 + (–21)2 – 94 Milk in 1 litre of A = litre
7
= 14 + 441 – 94 = 361 8
Þ ?= Milk in 1 litre of B = litre
361 = 19 13
576 ´ 34 842 ´ 18 400 ´ ? 9
110. (e) + = + 83.4 Milk in 1 litre of mixture = litre
100 100 100 13
Þ 195.84 + 151.56 = 4 × ? + 83.4 C.P. of 1 C.P. of 2
Þ 347.4 = 4 × ? + 83.4 litre of A litre of B
Þ 4 × ? = 347.4 – 83 .4 = 264 5 Average 8
`— price `—
264 7 13
Þ ?= = 66 9
4 `—
13
29241 47 171 47 1 2
´ ´ — —
111. (a) ?= = = 47 13 91
361 9 19 9
1 2
13 44 3 \ Required ratio = : =7:2
112. (b) + + ? = 13 + 13 91
4 7 28
17 + 22 + 33 + 13 + 15 + 20
91 + 176 3 116. (b) Required average =
Þ + ? = 13 + 6
28 28
120
267 3 = = 20 thousand
Þ + ? = 13 + 6
28 28
20 - 15 100 1
264 66 117. (a) Required per cent = ´ 100 = = 33 %
Þ + ? = 13 Þ + ? = 13 15 3 3
28 7
13 + 27 + 12
66 91 - 66 25 4 118. (b) Required average =
Þ ? = 13 – = = =3 3
7 7 7 7
113. (d) 2pr1= 308 where r1 = radius of circular playground 52 1
= = 17 thousands
3 3
r2 119. (d) Required ratio = 15 :18 = 5 : 6
120. (e) Required ratio = 18 : 27 = 2 : 3
121. (d) 122. (b) 123. (d) 124. (e) 125. (a)
126. (d) 127. (c) 128. (d) 129. (b) 130. (c)
r1 131. (a) 132. (d) 133. (c) 134. (c) 135. (c)
136. (d) 137. (c) 138. (a) 139. (e) 140. (d)
141. (a) 142. (d) 143. (d) 144. (b) 145. (c)
146. (e) 147. (c) 148. (d) 149. (c) 150. (c)
151. (b) 152. (b) 153. (e) 154. (d) 155. (a)
22
Þ 2× × r1 = 308 156. (b) 157. (c) 158. (b) 159. (d) 160. (e)
7 161. (a) 162. (d) 163. (b) 164. (a) 165. (b)
308 ´ 7 166. (c) 167. (c) 168. (b) 169. (b) 170. (c)
Þ r1 = = 49 metre 171. (e) 172. (c) 173. (a) 174. (b) 175. (d)
2 ´ 22
176. (d) 177. (d) 178. (e) 179. (e) 180. (a)
2
(
\ Area of the path = p r2 - r1
2
) 181. (b) 182. (a) 183. (c) 184. (c) 185. (e)
186. (a) 187. (b) 188. (d) 189. (a) 190. (a)
22 191. (a) 192. (c) 193. (d) 194. (e) 195. (c)
= (562 – 492)
7 196. (a) 197. (c) 198. (d) 199. (b) 200. (d)
y
o
u
rs
m
a
h
b
o
o
b
IBPS RRBS OFFICER SCALE-I EXAM 2013

.w
o
rd
p
Based on Memory

re
s
s
.c
REASONING ABILITY Ikon, Scorpio and Esteem, not necessarily in that order. None of

o
m
the ladies is a Consultant or a Lawyer. T is an Air hostess and she
DIRECTIONS (Qs. 1–5) : In the following questions, the symbols owns an Ikon car. R owns a Scorpio. M is not a Doctor. L is a
D, S @, © and # are used with the following meaning as Jeweller and he owns Corolla. W is a Lawyer and does not own
illustrated below : Alto. Z is a Consultant and owns Santro. The Doctor owns Esteem
car whereas the Professor owns Scorpio. The Travel Agent owns
‘X D Y’ means ‘X’ is neither greater than nor equal to Y. an Alto. None of the ladies own a Scorpio.
‘X S Y means ‘X is not smaller than Y. 6. What car does S own ?
‘X @ Y means ‘X is not greater than Y’. (a) Alto (b) Santro
‘X © Y’ means ‘X is neither greater than nor smaller than Y’.
(c) Lancer (d) Esteem
‘X # Y’ means ‘X is neither smaller than nor equal to Y.
(e) None of these
Now in each of the following questions assuming the given
7. Who owns the car Lancer ?
statements to be true, find which of the two Conclusions I and II
(a) Z (b) M
given below them is/are definitely true ?
(c) W (d) Data inadequate
Give answer (a) If only Conclusion I is true.
(e) None of these
Give answer (b) if only Conclusion II is true.
Give answer (c) if either Conclusion I or II is true. 8. What is the profession of R ?
Give answer (d) if neither Conclusion I nor II is true. (a) Professor (b) Travel Agent
Give answer (e) if both Conclusions I and II are true. (c) Doctor (d) Data inadequate
1. Statements : (e) None of these
R @ J, F S J, C S F 9. Who is the Doctor ?
Conclusions : (a) R (b) S
I. R © C (c) L (d) Data inadequate
II. C # R (e) None of these
2. Statements : 10. Who are the three ladies in the group ?
W @ P, W # E, D V (a) T, R, L (b) T, M, S
Conclusions : (c) W, T, M (d) Data inadequate
I. P # E (e) None of these
II. V © W DIRECTIONS (Qs. 11–15) : In each of the questions below are
3. Statements :
given three statements followed by three Conclusions numbered
J © R, P S R, Z # P
I, II and III. You have to take the given statements to be true even
Conclusions :
if they seem to be at variance from commonly known facts. Read
I. R D Z
all the Conclusions and then decide which of the given
II. J @ P
Conclusions logically follows from the given statements
4. Statements :
disregarding commonly known facts.
G @ O, N © O, H # G
Conclusions : 11. Statements :
I. O D H All flowers are toys.
II. G © N Some toys are trees.
5. Statements : Some angels are trees.
Q D B, M © B, K S M Conclusions :
Conclusions : I. Some angels’ are toys.
I. K © B II. Some trees are flowers.
II. Q D K III. Some flowers are angels.
DIRECTIONS (6–10) : Study the following information carefully (a) Only I follows (b) Only II follows
and answer the questions given below : (c) Only I and III follow (d) Only III follows
(e) None follows
Three ladies and four men are a group of friends i.e. R, M, T, S, L, 12. Statements :
W and Z. Each one has a different profession i.e. Lawyer, Travel Some pigeons are dogs.
Agent, Air hostess, Doctor, Professor, Consultant and Jeweller All dogs are cats.
and each one owns a different car i.e. Alto, Corolla, Santro, Lancer, All pigeons are horses.
y
o
u
rs
m
182 IBPS RRBs Officer Scale-I Exam 2013

a
h
b
Conclusions : (a) Watermelon (b) Papaya

o
I. Some horses are dogs. (c) Jackfruit (d) Pineapple

o
b
II. Some cats are pigeons. (e) Sweetlime

.w
III. Some horses are cats. 19. How many such pairs of letters are there in the ‘ENIGMATIC’

o
(a) All follow (b) Only II and III follow each of which has as many letters between them in the

rd
word as in the English alphabet ?

p
(c) Only I and III follow (d) Only I and II follow

re
(e) None of these (a) One (b) Two

s
(c) Three (d) None

s
13. Statements :

.c
All tables are streets. (e) More than five

o
20. How many such 6s are there in the following number series,

m
All streets are bottles.
All parrots are bottles. each of which is immediately preceded by 1 or 5 and
immediately followed by 3 or 9 ?
Conclusions :
26 3756 4296 1341 6391569 2316 54
I. Some streets are parrots.
321967163
II. Some bottles are tables.
(a) None (b) One
III. All tables are bottles. (c) Two (d) Three
(a) Only I follows (b) Only II follows (e) None of these
(c) Only III follows (d) Only II and III follow
(e) None of these DIRECTIONS (21-22) : Read the following information and
14. Statements : answer the questions that follow:
Some cartoons are boys. (i) ‘S × T’ means ‘S is the brother of T’.
All boys are windows. (ii) ‘S ¸ T’ means ‘T is the father of S.’
All jokers are windows. (iii) ‘S + T’ means ‘S is the sister of T.’
Conclusions : (iv) ‘S – T’ means ‘S is the mother of T’.
I. All jokers are boys. 21. Which of the following means R is the paternal uncle of B?
II. Some cartoons are windows. (a) B × Q ¸ L × R
III. Some jokers are cartoons. (b) B × D ¸ J × R × K
(a) Only I follows (b) Only II follows (c) R × P ¸ S × B
(c) Only III follows (d) Only I and II follow (a) Only (a) (b) Only (b)
(e) None of these (c) Only (c) (d) Both (a) and (b)
15. Statements : (e) None of these
Some saints are balls. 22. Which of the following statement(s) is/are superfluous to
All balls are bats. answer the above question ?
(a) (i) only (b) (ii) only
Some tigers are balls.
(c) (iii) only (d) Only (iii) and (iv)
Conclusions :
(e) None of these
I. Some bats are tigers.
23. Four of the following five are alike in a certain way and so
II. Some saints are bats. form a group. Which is the one that does not belong to that
III. All bats are balls. group ?
(a) Only I and III follow (b) Only II follows (a) 17 (b) 13
(c) Only I and II follow (d) Only III follows (c) 63 (d) 71
(e) None of these (e) 23
16. In a certain code language RUSTICATE is written as 24. In a certain code language, SAMBLE is written as =/ © $ S
‘QTTUIDBSD’. How would STATISTIC be written in that % and STAR is written as = @ / £. How would TREMBLE
code ? be written in the same code ?
(a) RSBUIRSJD (b) TUBUITUMB (a) @ £ % © $ S % (b) @ £ % $ S / ©
(c) RSBUITUHB (d) RSBUJTUHB (c) @ £ % / $ S % (d) = £ % © $ S %
(e) None of these (e) None of these
17. If it is possible to make only one meaningful word with the 25. If ‘sky’ means ‘star’; ‘star’ means ‘cloud’; ‘cloud’ means
second, the fifth, the seventh and the tenth letters of the ‘earth’, ‘earth’ means ‘tree’ and ‘tree’ means ‘book’, then
word OMNISCIENT, which of the following will be the third where do the birds fly ?
letter of that word ? If no such word can be made, give ‘J’ as (a) sky (b) star
the answer and if more than one such word can be made, (c) cloud (d) Data inadequate
give ‘Q’ as the answer. (e) None of these
(a) J (b) Q
(c) I (d) M DIRECTIONS (26-30) : Study the following information carefully
(e) S and answer the questions given below :
18. Four of the following five are alike in a certain way and so An organisation wants to recruit Management Trainees.
form a group. Which is the one that does not belong to that Following are the criteria for such recruitment. The candidate
group ? must be –
y
o
u
rs
m
IBPS RRBs Officer Scale-I Exam 2013 183

a
h
b
(a) a graduate in any discipline with at least 65 per cent marks. 31. What is the code for ‘is’ in the code language ?

o
(b) above 24 years and not above 30 years of age as on I. In the code language ‘shi tu ke’ means ‘pen is blue’.

o
b
01.11.2012. II. In the same code language ‘ke si re’ means ‘this is

.w
(c) having a postgraduate degree/diploma in management or wonderful’.

o
completed ICWA/CA/CS with at least 55 per cent marks. 32. Which train did Harish catch to go to office ?

rd
(d) having an experience of 0–2 years. I. Harish missed his usual train of 10.25 a.m. A train comes

p
re
(e) ready to be on probation for one year. in every 5 minutes.

s
If a candidate satisfies all the criteria except — II. Harish did not catch the 10.40 a.m. train or any train

s
.c
(i) at (c) above but is an engineer with minimum 70 per after that time.

o
33. How is Vandana related to Prabha ?

m
cent marks his/her case is to be referred to AGM-PA.
(ii) at (d) above but has secured at least 70 per cent marks I. Mallika’s sister Vandana is Rajesh’s wife. Prabha is
in postgraduation his/her case is to be referred to the Rajesh’s good friend.
GM – PA. II. Prabha is Rajesh’s brother’s wife and Vandana is
Based on the above criteria and information provided below, make Rajesh’ wife.
a decision in each case. You are not to assume anything. If the 34. Who earns the highest among the five friends ?
data given are not enough to take a decision mark your answer as I. Priya earns more than Pinky and Sheetal and less than
‘data inadequate’. These cases are given to you as on 31.10.2012. only Shilpa.
Mark answer (1) if candidate is to be referred to AGM-PA. II. Sheetal earns more thar Neetu but less than Pinky.
Mark answer (2) if data given is not sufficient to take any decision. 35. Who is sitting to the immediate right of Tanvi among five
Mark answer (3) if candidate is to be selected friends sitting around a circle facing the centre ?
Mark answer (4) if candidate is to be referred to GM-PA. I. Ansh is sitting exactly between Bindu and Suresh and
Mark answer (5) if candidate is not to be selected. Neel is sitting to the immedi ate right of Suresh.
26. Rajesh Chopra is a Postgraduate in Management with 60 II. Tanvi is sitting exactly between Bindu and Neel and
per cent marks and has been working since June 2011. He Ansh is sitting to the immediate right of Bindu.
was bom on 13th September, 1988. He is ready to work on DIRECTIONS (36-40) : Study the following information
probation of one year. carefully and answer the questions given below :
27. Roop Kamal is an Arts graduate and C.A. both with 66 per
cent marks. He has been working from January 2012 and is Seven people N, K, T, B, M and R have their weekly offs on
ready to join on probation of one year. His date of birth is different days of the week i.e. Sunday, Monday, Tuesday,
3rd January, 1988. Wednesday Thursday, Friday and Saturday not necessarily in
that order. Each of them has a liking for different cuisine i.e. Indian,
28. Indira Narayan is a Commerce graduate and postgraduate
Italian, Mexican, Chinese, Spanish, Continental and Thai, not
in finance both with 68 per cent marks. She was bom on
necessarily in that order. K likes Thai food and gets his weekly
26th September, 1983. She has been working from April 2011.
off on Thursday. B likes Italian food and does not have off on
29. Darshan Patil is Commerce graduate and postgraduate in
Sunday. M has weekly off on Saturday and R has his weekly off
finance both with 60 per cent marks. He is 26 years old and
on Tuesday. W likes continental food whereas the one who has
has been working from November 2011. He is ready to join
weekly off on Monday likes Mexican cuisine. T does not like
on probation for one year.
Spanish cuisine and has weekly off on Wednesday. The one who
30. Vanitha Mittal is Commerce graduate with 68 per cent marks likes Indian food does not have a weekly off on Tuesday or
and she has completed her MBA with 70 per cent marks in Wednesday.
2011. Her date of birth is 30th August, 1988 and she is 36. Who has a weekly off on Friday ?
ready to work on probation for one year. (a) T (b) R
DIRECTIONS (31-35) : Each of the questions below consists of (c) W (d) Data inadequate
a question and two statements numbered I and II given below it. (e) None of these
You have to decide whether the data provided in the statements 37. What cuisine does R like ?
are sufficient to answer the questions. Read both the statements (a) Continental (b) Indian
and— (c) Italian (d) Spaish
(e) None of these
Give answer (1) if the data in Statement I alone are sufficient to 38. On which day does N have weekly off ?
answer the question, while the data in Statement II alone are not (a) Tuesday (b) Friday
sufficient to answer the question. (c) Monday (d) Sunday
Give answer (2) if the data in Statement II alone are sufficient to (e) None of these
answer the question, while the data in Statement I alone are not 39. Who likes Chinese cuisine ?
sufficient to answer the question. (a) T (b) B
Give answer (3) if the data either in Statement I alone or in (c) R (d) N
Statement II alone are sufficient to answer the question. (e) None of these
Give answer (4) if the data in both the Statements I and II are not 40. On which day does W have weekly off ?
sufficient to answer the question. (a) Monday (b) Sunday
Give answer (5) if data in both the Statements I and II together (c) Wednesday (d) Data inadequate
are necessary to answer the question. (e) None of these
y
o
u
rs
m
184 IBPS RRBs Officer Scale-I Exam 2013

a
h
b
QUANTITATIVE APTITUDE DIRECTIONS (51–55) : Study the pie-charts carefully to answer

o
o
the questions that follow :

b
.w
DIRECTIONS (41-45) : What should come in place of the Per cent of employees and women working in different

o
question mark (?) in the following questions ? Departments of an Organisation.

rd
Percentage of employees working in different departments

p
41. 20.2 × 64 × 81.3 × 40.2 = 8?

re
N = 8450 = Total no. of employees
(a) 2.7 (b) 2.5

s
s
(c) 3.7 (d) 3.2

.c
IT

o
(e) None of these department HR

m
42. 83% of 6242 × 12% of 225 = ? 18% department
34%
(a) 146286.42 (b) 134263.18
(c) 139883.22 (d) 1562218.23 22% Marketing
(e) None of these 14% 12% department
Finance
1 6 3 department
43. 1 +1 + 3 = ? Administration
8 7 5 department
121 163
(a) 8 (b) 6 Percentage of women working in different departments
140 280 Total no. of women = 3500
197 117
(c) 9 (d) 7 IT
280 140 HR
department department
(e) None of these
44. ? ¸ 25 ¸ 12 = 248.76 21% 17%
(a) 74628 (b) 497.52 Marketing
(c) 62452 (d) 870.66 25% 23% department
14%
(e) None of these
Finance
45. (73)3 = ? department Administration
(a) 365127 (b) 298627 department
(c) 305867 (d) 389017
(e) None of these 51. Which department has the lowest number of Men working
in it ?
DIRECTIONS (46–50) : What should come in place of question (a) HR (b) Marketing
mark (?) in the following number series ? (c) Administration (d) Finance
46. 3 4 12 ? 576 27648 (e) IT
(a) 64 (b) 96 52. What is the average number of Men working in different
(c) 36 (d) 52 departments in the organisation?
(e) None of these (a) 890 (b) 900
(c) 1020 (d) 780
47. 4 12 60 420 ? 60060
(e) None of these
(a) 4620 (b) 3780
53. What is the respective ratio of number of women working
(c) 4200 (d) 5040
in Administration department to the total number of
(e) None of these employees working in that department ?
48. 6 349 565 ? 754 781 (a) 6 : 7 (b) 245 : 507
(a) 629 (b) 590 (c) 7 : 6 (d) 243 : 322
(c) 601 (d) 690 (e) None of these
(e) None of these 54. Number of women working in the Finance & IT department
49. 1.5 4 20 128 ? 10505 together form what per cent of total number of employees
(a) 1044 (b) 1048 in the organisation? (rounded off to two digits after decimal)
(c) 1548 (d) 1052 (a) 19.05 (b) 26.78
(e) None of these (c) 95.83 (d) 59.21
50. 89250 17850 7140 1428 571.2 ? (e) None of these
(a) 228.48 (b) 126.12 55. What is the number of men working in the HR department?
(c) 114.24 (d) 246.48 (a) 794 (b) 823
(e) None of these (c) 926 (d) 683
(e) None of these
y
o
u
rs
m
IBPS RRBs Officer Scale-I Exam 2013 185

a
h
b
DIRECTIONS (56-60): Study the table carefully and answer 61. For state B the per cent rise in population from the previous

o
year was the highest in which of the following years ?

o
the questions that follow.

b
(a) 2008 (b) 2006

.w
Number of applications received (in hundreds) for six (c) 2005 (d) 2004

o
different specialist posts by six different Banks in a year (e) 2007

rd
62. What was the average population of state B (in millions)

p
Specia-

re
list post P Q R S T U for all the years together ?

s
(a) 38.5 (b) 28.5

s
Bank

.c
A 25.5 38.4 43.7 18.8 28.3 40.6 (c) 35 (d) 26

o
m
B 35.3 44.2 42.0 26.6 31.2 35.9 (e) 37.5
C 38.8 41.1 38.6 23.9 24.4 23.3 63. What is the per cent rise in population of state A in 2007
D 26.6 39.6 47.2 15.4 38.9 28.5 from the previous year ?
E 29.0 35.5 30.3 29.1 42.0 20.9 1
F 32.3 33.4 37.8 22.4 30.3 41.8 (a) 25 (b) 33
3
56. Number of applications received for Specialist post T by Bank 1
C is approximately what per cent of that received by B? (c) 33 (d) 25
3
(a) 58 (b) 78 (e) None of these
(c) 63 (d) 86 64. What is the ratio between the total populations of states A
(e) 92 and B respectively for all the years together ?
57. What is the respective ratio of number of applications (a) 37 : 45 (b) 37 : 43
received by Bank D, E & F for Specialist post S to that
(c) 43 : 37 (d) 45 : 37
received by Banks A, B & C for the same ?
(a) 223:231 (b) 215:283 (e) None of these
(c) 231:223 (d) 283:215 65. Population of state A in 2005 is what per cent of its total
(e) None of these population for all the years together ?
58. Which Bank has received the lowest number of
applications for all Specialist posts together ? (a) 17 8 % (b) 16 5 %
33 33
(a) D (b) A
(c) C (d) E 8 8
(e) None of these (c) 16 % (d) 17 %
37 37
59. What is the average number of applications received by
Bank F for all the Specialist posts together ? (e) None of these
(a) 33 (b) 3450 66. Area of a rectangle is equal to the area of the circle whose
(c) 3300 (d) 34.5 radius is 21 cms. If the length and the breadth of the rectangle
(e) None of these are in the ratio of 14 : 11 respectively, what is its perimeter ?
60. Which Specialist post had the maximum number of (a) 142 cms. (b) 140 cms.
applicants from all Banks together ? (c) 132 cms. (d) 150 cms.
(a) P (b) Q (e) None of these
(c) T (d) U 67. Shri Ramlal purchased a TV set for ` 12,500 and spent
(e) None of these ` 300 on transportation and ` 800 on installation. At what
price should he sell it so as to earn an overall profit of 15%?
DIRECTIONS (61–65) : Study the following graph carefully
to answer these questions : (a) ` 14,560 (b) ` 14,375
(c) ` 15,460 (d) ` 15,375
POPULATIONS OF TWO STATES (IN MILLIONS) (e) None of these
OVER THE YEARS 68. Three girls start jogging from the same point around a
State A circular track and each one completes one round in 24
State B seconds, 36 seconds and 48 seconds respectively. After
50
POPULATION IN MILLIONS

how much time will they meet at one point ?


45 (a) 2 minutes, 20 seconds
40 (b) 2 minutes, 24 seconds
35 (c) 4 minutes, 12 seconds
30 (d) 3 minutes, 36 seconds
25 (e) None of these
20 69. A 240–metre long train running at the speed of 60 kmph will
15 take how much time to cross another 270–metre long train
10 running in opposite direction at the speed of 48 kmph?
5 (a) 17 seconds (b) 3 seconds
0 (c) 12 seconds (d) 8 seconds
2003 2004 2005 2006 2007 2008 (e) None of these
YEARS
y
o
u
rs
m
186 IBPS RRBs Officer Scale-I Exam 2013

a
h
x2 – 7x + 10 = 0

b
70. Sarita started a boutique investing an amount of ` 50,000. 76. I.

o
Six months later Neeta joined her with an amount of II. y2 + lly + 10 = 0

o
b
` 80,000. At the end of one year they earned a profit of 77. I. x2 + 28x + 192 = 0

.w
` 18,000. What is Sarita’s share in the profit ? II. y2 + 16y + 48 = 0

o
(a) ` 9000 (b) ` 8000

rd
78. I. 2x – 3y = – 3.5
(c) ` 12000 (d) ` 10000

p
II. 3x – 2y = – 6.5

re
(e) None of these
x2 + 8x + 15 = 0

s
71. A alone can make 100 baskets in 6 days and B alone can 79. I.

s
.c
make 100 baskets in 12 days. In how many days can A and II. y2 + lly + 30 = 0

o
B together make 100 baskets?

m
80. I. x= 3136
(a) 3 days (b) 5 days 2
II. y = 3136
1 1
(c) 2 days (d) 3 days
2 2 GENERAL AWARENESS
(e) None of these
72. Samir’s age is one–fourth of his father’s age and two–third 81. Which of the following is NOT a measure to control Inflation
of his sister Reema’s age. What is the ratio of the ages of adopted by the Government and/or RBI ?
Samir, Reema and their father respectively ? (a) Monetary Policy (b) Fiscal Policy
(a) 3 : 2 : 8 (b) 3 : 4 : 8 (c) Financial Inclusion (d) Price Control
(c) 2 : 3 : 8 (d) 4 : 3 : 8 (e) Bank Rate Policy
(e) None of these 82. As we all know the Wholesale Price Index is divided in five
main commodity groups. Which of the following is NOT
DIRECTIONS (73–75) : Study the information carefully to
considered separately as a group while calculating
answer the questions that follow.
Wholesale Price Index (WPI) ?
A bucket contains 8 red, 3 blue and 5 green marbles. (a) Services and/or Production by Public Sector Units
73. If 4 marbles are drawn at random, what is the probability (b) Agriculture
that 2 are red and 2 are blue ? (c) Import and Exports
11 3 (d) Mining
(a) (b) (e) Manufacturing
16 16
83. In terms of the economics, the total value of the output
11 3 (goods and services) produced and income received in a
(c) (d)
72 65 year by domestic residents of a cotintry put together is
(e) None of these called
74. If 2 marbles are drawn at random, what is the probability (a) Net National Product
that both are green ? (b) Gross National Product
1 5 (c) Gross National Income
(a) (b) (d) National Income
8 16
(e) None of these
2 3 84. Which of the following schemes was launched a few years
(c) (d)
7 8 ago to provide wage employment to the rural people in
(e) None of these India (The programme is being run successfully in addition
75. If 3 marbles are drawn at random, what is the probability to MNREGA) ?
that none is red ? (a) Indira Awas Yojana
3 1 (b) Accelerated Rural Water Supply Programme
(a) (b) (c) Total Sanitation Campaign
8 16
(d) Sampoorna Grameen Rozgar Yojana
1 3 (e) Pradhan Mantri Gram Sadak Yojana
(c) (d) 85. Which of the following is TRUE about the Rajiv Gandhi
10 16
(e) None of these National Drinking Water Mission ?
(A) To provide safe drinking water in urban slums
DIRECTIONS (76-80) : In the following questions two equations (B) To provide sustainable safe drinking water in rural
numbered I and II are given. You have to solve both equations areas
and (C) To provide sustainable and safe drinking water in those
Give answer if parts of the country where ample water is not available
(a) x > y (b) x > y like coastal areas and desert areas.
(c) x < y (d) x < y (a) Only A (b) Only B
(e) x = y (c) Only C (d) All A, B and C
or the relationship cannot be established (e) None of these
y
o
u
rs
m
IBPS RRBs Officer Scale-I Exam 2013 187

a
h
b
86. Which of the following is True about the India –USA 92. As per the reports in various newspapers/magazines etc.

o
agreement on developing gas resources ? the agricultural productivity in India is low in comparison

o
b
(A) Both the nations will cooperate in exploiting gas to other countries. What are the main reasons of the same?

.w
hydrates in Krishna, Godavari and Mahanadi basins (A) Lack of credit and marketing facilities

o
and deep sea of Andaman. (B) Uneconomic holdings

rd
(B) USA will help in establishment of an IIT exclusively (C) Outdated agricultural techniques

p
re
for research and training in the areas of gas technology. (a) Only A (b) Only B

s
(C) India will send its experts to find out more gas reserves (c) Only C (d) Both A and C

s
.c
in various parts of USA particularly in deep sea of (e) All A, B and C

o
93. An insurance scheme for the agriculture sector was

m
Alaska as they have special training in the same.
(a) Only A (b) Only B introduced by the Government of India. Which of the
(c) Only C (d) All A, B and C following represents the name of the same?
(e) None of these (a) NAIS (b) RIDF
87. Rural sanitation programme of the Central Government is a (c) ALCIL (d) CACP
very popular programme in India. Which of the following (e) None of these
was/were the objective (s) of the same ? 94. Which of the following ministries lays down the policies
(A) To provide home sanitation and programmes for the development and regulation of the
(B) To provide disposal of garbage country’s water resources ?
(C) To provide wage employment to poor of society in (a) Ministry of Rural Development
rural areas (b) Ministry of Agriculture
(a) Only A (b) Only B (c) Ministry of Environment and Forests
(c) Only C (d) Both A and C (d) Ministry of Tribal Affairs
(e) All A, B and C (e) Ministry of Water Resources
88. Which of the following is/are true about the economy of 95. Which of the following phenomenon is also known as di
China ? matic Migration ?
(a) White Revolution (b) Green Revolution
(A) China as a country is rich but Chinese are not so.
(c) Global Warming (d) Urbanisation
(B) China’s growth strategy has been based mainly on
(e) None of these
exports.
96. The G–20 is grouping of world’s most powerful 20
(C) Despite attaining rapid growth and also bringing millions
countries. In terms of economic representation, what
of people out of poverty, China remains a poor country.
percentage wolds’s economy is represented by G–20
(a) Only A (b) Only B
nations ?
(c) Only C (d) Both A and C
(a) 50% (b) 60%
(e) All A, B and C
(c) 85% (d) 70%
89. Who amongst the following is the author of the book “What (e) None of these
I Require from Life” ? 97. India is a permanent Member of which of the following
(a) J.B.S. Haldane (b) Nirupam Sen organisations ?
(c) Sonal Shah (d) Sonia Gandhi (a) NATO (b) Non Aligned Movement
(e) Manmohan Singh (c) G–8 (d) OPEC
90. Which of the following is / are major reforms the Government (e) None of these
has introduced in Banking Sector ? 98. Which of the following commissions is formed in order to
(A) Dismantling the complex system of interest rate guard against predatory and collusive behaviour of the
controls. business organisations and protect the interest of business
(B) Eliminating prior approval of the RBI for large loans. organisations in order to promote economic efficiency in
(C) Introduction of capital adequacy norms and few other the country ?
prudential norms (a) Knowledge Commission
(a) Only A (b) Only B (b) Farmers Commission
(c) Only C (d) All A, B and C (c) Administrative Reforms Commission
(e) None of these (d) Labour Commission
91. Which of the following is/are recent trends noticed in (e) Competition Commission
Indian economy that have raised concern about food 99. Which of the following is correct about the Rajiv Gandhi
security, farmer’s income and poverty ? Grameen Vidyutikaran Yojana?
(A) Slowdown in growth in agriculture sector (A) The scheme was launched in few backward states only.
(B) Widening economic disparities between irrigated and (B) The aim of the scheme was to provide electricity to all
rainfed areas villages having a population of 5000 and above.
(C) Increased non agricultural demand for land and water (C) The scheme achieved its target in less than 10 years
as a result of fast urbanisation time.
(a) Only A (b) Only B (a) Only A (b) Only B
(c) Only C (d) All A, B and C (c) Only C (d) All A, B and C
(e) None of these (e) None of these
y
o
u
rs
m
188 IBPS RRBs Officer Scale-I Exam 2013

a
h
b
100. Some agro processing units make an advance agreement 110. Which of the following dance forms can not be called a

o
with the farmers and decide to purchase the entire classical dance ?

o
b
production of certain fruits, vegetables and medicinal plants (a) Kathak (b) Kuchipudi

.w
on an agreed upon cost/price. This type of arrangement is (c) Manipuri (d) Odissi

o
known as ____. (e) Ghumar

rd
(a) Import farming (b) Crop farming 111. Bill Gates is associated with which of the following

p
re
(c) Adoptive farming (d) Loan farming companies ?

s
(e) Contract farming (a) Infosys (b) Microtech

s
.c
101. India’s first fully dedicated university for Humanities is (c) Intel (d) Google

o
(e) None of these

m
established in ____ .
(a) Lucknow (b) Delhi 112. Which of the following is NOT a major IT company of India?
(c) Jaipur (d) Mumbai (a) HAL (b) TCS
(e) Hyderabad (c) Infosys (d) NIIT
102. National Food Security Management scheme was launched (e) Wipro
by the Government of India to achieve which of the 113. The present band approved by Reserve Bank of India
following ? between REPO and reverse REPO is
(A) Demonstration of improved production technology (a) 0.5% (b) 1%
(B) Distribution of high yield variety seeds (c) 1.5% (d) 0.75%
(C) To distribute fertilizer subsidy to farmers directly in (e) None of these
cash 114. The first set of Fourteen Nationalized Banks were
nationalized in the year
(a) Only (A) (b) Only (B)
(a) 1947 (b) 1955
(c) Only (C) (d) Both (A) and (B)
(c) 1969 (d) 1973
(e) All (A), (B) and (C)
(e) 1981
103. What is the full form of CFSA as used in financial sectors?
115. Term LIBOR is related with which of the following?
(a) Corporation of Farmers and Subsidiary Allocation
(a) Stock market prices (b) Commodity prices
(b) Committee on Financial Sector Assessment
(c) Sensitive advances (d) Interest rates
(c) Committee on farming sector and Agronomy (e) None of these
(d) Commission to Float Scientific Agriculture 116. Which are the two critical variables that generally go into
(e) None of these monetary planning by RBI ?
104. Which of the following terms is NOT used in cricket ? (a) Demand and interest rates
(a) Hoops (b) Maiden over (b) Growth and Inflation
(c) Stumped (d) Hattrick (c) Price and supply
(e) Gulleys (d) Growth and interest rates
105. ‘Malegaon’ which was in news recently is a town in (e) Movement in money market
(a) Bihar (b) Gujarat 117. Who among the following was selected for the Tagore
(c) Madhya Pradesh (d) Chhattisgarh Award for Cultural Harmony in July 2013?
(e) Maharashtra (a) Zubin Mehta (b) Shiamak Dawar
106. ‘TRAI’ the name which we read very often in the (c) M S Chandramukhi (d) P K Iyengar
newspapers is a regulatory body associated with which of (e) None of these
the following sectors ? 118. Tennis star Marion Bartoli who won the women’s singles
(a) Transport (b) Travel & Tourism title at Wimbledon 2013 announced her retirement on
(c) Technical Education (d) Telecom August 14, 2013. Bartoli is the native of
(e) None of these (a) Czech Republic (b) Serbia
(c) France (d) Argentina
107. Iron ore is found mainly in which of the following states?
(e) None of these
(a) Uttar Pradesh (b) West Bengal
119. What was the rank of India in the Global Innovation Index
(c) Jharkhand (d) Uttarakhand
2013 published in July 2013 ?
(e) Haryana (a) 65th (b) 66th
108. Which of the following diseases is NOT caused by a virus? (c) 67th (d) 68th
(a) Cancer (e) 70th
(b) Rabies 120. The annual supplement to the Foreign Trade Policy (FTP)
(c) AIDS 2009–14, announced by the Commerce and Industry
(d) Severe Acute Respiratory Syndrome (SARS) Ministry on April 18, 2013 added 47 new products under
(e) Influenza MLF– PS. What is the full form of MLFPS?
109. Angelina Jolie who was on a visit to India recently is (a) Market – Linked Focus Product Scheme
associated with which of the following areas ? (b) Market – Linked Foreign Product Scheme
(a) Journalism (b) Sports (c) Moratorium – Linked Focus Product Securitisation
(c) Politics (d) Films (d) Market – Liquidity Finance Parity Scheme
(e) Social Service (e) Market – Linked Future Product Scheme
y
o
u
rs
m
IBPS RRBs Officer Scale-I Exam 2013 189

a
h
132. The ____ tells the computer how to use its components.

b
COMPUTER KNOWLEDGE

o
(a) utility (b) network

o
b
121. To move to the beginning of a line of text, press the ____ (c) application program (d) operating system

.w
key. (e) None of these

o
133. Files deleted from the hard disk are sent to the ____.

rd
(a) pageup (b) a
(a) Recycle Bin (b) floppy disk

p
(c) home (d) enter

re
(c) clipboard (d) motherboard
(e) None of these

s
(e) None of these

s
122. Computers use the ____ number system to store data and

.c
134. A ____ is a named set of characters that have the

o
perform calculations. characteristics.

m
(a) binary (b) octal (a) type face (b) type style
(c) decimal (d) hexadecimal (c) font (d) pico
(e) None of these (e) None of these
123. Physical components that make up your computer are 135. A ____ pre–designed document th at alr eady has
known as coordinating fonts, a layout, and a back ground.
(a) Operating System (b) Software (a) guide (b) model
(c) ruler (d) template
(c) Hardware (d) Web Browsers
(e) None of these
(e) None of these 136. The instructions that tell a computer how to carry out the
124. Which key is used in combination with another key to processing tasks are referred to as computer ____.
perform a specific task ? (a) programs (b) processors
(a) function (b) space bar (c) input devices (d) memory modules
(c) arrow (d) control (e) None of these
(e) None of these 137. RAM can be thought of as the ____ for the computer’s
125. Ctrl, Shift and Alt are called ____ keys. processor.
(a) modifier (b) function (a) factory (b) operating room
(c) alphanumeric (d) adjustment (c) waiting room (d) planning room
(e) None of these
(e) None of these
138. C, BASIC, COBOL, and Java are examples of ____
126. The pattern of printed lines on most products are called languages.
____. (a) low–level (b) Computer
(a) prices (b) OCR (c) System programming (d) High–level
(c) scanners (d) barcodes (e) None of these
(e) None of these 139. An area of a computer that temporarily holds data waiting
127. What is the permanent memory built into your computer to be processed is ____.
called ? (a) CPU (b) Memory
(a) RAM (b) ROM (c) Storage (d) File
(c) CPU (d) CD–ROM (e) None of these
(e) None of these 140. A ____ is a microprocessor–based computing device.
128. Various applications and documents are represented on (a) personal computer (b) mainframe
(c) workstation (d) server
the Windows desktop by ____.
(e) None of these
(a) Symbols (b) Labels
141. The taskbar is located ____.
(c) Graphs (d) Icons (a) one the Start menu
(e) None of these (b) at the bottom of the screen
129. Passwords enable users to ____. (c) on the Quick Launch toolbar
(a) get into the system quickly (d) at the top of the screen
(b) make efficient use of time (e) None of these
(c) retain confidentiality of files 142. Generally, you access the Recycle Bin through an icon
(d) simplify file structures located ____.
(e) None of these (a) on the desktop
130. When sending an e-mail, the ____ line describes the (b) on the hard drive
contents of the message. (c) on the shortcut menu
(d) in the Properties dialog box
(a) subject (b) to
(e) None of these
(c) contents (d) cc 143. A MODEM is connected in between a telephone line and a
(e) None of these ____.
131. An ____ is a program that makes the computer easier to (a) Network
use. (b) Computer
(a) utility (b) application (c) Communication Adapter
(c) operating system (d) network (d) Serial Port
(e) None of these (e) All of these
y
o
u
rs
m
190 IBPS RRBs Officer Scale-I Exam 2013

a
h
b
144. Where is data saved permanently ? 155. Unsolicited commercial email is commonly known as

o
(a) Memory (b) Storage (a) spam (b) junk

o
b
(c) CPU (d) Printer (c) hoaxes (d) hypertext

.w
(e) None of these (e) None of these

o
156. ____ is processed by the computer into information.

rd
145. Changing an existing document is called ____ the
(a) numbers (b) processor

p
document.

re
(a) creating (b) editing (c) input (d) data

s
(e) None of these

s
(c) modifying (d) adjusting

.c
157. A web site address is a unique name that identifies a specific

o
(e) None of these

m
____ on the web.
146. Which is not a basic function of a computer ?
(a) web browser (b) web site
(a) Copy text (b) Accept input
(c) PDA (d) link
(c) Process data (d) Store data
(e) None of these
(e) None of these 158. A device that connects to a network without the use of
147. The ____ is the box that houses the most important parts cables is said to be
of a computer system. (a) distributed (b) centralised
(a) software (b) hardware (c) open source (d) wireless
(c) input device (d) system unit (e) None of these
(e) None of these 159. Editing a document consists of reading through the
148. The term ____ refers to data storage systems that make it document you’ve created, then
possible for a computer or electronic device to store and (a) correcting your errors (b) printing it
retrieve data. (c) saving it (d) deleting it
(a) retrieval technology (b) input technology (e) None of these
(c) output technology (d) storage technology 160. The Internet allows you to
(e) None of these (a) send electronic mail
149. The term ____ refers to any computer component that is (b) view web pages
required to perform work. (c) connect to servers all around the world
(a) bootstrap (b) kernel (d) All of these
(c) resource (d) source code (e) None of these
(e) None of these
150. A computer system includes ____. ENGLISH LANGUAGE
(a) hardware (b) software DIRECTIONS (161–175) : Read the following passage carefully
(c) peripheral devices (d) All of these and answer the question given below it. Certain words have
(e) None of these been printed in bold to help you locate them while answering
151. ____ is the maximum amount of data that can be stored on some of the questions.
a storage medium.
Agriculture has always been celebrated as the primary sector
(a) Magnetic storage (b) Optical storage
in India. Thanks to the Green Revolution, India is now self-
(c) Solid – state storage (d) Storage capacity sufficient in food production. Indian agriculture has been
(e) None of these making technological advancement as well. Does that mean
152. The____ is responsible for performing calculations and everything is looking bright for Indian agriculture ? A
contains decision–making mechanisms. superficial analysis of the above points would tempt one to
(a) Central Processing Unit say yes, but the truth is far from it. The reality is that Indian
(b) Memory Unit farmers have to face extreme poverty and financial crisis, which
(c) Arithmetic and Logic Unit is driving them to suicides. What are the grave adversities that
(d) Output Unit drive the farmers to commit suicide, at a time when Indian
(e) None of these economy is supposed to be gearing up to take on the world
153. A ____ is a large and expensive computer capable of ?
simultaneously processing data for hundreds or thousands Indian agriculture is predominantly dependent on nature.
of users. Irrigation facilities that are currently available, do not cover the
(a) handheld computer (b) mainframe computer entire cultivable land. If the farmers are at the mercy of monsoons
(c) personal computer (d) tablet computer for timely water for their crops, they are at the mercy of the
government for alternative irrigation facilities. Any failure of
(e) None of these
nature, directly affects the fortunes of the farmers. Secondly,
154. Which process checks to ensure the components of the Indian agriculture is largely an unorganised sector, there is no
computer are operating and connected properly? systematic planning in cultivation, farmers work on lands of
(a) Booting (b) Processing uneconomical sizes, institutional finances are not available and
(c) Saving (d) Editing minimum purchase prices of the government do not in reality
(e) None of these reach the poorest farmer. Added to this, the cost of agricultural
y
o
u
rs
m
IBPS RRBs Officer Scale-I Exam 2013 191

a
h
b
inputs have been steadily rising over the years, farmers’ margins (C) To avail relief package announced by the government

o
of profits have been narrowing because the price rise in inputs is to support the family of those who commit suicide.

o
b
not complemented by an increase in the purchase price of the (a) Only A (b) Only B

.w
agricultural produce. Even today, in several parts of the country, (c) Only B and C (d) Only A and B

o
agriculture is a seasonal occupation. In many districts, farmers (e) None of these

rd
get only one crop per year and for the remaining part of the year, 163. According to the author why does the situation of

p
re
they find it difficult to make both ends meet. agricultural sector remain grim even after making several

s
The farmers normally resort to borrowing from money technological advances ?

s
.c
lenders, in the absence of institutionalized finance. Where (a) Indian farmers continue to face adversities from nature

o
institutional finance is available, the ordinary farmer does not

m
as well as the government.
have a chance of availing it because of the “procedures” involved (b) India has failed to match the technological advances
in disbursing the finance. This calls for removing the elaborate taking place in the rest of the world
formalities for obtaining the loans. The institutional finance, (c) Natural calamities have been very frequent in India
where available is mostly availed by the medium or large land (d) Banks have failed to provide adequate loans to the
owners, the small farmers do not even have the awareness of the farmers
existence of such facilities. The money lender is the only source (e) None of these
of finance to the farmers. Should the crops fail, the farmers fall 164. Which of the following is not true in context of the passage?
into a debt trap and crop failures piled up over the years give (a) Many farmers struggle to sustain themselves after
them no other option than ending their lives. reaping one crop in a year
Another disturbing trend has been observed where farmers (b) The government has relieved the farmers from any
commit suicide or deliberately kill a family member in order to elaborate formalities while availing the loans
avail relief and benefits announced by the government to support (c) India was made self–sufficient in food production
the families of those who have committed suicide so that their because of green revolution
families could atleast benefit from the Government’s relief (d) Some farmers commit suicide in order to avail relief
programmes. What then needs to be done to prevent this sad state package from the government
of affairs ? There cannot be one single solution to end the woes (e) None of these
of farmers. 165. What does the author suggest as opposed to providing
Temporary measures through monetary relief would not be temporary monetary relief to the farmers ?
the solution. The governmental efforts should be targeted at
(A) To improve the entire agricultural setup in India instead
improving the entire structure of the small where in the relief is
of providing relief in the face of adversities.
not given on a drought to drought basis, rather they are taught
(B) Providing the local money lender with large amount of
to overcome their difficulties through their own skills and
money so that small farmers can easily obtain loans
capabilities. Social responsibility also goes a long way to help
from them.
the farmers. General public, NGOs, Corporate and other
(C) Empowering the farmers so that they can sustain a
organisations too can play a part in helping farmers by adopting
livelihood throughout life without having to face the
drought affected villages and families and helping them to
desperation that adversity drives them to.
rehabilitate.
(a) Only A (b) Only A and B
The nation has to realise that farmers’ suicides are not minor
issues happening in remote parts a few states, it is a reflection of (c) Only B (d) Only A and C
the true state of the basis of our economy. (e) None of these
161. What does the author mean by “procedures” when he says 166. What are the adversities faced by Indian farmers according
that ‘farmers do not get a chance of availing institutional to the passage?
finance because of procedures involved in it’ ? (a) Lack of adequate irrigation facilities
(a) He refers to the government guideline of disbursing (b) Ownership of only a small piece of land by a majority
finance only to medium and large land owners? of farmers, which fails to generate any profit
(b) Refers to the strict government rule of providing loans (c) Lack of financial help to the farmers
to only such farmers who can guarantee a default– (d) All of these
free–tenure (e) None of these
(c) The formalities to avail these facilities are enormous 167. According to the passage why don’t farmers avail the
and too difficult for an ordinary farmer to understand institutional finance facilities ?
(d) Refers to the danger farmers must face from the local (a) Banks are not willing to provide loans to the farmers
money-lenders they availed the loan from government because of high risk associated with it
instead (b) Many NGOs and corporate organisations provide them
(e) None of these the loans in a hassle free manner
162. Why have many farmers resorted to killing family members? (c) Most of the farmers do not need finance in large scale
(A) It is difficult for the farmers to sustain their family’s since they work on small size lands
livelihood. (d) The local money-lenders charge lower interest rates
(B) So that the killed family member may get rid of the as compared to such financial facilities
persistent adversities. (e) None of these
y
o
u
rs
m
192 IBPS RRBs Officer Scale-I Exam 2013

a
h
b
168. According to the passage, how can general public lend a DIRECTIONS (176-180) : Which of the phrases (a), (b d), (c d)

o
helping hand to the struggling farmers ?

o
and (d) given below each statement should replace the phrase

b
(a) By adopting the affected families and helping them to printed in bold in the sentence to make it grammatically correct?

.w
rehabilitate If the sentence is correct as it is given and ‘No correction is

o
rd
(b) By acting as a relief worker in the drought hit areas required’, mark (e) as the answer.

p
(c) To provide help for building dams and better irrigation

re
176. The employees offers to work overtime without any
facilities

s
compensation to increase the profit.

s
(d) Not specified in the passage

.c
(a) offering to work (b) offer to working

o
(e) None of these (c) offered for work (d) offered to work

m
169. Why is the profit margin of farmers narrowing even after (e) No correction required
increasing the minimum purchase price? 177. He donated his entire wealth to an orphanage since he
(A) The minimum purchase price of the government is too didn’t have a children of his own.
low to make any profit. (a) have any children (b) had any child
(B) The quality of the inputs such as seeds and fertilizers (c) has a child (d) has any children
is very poor. (e) No correction required
(C) There has been a continuous rise in the price of 178. Psychologists are making best efforts to understand the
agricultural inputs which adversely affects the profit. reason behind unruly behaviour of adolescents.
(a) Only A (b) Only B (a) for understand the (b) to understanding the
(c) Only C (d) Only A and C (c) to understands the (d) to understood the
(e) No correction required
(e) None of these
179. Althoughhe is a reputed speaker, today his speech were
170. What is the author’s main objective in writing the passage?
irrational and full of ambiguities.
(a) Criticising the policy of providing relief packages to (a) speeches was irrational
the family of farmers who commit suicide (b) speech was irrational
(b) To appeal to the non institutional money lenders for (c) speech were irrationals
providing loans to farmers in a hassle-free manner (d) speech are irrational
(c) To applaud the dauntless spirit of the farmers (e) No correction required
(d) To highlight the drawbacks in the agriculture sector 180. He was told for report back to the camp in the middle of the
(e) None of these holidays when the tension at the border aggravated.
(a) He is told to (b) He was telling to
DIRECTIONS (171 – 173) : Choose the word which is most (c) He would told for (d) He was told to
similar in meaning to the word printed in bold as used in the (e) No correction required
passage.
DIRECTIONS (181–185) : Read each sentence to find out
171. BRIGHT whether there is any grammatical error in it. The error, if any,
(a) Glowing (b) Radiant will be in one part of the sentence. The number of that part is the
(c) Dazzling (d) Lustrous answer. If there is no error, the answer is (e) i.e. ‘No Error’.
(e) Promising (Ignore the errors of punctuation, if any).
172. GRAVE 181. He has been assigned (a)/ with that team (b)/ because he is
(a) Cemetery (b) Fatal (c)/ well qualified and experienced. (d)/ No error (e)
(c) Severe (d) Carve 182. The company’s new project (a)/ cannot be launched (b)/
(e) Trivial unless the approval (c)/ of the Board. (d)/ No error (e)
173. WOES 183. Instead criticizing (a)/ why don’t (b)/ you help (c)/ with the
(a) Suffering (b) Tragedy presentation ? (d)/ No error (e)
(c) Bitterness (d) Anger 184. Mr. Sethi was (a)/ not given a promotion (b)/ because he is
(e) Fear (c)/ frequent absent. (d)/ No error (e)
185. Inspite of (a)/ so many hardships (b)/ Amar has managed
DIRECTIONS (174-175) : Choose the word which is most (c) / to success. (d)/ No error (e)
opposite in meaning to the word printed in bold as used in the
DIRECTIONS (186–190): Rearrange the following six sentences
passage.
(A), (B), (C), (D), (E) and (F) in the proper sequence to form a
174. SUPERFICIAL meaningful paragraph; then answer the questions given below
(a) Careless (b) Profound them.
(c) Extreme (d) Articulate (A) Thus rapid development is still unable to meet demand.
(e) Fear (B) Surplus funds from hikes in passenger fares and cuts in
175. NARROWING staff have made this possible.
(a) Broadening (b) Stretching (C) This demonstrates that it has been transformed into a
(c) Changing (d) Increasing modern high standard design and high service reliable
(e) Extending system.
y
o
u
rs
m
IBPS RRBs Officer Scale-I Exam 2013 193

a
h
b
(D) However China’s railways are not problem free. made him known and (193) around the world. (194) many people

o
(E) China’s railways have been able to generate the funds who work to create humour he took it very seriously. He would

o
b
needed for the construction of new railway lines internally. sit sadly (195) the funniest cartoon concentrating on some way

.w
(F) For example at present about 2,80,000 cars are requested to improve it. Walt Disney (196) the opinions of those working

o
daily to transport goods but only half the requests can be with him but the (197) judgment was always his. He demanded a

rd
met. lot (198) people but he gave a lot too. When the economy was

p
re
186. Which of the following is the FIFTH sentence after not doing well he gave every one a (199) and though some (200)

s
rearrangement ? of this, it gave his employees’ morale a boost.

s
.c
(a) A (b) C 191. (a) yet (b) even

o
m
(c) D (d) E (c) and (d) till
(e) F (e) besides
187. Which of the following is the SIXTH (LAST) sentence 192. (a) from (b) where
after rearrangement ? (c) which (d) while
(a) A (b) B (e) that
(c) C (d) D 193. (a) respect (b) seen
(e) F (c) loved (d) entertained
188. Which of the following is the SECOND sentence after (e) laughed
rearrangement ? 194. (a) for (b) to
(a) A (b) B (c) without (d) not
(c) C (d) D (e) like
(e) E 195. (a) on (b) until
189. Which of the following is the THIRD sentence after (c) front (d) through
rearrangement ? (e) in
(a) A (b) B 196. (a) saw (b) concluded
(c) C (d) E (c) discussed (d) discouraged
(e) F (e) valued
190. Which of the following is the FIRST sentence after 197. (a) final (b) ultimately
rearrangement ? (c) important (d) hasty
(a) A (b) B (e) lasting
(c) C (d) D 198. (a) by (b) from
(e) E (c) with (d) to
(e) many
DIRECTIONS (191–200) : In the following passage there are
199. (a) advance (b) share
blanks each of which has been numbered. These numbers are
(c) fee (d) raise
printed below the passage and against each five words are
(e) profit
suggested one of which fits the blank appropriately. Find out
200. (a) credit (b) disapproved
the appropriate word in each case.
(c) criticized (d) offended
Although he is no longer alive, (191) his influence can be felt in (e) paid
the studio (192) he created cartoons and feature films which
y
o
u
rs
m
194 IBPS RRBs Officer Scale-I Exam 2013

a
h
b
o
o
b
.w
o
rd
(1 – 5): Conclusions:

p
DÞ< SÞ³ @Þ£ K © B Þ K = B : Not True

re
I.

s
ÓÞ = #Þ> II. Q D K Þ Q < K : True

s
.c
(6 – 10) : On the basis of given information, we can costruct

o
1. (c) Statements:

m
R@ JÞ R £ J the following table :
FSJ ÞF³J
CSFÞC³F Persons Sex Profession Car
Therefore, R Male Professor Scorpio
R£J£F£C
M Female Travel Agent Alto
Conclusions:
I. R © C Þ R = C : Not True T Female Air-Hostess Ikon
Either
II. C # R Þ C > R : Not True S Female Doctor Esteem
So either conclusion I or II is true.
L Male Jeweller Corolla
2. (a) Statements:
W @ PÞ W £ P W Male Lawyer Lancer
W#E ÞW>E Z Male Consultant Santro
EDV ÞE<V
Therefore, 6. (d) S owns Esteem who is a Doctor.
V>E<W£P 7. (c) W, the Lawyer, owns Lancer car.
So only conclusion is true.
8. (a) R is a Professor.
Conclusions:
I. P # E Þ P > E : True 9. (b) S is the Doctor.
II. V © W Þ V = W : Not True 10. (b) The three ladies are : M, T and S.
So all the conclusions follow.
3. (e) Statements: 11. (e) Toys
J©RÞJ=R
PSRÞP³R Trees
Z#PÞZ>P
Therefore, Angels
J=R£P<Z
Conclusions: OR
I. R D Z Þ R < Z : True
II. J @ P Þ J £ P : True Toys
So both the conclusions I and II are true. So all the
conclusions follow. Trees
4. (d) Statements:
G@OÞG£O
Angels
N©OÞN=O
H#GÞH>G Conclusions:
Therefore,
I. False
H>G£O=N
Conclusions: II. False
I. O D H Þ O < H : Not True III. False
II. G © N Þ G = N : Not True A + I Þ No conclusion. Hence none follows.
So none of the conclusion follows.
5. (b) Statements 12. (a)
QDBÞQ<B
Pigeons
Horses

M©BÞM=B
Dogs

Cats

KSMÞK³M
Therefore,
Q<B=M£K
y
o
u
rs
m
IBPS RRBs Officer Scale-I Exam 2013 195

a
h
R S BU I TU H B

b
Conclusions: S T AT I S T I C

o
I. True

o
b
–1
II. True

.w
III. True –1

o
+1

rd
So all the conclusions follow. +1

p
+0

re
13. (d) Bottles +1

s
Bottles

s
Streets +1

.c
–1

o
Parrots

m
Tables –1
17. (e) 1 2 3 4 5 6 7 8 9 10
O M N I S C I E N T
Conclusions: Specifiedletters Þ M, S, I, T
I. False Meaningful word
II. True Þ M I S T
III. True
Third letter
14. (b) 18. (d) Pineapple is grown underground.
Windows Windows
19. (a) 5 14 9 7 13 1 20 9 3
Boys Cartoons Jokers N I G M A T I C
E

20. (d) 5 /1 6 3 / 9
Conclusions:
I. False Such combinations are :
II. True 163 , 569 , 163
III. False 21. (b) Options (a)
So only conclusion II follows.
L R
(+) (?)
15. (c) Bats

Balls Saints B Q
(+)

Tigers The sex of R is not known


Option (b)
OR
J R K
(+)
Bats
Paternal uncle
Balls Saints B D
(+)

Tigers Option (c)


Conclusions: S B
(+)
I. True
–Nephew
II. True R P
III. False (+)
So only conclusions I and II follows.
Hence only (b) follows.
16. (c) R U S T I C A TI E Q T T U I D B S D 22. (d) Statements (iii) and (iv) are superfluous because
relationship between S and T is not required
–1
anywhere.
–1 23. (c) Except 63, all other numbers are Prime Numbers.
+1
+1 24. (a) S A M B L E
+0
¯ ¯ ¯ ¯ ¯ ¯
+1
+1 = / Ó $ S %
–1 S T A R
–1
¯ ¯ ¯ ¯
Therefore,
= @ / £
y
o
u
rs
m
196 IBPS RRBs Officer Scale-I Exam 2013

a
h
b
Therefore, Shilpa > Priya > Pinky > Sheetal > Neetu

o
T R E M B L E

o
b
¯ ¯ ¯ ¯ ¯ ¯ ¯

.w
Earns highest

o
@ £ %© $ S %

rd
25. (b) Birds fly in sky. But here sky is called star. 35. (a) From statement I

p
re
(26-30):
Ansh

s
s
CRITERIA Suresh Bindu

.c
CANDIDATE

o
(a) (b) (c) or (i) (d) or (ii) (e) Ans

m
Rajesh NG ü ü – ü – ü 2
RoopKamal ü ü ü – ü – ü 3
Neel Tanvi
Indira ü ü ü – ü – NG 2
Darshan × ü ü – ü – ü 5 So only statement I sufficient to answer the question
Vanitha ü ü ü – – ü ü 4 (36-40):
26. (b) No information is given regarding the criterion (a). On the bassis of given information we can construct the
27. (c) Roop Kamal does satisfy all the criteria. following table :
28. (b) No information is given regarding the criterion (e).
29. (e) Darshan Patil does not satisfy criterion (a). Persons Offs Cusine
30. (d) Vanitha Mittal does satisfy (a), (b), (c), (ii) and (e). N Monday Mexican Cusine
31. (e) From statement I K Thursday Thai Food
Shi tu ke Þ pen is blue T Wednesday Chinese
From statement II B Friday Italian Food
ke si re Þ this is wonder M Saturday Indian Food
From both statements W Sunday Continental Food
R Tuesday Spabnish Cuisine
shi tu ke Þ pen i s blue

ke is re Þ this i s wonder. 36. (e) B has weekly off on Friday.


code for is Þ ke 37. (d) R likes Spanish cuisine.
So data in both the statements together is necessary 38. (c) N has weekly off on Monday.
to answer the question. 39. (a) T likes Chinese cuisine.
32. (d) Both statements are not sufficient to answer the 40. (b) W has weekly off on Sunday.
question 41. (e) 20.2 ×26 × (23)1.3 × (22)0.2 = (23)?
33. (b) From statement I Þ (2)0.2+ 6 + 3.9 + 0.4 = 23?
Mallika « Vandana Û Rajesh Þ 3? = 10.5
( -) (+) 10.5
Þ ?= = 3.5
Prabha is Rajesh’s good friend 3
From statement II
83 12
42. (c) ? = 6242 × ´ 225 ´ = 139883.22
Vandana Û Rajesh «? Û Prabha 100 100
(–) (+) (+) (–)
43. (b) 9 13 18
Sister-in-law ?= + +
8 7 5
34. (e) From statement I 315 + 520 + 1008 1843 163
= = =6
Shilpa > Priya > Pinky & Sheetal 280 2850 280
From statement II
?
Pinky > Sheetal > Neetu 44. (a) = 248.76
25 ´ 12
From both statements
Þ ? = 25 × 12 × 248.76 = 74628
y
o
u
rs
m
IBPS RRBs Officer Scale-I Exam 2013 197

a
h
(d) ? = (73)3

b
45. = (70 + 3)3 Number of men = 1859 – 805 = 1054

o
o
= (70) + (3)3 + 3(70)2 (3) + 3(70)(3)2
3 Administration department

b
.w
= 343000 + 27 + 44100 + 1890 = 389017
12
´ 8450 = 1014

o
46. (e) The given series is based on the following pattern : Total number of employees =

rd
100

p
12 = 3 × 4

re
14

s
? = 12 × 4 = 48 Number of women = ´ 3500 = 490

s
100

.c
o
576 = 48 × 12 Number of men = 1014 – 490 = 524

m
27648 = 576 × 48 Finance department
Hence, 48 will replace the question mark.
14
×3 ×5 ×7 ×9 Total number of employees = ´ 8450 = 1183
47. (b) 4 12 60 420 3780 100
×11
25
Number of women = ´ 3500 = 875
41580 100
Hence, 3780 is the correct answer Number of men = 1183 – 875 = 308
48. (d) Identifying the pattern of given number series IT department :
3 3
6 +7 3
349 +6
565 +5
690 34
Total number of employees = ´ 8450 = 2873
3
100
+4
3
781
+3
754 21
Number of women = ´ 3500 = 735
100
Hence, 690 is the correct answer.
Number of men = 2873–735 = 2138
49. (b) Identifying the pattern of given number series
51. (d) Number of men in finance department = 308
2 2
×2+12 ×4+2 ×6+3
1.5 4 20 129 926 + 1054 + 524 + 308 + 2138
×8+4
2 52. (e) Required average =
5
2
×10+5
10505 1048
4950
So 10505 is the correct ansewr. = = 990
5
50. (c) The given series is based on the following pattern :
53. (b) Required ratio = 490 : 1014 = 245 : 507
89250 17850 7140 1428 571.2 114.24 54. (a) Number of women working in the Finance and IT
department
÷5 ÷ 2.5 ÷5 ÷ 2.5 ÷5
= 875 + 735 = 1610
Hence, 114.24 will replace the question mark.
Total number of employees = 8450
(51–55) :
HR department 1610
Required percentage = ´ 100 = 19.05
8450
18
Total number of employees = ´ 8450 = 1521 55. (c) Number of men working in HR department = 926
100
24.4
17 56. (b) Required percentage = ´ 100 = 78.205 » 78
Number of women = ´ 3500 = 595 31.2
100
57. (a) Number of applications received by the banks D,E
Number of men = 1521 – 595 = 926 and F for specialist post ‘S’
Marketing department = (15.4 + 29.1 + 22.4) hundreds = 66.9 hundreds
22 Number of applications received by the banks A,B
Total number of employees = ´ 8450 = 1859 and C for specialist post ‘S’
100
= (18.8 + 26.6 + 23.9) hundreds = 69.3 hundreds
23 Required ratio = 66.9 : 69.3
Number of women = ´ 3500 = 805
100
= 669 : 693 = 223 : 231
y
o
u
rs
m
198 IBPS RRBs Officer Scale-I Exam 2013

a
h
b
58. (d) Number of applications received by 66. (d) Area of rectangle = Area of circle

o
o
Bank A ® 25.5 + 38.4 + 43.7 + 18.8 + 28.3 + 40.6

b
22
(hundreds)

.w
= × 21 × 21 = 1386 sq. cm.
7

o
= 195.3 hundreds

rd
Let the length and breadth of rectangle be 14x and
Bank D ® 26.6 + 39.6 + 47.2 + 15.4 + 38.9 + 28.5

p
11x cm respectively. Then

re
(hundreds)

s
14x. 11x = 1386

s
= 196.2 hundreds

.c
o
Bank E ® 29 + 35.5 + 30.3 + 29.1 + 42 + 20.9 1386

m
Þ x2 = =9
(hundreds) 14 ´ 11
= 186.8 hundreds
\ x= 9 =3
Bank F ® 32.3 + 33.4 + 37.8 + 22.4 + 30.3 + 41.8
(hundreds) \ Perimeter of rectangle = 2 (14x + 11x)
= 198 hundreds = 50x = 50 × 3 = 150 cm
So Bank E had received lowest hnumber of applicants. 67. (e) Total cost of TV = ` (12500 + 300 + 800) = ` 13600
For a profit of 15%.
19800
59. (c) Required average = = 3300
6 æ 115 ö
Selling price = ` ç ´ 13600 ÷ = ` 15640
60. (e) Number of applicants in all banks for post : è 100 ø
Q ® (38.4 + 44.2 + 41.1 + 39.6 + 35.5 + 33.4) hundreds 68. (b) Required time = LCM of 24, 36, and 48 seconds
= 232.2 hundreds = 144 seconds = 2 minutes 24 seconds
R ® (43.7 + 42.0 + 38.6 + 47.2 + 30.3 + 37.8) 69. (a) Relative speed = (60 +48) kmph = 108 kmph
hundreds
æ 5ö
= 239.6 hundreds = ç 108 ´ ÷ m/sec = 30 m/sec.
è 18 ø
U ® (40.6 + 35.9 + 23.3 + 28.5 + 20.9 + 41.8) hundreds
= 191 hundreds Total distance = (240 + 270) metre = 510 metre
So specialist post R had received maximum number 510
of applicants. \ Time taken in crossing = = 17 seconds
30
61. (d) It is obvious from the graph. 70. (d) Ratio of equivalent capitals of Sarita and Neeta for
62. (e) Average population of state B over the years 1 month
= 50000 × 12 : 80000 × 6 = 5 : 4
æ 20 + 30 + 35 + 45 + 45 + 50 ö
=ç ÷ million
è 6 ø æ5 ö
\ Sarita’s share = ` ç ´18000 ÷ = ` 10000
è9 ø
225
= million = 37.5 million
6 1
71. (e) A’s 1 day’s work =
63. (b) Required percentage increase 6

40 - 30 1
= ´ 100 B’s 1 day’s work =
30 12

100 1 1 1 2 +1 1
= = 33 % \ (A + B)’s 1 day’s work = + = =
3 3 6 12 12 4
64. (a) Total population of state A over the years \ A and B together will make 100 baskets in 4 days.
= (15 + 25 + 30 + 30 + 40 + 45) million 72. (c) Samir’s age = x year (let)
= 185 million \ His father’s age = 4x years
Total population of state B over the years = 225 million
3
Required ratio = 185 : 225 = 37 : 45 Reema’s age = x years
2
30 600 8
65. (c) Required percentage = ´ 100 = = 16 % 3x
185 37 37 \ Required ratio = x : : 4x = 2 : 3 : 8
2
y
o
u
rs
m
IBPS RRBs Officer Scale-I Exam 2013 199

a
h
Þ

b
73. (d) Total number of marbles = 8 + 3 + 5 = 16 y2
+ 12y + 4y + 48 = 0

o
o
n(S) = Exhaustive number of cases Þ y (y + 12) + 4 (y + 12) = 0

b
.w
= Number of ways of drawing 4 marbles out of 16 Þ (y + 12) (y + 4) = 0

o
Þ y = – 12 or – 4

rd
16 ´15 ´ 14 ´ 13
= 16C4 = = 1820

p
1´ 2 ´ 3 ´ 4 Clearly, x < y

re
78. (c) 2x – 3y = – 3.5 … (i)

s
n(E) = Number of cases when 2 marbles are red and

s
.c
2 are blue. 3x – 2y = – 6.5 … (ii)

o
m
By equation (i) × 2 – equation (ii) × 3, we have
8´ 7 3´ 2
= 8C × 3C = ´ = 84
2 2
1´ 2 1´ 2 4 x - 6 y = -7
9 x - 6 y = -19.5
84 3 - + +
\ Required probability = =
1820 65 -5 x = 12.5

16 ´15
74. (e) n(S) = 16C2 = = 120 12.5
1´ 2 Þ x= = –2.5
-5
5´ 4 From equation (i)
n(E) = 5C2 = = 10
1´ 2 2 × (–2.5) – 3y = – 3.5
10 1 Þ 3y = – 5 + 3.5
Required probability = =
120 12 -1.5
Þy= = - 0.5
3
16 ´ 15 ´14
75. (c) n(S) = 16C2 = = 560 Clearly, x < y
1´ 2 ´ 3
79. (b) I. x2 + 8x + 15 = 0
Out of the three drawn marbles none is red.
Clearly they will be either blue or green. Þ x2 + 5x + 3x + 15 = 0
Þ x(x + 5) + 3(x + 5) = 0
8´ 7 ´ 6
\ n(E) = 8C = = 56 Þ (x + 5) (x + 3) = 0
3 1´ 2 ´ 3
Þ x = –5 or –3
n(E) 56 1 II. y2 + 11y + 30 = 0
\ Required probability = n(S) = =
560 10 Þ y2 + 6y + 5y + 30 = 0
76. (a) I. x2 – 7x + 10 = 0 Þ y (y + 6) + 5 (y + 6) = 0
Þ x2
– 5x – 2x + 10 = 0 Þ (y + 5) (y + 6) = 0
Þ x(x – 5) – 2(x – 5) = 0 Þ y = – 5 or – 6
Þ (x – 2) (x – 5) = 0 Clearly, x > y
Þ x = 2 or 5 80. (e) x = 3136 = ±56
II. y2 + 11y + 10 = 0
y2 = 3136
Þ y2 + 10y + y + 10 = 0
Þ y= 3136 = ±56
Þ y(y + 10) + 1(y + 10) = 0
Þ (y + 1) (y + 10) = 0 Clearly, x = y
Þ y = – 1 or – 10 81. (c) 82. (c) 83. (b) 84. (d)
Clearly, x > y 85. (b) 86. (a) 87. (e) 88 (e)
77. (d) I. x2 + 28x + 192 = 0 89 (a) 90. (c) 91. (d) 92. (e)
Þ x2 + 16x + 12x + 192 = 0 93. (a) 94. (e) 95. (d) 96. (e)
Þ x(x + 16) + 12(x + 16) = 0 97. (b) 98. (e) 99. (e) 100. (e)
Þ (x + 12) (x + 16) = 0 101. (b) 102. (d) 103. (c) 104. (a)
Þ x = – 12 or –16 105. (e) 106. (d) 107. (c) 108. (a)
II. y2 + 16y + 48 = 0 109. (d) 110. (e) 111. (e) 112. (a)
y
o
u
rs
m
200 IBPS RRBs Officer Scale-I Exam 2013

a
h
b
113. (b) 114. (c) 115. (d) 116. (b) 168. (a) It is mentioned in the last line of the fifth paragraph

o
o
117. (a) 118. (c) 119. (b) 120. (a) 169. (c) It is mentioned in the second paragraph

b
.w
121. (c) 122. (a) 123. (c) 124. (d) 170. (d) 171. (e) 172. (c)

o
125. (a) 126. (d) 127. (b) 128. (d) 173. (a) 174. (b) 175. (a) 176. (d)

rd
p
129. (c) 130. (a) 131. (a) 132. (d) 177. (a) 178. (e) 179. (b) 180. (d)

re
133. (a) 134. (b) 135. (d) 136. (a) 181. (b) To that team.

s
s
.c
137. (b) 138. (c) 139. (b) 140. (a) 182. (c)

o
m
141. (b) 142. (a) 143. (b) 144. (b) 183. (a) Instead of criticising.
145. (b) 146. (a) 147. (d) 148. (d) 184. (d) Frequently absent.
149. (c) 150. (d) 151. (d) 152. (c) 185. (d) To succeed.
153. (b) 154. (a) 155. (a) 156. (d) 186. (e) 187. (a) 188. (b)
157. (b) 158. (d) 159. (a) 160. (d) 189. (c) 190. (e) 191. (a) 192. (b)
161. (c) 193. (c) 194. (e) 195. (d) 196. (e)
162. (c) It is mentioned in the last few lines of the third 197. (a) 198. (b) 199. (b) 200. (b)
paragraph nad the first few lines of the fourth 161. (b) 162. (a) 163. (d) 164. (d)
paragraph.
165. (b) 166. (c) 167. (b) 168. (a)
163. (a) It can be inferred from the entire passage.
169. (a) 170. (d) 171. (c) 172. (d)
164. (b) It is mentioned in the third paragrpah how the
173. (a) 174. (b) 175. (d) 176. (d)
lenghty procedures make it difficult for the needy
farmers to avail loans. Nowhere in the passage is it 177. (c) 178. (d) 179. (d) 180. (c)
mentioned that the government has relieved the 181. (a) 182. (a) 183. (c) 184. (a)
farmers of the loans. 185. (c) 186. (b) 187. (e) 188. (a)
165. (d) It is mentioned in the last paragraph. 189. (c) 190. (c) 191. (c) 192. (e)
166. (d) 193. (b) 194. (a) 195. (d) 196. (c)
167. (e) 197. (e) 198. (b) 199. (d) 200. (d)
y
o
u
rs
m
a
h
b
o
IBPS RRBs OFFICER SCALE-II & III CWE

o
b
.w
EXAM 2013

o
rd
p
Based on Memory

re
s
s
.c
o
m
8. Four of the following five are alike in a certain way and
R EASONING ABILITY so form a group. Which is the one that does not belong
to that group ?
1. Among P, Q, R, S, T and U, R is taller than only P and (a) Cuckoo (b) Crow
U. S is shorter than only T and Q. If each of them has a (c) Bat (d) Parrot
different height, who among them will be the third from (e) Sparrow
top when they are arranged in descending order of their 9. How many such pairs of letters are there in the word
height ? TERMINATE each of which has as many letters between
(a) R (b) P them in the word as in the English alphbet ?
(c) S (d) Q (a) None (b) One
(e) None of these (c) Two (d) Three
2. Vikas walked 10 metres towards North, took a left turn and (e) More than three
walked 15 metres and again took a left turn and walked
10 metres and stopped walking. Towards which direction DIRECTIONS (Qs. 10): In each of the questions below are
was he facing when he stopped walking ? given three statements followed by three conclusions numbered
(a) South (b) South-West I, II & III. You have to take the given statements to be true
(c) South-East (d) Cannot be determined even if they seem to be at variance from commonly known
(e) None of these facts. Read all the conclusions and then decide which of the
3. How many such 7s are there in the following number given conclusions logically follows from the given statements
sequence each of which is immediately preceded by 5 and disregarding commonly known facts.
not immediately followed by 8?
235789345761935748325788925 10. If it is possible to make only one meaningful English word
(a) None (b) One with the second, the fifth, the sixth and the tenth letters
(c) Two (d) Three of the word STREAMLINE, which of the following will be
(e) More than three the third letter of that word ? If no such word can be
4. If it is possible to form a number with the first, the fourth made, give ‘X’ as the answer and if more than one such
and the seventh digits of the number 4671358, which is
word can be made, give ‘Y’ as the answer.
the perfect square of a two-digit odd number, which of the
(a) E (b) A
following will be the digit in the tenth place of that two
digit odd number ? If no such number can be formed, give (c) M (d) X
‘O’ as the answer and if more than one such number can (e) Y
be made, give ‘X’ as the answer. DIRECTIONS (Qs. 11-16): In each of the questions below are
(a) 2 (b) 9 given three statements followed by three conclusions numbered
(c) 3 (d) O
I, II & III. You have to take the given statements to be true
(e) X
even if they seem to be at variance from commonly known
5. Among P, Q, R, S and T, Q is younger than only S and
R and older than T. Who among them is the oldest ? facts. Read all the conclusions and then decide which of the
(a) S (b) R given conclusions logically follows from the given statements
(c) P (d) Data inadequate disregarding commonly known facts.
(e) None of these
11. Statements :
6. Pointing to a boy, Meena said “He is the only grandson
Some blades are papers.
of my grandfather”. How is the boy related to Meena ?
(a) Brother (b) Cousin Some papers are books.
(c) Uncle (d) Data inadequate Some books are pens.
(e) None of these Conclusions :
7. Four of the following five are alike in a certain way and I. Some pens are papers.
so form a group. Which is the one that does not belong II. Some books are blades.
to that group ? III. Some pens are blades.
(a) Fruit (b) Flower (a) Only I follow (b) Only II follow
(c) Leaf (d) Petal (c) Only III follow (d) None follow
(e) Tree (e) Only II and III follow
y
o
u
rs
m
202 IBPS RRBs Officer Scale-II & III CWE Exam 2013

a
h
12. Statements :

b
DIRECTIONS (Qs. 17 - 20) : Study the following information

o
Some pencils are marbles.

o
carefully and answer the given questions :

b
All marbles are buses.

.w
Some buses are trucks. A word and number arrangement machine when given an input
line of words and number rearranges them following a

o
Conclusions :

rd
I. Some trucks are pencils. particular rule in each step. The following is an illustration of

p
II. Some buses are pencils. input and rearrangement.

re
III. No truck is pencil.

s
Input : 51 pour 32 start now 23 46 house

s
(a) Only I follow

.c
Step I : 23 51 pour 32 start now 46 house

o
(b) Only II follow Step II : 23 start 51 pour 32 now 46 house Step

m
(c) Only either I or III and II follow Step III : 23 start 32 51 pour now 46 house Step
(d) Only either I or III follow
Step IV : 23 start 32 pour 51 now 46 house Step
(e) None of these
13. Statements : Step V : 23 start 32 pour 46 51 now house Step
Some trees are jungles. Step VI : 23 start 32 pour 46 now 51 house As per the
Some jungles are flowers. rules followed in the above steps, find out in each of the
All flowers are streets. following questions the appropriate step for the given
Conclusions: input.
I. Some streets are jungles. 17. Step II of an input is : 18 task bear cold dish 81 63 31 How
II. Some streets are trees. many more steps will be required to complete the
III. Some flowers are trees. rearrangement ?
(a) Only I follow (a) Three (b) Four
(b) Only II follow (c) Five (d) Six
(c) Only III follow (e) None of these
(d) Only I and II follow 18. Input: nice flower 34 12 costly height 41 56
(e) None of these
Which of the following will be step III ?
14. Statements :
All desks are tables. (a) 12 nice 34 height flower costly 41 56
All tables are chairs. (b) 12 nice 34 height 41 flower costly 56
Some chairs are sofas. (c) 12 nice 34 flower costly height 41 56
Conclusions: (d) 12 nice flower 34 costly height 41 56
I. Some sofas are desks. (e) None of these
II. Some chairs are desks. 19. Step II of an input is : 16 victory 19 36 53 store lake town
III. Some tables are desks. Which of the following will be step V ?
(a) Only I and II follow (a) 16 victory 19 town store 36 53 lake
(b) Only II and III follow (b) 16 victory 19 town 36 store 53 lake
(c) Only I and III follow (c) 16 victory 19 town 36 53 store lake
(d) All follow (d) There will be no such step
(e) None of these
(e) None of these
15. Statements :
Some cycles are bikes. 20. Input : milk pot 18 24 over goal 36 53
No bike is flower. Which of the following steps will be the last but one
All flowers are goats. (a) VI (b) V
Conclusions: (c) VII (d) VIII
I. No goat is cycle. (e) None of these
II. Some flowers are cycles. DIRECTIONS (Qs. 21- 25) : Study the following information
III. Some goats are bikes. carefully and answer the questions below :
(a) None follow
(b) Only I follow P, Q, R, S, T, V and W are seven friends working in a call
(c) Only II follow centre. Each of them has different day off in a week from
(d) Only III follow Monday to Sunday not necessarily in the same order. They
(e) Only II and III follow work in three different shifts I, II and III with at least two of
16. Statements : them in each shift.
All rivers are hills. R works in shift II and his day off is not Sunday. P’s day off
All hills are rocks. is Tuesday and he does not work in the same shift with either
Some rocks are sticks. Q or W. None of those who work in shift I has day off either
Conclusions:
on Wednesday or on Friday. V works with only T in shift III.
I. Some sticks are hills.
II. Some sticks are rivers. S’s day off is Sunday. V’s day off is immediate next day of that
III. Some rocks are rivers. of R’s day off. T’s day off is not on Wednesday. W’s day off
(a) None follow (b) Only I follow is not on the previous day of P’s day off. S works in shift I.
(c) Only II follow (d) Only III follow Q does not work in the same shift with R and his day off is
(e) Only II and III follow not on Thursday.
y
o
u
rs
m
IBPS RRBs Officer Scale-II & III CWE Exam 2013 203

a
h
21. Which of the following is W’s day off ? Assumptions:

b
o
(a) Tuesday (b) Monday I. The customers may totally stop buying vegetables

o
b
(c) Saturday (d) Data inadequate at higher prices.

.w
(e) None of these II. The customers may still buy vegetables from the

o
22. Which of the following is R’s day off ? retail vendors.

rd
(a) Friday (b) Thursday 30. Statement : A large number of students and parents

p
(c) Tuesday (d) Wednesday

re
stood in the queue to collect forms for admission to
(e) None of these

s
various undergraduate courses in the college.

s
23. Which of the following groups of friends work in shift II?

.c
Assumptons :

o
(a) RP (b) RV I. The college authority may be able to admit all those

m
(c) QWS (d) Data inadequate who stood in the queue.
(e) None of these II. The college authority may have adequate number of
24. Which of the following is Q’s day off ? forms for all those standing in the queue.
(a) Friday (b) Wednesday
(c) Thursday (d) Monday DIRECTIONS (Qs. 31-38): In the following questions, the
(e) None of these symbols $, @, ©, % and # are used with the following
25. Which of the following groups of friends work in shift I? meaning as illustrated below :
(a) RV (b) RP ‘P $ Q’ means ‘P is not smaller than Q’
(c) QWS (d) Data inadequate ‘P @ Q’ means ‘P is not greater than Q’
(e) None of these ‘P © Q’ means ‘P is neither greater than nor equal to Q’
DIRECTIONS (Qs. 26 - 30): In each question below is given ‘P % Q’ means ‘P is neither smaller than nor equal to Q’
a statement followed by two assumptions numbered I and II. ‘P # Q’ means ‘P is neither greater than nor smaller than Q’
An assumption is something supposed or taken for granted. Now in each of the following question assuming the given
You have to consider the statement and the following statements to be true, find which of the conclusions I, II and
assumptions and decide which of the assumptions is implicit III given below them is/are definitely true and give your
in the statement. answer accordingly.
Give answer (1) if only Assumption I is implicit. 31. Statements :
Give answer (2) if only Assumption II is implicit. M © T, T @ J, J # D
Give answer (3) if either I or II is implicit. Conclusions :
Give answer (4) if neither I nor II is implicit. I. D # T
Give answer (5) if both I and II are implicit. II. D % T
26. Statement : A nationalised bank issued an advertisement III. D % M
in the national dailies asking the eligible candidates for (a) All are true
applying for 100 posts of chartered accountants. (b) Only I is true
Assumptions : (c) Only II is true
I. The eligible chartered accountants may respond to (d) Only either I or II is true
the advertisement (e) Only either I or II and III are true
II. There may be adequate number of eligible chartered 32. Statements :
accountants who may want to join a nationalized H $ J, J © M, M @ T
bank. Conclusions :
27. Statement : The municipal authority announced before I. H % M
the onset of monsoon that the roads within the city will II. H $ T
be free of potholes during monsoon. III. T % J
Assumptions: (a) Only I is true
I. The roads were repaired so well that potholes may (b) Only III is true
not reappear. (c) Only II is true
II. People may not complain even if the potholes (d) Only I and II are true
reappear.
(e) None of these
28. Statement : “Our Europe Holiday Package costs less
than some of the holiday Packages within the country” 33. Statements :
- An advertisement by an Indian travel company. R @ N, N % E, E # K
Assumptions: Conclusions:
I. People may prefer to travel to foreign destinations I. R © K
than to the places within the country at comparable II. K % N
cost.
III. E % R
II. People generally take their travel decisions after
getting information from such advertisements. (a) None is true
29. Statement : The retail vegetable vendors increased the (b) Only I is true
prices of vegetables by about 20 percent due to non (c) Only II is true
availability of vegetables at lower prices at the wholesale (d) Only III is true
market. (e) Only II and III are true
y
o
u
rs
m
204 IBPS RRBs Officer Scale-II & III CWE Exam 2013

a
h
34. Statements : Mark answer (3) if the case is to be referred to General

b
o
K % N, N $ B, B © D Manager-Finance.

o
b
Conclusions : Mark answer (4) if the case is to be referred to President-

.w
I. D % N Finance.

o
Mark answer (5) if the candidate is to be selected.

rd
II. K % D

p
III. B © K 36. Geeta Kothari was born on 10th September 1980. She has

re
(a) None is true been working in the Finance Department of an organiza

s
tion for the past four years after completing her MBA

s
(b) Only II is true

.c
with Finance specialisation. She has secured 50 percent

o
(c) Only III is true marks in the selection process.

m
(d) Only I is true 37. Ravi Sharma has secured 60 percent marks in graduation.
(e) Only II and III are true He has been working in the Finance Department of an or
35. Statements : ganization for the past four years after completing his
T # A, A $ B, B @ D MBA in Finance with 75 percent marks. He was born on
Conclusions : 25th May 1983. He has secured 39 percent marks in the
selection process.
I. D # A
38. Ranjit Malhotra has secured 55 percent marks in graduation
II. D # T and 65 percent marks in MBA Finance. He has been
III. B @ T working as Deputy Manager Finance in an organization
(a) None is true for the past four years after completing his MBA. He was
(b) Only I is true bom on 24th February 1985. He has secured 60 percent
(c) Only II is true marks in the selection process.
(d) Only III is true 39. Deepak Banerjee was bom on 8th July 1977. He has
secured 65 ercent marks in both graduation and MBA-
(e) Only II and III are true
Finance. He has also secured 70 percent marks in the
DIRECTIONS (Qs. 36-40): Study the following information selection process. He has been working in the Accounts
carefully and answer the questions given below : Department of an organization for the past three years
after completing his MBA.
Following are the conditions for selecting Manager-Finance in
40. Sanket Chopra has been working as Deput Manager-
an organisation :
Finance in an organization for the past four years after
The candidate must — completing graduation with 65 percent marks. He has
(i) be a graduate in any discipline with at least 50 also done a diploma in Finance Management. He has
percent marks. secured 55 percent marks in the selection process. He
(ii) be a postgraduate in Management with specialisation was born on 3rd April 1983.
in Finance.
(iii) be at least 25 years and not more than 35 years as
on 1.2.2013.
QUANTITATIVE APTITUDE & DATA
(iv) have post qualification work experience of at least INTERPRETATION
two years in the Accounts/Finance department of an
organization DIRECTIONS (Qs. 41-45): What will come in place of the
(v) have secured at least 40 percent marks in the question mark (?) in the following series ?
selection process.
41. 12 22 69 272 1365 ?
In the case of a candidate who satisfies all other criteria EXCEPT (a) 8196 (b) 8184
(A) at (ii) above, but has worked as Deputy Manager- (c) 8195 (d) 6830
Finance in an organization for at least three years, (e) None of these
his/her case is to be referred to General Manager- 42. 1 ? 27 64 125
Finance. (a) 14 (b) 4
(B) at (v) above, but has secured at least 70 percent (c) 9 (d) 8
marks in postgraduation, his/her case is to be (e) None of these
referred to President-Finance. 43. 104 153 189 214 ?
In each question below, detailed information of one (a) 239 (b) 225
candidate is provided. You have to take one of the (c) 264 (d) 235
following courses of action based on the information (e) None of these
provided and the conditions and sub-conditions given 44. 15 17 32 49 81 130 ?
above and mark your answer accordingly. You are not to (a) 179 (b) 211
assume anything other than the information provided in (c) 194 (d) 226
case of each candidate. All these cases are given to you (e) None of these
as on 1.2.2013 45. 15 17 21 29 45 77?
Mark answer (1) if the candidate is not to be selected. (a) 109 (b) 125
Mark answer (2) if the data provided are not adequate (c) 141 (d) 173
to take a decision. (e) None of these
y
o
u
rs
m
IBPS RRBs Officer Scale-II & III CWE Exam 2013 205

a
h
Directions (Qs. 46-50): From the following, different committees 48. A Committee is 5 in which 2 men teachers, 2 women teach

b
o
are to be made as per the requirement given in each question. ers and 1 doctor are there

o
b
In how many different ways can it be done ? (a) 75 (b) 150

.w
10 men and 8 women out of which 5 men are teachers, 3 men (c) 214 (d) 20

o
doctors and businessmen. Among the women, 3 are teachers, (e) None of these

rd
2 doctors, 2 researchers and 1 social worker. 49. A Committee of 7.

p
re
46. A Committee of 5 in which 3 men and 2 women are there (a) 31824 (b) 1200

s
(a) 3360 (b) 8568 (c) 9600 (d) 15912

s
.c
(c) 4284 (d) 1680 (e) None of these

o
(e) None of these 50. A Committee of 3 in which there is no teacher and no

m
47. A Committee of 4 in which at least 2 women are there doctor
(a) 1260 (b) 1820 (a) 100 (b) 120
(c) 3060 (d) 1890 (c) 10 (d) 12
(e) None of these (e) None of these

DIRECTIONS (Qs. 51-55) : Study the following table carefully answer the questions given below :
Number of books of different prices bought over the months.

Months January March May July September November


Price
More than `5,000 50 106 2 30 25 75
`4,000 - `5,000 105 1000 40 105 400 375
`3,000 - `3,999 70 100 80 115 200 240
`2,000 - `2,999 300 500 100 216 135 300
`1,000 - `1,999 140 370 200 225 175 470
`500 - `999 200 700 15 400 75 530
Less than `500 65 135 111 118 25 65

51. In which month maximum number of books were bought? 57. The simple interest accrued on an amount of ` 16,500 at
(a) March (b) November the end of three years is ` 5,940. What would be the
(c) July (d) September compound interest accrued on the same amount at the
(e) None of these same rate in the same period? (rounded off to two digits
52. In which price range maximum books were bought in the after decimal)
given months taken together?
(a) ` 500 - ` 999 (b) ` 2000 - ` 2999 (a) ` 6681.31 (b) ` 6218.27
(c) ` 1000 - ` 1999 (d) ` 3000 - ` 3999 (c) ` 6754.82 (d) ` 6537.47
(e) None of these (e) None of these
53. In the price range of ` 1000 - ` 1999 the number of books 58. Prema decided to donate 15% of her salary to an
bought in January is what percent of the number of orphanage. On the day of donation she changed her
books bought in May in the same price range ?
(a) 30 (b) 70 mind and donated ` 1,896 which was 80% of what she
(c) 142.86 (d) 60 had decided earlier. How much is Prerna’s salary?
(e) None of these (a) ` 18,500 (b) ` 10,250
54. What is the difference between the number of books (c) ` 15,800 (d) Cannot be determined
bought in September and November ? (e) None of these
(a) 244 (b) 776
(c) 1020 (d) 1310 59. If the numerator of a fraction is increased by 600% and
(e) None of these the denominator is increased by 200%, the resulting
55. What is the ratio between the number of books in price
range ` 4000 - ` 5000 bought in January and March ? 4
(a) 1 : 10 (b) 15 : 100 fraction is 2 . What was the original fraction?
5
(c) 930 : 2911 (d) 21 : 200
(e) None of these
56. Prabha started a business investing ` 32,000. After 5 4 13
(a) (b)
months Amit joined her with a capital of ` 22,000. At the 7 12
end of the year the total profit was ` 16,409. What is
Prabha’s share in the profit ? 11 6
(a) ` 5,284 (b) ` 11,712 (c) (d)
(c) ` 10,182 (d) ` 4,697 12 5
(e) None of these (e) None of these
y
o
u
rs
m
206 IBPS RRBs Officer Scale-II & III CWE Exam 2013

a
h
60. The present ages of Trisha and Shalini are in the ratio of 67. What is the approximate average number of candidates

b
o
7 : 6 respectively.  After 8 years the ratio of their ages appearing for the Entrance Test from all the cities

o
b
will be 9 : 8. What is the difference in their ages ? together in the year 2007?

.w
(a) 4 years (b) 8 years (a) 2800000 (b) 2901000

o
(b) 2755000 (d) 2795000

rd
(c) 10 years (d) 12 years
(e) 2917000

p
(e) None of these

re
68. The number of candidates appearing for the Entrance

s
DIRECTIONS (Qs. 61-65) : What approximate value should Test from city B in the year 2008 is approximately what

s
.c
come in place of the question mark (?) in the following per cent of the number of candidates appearing for the

o
questions ? (Note : You are not expected to calculate the Entrance test from city C in the same year?

m
exact value.) (a) 75 (b) 112
(c) 124 (d) 133
61. 839.999 ÷ 48.007 = ?
(e) 97
(a) 9.5 (b) 23.5 69. What is the respective ratio of the number of candidates
(c) 11.5 (d) 28.5 appearing for the Entrance Test from cities A, B and C
(e) 17.5 together in the year 2007 to the number of candidates
62. 8000 = ? appearing for the Entrance Test from cities D, E and F
(a) 76 (b) 89 together from the same year ?
(c) 65 (d) 97 (a) 18 : 17 (b) 13 : 14
(c) 17 : 18 (d) 14 : 13
(e) 58
(e) None of these
63. 18.001 × 22.999 × 9.998 = ? 70. The number of candidates appearing for the Entrance
(a) 4560 (b) 6290 Test from city E in the year 2008 is what per cent of the
(c) 4140 (d) 5920 total number of candidates appearing for the Entrance
(e) 5080 Test from all the cities together in that year (rounded off
64. 99999 ÷ 99 ÷ 9 = ? to two digits after decimal)?
(a) 17.14 (b) 15.43
(a) 112 (b) 211
(c) 20.31 (d) 14.49
(c) 121 (d) 221 (e) None of these
(e) 222
65. 7984.986 + 2541.005 + 127.996 = ? DIRECTIONS (Qs. 71-75) : Study the table carefully to
answer the questions that follow.
(a) 11280 (b) 15600
(c) 14650 (d) 10654 Total number of 24500 people who are engaged in given
(e) 16500 professions and (of these) percentage of females and males

DIRECTIONS (Qs. 66-70) : Study the following graph Professions Percentage Percentage Percentage
carefully to answer the questions. of People of Females of Males
Number of candidates Appearing for an Entrance Test from Medical 11 60 40
Engineering 18 30 70
Various Cities (Number in lakhs) Law 24 45 55
Teaching 21 80 20
2007 2008 Banking 16 35 65
Management 10 44 56
40
71. What is the respective ratio of the total males in Medical
Number of Candidates

35
30 and Teaching profession together to the total number of
25 females in the same professions together ?
(a) 117 : 43 (b) 29 :183
20
(c) 183 : 29 (d) 43:117
15 (e) None of these
10 72. The total number ef people in Teaching profession is
5 what per cent ef the total number ef people in law
profession?
A B C D E F (a) 87.5 (b) 93
City (c) 68 (d) 79.5
(e) None of these
66. What is the respective ratio of the number of candidates 73. What is the total number of males from all the professions
appearing for the Entrance Test from city A and city D together ?
in the year 2008 ? (a) 11472 (b) 12784
(a) 11 : 12 (b) 13 : 11 (c) 12348 (d) 12453
(c) 12 : 11 (d) 11 : 13 (e) None of these
(e) None of these
y
o
u
rs
m
IBPS RRBs Officer Scale-II & III CWE Exam 2013 207

a
h
74. Females in Engineering profession are what per cent of

b
GERNAL AWARENESS

o
the males in Management profession? (Rounded off to

o
b
two digits after decimal) 81. The Foreign Exchange Reserves of India are kept in the

.w
(a) 71.71 (b) 96.43 custody of

o
(c) 83.16 (d) 68.54

rd
(a) World Bank
(e) None of these

p
(b) International Monetary Fund

re
75. What is respective ratio of the number of males in (c) Prime Minister Rahat Kosh

s
Banking profession to the number of males in Engineering (d) Reserve Bank of India

s
.c
profession? (e) None of these

o
(a) 17 : 7 (b) 28 : 55 82. The Reserve Bank of India issues coins and notes of

m
(c) 7 : 11 (d) 63 : 52 various denominations. At present RBI does not issue
(e) None of these coins of which of the following denominations ?
(a) 10 Rupees (b) 5 Rupees
DIRECTIONS (Qs. 76-80) : Study the following graph
(c) 25 paise (d) 2 Rupees
carefully to answer the questions that follow. (e) 1 Rupee
Total Number of Males and Females in Five Different 83. The European Union has adopted which of the following
Organisations as common currency ?
(a) Dollar (b) Dinar
Males Females (c) Yen (d) Peso
5000 (e) Euro
4500 84. Various Banks in the country have installed machines
Number of People

4000 which disburse money to general public. These machines


3500 are called
3000 (a) Coin dispensing machines
2500 (b) ATMs
2000 (c) Debit Card Machines
1500 (d) Ledger Machines
1000 (e) None of these
500 85. Which of the following names is NOT associated with
0 the insurance business in India ?
A B C D E (a) Bajaj Allianz (b) LIC
Organisations (c) GIC (d) Tata AIG
(e) GE Money
76. What is the average number of females from all the
86. Commodity Exchanges at various places in India are
organisations together ? trading in many metals like gold, silver, etc. Many times
(a) 3800 (b) 3550 we read in newspapers that gold no more has remained
(c) 3300 (d) 3150 the highest trading option and has been rcplaced by a
(e) None of these non traditional metal known as
77. The number of males from organisation A is approximately (a) copper (b) steel
what per cent of the total number of males from all the (c) zinc (d) white gold
organisations together ? (e) None of these
(a) 18 (b) 28 87. Which of the following is known as plastic money ?
(c) 11 (d) 31 (A) Demand Draft
(e) 36 (B) Credit Card
78. What is the difference between the total number of (C) Debit Card
females and the total number of males from all the (a) Only (A) (b) Only (B)
organisations together ? (c) Only (C) (d) Both (B) and (C)
(a) 1500 (b) 1750 (e) All (A), (B) and (C)
(c) 1800 (d) 2050 88. Many a times we see a term in newspapers ‘IPO’. What
(e) None of these is the full form of the same ?
79. What is the respective ratio of number of females from (a) Indian Public Offer
organisation C to the number of females from organisation (b) Institutional Purchase Offer
E? (c) Industrial Purchase Offer
(a) 14 : 17 (b) 17 : 14 (d) Indian Purchase Offer
(c) 14 : 15 (d) 15 : 14 (e) Initial Public Offer
(e) None of these 89. Many a times we read in financial newspapers a term/
80. The total number of males from organisations A and B name NMCEX. What is the full form of the same?
together are approximately what per cent of the total (a) New Multi Capital Exchange
number of males from organisations C, D and E together? (b) National Medium Commodity Exchange
(a) 58 (b) 75 (c) National Multi Commodity Exchange
(c) 69 (d) 83 (d) Net Marketable Commodity Exchange
(e) 52 (e) None of these
y
o
u
rs
m
208 IBPS RRBs Officer Scale-II & III CWE Exam 2013

a
h
90. Many Banks these days are entering into business of (C) Providing insurance cover to each and every citizen

b
o
offering loans against property. This business of the so that he/she can live a healthy and long life.

o
b
banks can be categorised under which of the following (a) Only (A) (b) Only (B)

.w
heads of banking? (c) Only (C) (d) Both (A) and (B)

o
(a) Corporate Banking (e) All (A), (B) and (C)

rd
(b) Personal Banking 99. Which of the following organisations recently issued

p
re
(c) Merchant Banking some guidelines related to ‘Participatory Notes’ as used

s
(d) Portfolio Management Service in financial world ?

s
.c
(e) None of these (a) RBI (b) IRDA

o
91. Which of the following organisations /banks has done a (c) SEBI (d) AMFI

m
commendable work in the field of micro finance and was (e) None of these
awarded Nobel Prize also in the past ? 100. As per news items published in various newspapers,
(a) Gramin Bank of Bangladesh Italy has offered its help in developing ‘SMEs’ in India.
(b) CRY What is the full form of ‘SME’?
(c) ASHA
(a) Small and Monopolistic Economy
(d) NABARD
(b) Small and Medium Enterprises
(e) None of these
(c) Speedy and Mechanical
92. Which of the following countries has not made any
(d) Small and Medium Level Economy
significant investment in India ?
(a) Japan (b) USA (e) None of these
(c) Nepal (d) Britain 101. Which of the following statements about the ‘exports’
(e) France from India is true ?
93. Which of the following is considered an informal method (A) Exports are showing good rate of growth.
of getting credit/finance ? (B) Exports are growing in volume but still they are
(a) Internet banking poorer than imports in terms of value.
(b) Branch visits (C) Exports are not improving the way they were
(c) Going to money lenders growing two years back.
(d) Tele Banking (a) Only (A)
(e) All of these (b) Only (B)
94. Which of the following is NOT a Public Sector Unit/ (c) Only (C)
Undertaking/Agency ? (d) Both (A) and (C)
(a) ECGC (b) SEBI (e) None of these
(c) SIDBI (d) Axis Bank 102. The Government of India keeps on clearing the proposal
(e) BHEL of setting up new SEZs at several places. What does the
95. Which of the following does not represent the name of letter ‘S’ represent in the ‘SEZ’?
a bank working in India ? (a) Soft (b) Small
(a) YES (b) HDFC (c) Special (d) Supportive
(c) TATA (d) Kotak Mahindra (e) None of these
(e) Axis 103. Which amongst the following is NOT a foodgrain ?
96. Which of the following is a type of tax levied by the (a) Wheat (b) Rice
Government on goods and services ? (c) Maize (d) Jowar
(a) SAT (b) NET (e) Cotton
(c) PAN (d) VAT 104. Various Government agenicies/organisations are given
(e) None of these responsibilities for implementing various policies/decisions
97. As per the news appeared in some major newspapers
of the Government of India. Policies about credit flow to
India is number two in getting ‘PE’. What is the full form
the agricultural/priority sector are framed/implemented
of ‘PE’ as used in the financial world?
through which of the following apex bodies ?
(a) Professional and Eco-friendly
(a) ECGC (b) NABARD
(b) Profitable Equity
(c) Profitable and Economical (c) UTI (d) IDBI Bank
(d) Private and Economical (e) None of these
(e) Private Equity 105. As we read now and then the Monetary and Credit
98. Many a times we read in newspapers about ‘Financial Policy reviewed and changes/corrections are made
Inclusion’. What does it really mean ? frequently.
[(Pick out correct statement(s)] Who amongst the following exactly takes this decision in
(A) Allow the merger and acquisition of Banks so that India ?
only few big banks exist and continue to cater to the (a) Deputy Chairman, Planning Commission
need of corporate sector. (b) Prime Minister
(B) Extending the network of banks in such away that (c) Finance Minister
people from lower strata of society also get the (d) Chairman, Finance Commission
benefit of services provided by banks. (e) None of these
y
o
u
rs
m
IBPS RRBs Officer Scale-II & III CWE Exam 2013 209

a
h
106. Which of the following is NOT a foreign bank working

b
(d) Chits and Money Lenders

o
in India ? (e) State Co-Operative Banks

o
b
(a) HSBC (b) Barclays 114. The term ‘Power of Attorney’ refers to

.w
(c) Standard Chartered (d) Yes Bank (a) Power of a person

o
(e) All are foreign banks

rd
(b) An authority to operate a Bank Account
107. Who amongst the following a the Chairman of the 14th

p
(c) An instrument by which a person is empowered to

re
Finance Commission ? act for another person

s
(a) Bimal Jalan (b) Y.V. Reddy

s
(d) All of these

.c
(c) C. Rangarajan (d) Vijay Kelkar
(e) None of these

o
(e) None of these

m
108. With which of the following nations India has the trade 115. What does BPLR stand for ?
relations? (a) Basic Priority Lending Rate
(a) Japan (b) Britain (b) Below Poverty Line Register
(c) South Africa (d) Yuganda (c) Benchmark Prime Lending Rate
(e) Iran (d) Base Primary Lending Rate
109. Your friend gets ` 10000 from his grand parents. He/she (e) None of these
wants to invest this amount in a bank in such a way that 116. Which of the following bodies/agencies decides the
he/she need not withdraw the amount partly or wholly distribution of tax income between Central & State
for two years. What kind of account should he/she open Governments in India?
in a bank? (a) Planning Commission
(a) Small Savings Accoun (b) Central Law Commission
(b) Current Account (c) Reserve Bank of India
(c) Term Deposit Account (d) Finance Commission
(d) Demat Account (e) Administrative Reforms Commission
(e) None of these 117. Who among the following is the chairman of the Tax
110. We often see the names of two personalities in
Administration Reform Commision (TARC) set up by the
newspapers/ magazines/electronic media. These names
Government of India on August 26,
are - Dr. C. Rangarajan and Dr. Bimal Jalan. In their lives,
one thing is common. That is 2013 to review the application of tax policies ?
(a) both are the chief economic advisor to Prime (a) Parthasarathi Shome
Minister of India (b) Y G Parande
(b) both are Members of Parliament (c) Sunita Kaila
(c) both are former Governors of RBI (d) M K Zutshi
(d) both were Finance Ministers of India (e) None of these
(e) there is nothing common in both 118. Which of the following steps was/were takes by the
111. According to the reports published in Newspapers/ Reserve Bank of India in July 2013 to suck liquidity and
magazines, Government of India and some NGOs have check the rupee’s slide?
announced/started many schemes to provide help/relief (a) The RBI raised the cost of borrowing by banks by
to farmers in general and the tragedy stricken farmers in two per cent to 10.25 per cent
particular. Which of the following cannot be considered (b) The RBI announced the sale of Government
such a scheme? Securities via open market operation
(a) Crop Insurance Scheme (c) The RBI fixed a cap on how much banks can borrow
(b) E-Chaupal
from the Central bank at 7.25 per cent using the repo
(c) Subsidy for chemical fertilisers
window
(d) Renovation of watershed/ waterbodies
(d) Only (a) and (b)
(e) Midday Meal Scheme
112. What does the term Depreciation mean as used in (e) All of the above
finance/banking operations? 119. Which of the following stock exchanges on July 5, 2013
(a) Closure of a plant due to lock out launched ‘LIX 15’ index for derivatives segment?
(b) Reduction in the value/ loss of equipment/plant (a) Bombay Stock Exchange (BSE)
over a time due to wear and tear (b) Delhi Stock Exchange (DSE)
(c) Loss incurred during a year due to plant breakdown (c) National Stock Exchange (NSE)
(d) Unusually high repair expenses incurred on the (d) Magadh Stock Exchange (MSE)
plant during the year (e) None of these
(e) None of these 120. The Annual Monetary Policy Statement 2013-14,
113. Which of the following cannot be called an organized announced on May 3, 2013 projected the Gross Domestic
sector in India ? Product growth for 2013-14 at
(a) Nationalised Banks (a) 5.3 per cent (b) 5.7 per cent
(b) Regional Rural Banks (c) 5.4 per cent (d) 5.5 per cent
(c) Co-operative Banks (e) None of these
y
o
u
rs
m
210 IBPS RRBs Officer Scale-II & III CWE Exam 2013

a
h
COMPUTER KNOWLEDGE 132. The term _____ designates equipment that might be

b
o
added to a computer system to enhance its functionality.

o
b
121. When you save to this, your data will remain intact even (a) digital device (b) system add-on

.w
when the computer is turned off _____ . (c) disk pack (d) peripheral device

o
(e) None of these

rd
(a) RAM
133. An email account includes a storage area, often called a

p
(b) motherboard
(n)

re
(c) secondary storage device (a) attachment (b) hyperlink

s
s
(d) primary storage device (c) mailbox (d) IP address

.c
(e) None of these

o
(e) None of these

m
122. The simultaneous processing of two or more programs 134. Data becomes _____ when it is presented in a format
by multiple processors is _____ . that people can understand and use.
(a) multiprogramming (b) multitasking (a) processed (b) graphs
(c) time-sharing (d) mult processing (c) information (d) presentation
(e) None of these (e) None of these
123. What type of device is a computer printer ? 135. A set of computer programs that helps a computer
(a) Input (b) Output monitor itself and function more efficiently is a/an _____ .
(c) Software (d) Storage (a) Windows (b) System Software
(e) None of these (c) DBMS (d) Application Software
124. The contents of _____ are lost when the computer turns (e) None of these
off. 136. A ____ is a collection of information saved as a unit.
(a) storage (b) input (a) folder (b) file
(c) path (d) file extension
(c) output (d) memory
(e) None of these
(e) None of these 137. Peripheral devices such as printers and monitors are
125. When you turn on the computer, the boot routine will considered to be
perform this test _____ . (a) hardware (b) software
(a) RAM test (b) disk drive test (c) data (d) information
(c) memory test (d) power-on-self-test (e) None of these
(e) None of these 138. Input, output, and processing devices grouped together
126. Personal computers can be connected together to form a represent a(n) _____ .
(a) server (b) supercomputer (a) mobile device
(c) enterprise (d) network (b) information processing cycle
(e) None of these (c) circuit board
127. Which elements of a Word document can be displayed (d) computer system
in colour? (e) None of these
(a) Only graphics 139. Most Web sites have a main page, the ____, which acts
(b) Only text as doorway to the rest of the Web site pages.
(c) All elements (a) search engine (b) home page
(d) All elements, but only if you have a colour printer (c) browser (d) URL
(e) None of these
(e) None of these
140. Which of the the following system components is the
128. A complete electronic circuit with transistors and other brain of the computer ?
electronic components on a small silicon chip is called a (a) Circuit board (b) CPU
(n) (c) Memory (d) Network card
(a) Workstation (b) CPU (e) None of these
(c) magnetic disk (d) integrated circuit 141. Which of the following is not true about computer files?
(e) None of these (a) They are collections of data saved to a storage
129. To access properties of an object, the mouse technique medium
to use is _____ . (b) Every file has a filename
(a) right-clicking (b) shift-clicking (c) A file extension is established by the user to
(c) dragging (d) dropping indicate the file’s contents
(e) None of these (d) All files contain data
130. What is the term for unsolicited e-mail ? (e) None of these
(a) newsgroup (b) usenet 142. allows voice conversations to travel over the Internet.
(c) backbone (d) flaming (a) Internet telephony (b) Instant messaging
(e) spam (c) E-mail (d) E-commerce
131. You use a(n) _____ , such as a keyboard or mouse, to (e) None of these
input information. 143. You use a(n) _____ , such as a keyboard or mouse, to
input information.
(a) output device (b) input device
(a) storage device (b) processing device
(c) storage device (d) processing device
(c) input device (d) output device
(e) None of these
(e) None of these
y
o
u
rs
m
IBPS RRBs Officer Scale-II & III CWE Exam 2013 211

a
h
144. Which of the following is not true concerning user IDs 155. Which of the following is not an integral part of

b
o
and passwords ? computer?

o
b
(a) When you enter your user ID and password, the (a) CPU (b) Mouse

.w
computer knows it is you (c) Monitor (d) UPS

o
(b) If your computer asks for a user ID and password, (e) None of these

rd
you can create your own 156. Which of the following is/are not part of the CPU ?

p
(c) Sometimes you are assigned a user ID and password

re
(a) Primary storage (b) Registers
for security reasons

s
(c) Control unit (d) ALU

s
(d) You should share your user ID and password with

.c
(e) None of these

o
at least one other person 157. The device that reconciles the differences between

m
(e) None of these computers and phones is the—
145. One who designs, writes, tests and maintains computer (a) LAN (b) wand reader
programs is called a (c) TCP/IP (d) scanner
(a) User (b) Programmer (e) modem
(c) Designer (d) Operator
158. A combination of hardware and software that allows
(e) None of these
communication and electronic transfer of information
146. Items such as names and addresses are considered
(a) information (b) input between computers is a _____.
(c) records (d) data (a) Network (b) Backup system
(e) None of these (c) Server (d) Peripheral
147. Most mail programs automatically complete the following (e) Modem
two parts in an e-mail 159. Which of the following represents the fastest data
(a) From : and Body : (b) From : and Date : transmission speed ?
(c) From : and To : (d) From : and Subject : (a) bandwidth (b) gbps
(e) None of these (c) kbps (d) kbps
148. Where is the disk put in a computer ? (e) mbps
(a) in the modem (b) in the hard drive 160. What is a major feature of the World Wide Web that
(c) into the CPU (d) in the disk drive makes it simple to learn and use ?
(e) None of these (a) Database interface
149. The name a user assigns to a document is called a(n): (b) Graphical text interface
(a) filename (b) program (c) Graphical user interface
(c) record (d) data (d) Point-to-Point Protocol
(e) None of these (e) None of these
150. An e-mail address typically consists of a user ID
followed
by the _____ sign and the name of the e-mail server that ENGLISH LANGUAGE
manages the user’s electronic post office box.
(a) @ (b) # DIRECTIONS (Qs. 161-170) : Read the following passage
(c) & (d) * carefully and answer the questions given below it. Certain
(e) None of these words have been printed in bold to help you locate them
151. A personal computer is designed to meet the computing while answering some of the questions.
needs of a(n) Crude oil has had a long history, and an interesting one.
(a) individual (b) department It is probably one single natural resource that has been
(c) company (d) city instrumental in producing maximum conflicts and wars during
(e) None of these the last century. With such a track record, it is sur- prising that
152. Devices that enter information and let you communicate
in the given environment, wherein oil prices are breaking all
with the computer are called
records, and has become the biggest cause of concern across
(a) Software (b) Output devices
(c) Hardware (d) Input devices the globe, why is it that there is silence from some parts of the
(e) Input /Output devices world? It is intriguing especially when some have been paying
153. An electronic device, operating under the control of a much higher price on fuel, as compared to their poorer
information, that can accept data, process the data, counterparts. It is surprising, more so, when one realizes that
produce output and store the results for future use the number of oil producing nations is only a handful and
(a) Input (b) Computer compared to the number of oil consuming nations. While a
(c) Software (d) Hardware proactive action from the mighty and powerful can bring in a
(e) None of these huge respite for the world, why is it that they choose to be
154. What is the function of the Central Processing Unit of a silent? Also, keeping in mind the fact that a few nations - both
Computer ? powerful and weak have some of the largest unused oil
(a) Creates invoices reserves, their silence ‘and lack of any serious action look at
(b) Performs calculations and processing the more aberrant. Looking at ‘the manner things are shaping
(c) Deletes Data up, it just cannot be ruled out that possibly these nations have
(d) Corrupts the data a definite interest in the increasing prices of oil. The
(e) None of these question then is what could their underlying interest be ?
y
o
u
rs
m
212 IBPS RRBs Officer Scale-II & III CWE Exam 2013

a
h
In all probability, what we are seeing right now is just a 163. What does the author mean by ‘dangling the carrot’ in

b
o
precursor to the larger picture, well laid out by them may be the passage ?

o
b
for the good. With rising prices, the OPEC might be delighted (a) Some countries are stringently opposing the

.w
right now, but perhaps they are unable to see what is going conversion to alternate forms of fuel

o
to hit them. For this unprecedented rise in oil is creating a most (b) Some countries have been luring other countries to

rd
demanding environment for alternative fuels. As and when the change over the alternate fuels in order to boost

p
re
reserves deplete and demand really grows, more than anyone their own business

s
else, it is OPEC that would be badly hit, much to the glee of (c) Some countries are making the effort to save

s
.c
some nations having unused reserves which have the most to environment by using natural gas instead of oil as

o
m
gain by using blackmailing tactics. Eventually, the countries at a fuel
the receiving end would be forced to create an enabling (d) Both (a) and (b)
environment for alternative energy. This trend is already (e) None of these
visible, with India showing the way with its nuclear deal and 164. Why, according to the author, OPEC though delighted
other countries dangling the carrot of complete conversion of currently, would be in a difficult situation later ?
natural gas towards other countries in order to boost their own (a) All the oil reserves on the earth will soon be exhausted
business. By doing so they are creating a tactical pressure on (b) Powerful nations will try and dominate OPEC later on
countries. As they know that the further the oil prices get (c) As the oil reserves which are being used currently
pushed, the more the globe would become attracted and ready deplete in the time to come, nations having unused
for alternative and non-conventional fuel. Moreover, for many oil reserves would arm-twist OPEC
of the countries, it might become more expensive to invest in (d) Nations will lose faith in OPEC owing to very high
newer technologies to conform to the emission norms than to fuel prices
shift to alternative sources, making the market even more (e) None of these
attractive. And it is then that the organisations which have 165. What, according to the author, makes the market of
already invested billions of dollars in alternate fuels would alternate sources very attractive?
mop up the global energy market completely. (a) Many countries have already made a substantial
The second possible reason why some nations of the profit in the industry alternate fuel
world are not too enthusiastic to mitigate the price of oil is (b) Such measures do not cause harm to the environment
because the rising oil price is perhaps one of the major (c) Only a few countries of the world will have to
deterrents to growth of other countries. Perhaps they were access to alternate forms of fuel
hand in glove in the mechanism to raise the oil price and (d) The use of alternate sources has been forced upon
knowing well that this would make these nations feel the certain countries by powerful countries
pinch. And now, the rising oil prices have put these nations (e) None of these
in a fix. As the pressure of inflationary tendencies increases 166. Which of the following can be the most appropriate title
these countries cannot afford either to sit back and remain for the given passage ?
entrapped in the larger conspiracy. No wonder then that these (a) Reasons behind silence on increase in oil prices
are frantically looking around the globe - especially Africa - for (b) Nuclear power as an alternate source of energy
newer reserves ! As it is said, oil has a long history and the (c) The monopoly of oil reserves by powerful countries
legacy continues. (d) The repercussion of shifting to alternate forms of
161. Which nations does the author refer to the phrase ‘these energy
nations have a definite interest in the increasing prices (e) Oil reserves as deterrent of growth of the nations
of oil’ ?
DIRECTIONS (Qs. 167-168) : Choose the word which is most
(a) The mighty and powerful nations
similar in meaning to the word printed in bold as used in the
(b) Nations having unused oil reserves
passage.
(c) Nations which have exhausted their oil reserves
(d) The nations which have shifted to non-conventional 167. INSTRUMENTAL
fuels (a) Non-vocal (b) Reasonable
(e) None of these (c) Creditable (d) Responsible
162. Which of the following is not true in context of the (e) Liable
passage? 168. ABERRANT
(a) Some nations have kept a silence upon the rising oil (a) Curious (b) Abnormal
price in order to inhibit the growth of other nations (c) Spoilt (d) Inferior
(b) It is more cost effective to shift to alternate forms of (e) Nonfunctional
energy than to invest in technology for conforming
to the emission norms Directions (Qs. 169-170) : Choose the word/phrase which is
(c) Some nations have unused oil reserves which would most opposite in meaning to the word printed in bold as used
earn these nations heavy profits once the oil in the passage.
reserves elsewhere deplete 169. PROACTIVE
(d) India has had a nuclear deal for its energy needs (a) Quick (b) Over powered
(e) All are true
y
o
u
rs
m
IBPS RRBs Officer Scale-II & III CWE Exam 2013 213

a
h
(c) Dormant (d) Delicate (e) No correction required

b
o
(e) Brittle

o
DIRECTIONS (Qs. 181-185): Rearrange the following six

b
170. MITIGATE sentences (A), (B), (C), (D), (E) and (F) in the proper

.w
(a) Fright (b) Decline sequence to form a meaningful paragraph and then answer

o
(c) Degeneration (d) Worsen

rd
the questions which follow:
(e) Deprivation

p
(A) The only way in which this problem can be solved

re
DIRECTIONS (Qs. 171-175) : Read each sentence to find out
is by making artificial blood which has remained a

s
s
whether there is any grammatical error in it. The error, if any,

.c
distant dream for science.
will be in one part of the sentence. The number of that part

o
(B) Donation of blood is considered to be the most

m
is the answer. If there is no error, the answer is ‘e’. (Ignore
noble of all the charities.
the errors of punctuation, if any).
(C) If they succeed, it would be noted as one of the
171. No sooner did the students (a)/ seen the principal most important inventions in the history of mankind.
approach (b)/ than they ran (c)/ from the playground. (d)/ (D) This is because this donated magic potion can give
No error (e) life to another person in an emergency.
172. My elder sister and I am (a)/ interested in painting (b)/ (E) A group of scientists, however, has dedicated
and therefore have joined (c)/ the coaching classes. (d)/ themselves towards making this a reality.
No error (e) (F) A growing problem however is that the requirement
173. Only after a lot of persuasion (a)/ the illiterate villager for safe blood is increasing whereas the number of
allowed (b)/ himself’s girl child (c) / to study in the donors is decreasing.
school. (d)/ No error (e) 181. Which of the following sentence should be the FIRST
174. The need for alternate (a)/ sources of energy are (b)/ after rearrangement ?
essential to conserve (c)/ the environment. (d)/ No error (a) A (b) B
(e) (c) C (d) D
175. I have being working (a)/ in that organisation for five (e) E
years (b)/ but now I work (c)/ for a different company. 182. Which of the following sentence should be the SECOND
(d)/ No error (e) after rearrangement ?
DIRECTIONS (Qs. 176-180) : Which of the phrases (a), (b), (a) A (b) B
(c) and (d) given below each statement should replace the (c) C (d) D
phrase printed in bold in the sentence to make it grammatically (e) F
correct ? If the sentence is correct as it is given and ‘No 183. Which of the following sentence should be the THIRD
correction is required’, mark (e) as the answer. after rearrangement ?
(a) A (b) D
176. Many students waits anxiously at the college gate to (c) C (d) E
know their results. (e) F
(a) student waited anxiouly 184. Which of the following sentence should be the FIFTH
(b) students waiting anxiously
after rearrangement ?
(c) students waited anxiously
(d) students waited anxious (a) A (b) B
(e) No correction required (c) C (d) E
177. Through a fortuitous circumstance Rakhi met her (e) F
childhood friend on the bus in which she was travelling. 185. Which of the following sentence should be the SIXTH
(a) she was travelled (b) she did travel (LAST) after rearrangement ?
(c) she has travelling (d) she were travelling (a) A (b) B
(e) No correction required (c) C (d) D
178. The opposition party has alleged that the prices of (e) E
essential commodities are soaring like never before on DIRECTIONS (Qs. 186-190) : Each question below has two
the last three decades. blanks, each blank indicating that something has been
(a) before on the next (b) before in the last
omitted. Choose the set of words for each blank that best fits
(c) before at the last (d) previously in the next
(e) No correction required the meaning of the sentence as a whole.
179. The salaries and the perks of the employees in this 186. One of the TV cameramen was ____ in the leg by a bullet
institution are not in according with the rest of the when he was ____ in the middle of a gun fight between
industry. two gangs.
(a) are not in accordance (b) is not in accordance
(a) hurt, entered (b) shot, caught
(c) are not according (d) is not on accordance
(e) No correction required (c) injured, came (d) stabbed, trapped
180. The soldiers deployed in the town were instructed to (e) beaten, engulfed
exercising restraint and handle the situation peacefully. 187. Although the fire was very small everyone ____ and
(a) was instructed to exercising rushed out of the cinema hall ____ complete chaos.
(b) were instructed for exercise (a) shouted, leading (b) died, producing
(c) were instructed to exercise (c) injured, resulting (d) scared, making
(d) was instructing to exercising
y
o
u
rs
m
214 IBPS RRBs Officer Scale-II & III CWE Exam 2013

a
h
(e) panicked, causing small town. A small handpainted sign which greets visitors

b
o
188. The______of pesticides and fertilizers while growing the outside Bekoji (199) ‘‘Welcome to the Village of Athletes”.

o
b
vegetables and fruits is one of the greatest______to the Children here start running at an early age, (200) great

.w
health these days. distances to fetch water and firewood or to reach school. At

o
(a) production, dangers (b) consumption, problem the Olympics, runners from this small town are likely to win

rd
(c) overuse, threats (d) application, rewards more medals than those from developed countries. It will give

p
re
(e) amount, injury their families a way out of poverty.

s
189. The group’s mission is to peace between the nations and 191. (a) accepted (b) common

s
.c
help different people______each other better. (c) alike (d) similar

o
(a) maintain, understand (b) remain, address (e) popular

m
(c) keep, interact (d) advise, find 192. (a) alleviate (b) forgot
(e) communicate, friendly (c) prevent (d) reduce
190. History ___ the Kings and influential leaders whereas (e) leave
the real battle was ____ by the soldiers who remain little 193. (a) changes (b) arrange
known and celebrated in the texts. (c) control (d) transform
(a) remembers, played (b) highlights, underwent (e) shift
(c) biased, lead (d) glorifies, fought 194. (a) further (b) more
(e) writes, done (c) greater (d) over
DIRECTIONS (Qs. 191-200) : In the following passage there (e) larger
are blanks, each of which has been numbered, These numbers 195. (a) concentrated (b) rival
are printed below the passage and against each, five words (c) focused (d) playful
are suggested, one of which fits the blank appropriately. Find (e) performed
out the appropriate word in each case. 196. (a) convince (b) optimist
Without doubt there is one thing (191) to all of us we have (c) intended (d) privilege
played a game at some time in our lives. Most of us play to (e) cenfident
relax or have fun, but for many playing a game or a sport is 197. (a) boast (b) suspicion
a way to (192) poverty behind. Infact, in many African (c) risk (d) worship
countries, playing a sport professionally can (193) the lives of (e) precaution
a person’s entire family. 198. (a) existing (b) that
For example, in the small town of Bekoji, in Ethiopia (194) (c) comprising (d) consisting
than a hundred boys and girls can be seen running at dawn (e) for
everyday. Each of these youth is (195) and serious and their 199. (a) warn (b) inform
coach is (196) that one of them will be a world champion. This (c) notices (d) reads
seems like an idle (197) but it is virtually a guarantee in this (e) wish
small community (198) mainly farmers. Many of the fastest 200. (a) covering (b) driving
male and female distance runners in the world hail from this (c) measuring (d) following
(e) competing
y
o
u
rs
m
IBPS RRBs Officer Scale-II & III CWE Exam 2013 215

a
h
b
o
o
b
.w
o
rd
1. (c) R > P, U

p
Buses

re
T, Q > S

s
T, Q > S > R > P, U 12. (c) Trucks Pencils

s
Marbles

.c
2. (a) 15 m

o
N

m
10 m

10 m

W E OR

S Buses
Starting Piont
Trucks Marbles Pencils
3. (c) 5 7 8
Such combinations are :
576 ; 574 OR

4. (a) Buses
1 2 3 4 5 6 7
4 6 7 1 3 5 8 Marbles Pencils

841 = (29)2 Trucks


Digit at tenth place = 2
5. (d) S, R > Q > T Conclusions:
I. False
P ù
So no information about P from the given data. II. True ú Either Complementary pair
6. (d) The boy is only grandson of Meena’s grandfather. ú
û
However, there may be granddaughters of Meena’s
III. False
grandfather.
So only either I or III and II follow.
Therefore, the boy is either brother or cousin of
Meena. 13. (a) Streets
So no sufficient data is there.
7. (e) All others are parts of a tree.
8. (c) Except bat, all others are birds. Bat is a mammal. Flowers

9. (c) 20 5 18 13 9 14 1 20 5 Jungles
T E R M I N A T E Trees

10. (e) 1 2 3 4 5 6 7 8 9 10 Conclusions:


I. True
S T R E A M L I N E II. False
Meaningful words Þ MATE, MEAT, TEAM, TAME III. False
11. (d) Blades Books So only conclusion I follows.

14. (b) Chairs

Papers Tables
Pens
Conclusions: Desks
I. False Sofas
II. False
III. False
All the three premises are I-type.
Hence no conclusion follows OR
y
o
u
rs
m
216 IBPS RRBs Officer Scale-II & III CWE Exam 2013

a
h
b
continues till all the words and numbers are arranged

o
Chairs according to pattern.

o
b
17. (c) Step II : 18 task bear cold dish 81 63 31

.w
Tables
Step III : 18 task 31 bear cold dish 81 63

o
rd
Desks Step IV : 18 task 31 dish bear cold 81 63

p
Sofas Step V : 18 task 31 dish 63 bear cold 81

re
s
Step VI : 18 task 31 dish 63 cold bear 81

s
.c
Step VII : 18 task 31 dish 63 cold 81 bear

o
So five more steps are required.

m
Conclusions:
I. False 18. (a) So five more steps are required.
II. True Input : nice flower 34 12 costly height 41 56
III. True. Conversion of the first premise. Step I : 12 nice flower 34 costly height 41 56
Step II : 12 nice 34 flower costly height 41 56
15. (a) Goats Step III : 12 nice 34 height flower costly 41 56
19. (d) Step II : 16 victory 19 36 53 store lake town
Flowers
cycles bikes Step III : 16 victory 19 town 36 53 store lake
Step IV : 16 victory 19 town 36 store 53 lake
Step IV : is the last step.
OR
20. (b) Input : milk pot 18 24 over goal 36 53
Goats Step I : 18 milk pot 24 over goal 36 53
Step II : 18 pot milk 24 over goal 36 53
Flowers Step III : 18 pot 24 milk over goal 36 53
cycles bikes
Step IV : 18 pot 24 over milk goal 36 53
Step V : 18 pot 24 over 36 milk goal 53
Conclusions:
I. False Step VI : 18 pot 24 over 36 milk 53 goal
II. False So step V is last but one.
III. False (21–25) :
16. (d) Rocks Person Shift Off Day
Hills
P II Tuesday
Rivers Sticks Q I Monday
R II Wednesday
S I Sunday
T III Friday
OR
V III Thursday
Rocks W I Saturday
Hills
21. (c) The off day of W is Saturday.
Rivers 22. (d) The off day of R is Wednesday.
23. (a) P and R work in shift II.
Sticks
24. (d) The off day of Q is Monday.
25. (c) Q, S and W work in shift I.
Conclusion: 26. (e) Both the assumptions are implicit because daily
I. False newspapers provide instant reach. Again 100
II. False vacancies of Chartered Accountants were announced
III. True assuming sufficient eligible candidates may join the
So only conclusion III follows. nationalised bank.”
(17–20) : 27. (a) Only assumption I is implicit because repairing of
roads is carried out so efficiently to leave potholes.
If we examine the given input and various steps of
Assumption II is not implicit because it is people’s
rearrangement we would find that one number and one word
right to complain against any pothole.
are arranged alternately. Pattern followed is one word get
arranged according to alphabetical order in one step and the 28. (e) Both the assumption are implicit because main
number is arranged ascending order in next step. The pattern consideration for people is cost factor. People would
y
o
u
rs
m
IBPS RRBs Officer Scale-II & III CWE Exam 2013 217

a
h
b
prefer foreign destination at competitive prices. Conclusions

o
Advertisement provides sufficient information on I. D # A Þ D = A : Not True

o
b
this. II. D # T Þ D = T : Not True

.w
29. (b) Only assumption II is implicit because customers III. B @ T Þ B < T : True

o
rd
prefer to buy vegetables from retail vondors as there So only conclusion III follows.

p
is a lot of innovation in retail sector.
(36–40) :

re
30. (b) Only assumption II is implicit because college

s
s
authority cannot admit all those standing in the CRITERIA

.c
queue. CANDIDATE

o
(i)(ii) or A (iii) (iv) (v) or B Ans

m
(31 –35) :
Geeta NG ü – ü ü ü – 2
$Þ ³ @Þ £ ÓÞ < Ravi – – –
ü ü ü ü 4
% Þ> # Þ = – –
Ranjit ü ü ü ü – 5
31. (e) M © T Þ M < T Deepak ü ü – – û ü – 1
T@JÞT£J – – –
Sankot ü ü ü ü 3
J#DÞJ=D
Therefore, M < T £ J = D 36. (b) Data inadequate
Conclusions 37. (d) Ravi Sharma satisfies conditions (i), (ii), (iii), (iv) and
(B). So his case should be referred to President –
I. D # T Þ D = T : Not True Finance.
II. D % T Þ D > T : Not True 38. (e) Ranjit Malhotra satisfies all the criteria. So he can be
III. D % M Þ D > M : True selected.
Either I or II and III are true. 39. (a) Deepak Banerjee does not satisfy condition (iii). So
32. (b) H $ J Þ H ³ J he cannot be selected.
J©MÞJ<M 40. (c) Sanket Chopra satisfies conditions (i), (A), (iii), (iv)
and (v). So his case should be referred to General
M@TÞM£T
Manager — Finance.
Therefore, H ³ J < M £ T 41. (b) The given series is based on the following pattern :
Conclusions
12 22 69 272 1365 8184
I. H % M Þ H > M : Not True
II. H $ T Þ H ³ T : Not True ×2–2 ×3+3 ×4–4 ×5+5 ×6–6
III. T % J Þ T > J : True Hence, 8184 will replace the question mark.
So only conclusion III is true. 42. (d) The given series is based on the following pattern :
33. (a) R @ N Þ R £ N 1 = 13
N%EÞN>E
? = 23 = 8
E#KÞE=K
27 = 33
Therefore, R £ N > E = K
64 = 43
Conclusions
125 = 53
I. R © K Þ R < K : Not True
Hence, 8 will replace the question mark.
II. K % N Þ K > N : Not True 43. (e) Identifying the pattern for given series
III. E % R Þ E > R : Not True 2 2 2
+7 +6 +5
So none of conclusion is true. 104 153 189 214
2
34. (c) K % N Þ K > N +6
N$BÞN³B 230
B©DÞB<D 44. (b) Identifying the pattern for given series
Therefore, K > N > B < D 15 + 17 32
Conclusions 17 + 32 49
I. D % N Þ D > N : Not True 32 + 49 81
II. K % D Þ K > D : Not True 49 + 81 130
III. B © K Þ B < K : True 81 + 130 211
So only conclusion III follows. 45. (c) The given series is based on the following pattern :
35. (d) T # A Þ T = A
15 17 21 29 45 77 141
A$BÞA>B
B@DÞB<D +2 +4 +8 +16 +32 +64
Therefore, T = A > B < D Hence, 141 will replace the question mark.
y
o
u
rs
m
218 IBPS RRBs Officer Scale-II & III CWE Exam 2013

a
h
` 3000 – ` 3999 Þ 70 + 100 + 80 + 115 + 200 + 240 = 805

b
46. (a) The committee consists of 3 men and 2 women.

o
Out of 10 men. 3 men can be selected in 10C3 ways. ` 2000 – ` 2999 Þ 300 + 500 + 100 + 216 + 135 + 300 =1551

o
b
and out of 8 women can be selected in 8C2ways ` 1000 – ` 1999 Þ 140 + 370 + 200 + 225 + 175 + 470 = 1580

.w
` 500 – ` 999 Þ 200 + 700 + 15 + 400 + 75 + 530 = 1920
\ Total number of selections = 10C3 ´ 8C 2

o
rd
Less than ` 500 Þ 65 + 135 + 111 + 188 + 25 + 65 = 589

p
10 ´ 9 ´ 8 8 ´ 7 So maximum number of books bought in price range

re
= ´

s
1´ 2 ´ 3 1 ´ 2 of ` 4000 – 5000.

s
.c
= 3360 140

o
53. (b) Required percentage = ´ 100 = 70

m
47. (d) Let’s look at following cases : 200
(i) 2 women & 2 men 54. (c) Number of books bought in September
(ii) 3 women & 1 man = (25 + 400 + 200 + 135 + 175 + 75 + 25) = 1035
(iii) 4 women Number of books bought in November
\ Total number of selections = (75 + 375 + 240 + 300 + 470 + 530 + 65) = 2055
= 8C2 × 10C2 + 8C3 × 10C1 + 8C4 So Required difference = 2055 – 1035 = 1020.
8 ´ 7 10 ´ 9 8 ´ 7 ´ 6 ´10 8 ´ 7 ´ 6 ´ 5 55. (d) Required ratio = 105 : 1000 = 21 : 200
= ´ + +
1´ 2 1´ 2 1´ 2 ´ 3 1´ 2 ´ 3 ´ 4 56. (b) Ratio of the equivalent capitals of Prabha and Amit
= 1260 + 560 + 70 = 1890 of 1 month
48. (b) Out of 5 men 2 teachers, can be selected in 5C2 ways. = 32000 × 12 : 22000 × 7
Out of 3 women teachers, 2 can be selected in 3C2 = 32 × 12 : 22 × 7 = 192 : 77
ways. Total profit = `16409
Out of 5 doctors 1 can be selected in 5C1 ways. 192
\ Total number of selections = 5C2 × 3C2 × 5C1 \ Prabha’s share = × 16409 = `11712
(192 + 77)
5´ 4 3´ 2
= ´ ´5 SI ´ 100
1 ´ 2 1´ 2 57. (a) Rate =
Principal ´ Time
= 10 ×3 × 5 = 150 5940 ´ 100
= = 12% per annum
49. (a) Out of 18 persons, a committee of 7 persons is to be 500 ´ 3
formed.
Total number of selections = 18C7 éæ R ö
T ù
\ CI = P ç 100 ÷ - 1ú
ê 1 +
18´17´16´15´14´13´12 êëè ø úû
= = 31824
1´ 2´3´ 4´5´ 6´ 7
50. (c) The committee has no teachers and no doctor. éæ 12 ö
3 ù
Out of 18 persons, there are 8 teacher and 5 doctors. = 16500 êç1 + 100 ÷ - 1ú
êëè ø úû
\ Total number of selections
= 16500 [(1.12)3 –1]
= Number of ways of selecting 3 persons out of
remaining 5 persons = 16500 × 0.404928 = ` 6681.31
58. (c) Let Prerna’s salary be ` x
5´ 4 ´3 According to the question,
= 5C3 = = 10
1´ 2 ´ 3 80% of 15% of x = 1896
51. (a) Number of books bought in :
15 4
January Þ 50 +105 + 70 + 300 +140 + 200 + 65 = 930 Þ x× ´ = 1896
100 5
March Þ 106 +100 +100 + 500 + 370 + 700 +135 = 2911
1896 ´ 5 ´100
May Þ 2 + 40+ 80 +100+ 200 +15 +111= 548 \ x= = `15800
15 ´ 4
July Þ 30 +105+115 + 216 + 225 + 400 +188 =1279
September Þ 25+ 400+ 200+135+175+ 75 + 25 = 1035 a
59. (d) Let the original fraction is
NovemberÞ 75+ 375 + 240 + 300 + 470 + 530 + 65= 2055 b
So maximum number of books were bought in March. Numerator is increased by 600%,
52. (e) Number of books in the price range : 600
More than ` 5000 Þ 50 + 106 + 2 + 30 + 25 + 75 = 288 a®a+ ´ a = 7a
100
` 4000 – ` 5000 Þ 105 + 1000 + 40 + 105 + 400 + 375
= 2025
y
o
u
rs
m
IBPS RRBs Officer Scale-II & III CWE Exam 2013 219

a
h
b
Denominator is increased by 200%, Number of females = 2695 – 1078 = 1617

o
Number of people in teaching profession

o
200

b
b ®b+ ´ b = 3b

.w
100 21
= 24500 × = 5145

o
100

rd
7 a 14
=

p
According to the question
1

re
3b 5
Number of males = 5145 × = 1029

s
5

s
a 14 ´ 3 42 a 6

.c
= = or = Number of females = 5145 – 1029 = 4116

o
b 5 ´ 7 35 b 5

m
\ Required ratio = (1078 + 1029) : (1617 + 4116)
60. (a) Let Trisha’s and Shalini’s present ages be 7x and 6x
= 2107 : 5733 = 43 : 117
years respectively.
72. (a) Number of people in teaching profession = 5145
7x + 8 9 Number of people in law profession
After 8 years, =
6x + 8 8
24
Þ 56x + 64 = 54x + 72 = 24500 × = 5880
100
Þ 2x = 72 – 64 = 8
Þ x=4 5145
\ Required percentage = × 100 = 87.5
\ Required difference = 7x – 6x Þ x = 4 years 5880
61. (e) ? » 840 ÷ 48 » 17.5 Alternative
62. (b) ? = 8000 » 89 Percentage of people in teaching profession = 21
63. (c) ? = 18 × 23 × 10 » 4140 Percentage of people in law profession = 24

99999 21
\ Required percentage = × 100 = 87.5
64. (a) ? = » 112 24
99 ´ 9
73. (c) Number of males :
65. (d) ? » 7985 + 2541 + 128 » 10654
Medical Þ 1078
66. (b) Required ratio = 32.5 : 27.5 = 13 : 11
67. (e) Required average number of candidates 18 70
Engineering Þ 24500 × ´ = 3087
100 100
æ 27.5 + 27.5 + 30 + 32.5 + 22.5 + 35 ö
= ç ÷ lakh
è 6 ø 24 55
Law Þ 24500 × ´ = 3234
100 100
æ 175 ö
= ç ÷ lakh Teaching Þ 1029
è 6 ø
= 29.17 lakh = 2917000 16 65
Banking Þ 24500 × ´ = 2548
100 100
30
68. (d) Required percentage = × 100 » 133 10 56
22.5 ´
Management Þ 24500 × = 1372
69. (c) Required ratio = (27.5 + 27.5 + 30) : (32.5 + 22.5 +35) 100 100
= 85 : 90 = 17 : 18 \ Total number of males
70. (a) Total number of candidates appearing for the test = 1078 + 3087 + 3234 + 1029 + 2548 + 1372
from all the cities in 2008 = 12348
= (32.5 + 30 + 22.5 + 27.5 + 30 + 32.5) lakh = 175 lakh 74. (b) Number of females in Engineering profession
Number of candidates appearing in the test from city
18 30
E in 2008 = 24500 × ´ = 1323
100 100
= 30 lakh
Number of males in Management = 1372
30
\ Required percentage = × 100 = 17.14 1323
175 Required percentage = × 100 = 96.43
1372
71. (d) Number of people in medical profession
75. (d) Required ratio = 2548 : 3087 = 63 : 52
11 76. (e) Required average number of females
= 24500 × = 2695
100
2750 + 4000 + 4250 +3750 + 3500 18250
40 = = = 3650
5 5
Number of males = 2695 × = 1078
100
y
o
u
rs
m
220 IBPS RRBs Officer Scale-II & III CWE Exam 2013

a
h
b
77. (a) Total number of males from all organisations 141. (c) 142. (a) 143. (c) 144. (d)

o
o
= 3000 + 3750 + 4000 + 2500 + 3250 = 16500 145. (b) 146. (d) 147. (b) 148. (d)

b
.w
3000 149. (a) 150. (a) 151. (a) 152. (d)

o
\ Required percentage = ×100 » 18

rd
16500 153. (b) 154. (b) 155. (d) 156. (a)

p
157. (e) 158. (a) 159. (c) 160. (c)

re
78. (b) Required difference = (18250 – 16500) = 1750

s
161. (a) It is mentined at the end of the first paragraph.

s
79. (b) Required ratio = 4250 : 3500 = 17 : 14

.c
162. (e) All the statements are mentioned in the passage.

o
80. (c) Total number of males from organisations A and B

m
= 3000 + 3750 = 6750 163. (b) It is mentined towards the end of the second
Total number of males from organisations C, D and E paragraph.

= 4000 + 2500 + 3250 = 9750 164. (e) 165. (e)


166. (a) The tone of the passage suggests the same and the
6750 reasons what countries would do in response to
\ Required percentage = × 100 » 69
9750 continuously rising price of oil.
81. (d) 82. (c) 83. (e) 84. (b) 167. (d)
85. (e) 86. (a) 87. (d) 88 (e) 168. (b) Aberrant means divergong from the normal.
89 (c) 90. (e) 91. (a) 92. (c) 169. (c) Proactive means acting on a situation before things
93. (c) 94. (d) 95. (c) 96. (d) happen.
97. (e) 98. (d) 99. (c) 100. (b) 170. (d) 171. (b) 172. (e)
101. (d) 102. (c) 103. (e) 104. (b) 173. (c) His girl child.
105. (e) 106. (e) 107. (d) 108. (d) 174. (b) Sources of energy is.
109. (c) 110. (c) 111. (e) 112. (b) 175. (a) I have been working in.
113. (d) 114. (c) 115. (c) 116. (d) 176. (c) 177. (e) 178. (b) 179. (a)
117. (a) 118. (e) 119. (c) 120. (b) 180. (c) 181. (b) 182. (d) 183. (e)
121. (c) 122. (b) 123. (b) 124. (d) 184. (d) 185. (c) 186. (b) 187. (e)
125. (d) 126. (d) 127. (c) 128. (d) 188. (c) 189. (a) 190. (d) 191. (b)
129. (a) 130. (e) 131. (b) 132. (d) 192. (a) 193. (d) 194. (b) 195. (c)
133. (c) 134. (c) 135. (b) 136. (b) 196. (e) 197. (c) 198. (c) 199. (d)
137. (a) 138. (d) 139. (b) 140. (b) 200. (a)
y
o
u
rs
m
a
h
b
o
o
SBI PO EXAM 2013

b
.w
o
rd
p
Based on Memory

re
s
s
.c
o
m
Maths and Economics but not necessarily in the same order.
REASONING (High Level) A likes Maths and studies in the 5th standard with only one other
friend who likes Marathi. I studies with two other friends. Both
DIRECTIONS (Qs. 1-5): Read each statement carefully and
the friends who study with I like languages. (Here languages
answer the following questions.
include only Hindi, Marathi and English). D Studies in the 6th
1. Which of the following expressions will be true if the standard with only one person and does not like civics. E studies
expression R > O = A > S > T as definitely true? with only one friend. The one who likes history does not study in
(a) O > T (b) S < R 5th or 6th standard. E does not like languages. C does not like
(c) T < A (d) S = O English, Hindi or Civics.
(e) T < R 6. Which combination represents E’s favourite subject and the
2. Which of the following symbols should replace the questions standard in which he studies?
mark (?) in the given expression in order to make the (a) Civics and 7th (b) Economics and 5th
expression ‘P > A’ as well as ‘T £ L’ definitely true? (c) Civics and 6th (d) History and 7th
P> L?A³N=T (e) Economics and 7th
(a) £ (b) > 7. Which of the following is I’s favourite subject?
(c) < (d) ³ (a) History
(e) either £ or < (b) Civics
3. Which of the following symbols should be placed in the (c) Marathi
blank spaces respectively (in the same order from left to (d) Either English or Marathi
right) in order to complete the given such a manner that (e) Either English or Hindi
makes the expressions ‘B > N’ as well as ‘D £ L’ definitely 8. Which amongst the following combinations studies in the
true? 7th standard?
B ––L––O––N––D (a) G (b) C
(a) =. =. ³, ³ (b) >, ³, =, > (c) E (d) D
(c) >, < =, £ (d) <. =, =, ³ (e) Either D or B
(e) >, = ³, > 9. Which of the following is definitely correct?
4. Which of the following should be placed in the blank spaces (a) I and Hindi (b) G and English
respectively (in the same order from left to right) in order to (c) C and Marathi (d) B and Hindi
complete the given expression in such a manner that makes (e) E and Economics
the expression ‘A < P’ definitely false? 10. Which of the following subjects does G like?
__ £ __ < __> __ (a) Either Maths or Marathi
(a) L, N, P, A (b) L, A, P, N (b) Either Hindi or English
(c) A, L, P, N (d) N, A, P, L (c) Either Hindi or Civics
(e) P, N, A, L (d) Either Hindi or Marathi
5. Which of the following symbols should be placed in the (e) Either Civics or Economics
blank spaces respectively (in the same order from left too
DIRECTIONS (Qs.11-15): Study the following information
right) in order to complete the given expression in such a
carefully and answer the given questions.
manner that makes the expression ‘F > N’ and ‘U > D’
definitely false? In a certain code language- ‘economics is not money’ is written
F__O__U__N__D as, ‘ka la ho ga’ ‘demand and supply economics’ is written as, ‘mo
(a) <. <. <, = (b) <, =, =, > ta pa ka’ money makes only part’ is written as, ‘zi la ne ki’ demand
(c) <, =, =, < (d) ³, =, =, ³ makes supply economics’ is written as, ‘zi mo ka ta’
(d) >, >, =, < 11. What is the code for ‘money’ in the given code language?
DIRECTIONS (Qs.6-10): Study the following information (a) ga (b) mo
carefully and answer the given questions. (c) pa (d) ta
(e) la
A, B, C, D, E, G, and I are seven friends who study in three different 12. What is the code for ‘supply’ in the given code language?
standards namely 5th, 6th and 7th such that not less than two (a) only ta (b) only mo
friends study in the same standard. Each friend also has a different (c) either pa or mo (d) only pa
favourite subject namely History, Civics, English, Marathi, Hindi, (e) either mo or ta
y
o
u
rs
m
222 SBI PO Exam 2013

a
h
b
13. What may be the possible code for ‘demand only more’ in 19. What is the position of ‘nice’ from the left end in the final

o
the given code language? step?

o
b
(a) xi ne mo (b) mo zi ne (a) Fifth (b) Sixth

.w
(c) ki ne mo (d) mo zi ki (c) Seventh (d) Eighth

o
(e) xi ka ta (e) Ninth

rd
14. What may be the possible code for ‘work and money’ in the 20. Which element is third to the right of ‘family’ in Step V?

p
re
given code language? (a) beautiful (b) 17

s
(a) pa ga la (b) pa la tu (c) proud (b)` 97

s
.c
(c) mo la pa (d) tu la ga (c) 32

o
m
(e) pa la ne
DIRECTIONS (Qs.21-25): Study the following information
15. What is the code for ‘makes’ in the given code language?
(a) mo (b) pa carefully and answer the given questions.
(c) ne (d) zi If A + B means A is the father of B
(e) ho If A × B means A is the sister of B
If A $ B means A is the wife of B
DIRECTIONS (Qs.16-20): Study the following information If A % B means A is the mother of B
carefully and answer the given questions.
If A ¸ B means A is the son of B
When a word and number arrangement machine is given an input 21. What should come in place of the question mark, to establish
line of words and numbers, it arranges them following a particular that J is the brother of T in the expression?
rule. The following is an illustration of input and rearrangement: J ¸P%H?T %L
(All the numbers are two digit numbers) (a) × (b) ¸
Input : 40 made butter 23 37 cookies salt extra 52 86 92 fell now 19 (c) $ (d) Either ¸ or ×
Step I : butter 19 40 made 23 37 cookies salt extra 52 86 92 fell (e) Either + or ¸
now 22. Which among the given expressions indicate that M is the
Step II: cookies 23 butter 19 40 made 37 salt extra 52 86 92 fell
now daughter of D?
Step III: extra 37 cookies 23 butter 19 40 made salt 52 86 92 fell (a) L % R $ D + T × M (b) L + R $ D + M × T
now (c) L % R % D + T ¸ M (d) D + L $ R + M × T
Step IV: fell 40 extra 37 cookies 23 butter 19 made salt 52 86 92 (e) L $ D ¸ R % M ¸ T
now 23. Which among the following options is true if the expresssion
Step V: made 52 fell 40 extra 37 cookies 23 butter 19 salt 86 92 ‘I + T % J × L ¸ K’ is definitely true?
now (a) L is the daughter of T (b) K is the son-in-law of I
Step VI: now 86 made 52 fell 40 extra 37 cookies 23 butter 19 salt (c) I is the grandmother of L (d) T is the father of L
92 (e) J is the brother of L
Step VII: salt 92 now 86 made 52 fell 40 extra 37 cookies 23 butter 24. Which among the following expression is true if Y is the son
19 of X is definitely false?
Step VII is the last step of the above arrangement as the intended
arrangement is obtained. (a) W % L × T × Y ¸ X (b) W + L × T × Y ¸ X
(c) X + L × T × Y ¸ W (d) W $ X + L + Y + T
As per the rules followed in the given steps, find out the
(e) W % X + T × Y ¸ L
appropriate steps for the given input.
Input: 32 proud girl beautiful 48 55 97 rich family 61 72 17 nice life 25. What should come in place of the question mark, to establish
that T is the sister-in-law of Q in the expression?
16. How many steps will be required to complete the given input? R%T ×P?Q +V
(a) Five (b) Six (a) ¸ (b) %
(c) Seven (d) Eight (c) × (d) $
(e) Nine (e) Either $ or ×
17. Which of the following is the third element from the left end
of step VI? DIRECTIONS (Qs.26-30): Study the following information
(a) beautiful (b) life carefully and answer the given questions.
(c) 61 (d) nice
Eight people - E, F, G, H, J, K, L and M are sitting around a circular
(e) 17
table facing the centre. Each of them is of different profession
18. Which of the following is step III of the given input?
-Chartered Accountant, Columnist, Doctor, Engineer, Financial
(a) Proud 72 girl 48 family 32 beautiful 17 55 97 rich 61 nice
Analyst, Lawyer, Professor and Scientist but not necessarily in
life
the same order. F is sitting second to the left of K. The Scientist is
(b) life 55 girl 48 family 32 beautiful A proud 97 rich 61 72 an immediate neighbour of K. There are only three people between
nice. the Scientist and E. Only one person sits between the Engineer
(c) girl 48 family 32 beautiful 17 proud 55 97 rich 61 72 nice and E. The Columnist is to the immediate right of the Engineer. M
life is second to the right of K. H is the Scientist. G and J are immediate
(d) family 32 beautiful 17 proud girl 48 55 97 rich 61 72 nice neighbours of each other. Neither G nor J is an Engineer. The
life Financial Analyst is to the immediate left of F. The Lawyer is
(e) girl 48 life 55 family 32 beautiful 17 proud 97 rich 61 72 second to the right of the Columnist. The Professor is an immediate
nice
y
o
u
rs
m
SBI PO Exam 2013 223

a
h
neighbour of the Engineer. G is second to the right of the Chartered

b
DIRECTIONS (Qs. 36-38): Each of the questions below consists

o
Accountant.

o
of a question and two statements numbered I and II given below it.

b
26. Who is sitting second to the right of E? You have to decide whether the data given in the statements are

.w
(a) The Lawyer (b) G sufficient to answer the questions. Read both the statements and-

o
(c) The Engineer (d) F

rd
(e) K Give answer (a) if the data to statement I alone are sufficient to

p
re
27. Who amongst the following is the Professor? answer the question, while the data in statement II alone are not

s
(a) F (b) L sufficient to answer the question.

s
.c
(c) M (d) K Give answer (b) if the data to statement II alone are sufficient to

o
(e) J answer the question, while the data in statement I alone are not

m
28. Four of the following five are alike in a certain way based on sufficient to answer the question.
the given arrangement and hence form a group. Which of Give answer (c) if the data either in statement I alone or statement
the following does not belong to that group? II alone are sufficient to answer the question.
(a) Chartered Accountant - H Give answer (d) if the data neither in statement I nor II together
(b) M - Doctor are sufficient to answer the questions.
(c) J - Engineer Give answer (e) if the data in both statement I and II together are
(d) Financial Analyst - L necessary to answer the questions.
(e) Lawyer - K 36. Seventeen people are standing in a straight line facing south.
29. What is the position of L with respect to the Scientist? What is Bhavna’s position from the left end of the line?
(a) Third to the left (b) Second to the right I. Sandeep is standing second to the left of Sheetal. Only
(c) Second to the left (d) Third to the right five people stand betweeen Sheetal and the one who
(e) Immediate right is standing at the extreme right end of the line. Four
30. Which of the following statements is true according to the people stand between Sandeep and Bhavna.
given arrangement? II. Anita is standing fourth to the left of Sheetal. Less
(a) The Lawyer is second to the left of the Doctor than three people are standing between Bhavna and
(b) E is an immediate neighbour of the Financial Analyst Anita.
(c) H sits exactly between F and the Financial Analyst 37. Five letters - A, E, G, N and R are arranged left to right
(d) Only four people sit between the Columnist and F according to certain conditions. Which letter is placed third?
(e) All of the given statements are true I. G is placed second to the right of A. E is to the immediate
right of G. There are only two letters between R and G.
DIRECTIONS (Qs. 31-35): In each of the questions below, two/ II. N is exactly between A and G. Neither A nor G is at the
three statements are given followed by conclusions/group of extreme end of the arrangement.
conclusions numbered I and II. You have to assume all the statements 38. Six people - S, T, U, V, W and X are sitting around a circular
to be true even if they seem to be at variance from the commonly table facing the centre. What is T’s position with respect to X?
known facts and then decide which of the given two conclusions I. Only two people sit between U and W. X is second to the
logically follows from the information given in the statements. left of W. V and T are immediate neighbours of each other.
II. T is to the immediate right of V. There are only two
Give answer (a) if only conclusion I follows people between T and S. X is an immediate neighbour
Give answer (b) if only conclusion II follows of S but not of V.
Give answer (c) if either I or II follows
Give answer (d) if neither I nor II follows DIRECTIONS (Qs.39-40): Study the following information
Give answer (e) if both I and II follows carefully and answer the given questions.
31-32. Statements: Some squares are circles.
The convenience of online shopping is what I like best about it.
No circle is a triangle.
Where else can you shop even at midnight wearing your night
No line is a square.
suit? You do not have to wait in a line till the shop assistant is
31. Conclusions: I. All squares can never be triangles.
ready to help you with your purchases. It is much better experience
II. Some lines are circles.
as compared to going to a retail store. – A consumer’s view.
32. Conclusions: I. No triangle is a square.
II. No line is a circle. 39. Which of the following can be a strong argument in favour
33-34: Statements: All songs are poems. of retail store owners?
All poems are rhymes. (a) Online shopping portals offer a great deal of discounts
No rhymes is a paragraph. which retail stores offer only during the sale season.
33. Conclusions: I. No song is a paragraph. (b) One can compare a variety of products online which
No poem is a paragraph. cannot be done at retail stores.
(c) Many online shopping portals offer the ‘cash on
34. Conclusions: I. All rhymes are poems.
delivery’ feature which is for those who are sceptical
All songs are rhymes.
about online payments.
35. Statements: Some dews are drops. All drops are stones. (d) Many consumers prefer shopping at retail stores which
Conclusions: I. Atleast some dews are stones. are nearer to their houses.
II. Atleast some stones are drops. (e) In online shopping the customer may be deceived as
he cannot touch the product he is paying for.
y
o
u
rs
m
224 SBI PO Exam 2013

a
h
b
40. Which of the following can be inferred from the given hall early stole the mobile phones kept in the bags of the

o
information? (An inference is something that is not directly

o
students who were writing the exam.

b
stated but can be inferred from the given information) (E) The school authorities must ask the students to leave their

.w
(a) One can shop online only at night.
phones in the custody of the invigilator before the exam in

o
(b) Those who are not comfortable using computers can

rd
never enjoy the experience of online shopping. order to avoid thefts of mobile phones.

p
(F) None of the other students were carrying their phones in the

re
(c) All retail stores provide shopping assistants to each

s
and every customer. exam hall.

s
.c
(d) The consumer whose view is presented has shopped 43. Which of the following among (A), (B), (C) and (D) may be a

o
at retailo stores as well as online. strong argument in favour of, the three students who were

m
(e) The consumer whose view is presented does not have
any retail stores in her vicinity. caught with the mobile phone?
41. Read the following information carefully and answer the given (a) Only (A) (b) Both (A) and (B)
question: (c) Both (C) and (D) (d) Only (C)
Many manufacturing companies are now shifting base to (e) Both (B) and (D)
the rural areas of the country as there is a scarcity of space 44. Which of the following among (A), (B), (E) and (F) may be
in urban areas. Analysts say that this shift will not have a the reason behind the school making such announcements
huge impact on the prices of the products manufacutured
by these companies as only about 30% consumers live in before the exam?
urban areas. (a) Only (B) (b) Both (B) and (E)
Which of the following may be consequence of the given (c) Only (F) (d) Only (A)
information? (e) Both (E) and (F)
(a) The prices of such products will decrease drastically 45. Which of the following among (A), (B), (D) and (F) can be an
in the urban areas. immediate course of action for the invigilator?
(b) People living in urban areas will not be allowed to work
in such manufacturing companies. (a) Only (B) (b) Both (A) and (D)
(c) These manufacturing companies has set-ups in the (c) Only (A) (d) Both (D) and (F)
urban areas before shifting base. (e) Only (F)
(d) Those who had already migrated to the urban areas
will not shift back to rural areas. DIRECTIONS (Qs.46-50): In each of the questions given below
(e) The number of people migrated from rural to urban which one of the five answer figures on the right should come after
areas in search of jobs may reduce. the problem figures on the left, in the sequence were continued?
42. Read the following information carefully and answer the given
question:
46.
‘Pets are not allowed in the park premises’ - A notice put up
at the park entrance by the authority that is responsible for
maintenance of the park.
Which of the following can be an assumption according to
the given information? (An assumption is something that is
supposed or taken for granted) Answer Figure:
(a) At least some people who visit the park have pets.
(b) This is the only park which does not allow pets
(c) People who ignored this notice were fined
(d) There are more than one entrances to the park
(e) Many people have now stopped visiting the park
DIRECTIONS (Qs.43-45): Study the following information (a) (b) (c) (d) (e)
carefully and answer the given questions.
Despite repeated announcements that mobile phones were not 47. C
T S
allowed in the examination hall, three students were caught with T C
C C
their mobile phones. S S T CS
T T
(A) Mobile phones nowadays have a lot of features and it is S
easy to cheat with their help.
(B) The invigilator must immediately confiscate the mobile Answer Figure:
phones and ask the students to leave the exam hall
immediately. C C
(C) Mobile phones are very expensive and leaving them in bags T C
C ST C TS
S S S
outside the exam hall is not safe.
T T
(D) There have been incidents where students who left the exam
(a) (b) (c) (d) (e)
y
o
u
rs
m
SBI PO Exam 2013 225

a
h
b
48. × C B O × O 51. In which city is the difference between the cost of one kg of

o
o
O B C B apple and cost of one kg of guava second lowest?

b
.w
× × C (a) Jalandhar (b) Delhi

o
B O C B O × C (c) Chandigarh (d) Hoshiarpur

rd
p
Answer Figure: (e) Ropar

re
52. Cost of one kg of guava in Jalandhar is approximately what

s
× C C × C × C C ×

s
.c
B O percent of the cost of two kgs of grapes in Chandigarh?

o
B

m
(a) 66 (b) 24
B O B O O × C O B (c) 28 (d) 34
(a) (b) (c) (d) (e) (e) 58
49. 53. What total amount will Ram pay to the shopkeeper for
purchasing 3 kgs of apples and 2 kgs of guavas in Delhi?
(a) ` 530/- (b) ` 450/-
(c) ` 570/- (b) ` 620/-
Answer Figure: (e) ` 490/-
54. Ravinder had to purchase 45 kgs of grapes from Hoshiarpur.
Shopkeeper gave him discount of 4% per kg. What amount
did he pay to the shopkeeper after the discount?
(a) ` 8,280/- (b) ` 8,104/-
(a) (b) (c) (d) (e)
(c) ` 8,340/- (b) ` 8,550/-
50. 8 z D x D x O 8 (e) ` 8,410/-
55. What is the respective ratio between the cost of one kg of
D 3 T x O T z 8 8 T z S D
apples from Ropar and the cost of one kg of grapes from
z x T 8 D O O z x Chandigarh?
Answer Figure: (a) 3: 2 (b) 2 : 3
z 8 S z D O z (c) 22 : 3 2 (d) 42 : 92
(e) 92 : 4 2
D S O x 8 z O x z D x x B O
DIRECTIONS (Qs. 56-60) : Study the following information
x 8 D 8 S 8 S D 8 carefully and answer the given questions.
(a) (b) (c) (d) (e)
Number of students (in thousands) in two
DATA ANALYSIS AND INTERPRETATION different universities in six different years
Number of students
Univeristy - 1
DIRECTIONS (Qs.51-55): Study the following information
carefully and answer the given questions. 2007 University - 2
Cost of three different fruits (in rupees per kg. in five different cities) 35
Grapes 30
Cost of three different fruits (in rupees per Guava 25
kg. in five difference cities) Apple 20
2012 2008
200
15
10
180 5
Cost of Fruits (rupees per kg.)

160
140 0
120
100
80 2011 2009
60
40
20
0 2010
Jalandhar Delhi Chandigarh Hoshiarpur Ropar
Cities
y
o
u
rs
m
226 SBI PO Exam 2013

a
h
b
56. What was the difference between the number of students in 62. What is the respective ratio between number of players

o
university - 1 in the year 2010 and the number of students in participating in baseketball from school-I and the number of

o
b
university - 2 in the year 2012? players participating in Kho-Kho from school-3?

.w
(a) Zero (b) 5,000 (a) 5 : 7 (b) 7 : 9 (c) 7 : 8 (d) 9 : 7 (e) 5 : 8

o
63. In which school is the number of players participating in

rd
(c) 15,000 (d) 10,000
hockey and basketball together second lowest?

p
re
(e) 1,000 (a) School - 1 (b) School - 2 (c) School - 3

s
57. What is the sum of the number of students in university - 1 (d) School - 4 (e) School - 5

s
.c
in the year 2007 and the number of students in university - 2 64. Number of players participating in Kho-Kho from school-4

o
m
in the year 2011 together? is what percent of number of players participating in hockey
(a) 50000 (b) 55000 from school-2?
(c) 45000 (d) 57000 (a) 42 (b) 48 (c) 36 (d) 40 (e) 60
65. 25% of the number of the players particpating in hockey
(e) 40000
from School-5 are females. What is the number of the hockey
58. If 25% of the students in university - 2 in the year 2010 were players who are males in school-5?
females, what was the number of male students in the (a) 15 (b) 18 (c) 30 (d) 21 (e) 27
university -2 in the same year?
(a) 11250 (b) 12350 DIRECTIONS (Qs.66-70): Study the following bar-graph
carefully and answer the following questions.
(c) 12500 (d) 11500
(e) 11750 Earnings (in rupees) of three different
59. What was the present increase in the number of students in persons on four different days Naveen
University - I in the year 2011 as compared to the previous Gita
year? Rahul
(a) 135 (b) 15
Thursday
(c) 115 (d) 25
(e) 35
60. In which year was the difference between the number of Wednesday
students in university - I and the number of students in
university - 2 highest? Tuesday
(a) 2008 (b) 2009
(c) 2010 (d) 2011 Monday
(e) 2012
DIRECTIONS (Qs. 61-65) : Study the following information 0 100 200 300 400 500
carefully and answer the given questions.
66. What is Gita’s average earnings over all the days together?
Number of players participating in three different games in five (a) ` 285/- (b) ` 290/- (c) ` 320/-
different schools.
(d) ` 310/- (e) ` 315/-
Number of players participating in three 67. What is the total amount earned by Rahul and Naveen
different games in five different schools together on Tuesday and Thursday together?
(a) ` 1,040/- (b) ` 1,020/- (c) ` 980/-
90 (d) ` 940/- (e) ` 860/-
80 Hockey
68. Gita donated her earnings of Wednesday to Naveen. What
Number of Players

70 Basketball
was Naveen’s total earnings on Wednesday after Gita’s
60 Kho-Kho
50 donation?
40 (a) ` 520/- (b) ` 550/- (c) ` 540/-
30 (d) ` 560/- (e) ` 620/-
20 69. What is the difference between Rahul’s earnings on Monday
10
0
and Gita’s earnings on Tuesday?
School-1 School-2 School-3 School-4 School-5 (a) ` 40/- (b) ` 20/- (c) ` 50/-
Schools
(d) ` 30/- (e) ` 10/-
70. What is the respective ratio between Naveen’s earnings on
61. What is the total number of players participating in hockey Monday, Wednesday and Thursday?
from all the five schools together? (a) 7 : 3 : 5 (b) 8 : 6 : 5 (c) 8 : 7 : 4
(a) 324 (b) 288 (c) 342 (d) 284 (e) 248 (d) 9 : 5 : 4 (e) 6 : 2 : 3
y
o
u
rs
m
SBI PO Exam 2013 227

a
h
b
DIRECTIONS (Qs. 71-75) : Study the following pie-chart and 72. In management profession three-fourth of the number of

o
employees are female. What is the number of male employees

o
answer the following questions.

b
in management profession?

.w
Percentagewise distribution of employees in
(a) 1239 (b) 1143

o
six different professions

rd
Total number of employees = 26800 (c) 1156 (d) 1289

p
re
(e) 1139

s
73. 25% of employees from film production profession went on

s
9% ng

.c
a strike. What is the number of employees from film
ri

o
i n ee

m
Management production who have not participated in the strike?
E ng

17%
Teaching (a) 3271 (b) 3819
15% (c) 3948 (d) 1273
Industries
13%
(e) 1246
74. What is the total number of employees in engineering
Film Production
19% profession and industries together?
Medical (a) 5698 (b) 5884
27%
(c) 5687 (d) 5896
(e) 5487
75. In teaching profession if three-fifth of the teachers are not
71. What is the difference between the total number of permanent, what is the number of permanent teachers in the
employees in teaching and medical profession together and teaching profession?
the number of employees in management profession?
(a) 1608 (b) 1640
(a) 6770 (b) 7700
(c) 1764 (d) 1704
(c) 6700 (d) 7770
(e) 7670 (e) 1686

DIRECTIONS (Qs.76-80): Study the table carefully to answer the questions that follow:
Monthly Bill (in rupees) of landline phone, electricity, laundry and mobile phone paid by three different people in five different months.

Monthly Bills
Month Landline Phone Electricity Laundry Mobile Phone
Ravi Dev Manu Ravi Dev Manu Ravi Dev Manu Ravi Dev Manu
March 234 190 113 145 245 315 93 323 65 144 234 345
April 124 234 321 270 220 135 151 134 35 164 221 325
May 156 432 211 86 150 98 232 442 132 143 532 332
June 87 123 124 124 150 116 213 324 184 245 134 125
July 221 104 156 235 103 131 413 532 143 324 432 543

76. What is the total amount of bill paid by Dev in the month of (a) ` 180/- (b) ` 176/-
June for all the four commodities? (c) ` 190/- (d) ` 167/-
(a) ` 608/- (b) ` 763/- (e) ` 196/-
(c) ` 731/- (d) ` 683/- 79. In which months respectively did Manu pay the second
(e) ` 674/- highest mobile phone bill and the lowest electricity bill?
77. What is the average electricity bill paid by Manu over all the (a) April and June (b) April and May
five months together? (c) March and June (d) March and May
(a) ` 183/- (b) ` 149/- (e) July and May
(c) ` 159/- (d) ` 178/- 80. What is the respective ratio between the electricity bill paid
(e) ` 164/- by Manu in the month of April and the mobile phone bill
78. What is the difference between the mobile phone bill paid paid by Ravi in the month of June?
by Ravi in the month of May and the laundary bill paid by (a) 27 : 49 (b) 27 : 65
Dev in the month of March? (c) 34 : 49 (d) 135 : 184
(e) 13 : 24
y
o
u
rs
m
228 SBI PO Exam 2013

a
h
b
DIRECTIONS (Qs.81-85): Study the table carefully to answer the questions that follow:

o
o
b
Station Arrival time Departure Halt time (in Distance travelled No. of Passengers boarding

.w
Name time minutes) from origin (in km) the train at each station

o
rd
Dadar Starting point 12.05 am - 0 km 437

p
Vasai Road 12.53 am 12.56 am 3 minutes 42 km 378

re
Surat 4.15 am 4.20 am 5 minutes 257 km 458

s
s
.c
Vadodara 6.05 am 6.10 am 5 minutes 386 km 239

o
Anand Jn. 6.43 am 6.45 am 2 minutes 422 km 290

m
Nadiad Jn. 7.01 am 7.03 am 2 minutes 440km 132
Ahmedabad 8.00 am 8.20 am 20 minutes 486 km 306
Bhuj 5.40 pm Ending point – 977 km None

81. What is the distance travelled by the train from Surat to 84. If halt time (stopping time) of the train at Vadodara is
Nadiad Jn.? decreased by 2 minutes and increased by 23 minutes at
(a) 176 km (b) 188 km (c) 183 km (d) 193 km (e) 159 km Ahmedabad. At what time will the train reach Bhuj?
82. How much time does the train take to reach Ahmedabad (a) 6.10 am
after departing from Anand Jn. (including the halt time)? (b) 6.01 pm
(a) 1 hr. 59 min (b) 1 hr. 17 min. (c) 6.05 am
(c) 1 hr. 47 min. (d) 1 hr. 45 min. (d) 6.50 pm
(e) 1 hr. 15 min. (e) 6.07 pm
83. What is the respective ratio between the number of 85. Distance between which two stations is second lowest?
passengers boarding from Vasai Road and from Ahmedabad (a) Nadiad Jn. to Ahmedabad
in the train? (b) Anand Jn. to Nadiad Jn.
(a) 21 : 17 (b) 13 : 9 (c) Dadar to Vasai Road
(c) 21 : 19 (d) 15 : 13 (d) Anand Jn. to Vadodara
(e) 13 : 15 (e) Vasai Road to Surat
DIRECTIONS (Qs.86-90): Study the table carefully to answer the questions that follow.

Temperature
Month Bhuj Sydney Ontario Kabul Beijing
Max. Min. Max. Min. Max. Min. Max. Min. Max. Min.
st
1 September 24 14 12 2 5 1 34 23 12 9
st
1 October 35 21 5 –1 15 6 37 30 9 3
st
1 November 19 8 11 3 4 0 45 36 15 1
st
1 December 9 2 –5 –9 – 11 –7 31 23 2 –3
st
1 January –4 –7 11 – 13 – 14 – 19 20 11 5 – 13

86. What is the difference between the maximum temperature of 88. In which month (on 1st day) is the difference between
Ontario on 1st November and the minimum temperature of maximum temperature and minimum temperature of Bhuj
Bhuj on 1st January? second highest?
(a) 3°C (b) 18°C (a) 1st September (b) 1st October
(c) 15°C (d) 9°C (c) 1st November (d) 1st December
(e) 11°C st
(e) 1 January
87. In which month respectively the maximum temperature of 89. What is the average maximum temperature of Beijing over all
Kabul is second highest and minimum temperature of Sydney the months together?
is highest? (a) 8.4°C (b) 9.6°C
(a) 1st October and 1st January (c) 7.6°C (d) 9.2°C
(b) 1st October and 1st November (e) 8.6°C
(c) 1st December and 1st January
90. What is the respective ratio between the minimum
(d) 1st September and 1st January
temperature of Beijing on 1st September and the maximum
(e) 1st December and 1st September
y
o
u
rs
m
SBI PO Exam 2013 229

a
h
temperature of Ontario on 1st October?

b
95. What is the average number of petrol engine cars in all the

o
states together?

o
(a) 3 : 4 (b) 3 : 5

b
(a) 86.75 (b) 89.25

.w
(c) 4 : 5 (d) 1 : 5
(c) 89.75 (d) 86.25

o
(e) 1 : 4 (e) 88.75

rd
DIRECTIONS (Qs.91-95): Study the pie-chart and table 96. A bag contains 7 blue balls and 5 yellow balls. If two balls

p
re
carefully to answer the questions that follow: are selected at random, what is the probability that none is

s
yellow?

s
.c
Pie-chart showing percentagewise

o
distribution of cars in four different states 5 5

m
Total cars = 700 (a) (b)
Distribution of Cars
35 22
7 7
(c) (d)
22 33
State - 1
14% 7
State - 4 (e)
66
26%
97. A die is thrown twice. What is the probability of getting a
sum 7 from both the throws?
State - 2
28% 5 1
(a) (b)
18 18
Sta te - 3
32%
1 1
(c) (d)
9 6
5
(e)
Table showing ratio between diesel and petrol engine cars which 36
are distributed among four different states DIRECTIONS (Qs.98-100): Study the information carefully to
Diesel Engine Petrol Engine answer these questions.
State
Cars Cars In a team there are 240 members (males and females). Two-third of
State - 1 3 4 them are males. Fifteen percent of males are graduates. Remaining
males are non-graduates. Three-fourth of the females are
State - 2 5 9 graduates. Remaining females are non-graduates.
State - 3 5 3 98. What is the difference between the number of females who
are non-graduates and the number of males who are
State - 4 1 1 graduates?
(a) 2 (b) 24 (c) 4 (d) 116 (e) 36
91. What is the difference between the number of diesel engine
cars in state-2 and the number of petrol engine cars in 99. What is the sum of the number of females who are graduates
state-4? and the number of males who are non-graduates?
(a) 159 (b) 21 (a) 184 (b) 96 (c) 156 (d) 84 (e) 196
(c) 28 (d) 34 100. What is the ratio between the total number of males and the
(e) 161 number of females who are non-graduates?
92. Number of petrol engine cars in state-3 is what percent more (a) 6 : 1 (b) 8 : 1 (c) 8 : 3 (d) 5 : 2 (e) 7 : 2
than the number of diesel engine cars in state-1?
(a) 100 (b) 200 GENERAL AWARENESS,
(c) 300 (d) 125
(e) 225 MARKETING & COMPUTERS
93. If 25% of diesel engine cars in state-3 are AC and remaining
cars are non-AC, what is the number of diesel engine cars in 101. Who is the Captain of Australian Cricket Team, which
currently (March 2013) visited India?
state-3 which are non-AC?
(a) Michael Clarke (b) Shane Watson
(a) 75 (b) 45 (c) Shane Warne (d) Michael Hussey
(c) 95 (d) 105 (e) Ricky Ponting
(e) 35 102. Government as part of the recaptalisation plan, infused capital
94. What is the difference between the total number of cars in in State Bank of India recently. Indicate the approximate
state-3 and the number of petrol engine cars in state-2? capital infusion.
(a) 9 6 (b) 10 6 (a) ` 500 cr. (b) ` 1,000 cr.
(c) 112 (d) 10 2 (c) ` 1,500 cr. (b) ` 2,000 cr.
(e) 9 8 (e) ` 3,000 cr.
y
o
u
rs
m
230 SBI PO Exam 2013

a
h
b
103. The NRIs while investing in the equity of a company cannot 113. Banking Ombudsman is appointed by _____________.

o
buy more than prescribed percentage of the paid up capital (a) Government of India (b) State Governments

o
b
of that company. What is the prescribed percentage? (c) RBI (d) E C G C

.w
(a) 2% (b) 3% (c) 4% (d) 5% (e) 6% (e) Exim Bank

o
rd
104. Every year March 20 is celebrated as what day? 114. The Holidays for the Banks are declared as per _________.

p
(a) World Sparrow Day (a) Reserve Bank Act

re
(b) International Women’s Day (b) Banking Regulation Act

s
s
(c) World Cuckoo Day (c) Negotiable Instruments Act

.c
(d) International Child Day

o
(d) Securities and Exchange Board of India Act

m
(e) International Mother’s Day
(e) Companies Act
105. One of the famous Indian Sportsperson released his/her
115. Interest on Savings deposit nowadays is _____________.
autobiography - ‘Playing to Win’ in November 2012. Name
(a) Fixed by RBI
the sports person correctly.
(a) Saina Nehwal (b) Mary Kom (b) Fixed by the respective Banks
(c) Yuvraj Singh (d) Sachin Tendulkar (c) Fixed by the Depositors
(e) Sushil Kumar (d) Fixed as per the contract between Bank and the
106. Which of the following term is associated with the game of Consumer Court
Cricket? (e) Not paid by the Bank
(a) Bouncer (b) Love 116. Interest below which a bank is not expected to lend to
(c) Goal (d) Mid Court customers is known as _____________.
(e) Collision (a) Deposit Rate (b) Base Rate
107. Who is the author of the book ‘Women of Vision’? (c) Prime Lending Rate (d) Bank Rate
(a) Ravinder Singh (b) Preeti Shenoy (e) Discount Rate
(c) Amish Tripathi (d) Durjoy Dutta
117. The customers’ by opening and investing in the Tax Saver
(e) Alam Srinivas
Deposit Account Scheme in a Bank would get benefit under
108. Prof. Muhammad Yunus, the receipient of the Nobel Peace
Prize 2006 is the exponent of which of the following concepts _____________.
in the field of banking? (a) Sales Tax (b) Customs Duty
(a) Core Banking (c) Excise Duty (d) Professional Tax
(b) Micro Credit (e) Income Tax
(c) Retail Banking 118. In banking business, when the borrowers avail a Term Loan,
(d) Real Time Gross Settlement initially they are given a repayment holiday and this is referred
(e) Internet Banking as _____________.
109. Invisible Export means export of _____________.
(a) Subsidy (b) Interest Waiver
(a) Services
(b) Prohibited goods (c) Re-phasing (d) Interest concession
(c) Restricted goods (e) Moratorium
(d) Goods as per OGL list 119. One of IT company from India has become the first Indian
(e) Other than those given as options. Company to trade on NYSE Euronext London and Paris
110. The European Union has adopted which of the following as markets from February 2013 onwards. Which is this
a common currency? company?
(a) Dollar (b) Dinar (a) Wipro Infotech Ltd.
(c) Yen (d) Euro
(b) L & T Infotech
(e) Peso
111. Banks in India are required to maintain a portion of their (c) HCL Technologies Limited
demand and time liabilities with the Reserve Bank of India. (d) Infosys Technologies Limited
This portion is called _____________. (e) Polaris Financial Technology Ltd.
(a) Statutory Liquidity Ratio 120. BSE (Bombay Stock Exchange), the oldest stock exchange
(b) Cash Reserve Ratio in Asia has joined hands with one more international index
(c) Bank Deposit in February 2013. This association has resulted in change of
(d) Reserve Repo name of BSE index. What is the change of name effected?
(e) Government Securities
(a) Dow Jones BSE Index
112. Pre-shipment finance is provided by the banks only to
_____________. (b) NASDAQ BSE Index
(a) Credit Card Holders (c) S & P BSE Index
(b) Students aspiring for further studies (d) Euronext BSE Index
(c) Brokers in equity market (e) Other than those given as options.
(d) Exporters
y
o
u
rs
m
SBI PO Exam 2013 231

a
h
b
121. A non-performing asset is _____________. 129. Which of the following uses a handheld operating system?

o
(a) A supercomputer (b) A personal computer

o
(a) Money at call and short notice.

b
(c) A laptop (d) A mainframe

.w
(b) An asset that ceases to generate income
(c) Cash balance in till (e) A PDA

o
rd
130. To display the contents of a folder in Windows Explorer you
(d) Cash balance with RBI

p
should __________.

re
(e) Balance with other banks
(a) click on it (b) collapse it

s
122. RBI has released its guidelines for entry of new banks in the

s
(c) name it (d) give a password

.c
private sector in the month of February 2013. One of the

o
(e) rename it

m
norms is at least a prescribed percentage of branches of new 131. The CPU comprises of Control, Memory, and ________
bank should be set in unbanked rural centres with a units.
population of upto 9,999 people. What is the percentage of (a) Microprocessor (b) Arithmetic/Logic
such branches prescribed in the norms? (c) Output (d) ROM
(a) 10% (b) 15% (e) Input
(c) 18% (d) 25% 132. __________ is the most important/powerful computer in a
(e) Other than those given as options typical network.
123. The Union Budget for 2013-14 proposed by the Finance (a) Desktop (b) Network client
Minister on 28th February 2013 announced introduction of (c) Network server (d) Network station
new variety of bonds by the Government. What is the name (e) Network switch
of these bonds? 133. A(n) ________ appearing on a web page opens another
(a) Deep discount bonds (b) Zero Coupon bonds document when clicked.
(c) Bullet Bonds (d) Inflation Indexed Bonds (a) anchor (b) URL
(e) Inflation Variable Bonds (c) hyperlink (d) reference
124. Government usually classifies its expenditure in term of (e) heading
planned and non-planned expenditure. Identify, which is the 134. Which of the following refers to the rectangular area for
correct definiton of Planned expenditure? displaying information and running programs?
(a) It represents the expenditure of all the State (a) Desktop (b) Dialog box
Governments. (c) Menu (d) Window
(e) Icon
(b) It represents the total expenditure of the Central
135. __________ is a Window utility program that locates and
Government.
eliminates unnecessary fragments and rearranges files and
(c) It is the expenditure which is spent through centrally
unused disk space to optimize operations.
sponsored programmes and flagship schemes of
(a) Backup (b) Disk Cleanup
Government.
(c) Disk Defragmenter (d) Restore
(d) It represents the expenditure incurred on Defence.
(e) Disk Restorer
(e) Other than those given as options.
136. Which of the following refers to too much electricity and
125. Which of the following organization is made specifically
may cause a voltage surge?
responsible for empowering Micro, Small and Medium
(a) Anomaly (b) Shock
enterprises in India?
(c) Spike (d) Virus
(a) NABARD (b) RBI
(e) Splash
(c) SIDBI (d) E C G C 137. The software that is used to create text-based documents
(e) SEBI are referred to as ___________.
126. ‘C’ in CPU denotes _____________. (a) DBMS (b) suites
(a) Central (b) Common (c) spreadsheets (d) presentation software
(c) Convenient (d) Computer (e) word processors
(e) Circuitry 138. ___________ devices convert human-understandable data
127. A joystick is primarily used to/for ___________. and programs into a form that the computer can process.
(a) Control sound on the screen (a) Printing (b) Output
(b) Computer gaming (c) Solid State (d) Monitor
(c) Enter text (e) Input
(d) Draw pictures 139. Effective Selling Skills depends on __________
(e) Print text (a) Number of languages known to the DSA
128. Which is not a storage device? (b) Data on marketing staff
(a) A CD (b) A DVD (c) Information regarding IT market
(c) A floppy disk (d) A printer (d) Knowledge to related markets
(e) A Hard disk (e) Ability to talk fast
y
o
u
rs
m
232 SBI PO Exam 2013

a
h
b
140. A Direct Selling Agent (DSA) is required to be adept in
GENERAL AWARENESS

o
__________.

o
b
(a) Surrogate marketing (b) Training skills

.w
DIRECTIONS (Qs.151-160): Read the following passage
(c) Communication skills (d) Market Research

o
carefully and answer the questions given below it. Certain words

rd
(e) OTC Marketing have been printed in bold to help you locate them while

p
141. Leads can be best sourced from __________. answering some of the questions.

re
(a) Foreign customers (b) Yellow pages

s
s
(c) Dictionary (d) List of vendors Financial Inclusion (FI) is an emerging priority for banks that

.c
(e) Local supply chains have nowhere else to go to achieve business growth. The viability

o
m
142. A successful marketing person requires one of the following of FI business is under question, because while banks and their
qualities __________. deliver partners continue to make investments, they haven’t seen
(a) Empathy (b) Sympathy commensurate returns. In markets like India, most programs are
(c) Insistence (d) Aggressiveness focused on customer on-boarding, an expensive process which
(e) Pride people often find difficult to afford, involving issuance of smart
143. Innovation in marketing is same as _________. cards to the customers. However, large scale customer acquisition
(a) Abbreviation (b) Communication hasn’t translated into large scale business, with many accounts
(c) Creativity (d) Aspiration lying dormant and therefore yielding no return on the bank’s
(e) Research work investment. For the same reason, Business Correspondent Agents
144. Market segmentation can be resorted to by dividing the target who constitute the primary channel for financial inclusion are
group as per _________. unable to pursue their activity as a full-time job. One major reason
(a) Income levels of customers for this state of events is that the customer on-boarding process
(b) Age of the employees is often delayed after the submission of documents (required to
(c) Needs of the sales persons validate the details of the concerned applicant) by the applicant
(d) Marketing skills of the employees and might take as long as two weeks. By this time the initial
(e) Size of the Organisation enthusiasm of applicants fades away. Moreover, the delivery
145. Post-sales activities include _________. partners don’t have the knowledge and skill to propose anything
(a) Sales presentation other than the most basic financial products to the customer and
(b) Customer Feedback hence do not serve their banks’ goal of expanding the offering in
(c) Customer identification unbanked markets.
(d) Product design Contrary to popular perception, the inclusion segment is not a
146. The ‘USP’ of a product denotes __________. singular impoverished, undifferentiated mass and it is important
(a) Usefulness of the product to navigate its diversity to identify the right target customers for
(b) Drawbacks of the product various programs. Rural markets do have their share of rich people
(c) Main functions who do not use banking services simply because they are
(d) Number of allied products available incovenient to access or have low perceived value. At the same
(e) High selling features of a product
time, urban markets, despite a high branch density, have multitude
147. The competitive position of a company can be improved by
of low wage earners outside the financial net. Moreover, the
__________.
(a) increasing the selling price branch timings of banks rarely coincide with the off-work hours
(b) reducing the margin of the labour class.
(c) ignoring competitors Creating affordability is crucial in tapping the unbanked market.
(d) increasing the cost price No doubt pricing is a tool, but banks also need to be innovative in
(e) understanding and fulfilling customers’ needs right-sizing their proposition to convince customers that they
148. Value added services means _________. can derive big value even from small amounts. One way of doing
(a) Low Cost products this is to show the target audience that a bank account is actually
(b) High cost products a lifestyle enabler, a convenient and safe means to send money -
(c) At par services to - family or make a variety of purchases. Once banks succeed in
(d) Additional services for the same cost hooking customers with this value proposition they must sustain
(e) Giving discounts their interest by introducing a simple and intuitive user application,
149. The target market for Debit Cards is _________. unbiquitous access over mobile and other touch points, and
(a) All existing account-holders adopting a banking mechanism which is not only secure but also
(b) All agriculturists reassuring to the customer. Technology is the most important
(c) All DSAs
element of financial inclusion strategy and an enabler of all other.
(d) All outsourced agents
The choice of technology is therefore a crucial decision, which
150 A good Brand can be built up by way of ________.
(a) Customer grievances could make or mar the agenda. Of the various selection criteria,
(b) Break-down of IT support cost is perhaps the most important. This certainly does not mean
(c) Old age buying the cheapest package, but rather choosing that solution
(d) Large number of products which by scaling transactions to huge volumes reduces per unit
(e) Consistent offering of good services
y
o
u
rs
m
SBI PO Exam 2013 233

a
h
b
operating cost. An optimal mix of these strategies would no doubt 156. According to the passage, which of the following ways may

o
offer an innovative means of expansion in the unbanked market. help banks to sustain the interest of their customers after

o
b
151. Which of the following facts is true as per the passage? hooking them?

.w
(a) People from rural areas have high perceived value of (A) Adoption of a banking mechanism which is not only

o
banking services.

rd
secure but reassuring to the customers
(b) Cost is not a valid criterion for technological package

p
(B) Increasing the number of delivery partners in rural

re
selection for financial inclusion initiatives
market

s
(c) The inclusion segment is a singular impoverished,

s
(C) Introduction of a simple and intuitive user application

.c
undifferentiated mass

o
(d) The branch timings of banks generally do not coincide (a) Only (A) (b) Only (C)

m
with the off-work hours of the labour class in urban (c) Only (B) (d) All, (A), (B) and (C)
markets (e) Both (A) and (C)
(e) All the given statements are true DIRECTIONS (Qs.157-158): Choose the word which is most similarly
152. According to the passage, for which of the following reasons in meaning to the word printed in bold as used in the passage.
do the delivery partners fail to serve their bank’s goal to
expand in the unbanked markets? 157. Multitude
(A) They do not have adequate client base to sell their (a) Impoverished (b) Handful
financial products (c) Acknowledged (d) Plenty
(B) They do not have adequate knowledge and skills to (e) Solitude
explain anything beyond basic financial products to
158. Ubiquitous
the customers.
(a) Quintessential (b) Popular
(C) They do not have the skills to operate advanced
technological aids that are a prerequisite to tap the (c) Omnipresent (d) Simplified
unbanked market. (e) Abnormal
(a) Only (B) (b) Only (C) DIRECTIONS (Qs.159-160): Choose the word which is most similarly
(c) All (A), (B) and (C) (d) Only (A) in meaning to the word printed in bold as used in the passage.
(e) (B) and (C)
153. According to the passage, for which of the following reasons 159. Dormant
is the viability of financial inclusion under question? (a) Emaciated (b) Pertinent
(a) The banks always prefer the cheapest package (to cut (c) Cornered (d) Rejected
cost) while making a choice of technology to be used (e) Active
(b) The Business Correspondent Agents are highly 160. Delayed
demotivated to pursue their activity as a full-time job (a) Perturbed (b) Popularised
(c) The investments made by banks and its delivery (c) Expedited (d) Stabilised
partners are not yielding equal amount of returns
(e) Repressed
(d) The banks do not have adequate number of delivery
partners required to tap the unbanked market DIRECTIONS (Qs.161-170): Read the following passage
(e) The banks do not have adequate manpower to explore carefully and answer the questions given below it. Certain words
the diversity of right target customers for various have been printed in bold to help you locate them while
programs answering some of the questions.
154. In the passage, the author has specified which of the
following characteristics of the customer on-boarding The evolution of Bring Your Own Device (BYOD) trend has been
process? as profound as it has been rapid. It represents the more visible
(a) It involves collection of documents from the applicants sign that the boundaries between personal life and work life are
in order to validate their details. blurring. The 9 am - 5 pm model of working solely from office has
(b) It involves issuance of smart cards to the customers become archaic and increasingly people are working extended
(c) It suffers from latency as it takes a long time after hours from a range of locations. At the vary heart of this evolution
submission of documents by the customer. is the ability to access enterprise networks from anywhere and
(d) It is an expensive process which people find difficult anytime. The concept of cloud computing serves effectively to
to afford
extend the office out of office. The much heralded benefit of BYOD
(e) All of the given characteristics have been specified
is greater productivity. However, recent research has suggested
155. What did the author try to highlight in the passage?
(A) The ailing condition of financial inclusion business at that this is the greatest myth of BYOD and the reality is that
present BYOD in practice poses new challenges that may outweigh the
(B) Strategies that may help bank to expand in the benefits. A worldwide survey commissioned by Fortinet chose to
unbanked market look at attitudes towards BYOD and security from the users’s
(C) Role of government in modifying the existing financial point of view instead of the IT managers. Specifically the survey
inclusion policies was conducted in 15 territories on a group of graduate employees
(a) Both (A) and (B) (b) All (A), (B) and (C) in their early twenties because they represent the first generation
(c) Only (C) (d) Only (A) to enter the workplace with an expectation of own devide use.
(e) Only (B)
y
o
u
rs
m
234 SBI PO Exam 2013

a
h
b
Moreover, they also represent tomorrow’s influences and decision of devices

o
makers. The survey findings reveal that for financial organizations, (c) They may lack mature IT strategies and policies

o
b
the decision to embrace BYOD is extremely dangerous. Larger required to protect confidential data

.w
organizations will have mature IT strategies and policies in place. (d) They cannot afford to deal with damage liability issues

o
of employee-owned devices

rd
But what about smaller financial businesses? They might not

p
have such well developed strategies to protect confidential data. (e) Their employees have a tendency to change jobs

re
Crucially, within younger employee group, 55% of the people frequently

s
s
share an expectation that they should be allowed to use their own 164. According to the passage, the expectation of younger

.c
employees that they should be allowed to use their own

o
devices in the workplace or for work purposes. With this

m
expectation comes the very real risk that employees may consider devices in the workplace, entails which of the following risks?
contravening company policy banning the use of own devices. (A) Younger employees may deliberately transfer
The threats posed by this level of subversion cannot be confidential data of their companies to rivals if they
overstated. The survey casts doubt on the idea of BYOD leading are not allowed to use their own devices for work
to greater productivity by revealing the real reason people want purpose
to use their own devices. Only 26% of people in this age group (B) Younger employees may strongly feel like leaving the
cite efficiency as the reason they want to use their own devices, company if it presents usage of own device and join
while 63% admit that the main reason is so they have access to some other company that does not have such stringent
their favourite applications. But with personal applications so policies
close to hand, the risks to the business must surely include (C) Younger employees may consider flouting company
distraction and time wasting. To support this assumption 46% of policy prohibiting usage of their own devices in the
people polled acknowledged time wasting as the greatest threat workplace or for work purposes
to the organization, while 42% citing greater exposure to theft or (a) Only (C) (b) Only (B)
loss of confidential data. Clearly, from a user perspective there is (c) Both (A) and (C) (d) Only (A)
great deal of contradiction surrounding BYOD and there exists an (e) All (A), (B) and (C)
undercurrent of selfishness where users except to use their own 165. According to the findings of the survey conducted by
devices, but mostly for personal interest. They recognize the risks Fortinet, why do majority of employees prefer using their
to the organization but are adamant that those risks are worth own devices for work purpose?
taking. (a) As they often find that the devices provided by the
161. According to the passage, for which of the following reasons company lack quality
did Fortinet conduct the survey on a group of graduate (b) As they have access to their favourite applications
employees in their early twenties? while working
(A) As this group represents the future decision makers (c) As majority of them believe that output doubles when
(B) As this group represents the first generation who they use their own devices for work purpose
entered the workforce with a better understanding of (d) As handling data from their own devices reinforces
sophisticated gadgets their sense of responsiblity
(C) As this group represents the first generation to enter (e) None of these
the workplace expecting that they can use their own 166. What is/are the author’s main objective(s) in writing the
devices for work purpose passage?
(a) All (A), (B) and (C) (b) Only (C) (A) To break the myth that BYOD promotes employee
(c) Both (A) and (C) (d) Only (A) efficiency and organisational productivity
(e) Only (B) (B) To suggest ways to control subversion across levels
162. Which of the following is not true about BYOD? of corporate chain of command
(a) BYOD enables employees to access enterprise network (C) To throw light upon the fact that employees even after
from anywhere and anytime knowing the risks involved, prefer to use their own
(b) Due to evolution of BYOD trend the 9am - 5pm model devices for work purpose mostly for personal benefits
of working solely from office has become outdated (a) Both (A) and (C) (b) All (A), (B) and (C)
(c) Recent research has confirmed that BYOD boosts (c) Only (C) (d) Only (A)
organisational productivity (e) Only (B)
(d) The concept of cloud computing facilates the BYOD DIRECTIONS (Qs.167-168): Choose the word which is most similar
trend in meaning to the word printed in bold as used in the passage.
(e) None of these
163. According to the passage, why would the decision to 167. Heralded
embrace BYOD prove dangerous to smaller financial (a) Suspected (b) Publicised
(c) Dragged (d) Objective
businesses?
(e) Benefit
(a) Their employees have poor knowledge about their
168. Outweigh
devices which in turn pose a threat to the confidential
(a) Control (b) Venture
data of the organisation (c) Perish (d) Determine
(b) Their employees are more vulnerable to misplacement (e) Surpass
y
o
u
rs
m
SBI PO Exam 2013 235

a
h
b
DIRECTIONS (Qs.169-170): Choose the word which is most opposite 180. Lack of financing options, __________ with HR and

o
technological __________ , make small and medium

o
in meaning to the word printed in bold as used in the passage

b
enterprises sector the most vulnerable component of our

.w
169. Embrace economy.

o
(a) Contradict (b) Disobey

rd
(a) except, loophole (b) coupled, challenges
(c) Curtail (d) Reject

p
(c) armed, benefits (d) registered, strategies

re
(e) Obscure (e) None of these

s
170. Subversion

s
DIRECTIONS (Qs.181-185): Rearrange the following six

.c
(a) Compliance (b) Sanity

o
sentences (A), (B), (C), (D) and (F) in the proper sequence to form
(c) Popularity (d) Destabilisation

m
a meaningful paragraph, then answer the questions given below
(e) Clarity
them.
DIRECTIONS (Qs.171-175): Read each sentence to find out
whether there is any grammatical mistake/error in it. The error if (A) The group desired to enhance the learning experience in
any, will be in any part of the sentence. Mark the number of that schools with an interactive digital medium that could be used
part with error as your answer. If there is ‘No error’, mark (e). within and outside the class-room
(B) Then the teacher can act on the downloaded data rather
171. There cannot be any situation where/(a) somebody makes
than collect it from each and every student and thereby
money in an asset/ (b) located in India and does not pay tax/
(c) either to India or to the country of his origin./ (d) No error save his time and effort.
(e). (C) Eductor, decided the group of engineers, all alumni of the
172. India has entered a downward spiral / (a) where the organised, Indain Institute of technology, when the founded Edutor
productive/ (b) and law abide sectors are subjec to / (c) Technologies in August 2009.
sevage amounts of multiple taxes./ (d) No error (e). (D) They can even take tests and submit them digitally using
173. The bank may have followed/ (a) an aggressive monetary the same tablets and the teachers in turn can download the
tightening policy/ (b) but its stated aim of / (c) curbin inflation tests using the company’s cloud services.
have not been achieved/ (d) No error (e). (E) With this desire they created a solution that digitzes school
174. Equal opportunities for advancement/ (a) across the length textbooks and other learning materials so that students no
and breadth / (b) of an organisation will/ (c) keep many longer need to carry as many books to school and back as
problems away. /(d) No error (e).
before, but can access their study material on their touch-
175. A customised data science degree/ (a) is yet to become/(b) a
screen tablets.
standard programme/ (c) to India’s premier educational
institutes./ (d) No error (e) (F) A mechanic works on motors and an accountant has his
computer. Likewise, if a student has to work on a machine or
DIRECTIONS (Qs.176-180): Each question below has two device, what should it be called?
blanks, each blank indicating that something has been omitted. 181. Which of the following sentences should be the FIRST after
Choose the set of words for each blank that best fits the meaning rearrangement?
of the sentence as a whole. (a) F (b) D
176. When you want to digitalise a city __________ with millions, (c) A (d) C
you don’t bet __________ the odds. (e) E
(a) proceeding, into (b) teeming, against 182. Which of the following sentences should be the THIRD
(c) undergoing, adhere (d) dangling, for after rearrangement?
(e) falling, above (a) A (b) B
177. The numbers __________ by the legitimate online music
(c) D (d) E
service providers indicate that a growing number of users
are __________ to buy music. (e) F
(a) morphed, ignoring (b) labelled, thriving 183. Which of the following sentences should be the SIXTH
(c) figured, fanatic (d) painted, interested (LAST) after rearrangement?
(e) touted, willing (a) A (b) F
178. In India is __________ on protecting its resources, (c) E (d) B
international business appears equally __________ to (e) D
safeguard its profit. 184. Which of the following sentences should be the FOURTH
(a) dreaded, fragile (b) stubborn, weak after rearrangement?
(c) bent, determined (d) approaching, settled
(a) A (b) F
(e) None of these
179. Brands __________ decision-simplicity strategies make (c) E (d) B
full use of available information to __________ where (e) D
consumers are on the path of decisions making and direct 185. Which of the following sentences should be the FIFTHafter
them to the best market offers. rearrangement?
(a) diluting, divulge (b) tempting, maintain (a) A (b) D
(c) imputing, overdrive (d) pursuing, assess (c) C (d) E
(e) employing, trust (e) F
y
o
u
rs
m
236 SBI PO Exam 2013

a
h
b
DIRECTIONS (Qs.186-195): In the following passage there are 193. (a) aid (b) jeopardise

o
(c) promotes (d) endure

o
blanks, each of which has been numbered. These numbers are

b
printed below the passage and against each, five words/phrases (e) cater

.w
are suggested, one of which fits the blank appropriately. Find 194. (a) acute (b) utilising

o
(c) restricting (d) inspiring

rd
out the appropriate word/phrase in each case.
(e) increased

p
re
There is a considerable amount of research about the factors that 195. (a) both (b) besides

s
make a company innovate. So is it possible to create an environment (c) combining (d) participating

s
.c
(186) to innovation? This is a particularly pertinent (187) for (e) also

o
India today. Massive problems in health, education etc. (188) be

m
DIRECTIONS (Qs.196-200): In each of the following questions
solved using a conventional approach but (189) creative and four words are given of which two words are most nearly the
innovative solutions that can ensure radical change and (190). same or opposite in meaning. Find the two words which are
There are several factors in India’s (191). Few countries have the most nearly the same or opposite in meaning and indicate the
rich diversity that India or its large, young population (192). While number of the correct letter combination, by darkening the
these (193) innovation policy interventions certain additional steps appropriate oval in your answer sheet.
also required. These include (194) investment in research and
development (195) the government and the private sector, easy 196. (A) Consent (B) Nascent
transfer of technology form the academic world etc. To fulfill its (C) Emerging (D) Insecure
promise of being prosperous and to be at the forefront, India (a) A–C (b) B-D
must be innovative. (c) B–D (d) A–D
186. (a) stimuli (b) conducive (e) A–B
(c) incentive (d) facilitated 197. (A) Elated (B) Eccentric
(e) impetus (C) Explicit (D) Abnormal
187. (a) objective (b) controversy (a) A–B (b) B-D
(c) doubt (d) question (c) A–C (d) A–D
(e) inference (e) D–C
188. (a) cannot (b) possibly 198. (A) Abundance (B) Incomparable
(c) should (d) never (C) Projection (D) Plethora
(e) must (a) A–C (b) A-B
189. (a) necessary (b) apply (c) C–D (d) B–D
(c) need (d) consider (e) A–D
(e) requires 199. (A) Purposefully (b) Inaccurately
190. (a) quantity (b) advantages (C) Inadvertently (d) Unchangeably
(c) increase (d) chaos (a) A–C (b) A–B
(e) growth (c) B–C (d) B–D
191. (a) challenges (b) praises (e) A–D
(c) favour (d) leverage 200. (A) germane (B) generate
(e) esteem (C) reliable (D) irrelevant
192. (a) blessed (b) enjoys (a) B–D (b) B–C
(c) endows (d) prevails (c) A–B (d) C–D
(e) occurs (e) A–D
y
o
u
rs
m
SBI PO Exam 2013 237

a
h
b
o
o
b
.w
o
rd
1. (a) R > O = A > S > T

p
6. (c) 7. (a) 8. (a)

re
or, O > S > T 9. (c) 10. (b)

s
Therefore, O > T

s
(11-15)

.c
2. (b) P > L > A ³ N = T Economics is not money - ka la ho ga .......(i)

o
(d) B > L = O = N ³ D

m
3. demand and supply economics - mo ta pa ka ....... (ii)
4. (e) By options: money makes only part - zi la ne ki ...... (iii)
(1) L < N < P > A i.e., A < P, definitely true demand makes supply economics - zi mo ka ta ... (iv)
(b) L < A < P > N i.e., A < P, definitely true From (i) & (iii)
(c) A < L < P > N i.e., A < P, definitely true money ® ‘la’
(d) N < A < P > L i.e., A < P, definitely true From (iii) & (iv)
(e) P < N < A > L i.e., A > P, definitely false makes ® ‘zi’
5. (c) F < O = U = N < D From (i) & (iv)
(6-10): economics – ‘ka’
Also, and - ‘pa’
Standard Subject demand - ‘mo’ or ‘ta’
supply - ‘mo’ or ‘ta’
A 5th Maths only - ‘ne’ or ‘ki’
B 7th Hindi or English part - ‘ne’ or ‘ki’
is - ‘ho’ or ‘ga’
C 5th Marathi not - ‘ho’ or ‘ga;
11. (e)
D 6th Economics
12. (e)
E 6th Civics 13. (a)
14. (b)
G 7th Hindi or English 15. (d)
I 7th History

16-20:

Input 32 proud girl beautiful 48 55 97 ric h family 61 72 17 nice life


Step I beautiful 17 32 proud girl 48 55 97 rich family 61 72 nice life
Step II family 32 beautiful 17 proud girl 48 55 97 rich 61 72 nice life
Step III girl 48 family 32 beautiful 17 proud 55 97 rich 61 72 nice life

Step IV life 55 girl 48 family 32 beautiful 17 proud 97 rich 61 72 nice


Step V nice 61 life 55 girl 48 family 32 beautiful 17 proud 97 rich 72

Step VI proud 72 nice 61 life 55 girl 48 fa mily 32 beautiful 17 97 rich


Step VII r ich 97 proud 72 nice 61 life 55 girl 48 family 32 beautiful 17

21. (a) J ¸ P % H × T % L can be represented in diagram. As


16. (c) 17. (d) 18. (c) 19. (a) follows.
20. (b)
(21-25):
A +B Þ A is the father of B. P
- Male
A ×B Þ A is the sister of B.
A $B Þ A is the wife of B. - Female
J = H = T
A %B Þ A is the mother of B. = - Sibling Relation
A ¸B Þ A is the son of B.
- Spouse Relation
L
y
o
u
rs
m
238 SBI PO Exam 2013

a
h
b
22. (b) L + R $ D + M × T 26. (b)

o
27. (d)

o
b
28. (c)

.w
L
29. (b)

o
rd
30. (a)

p
R D

re
s
31-32:

s
.c
M =T

o
m
Squares Circles Line
23. (b) I + T % J × L ¸ K

I line
Triangles

T K

Triangles
J = L

24. (d) W $ X + L + Y + T
31. (a) Conclusion I - True
Conclusion II - False
W X 32. (e)
33-34:

L
Rhymes
Poems
Y

Songs Paragraph
T

25. (d) R % T × P $ Q + V

R
33. (e) Conclusion I - True
Conclusion II - True
T = P Q 34. (b) Conclusion I - False
Conclusion II - True
V
Stones
(26-30):
(Columnist)
(Chartered M Drops
35. (e) Dews
Accountant)E L (Engineer)

(Lawyer) J K (Professor) Conclusion I - True


Conclusion II - True
36. (e) Combining statements I and II together. We get
position of Bhavna
G H (Scientist)
(Financial 17 11 9 7 4
F
Analyst) (Doctor) R L
Sheetal Sandeep Anita Bhavna
y
o
u
rs
m
SBI PO Exam 2013 239

a
h
b
44. (d) Clearly A as mobiles used as cheating.

o
L R 45. (a) B among the given statements is the only course of

o
37. (c) Statement I :

b
R A N G E action for the invigilator.

.w
46. (d) 47. (c) 48. (b) 49. (b) 50. (c)

o
L R

rd
51. (b) Difference between cost of 1 kg apple and cost of 1 kg
Statement II:

p
A N G guava in 5 cities.

re
J 160 – 60 = 100

s
s
D 130 – 90 = 40

.c
38. (b) Statement I : T / V C 180 – 120 = 60

o
m
V / T H 90 – 30 = 60
R 40 – 20 = 20
\ Cost is second lowest in Delhi.
52. (d) Cost of 1 kg guava in Jalandhar = ` 60
U W Cost of 2 kg grapes in chandigarh = ` 90 × 2 = ` 180
60
%= ´ 100 = 33.3 » 34%
180
53. (c) Cost of 3 kgs apples for Ram = 3 × 130 = ` 390
X Cost of 2 kgs guavas for Ram = 2 × 90 = ` 180
Total cost that Ram pay = 390 + 180 = ` 570
From Statement I: We cannot get the exact position of 54. (a) Total cost of 45 kgs grapes from Hoshiarpur = 45 × 190
T. = ` 8550
8550 ´ 4
Statement II: S After discount 4% Ravinder paid = 8550 –
100
X = ` 8208
55. (c) Cost of 1 kg apples from Ropar :
Cost of 1kg grapes from chandigarh
40 : 90
4 : 9 or 22 : 32
56. (a) Number of students in university I in 2010 = 20,000
Number of students in university II in 2012 = 20,000
V Difference = 20,000 – 20,000 = 0
T 57. (e) Number of students in university I in 2007 = 10,000
Number of students in university II in 2011 = 30,000
From Statement II: We get the position of T and X. Total students = 10,000 + 30,000 = 40,000
39. (e) In online shopping the customer may be deceived as 58. (a) Total no. of students in Uni-2 in 2010 = 15,000
he cannot touch the product he is paying for. No. of females students = 25% of 15000
40. (d) The customer whose view is presented has shopped
25
at retail stores as well as online. = ´ 15, 000 = 3750
41. (e) The number of people migrating from rural to urban 100
areas in search of jobs may reduce. No. of male students = 15,000 – 3750 = 11250
42. (a) At least some people who visits the park have pets. 59. (d) Number of students in university I in 2011 = 25,000
43-45. Number of students in university I in 2010 = 20,000
A supports the given information as Mobile phones can aid Increase = 25000 – 20000 = 5000
in cheating so it has been banned in the examination hall. 5000
B is an immediate course of action the invigilator should Increase % = ´ 100 = 25%
20000
take as the 3 students have violated the rule. It does not
matter whether they were using their mobile to cheat at that 60. (e) Difference between Number of students in university I
time or not. & Number of students in university II for the year
C is a reason because of which students would not like to 2007 – 10,000
leave their mobile in their bag outside classroom. 2008 – 10,000
D is again a reason as C. 2009 – 10,000
E can be a possible course of action for student's worry of 2010 – 5,000
their mobile being stolen while they are in the examination 2011 – 5,000
hall. 2012 – 15,000
F is statement which supports the rule. \ Difference was highest in year 2012.
43. (c) Clearly C & D as stated above.
y
o
u
rs
m
240 SBI PO Exam 2013

a
h
b
61. (b) No. of hockey players in all schools 3

o
= 68 + 80 + 54 + 50 + 36 = 288 75. (a) th of the teachers are not permanent

o
5

b
62. (c) No. of basket-ball players in school-1 = 42

.w
No. of Kho-Kho players in school-3 = 48

o
2
\ th of the teachers are permanent

rd
Ratio = 42 : 48 = 7 : 8
5

p
63. (b) No. of player = 80 + 22 = 102

re
64. (d) No. of Kho-Kho players in school-4 = 32 26800 ´15 2

s
´ = 1608

s
No. of hockey players in school-2 = 80 \ No. of permanent teachers =

.c
100 5

o
32

m
´100 = 40% 76. (c) Total amount paid by Dev in June for all commodities =
%=
80 123 + 150 + 324 + 134
65. (e) No. of hockey players in school-5 = 36 = ` 731
No. of females = 25%, No. of males = 75% 77. (c) Average electricity bill paid by Manu in all five months
36 ´ 75 = 315 + 135 + 98 + 116 + 131
No. of males = = 27
100 795
= = ` 159
66. (b) Gita's average earning over all the day's 5
400 + 420 + 200 + 140 1160 78. (a) Mobile phone bill paid by Ravi in May = ` 143
= = = ` 290
4 4 Loundry bill paid by Dev in March = ` 323
67. (d) Rahul earned on Tuesday and Thursday Difference = 323 – 143 = ` 180
= 180 + 340 = ` 520 79. (d)
Total = 520 + 420 = ` 940 80. (a) Electricity bill paid by Manu in April = 135
68. (c) Gita donated Naveen = ` 420
Mobile bill paid by Ravi in June = 245
Naveen's total earning on Wednesday
= 420 + 120 = ` 540 Ratio = 135 : 245 = 27 : 49
69. (a) Rahul's earning on Monday = ` 240 81. (c) Distance travelled by train from Surat to Nadiad Jn.
Gita's earning on Tuesday = ` 200 = 440 – 257 = 183 kms
Diff = 240 – 200 = ` 40 82. (a) Time taken to reach Ahmedabad
70. (e) Naveen's earning on Monday, Wednesday and = 8 : 00 Am – 6 : 43 Am = 1 hour 17 min.
Thursday
83. (a) Ratio between No. of passengers boarding from Vasai
= 360 : 120 : 180
Road and from Ahmedabad
=6:2:3
71. (c) No. of employees in teaching and medical = 42% = 378 : 306 = 21 : 17
No. of employees in management = 17% 84. (b) Total time increase = 23 – 2 = 21 min.
Difference = 42 – 17 = 25% \ Train will reach Bhuj at = 5:40PM + 21 min = 6:01 PM
26800 ´ 25 85. (d) Distance between Anand Jn. to Vadodara is second
= = 6700 lowest.
100
86. (e) Difference between the max. temperature of Ontario on
3 1st Nov. and the min. temperature of Bhuj on 1st Jan. =
72. (e) In management employees are female 4 – (7) = 11°C
4
87. (b)
1
\ employees in management are male 88. (c) Difference between max. and min. temperature of Bhuj
4
Sept. = 10°C
26800 ´17 1 Oct. = 14°C
\ No. of male employees = ´ = 1139
100 4 Nov. = 11°C
73. (b) In film production employees on strike = 25% Dec. = 7°C
No. of employees not in the strike = 75%
Jan. = 3°C
\ No. of employees not in strike
89. (e) Average of max. temperature of Beijing over all the
26800 ´19 75
= ´ = 3819 43
100 100 months = = 8.6°C
5
74. (d) Total No. of employees in engineering industries
= 9 + 13 = 22% 90. (b) Ratio between minimum temperature of Beijing on 1st
sept. and maximum temperature of Ontario on 1st Oct.
26800 ´ 22 = 9 : 15 = 3 : 5
\ = 5896
100
y
o
u
rs
m
SBI PO Exam 2013 241

a
h
b
From (91 – 95) 100. (b) Ratio between total No. of males and no. of non-grad

o
Total Cars = 700 females

o
b
= 160 : 20 = 8 : 1

.w
State Total Diesel Petrol 101. (a) Michael Clarke was the captain of Australian team in

o
rd
Cars Cars Cars the first three test matches of the series. Shane Watson

p
1 98 42 56 captained the side in the fourth and final test match

re
2 196 70 126 which was played in New Delhi.

s
s
3 224 140 84 102. (e) State Bank of India, the country's largest lender, received

.c
o
4 182 91 91 Rs. 3,004 crore from the government as part of state-

m
run banks. After the infusion, the government's
91. (b) Difference between No. of diesel cars in S-2 and No. of shareholding in the bank has increased to 62.31 per
pertrol cars in S-4 cent.
= 91 – 70 = 21 103. (d) An NRI can purchase up to a maximum of 5% of the
92. (a) Petrol cars in S-3 = 84 aggregate paid up capital of the company (equity as
Diesel cars in S-1 = 42 well as preference capital) or the aggregate paid up
value of each series of convertible debentures as the
84 - 42
%= ´100 = 100 % case may be. Total investment by NRIs cannot exceed
42 10% of the paid-up capital in an Indian company.
93. (d) 25% diesel cars in S-3 are AC 104. (a) March 20 is observed as World House Sparrow Day
\ 75% diesel cars in S-3 are non-AC (WHSD) to focus on raising awareness about the
decline of the common House Sparrow.
140 ´ 75
\ No. of non-AC cars = = 105 105. (a) Playing to Win: My Life... on and Off Court is the
100
autobiography of Saina Nehwal. She has the distinction
94. (e) Difference between total cars in S-3 and petrol cars in of being the first Indian to win a medal in Badminton at
S-2 the Olympics in 2012.
= 224 – 126 = 98 106. (a) In the sport of cricket, a bouncer (or bumper) is a type
95. (b) Average of petrol cars in all states together of delivery, usually bowled by a fast bowler which is
pitched short so that it bounces on the pitch well short
56 + 126 + 84 + 91 357
= = = 89.25 of the batsman and rears up to chest or head height.
4 4 107. (e) 'Women of Vision: Nin e Business Leaders in
96. (c) Blue balls = 7 Conversation' is a book by Alam Srinivas. It is a
None-ball out of two yellow collection of freewheeling chats with women such as
Yellow balls = 5 television and film producer Ekta Kapoor: ICICI Bank
\ Both balls are blue Ltd. managing director and CEO Chanda Kochchar,
Total balls = 12 Biocon Ltd's Kiran Mazumdar. Shaw and Shobhana
Bhartia, chairperson of HT Media Ltd.
7 6 7 108. (b) Muhammand Yunus is a Bangladeshi banker and
\ P (both blue balls) = ´ =
12 11 22 economist who developed the concepts of microcredit
97. (d) Total possible outcomes when A die is thrown twice and microfinance.
= 36 109. (a) Inversible Exports are services which are sold to other
Outcome for getting a sum 7 from both throwns nations by domestic firms, like banking, insurance, and
= 6{(1, 6), (2, 5), (3, 4), (4, 3), (5, 2), (6, 1)} tourism.
6 1 110. (d) The euro is the currency used by the Insitututions of
\ P(E) = =
36 6 the European Union and is the official currency of the
98–100. eurozone, which consists of 17 of the 27 member states
Total members = 240 of the European Union. It is the second largest reserve
currency as well as the second most traded currency
Males Females
in the world after the United States dollar.
Total 160 80
Graduates 24 60 111. (b) The Cash Reserve Ratio is the amount of funds that
Non-graduates 136 20 the banks are bound to keep with Reserve Bank of
India, with referece to the demand and time liabilities
98. (c) Difference between No. of non-graduates females and
(NDLT) to ensure the liquidity and solvency of the
no. of graduates males = 24 – 20 = 4.
Banks. This proportion is specified by RBI and could
99. (e) Sum of (graduates females and non-graduates males)
= 60 + 136 = 196 change from time to time.
y
o
u
rs
m
242 SBI PO Exam 2013

a
h
b
112. (e) Pre-Shipment Finance is issued by a financial institution loan is considered to be a non-performing asset.

o
when the seller wants the payment of the goods before 122. (d) According to the conditions stipulated by the RBI, the

o
b
shipment. Pre-shipment or post-shipment finance can bank shall open at least 25 per cent of its branches in

.w
be provided to exporters of IT and software services in unbanked rural centres (population up to 9,999 as per

o
rd
case of specific orders from abroad. the latest census). Besides, the bank shall comply with

p
the priority sector lending targets and subtargets as

re
113. (c) According to the Banking Ombudsman Scheme, 2006,
applicable to the existing domestic banks.

s
the Banking Ombudsmand is a senior official appointed

s
123. (d) The Union Budget 2013-14 announced the proposal to

.c
by the Reserve Bank of India to redress customer
launch Inflation Indexed Bonds or Inflation Indexed

o
complaints against deficiency in certain banking

m
National Security Certificates to protect savings from
services. inflation. These bonds will protect the principal and
114. (c) In terms of Section 25 of the Negotiable instruments the interest components from inflation risks.
Act, 1881, the expression "public holiday" include 124. (b) Planned expenditures are estimated after discussions
Sunday and any other day declared by the Central between each of the ministries concerned and the
Government by notification in the Official Gazette to Planning Commission. The expenditure incurred on the
be a public holiday. However, this power was delegated items relating to five year plans is termed as plan
by the Central Government to State Governments vide expenditure. Such expenditure is incurred by the Central
the Government of India, Ministry of Home Affairs in Government.
June 1957. 125. (c) The Small Industries Development Bank of India
(SIDBI) was set up in 1990 with the mission to empower
115. (b) With effect from October 25, 2011, saving bank deposit
the Micro, Small and Medium Enterprises (MSME)
interest rate stands deregulated. Accordingly, banks sector with a view to contributing to the process of
are free to determine their savings bank deposit interest economic growth, employment generation and
rate. balanced regional financial institution responsible for
116. (b) The Base Rate is the minimum interest rate of a Bank promotion, flancing and development of the section.
below which it cannot lend, except in cases allowed by 126. (a) CPU is the abbreviation for Central Processing Unit.
the Reserve Bank of India. The Base Rate system 127. (b) Joysticks are primarily used for computer gaming.
replaced the Benchmark Prime Lending Rate (BPLR These are input devices that are connected to a
system) with effect from July 1, 2010. computer to manage game controls such as moving
front, behind and sideways, shooting or other gaming
117. (e) In the Finance Act 2006, Section 80C of Income Tax
tasks. Hardcore games prefer joysticks over keyboard
Act 1961 was amended to include deposits placed with
controls.
a scheduled bank to qualify for deduction under 80C.
128. (d) Storage devices are data storage devices that are used
The amount of investment up to Rs. 1,00,000 under the in the computers to store data. CD, DVD, floppy disk
notified deposit scheme will be eligible for deduction and Hard disk are all types of storage devices. Printers
under Section 80C of Income Tax Act for individuals/ are output devices used to prepare permanent output
Hindu Undivided Families. on paper.
118. (e) The Repayment holiday is called moratorium. It is 129. (e) A handheld operating system (also known as mobile
actually a delay granted in the repayment of a debt or operating system) is an operating system that operates
in the fulfuillment of another legal obligation. on smart phones, tablet, PDA (personal digital
assistant) or other digital mobile devices.
119. (d) Indian IT Services and out sourcing giant Infosys got
130. (a) By double-clicking a folder in the main window or in
listed on the NYSE Euronext London and Paris markets
from February 20, 2013 becoming the first Indian the list along the left side to open the folder (also called
company to trade on the stock exchange. The left pane in windows explorer), the contents of the folder
Bangalore-based company is also listed on the Bombay gets displayed.
Stock Exchange (BSE) and National Stock Exchange of 131. (b) The CPU comprises of control unit (CU), memory and
India (NSE). arithmetic/logic unit (ALU). ALU performs arithmetic
120. (c) On 19 February 2013 BSE entered into Strategic and logic operations and the CU extracts instructions
Partnership with S&P Dow Jones indices and the from memory (called main memory/primary memory/
SENSEX was renamed as "S&P BSE Sensex." It will RAM) and decodes and executes them, calling on the
use the S&P brand for Sensex and other indices.
ALU when necessary.
121. (b) A non-performing asset (NPA) is defined as a credit
facility in respect of which the interest and/or 132. (c) A network server is a computer that provides various
installment of principal has remained 'past due' for a shared resources to workstations and other servers
specified period of time. Once the borrower has failed on a computer network. They are built with more
to make interest or principal payments for 90 days the powerful components than individual workstations.
y
o
u
rs
m
SBI PO Exam 2013 243

a
h
b
133. (c) A hyperlink (or link) is a word, group of words or image 145. (b) The purpose of post sales activities is to support the

o
o
that appears on a web page which can be clicked on to customer in use of the product or service and thereby

b
.w
jump to another document. A link is a part of anchor continuing customer satisfaction. It includes delivering

o
tag. support to agreed expectations, handling client

rd
134. (d) Each program displays information in one or more feedback, strengthening client relationships, offering

p
re
windows-rectangular areas on the screen that display additional benefits, etc.

s
s
information from a running program. Each program's 146. (e) USP (unique selling proposition) denotes high/unique

.c
selling features of a product i.e., USP helps in creating

o
window(s) can be minimized, maximised or restored.

m
135. (c) Disk fragmentation is a utility program that locates and a unique place in the mind's of customers relative of
eliminates unnecessary fragments. As operations & competition.
processes are loaded & deleted from computer memory, 147. (e) The competitive position of company can be improved
free memory space is broken in little pieces. by being customer-centric not product-centric i.e., not
136. (c) A power surge is a period of abnormally high voltage viewing product in isolation, but as a part of a full-long
in an electrical system, which pushes too much current lasting relationship with a customer. This means
(or electricity) through electrical devices. A brief power offering those products and services that fulfill their
surge that lasts less than one power cycle is called a needs.
voltage spike. 148. (d) A value added service is something extra that is
137. (e) A word processor is a software application that is used provided to a customer at no additional charge. For
to produce, edit and format mainly text documents such example, a sample product is given for free when
as letters, memos, reports, labels, etc. another related product is bought whether at regular
138. (e) It is the function of the input devices to take the input or discounted price.
from the user (which is in human readable form) and 149. (a) For a bank, the foremost target market for selling their
convert it into machine readable form before it is debit cards is all their existing account holders.
presented to the computer. 150. (e) Brands loyalty built through goodwill means providing
consistent good serivces to their customers. For
139. (d) Effective selling skill depends on market information
i.e., knowledge of related markets. It involves details example, providing effective post sales maintenance
about customer profile and product mix. & support services: listening & responding properly
when customer provides feedback; etc.
140. (c) A DSA markets and sells goods to customers without
151. (d) In paragraph 2 of the passage, it is mentioned that
the benefit of a retail, online or catalog store. No
bank branch timings rarely coincide with off work hours
educational requirements is necessary for DSA
of labour class.
position. But DSA should have good oral and written
152. (a) in paragraph 1, the last 5 lines explain this point. There
communication skills.
is no mention of the other points in the passage. Hence,
141. (b) Lead generation can be achieved by browsing
1 is the correct answer.
telephone directory, yellow pages, internet sites, list
153. (c) paragraph 1 , line 3rd to 6th explain this aspect.
of existing customers, etc.
154. (e) all the points are covered in paragraph 1.
142. (a) Five essential qualities of a successful marketing person
155. (a) the role of government is nowhere mentioned, the other
are empathy, focus, responsibility, optimism and ego-
2 points are covered, hence (a) is the correct answer
drive (competitiveness). Empathy is the ability to
156. (e) the passage explains only points A and C, hence (e) is
identify with the customers, to feel what they are feeling
the correct answer.
and make customers feel respected. A sales person
157. ‘(d) multitude means ‘a large number of people or things,
showing empathy can gain trust and establish rapport
hence plenty (d) is correct
with customers.
158. (c) ubiquitous means-‘present, appearing, or found
143. (c) Innovation marketing involves applying the principle
everywhere’, hence (c) omnipresent is the correct
of creativity to find new ways of marketing effectively.
answer
144. (a) Market segmentation helps to determine targets groups
or indentifying prospects. It can be resorted to by way 159. (e) dormant means inactive, hence opposite would be
of- segmenting by customer's age, segmenting by active (e)
customer's income, segmenting geographically, 160. (c) delayed means late, so opposite would be to rush or
segmenting by customer's tastes & preferences, etc. hurry, hence expedited (c) is correct
y
o
u
rs
m
244 SBI PO Exam 2013

a
h
b
161. (c) as per paragraph 1 A and C are correct, hence correct 181. (a) sentence F is first ( correct sequence- F,C,A,E,D,B)

o
o
answer is (c) 182. (a) A is the third sentence

b
.w
162. (c) paragraph1 says (in line 13-14) that it is a myth... hence 183. (d) sentence B will be the correct answer.

o
answer is (c) 184. (c) E

rd
163. (c) paragraph 1 – last few lines

p
185. (b) D

re
164. (a) paragraph 2 , the 4th and 5th line explains this point.
186. (b) conducive ; the other words do not fit in correctly

s
s
165. (b)

.c
187. (d) question
166. (a) the passage does not talk of point B, hence A and C is

o
m
188. (a) cannot fits correctly
correct (a)
189. (c) need
167. (a) means indicate, hence (a) suspected is correct
168. (e) outweigh means to be more significant- hence it means 190. (e) growth
surpass 191. (c) favour
169. (d) embrace means to accept, hence opposite would be 192. (b) enjoys
reject (d) 193. (a) aid is used correctly meaning facilitate
170. (a) compliance 194. (e) increased
171. (d) it should be ‘either in India or in the country of his 195. (a) both
origin 196. (c) emerging/ nascent mean almost the same (just coming
172. (c) and law abiding .... up)
173. (d) .... has not been achieved instead of have 197. (b) eccentric / abnormal mean almost the same (strange
174. (e) the sentence is correct ( no error) behaviour)
175. (d) in India’s ..... in place of to India’s ... 198. (e) abundance and plethora mean the same ( large quantity
176. (b) of something)
177. (e) others do not fit
199. (a) purposefully / inadvertently are opposites , former
178. (c)
means deliberately and latter by mistake
179. (d)
200. (e) germane / irrelevant are opposites – germane means
180. (b)
relevant
y
o
u
rs
m
a
h
b
o
IBPS SPECIALIST (I.T.) OFFICER

o
b
.w
EXAM 2013

o
rd
p
Based on Memory

re
s
s
.c
o
m
REASONING ABILITY 4. Statements
All books are journals.
DIRECTIONS (Qs. 1-5) : In each of the following questions two/ All diaries are journals.
three statements are given followed by two Conclusions
Conclusions:
numbered I and II. You have to take the given statements to be
true even if they seem to be at variance from commonly known I. All journals are books.
facts. Read both the conclusions and then decide which of the II. Some diaries being books is a possibility.
given conclusions logically and definitely follows from the given (a) Either Conclusion I or Conclusion II follows
statements disregarding commonly known facts. (b) Only Conclusion I follows
1. Statements (c) Both Conclusion I and Conclusion II follows
All beans are pulses (d) Neither Conclusion I nor Conclusion II follows.
All pulses are crops. (e) Only Conclusion II follows
No crop is seed. 5. Statements
Conclusions: All beans are pulses.
I. All crops are pulses. All pulses are crops.
II. All beans are crops. No crop is seed.
(a) Only Conclusion II follows. Conclusions:
(b) Neither Conclusion I nor Conclusion II follows. I. No seed is a bean.
(c) Either Conclusion I or Conclusion II follows II. No bean is a pulse.
(d) Only Conclusion I follows (a) Both Conclusion I and Conclusion II follow.
(e) Both Conclusion I and Conclusion II follow. (b) Neither Conclusion I nor Conclusion II follows.
(Qs. 2-3) Statements
(c) Only Conclusion I follows.
No fruit is vegetable
(d) Either Conclusion I or Conclusion II follows
All potatoes are vegetables.
(e) Only Conclusion II follows
Some fruits are apples.
2. Conclusions: DIRECTIONS (Qs. 6) : Read the given statements carefully and
I. Some apples are potatoes answer the question given below:
II. Some potatoes being fruits is a possibility. 6. The project of the road construction (work) has crossed its
(a) Both Conclusion I and Conclusion II follow. first deadline as far as pre-monsoon road works are
(b) Only Conclusion II follows concerned. In the major city the road works are given great
(c) Either Conclusion I or Conclusion II follows emphasis and these are the places where road work has
(d) Only Conclusion I follows been completed.
(e) Neither Conclusion I nor Conclusion II follows. Which of the following can be concluded from the
3. Conclusions: statements given above?
I. No fruit is a potato. (a) The start the work of the road one has to go through a
II. Atleast some apples are fruits. lot of tiresome paperwork before starting the repair
(a) Both Conclusion I and Conclusion II follows work which delays the whole work of the road
(b) Either Conclusion I or Conclusion II follows (b) It takes several hours while travelling via these roads.
(c) Only Conclusion II follows (c) The work of the road is going on
(d) Neither Conclusion I nor Conclusion II follows. (d) They will start the road works well in advance
(e) Only Conclusion I follows (e) None of these
y
o
u
rs
m
246 IBPS Specialist (I.T.) Officer Exam 2013

a
h
10. Which of the following is possibly the height of Q?

b
DIRECTIONS (Qs. 7-8) : Read the following information carefully

o
(a) 148 cm (b) 156 cm

o
and answer the questions which follow:

b
(c) 152 cm (d) 150 cm

.w
Supermarkets are growing at a fast pace than Kirana Stores. (e) 158 cm

o
Kirana Stores are such places where customer go to purchase 11. How many persons are shorter than Q?

rd
their necessities. In place of five Kirana stores one or two (a) Two (b) Three

p
re
supermarkets are being established. It has been found that (c) Four (d) Cannot be determined

s
customers’ requirements are looked after by trained staff. It has (e) One

s
.c
been found that food products are low life products which are

o
manufactured from local manufacturers. These products are DIRECTIONS (Qs. 12-13) : Read the following statements

m
typically purchased by the customer on the assurance. The carefully and answer the questions given below:
markets is appealing to supermarkets and retail outlet owners are Mobile technology has played great role in growth and
setting up their business in other areas where there are less Kirana development of society. Earlier cellphone was used as a medium
stores. of conversation only. Now mobile phones also support a wide
7. Which of the following can be a good argument in favour of variety of other services, such as, texting, email, internet access
shopping, from Kirana stores instead of supermarkets? etc. The price of mobile phones is also decreasing and people are
(a) People prefer supermarkets because they offer a larger being encouraged to buy a mobile phone set at a cheaper rate.
range of products. i.e., products other than FMCG and The mobile technology and smartphones have the capabilities of
they can buy everything under one roof. handling video calls, sharing large files. Mobile technology had
(b) People end-up buying other irrelevant things along made it more efficient to conduct business. Video calls and taking
with those on their shopping lists in Supermarkets and photographs have become possible as mobile phone has in built
then they have to stand in long queues at the billing camera. Therefore, there is no need to carry around a camera
counters. everywhere you go,.
(c) Most Kirana stores are closed atleast one day in a 12. Which of the following cannot be inferred from the given
week whereas supermarkets are open 365 days a year. information? (An inference is something that is not directly
(d) Kirana stores do not accept debit and credit cards. stated but can be inferred from the given information)
(e) Very few Kirana stores sell products at a bargained (a) One can share photos and videos via mobile phones
price. provided that the other person has a similar device.
8. Which of the following can be inferred from the given (b) Many features are being added to mobile phones now-
information? (An interference is something that is not a-days
directly stated but can inferred from the given information). (c) The other features of mobile phones are used as useful
(a) Most supermarkets and retail outlet owners choose to as the built in camera
set up businesses in areas that have very few Kirana (d) Mobile phones can be used for purposes other than
stores. making calls
(b) People tend to trust retail outlets and supermarkets (e) Technological advances are taking place in fields other
when it comes to buying high value products as than cellphones as well.
opposed to buying them from local shops. 13. Which of the following can be concluded from the given
(c) If there are two or more Kirana stores in a typical information?
residential area the competition among them is very (a) Buying a phone with a camera is more convenient than
high. buying two different devices.
(d) Kirana stores owners are buying to acquire franchisee (b) Mobile phones that are basic last longer than the ones
of supermarkets as the local shops have lost their charm with added features
and business. (c) Not many people are interested in clicking pictures
(e) Kirana stores do not sell the products which consumers with mobile phones
on their regular shopping lists, so they have to depend (d) It is possible to share pictures with someone having a
on supermarkets. similar camera
(e) No one will buy camera now onwards.
DIRECTIONS (Qs. 9-11) : Read the following information
carefully and answer the questions given below. DIRECTIONS (Qs. 14-15) : Study the following information to
answer the given questions.
Among five persons - P, Q, R, S and T – each has different height.
Only two persons are shorter than S. T is shorter than S but taller ‘TRAVEL’ is related to ‘UDKUSB’ and ‘CORNER’ is related to
than R. The one who is the second tallest among them is of 158 ‘MDQDPS’.
centimetre. 14. ‘SURVEY’ is related to _______.
9. Which of the following statements is definitely true with (a) UDXSTV (b) UXDTSV
respect to the given information? (c) TVSUDX (d) UDXTVS
(a) R is definitely 150 cm. (e) UDXVTS
(b) None of the given statements is true 15. ‘GROUPS’ is related to ______.
(c) T is shorter than S (a) TORHSP (b) TOHRSP
(d) Q is shorter than S (c) TORPHS (d) ROTHSP
(e) P is possibly 153 cm (e) ROTPHS
y
o
u
rs
m
IBPS Specialist (I.T.) Officer Exam 2013 247

a
h
b
DIRECTIONS (Qs. 16-20) : Study the following information to 23. Effect: As a step to regulate private hospitals, the static

o
health department is framing rules to ensure all such hospitals

o
answer the given questions.

b
are registered with it.

.w
In a certain code ‘8 2 9’ means ‘how art thou, ‘9 5 8’ means ‘thou Which of the following can be a possible cause of the above

o
art good’ and ‘1 5 8 7 3’ means ‘thy good’ and ‘thou bad’. statement?

rd
16. What may be the possible code for ‘thy’? (a) The department realised the private hospitals charge

p
re
(a) 1 or 7 (b) 7 much less for treatment as compared to government

s
(c) 3 (d) 5 hospitals.

s
.c
(e) 1 or 7 or 3 (b) Government run hospitals do not maintain the same

o
standards as private hospitals.

m
17. What is the code for ‘thou’?
(a) 9 (b) 8 (c) The department realised that several hospitals were
(c) 2 (d) 5 rejecting cases stating lack of infrastructure.
(d) Apart from the number of doctors, nurses and beds,
(e) None of these
the kind of procedure a hospital can carry out based
18. What is the code for ‘how’?
on its infrastructure will also be registered and detailed.
(a) 5 (b) 8 (e) Private hospitals nor registering with the department
(c) 2 (d) 9 shall be forced to do so and will have to pay hefty
(e) None of these penalties.
19. Which of the following may possibly be the code for ‘thou 24. Statement: The constable has been recommended for a
no good’? suitable reward by his superior in recognition of his sincere
(a) 5 0 8 (b) 7 8 0 duty and busting of several gangs of criminals actively
(c) 5 0 7 (d) 7 0 8 involved in the loot and incidents of pick-pocketing.
(e) None of these Which of the following can be a possible assumption of the
20. What is the code for ‘how good thou art’? above statement?
(a) 7 5 8 9 (b) 8 2 9 5 (a) The superior is certain that the recommendation would
(c) 7 1 8 3 (d) 8 7 9 5 be denied.
(e) None of these (b) The number of criminals apprehended by this particular
constable was exceptionally high.
21. Effect: The temple at the religious site wears a deserted look
(c) The constable desires to be monetarily compensated
with the number of devotees trickling down.
for his efforts.
Which of the following can be a possible causes of the (d) The superior wants to set an example for his other
above effect? juniors by recommending the reward.
(a) A structural engineer had visited the temple a month (e) Rewards recognising the sincerity and
back and had declared the structure unsafe. accomplishments of policemen are given.
(b) The temple is facing drastic depletion of its funds which 25. Statement: The college has finally received accreditation
had accumulated over the years due to offerings made and has gained the status of a deemed university.
by devotees. Which of the following can be a possible effect of the above
(c) The local corporation decided to donate a huge amount statement?
of money to the temple for its renovation. (a) The principal of the college will now have to be retired
(d) The village housing the religious site has qualified government official.
priests to perform religious ceremonies. (b) Number of students seeking admission to this college
(e) A famous actor recently visited the temple and paid in the next academic year would drop significantly.
his respects to the deity. (c) The college will charge lesser fees from all the students
22. Statements: The Income Tax authorities carried out raids at despite not getting a grant from the government
three different business houses in the city last week. (d) The college will reduce the number of courses that it
Which of the following can be a possible effect of the above runs by a significant margin.
statement? (e) The reputation of the college amongst the student
(a) The three business houses are regular defaulters in population in general has improved.
payment of their income tax. DIRECTIONS (Qs. 26-27) : In these questions, relationship
(b) The Income Tax department had received a tip off about between different elements is shown in the statements. These
the illegal activities going on in the three business statements are followed by two conclusions.
houses. Mark answer If
(c) The Government decided to look into the matter and (a) Only conclusion I follows.
has appointed an inquiry committee (b) Only conclusion II follows.
(d) Other business houses took immediate action to clear (c) Either conclusion I or II
off all their income tax dues in order to avoid a raid on follows.
their establishments. (d) Neither conclusion I nor II
(e) The authorities intend to conduct raids in several other follows.
business houses in the vicinity. (e) Both conclusion I and II
follows.
y
o
u
rs
m
248 IBPS Specialist (I.T.) Officer Exam 2013

a
h
26. Statement: E < F £ G = H > S

b
Mark answer (c) If the data is inadequate i.e. from the facts given,

o
Conclusions: you cannot say whether the inference is likely to be true or false.

o
b
I. G > S Mark answer (d) If the inference is “probably false” though not

.w
II. F £ H “definitely false” in the light of the fact given.

o
27. Statement: P £ Q < W = L

rd
Mark answer (e) If the inference is “definitely false” i.e., it can
Conclusions:

p
not possibly be drawn from the facts given or it contradicts the

re
I. L > P given facts.

s
II. Q £ L

s
(Note: Each of the five questions has only one distinct answer

.c
i.e., no two questions can have the same answer. If you get the

o
DIRECTIONS (Qs. 28-30) : Study the following information to

m
answer the given questions: same answer for more than one question, consider both again
and decide which one of the two would more definitely be that
A word and number arrangement machine when given an answer and in the same way review the others also).
input line of words and numbers rearranges them following a Cardiovascular disease is so prevalent that virtually all
particular rule. The following is an illustration of input and businesses are likely to have employees who suffer from, or may
rearrangement. develop, this condition. Research shows that between 50-80
(All numbers in these questions are two digit numbers) percent of all people who suffer a heart attack are able to return to
Input : 16 today 32 waiting 21 are 11 people 46 bus 66 long work. However, this may not be possible if they have previously
Step I : 16 today 32 waiting 21 11 people 46 bus 66 long are been involved in heavy physical work. In such cases, it may be
Step II : 16 today 32 waiting 21 people 46 bus 66 long 11 are possible to move the employee to lighter duties, with appropriate
Step III : 16 today 32 waiting 21 people 46 66 long bus 11 are retraining where necessary. Similarly, high-pressure, stressful
Step IV : today 32 waiting 21 people 46 66 long 16 bus 11 are work, even where it does not involve physical activity, should
Step V : today 32 waiting people 46 66 21 long 16 bus 11 are also be avoided. Human Resource managers should be aware of
Step VI : today 32 waiting 46 66 people 21 long 16 bus 11 are the implications of job roles for employees with a cardiac
Step VII : today waiting 46 66 32 people 21 long 16 bus 11 are condition.
Step VIII : waiting 46 66 today 32 people 21 long 16 bus 11 are 31. Employees who suffer from cardiovascular disease are
Step IX : waiting 66, 46 today 32 people 21 long 16 bus 11 are
mostly unable to return to work.
Step X : 66 waiting 46 today 32 people 21 long 16 bus 11 are
32. Employees suffering from Cardiovascular diseases are
Step X is the last Step of the arrangement of the above input as
unable to handle stressful situations.
the intended arrangement is obtained.
33. Employees above the age of 50 are found to suffer from
Now, answer the questions based on the following input.
cardiovascular disease.
Input: 23 you 13 wake 81 me 43 before 72 go 34 up
34. Physical and stressful work definitely leads to a heart attack.
28. Which of these words / numbers would be fourth (from left
35. Heart disease can affect employees in any type of business.
side) in step IV for the input
(a) me (b) 48 DIRECTIONS (Qs. 36-40) : Study the following information to
(c) 81 (d) wake answer the given questions:
(e) None of these
29. The following stands for which step of the reaarrangement? J, P, Q, R, S, T, U and V are four married couples sitting in a
you wake 81 43 72 34 up me 23 go 13 before circle facing the centre. The profession of the males within the
(a) Step IX (b) Step IV group are lecturer, lawyer, doctor and scientist. Among the males,
(c) Step VI (d) Step V only R (the lawyer) and V (the scientist) are sitting together. Each
(e) None of these man is seated besides his wife. U, the wife of the lecturer is seated
30. Which of the following would be Step II for the above input? second to the right of V. T is seated between U and V. P is the wife
(a) 23 you wake 81 me 43 72 34 up go 13 before of the doctor. Q is not the doctor. S is a male.
(b) 23 you 13 wake 81 me 43 72 go 34 up before 36. Which of the following is P’s position with respect to S?
(c) 23 you wake 81 me 43 72 go 34 up before 13 (a) Second to the right (b) Second to the left
(d) 23 you wake 81 me 43 72 go 34 up 13 before (c) Immediate right (d) Immediate left
(e) None of these (e) Third to the left
37. Which of the following is J’s position with respect to T?
DIRECTIONS (Qs. 31-35) : Below is given a passage followed
(a) Third to the left (b) Fourth to the right
by several possible interferences which can be drawn from the
(c) Third to the right (d) Opposite T
facts stated in the passage. You have to examine each inference
(e) Second to the right
separately in the context of the passage and decide upon its
38. Which of the following is not true regarding the couples?
degree of truth or falsity.
(a) P is the wife of S
Mark answer (a) If the inference is “definitely true” i.e. it properly (b) T is the wife of Q
follows from the statement of facts given. (c) R is the husband of J
Mark answer (b) If the inference is “probably true” though not (d) J and S are seated adjacent to each other
“definitely true” in the light of the facts given. (e) All are true
y
o
u
rs
m
IBPS Specialist (I.T.) Officer Exam 2013 249

a
h
b
39. The wives of which two husbands are immediate 46. The number of cases concerning a non-contagious chemical

o
neighbours? poisoning is on the rise among the local villagers.

o
b
(a) UT (b) SR Which of the following could possibly be a Cause of the

.w
(c) VQ (d) RV statement given above?

o
(e) None of these (a) The government has ordered an immediate enquiry into

rd
the matter.

p
40. Four of the following are alike in a certain way based on

re
their seating position in the above arrangement and so form (b) Unless timely treatment is provided to the patients,

s
the poisoning can prove to be fatal.

s
a group. Which is the one that does not belong to the group?

.c
(a) RSJ (b) TRV (c) Many factories surrounding the village do not dispose

o
off their wastes appropriately,.

m
(c) UTV (d) SQP
(e) UPQ (d) The only hospital in the area is not equipped to treat
chemical poisoning related ailments
DIRECTIONS (Qs. 41-44) : Study the following information to (e) A study done a couple of years ago had reported that
answer the given questions: a large number of farmers had stopped using chemical
fertilizers owing to their ill-effects on health.
Seven friends T, U, V, W, X, Y and Z are sitting in a straight 47. Small grocery shop owners have gone on a day’s strike in
line facing north. W sits fifth to the right to T. W does not sit at order to protest against the entry of a foreign retail store
any of extreme ends. Two people sit between Z and X. Y sits third chain in the city.
to the left of U. Y sits exactly in the middle. Z is not an immediate Which of the following statements can be an Effect of the
neighbour of Y. statement given above?
41. What is the Z’s position with respect to W? (a) The small grocery shop owners perceive the big retail
(a) Second to the left (b) Third to the right giants as sharks, trying to dominate the available market
(c) Fourth to the left (d) Third to the left space.
(e) Fourth to the right (b) Foregin investment in retail has shown a tremendous
42. Who is second to the right of T? growth in India since a couple of years.
(a) Y (b) X (c) There have been several countries in which retail chain
(c) U (d) V giants have totally wiped out small scale grocery shop.
(e) None of these (d) The government agreed to take all political steps
43. Four of the following five are alike in a certain way based on required to safeguard the interests of small scale
their seating positions in the above line and so form a group, retailers.
which is the one that does not belong to the group? (e) A recent survey indicated that a large number of people
(a) UW (b) XV prefer to buy their daily daily grocery stores rather
than big retail chain stores.
(c) ZT (d) YV
(e) WX DIRECTIONS (Qs. 48-50) : Read the following statements
44. If all the seven friends are made to sit alphabetically from carefully and answer the questions which follow.
right to left. Positions of how many will remain unchanged?
(a) None (b) One 48. The ministry of sports has been advised by a committee to
(c) Two (d) Three take the highest award in the field of sports back from two
(e) Four players who were allegedly-involved in match fixing.
Which of the following statements would weaken the
DIRECTIONS (Qs. 45-47) : Read the given statements carefully argument put forward by the committee to the sports
and answer the questions which follow. ministry?
45. After the government announced a compensatory policy (a) A good conduct in the past and a lack of evidence
for farmers in return for the acquisition of their land, the against the players make the case against them very
farmers demanded a written proof of the policy. weak.
Which of the following could possibly be a Cause for (b) The ministry of sports has never declined the
demanding the written proof? recommendations made by the committee earlier.
(a) The farmers were unable to contest the inadequate (c) Taking the award back from the players would set a
good example to other players for avoiding such
compensation for their land in earlier such policies.
actions in the future.
(b) The compensation promised in the policy was grossly
(d) There have been past cases where the award had to be
inadequate.
taken back from the players owing to some misconduct
(c) The farmers were unwilling to surrender their lands to
later on.
the government. (e) The committee is constituted of some of the most
(d) The farmers demand a certain percentage of share out respected from the fields of sports and politics.
of the revenue generated by the government off their 49. Many organizations have been resorting to recruitment
lands. based upon performance at graduate post-graduate level
(e) The only option to enforce the government officials to exams rather than conducting exams for the same purpose.
frame a compensatory policy for the farmers was to Which of the following statements would strengthen the
demand a proof. argument given in the above statement?
y
o
u
rs
m
250 IBPS Specialist (I.T.) Officer Exam 2013

a
h
b
(a) A recent study shows no link of past performance with DIRECTIONS (Qs. 53-58) : In the following questions

o
the performance in recruitment exams.

o
information about number of candidates interviewed by five

b
.w
(b) The graduate/post-graduate exams are considered to commercial firms on different working days has been provided.

o
be severely deficient in training in job related You are required to read the table carefully and answer the

rd
environment questions given below:

p
re
(c) Organisations which had undertaken recruitment on
Number of candidates interviewed by five firms on

s
s
the basis of graduate / post-graduate exams report a

.c
different working days
significant drop in the quality of the recruited

o
m
employees.
(d) Such policies would add to unemployment amongst Working Firms
students having below average performance in Day A B C D E
graduation or post-graduation. Monday 17 18 23 25 18
Tuesday 21 19 14 28 25
(e) Such policies could save time, money and resources Wednesday 23 22 23 12 18
of the organisation which are wasted in the conduct of Thursday 24 14 12 23 18
recruitment examinations. Friday 10 10 16 15 22
50. According to a recent government directive, all bank Saturday 17 26 20 20 24
branches in rural areas should be computerized.
Which of the following statements would weaken the 53. What is the respective ratio between the number of
government’s argument? candidates interviewed by firm D on Friday and Saturday
(a) Computerisation of bank branches in urban areas has together and that of candidates interviewed by firm B on
helped in making their performance more efficient and the same days?
fast. (a) 35 : 38 (b) 39 : 40
(b) Lack of skilled and qualified manpower has been (c) 43 : 44 (d) 45 : 46
suitably substituted by computers in banks. (e) 35 : 36
(c) Non-computerised bank branches in the rural areas 54. The number of candidates interviewed by firm C on
have been proved to be as efficient as their Wednesday is what percent of total number of candidates
computerized counterparts. interviewing by all the firms on the same day?
(a) 24 (b) 23
(d) The government has introduced a special test for
computer knowledge in all recruitment exams for banks. (c) 38 (d) 29
(e) None of these
(e) Unemployment in the rural areas could be controlled
55. In which firm the number of candidates interviewed decreased
by training more and more professionals in computers.
consistently from Monday to Saturday?
(a) B (b) None
QUANTITATIVE APTITUDE (c) D (d) C
(e) A
51. A 320 metre long train crosses a pole in 50 seconds. What is
56. What is the number of candidates interviewed by all the
the speed of train?
firms on Monday?
(a) 4.6 m/sec (b) 7.2 m/sec (a) 101 (b) 102
(c) 5.4 m/sec (d) 6.4 m/sec (c) 114 (d) 98
(e) Cannot be determined (e) 96
52. In a school some sweets were to be distributed among 420 57. By what percent the number of candidates interviewed by
children on the occasion of Teacher’s day. But 140 children firm E on Tuesday increased with respect to that of
remained absent on that particular day and hence each child interviewed on the preceding day?
got one sweat extra. How many sweats each child would (a) 45 (b) 26
have got originally? (c) 61 (d) 56
(a) Cannot be determined (e) 39
(b) 2 58. In how many ways the letters of the word VISITING can be
rearranged?
(c) 5
(a) 6720 (b) 5720
(d) 4 (c) 720 (d) 7620
(e) 1 (e) None of these
y
o
u
rs
m
IBPS Specialist (I.T.) Officer Exam 2013 251

a
h
b
DIRECTIONS (Qs. 59-64) : In the following pie-chart percent of DIRECTIONS (Qs. 65-69) : What value should come in place of

o
o
students enrolled in different cultural activities of a school has the questions mark (?) in the following questions ?

b
.w
been shown. You are required to study the pie-chart carefully 65. 18.5 × 21.4 × ? = 6255.22

o
and answer the questions given below:

rd
(a) 15.8 (b) 14.6

p
Number of students = 1800 (c) 17.4 (d) 17.2

re
(e) 16.4

s
s
66. 1.5 × 78 ¸ 0.5 = ?

.c
o
(a) 238 (b) 234

m
Karate (c) 243 (d) 216
10% % (e) 261
et7 67. 302.46 + 395.72 – 123.47 = ?
k
ic
Dance Cr (a) 576.77 (b) 547.17
38% (c) 547.77 (d) 574.71
(e) 577.71
Painting
32% 68. 3
4096 ¸ 3 64 = 3 ?
5%
ic 8%
ts

(a) 16 (b) 8
or
Mus
Sp

(c) 64 (d)
4
69. ? % of 800 = 293 – 22% of 750
(a) 14 (b) 18
59. What is the number of students who opted for dancing? (c) 12 (d) 16
(a) 648 (b) 684 (e) 20
(c) 864 (d) 664
DIRECTIONS (Qs. 70-74) : In the following table, the number of
(e) None of these
employees working in five companies and the corresponding
60. What is the respective ratio between the number of students
ratio of male and female employees have been given. You are
who opted for painting and that who opted for Karate? required to study the table carefully and answer the questions.
(a) 3 : 7 (b) 5 : 16
(c) 16 : 5 (d) 5 : 7
Company Number of Employees
(e) None of these
L 400
61. By what percent is the number of candidates who opted for
music more than those who opted for sports? M 600
(a) 30 (b) 55 N 800
(c) 45 (d) 60 O 1000
(e) None of these P 1200
62. How many students have opted for cricket and painting Company Male : Female
together? L 12 : 8
(a) 702 (b) 602 M 15 : 5
(c) 660 (c) 645 N 4:1
(e) None of these O 13 : 12
63. By what percent is the number of students who opted for P 7:5
cricket and dancing together more than those who opted
for painting and music together? 70. What is the respective ratio between the number of females
in company P and number of females in company L?
(a) 14.5 (b) 12.5
(a) 8 : 25 (b) 25 : 8
(c) 16.5 (d) 21.5
(c) 3 : 16 (d) 16 : 3
(e) None of these
(e) None of these
64. What is the respective ratio between the number of students 71. The number of female employees working in company O is
who opted for dancing and karate to those who opted for what percent of total employees working in that company?
painting (a) 24 (b) 12
(a) 2 : 3 (b) 1 : 3 (c) 48 (d) 13
(c) 3 : 1 (d) 3 : 2 (e) None of these
(e) None of these
y
o
u
rs
m
252 IBPS Specialist (I.T.) Officer Exam 2013

a
h
b
72. What is the average number of employees in all companies 80. What is the average number (in thousand) of units

o
together? manufactured by all the companies together?

o
b
(a) 800 (b) 775 (a) 41 (b) 40

.w
(c) 760 (d) 600 (c) 43 (d) 44

o
rd
(e) None of these (e) 46

p
73. The number of female employees in company M is 81. What is the respective ratio between the number of units

re
(a) 450 (b) 150 sold by company S and that sold by company Q?

s
s
(a) 7 : 6 (b) 1 : 2

.c
(c) 250 (d) 350

o
(e) None of these (c) 1 : 3 (d) 3 : 1

m
74. The total number of male employees working in companies (e) None of these
N and P together is 82. Which company sold highest percentage of units as
compared to manufactured by it?
(a) 1240 (b) 1360
(a) Q (b) R
(c) 1340 (d) 1260
(c) P (d) T
(e) None of these (e) S
DIRECTIONS (Qs. 75-79) : What approximate value should come 83. What is the average number of units (in thousand) sold by
all the companies together?
in the following questions at the questions places.
(a) 25.7 (b) 30
(You are not required to calculate the exact value). (c) 27.7 (d) 28.8
75. (13.001)3 = ? (e) None of these
(a) 1900 (b) 2200 84. What is the respective ratio between the units manufactured
(c) 2000 (d) 1800 by company P and company S?
(e) 2100 (a) 7 : 8 (b) 8 : 7
76. 55.003 × 54.998 + 5.001 = ? (c) 3 : 5 (d) 5 : 3
(e) None of these
(a) 3500 (b) 3630
85. From a well-shuffled pack of 52 playing cards, one card is
(c) 2540 (d) 3030
drawn at random. What is the probability that the card drawn
(e) 2750 will be a black king?
77. 50.001% of 99.99 ¸ 49.999 = ?
(a) 1 (b) 0.1 1 7
(a) (b)
(c) 0.01 (d) 0.02 26 13
(e) None of these 3 9
78. 999.0001 + 899.999 – 349.88 = ? (c) (d)
13 13
(a) 1549 (b) 1560
(c) 1449 (d) 1460 1
(e) None of these (e)
13
79. (2.0001)3 × (1.999)–2 ¸ (3.999)–4 = ? 86. A horse is tethered to a peg with a 14 metre long rope at the
(a) 32 (b) 16 corner of a 40 metre long and 24 metre wide rectangular
(c) 64 (d) 256 grass-field. What area of the field will the horse graze?
(e) 512 (a) 154 m2 (b) 308 m2
(c) 240 m 2 (d) 480 m2
DIRECTIONS (Qs. 80-84) : In the following table the number of (e) None of these
units manufactured and sold by five companies- P, Q, R, S and T
2
has been given. Study the given data carefully and answer the 87. Farah was married 8 years ago. Today her age is 1 times
questions. 7
to that at the time of marriage. At present her daughter’s

Number of units Number of 1


age is th of her age. What was her daughter’s age 3 years
Company manufactured units sold (in 6
(in thousand) thousand) ago?
(a) 6 years (b) 7 years
P 35 20 (c) 3 years (d) Cannot be determined
Q 45 30 (e) None of these
R 35 25 88. Manish brought 25 kg of rice at Rs. 32 per kg and 15 kg of
rice at Rs. 36 per kg. what profit did he get when he mixed
S 40 35
the two varieties together and sold it at Rs. 40.20 per kg?
T 50 40 (a) 25% (b) 40%
(c) 30% (d) 20%
(e) None of these
y
o
u
rs
m
IBPS Specialist (I.T.) Officer Exam 2013 253

a
h
ENGLISH LANGUAGE

b
DIRECTIONS (Qs. 89-93) : In each of the following questions

o
o
two equations are given. Solve these equations and give answer:

b
.w
(a) if x ³ y, i.e., x is greater than or equal to y. DIRECTIONS (Qs. 101-110) : Each question below has two

o
blanks, each blank indicating that something has been omitted.

rd
(b) if x > y, i.e., x is greater than y.

p
(c) if x £ y, i.e., x is less than or equal to y. Choose the set of words for each blank that best fits the meaning

re
(d) if x < y, i.e., x is less than y. of the sentence as a whole.

s
s
(e) x = y or no relation can be established between x and y

.c
101. A plethora of cultural talent ______ the spectators busy at

o
89. I. x2 + 5x + 6 = 0

m
a talent hunt programme where students from various
II. y2 + 7y + 12 = 0
colleges get together to ______ their mettle in various
90. I. x2 + 20 = 9x
contests.
II. y2 + 42 = 13y
(a) tried, show (b) kept, prove
91. I. 2x + 3y = 14
II. 4x + 2y = 16 (c) caught, puzzle (d) held, learn
(e) helped, mention
92. I. x = 625
102. The _____ of Chinese plastic thread should be banned as it
II. x = 676 _____ thousands of birds every year during the kite flying
93. I. x2 + 4x + 4 = 0 season.
II. y2 – 8y + 16 = 0 (a) sale, cripples (b) sell, kills
(c) sale, disturbs (d) sell, disables
DIRECTIONS (Qs. 94-98) : What will come at the place of
question mark (?) in the following number series? (e) sale, saves
103. Harish _____ with me some basic techniques that one can
94. 3 1731 ? 3243 3459 3523 _____ at home without even owning a drumset.
(a) 2371 (b) 2731
(a) made, shred (b) imparted, try
(c) 3731 (d) 3127
(c) learned, balance (d) fits, rechearses
(e) None of these
95. 10 18 34 ? 130 258 (e) shared, practise
(a) 32 (b) 60 104. In the present case, the facts clearly _____ that the required
(c) 66 (d) 68 reasonable degree of care and caution was not _____ by
(e) None of these hospital in the treatment of the patient.
96. 4 10 33 136 ? 4116 (a) reflect, compressed
(a) 822 (b) 534 (b) fix, advancement
(c) 685 (d) 745 (c) show, proceeded
(e) 548
(d) observe, considered
97. 4000 800 ? 80 40 8
(e) indicate, taken
(a) 140 (b) 100
(c) 400 (d) 160 105. Students from the Middle East and the African countries
(e) 200 have _____ been _____ contributors to the pool of foreign
98. 3 4 16 75 364 ? students in the university.
(a) 783 (b) 828 (a) traditionally, major
(c) 1293 (d) 1945 (b) conservatively, crucial
(e) None of these (c) suprisingly, most
99. The area of circle is seven times the numerical value of its (d) intuitively, salient
circumference. What is the circumference of the circle?
(e) annually, lucid
(a) 616 units (b) 132 units
106. The minister _____ about various _____ being provided
(c) 88 units (d) Cannot be determined
by officials for the pilgrims including direction wise colour
(e) None of these
coded passenger enclosures.
100. The compound interest earned on a sum is 3 years at 15%
per annum compounded annually is ` 6500.52. What is the (a) ceased, functions
sum? (b) enquired, facilities
(a) ` 12480 (b) ` 10500 (c) relied, opportunities
(c) ` 14800 (d) ` 13620 (d) shy, evidence
(e) None of these (e) asked, deity
y
o
u
rs
m
254 IBPS Specialist (I.T.) Officer Exam 2013

a
h
b
107. It is _____ to drive the evening and night when the _____ 119. It is all well known that (a) / women are generally in favour

o
of accidents looms large due to non-functional streetlights.

o
of (b) / light topics like jokes and expressions (c) / that

b
(a) juvenile, planning

.w
causing laughter all around. (d) / No Error (e).
(b) easy, risk

o
120. The evening breeze (a) / won’t carrying the poetry (b) / of

rd
(c) instinctive, fame peace beyond (c) / the school building (d). No Error (e).

p
(d) difficult, threat

re
121. We are bring in the idea that (a) / European rehabilitation
(e) natural, feature

s
focussed (b) / on a multi-disciplinary approach (c) / towards

s
.c
108. An overcast sky made Sunday less than _____ for city
chronic pain. (d) / No Error (e).

o
residents, who going by weather experts have more in _____.

m
122. If parents are able to (a) / get their children into schools (b)
(a) difficult, fate (b) perfect, store
/ that are far away, the (c) / next challenge is transportation.
(c) holiday, sky (d) dream, sharing
(d) / No Error (e).
(e) steady, fame
109. Many teachers _____ the lack of professional freedom as 123. The perception of others (a) / particularly family members
the _____ for leaving the job. (b) / changed when he (c) / qualified on a government job.
(a) cited, reason (d) / No Error (e).
(b) explained, force 124. Every house should (a) / have the device as it (b) / protects
(c) claimed, understanding people from a (c) / common household disaster. (d). No
(d) argued, culprit Error (e).
(e) believe, ground 125. Automated baggage handling systems are (a) / ensuring
110. Skeptics would not _____ that the earth actually moves, let that on the time passengers (b) / are out of the plane their (c)
alone that it _____ around the sun. / baggage is already waiting for them. (d) / No Error (e)
(a) permit, orbits
(b) accept, revolves DIRECTIONS (Qs. 126-140) : Read the following passage
(c) experiment, circles carefully and answer the questions given below it.
(d) assume, went
Certain words have been printed in bold to help you locate
(e) challenge, spins
them while answering some of the questions.
DIRECTIONS (Qs. 111-125) : Read each sentence to find out Indeed the western recession is really the beginning of good
whether there is any grammatical error or idiomatic error in it. news for India ! But to understand that we will have to move away
The error, if any, will be in one part of the sentence. The number for a while from the topic of western recession ........ to the Japanese
of that part is the answer. If there is “No Error” the answer is (e). recession! For years the Japanese style of management has been
(Ignore errors of punctuation if any.) admired. However, over the last decade or so. one key question
111. The couple’s work in (a) / up-grading rural technicians (b) / has sprung up ‘if Japanese management style is as wonderful as
has set a benchmarking (c) / for future generations. (d) / No described then why has Japan been in a recession for more than
Error(e). a decade?’
112. It has taking almost (a) / a year for India (b) / to let its The answer to this question is very simple. Culture play a
pessimism (c) / translate into fewer jobs. (d) / No Error (e). very important part in shaping up economies. What succeeds in
113. The city needs an airport (a) / that can efficiently manage one culture falls in another. Japanese are basically no materialistic.
(b) / a constantly flow of (c) / passengers and flights. (d) / And however rich they become unlike others, they cannot just
No Error (e). keep throwing and buying endlessly. And once they have
114. This group of (a) / rural achievers is very (b) / different than
everything they need; there is a saturation point. It was only
the (c) / ones in the past. (d) / No Error (e).
when companies like Toyota realized that they cannot keep selling
115. The government has announced (a) / plans to creating (b) /
one million new (c) / training places. (d) / No Error (e). cars endlessly to their home market that they went really aggressive
116. The argument assumes that (a) / early detection of the in the western markets - and the rest is history. Japanese companies
disease (b) / will lead to an immediate drop in (c) / the mortality grew bigger by catering to the world markets when their home
rating from this disease. (d) / No Error (e). markets shrunk.
117. The two most important numbers (a) / which the mandarins And the markets have to shrink finally after attaining a level
of an (b) / economy have to watch (c) / are inflation and of affluence! And that’s great for the world because earth needs
unemployment. (d) / No Error. (e). sustainable development. It does not need monstrous consumers
118. Witnessed the young soldier’s ability (a) / to repeatedly hit who keep consuming at the cost of the environment and the earth.
bull’s eye at (b) / arms training, instructors pushed him (c) / There should be limits to growth so that consumers are not
to participate in the Army marksmanship competition. (d) / converted into material dustbins for the profit of a handful of
No Error (e). corporations.
y
o
u
rs
m
IBPS Specialist (I.T.) Officer Exam 2013 255

a
h
b
Owing to the materialistic culture elsewhere, it was possible (b) Japanese banks have provided loans indiscriminately

o
o
to keep selling newer products to the consumers despite having to the creditworthy as well as non creditworthy people.

b
existing ones which served equally well. They were lured through

.w
(c) Because Japanese markets have been going through a
advertising and marketing techniques of ‘dustbinisation’ of the

o
period of continuous recession since the last decade.

rd
customer; and then finally, once they became ready customers, (d) The unlimited growth of the Japanese makes has come

p
they were given loans and credits to help them buy more and

re
at the cost of the western market.

s
more. When all the creditworthy people were given loans to a

s
(e) None of these

.c
logical limit, they ceased to be a part of the market. Even this

o
would have been understandable. If it could work as an eye opener. 128. Why does the author foresee the markets being created in

m
Instead of taking the ‘Right Step’ as Toyota did, they preferred to the developing countries instead of America and Europe?
take a ‘shortcut’. Now banks went to the non creditworthy people (a) All developing countries have materialistic culture.
and gave them loans. The people expectedly defaulted and the (b) Developed countries are willing to make an effort to
entire system collapsed. achieve globalization.
Now like Toyota western companies will learn to find new (c) American and European markets have had a large
markets. They will now lean towards India because of its common number of credit defaulters.
man! The billion plus population in the next 25 years will become, (d) Recession has not hit the markets of developing
a consuming middle-class. Finally, the world’s attention will shift countries yet.
to the developing world. Finally, there will be a real surge in (e) None of these
income of these people and in the next fifty odd years, one can
129. According to the author, what is the main cause of Japanese
really hope to see an equal world in terms of material plenty, with
recession?
poverty being almost nonexistent! And this will happen not by
selling more cars to Americans and Europeans. It will happen by (a) Only a handful of corporations earned profits and not
creating markets in India. China, Latin America and Africa, by the people in general.
giving their people purchasing power and by making products (b) Non creditworthy people defaulted which led to a
for them. collapse of the entire system.
The recession has made us realize that it is not because of (c) Consumers were sold newer products which were
worse management techniques, but because of limits to growth. similar in quality to the existing ones.
And they will realize that it is great for planet earth. After all, how (d) Japanese do not purchase endlessly and thus when
many cars and houses must the rich own before calling it enough? products had been sold to every customer, the markets
It’s time for them to look at others as well. Many years back, to slowed down.
increase his own profits. Henry Ford had started paying his (e) None of these
workers more, so that they could buy his cars. In similar fashion,
130. How does the author foresee the future globalization as an
now the developed world will pay the developed world will pay
analogy to Henry Ford’s example?
the developing world people so that they can buy their cars and
washing machines. (A) Car companies would start selling cars in developing
The recession will kick-start the process of making the entire countries as well.
world more prosperous, and lay the foundation of limits to growth (B) By paying the developing world the developed world
in the west and the foundation of real globalization in the world – would increase its own profit, in turn bringing affluence
of the globalization of prosperity. And one of its first beneficiaries to developing world as well.
will be India. (C) To earn profit, the companies in developing countries
126. What does the author mean by the “Right Step” in the would move to foreign land.
passage? (a) Only A (b) Only B
(a) Giving loans to creditworthy people only (c) Only C (d) Only A and C
(b) Considering market growth along with environment (e) None of these
protection.
131. According to the passage, which of the following was NOT
(c) Restricting people to buy only such products which
an effect of providing loans and credits to the customers?
are needed by them.
(d) To start looking at newer avenues and markets. (A) The non creditworthy people defaulted.
(e) None of these (B) People bought new products which were not needed.
127. Although admired since years, why did the scepticism over (C) Poverty became non existent.
the Japanese management style start since the last decade? (a) Only A (b) Only B
(a) Japanese companies have been moving out of their (c) Only A and B (d) Only B and C
home markets since the last decade. (e) Only C
y
o
u
rs
m
256 IBPS Specialist (I.T.) Officer Exam 2013

a
h
b
132. Why is recession the beginning of good news for India in 139. CONSUMING

o
the author’s view? (a) Destroying (b) Exhausting

o
b
(A) India can provide an attractive market to the western (c) Greedy (d) Curtailing

.w
companies. (e) Spending

o
rd
(B) India has remained largely unaffected by recession 140 SURGE

p
owing to its huge population. (a) Decrease (b) Deteriorating

re
(C) Indians keep purchasing products despite owning (c) Weakening (d) Atrophy

s
s
equally good products.

.c
(e) Crumble
(a) Only C (b) Only B

o
m
(c) Only A (d) Only B and C DIRECTIONS (Qs. 141-150) : In the following passage there
(e) None of these are blanks, each of which has been numbered. These numbers
133. What does the author mean by ‘Dustbinisation’ of the are printed below the passage and against each, five words are
customer? suggested, one of which best fits the blank appropriately. Find
(a) Convincing the customer to buy products he does not out the appropriate word in each case.
need.
(b) Denying the non creditworthy people of any loans. Twenty years (141) now, nearly 60% of the world’s
(c) Denying more loans to people who have already taken population will live in urban areas. The impact of urbanization
loans to a logical limit. might not all be positive on India as urban expansion is happening
(d) Moving from old customers at the home market to at a much (142) rate than infrastructure expansion.
foreign markets. Sustainability issues need to be (143) so that economic
(e) None of these development is not at the (144) of public health. Some urban
134. Why according to the author is the current recession great services that ought to be in (145) in a city like water, electricity,
for ‘Planet Earth’?
transport etc. need special consideration.
(A) It will make people non-materialistic like the Japanese.
TERI has put together a detailed report that (146)
(B) The unlimited market growth which caused hazards to
sustainability in the provision of basic urban services in Indian
the environment would be checked to a certain extent.
(C) Banks will now provide loans only to the creditworthy cities.
people. (147) public transport is a major reason for the proliferation
(D) Developing countries will also be benefited by shifted of private vehicles on the road. Respiratory illness in children
markets. living in urban areas is on the (148) with more cases of Asthma
(a) Only A (b) Only B and D being (149) because of pollution. The future of cities of Indian
(c) Only A and B (d) Only B dreams depends on (150) we can build better cities today.
(e) None of these 141. (a) till (b) since
(c) from (d) after
DIRECTIONS (Qs. 135-137) : Choose the word which is most (e) on
similar in meaning to the word printed in bold as used in the 142. (a) faster (b) slower
passage.
(c) changed (d) speed
135. CATERING (e) quick
(a) Considering (b) Lending 143. (a) speculated (b) believed
(c) Supplying (d) Working (c) imagined (d) considered
(e) Indulging (e) understand
136. KEY 144. (a) payment (b) rate
(a) Foundation (b) Solution (c) costs (d) charge
(c) Requisite (d) Difficult (e) expense
(e) Important 145. (a) abundance (b) large
137. AGGRESSIVE (c) functional (d) vicinity
(a) Violent (b) Determined
(e) location
(c) Demanding (d) Offensive
146. (a) bring (b) emphasizes
(e) Brutish
(c) speculates (d) postulates
DIRECTIONS (Qs. 138-140) : Choose the word/phrase which is (e) requests
most opposite in meaning to the word printed in bold as used in 147. (a) Good (b) Competent
the passage. (c) Absence (d) Inadequate
(e) Sufficient
138. PROSPEROUS
148. (a) multiplication (b) expansion
(a) Distressed (b) Helpless
(c) rise (d) inflation
(c) Worse (d) Worthless
(e) Underprivileged (e) grow
y
o
u
rs
m
IBPS Specialist (I.T.) Officer Exam 2013 257

a
h
b
149. (a) produced (b) develop 157. In Oracle, what is the default number of transactions that

o
MAXTRANS is set to if not specified?

o
(c) composed (d) resulted

b
(e) reported (a) 512 (b) 10

.w
150. (a) if (b) whether (c) 40 (d) 1

o
(e) 255

rd
(c) unless (d) provided

p
158. To which pin on the RAM chip does the address decoder
(e) weather

re
connect in order to signal which memory chip is being

s
s
accessed?

.c
PROFESSIONAL KNOWLEDGE (I.T.) (a) The address input (b) The output enable

o
m
(c) The chip enable (d) The data input
(e) The data output
151. In Oracle, who owns the data dictionary? 159. Which of the following switching methods provides the
(a) Oracle (b) SYS greatest frame throughput?
(c) The DBA (d) SYSTEM (a) Store-and-forward switching
(e) None of these (b) Frame-tag switching
152. The reason the data outputs of most ROM ICs are tri-state (c) Cut-through switching
outputs is to: (d) ISL switching
(a) allow for three separate data input lines. (e) Store-and-backward switching
160. Which kind of lock includes a keypad that can be used to
(b) allow the bidirectional flow of data between the bus
control access into areas?
lines and the ROM registers.
(a) Cipher (b) Warded
(c) permit the connection of many ROM chips to a common (c) Device (d) Tumbler
data bus. (e) Typelock
(d) isolate the registers from the data bus during read 161. A gateway is:
operations. (a) a device that enables users to send information
(e) None of these simultaneously over a network without slowing down
153. To drop a column that is used as a foreign key, first: transmission
(a) drop the primary key (b) an electronic device that connects several computing
(b) drop the table devices to a network
(c) a point in one network that is an entrance point to
(c) drop the foreign key constraint
another network
(d) all of these
(d) a device that links two or more segments of a network
(e) none of these and helps direct traffic
154. In the straight CGI approach to database connectivity on (e) None of these
the internet: 162. Which of the following is considered a vulnerability of
(a) the external program is located between the client and SNMP?
the web server (a) Clear text community strings
(b) the external program is located between the database (b) Its use of TCP
server and the database (c) The fact that it is on by default in Windows 2000 server
(c) the external program is located between the web server (d) The fact that it is on by default in Windows XP
Professional
and the database server
(e) None of these
(d) there is no external program
163. Network routing information distributed among routers is
(e) None of these stored in which of the following?
155. What is the name of the protocol used to eliminate loops? (a) Flash memory (b) Route table
(a) Switching (b) ISL (c) Metric table (d) NVRAM
(c) Frame tagging (d) Spanning Tree Protocol (e) Router memory
(e) Scanning 164. If the destination did not receive a segment, how will the
156. The effect of the ROLLBACK command in a transaction is TCP host know to resend the information?
the following: (a) The ACK received will not include the segment number
(a) Undo all changes to the database resulting from the that was not received.
(b) The ACK received will include the segment number
execution of the transaction
that was not received.
(b) Undo the effects of the last UPDATE command
(c) The sending host will send a PACK to verify segment
(c) Restore the content of the database to its state at the receipt
end of the previous day (d) The destination host will send a YACK message back
(d) Make sure that all changes to the database are in effect to the sending host
(e) None of these (e) None of these
y
o
u
rs
m
258 IBPS Specialist (I.T.) Officer Exam 2013

a
h
b
165. What are the effects of mixing RAM modules with different 174. In Oracle, which is more appropriate to store a small list of

o
o
speed ratings? values in a single column in each row in your address table?

b
.w
(a) The system runs at the speed of the slowest RAM (a) ORACARRAY (b) Nested table

o
stick. (c) Index organized table (d) Index

rd
(b) The system runs normally. (e) VARRAY

p
re
(c) The system runs at the speed of the memory bus. 175. While searching a website, you have been unable to find

s
information that was on the site several months ago. What

s
(d) The system may not run, or it crashes periodically.

.c
(e) None of these might you do to attempt to locate that information?

o
m
166. What kind of scheme is the HTTP protocol? (a) Visit Google’s cached page to view the older copy.
(a) get/put (b) store/forward (b) Forget about it, as there is no way to find this
(c) queuing (d) search/return information.
(e) request/response (c) Visit a partner site of the organization to see if it is
167. What type of error does a memory parity error create? there.
(a) A fatal exception error (d) Use the wayback machine.
(b) An NMI error (e) None of these
(c) A corrupt Windows operating system file 176. Which layers of the OSI, model are included in the lower
(d) A GPF error layers?
(e) A GPA error (a) Application, Session, Presentation
168. Which is not a part of the Oracle database block? (b) Physical, Transport, Data Link, Network
(a) Header (b) Row directory (c) Data link, Physical, Network
(c) Data (d) Freelists (d) Session, Data Link, Physical
(e) None of these (e) None of these
169. Which of the following cannot be used as the scope when 177. From smallest to largest, rank the following logical pieces of
the database: data block, tablespace, extent, segment.
using a JavaBean with JSP?
(a) tablespace, segment, extent, data block
(a) Page (b) Request
(b) data block, segment, extent, tablespace
(c) Session (d) Application
(c) segment, extent data block, tablespace
(e) Response
(d) data block, extent, segment, tablespace
170. Which of the following is used for integrity?
(e) None of these
(a) DES (b) Diffie-Hellman
178. Where are cookies stored?
(c) SED (d) AES
(a) On the server (b) In web. xml
(e) MD5
(c) On the client (d) In HTML
171. What is ISL used for?
(e) None of these
(a) To allow an Ethernet interface to understand frame
179. How is power supplied to a low-power USB device?
tags
(a) Through a power cable
(b) To make two Ethernet interfaces appear as one
(b) From an external power supply
(c) To connect an Ethernet switch with a high-speed core
(c) Directly from the computer’s power supply
switch such as ATM
(d) Through the USB cable
(d) To allow simultaneous routing and switching
(e) None of these
(e) None of these
180. When you are selecting a mouse for a particular computer
172. Which of the following describes programs that can run
system, what is the most important consideration?
independently, travel from system to system, and disrupt
(a) The type of drivers that come with the mouse
computer communications?
(b) The length of the mouse cord
(a) Trojans (b) Viruses (c) The type of connector the mouse is equipped with
(c) Idlers (d) Droppers (d) The number of buttons the mouse has
(e) Worms (e) None of these
173. You need to configure a switch from a remote subnet. Which 181. Programs that automatically submit your search request to
of the following must be configured on the switch? several search engines simultaneously are called:
(a) Console port (b) IP (a) Metasearch engines (b) Webcrawlers
(c) Hostname (d) SNMP (c) Spiders (d) Hits
(e) Default gateway (e) None of these
y
o
u
rs
m
IBPS Specialist (I.T.) Officer Exam 2013 259

a
h
b
182. You are planning on using a single network that supports 189. If 20 people need to communicate using symmetric-key

o
o
208 users. Which IP address class would you choose to be cryptography ..........symmetric keys are needed.

b
.w
the most efficient? (a) 190 (b) 200

o
(a) Class A (b) Class B (c) 20 (d) 19

rd
p
(c) Class C (d) Class D (e) 210

re
(e) Class E

s
190. What should you do as soon as you have successfully

s
.c
183. Which of the following options is not a JSP implicit object? created the database and the data dictionary?

o
m
(a) Out (b) In (a) Open the database for users to start adding data
(c) Response (d) Page (b) Start adding a bunch of tablespaces to allow users to
(e) None of these use
184. In Oracle, which statement is true about segments? (c) Leave it running for a few days to make sure that
nothing bad is going to happen
(a) Each table has its own segment
(d) Shut it down and take a cold backup
(b) A segment is created every time an extent is created,
extended, or altered (e) None of these
(c) An index segment is created every time a table is 191. Which of the following is the default mask of a class A IP
created address?
(d) A segment is deleted whenever a table is truncated (a) 255.0.0.255 (b) 255.255.0.0
(e) None of these (c) 255.0.0.0 (d) 255.255.255.0

185. All of the following are basic principles of networks, except: (e) None of these

(a) each computer must have a network card 192. What is Internet 2?

(b) there must be communications media connecting the (a) A new type of cabling system for Wide Area Networks
network hardware devices (b) A second local area network that can connect with
(c) there must be at least one connecting device another online LAN to share access

(d) each computer must have software that supports the (c) A new standard for Internet browsers
movement of information (d) An association to develop advanced Internet
(e) None of these technology
(e) None of these
186. Which of the following could be a legitimate Media Access
Control address? 193. Which one of the following objects is passed to a JavaBean
(a) 192.168.254.3 (b) 3FA2. 4756.F9A3 when one of its properties is set via a JSP action?
(a) ServletRequest
(c) A5514 (d) C1.3A.77.5B
(b) HttpServletRequest
(e) None of these
(c) ServeletResponse
187. The term, “hypertext,” means:
(d) HttpServeletResponse
(a) Non-sequential writing
(e) None of these
(b) Hypermedia
194. If you want to locate the hardware address of a local device,
(c) Blinking text
which protocol would you use?
(d) Text with heavy formatting
(a) ARP (b) RARP
(e) None of these
(c) ICMP (d) PING
188. You receive an e-mail message that informs you that a terrible
(e) PONG
virus has been unleashed on the internet and that you should
195. In Oracle, which tablespace is the minimum required when
warn all the people in your Address Book. Which of the
following have you most likely encountered? creating a new database?

(a) A virus hoax (b) A filter (a) Undo tablespace (b) Temporary tablespace
(c) System tablespace (d) Users tablespace
(c) Spam (d) A virus
(e) None of these
(e) A worm
y
o
u
rs
m
260 IBPS Specialist (I.T.) Officer Exam 2013

a
h
b
196. Database redesign is not terribly difficult if the: (c) Business logic is mostly processed on clients

o
o
(a) database is structured (d) Business logic may be processed on the server

b
.w
(b) database is well-designed (e) None of these

o
(c) database has no data

rd
199. The process by which the structure of the database is

p
(d) database is relatively small modified to eliminate hidden dependencies and repeating

re
s
(e) database is relatively large groups is:

s
.c
197. Which of the following are normally used to initialize a (a) enforcing data integrity

o
m
computer system’s hardware? (b) enforcing referential integrity
(a) Bootstrap memory (c) indexing
(b) Volatile memory (d) normalization
(c) External mass memory
(e) none of these
(d) Static memory
200. Which of the following function is not performed by TCP?
(e) Random access memory
(a) Flow control
198. Which of the following statements is not true about two-
(b) Sequencing
tier-client-server database architecture?
(a) SQL statements are processed on the server (c) Error checking

(b) SQL statements may be processed on some of the (d) Subnetting


clients (e) None of these
y
o
u
rs
m
IBPS Specialist (I.T.) Officer Exam 2013 261

a
h
b
o
o
b
.w
o
rd
p
re
Conclusion I: True

s
s
Conclusion II: True

.c
1. (a) Crops \ Both conclusion I and conclusion II follows.

o
m
Pulses
4. (e) Journals
Beans Seed Journals Books

Books or Diaries

Conclusion I: False
Conclusion II: True Conclusion I: False
\ Only conclusion II follows Conclusion II: True
\ Only conclusion II follows.
Vegetables 5. (c)
Fruit
2. (e)
Potatoes
Crops
Apples
Pulses

or Beans Seed

Vegetables
Fruit
Potatoes
Conclusion I: True
Apples Conclusion II: False
\ Only conclusion I follows.
6. (c) It is clearly mentioned that the project of road
Conclusion I: False construction has crossed its first deadline. Therefore.
Conclusion II: False Option (d) is not correct.
\ Neither conclusion I nor conclusion II follows. The Conclusion - the work of the road is going on - is
true.
3. (a)
7. (b) Option (b) strengthens the statement.
Vegetables
Fruit 8. (a) Obviously option (a) can be inferred from the given
Potatoes information.
Apples > S>
(9-11) :
> S>T>R
or Either P or Q is of 158 cm.
9. (a) It is clear that T is shorter than S.
Vegetables 10. (e) Q is either 158 cm high or more than 158ccm
Fruit 11. (d) Either three of four persons are shorter than Q
Potatoes P > Q > S >T > R or Q > P > S > T > R
12. (e) From the given information we cannot infer option (e)
Apples 13. (d) Obviously option (d) is correct.
y
o
u
rs
m
262 IBPS Specialist (I.T.) Officer Exam 2013

a
h
b
(14-15) : 26. (e) E < F £ G = H > S

o
Conclusion I : G > S : True

o
b
14. (d) –1 Conclusion II : F £ H : True

.w
–1 27. (a) P £ Q < W = L

o
–1

rd
Conclusion I : L > P : True
T RAV E L U D KUS B

p
+1 Conclusion II : Q £ L : True

re
+1 (28-30) : After careful analysis of the given input and various

s
s
steps of rearrangement it is evident that words and

.c
+1

o
numbers are rearranged from right to left. In the first

m
step one word is rearranged and in the second step
–1 one number is rearranged. The words are rearranged in
–1 alphabetical order and the numbers are arranged in
–1 ascending order from right to left.
CO R N E R MDQDPS Input : 23 you 13 wake 81 me 43 before 72 go 34 up
+1
+1 Step I : 23 you 13 wake 81 me 43 72 go 34 up before
Step II : 23 you 13 wake 81 me 43 72 go 34 up 13 before
+1
Step III : 23 you wake 81 me 43 72 34 up go 13 before
Similarly Step IV : you wake 81 me 43 72 34 up 23 go 13 before
Step V : you wake 81 43 72 34 up me 23 go 13 before
–1 Step VI : you wake 81 43 72 up 34 me 23 go 13 before
–1 Step VII : you wake 81 72 48 up 34 me 23 go 13 before
–1 Step VIII : you 81 72 wake 43 up 34 me 23go 13 before
SUR V E Y U D X T VS Step IX : 81 you 72 wake 43 up 34 me 23 go13 before
+1 28. (a) me
+1 29. (d) It is step V.
+1 30. (d) Option (d) is step II.
31. (e) It is mentioned in the passage that 50 - 80 per cent of all
15. (a) people who suffer a heart attack are able to return to
work. Therefore, the inference is definitely false.
–1 32. (b) It is mentioned in the passage that the persons who
–1
have suffered a heart attack should avoid high pressure,
–1
T ORHSP
stressful work. Therefore, it may be concluded that the
GR O U P S
+1 inference is probably true.
+1 33. (c) There is no information about this inference.
+1 34. (d) The use of term ‘definitely’ in the inference makes it
doubtful. Therefore, the inference is probable false.
(16-20) : 35 (a) Consider the very first line of the passage. It is clear
from the first line of the passage that the inference is


definitely true.
8 2 9 how art thou (36 - 40): Sitting arrangement


(Wi
good U fe
9 5 8 thou art Fem of Q)
ale (Wife of V)

Male, Lecturer
1 5 8 7 3 thy good and thou bad. Q T Female

16. (e) The code for ‘thy’ may be ‘1’ or ‘7’ or ‘3’
(Wife of S) Scientist
17. (b) The code for ‘thou’ is ‘8’ Female Male
18. (c) The code for ‘how’ is ‘2’ P V
19. (a) thou Þ 8; good Þ 5;
The code for ‘no’ may be ‘0’
20. (b) How Þ 2; good Þ 5;
thou Þ 8; art Þ 9.
21. (a) Clearly option (a) is the cause. S R
Male, Doctor Lawyer, Male
22. (d) Clearly option (d) is the effect.
23. (c) Clearly option (c) is the cause. J
24. (e) Obviously option (e) is an assumption. (Wife of R)
25. (e) Obviously option (e) is the effect. Female
y
o
u
rs
m
IBPS Specialist (I.T.) Officer Exam 2013 263

a
h
b
36. (d) P is to the immediate left of S. 57. (e) Percentage increase

o
37. (a) J is third to the left of T .

o
25 - 18 7

b
38. (b) T is, the wife of V = ´ 100 = ´ 100 = 38.88 » 39%

.w
18 18
39. (c) Wives of Q and V are immediate neighbours.

o
rd
40. (c) Except in UTV, in all othes the third person is sitting 58. (a) The word VISITING has 8 letters in which I comes

p
between the first and the second persons. In UTV, the thrice.

re
second person is sitting between the first and the third \ Number of arrangements

s
s
persons.

.c
8!

o
(41 - 44) : =

m
3!
L T Z V Y X W U R = 8 ´ 7 ´ 6 ´ 5 ´ 4 = 6720
E I
G 59. (b) Number of students who opted for dancing
F
T H
T 1800 ´ 38
= = 684
100
41. (c) Z is fourth to the left of W.
42. (d) V is second to the right of T. 60. (c) Required ratio = 32 : 10
43. (b) Except in XV, in all others first person is to the = 16 : 5
immediate right of the second person. 61. (d) Required percentage
8-5
Z Y X W V U T = ´ 100 = 60%
L T Z V Y X W U R 5
E I
G 62. (a) Number of students who opted for cricket and painting
F H
T T 1800 ´ (32 + 7)
=
44. (a) 100
45. (a)
Obviously option (a) is the cause. = 702
46. (c)
Obviously option (c) is the cause. 63. (b) Required percentage
47. (d)
Option (d) may be and effect.
48. (a)
Option (a) would weaken the argument put forward by 45 - 40
= ´ 100
the committee to the sports ministry. 40
49. (e) Option (e) would strengthen the argument. 5
= ´ 100 = 12.5
50. (c) Option (c) would strengthen the argument of 40
government.
64. (d) Require ratio = 48 :32
51. (d) Speed of train
= 3 :2
320
= = 6.4m/sec. 6255.22
50 65. (a) ? = = 15.8
18.5 ´ 21.4
52. (b) Let the number of all sweets be x, then
1.5 ´ 78
x
-
x
=1 66. (b) ? = = 234
0.5
280 420
67. (d) ? = 302.46 + 395.72 - 123.47
3x - 2 x
Þ =1 = 698.18 - 123.47 = 574.71
840
Þ x = 840 68. (c) 3
? = 3 4096 + 3 64

840 = 3 16 ´ 16 ´ 16 + 3 4 ´ 4 ´ 4
\ Required answer = =2
420 = 16 ÷ 4 = 4
53. (e) Required ratio \ ? = 4 ´ 4 ´ 4 = 64
= (15 + 20) : (10 + 26) = 35 : 36
800 ´ ? 750 ´ 22
54. (b) Required percentage 69. (d) = 293 -
100 100
23
= ´ 100 = 23 Þ 8 ´ ? = 293 - 165 = 128
98
55. (b) It is obvious from the table. 128
Þ ?= = 16
56. (a) Required number of persons interviewed = 101 8
y
o
u
rs
m
264 IBPS Specialist (I.T.) Officer Exam 2013

a
h
b
70. (b) Females in company P 35

o
Company S Þ ´ 100 = 87.5

o
5 40

b
= ´1200 = 500

.w
12
Females in company L 40

o
Company T Þ ´ 100 = 80

rd
8 50

p
= 400 ´ 83. (b) Required average

re
20

s
= 160 150

s
= = 30 thousand

.c
\ Require percentage 5

o
m
84. (a) Required ratio = 35 : 40 = 7 : 8
= 500 :160
85. (a) Total possible outcomes = 52C1= 52
= 25 : 8 Favourable outcomes = 2
71. (c) Require percentage \ Required probability
12 2 1
= ´ 100 = 48 = =
25 52 26
72. (a) Require average 86. (a) 40 m
4000 D C
= = 800
5
73. (b) Females in company M
5
= ´ 600
20
= 150 24 m
74. (c) Males in company N and company P
4 7 m
= ´ 800 + 1200 ´ 14
5 12
= 640 + 700 = 1340 A 14 metre B
3 3
75. (b) ? = (13.001) = (13) 1 2
= 2197 = 2200 Required area = ´ pR
4
76. (d) ? = 55 ´ 55 + 5 1 22
= ´ ´ 14 ´ 14
= 3025 + 5 = 3030 4 7
= 154 sq.metre
100 ´ 50
77. (a) ?= ¸ 50 = 1 87 .(a) Let Farah’s age 8 years ago be x years
100
Farah’s present age = ( x + 8) years
78. (a) ? = 999 + 900 - 350
9x
= 1549 \x+8 = Þ 7 x + 56 = 9 x
7
79. (e) ? = 23 ´ (2)-2 ¸ (4) -4 Þ 2 x = 56
2 Þ x = 28
= = 2 ´ 28 = 29 = 512
-4 Farah’s present age = 28 + 8 = 36 years
(4)
80. (a) Required average Her daughter’s age 3 years ago
35 + 45 + 35 + 40 + 50 1
= = 36 ´ = 6 years
5 6
205 88. (d) C.P. of 40 kg of mixture
= = 41 thousand
5 = `[(25 ´ 32) + (15 ´ 36)]
81. (a) Required ratio =35 : 30
=7:6 = ` (800 + 540)
82. (e) Percentage sale: = ` 1340
20 S.P.of 40 kg of mixture
Company P Þ ´ 100 = 57
35 = ` (40 ´ 40.2)
30 Profit= ` (1608 - 1340) = ` 268
Company Q Þ ´ 100 = 66.7
45
25 268
Company R Þ ´ 100 = 71.4 Profit % = ´ 100 = 20%
35 1340
y
o
u
rs
m
IBPS Specialist (I.T.) Officer Exam 2013 265

a
h
3523 - 3459 = 64 = 43

b
89. (a)

o
3459 - 3243 = 216 = 63

o
I. x 2 + 5x + 6 =0

b
\ ? = 3243 - 83 = 3243 - 512 = 2731

.w
Þ x 2 + 2x + 3x + 6 =0 2731 - 1731 = 1000 = 103

o
1731 - 3 =1728 =123

rd
Þ x(x + 2) + 3(x + 2) =0
95. (c)t The pattern is :

p
Þ (x + 3)(x + 2) =0

re
10 ´ 2 - 2 = 20 - 2 = 18

s
Þ x =-3 or - 2

s
.c
18 ´ 2 - 2 = 36 - 2 = 34

o
II. y 2 + 7 y + 12 = 0

m
34 ´ 2 - 2 = 68 - 2 = 66
Þ y 2 + 4 y + 3y + 12 = 0
66 ´ 2 - 2 = 132 - 2 = 130
Þ y (y + 4) + 3(y + 4) = 0
96. (c) The pattern is :
Þ (y + 3)(y + 4) = 0
Þ y = -3 or - 4 4 ´ 2 + 2 = 10
On comparing the value of equ.(i) and equ.(ii) 10 ´ 3 + 3 = 33
x³y 33 ´ 4 + 4 = 136
90. (d) 136 ´ 5 + 5 = 680 + 5 = 685
I. x 2 - 9 x + 20 = 0 97. (c) The pattern is :
Þ x 2 - 5 x - 4 x + 20 = 0 4000 ¸ 5 = 800
Þ x( x - 5) - 4 ( x - 5) = 0 800 ¸ 2 = 400
= ( x - 4) ( x - 5) = 0 400 ¸ 5 = 80
x = 4 or 5 80 ¸ 2 = 40
40 ¸ 5 = 8
II. y 2 - 13 y + 42 = 0
98. (d) The pattern is :
Þ y 2 - 7 y - 6 y + 42 = 0
Þ y ( y - 7) - 6( y - 7) = 0 3 ´ 1 + 13 = 4
Þ ( y - 6)( y - 7) = 0 4 ´ 2 + 23 = 8 + 8 = 16
Þ y = 6 or 7 16 ´ 3 + 33 = 48 + 27 = 75
Here, y > x
75 ´ 4 + 43 = 300 + 64 = 364
91. (d) 2 x + 3 y = 14 ...(I)
364 ´ 5 + 53 = 1820 + 125
4 x + 2 y =16 ...(II)
By equation (I) × 2 – equation II. = 1945
4 x + 6y - 4 x - 2 y = 28 - 16
Þ 4 y =12 Þ y = 3 99. (c) pR 2 = 7 ´ 2pR
From equation I, Þ R=14
2 x + 3 ´ 3 = 14 \ Circumference of circle
5 22
Þ 2 x = 14 - 9 = 5 Þ x = =2pR=2 ´ ´ 14
2 7
Here, y > x = 88units
92. (e) I. x = 625 = ± 25 100. (a) Let the principal be ` x, then

II. y = 676 = ±26 3


æ 15 ö 650052
No relation can be established between x and y. x ç1+ ÷ -x=
93. (d) I. x 2 + 4x + 4 = 0 è 100 ø 100
( x + 2)2 = 0 Þ x = – 2 23 23 23 650052
Þ x´ ´ ´ - x =
II. y 2 - 8 y + 16 = 0 20 20 20 100
650052 ´ 8000
Þ ( y - 4)2 = 0 Þ 12167 x - 8000 x =
100
Þ y =4
650052 ´ 8000
Here, y > x Þx= = 156 ´ 80 = ` 12480
94. (b) The pattern is : 100 ´ 4167
y
o
u
rs
m
266 IBPS Specialist (I.T.) Officer Exam 2013

a
h
b
101. (b) Keep busy = keep engaged/involved. 135. (c) The meaning of the word coater (Verb) as used in the

o
passage is : to provide the things that a particular type

o
102. (a) Cripple (Verb) =to damage somebody’s body so that

b
they are no longer able to walk or move normally: of person wants.

.w
disable. Look at the sentence:

o
rd
103. (e) Share (Verb) = to tell other people about your ideas,
They only publish novels which cater to the

p
experiences and feelings.

re
massmarket.
104. (e) Indicate (Verb) = to show that something is true or

s
Hence, the words catering and supplying are

s
exists.

.c
105. (a) Traditionally (Adverb) = not changing for a long time. synonymous.

o
m
106. (b) Enquire about = to ask somebody for some information. 136. (e) The meaning of word Key (Adjective) as used in the
107. (d) Threat = the possibility of trouble, danger or disaster. passage is: most important: essential: critical: vital.
108. (b) Perfect = Excellent; very good. Look at the sentence:
In store = waiting to happen to somebody
109. (a) He played a key role in the dispute
110. (b) Hence, the words key and important are synonymous.
111. (c) Here, has set a benchmark......should be used. 137. (b) The meaning of the word Aggressive (Adjective) as
Benchmark = something which can be measured and used in the passage is : acting with force and
used as a standard that other things can be compared determination in order to succeed.
with.
112. (a) Structure of sentence in Present Perfect: Look at the sentence:
Subject + has/have +V3 (Past participle). A good sales person has to be aggressive in today’s
Hence, It has taken almost.........should be used. competitive market.
113. (c) An adjective qualifies a noun. Hence, a constant Hence, the words aggressive and determined are
(Adjective) flow (Noun)...........should be used. synonymous.
114. (c) There is a preposition related error, Hence, different
138. (e) The meaning of the word Prosperous (Adjective) as
from the .........should be used.
used in the passage is : rich and successful; affluent.
Look at the sentence :
American english is significantly different from British The word Underprivileged (Adjective) means: having
English. less money and fewer opportunities than others;
115. (b) Infinitive = To + V1(Plural Verb) disadvantaged.
Hence, to create ......should be used here. Hence, the words prosperous and under privileged
116. (d) Here, mortality rate from this disease....... should be are antonymous.
used.
139. (a) The meaning of the word Consume (Verb) as used in
Rate = a measurement of the number of time something
the passage is : to use something.
happens or exists during a particular period.
Rating = a measurement of how good, popular, Hence. the words Consuming and destroying and
important somebody/something is in relation to other antonymous
things. 140. (a) The meaning of the word Surge (Noun) as used in the
117. (e) passage is: a sudden increase of feeling: a sudden
118. (a) Having witnessed (Past Participle) the young soldier’s increase in the amount.
ability.........should be used. Look at the sentence:
119. (d) Here, Present Simple i.e. that cause laughter all around We are having trouble keeping up with the recent surge
.............should be used as the sentence is in the present in demand.
Tense.
Hence, the words Surge and decrease are antonymous.
120. (b) Infinitive without to follows wouldn’t/shouldn’t. 141. (c) 142. (a) 143. (d) 144. (e) 145. (a)
121. (a) Here, we are bringing in the idea......should be used. 146. (b) 147. (d) 148. (c) 149. (e) 150. (b)
122. (d) 151. (b) 152. (c) 153. (c) 154. (c) 155. (d)
156. (a) 157. (a) 158. (c) 159. (c) 160. (a)
123. (d) There is preposition related error. Hence, qualified for
161. (c) 162. (a) 163. (e) 164. (b) 165. (d)
a government job.........should be used.
166. (e) 167. (b) 168. (d) 169. (e) 170. (e)
124. (b) Here, have the device which (Pronoun) ....... should be 171. (a) 172. (e) 173. (b) 174. (c) 175. (a)
used. 176. (b) 177. (d) 178. (c) 179. (d) 180. (c)
125. (b) There is preposition related error.Hence, ensuring that 181. (a) 182. (c) 183. (b) 184. (b) 185. (d)
at the time passengers.......should be used. 186. (a) 187. (a) 188. (d) 189. (a) 190. (b)
126. (d) 127. (c) 128. (e) 129. (d) 130. (b) 191. (d) 192. (d) 193. (b) 194. (d) 195. (c)
131. (e) 132. (c) 133. (c) 134. (b) 196. (b) 197. (a) 198. (c) 199. (b) 200. (e)
y
o
u
rs
m
a
h
b
o
o
b
.w
IBPS BANK PO/MT CWE

o
rd
p
re
EXAM 2013

s
s
.c
Based on Memory

o
m
REASONING ABILITY 3. Ramkumar is an Engineering graduate in computers with
78% marks passed out in 2007 at the age of 23 yr. Since, then
DIRECTIONS (Qs. 1-5) : Study the following information he is working as a Software Manager in an engineering firm.
carefully and answer the questions given below: He doesn't want to sign the bond for ` 50000. He has cleared
the selection test with 72% marks.
An organization wants to recruit system analysts. The following
4. Nishant is an Electronics Engineer passed out in June, 2010
conditions apply.
at the age of 22 yr. Since, then he is working as a Programmer
The candidate must
in a software company. He has passed the selection test
(i) be an engineering graduate in computer/IT with at least 60%
with 66% marks and is willing to sign the bond.
marks.
(ii) have working experience in the field of computer at least for 5. Kalyani is an Engineer with 72% marks in Telecommunication.
2 yr after acquiring the requisite qualification. She has just completed 27 yr of age. She has cleared the
(iii) have completed minimum 25 yr and maximum 30 yr of age as selection test with 59% marks. She is willing to sign the
on 1.12.2013. bond.
(iv) be willing to sign a bond for ` 50000. DIRECTIONS (Qs. 6) : Analyse the following passage and answer
(v) have secured minimum 55% marks in selection test. However, the question.
if a candidate fulfils all other conditions
Except Some words are highly inflammable. Fusion is one them. You can
A. at (i) above, but is an Electronics Engineer with 65% or more get two sets of people into a war mode by just uttering the words
marks the case is to be referred to the General Manager 'fusion music'. One set will breathe fire and say it violates the
(GM)-IT. purity of music the other set will tell you earnestly that it opens up
B. at (iv) above, but has an experience of atleast 5 yr as a the borders of music.
Software Manager, the case is to be referred to the VP. 6. From the purists perspective, the 'war' between the two set
In each question below, detailed information of candidate is given. You
of people can best be
have to carefully study the information provided in each case and take
(a) categorized as an ideological conflict between two ideas
one of the following courses of actions based on the information and
the conditions given above. You are not to assume anything other than (b) termed as a conflict between generations the younger
the information provided in each question. All these cases are given to versus the older generation
you as on 01.12.2013. You have to indicate your decision by marking (c) an attempt to preserve the core principles
answers to each question as follows: (d) seen as an attempt of people at the margin to occupy
Give Answer: centre stage
(a) If the case is to be referred to VP (e) seen as preserving the social identity of purists
(b) If the case is to be referred to GM 7. Unlike other retail outlets, where items are purchased in any
(c) If the data provided is not sufficient to take a decision number of units the customer wants, in super-markets items
(d) If the candidate is to be selected are grouped in bulk packages. This bulk buying offers saving
(e) If the candidate is not to be selected to the customer. The option to buy at wholesale prices by
1. Ms. Suneeta is an IT Engineer with 60% marks at graduation buying in bulk makes super-market a practical choice for
as well as in selection test. She is working as a Software budget-conscious consumers.
Engineer for last 3 yr after completing engineering degree Which of the following assumption may be drived from the
and has completed 27 yr of age. She is willing to sign the above information.
bond of ` 50000. (a) Super-markets often have greater buying power and
2. Rakesh Rao is a Computer Engineer Graduate and thereafter
lower overhead costs, so they can offer a greater variety
is working as a Software Manager for last 6 yr. He has secured
of products than regular retail outlets
72% marks at graduation and 67% marks in selection test.
His date of birth is 5th December, 1984. He is not willing to (b) Super-markets are often more conveniently located and
sign the bond for ` 50000. have better parking facilities
y
o
u
rs
m
a
h
268 IBPS Bank PO/MT CWE Exam 2013

b
o
o
(c) The emergence of super-markets has caused many small DIRECTIONS (Qs. 11-13) : Study the information given below

b
retail stores to close down and thus eliminate

.w
carefully to answer the following questions
competitions

o
rd
(d) It is economically wise to buy single items since bulk In a certain code language the following lines written as:

p
packages seldom offer significant savings 'lop eop aop fop' means 'Traders are above laws'

re
(e) The financial savings from purchasing bulk packages 'fop cop bop gop' means 'Developers were above profitable'

s
s
may outweigh the inconvenience of being unable to 'aop bop uop qop' means 'Developers stopped following traders'

.c
purchase in any number of units that suits the

o
'cop jop eop uop' means 'Following maps were laws'

m
customers' need 11. 'Developers are following laws' would be correctly written as
8. Nations do not complete with each other in the way (a) 'bop cop uop eop' (b) 'lop bop eop uop'
corporations do. (c) 'oup cop lop aop' (d) 'gop cop uop qop'
Which of the following most favours the weakness of the (e) None of these
argument? 12. 'qop gop cop eop' would correctly mean
(a) Trade deficit is a sign of national strength, profits are a (a) profitable laws were stopped
sign of corporate strength (b) developers stopped following laws
(b) Increase in human development index improves national (c) traders were above profitable
standing, increase in market share improves corporate (d) were laws profitable traders
standing (e) None of the above
(c) Climate change negotiations lead to global 13. 'aop qop bop' would correctly mean
improvement; CSR initiatives lead to image (a) following were above
improvement (b) traders stopped developers
(d) Nations go to war to capture territory, corporates (c) developers are laws
(d) traders above stopped
contend against each other to capture market share
(e) laws are stopped
(e) None of the above
9. Civilization has taught us to be friendlier towards one DIRECTIONS (Qs. 14-18) : In each of the questions below are
another given four statements followed by three conclusions numbered I,
Which of the following most favours the strengthens of the II and III. You have to take the given statements to be true even if
they seem to be at variance from commonly known facts. Read all
argument?
the conclusions and then decide which of the given conclusions
(a) Cats are loyal to their children, whereas men are loyal
logically follows from the given statements disregarding
to their communities commonly known facts.
(b) Elephants move in a herd, whereas men live in nuclear
families 14. Statements: All petals are flowers. Some flowers are buds.
(c) Lions protect their own territories, whereas men capture Some buds are leaves. All leaves are plants.
other men's territories Conclusions: I. Some petals are not buds.
(d) Nilgai and Cheetal stay together, whereas men of one II. Some flowers are plants.
race dominate another III. No flower is plant.
(e) None of the above (a) Only I follows (b) Either II or III follows
(c) I and II follow (d) Only III follows
10. The mushrooming of business schools in the country is a
(e) None of the above
cause for shortage of faculty with Ph.D qualification. In
15. Statements: Some pens are keys. Some keys are locks. All
addition, the higher pay and generous fringe benefits given locks are cards. No card is paper
by industry has encouraged qualified people to not seek Conclusions:
academic positions. I. No lock is paper.
Which of the following statements, if true, would tend to II. Some cards are keys.
STRENGTHEN the argument? III. Some keys are not paper.
(a) The average salary for industry positions in Gujarat is (a) I and II follow (b) Only I follows
more than the average salary for faculty positions in (c) Only II follows (d) All follow
some business schools in Ahmedabad by around 30% (e) None follows
(b) The average salary for industry positions in Gujarat is 16. Statements: Some pearls are gems. All gems are diamonds.
less than the average salary for faculty positions in a No diamond is stone. Some stones are corals.
top business school in Ahmedabad by around 30% Conclusions:
(c) The average salary for recent Ph. D graduates in the I. Some stones are pearls.
industry is 20% higher than that in academics II. Some corals being diamond is a possibility.
(d) The rate of growth of salaries for the industry positions III. No stone is pearl.
has been higher than the rate of growth of salaries for (a) Only I follows (b) Only II follows
academic positions for the past three years (c) Either I or III follows (d) I and II follow
(e) None of the above (e) None of these
y
o
u
rs
m
a
h
IBPS Bank PO/MT CWE Exam 2013 269

b
o
o
17. Statements: Some apartments are flats. Some flats are 22. Who sits to the immediate right of E ?

b
buildings. All buildings are bungalows. All bungalows are (a) The person who works for Marketing department

.w
gardens. (b) C

o
rd
Conclusions: (c) B

p
I. All apartments being building is a possibility (d) The person who works for HR department

re
(e) A
II. All bungalows are not buildings.

s
23. Who amongst the following sits exactly between C and the

s
.c
III. No flat is garden. person who works for HR department?

o
(a) None follows (b) Only I follows (a) B

m
(c) Either I or III follows (d) II and III follow (b) The person who works for Marketing department
(e) Only II follows (c) The person who works for Operations department
18. Statements: All chairs are tables. All tables are bottles. Some (d) D
bottles are jars. No jar is bucket. (e) G
Conclusions: 24. Who amongst the following sit between the persons who
I. Some tables being jar is a possibility. work for Marketing and Investment Banking departments
when counted for the left hand side of the person working
II. Some bottles are chairs.
for Marketing department?
III. Some bottles are not bucket. (a) F and G (b) E and C
(a) Only I follows (b) I and II follow (c) C and B (d) F and D
(c) All follow (d) Only II follows (e) B and D
(e) None of these 25. How many people sit between the person who works for
19. A person starts from point P in East and moves 12 m to point Operations department and A, when counted from the right
Q. Then, he moves right 8 m to point R. Again he moves hand side of A?
right for 6 m to point S. Then, he moves 6 m in the North to (a) One (b) Two
point T. Finally from there he goes to left for 6 m to point U. (c) Three (d) Four
Which of three point he would form a triangle whose all the (e) More than four
angles are less than 90°? DIRECTIONS (Qs. 26-30) : In these questions the symbols @. #,
(a) PTQ (b) QTR $, % and « are used with different meanings as follow.
(c) UTS (d) TSR
'A @ B ' means 'A is not smaller then B'.
(e) SQR
'A # B ' means 'A is neither smaller than nor equal to B'.
DIRECTIONS (Qs. 20-25) : Read the following information
'A $ B' means 'A is neither greater than nor smaller than B'.
carefully and answer the questions that follow.
'A % B' means 'A is not greater than B'.
Seven friends A, B, C, D, E, F and G are sitting around a circular 'A « B' means 'A is neither greater than nor equal to B'.
table facing either the centre or outside. Each one of them belongs In each questions, four statements showing relationships have
to a different department viz. Finance, Marketing Sales, HR, Cor- been given, which are followed by three conclusions I, II and III.
porate Finance, Investment Banking and Operations but not nec- Assuming that the given statements are true, find out which con-
essarily in the same order. clusion (s) is/are definitely true?
C sits third to the right of G.G faces the centre. Only one person 26. Statements: V $ Y, Y @ Z, Z % X, X # T
sits between C and the person working in the HR department Conclusions:
immediate neighbours of C face outside. Only one person sits I. T # Z II. X # Y
between F and D. Both F and D face the centre. D does not work III. Z « Y
in the HR department. A works in Investment Banking Depart- (a) None follows (b) Only I follows
ment. A faces the centre. Two people sit between the persons who (c) II and III follow (d) I and III follow
work in Investment Banking and Marketing Departments. The (e) Only III follows
person who works in Corporate Finance sits to the immediate left 27. Statements: R @ J, J % F, F « E, E % M
of E. C faces same direction as E. The person who works in corpo- Conclusions:
rate finance sits to the immediate left of the person who works for I. M # J II. F % M
Finance department. III. M « R
20. For which of the following departments does B work? (a) Only I follows (b) Only II follows
(c) Only III follows (d) I and II follow
(a) Finance (b) Marketing
(e) All follow
(c) HR (d) Corporate Finance
28. Statements: H#R, R@L, L « W, W%F
(e) Operations Conclusions:
21. What is position of B with respect to the person who works I. H # J II. F # L
for Sales department? III. H $ F
(a) Immediate right (b) Third to the left (a) Only I follows (b) I and II follow
(c) Second to the right (d) Second to the left (c) II and III follow (d) Either I or II follows
(e) Fourth to the right (e) All follow
y
o
u
rs
m
a
h
270 IBPS Bank PO/MT CWE Exam 2013

b
o
o
29. Statements: M # K, M $ F, F % Q, Q « H 35. Who among J, T, W, R and Q reached the office first?

b
Conclusions: Statements:

.w
I. H # K I. J reached before Q, R and T but after W.

o
rd
II. Q # K II. Q reached before R but after W.

p
III. Q @ M 36. Village 'F' is in which direction with respect to village 'K'?

re
(a) I and II follow (b) Either I or II follows Statements:

s
s
(c) All follow (d) II and III follow I. Village 'J' is to the East of village 'F' and to the North of

.c
(e) None of the above village 'K'.

o
m
30. Statements: D « Q, Q $ L, L # T, T % H II. Village 'R', which is to the South of village 'F' is to the
Conclusions: West of village 'K'.
I. D « L 37. The increase in the number of newspaper articles exposed
II. L @ H as fabrications serves to bolster the contention that
III. H # L publishers are more interested in boosting circulation than
(a) Only I follows (b) I and II follow in printing the truth. Even minor publications have staff to
(c) Either II or III follows (d) All follow check such obvious fraud.?
(e) None follow Which of the following may be the assumption of the given
31. In a code language 'PROVIDE' is written as 'MULYFGB', then argument?
what will be code for 'BECAUSE' in same languages (a) Newspaper stories exposed as fabrication are a recent
(a) YZHDRVB (b) ZHYDRVB phenomenon
(c) YHZDRVB (d) ZYDHVBR (b) Everything a newspaper print must be factually
(e) None of these verifiable
(c) Fact checking is more comprehensive for minor
DIRECTIONS (Qs. 32-36) : Each of the questions below consists publications that for major ones
of a question and two statements numbered I and II are given (d) The publishers of newspapers are the people who
below it. You have to decide whether the data provided in the decide what to print in their newspapers
statements are sufficient to answer the question. Read both the (e) None of the above
statements and answer the questions that follow. 38. The rate of violent crime in this state is upto 30% from last
Give answer: year. The fault lies entirely in our system of justice. Recently
(a) If the data is Statements I alone are sufficient to answer the our judges' sentences have been so lenient that criminals
question, while the data in Statement II alone are not sufficient can now do almost anything without fear of a long prison
to answer the question term.
(b) If the data in Statement II alone are sufficient to answer the The argument above would be weakened if it were true that
question, while the date in Statement I alone are not sufficient (a) 85% of the other States in the nation have lower crime
to answer the question rates than does this state
(c) If the data in Statement I alone or in Statement II alone is (b) white-collar crime in this state has also increased by
sufficient to answer the question over 25% in the last year
(d) If the data in both the Statements I and II are not sufficient (c) 35% of the police in this state have been laid off in the
to answer the question last year due to budget cuts
(e) If the data in both the Statements I and II together are (d) polls show that 65% of the population in this state
necessary to answer the question opposes capital punishment
32. How many children are there in the group if no two children (e) None of the above
have same weight? 39. All German philosophers, except for Marx, are idealists. From
Statements: which of the following can the statement above be most
I. Sahil is fifth from the top in order of weight if all the properly inferred?
children in the group were arranged in descending (a) Except for Marx, if someone is an idealist, then he or
order. she is a philosopher, as long as he German
II. Ramesh, who in heavier than 14 children in the group is (b) Marx is the only non-German philosopher who is an
immediately next to Sahil in weight. idealist
33. What is the code for 'healthy' in the code language? (c) If a German is an idealist, then he or she is a philosopher,
Statements: as long as he or she is not Marx
I. In the code language eat healthy food' is written as 'ka (d) Aside from the philosopher Marx, If someone is a
ma re'. German, then he or she is an idealist
II. In code language 'food for healthy people' is written as (e) None of the above
'ta ma jo re'. 40. During the SARS days, about 23, 500 doctors who had treated
34. How many brothers does 'H' have? SARS sufferers died and about 23, 670 doctors who had not
Statements: engaged in treatment for SARS sufferers died. On the basis
I. 'H' is sister of 'K' who is son of 'T'. of those Figures, it can be concluded that it was not much
II. T is mother of 'K' who is brother of 'H'. more dangerous to participate in SARS treatment during the
y
o
u
rs
m
a
h
IBPS Bank PO/MT CWE Exam 2013 271

b
o
o
SARS day than it was not to participate in SARS treatment. DIRECTIONS (Qs. 45-50) : Given an input line the machine

b
Which of the following would reveal most clearly the

.w
arranges the words and numbers in steps in a systematic manner
absurdity of the conclusion drawn above?

o
as illustrated below:

rd
(a) Counting deaths among doctors who had participated

p
in SARS treatment in addition to deaths among doctors Input line 59 dress fine shine 32 66 72 offer

re
who had not participated is SARS treatment Step I 72 56 dress fine shine 32 66 offer

s
s
(b) Expressing the difference between the numbers of Step II 72 shine 56 dress fine 32 66 offer

.c
o
deaths among doctors who had treated SARS sufferers Step III 72 shine 66 56 dress fine 32 offer

m
and doctors who had not treated SARS suffers as a Step IV 72 shine 66 offer 56 dress fine 32
percentage of the total number of deaths Step V 72 shine 66 offer 56 fine dress 32
(c) Separating deaths caused by accidents during the
Step VI 72 shine 66 offer 56 fine 32 dress
treatment to SARS suffers from deaths caused by infect
of SARS suffers STEP VI is the last step and the output in Step VI is the final
(d) Comparing death rates per thousand members of each output.
group rather than comparing total numbers of deaths As per the rules followed in the above steps, find out in each of
(e) None of the above the following questions the appropriate step for the given input.
45. Step IV of an input is '62 sound 56 sleep roam present 33 49'.
DIRECTIONS (Qs. 41-44) : Read the following passage carefully What will be the input definitely?
and answer the Question given below it. (a) Sound 62 sleep 56 roam present 33 49
Six friends Abhishek, Deepak, Mridul, Pritam, Ranjan and Salil (b) Sleep sound 62 56 roam present 33 49
married within a year in the months of February, April, July, Sep- (c) 62 Sound sleep 56 roam present 33 49
tember, November and December and in the cities of Ahmedabad, (d) Cannot be determined
Bengaluru, Chennai, Delhi, Mumbai and Kolkata, but not neces- (e) None of the above
sarily following the above order. The brides' names were Geetika, 46. Which of the following will be the third step for input 'jockey
Jasmine, Hema, Brinda, Ipsita and Veena, once again not follow- firm 36 43 growth chart 22 45 ?
ing any order. The following are some facts about their weddings. (a) 45 jockey 43 growth firm 36 chart 22
(b) 45 jockey 43 firm growth 36 chart 22
(i) Mridul's wedding took place in Chennai, however he was
(c) 45 jockey 43 growth 36 firm chart 22
not married to Geetika or Veena
(d) 45 jockey 43 firm 36 growth chart 22
(ii) Abhishek's wedding took place in Ahmedabad and Ranjan's
(e) None of the above
in Delhi; however neither of them was married to Jasmine or
47. Step II of an input is '53' window 42 50 door lock key 36'. How
Brinda
many more steps will be required to complete the
(iii) The wedding in Kolkata took place in February
arrangement?
(iv) Hema's wedding took place in April, but not in Ahmedabad (a) Three (b) Four
(v) Geetika and Ipsita got married in February and November (c) Five (d) Six
and in Chennai and Kolkata but not following the above (e) None of these
order 48. What will be the fifth step of an input whose first step is '85
(vi) Pritam visited Bengaluru and Kolkata only after his marriage journey train 36 54 daily 28 mansion'?
in December (a) 85 train 54 mansion 28 journey daily 36
(vi) Salil was married to Jasmine to September (b) 85 train 54 mansion journey 36 daily 28
41. Hema's husband is (c) 85 train 54 mansion 36 journey daily 28
(a) Abhishek (b) Deepak (d) There is no such step
(c) Ranjan (d) Pritam (e) None of the above
(e) Mridul 49. Which step will be the last step for an input whose second
42. Deepak's wedding took place in step is '63' Sour 18 56 grapes healthy 32 rise'?
(a) Bengaluru (b) Mumbai (a) IV
(c) Kolkata (d) Delhi (b) V
(e) Chennai (c) VIII
43. In Mumbai, the wedding of one of the friends took place in (d) VII
the month of (e) None of these
(a) April (b) September 50. Which word/number will be sixth from right in step fifth
(c) November (d) December whose second step is '63 Sour 18 56 grapes healthy 32 rise'?
(e) July (a) Rise
44. Salil's wedding was held in (b) 56
(a) Bengaluru (b) Chennai (c) Sour
(c) Kolkata (d) Delhi (d) 32
(e) Mumbai (e) 18
y
o
u
rs
m
a
h
272 IBPS Bank PO/MT CWE Exam 2013

b
o
o
51. To which of the following has the author attributed the 2008
ENGLISH LANGUAGE

b
Asian financial crisis?

.w
A. Reluctance or Asian governments to taper off the

o
DIRECTIONS (Qs. 51-58) : Read the following passage carefully

rd
economic stimulus.
and answer the questions given below it. Certain words have

p
B. Greed of Asian investors causing them to trad stocks

re
been printed in bold to help you to locate them while answering of American companies at high prices.

s
s
some of the questions. C. Inflated real estate prices in Asian countries.

.c
(a) None (b) Only (A)

o
The great fear in Asia a short while ago was that the region would

m
(c) Only (C) (d) (A) and (B)
suffer through the wealth destruction already taking place in the (e) Only (B)
U.S as a result of the financial crisis. Stock markets tumbled as 52. What does the author want to convey through the phrase
exports plunged and economic growth deteriorated. Lofty property "The world has not changed it has just moved places"?
prices in China and elsewhere looked set to bust as credit (a) At present countries are more dependent on Asian
economies than on the US economy
tightened and buyers evaporated. But with surprising speed, fear
(b) Economies have become interlinked on account of
in Asia swung back to greed as the region shows signs of recovery globalisation
and property and stock prices are soaring in many parts of Asia. (c) Asian governments are implementing the same
Why should this sharp Asian turnaround be greeted with economic reforms as developed countries
skepticism? Higher asset prices mean households feel wealthier (d) All economies are susceptible to recession because of
and better able to spend, which could further fuel the region's the state of the US economy
nascent rebound. But just as easily, Asia could soon find itself (e) None of the above
53. Which of the following can be said about the Chinese
saddled with overheated markets similar to the U.S. housing market. government's efforts to revive the economy?
In short, the world has not changed, it has just moved placed. (a) These were largely unsuccessful as only the housing
The incipient bubble is being created by government policy. In market improved
response to the global credit crunch of 2008. Policy makers in (b) The governments only concern was to boost investor
Asi< slashed interest rates and flooded financial sectors with cash confidence in stocks
in frantic attempts to keep loans flowing and economies growing. (c) These efforts were ineffectual as the economy recovered
owing to the US market stabilising
Thesf steps were logical for central bankers striving to reverse a
(d) These were appropriate and accomplished the goal of
deepening economic crisis. But there is evidence that there is too economic revival
much easy money around. It's winding up in stocks and real estate, (e) They blindly imitated the economic reforms adopted
pushing prices up too far and too fast for the undenying economic by the US
fundamentals. Much of the concern is focused on China where 54. Why do experts predict that Asian policymakers will not
government stimulus efforts have been large and effective, Money withdraw fiscal stimulus?
ir China has been especially easy to find. Aggregate new bank A. The US economy is not likely to recover for a long time.
lending surged 201% in first half of 2009 from the same period a B. Stock markets are yet to regain their former levels.
C. Fear of revolt by greedy citizens.
yea earlier, to nearly 51.1 turn on. Exuberance over a quick recovery
(a) None of these (b) Only (C)
which was given a boost by China's surprisingly strong 7.9% GDI
(c) (A) and (C) (d) Only (B)
growth in the second quarter has buoyed investor sentiment not (e) (B) and (C)
just for stocks but also for real estate. 55. What do the statistics about loans given by Chinese banks
Former U.S. Federal Reserve Chairman Alan Greenspan argued in 2009 indicate?
that bubbles could only be recognised in hand sight. But investor (a) There was hardly any demand for loans in 2008
who have been well schooled in the dangers of bubbles over the (b) The Chinese government has borrowed funds from the
past decade are increasingly wary that prices have risen too far US
ant that the slightest bit of negative, economic news could knock (c) China will take longer than the US to recover from the
economic crisis
markets for a loop. These fears are compounded by the possibility
(d) The GDP of China was below expectations
that Asia's central bankers will begin taking stops to shut off the (e) None of the above
money. Rumours that Beijing was on the verge of tightening cred 56. Why has investor confidence in the Chinese stock market
led to Shanghai stocks plunging 5%. Yet many economists believe been restored?
that, there is close to a zero possibility that the Chines government A. Existing property prices which are stable and affordable.
will do anything this year that constitutes tightening. And without B. The government has decided to tighten credit.
a major shift in thinking, the easy-money condition will stay in C. Healthy growth of the economy indicated by GDP
place. In a global economy that has produced more dramatic ups figures.
(a) Only (C) (b) (A) and (B)
and downs than anyone thought possible over the pas two years.
(c) All (A), (B) and (C) (d) Only (B)
Asia may be neading for another disheartening plunge.
(e) None of these
y
o
u
rs
m
a
h
IBPS Bank PO/MT CWE Exam 2013 273

b
o
o
57. What is the author's main objective in writing the passage? The common sense solution advocated by the government is to

b
(a) Illustrating that Asian economies are financially more adopt the business correspondent model. Where in bank agents

.w
sound than those of developed countries will go to villages to make cash payments and duly record them on

o
rd
(b) Disputing financial theories about how recessions can handheld, electronic devices. This solution is based on the wrong

p
be predicted and avoided diagnosis that distance separating villages from banks is the main

re
(c) Warning Asian countries about the dangers of issue. In order to accelerate payments, clear timelines for every

s
s
favouring fast growth and profits over sound economic- step of the payment process should be incorporated into the system

.c
principles

o
as Programme Officers often have no data on delays and cannot

m
(d) Extolling China's incredible growth and urging other exert due pressure to remedy the situation. Workers are both
countries to emulate it clueless and powerless with no provision for them to air their
(e) Advising governments about the changes in policy to grievances and seek redress. In drought affected areas the system
strengthen economic fundamentals of piece rate work can be dispensed with where work measurement
58. Why does the author doubt the current resurgence of Asian is not completed within a week and wages may be paid on the
economics? basis of attendance. Buffer funds can be provided to gram
(a) Their economies are too heavily reliant on the American
panchayats and post offices to avoid bottlenecks in the flow of
economy which is yet to recover
funds. Partial advances could also be considered provided wage
(b) Central banks have slashed interest rates too abruptly
payments are meticulously tracked. But failure to recognise
which is likely to cause stock markets to crash
problems and unwillingness to remedy them will remain major
(c) With their prevailing economic conditions they are at
risk for a financial crisis threats to the NREGS.
(d) Their GDP has not grown significantly during the last 59. What impact have late wage payments had on NREGS
financial year workers?
(e) None of the above (a) They cannot obtain employment till their dues are
cleared
DIRECTIONS (Qs. 59-65) : Read the following passage carefully
(b) They have benefited from the compensation awarded
and answer the questions given below it. Certain words have
to them
been printed in bold to help you to locate them while answering
(c) They have been unable to provide for their families
some of the questions.
(d) They have been ostracised by their families who
Delays of several months in National Rural Employment Guarantee depend on them for sustenance
Scheme (NREGS) wage payments and work sites where labourers (e) None of the above
have lost all hope of being paid at all have become the norm in 60. Which of the following factors has not been responsible for
many states. How are workers who exist on the margins of untimely payment of NREGS wages?
subsistence supposed to feed their families? Under the scheme, (a) Communication delays between agencies implementing
workers must be paid within 1 5 days, failing which they are entitled, the scheme
to compensation under the Payment of Wages Act - upto 3000 per (b) Improper record keeping
aggrieved worker. In reality, compensation is received in only a (c) Behind schedule release of payments by banks
few isolated instances. It is often argued by officials that the main (d) Drought conditions prevalent in the country
reason for the delay is the inability of banks and post offices to (e) Delays in work measurement
handle mass payments of NREGS wages. Though there is a grain 61. What has the outcome of disbursing NREGS wages through
of truth in this, as a diagnosis it is misleading. The 'jam' in the banks been?
banking system has been the result of the hasty switch to bank (a) Theft of funds by administration officials responsible
payments imposed by the Central Government against the for the scheme has reduced
recommendation of the Central Employment Guarantee Council (b) Increased work load for local government officials
which advocated a gradual transition starting with villages (c) Protests by workers who have to travel long distances
to the nearest bank to claim their wages
relatively close to the nearest bank. However, delays are not
(d) Time consuming formalities have to be completed by
confined solely to the banking system. Operational hurdles include
workers
implementing agencies taking more than fifteen days to issue
(e) None of the above
payment orders, viewing of work measurement as a cumbersome 62. According to the passage, which of the following has/have
process resulting in procrastination by the engineering staff and been the consequence (s) of delayed wage payments?
non maintenance of muster rolls and job card etc. But behind A. Compensation to victimised workers has amounted to
these delays lies a deeper and deliberate 'backlash' against the crores.
NREGS. With bank payments making it much harder to embezzle B. Banks will no longer be entrusted with remitting wages.
NREGS funds, the programme is seen as a headache by many C. Regulations to ensure punctual wage payments have
government functionaries the workload has remained without the come into force.
"inducements". Slowing down wage payments is a convenient (a) None of these (b) Only (A)
way of sabotaging the scheme because workers will desert NREGS (c) (A) and (C) (d) (A) and (B)
worksites. (e) (B) and (C)
y
o
u
rs
m
a
h
274 IBPS Bank PO/MT CWE Exam 2013

b
o
o
63. To which of the following has the author attributed the delay 67. Which of the following should be the SECOND sentence

b
in wage payments? after rearrangement?

.w
(a) Embezzlement of funds by corrupt bank staff (a) A (b) B

o
rd
(b) Lack of monitoring by the Central Employment (c) C (d) D

p
Guarantee Council (e) F

re
(c) An attempt to derai1 the NREGS by vested interests 68. Which of the following should be the THIRD sentence after

s
s
(d) Overworked bank staff deliberately delay payments to rearrangement?

.c
protest against extra work

o
(a) A (b) B

m
(e) Engineers efforts to wreck the NREGS because of low (c) C (d) D
wages
(e) E
64. Which of the following is NOT true in the context of the
69. Which of the following should be FIFTH sentence after
passage?
A. Workers are reluctant to open bank accounts as rearrangement?
branches are not conveniently located. (a) A (b) B
B. Local officials often delay wage payments in drought (c) C (d) D
prone areas to benefit workers. (e) E
C. The Government has not implemented ever)' 70. Which of the following should be the LAST (SIXTH)
recommendation of the Central Employment Guarantee sentence after rearrangement?
Council. (a) A (b) B
(a) Only (B) (b) (A) and (B) (c) C (d) D
(c) (B) and (C) (d) (A) and (C) (e) E
(e) All of these DIRECTIONS (Qs. 71-80) : In the following passage, there are
65. Which of the following can be considered a deficiency in blanks each of which has been numbered. These numbers are
the NREGS? printed below the passage and against each five words have
(a) Lack of co-ordination among Programme Officers been suggested, one of which fills the blanks appropriately. Find
(b) Local officials are unaware of correct operational out the appropriate word in each case.
procedures
(c) Workers have no means of obtaining redressal for Traditional bank architecture is based on bank branches. These
untimely wage payments branches ensure the physical (71) of a customer's savings. A
(d) Disbursing wages through banks instead of readily customer may go there to deposit and withdraw money, (72) loans
accessible post offices and (73) in other financial transactions. In the past two decades
(e) The Central Employment Guarantee Council is reluctant banking architecture has changed the Automated Teller Machine
to award compensation to workers (ATM) has been a big (74) and credit and debit cards have created
DIRECTIONS (Qs. 66-70) : Rearrange the following sentences new financial spaces. (75) the bank branch has remained the
bedrock of the banking system after all a person needs a bank
(A), (B), (C), (D) (E) and (F) into a meaningful paragraph and
account in a branch before he can operate a debit or ATM card.
then answer the questions given below it.
This may be about to change as technocrats now (76) cell phones
(A) Moreover salaries in public sector enterprises are not as as the new architecture of virtual banks. This has the potential to
competitive as those offered by private or foreign corporates, make branches (77). Cell phone banking looks especially relevant
connection for India, since it can penetrate the countryside cheaply and (78).
(B) This trend should be a wake up call for stakeholders to The world over cell phones are spreaing at a (79) rate and in India
examine why employees are seeking better opportunities alone new cell phone connection are growing at the rate of six
with private companies in India and abroad. million a month a rate of customer (80) that no bank can dream of.
(C) Public Sector Enterprises (PSEs) have been experiencing 71. (a) Knowledge (b) security
severe challenges in attracting motivating and retaining their (c) presence (d) confidentiality
key staff. (e) guarantee
(D) Having identified these as the reasons employees leave PSEs 72. (a) negotiate (b) advance
it is important empower stakeholders to find ways to remedy (c) credit (d) disburse
the situation. (e) sanction
(E) One reason is that young employees lured away to private 73. (a) pursue (b) interact
firms are more willing to undertake professional risks. (c) operate (d) enable
(F) Employees in specialist roles especially have become (e) engage
increasingly difficult to retain. 74. (a) drawback (b) hurdle
66. Which of the following should be the FIRST sentence after (c) consequence (d) luxury
rearrangement? (e) innovation
(a) A (b) B 75. (a) Despite (b) Although
(c) C (d) D (c) Even (d) Yet
(e) E (e) Until
y
o
u
rs
m
a
h
IBPS Bank PO/MT CWE Exam 2013 275

b
o
o
76. (a) View (b) realize DIRECTIONS (Qs. 86-90) : Which of the phrases (a), (b), (c)

b
(c) Display (d) engineer

.w
and (d) given below each statement should be placed in the
(e) assess blank space provided so as to make a meaningful and

o
rd
77. (a) essential (b) obsolete grammatically correct sentence? If none of the sentences is

p
(c) extant (d) retreat appropriate, mark (e) i.e., 'None of the above' as the answer.

re
(e) expired

s
s
78. (a) moderately (b) occasionally 86. Overlooking the fact that water scarcity intensifies during

.c
(c) compulsorily (d) indiscriminately summer, …………

o
m
(e) effectively (a) the government issued guidelines to all builders to limit
79. (a) phenomenal (b) gradual their consumption to acceptable limits
(c) proportionate (d) competitive (b) provision for rainwater harvesting has been made to
(e) projected aid irrigation in drought prone area
80. (a) discount (b) base (c) the water table did not improve even after receiving
(c) expansion (d) satisfaction normal monsoon in the current year
(e) relationship (d) many residential areas continue to use swimming pools,
wasting large quantities water
DIRECTIONS (Qs. 81-85) : Each question below has two blanks, (e) None of the above
each blank indicating that something has been omitted. Choose 87. Refuting the rationale behind frequent agitations for
the set of words for each blank that best fits the meaning of the formation of separate states, a recent report …………
sentence as a whole. (a) proved that such agitations result in loss of
81. In an effort to provide ……………. for higher education to governmental property
all, most of the universities have been providing education (b) indicated that the formation of small states does not
necessarily improve the economy
without adequate infrastructure, thus churning out
(c) suggested that only large scale agitations have been
……………. graduates every year.
effective in bringing out desired change in the past
(a) chances, fresh
(d) recommended dividing large states into smaller ones
(b) platform, capable
to improve governance
(c) opportunities, unemployable
(e) None of the above
(d) prospects, eligible 88. Achieving equality for women is not only a laudable goal,
(e) policy, incompetent ..............
82. The move to allow dumping of mercury ……………. An (a) political reforms are also neglected preventing women
outcry from residents of the area who …………….that high from entering legislatures and positions of power
levels of mercury will affect their health and destroy (b) the problem is also deep rooted in the society and
ecologically sensitive forest area. supported by it
(a) resulted, insist (b) provoked, fear (c) their empowerment is purposefully hampered by people
(c) incited, determined (d) activated, accept with vested interests in all sections of the society
(e) angered, believe (d) it is also equally difficult to achieve and maintain for a
83. Even as the …………….elsewhere in the world are struggling long term
to come out of recession, Indian consumers are splurging (e) None of the above
on consumer goods and to …………….this growth, 89. he has lost most of his life's earning in the stock market but
companies are investing heavily in various sectors. .....................
(a) economies, meet (b) countries, inhibit (a) he still seems to be leading his life luxuriously and
(c) governments, measure (d) nations, inflict extravagantly
(e) companies, counter (b) he could not save enough to repay his enormous debts
84. Drawing attention to the pitfalls of……………. solely on (c) stock market is not a safe option to invest money unless
Uranium as a fuel for nuclear reactors, Indian scientists done with caution
warned that Uranium will not last for long and thus research (d) experts have been suggesting to avoid investments in
on Thorium as its must be revived. stock market because of its unpredictable nature
(e) None of the above
(a) using, substitute
90. ……………..or else they would not keep electing him year
(b) believing, replacement
after year.
(c) depending, reserve
(a) The party leader gave a strong message to the mayor
(d) reckoning, option for improving his political style
(e) relying, alternative (b) Owing to numerous scandals against the mayor, he
85. has been taken against some wholesale drug dealers for was told to resign from the post immediately
dealing in surgical items without a valid license and (c) The mayor threatened the residents against filing a
maintaining a stock of …………….drugs. complaint against him
(a) Note, overwhelming (b) Step, impressive (d) The residents must really be impressed with the political
(c) Execution, outdated (d) Action, expired style of their mayor
(e) Lawsuit, invalid (e) None of the above
y
o
u
rs
m
a
h
276 IBPS Bank PO/MT CWE Exam 2013

b
o
o
QUANTITATIVE APTITUDE 5 4

b
(a) (b)

.w
7 7

o
DIRECTIONS (Qs. 91-95) : In each of the following questions, a 3

rd
question is followed by information given in three Statements I, (c) (d) Cannot be determined
8

p
re
II and III. You have to study the question along with the statements (e) None of these

s
and decide the information given in which of the statement(s) is 97. If the positions of the digits of a two-digit number are

s
.c
necessary to answer the question. interchanged, the number obtained is smaller than the

o
original number by 27. If the digits of the number are in

m
91. In how many days 10 women can finish the work? the ratio of 1 : 2, what is the original number?
I. 10 men finish the work in 6 days. (a) 36 (b) 63
3 (c) 48 (d) Cannot be determined
II. 10 women and 10 men finish the work in 3 days.
7
III. If 10 men work 3 days and after that 10 women are (e) None of these
98. One of the angles of a quadrilateral is thrice the smaller
deployed to work for men, the rest work is finished in 4
angle of a parallelogram. The respective ratio between
days. the adjacent angles of the parallelogram is 4:5.
(a) I and II (b) Any two of three Remaining three angles of the quadrilateral are in ratio
(c) I and III (d) II and III 4 : 11: 9 respectively. What is the sum of the largest and
(e) None of these the smallest angles of the quadrilateral?
92. What is the present age of Sabir? (a) 255° (b) 260°
I. The present age of Sabir is half of his father's age. (c) 265° (d) 270°
II. After five years the ratio of ages of Sabir and his father (e) None of these
is 6 : 11. 99. An aeroplane flies with an average speed of 756 km/h.
III. Sabir is younger to his brother by five years. A helicopter takes 48 h to cover twice the distance
covered by aeroplane in 9 h. How much distance will
(a) I and II (b) I and III
the helicopter cover in 18 h? (Assuming that flights are
(c) II and III (d) All of these non-stop and moving with uniform speed.)
(e) Cannot be determined (a) 5010 km (b) 4875 km
93. What is two digit number? (c) 5760 km (d) 5103 km
I. The difference between the number and the number (e) None of these
formed by interchanging the digit is 27.
II. The difference between two digits is 3. DIRECTIONS (Qs. 100-104) : Study the following table carefully
III. The digit at unit's place is less than that at ten place and answer the question given below.
by 3. Number of People taking Fresh Loans from Different Banks
(a) I and II (b) I and either II or III over the Year and the Percentage of Defaulters Amongst
(c) I and III (d) All of these them each Year
(e) None of these
94. What is the rate of interest Percent per annum? Bank
Year
I. An amount doubles itself in 5 yr on simple interest; P Q R S T
II. Difference between the compound interest and the 2004 27361 26345 25467 28246 30164
simple interest earned on a certain amount in two years 2005 32081 27456 32461 29435 35128
is ` 400.
2006 25361 28637 32652 29565 32443
III. Simple interest earned per annum is ` 2000.
(a) Only I (b) II and III 2007 23654 29045 32561 28314 36152
(c) Any two of three (d) I or II and III 2008 36125 30467 25495 23764 35463
(e) Only I or II and III 2009 35465 31963 27649 24356 33214
95. What is the cost of flooring the rectangular hall? 2010 34135 31974 28283 26553 31264
I. Length and the breadth of the hall are in the ratio of
3:2 Approximate Percentage of Defaulters Among them
II. Length of the hall is 48 m and cost of flooring is ` 850 each Year
Bank
Year
per sq m. P Q R S T
III. Perimeter of the hall is 160 m and cost of flooring is 2004 12 9 15 13 19
` 850 per sq m.
2005 24 8 17 20 23
(a) I and II (b) I and III
(c) Only III (d) I and either II or III 2006 22 13 16 21 25
(e) Any two of the three 2007 18 11 18 22 19
96. If the numerator of a fraction is increased by 20% and the 2008 12 10 13 23 18
denominator is increased by 25%, the fraction obtained is 2009 11 20 11 22 21
3
. What was the original fraction? 2010 9 21 1.2 21 23
5
y
o
u
rs
m
a
h
IBPS Bank PO/MT CWE Exam 2013 277

b
o
o
100. Approximately how many people taking a loan from Bank S 110. What is the ratio of the number of appeared aspirants from

b
.w
in the year 2006 were defaulters? States C and E together to that of the appeared aspirants
(a) 6490 (b) 6210 from States A and F together?

o
rd
(c) 5020 (d) 6550 (a) 17:33 (b) 11:13

p
(e) 5580

re
(c) 13:27 (d) 17:27

s
101. Approximately what was the difference between the number (e) None of these

s
.c
of defaulters of Bank Q in the year 2004 and 2005? 111. In which state the percentage of qualified aspirants to

o
(a) 175 (b) 125

m
appeared aspirants is the least?
(c) 190 (d) 205 (a) C (b) F
(e) 140
(c) D (d) E
102. In which of the following years was the number of
(e) G
defaulters of Bank R, the maximum among the given
112. What is the difference in the number of qualified aspirants
years?
(a) 2005 (b) 2006 in states D and G?
(c) 2007 (d) 2010 (a) 690 (b) 670
(e) None of these (c) 780 (d) 720
103. In which of the following years was the difference in (e) None of these
number of people taking loan from Bank P from the 113. What is the percentage of qualified aspirants with respect
previous year the highest? to appeared aspirants from states B and C taken together?
(a) 2008 (b) 2006 (Rounded off to two decimal places.)
(c) 2007 (d) 2005 (a) 23.11 (b) 24.21
(e) None of these (c) 21.24 (d) 23
104. Approximately what was the total number of defaulters of (e) None of these
Bank T in the years 2007 and 2008 together? 114. What is the ratio between number of candidates qualified
(a) 14110 (b) 13620
from States B and D together and the number of candidates
(c) 13250 (d) 14670
appeared from States 'C' respectively?
(e) 15330
(a) 8:37 (b) 11:12
DIRECTIONS (Qs. 105-109) : In the following number series, a
(c) 37 : 40 (d) 7 : 37
wrong number is given. Find out the wrong number.
(e) None of the above
105. 29, 37, 21, 43 ,13, 53, 5 DIRECTIONS (Qs. 115-119) : In the following questions, two
(a) 37 (b) 53 (c) 13 (d) 21 (e) 43 equations numbered I and II are given. You have to solve both
106. 600, 125, 30, 13, 7.2, 6.44, 6.288 the equations and give answers.
(a) 6 (b) 10 (c) 15 (d) 12
(e) None of these (a) if x > y
107. 80, 42, 24, 13.5, 8.75, 6.375, 5.1875 (b) if x ³ y
(a) 8.75 (b) 13.5 (c) 24 (d) 6.375 (e) 42 (c) if x < y
108. 10, 8, 13, 35, 135, 671, 4007 (d) if x £ y
(a) 8 (b) 671 (c) 135 (d) 13 (e) 35 (e) if x = y or the relationship cannot be established
109. 150, 290, 560, 1120, 2140, 4230, 8400 115. I. 12x2 + 11x + 12 = 10x2 + 22x
(a) 2140 (b) 560 (c) 1120 (d) 4230 (e) 290 II. 13y2 – 18y + 3 = 9y2 – 10y
DIRECTIONS (Qs. 110-114) : These questions are based on the
following data. The distribution of appeared and qualified 18 6 12 8
116. I. 2
+ - 2 = 2
aspirants in competitive examination from different States. x x x x

Total appeard aspirants = 45000 Total qualified aspirants = 9000 II. y3 + 9.68 + 5.64 = 16.95

117. I. 1225x + 4900 = 0


II. (81)1/4 y + (343)1/3 = 0

(2)5 + (11)3
118. I. I. = x3
6
II. 4y3 = – (589 ¸ 4) + 5y3
y
o
u
rs
m
a
h
278 IBPS Bank PO/MT CWE Exam 2013

b
o
o
119. I. (x7/5 ¸ 9) = 169 ¸ x3/5 124. For all the given plans, what was the difference in expenditure

b
on education and health?

.w
II. y1/4 × y1/4 × 7 = 273 ¸ y1/2
(a) ` 220400000 (b) ` 224000000

o
DIRECTIONS (Qs. 120-124) : Study the following table to answer

rd
(c) ` 22040000000 (d) ` 220400000000
these questions.

p
(e) None of these

re
125. The respective ratio between the present ages of son, mother,

s
Plan of Public Sector Under Various Plans Sector-wise

s
.c
Expenditure out of that total expenditure (in million) father and grandfather is 2 : 7 : 8 : 12. The average age of son

o
and mother is 27 yr. What will be mother's age after 7 yr?

m
Plan I II III IV V VI
(a) 40 yr (b) 41 yr
Public sector plan
19600 46720 85770 157240 394260 97500 (c) 48 yr (d) 49 yr
expenditure
(e) None of these
Social service 4180 7440 12960 24620 63720 14035
126. In an examination, Raman scored 25 marks less than Rohit.
Education 1530 2730 5890 7860 13360 25240 Rohit scored 45 more marks than Sonia. Rohan scored 75
Health 980 2140 2260 3370 7610 18210 marks which is 10 more than Sonia. Ravi's score is 50 less
Family Planning - 20 250 2780 4920 10100 than, maximum marks of the test. What approximate
Housing and percentage of marks did Ravi score in the examination, if he
330 800 1280 2470 11500 24880
urban services gets 34 marks more than Raman?
Water supply (a) 90 (b) 70
- - 1060 4740 10920 39220
and sanitation (c) 80 (d) 60
Social welfare and
(e) 85
1340 1750 2220 3400 15410 22700 127. 8 men and 4 women together can complete a piece of work in
related area
6 days. The work done by a man in one day is double the
120. In various plans in the ratio of expenditure on public sector, work done by a woman in one day. If 8 men and 4 women
which of the following graphs explain best the expenditure started working and after 2 days 4 men left and 4 new women
on water supply and sanitation? joined, in how many more days will the work be completed?
(a) 5 days (b) 8 days
(a) (b)
(c) 6 days (d) 4 days
(e) 9 days
128. Mr Giridhar spends 50% of his monthly income on household
items and out of the remaining he spends 50% on transport,
25% on entertainment, 10% on sports and the remaining
amount of ` 900 is saved. What is Mr Giridhar's monthly
(c) (d)
income?
(a) ` 6000 (b) ` 12000
(c) ` 9000 (d) Cannot be determined
(e) None of these
129. The cost of fencing a circular plot at the rate of `15 per m is
(e) None of these ` 3300. What will be the cost of flooring the plot at the rate
121. The ratio of public sector expenditure to the expenditure on of ` 100 per sq m?
social services was highest in which plan? (a) ` 385000 (b) ` 220000
(a) I (b) VI (c) ` 350000 (d) Cannot be determined
(c) V (d) II (e) None of these
(e) None of these 130. The simple interest accrued on a sum of certain principal in
122. In the successive plans in the ratio of public sector 8 yr at the rate of 13% per year is ` 6500. What would be the
expenditure there was a continuous decrease in which compound interest accrued on that principal at the rate of
sector? 8% per year in 2 yr?
(a) In no sector (b) Health (a) ` 1040 (b) ` 1020
(c) ` 1060 (d) ` 1200
(c) Education (d) Social services
(e) None of these
(e) Social welfare and related areas
123. For plan VI out of public sector expenditure, what per cent DIRECTIONS (Qs. 131-135) : Study the following information
carefully to answer the questions that follow.
of expenditure is on Housing and Urban services?
(a) 0.35 (b) 25 There are two trains A and B. Both trains have four different types
(c) 25.5 (d) 2.5 of coaches viz. General coaches, sleeper coaches, first class
(e) 20.5 coaches and AC coaches. In train A, there are total 700 passengers.
y
o
u
rs
m
a
h
IBPS Bank PO/MT CWE Exam 2013 279

b
o
o
Train B has 30% more passengers than train A. 20% of the Total number of employees = 3250

b
passengers of train A are in general coaches. One-fourth of the

.w
Respective Ratio of Men to Women in each Department
total number of passengers of train A are in AC coaches. 23% of

o
rd
the passengers of train A are in sleeper class coaches. Remaining Department Men Women

p
passengers of train A are in first class coaches. Total number of Production 4 1

re
passengers in AC coaches in both the trains together is 480. 30%

s
HR 12 13

s
.c
of the number of passengers of train B is in sleeper class coaches,
IT 7 3

o
10% of the total passengers of train B are in first class coaches.

m
Remaining passengers of train B are in general class coaches. Marketing 3 2
131. What is the ratio of the number of passengers in first class Accounts 6 7
coaches of train A to the number of passengers in sleeper 136. What is the number of men working in the Marketing
class coaches of train B? department?
(a) 13 : 7 (b) 7 : 13 (a) 462 (b) 454
(c) 32 : 39 (d) Data inadequate (c) 418 (d) 424
(e) None of these (e) None of these
132. What is the total number of passengers in the general coaches 137. What is the respective ratio of the number of women working
of train A and the AC coaches of train B together? in the HR department to the number of men working in the IT
(a) 449 (b) 459 department?
(c) 435 (d) 445 (a) 11:12 (b) 17:29
(e) None of these (c) 13:28 (d) 12:35
133. What is the difference between the number of passengers in (e) None of these
the AC coaches of train A and total number of passengers in 138. The number of men working in the production department
sleeper class coaches and first class coaches together of of the organisation forms what per cent of the total number
train B? of employees working in that department?
(a) 199 (b) 178 (a) 88% (b) 90%
(c) 187 (d) 179 (c) 75% (d) 65%
(e) None of these (e) None of these
134. Total number of passengers in general class coaches in both 139. The number of women working in the IT department of the
the trains together is approximately. What percentage of organization forms what per cent of the total number of
total number of passengers in train B? employees in the organization from all departments together?
(a) 35 (b) 42 (a) 3.2% (b) 4.8%
(c) 45 (d) 38 (c) 6.3% (d) 5.6%
(e) 31 (e) None of these
135. If cost of per ticket of first class coach ticket is ` 450, what 140. What is the total number of men working in the organization?
total amount will be generated from first class coaches of (a) 2198 (b) 2147
train A? (c) 2073 (d) 2236
(a) ` 100080 (b) ` 108000 (e) None of these
(c) ` 100800 (d) ` 10800
(e) None of these GENERAL AWARENESS
DIRECTIONS (Qs. 136-140) : Study the following pie chart
With special reference to Banking Industry
carefully to answer the questions.
141. Which of the following is a receipt listed in India and traded
Degree Wise Break-up of Employees Working in Various
in rupees declaring ownership of shares of a foreign
Departments of an Organization and
company?
the ratio of Men to Women
(a) Indian Depository Receipt (IDR)
(b) European Depository Receipt (EDR)
(c) Global Depository Receipt (GDR)
Accounts
50.4° Production (d) American Depository Receipt (ADR)
136.8° (e) Luxemburg Depository Receipt (LDR)
Marketing 142. A bank without any branch network that offers its services
79.2° remotely through online banking, telephone/mobile banking
and interbank ATM network alliances is known as
IT HR (a) Universal Banking (b) Indirect Bank
57.6° 36° (c) Door Step Bank (d) A Direct Bank
(e) Unit Banking
y
o
u
rs
m
a
h
280 IBPS Bank PO/MT CWE Exam 2013

b
o
o
143. Which of the following Indian Universities is Asia's largest (a) identity theft (b) hacking

b
residential university? (c) money laundering (d) espionage

.w
(a) Allahabad University (e) phishing

o
153. Deepak Lathore is related to which of the following sports?

rd
(b) Utkal University
(a) Hockey (b) Cricket

p
(c) Banaras Hindu University

re
(d) Anna University (c) Badminton (d) Football

s
(e) Weightlifting

s
(e) Jawaharlal Nehru University

.c
144. In October, 2013, which country has confirmed plans to create 154. Who among the following is the author of the book "The

o
Lowland"?

m
a secure mail service to protect its citizens and businesses
against foreign espionage? (a) Jhumpa Lahiri (b) Amitav Ghosh
(a) Mexico (b) Brazil (c) Salman Rushdie (d) Hamid Ansari
(c) Sweden (d) Germany (e) Chetan Bhagat
(e) None of these 155. Which of the following Indian actresses has recently
145. The campaign name 'Heal India' aims to create awareness (October, 2013 been honoured at the British House of
about which of the following diseases? Commons for her contribution to the global entertainment
(a) Mental illness (b) AIDS industry?
(c) Leprosy (d) Alzheimer (a) Shabana Azmi (b) Kareena Kapoor
(e) None of these (c) Nandita Das (d) Aishwarya Rai Bachchan
146. The target set by the UIDAI for issuance of Aadhaar cards (e) Vidya Balan
upto 2014 is 156. Who among the following is the current Chief Election
(a) 50 crore cards (b) 55 crore cards Commissioner (CEC) of India?
(c) 45 crore cards (d) 40 crore cards (a) KG Balakrishnan
(e) 60 crore cards (b) Ranjit Sinha
147. Which of the following nations has signed a com-prehensive (c) Montek Singh Ahluwalia
free trade agreement with European Union? (d) VS Sampath
(a) Japan (b) China (e) SY Quraishi
(c) Russia (d) Canada 157. The part of a company's earnings or profits which are paid
(e) None of these out to shareholders is known as
148. According to the provisions of the income Tax Act, 1961 a (a) capital gains (b) taxes
resident individual is categorised as a 'very senior citizen' (c) interest on borrowings (d) dividends
when he is (e) penal interest
(a) 80 yr of age or older (b) 75 yr of age or older 158. NABARD is responsible for regulating and supervising the
(c) 90 yr of age or older (d) 85 yr of age or older functions of
(e) 65 yr of age or older (a) Investment and Industrial Finance Banks
149. Who among the following has recently been conferred with (b) Cooperative Banks and Regional Rural Banks
the first Yash Chopra Memorial Award? (c) Corporate Finance and Overseas Banking Units
(a) Other than those given as options (d) Private Sector and Multinational Banks
(b) Madhuri Dixit Nene (e) Reserve Bank of India
(c) Ramesh Sippy 159. The government of India has announced a 'funding for
(d) Lata Mangeshkar lending' scheme. Who are the beneficiaries for this scheme?
(e) AR Rahman (a) Commercial Banks
150. Which of the following is an investment advisory discipline? (b) Regional Rural Banks
(a) Corporate Industrial Finance (c) Micro-finance Institutions
(b) Offshare Banking (d) Finance Departments of the State Governments
(c) Wholesale Banking (e) None of the above
(d) Wealth Management 160. The arrangement under which banks sell insurance products
(e) Trade Finance acting as the agents of the respective companies is called
151. The Aadhar-enabled Payment System (AEPS) is a bank-led the
model that facilitates banking facilities through banking (a) Insurance joint venture
correspondents across banks. However, Aadhaar- enabled (b) Bancassurance Model
basic types of banking transactions do 'not' include (c) Hybrid Insurance Model
(a) Aadhaar to Aadhaar funds transfer (d) Insurance Broking
(b) Small overdraft facility (e) Integrated Model
(c) Cash withdrawal 161. The concept of'Micro Credit' essentially concentrates on
(d) Balance enquiry (a) consumption smoothening as and when needed
(e) Cash deposit (b) providing safe place to hold savings
152. A type of fraud wherein criminals use an innocent person's (c) accepting deposits
details to open or use an account to carry out financial (d) provision of credit to the poor
transactions is known as (e) facility to transfer money
y
o
u
rs
m
a
h
IBPS Bank PO/MT CWE Exam 2013 281

b
o
o
162. With effect from July 1, 2012, for calculation of lending rates, (c) Reserve Bank of India (RBI)

b
the Reserve Bank of India has advised banks to switch over (d) World Bank

.w
to the (e) Department of Financial Services (DFS)

o
(a) MSF Rate System 173. A bank's 'fixed deposit' is also referred to as a

rd
(b) Reverse Repo Rate System (a) term deposit (b) savings bank deposit

p
re
(c) Bank Rate System (c) current deposit (d) demand deposit

s
(d) Repo Rate System (e) home savings deposit

s
.c
(e) Base Rate System 174. To improve access of the poor to banking, RBI has advised

o
163. An Equity share is also commonly referred to as banks to open branches with minimum infrastructure

m
(a) ordinary share (b) debenture supporting up to 8 to 10 Business Correspondents (BC) at a
(c) convertible share (d) security receipt reasonable distance of 3-4 km. Such branches are known as
(e) preferred stock? (a) Nodal branches (b) Micro branches
164. Which among the following Companies in India has a tie-up (c) Mini branches (d) Ultra small branches
with the Japanese Financial Nomura for insurance market? (e) Satellite branches
(a) ICICI Bank 175. Which of the following services relate to execution of
(b) UCO Bank
transactions directly with consumers, rather than
(c) Kotak Finance
corporations or other banks?
(d) Life Insurance Corporation of India
(e) None of the above (a) Wholesale Banking Services
165. The seed capital of Bhartiya Mahila Bank is (b) Industrial Banking Services
(a) ` 2000 crore (b) ` 1000 crore (c) Investment Banking Services
(c) ` 4000 crore (d) ` 3000 crore (d) Corporate Banking Services
(e) None of these (e) Retail Banking Services
166. The World Health Organization (WTHO) is a specialised 176. World Food Day is celebrated every year around the world on
agency of the United Nations (UN) that is concerned with (a) July 4
international Public Health. It is headquartered at (b) October 16
(a) Sweden (b) Switzerland (c) Other than those given as options
(c) United Kingdom (d) France (d) August 12
(e) Germany (e) May 18
167. The process by which a life insurance policyholder can 177. As per Census 2011, which of the following Indian states
transfer all rights, title and interest- under a policy contract has the lowest population density?
to a third person is known as (a) Nagaland (b) Manipur
(a) Assignment of the policy (c) Arunachal Pradesh (d) Himachal Pradesh
(b) Hypothecation of the policy (e) Meghalaya
(c) Reinvestment of the policy 178. The minimum age for becoming a member of Rajya Sabha is
(d) Negotiation of the policy (a) 28 yr (b) 40 yr
(e) Nomination of the policy (c) 30 yr (d) 35 yr
168. Which of the following communities is 'not' notified as a (e) 25 yr
'minority community' by the ministry of Welfare, Government
179. Banks' mandatory lending to farmers for agriculture, micro
of India?
and small enterprises and other weaker sections where in
(a) Sikhs (b) Zoroastrians
(c) Buddhists (d) Jains banks are required to lend up to 40% of the loans is generally
(e) Christians described as
169. In October, 2013, which of the following countries has decided (a) Para banking
to scrap its two currency system? (b) Sub-prime lending
(a) Mongolia (b) Sweden (c) Retail lending
(c) Cuba (d) Jamaica (d) Non-priority sector lending
(e) None of these (e) Priority sector lending
170. Cheraw, the Bamboo Dance, is of which Indian states? 180. Which of the following statements regarding the 'Direct
(a) Mizoram (b) Sikkim Benefits Transfer Scheme (DBT)' of the government of India
(c) Arunachal Pradesh (d) Manipur is 'not' true?
(e) Asom (a) The scheme covers LPG subsidies, pension payments
171. 8th National Conference of Krishi Vigyan Kendra-2013 was and scholarships
held at (b) Indirect transfers of benefits are more prone to leakage
(a) Chennai (b) Mumbai than direct transfers
(c) Lucknow (d) Kolkata (c) Under DBT, money is directly transferred into bank
(e) Bangalore accounts of beneficiaries
172. Which of the following institutions is regarded as the 'Lender (d) The scheme was launched on January 1, 2013 to cover
of the Last Resort' by Banks in India? 20 districts initially
(a) State Bank of India (SBI) (e) The scheme is likely to increase the subsidy bill of the
(b) The State Treasury government?
y
o
u
rs
m
a
h
282 IBPS Bank PO/MT CWE Exam 2013

b
o
o
191. VIRUS stands for
COMPUTER KNOWLEDGE

b
(a) Very Important Record User Searched

.w
181. Macros stored in the global macro sheet can be used (b) Verify Interchanged Result Until Source

o
(c) Virtual Information Resource Under Seize

rd
(a) in the current document only
(d) Very Important Resource Under Search

p
(b) in any document

re
(c) can be used only with other macros of the global macro (e) None of the above

s
192. Unlike filters queries can be saved as in a database.

s
sheet

.c
(d) not consistent behaviour (a) objects (b) filters

o
(c) database (d) Any of the above

m
(e) None of the above
182. About pasting from the clip board (e) None of these
(a) a part of the clip board contents can be pasted 193. Table of contents can be prepared by using
(b) whole of the contents of clip board can be pasted (a) macros
(c) sometimes (a) and sometimes (b) (b) headings as H1, H2, H3 and more in the document
(d) (a) and (b) (c) by table of contents in tools menu
(e) None ol the above (d) (b) and (c)
183. One of the following statements is not true for BUFFERS (e) None of the above
command 194. Table in Word is a grid of rows and columns, with each cell
(a) increasing numbers of BUFFERS can speed program
can have
execution, but only to a certain extent
(b) the more buffers that exist the more sectors can be (a) text or graphics (b) only text
stored In memory; hence fewer accesses of disk are (c) only graphics (d) both
necessary (e) None of these
(c) The BUFFERS command is used to establish the number 195. What is a database ?
ol disk buffers set up by MS-DOS during booting (a) It is a collection of data arranged in rows
(d) All of the above (b) It is a collection of data arranged in columns
(e) None of the above (c) It is a collection of data arranged in rows and columns
184. EPROM can be used for (d) All of the above
(a) erasing the contents of ROM (e) None of the above
(b) reconstructing the contents of ROM 196. Which switch should be used in the DIR command to view
(c) erasing and reconstructing the contents of ROM files in a directories ?
(d) duplicating the ROM (a) /P (b) /W
(e) None of the above (c) /S (d) /L
185. Attributes can be defined for (e) None of these
(a) entity (b) switch board 197. When a key is pressed on the keyboard, which standard is
(c) macro (d) pages
(e) None of the above used for converting the keystroke into the corresponding
186. Where will we find the referential integrity command ? bits?
(a) Tools (b) View (a) ANSI
(b) Format (d) Table (b) ASCII
(e) None of these (c) EBCDIC
187. Anything that is typed in a worksheet appears (d) ISO
(a) in the formula bar only (e) None of the above
(b) in tho active cell only 198. External database is
(c) in both active cell and formula bar (a) Database created in EXCEL
(d) in tho formula bar first and when we press ENTER it (b) Database created using DBMS package
appears in active cell (c) Database created in MS-Word
(e) None of the above (d) All of the above
188. Which bar is usually located below the Title Bar that provides (e) None of the above
categorised options?
199. Which command we will give if we want to show the database
(a) Menu Bar (c) Status Bar
(b) Toolbar (d) Scroll Bar objects with it's decription ?
(e) None of the above (a) Details
189. A pixel is (b) Show
(a) a computer program that draws picture (c) List
(b) a picture stored in the secondary memory (d) Any of the above
(c) the smallest resolvable part of a picture (e) None of the above
(d) a virus 200. Word allows user to import graphics from
(e) None of the above (a) the library which comes bundled with Word
190. How many types of cell references are available in Excel? (b) any where in the computer
(a) 3 (b) 4 (c) various graphics format like gif, bmp, png, etc
(c) 8 (d) 10 (d) only gif format
(e) None of these (e) None of the above
y
o
u
rs
m
a
h
IBPS Bank PO/MT CWE Exam 2013 283

b
o
o
b
.w
o
rd
p
re
Solutions (1-5) 14. (b) According to question,

s
s
.c
o
Candidate I II III IV V (A) (B)

m
Suneeta P P P P P Petals Buds Leaves
Rakesh P P P – P P
Flowers Plants
Ramkumar P P P – P P
OR
Nishant – P P P P
Kalyani û P P P P
Flowrs Plants
1. (d) Suneeta Fulfils all conditions so, she is to be selected.
2. (a) Rakesh Rao fulfils condition (B) instead of (IV) so, Petals
Leaves
his case is to be referred to VP.
3. (a) Ramkumar fulfils condition (B) instead of (IV) so, his
case is to be referred to VP. Buds
4. (c) Percentage marks of Nishant in graduation is not
given so, data is insufficient. Conclusions I. false
5. (e) Kalyani is telecommunication engineer so, she is not II. false ù
or
to be selected. III. false úû
6. (b) A war between the original and the improvised.
Hence, only either II or III follows.
7. (e) Buying in wholesale and bulk depends upon the
15. (d) According to question
choice of the customer’s choice and may cause of
financial saving.
8. (d) This option says that aggressiveness is a common
way for both. Pens Keys Locks Paper
9. (a) Cultivation taught us the moral duties regarding the
Cards
children and relatives.
11-13: OR
lop eop aop fop - Traders are above laws ® (i)
fop cop bop gop - Developers were above profitable ® (ii) Paper
aop bop uop qop - Developers stopped following traders ®
(iii)
cop job cop uop - Following maps were laws ® (iv)
Pens Keys
From (i) and (ii), fop - above
From (i) and (iii), aop - traders
From (ii) and (iii), bop - developers Locks
From (ii) and (iv), cop - were Cards
From (iii) and (iv), uop - following
From (i) and (iv), eop - laws Conclusions I. True
Therefore, remaining codes are II. True
lop - are [from (i)] III. True
gop - profitable [from (ii)] Hence, All conclusions follow.
qop - stopped [from (iii)] 16. (e) According to question,
jop - maps [from (iv)]
11. (b) Developers are following laws = bop lop uop eop
= lop bop eop uop gems Stone Corals
Pearls
12. (a) qop gop cop eop = Stopped profitable were laws
= profitable laws were stopped. Diamonds
13. (b) aop qop bop = traders stopped developers.
y
o
u
rs
m
a
h
284 IBPS Bank PO/MT CWE Exam 2013

b
o
o
OR OR

b
.w
o
rd
Pearls gems

p
re
Diamonds Jar

s
s
.c
Stone Corals Chair

o
m
Tables
I. False ù Bottles
Conclusion II. True úú or Bucket
III.False úû

Hence, only conclusions II and either I or III follow.


17. (a) According to question,
Conclusions, I. P, II. P, III. P
Hence, All I, II and III follow.

Flats 19. (b) 12 m N


P Q

Apartments Building 6m
U T 8m W E
Bungalow
6m
Gardens
S 6m R S

Therefore, three points Q, T and R, would form a triangle


Or
whose all the angles are less than 90°
20-25 :

e)
Flats inanc
B (F
C (Corporate finance)

Apartments Building A
(Investment
E (Marketing)
Bungalow Banking)
Gardens

D F(HR)
Conclusions I. P, II. X, III. X (Sales) G
(Operations)
Hence, only conclusion I follows.
18. (c) According to question, 20. (a) B works for Finance department.
21. (d) Second to the left of D.
22. (d) F, who works for HR department.
23. (b) E, who works for Marketing department.
24. (c) C and B are sit between them.
Chair 25. (a) Only one i.e. D
26-30 :
Tables Jar Bucket
«Þ< # Þ> @ Þ³
Bottles % Þ£ $ Þ=
y
o
u
rs
m
a
h
IBPS Bank PO/MT CWE Exam 2013 285

b
o
o
26. (a) V$Y Þ V = Y 31. (c) As,

b
.w
Y@ Z Þ Y ³ Z P R O V I D E

o
Z%XÞZ<X

rd
X #T Þ X > T –3 +3 –3 +3 –3 +3 –3

p
re
From all above statements,

s
s
V =Y ³ Z£ X > T

.c
M U L Y F G B
Conclusions I. T # Z Þ T > Z (False)

o
m
II. X # Y Þ X > Y (False) B E C A U S E
III. Z « Y Þ Z < Y (False)
None follows. –3 +3 –3 +3 –3 +3 –3
27. (a) R@JÞR³J Similarly,
J%FÞJ£F Y H Z D R V B
F«EÞF<M
32. (e) From statements (I) and (II)
E% MÞE<M
According to weight, the position of Sahil is 5th.
From all above statements, R ³ J £ F < E £ M
Conclusions I: M # J Þ M > J (True) According to weight, the position of Ramesh is 5th.
II. F % M Þ F £ M (False) So, the number of children in group = 5 + 15 = 20
III. M « R Þ M < R (False) So, both statements are required to given the answer.
Only (I) follows. 33. (d) From statement I,
28. (b) H# RÞH>R eat healthy food ® Ka ma re
R@ LÞR³L From statement II.
L«WÞL<W food for healthy people ® ta ma jo re
W%FÞW£F So, from both statements, code of ‘healthy’ can not be
From all above statements, H > R > L < W £ F determined exactly but it may be ‘ma’ or ‘re’.
Conclusions I. H # L Þ H > L (True)
34. (c) From statement I,
II. F # L Þ F > L (True) T
III. H $ F Þ H = F (False)
Son
29. (e) M#JÞM>F
M$FÞM=K Sister
H K
F%QÞF£Q
Q«HÞQ<H Brother
From all above statements, So, it is clear from statement I that K is the only brother
of H.
K<M=F£Q<H
From Statement II.
Conclusions. I. H # K Þ H > K(True)
II. Q # K Þ Q > K (True) T
III. Q @ M Þ Q ³ M (True)
So, all follow. Mother Son
30. (e) D «QÞD<Q
K H
Q$LÞQ=L Brother
L# TÞL>T So, from statement (II), K is the only brother of H. So,
T% HÞT £H either I or II is sufficient.
From all above statemens, 35. (a) From statement (I). W > J > (Q, R, T)
D< Q= L> T £H So, W reached office first.
From statement (II), (J, T) > W > Q > R
Conclusions. I. D « L Þ D < L (True)
It is not clear from statement (II) that who reached the
II. L @ H Þ L ³ H (False) office first either J or T.
III. H # L Þ H > L (False) So, for giving the answer. Statement I is sufficient but
Only (I) and either (II) or (III) follow. statement II is not sufficient.
y
o
u
rs
m
a
h
286 IBPS Bank PO/MT CWE Exam 2013

b
o
o
48. (c) Step I 85 journey train 36 54 daily 28 mansion

b
J N Step II 85 train journey 36 54 daily 28 mansion

.w
F N-W
36. (c) Step III 85 train 54 journey 36 daily 28 mansion

o
rd
Step IV 85 train 54 mansion journey 36 daily 28

p
W E Step V 85 train 54 mansion 36 journey daily 28

re
K 49. (e) Step II 63 sour 18 56 grapes healthy 32 rise

s
s
Step III 63 sour 56 18 grapes healthy 32 rise

.c
o
S Step IV 63 sour 56 rise 18 grapes healthy 32

m
Step V 63 sour 56 rise 32 18 grapes healthy
From statement I, village F is in North-West direction of Step VI 63 sour 56 rise 32 healthy 18 grapes
village K. So, the last step is step VI.
From statement II, 50. (b) As solved in Qs. 49. So, 56 is the sixth number from the
right in step V.
N 51. (c) 52. (e) 53. (d) 54. (a) 55. (e)
N-W 56. (a) 57. (c) 58. (c) 59. (e) 60. (d)
F
61. (b) 62. (a) 63. (c) 64. (b) 65. (c)
W E 66. (c) 67. (e) 68. (e) 69. (b) 70. (d)
71. (b) 72. (d) 73. (c) 74. (e) 75. (d)
76. (a) 77. (b) 78. (e) 79. (a) 80. (c)
R K 81. (c) 82. (b) 83. (a) 84. (e) 85. (d)
S 86. (d) 87. (b) 88. (e) 89. (a)
From statement II, village F is in North-West direction 90. (d)
91. (b) From I and II, 10 women can finish the work in 1 day
of village K.
So, either I or II is sufficient to answer the question. 7 1 7-4 1
= - = =
37. (b) The exposed articles by the publication must be verified. 24 6 24 8
\ 10 women can finish the work in 8 days.
38. (c) Thus the increase in crime rate has been contributed by
From II and III,
other factors, not leniency in the punishment. Let 10 men can finish the work in x days and 10 women
39. (e) None of the conclusion can be inferred from the above can finish the same work in y days.
statement. So, option (e) is the right choice. 1 1 7
40. (d) Most logically such comparison should reveal mortality Hence, + = ...(i)
x y 24
rate per thousand doctors indulged in SARS treatment
and not indulged in treatment. 3 4
Sol. (41 - 44) and from III – II, + =1 ...(ii)
x y
Mridul Abhishek Ranjan Salil Deepak Pritam from (i) & (ii)
Place Chennai Ahmedabad Delhi Bengaluru Kolkata Mumbai y = 8 days
Month November July April September February December 3 4
Again from I and III + = 1 Þ y = 8 days
Brides Ipsita Veena Hema Jasmine Geetika Brinda 6 y
41. (c)
Ranjan is Hema’s Husband. 92. (a) From I, Let present age of Sabir be x yr and age of his
42. (c)
Deepak’s wedding took place in Kolkata. father be 2x yr.
43. (d)
It’s in the month of December.
x+5 6
44. (a)
Salil’s wedding was held in Bengaluru. From I and II, = Þ 12x + 30 = 11x + 55.
45. (d)
Input cannot be determined. 2x + 5 11
46. (a)
Input: Jockey firm 36 43 growth chart 22 45 x = 25 yr
Step I: 45 Jockey firm 36 43 growth chart 22 From I and II, age of Sabir = 25 yrs.
Step II: 45 Jockey 43 firm 36 growth chart 22 Hence, only from I & II, age of Sabir and his father can
Step III: 45 Jockey 43 growth firm 36 chart 22. be obtained.
47. (b) Step II 53 window 42 50 door lock key 36 93. (e) Let two digit number be 10x + y.
Step III 53 window 50 42 door lock key 36
27 27
Step IV 53 window 50 lock 42 door lock key 36 From I, either x – y = =3Þ y–x = =3
Step V 53 window 50 lock 42 door lock key 36 9 9
Step VI 53 window 50 lock 42 door lock key 36 From II, x – y = 3 Þ y – x = 3
the step VI is the last step of the arrangement. From III, x – y = 3
Therefore four more steps are required to complete Hence, Even by (I) + (II) + (III) we cannot obtain the
arrangement. number.
y
o
u
rs
m
a
h
IBPS Bank PO/MT CWE Exam 2013 287

b
o
\ Distance covered by helicopter in 18 h

o
94. (e) From I, If P = 100

b
A = 200 and SI = 200 – 100 = 100 = 283.5 × 18 = 5103 km

.w
100. (b) Required number of people = 21% of 29565

o
SI ´100 100 ´100

rd
Rate = = = 20% = 6208.65 = 6210
P´T 100 ´ 5

p
101. (a) Required difference = 9% of 26345 – 8% of 27456

re
= 2371.05 – 2196.48 = 174.57 = 175

s
400 ´ 100

s
From II and III, Rate = = 20% 102. (c) Number of defaulters of Bank R in the year

.c
2000 ´ 1 2004 = 15% of 25467 = 3820.05

o
m
Hence, either I alone or II + III will be sufficient. 2005 = 17% of 32461 = 5518.37
95. (e) From I and II. 2006 = 16% of 32652 = 5224.32
Length = 3x = 48 m 2007 = 18% of 32561 = 5860.98
\ x = 16 2008 = 13% of 25495 = 3314.35
Breadth = 2x = 32 m 2009 = 11% of 27649 = 3041.39
Hence, Area of floor = 48 × 32 2010 = 12% of 28283 = 3393.96
Cost of flooring = 48 × 32 × 850 = ` 1305600 Hence, maximum number of defaulters of Bank R is in
From I and III, 2(l + b) = 160 the year 2007.
Þ 2(3x + 2x) = 160 Þ 10x = 160 103. (a) Difference of number of people taking loan from Bank P
\ x = 16 from the previous year in the year
\ Length = 3 × 16 = 48 m 2005 = 32081 – 27361 = 4720
2006 = 32081 – 27361 = 6720
Breadth = 2 × 16 = 32 m
2007 = 25361 – 23654 = 1707
Cost of flooring = (48 × 32) × 850 = ` 1305600
2008 = 36125 – 35465 = 12471
Similarly, from II and III, we can find I = 48 m and b = 32 m
2009 = 36125 – 35465 = 660
and Total cost of flooring = ` 1305600
2010 = 35465 – 34135 = 1330
x Hence, the year is 2008.
96. (e) Let fraction be . 104. (c) Required number of defaulters
y
= 19% of 36152 + 18% of 35463
x ´ 120% 3 = 6868.88 + 6383.34 = 13252.22 » 13250
\ According to the question, y ´125% = 5
+8 +8
x 3 125 5 45
Þ = ´ =
y 5 120 8 105. (e) 29 37 21 43 13 53 5
97. (b) Let one’s digit = x
ten’s digit = 2x –8 –8 –8
Number = 10 (2x) + x = 21 x
After interchange the digit number = 12 x 11
\ 21x – 12x = 27 600 125 30 13 7.2 6.44 6.288
9x = 27 106. (e)
x= 3 ¸5+5 ¸5+5 ¸5+5 ¸5+5 ¸5+5 ¸5+5
\ one’s digit = 3
Let’s digit = 2 × 3 = 6 23
Number = 10 × 6 + 3 = 63 80 42 24 13.5 8.75 6.375 5.1875
98. (b) Let the adjacent angles of the parallelogram be 4x and 107. (c)
5x.
¸2+2 ¸2+2 ¸2+2 ¸2+2 ¸2+2 ¸2+2
\ 4x + 5x = 180 or 9x = 180 \ x = 20
One angle of quadrilateral = 3 × 80° = 240° 669
Again, sum of angles of quadrilateral 10 8 13 35 135 671 4007
4y + 11y + 9y + 240° = 360° 108. (b)
24y = 120° \ y = 5
Hence, the sum of the largest and the smallest angles of × 1 – 2 × 2 – 3× 3 – 4 × 4 – 5 × 5 – 6 × 6 – 7
the quadrilateral = 4 × 5 + 240 = 260° 1090
99. (d) Distance covered by the aeroplane in 9 h
= 9 × 756 = 6804 km 150 290 560 1120 2140 4230 8400
109. (c)
2 ´ 6804
Speed of helicopter = = 283.5 km/h × 2 – 10× 2 – 20 × 2 – 30× 2 – 40 × 2 – 50× 2 – 60
48
y
o
u
rs
m
a
h
288 IBPS Bank PO/MT CWE Exam 2013

b
o
o
110. (a) Required ratio = [45000 × (8 + 9)%] : [45000 × (15 + 18)%] 117. (a) I. 35x + 70 = 0

b
= 17 : 33.

.w
-70
\x= = -2

o
9000 ´ 7% 35

rd
111. (e) Qualified aspirants from C = ´ 100 = 17.5%
45000 ´ 8% II. (81)1/4 y + (343)1/3 = 0

p
re
Þ 3y + 7 = 0 Þ 3y = – 7
9000 ´ 21%

s
´ 100 = 24.70%

s
From D =

.c
45000 ´ 17% 7
\ y= – = – 233 \ x > y

o
3

m
9000 ´ 14%
From E = ´ 100 = 31.11%
45000 ´ 9% ( 2 )5 + (11)3
118. (a) I. = x3
9000 ´ 11% 6
From F = ´ 100 = 12.22%
45000 ´ 18% 32 + 1331 1363
Þ = x3 Þ = x3
9000 ´ 13% 6 6
From G = ´ 100 = 11.81%
45000 ´ 22% \ x3 = 227.167
Per cent is least in G. -589 589
112. (d) Required difference = 9000 × (21% – 13%) II. 4y3 = + 5y 3 Þ = y3
4 4
8 \ y3 = 147.25 \ x > y
= 9000 ´ = 720
100 119. (d) I. x7/5 ¸ 9 = 169 ¹ x3/5
9000 ´ (16 + 7 ) ´100 x 7 / 5 169
113. (b) Required per cent = = 24.21% =
45000 ´ (11 + 8 ) 9 x3 / 5
114. (c) Number of candidates qualified from State (B + D) Þ x 10/5 = 9 × 169 Þ x2 = 9 × 169
(16 + 21) x = ± (3 ´ 13) = ±39
= 9000 × = 90 × 37 = 3330
100
273
Number of candidates appeared from states II. y1/4 × y1/4 × 7 =
y1/ 2
8
= 45000 × = 3600
100 273
y= = 39
3330 7
Required ratio = = 37 : 40 x£y
3600
115. (b) I. 12x2 + 11x + 12 = 10x2 + 22x 120. (a) From table we can say that expenditure on water supply
2x2 – 11x + 12 = 0 and sanitation are increasing in every plan. So, the graph
2x2 – 8x – 3x + 12 = 0 represent in option (a) is best explain of it.
(x – 4) (2x – 3) = 0 121. (b) By watching table, we see that the ratio of public sector
x = 4, x = 3/2 expenditure to the expenditure on social service was
II. 13y2 – 18y + 3 = 9y2 – 10y highest in the VIth plan.
4y2 – 8y + 3 = 0 122. (a) From table, we can say that in all the sector, there is no
4y2 – 6y – 2y + 3 = 0 continuous decrease.
(2y – 3) (2y – 1) = 0 24880
123. (d) Required percentage = ´ 100%
3 1 975000
y= , = 2.551% = 2.5%
2 2 124. (c) Total expenditure on education in all the plans
\x³y = 1530 + 2730 + 5890 + 7860 + 13360 + 25240
18 6 12 8 = 56610 million
116. (c) 2
+ - 2 = 2
x x x x Total expenditure on health in all the plans
= 980 + 2140 + 2260 + 3370 + 7610 + 18210 = 34570 million
18 + 6x - 12 8
Þ = Þ 6x + 6 = 8 \ Required difference = (56610 – 34570)
2
x x2 = 22040 million = ` 22040 × 1000000
2 = ` 22040000000
\ x= = 0.33
6 125. (d) Total age of son and mother
II. y3 + 9.68 + 5.64 = 16.95 2x + 7x = 2 × 27
Þ y3 = 16.95 – 15.32 9x = 54
x= 6
Þ y3 = 1.63 = y = 3 1.63
y
o
u
rs
m
a
h
IBPS Bank PO/MT CWE Exam 2013 289

b
o
\ Mother’s age after 7 yr = 7x + 7 = 7 × 6 + 7 = 49 yr

o
224 7 ´ 32

b
126. (b) Rohan’s marks = 75 \ Required ratio = = = 32 : 39

.w
273 7 ´ 39
Sonia’s marks = 65

o
132. (d) Passengers in the general coaches of train A = 140

rd
Rohit’s marks = 65 + 45 = 110
Passengers in the AC coaches of train B = 305

p
Raman’s marks = 110 – 25 = 85

re
\ Total = 140 + 305 = 445

s
Ravi got marks = 85 + 34 = 119

s
133. (e) The number of passengers in AC coaches of train

.c
Total maximum marks = 119 + 50 + 169 A = 175

o
m
119 Total number of passengers in sleeper class coaches
Percentage of Ravi’s mark = ´ 100% = 70.4% = 70% and first class coaches together of train B
169
= (273 + 91) = 364
127. (a) 1M = 2W
\ Their difference = 364 – 175 = 189
(8M + 4W) × (6 days – 2 days) = (4M + 8W) × x days 134. (b) Passengers in general class coach of train A and train B
(8 × 2W + 4W) × (6 – 2) days = (140 + 241) = 381
= (4 × 2W + 8W) × x days Total number of passengers in train B = 910
(16 + 4)W × 4 days = 16W × x days
381
\ Required percentage = ´ 100% = 41.8% » 42%
20 ´ 4 910
\ x= = 5 days [M1D1 = M2D2]
16 135. (c) The cost of per ticket of first class coach = ` 450
128. (b) Let total monthly income of Mr. Giridhar be ` x. Number of Passenger in first class coaches of Train
According to question, A = 224.
Total amount = 450 × 224 = ` 100800
50 15 136. (e) Number of men working in the marketing department
\ x´ ´ = 900
100 100
79.2° 3
x = ` 12000 3250 ´ ´ = 429
360° 5
Hence, monthly income of Mr. Giridhar = ` 12000.
3300 36° 13
129. (a) Circumference of circular plot = = 220 3250 ´ ´
15 360° 25 = 13 : 28
137. (c) \ Required ratio = 57.6° 7
Þ 2pr = 220 3250 ´ ´
360° 10
220 55 ´ 7
\ r= ´7 = = 35 m 138. (e) Number of men working in the production department
2 ´ 22 11
136.8° 4
Total cost of flooring the plot = pr2 × 100 = 3250 ´ ´ = 988
360° 5
22 Total number of employees in production department
= ´ 35 ´ 35 ´ 100 = ` 385000
7
136.8°
3250 ´ = 1235
SI ´100 6500 ´ 100 360°
130. (a) P = = = 6250
R ´T 8 ´ 13
988
2 Required percentage = ´ 100% = 80%
æ 8 ö 1235
CI = 6250 ç 1 + ÷ – 6250 = ` 1040 139. (b) Number of women working in IT department
è 100 ø
Sol. (Q. Nos. 131-135) 57.6° 3
= 3250 × ´ = 156
Number of Passengers in train A = 700 360° 10
Number of Passenger in train B = 30% of 700 = 910 Total number of employees = 3250
First 156
Coaches General Sleeper AC Total Required percentage ´ 100% = 4.8%
Class 3250
Train A 140 161 224 175 700 140. (b) Number of men working in accounts department
Train B 241 273 91 305 910 50.4° 6
= 3250 ´ ´ = 210
Total 381 434 315 480 1610 360° 13
Number of men working in marketing department
131. (c) Number of passengers in first class coaches of train A =
224 Number of passengers in sleeper class coaches of 79.2° 3
= 3250 ´ ´ = 429
train B = 273 360° 5
y
o
u
rs
m
a
h
290 IBPS Bank PO/MT CWE Exam 2013

b
o
o
Number of men working in IT department 154. (a) 155. (b) 156. (d) 157. (d)

b
.w
57.6° 7 158. (d) The rural financial system in the country calls for a strong
= 3250 ´ ´ = 364

o
360° 10 and efficient credit delivery system, capable of taking

rd
care of the expanding and diverse credit needs of

p
Number of men working in HR department

re
agriculture and rural development. More than 50% of

s
36° 12

s
= 3250 ´ ´ = 156 the rural credit is disbursed by the Co-operative Banks

.c
360° 25

o
and Regional Rural Banks. NABARD is responsible for

m
Number of men working in production department
regulating and supervising the functions of Co-operative
136.8° 4 banks and RRBs. In this direction NABARD has been
= 3250 ´ ´ = 988
360° 5 taking various initiatives in association with Government
Hence, total number of men working in the organization of India and RBI to improve the health of Co-operative
= 210 + 429 + 364 + 156 + 988 = 2147 banks and Regional Rural Banks.
141. (a) An IDR is a receipt, declaring ownership of shares of a 159. (a) 160. (b)
foreign company. These receipts can be listed in India 161. (d) Microcredit is the term used to identify small loans that
and traded in rupees. Just like overseas investors in the are made to individuals and entities that would
US-listed American Depository Receipts (ADRs) of otherwise not be able to obtain any type of credit. The
Infosys and Wipro get receipts against ownership of main function of microcredit is to provide financial
shares held by an Indian custodian, an IDR is proof of services to those poor who do not qualify for standard
ownership of foreign company's shares. The IDRs are sources of credit and assist them in achieving a better
denominated in Indian currency and are issued by a quality of life.
domestic depository and the underlying equity shares 162. (e) 163. (a) 164. (d) 165. (b)
are secured with a custodian. 166. (b) The World Health Organization (WHO) is a specialized
142. (d) 143. (c) 144. (b) 145. (c) agency of the United Nations (UN) that is concerned
146. (e) The UIDAI has set a target of enrolment for 60 crore with international public health. It was established on 7
people for Aadhaar Numbers till the end of this year. April 1948, headquartered in Geneva, Switzerland. WHO
The government has already started Aadhaar based is a member of the United Nations Development Group.
direct benefit transfer (DBT) scheme in 43 districts 167. (a) 168. (d) 169. (c) 170. (a) 171. (e)
within 3 months in the country and such type of 172. (c) 173. (a) 174. (d) 175. (e) 176. (b)
implementation will help in reducing irregularities. 177. (c) The population density of Arunachal Pradesh is even
147. (d) 148. (a) less than 20 people per kilometer square of area,
149. (d) In a tribute to the legendary filmmaker Yash Chopra on according to Arunachal census of 2011. Population
the eve of his first death anniversary, melody queen growth rate is more than 25% here, but has shown
Lata Mangeshkar was conferred the first Yash Chopra decrement since the last census.
Memorial Award at a glittering function in Mumbai. 178. (c) 179. (e)
150. (d) Wealth management as an investment-advisory 180. (e) The Government launched Direct Cash Transfer scheme
discipline incorporates financial planning, investment on 1 January 2013 to transfer cash into bank accounts
portfoliomanagement and a number of aggregated of beneficiaries across 20 districts in the country. The
financial services. High-net-worth individuals (HNWIs), money is directly transferred into bank accounts of
small-business owners and families who desire the beneficiaries having Aadhar cards. LPG and kerosene
assistance of a credentialed financial advisory specialist subsidies, pension payments, scholarships and
call upon wealth managers to coordinate retail banking, employment guarantee scheme payments as well as
estate planning, legal resources, tax professionals and benefits under other government welfare programmes
investment management. will be made directly to beneficiaries. The money can
151. (b) 152. (e) then be used to buy services from the market.
153. (e) Indian weightlifter, Deepak Lathore won Gold and two 181. (b) 182. (b) 183. (d) 184. (c) 185. (a)
Silver medals in the Boys' 50 Kg category of the 186. (e) 187. (c) 188. (a) 189. (c) 190. (a)
Commonwealth Weightlifting Championships on 25 191. (c) 192. (a) 193. (b) 194. (a) 195. (c)
November 2013 at Penang, Malaysia. 196. (c) 197. (a) 198. (b) 199. (a) 200. (c)
y
o
u
rs
m
a
h
b
o
o
b
IBPS BANK PO/MT CWE EXAM 2014

.w
o
rd
p
re
Based on Memory

s
s
.c
o
m
REASONING ABILITY 4. Statements
I. All ACs are DCs.
DIRECTIONS (Qs. 1–5): Every question below has a three II. Some DCs are ECs.
statement, followed by four conclusions numbered I, II, III III. All ECs are YYs.
and IV. You have to consider every given statement as true, Conclusions
even if it does not conform to the well known facts. Read the I. Some ACs are ECs.
conclusions and then decide which of the conclusions can be II. Some YYs are DCs.
logically derived. III. No ACs is ECs.
1. Statements IV. All DCs are ACs.
I. Some toys are pens. (a) I and III (b) Only II
II. Some pens are papers. (c) I and II (d) II and either I or III
III. Some papers are black. (e) None of these
Conclusions 5. Statements
I. Some toys are black. I. Some newspapers are radios.
II. No pen is black. II. Some radios are televisions.
III. No toy is black. III. No television is a magazine.
IV. Some pens are black. Conclusions
(a) None follows I. No newspaper is a magazine.
(b) Either II or IV II. No radio is a magazine.
(c) Either I or III and either II or IV III. Some radios are not magazine.
(d) Either I or IV IV. Some newspapers are televisions.
(e) All of the above (a) Only I follows (b) Only III follows
2. Statements (c) Either I or II follows (d) Both I and II follow
I. Some books are copies. (e) None of these
II. All copies are green.
DIRECTIONS (Qs. 6-10): Study the following information
III. Some green are yellow.
carefully and answer the questions based on it.
Conclusions
I. All copies are yellow. Ten students A, B, C, D, E, F, G, H, I and J are sitting in a row facing
II. Some yellow are green. west.
III. Some copies are yellow. (i) B and F are not sitting on either of the edges.
IV. All green are copies. (ii) G is sitting to the left of D and H is sitting to the right
(a) Only II (b) Either III or IV only of J.
(c) Either II or IV only (d) All of these (iii) There are four persons between E and A.
(e) None of these (iv) I is to the right of B and F is to the left of D.
3. Statements (v) J is in between A and D and G is in between E and F.
I. All jugs are glasses. (vi) There are two persons between H and C.
II. All glasses are cups. 6. Who is sitting at the seventh place counting from left?
III. All jugs are pens. (a) H (b) C
Conclusions
(c) J (d) Either H or C
I. All pens are jugs.
II. Some glasses are pens. (e) None of these
III. Some cups are pens. 7. Who among the following is definitely sitting at one of the
IV. All pens are cups. ends?
(a) All follow (b) Only II (a) C (b) H
(c) Only II and III (d) Only III and IV (c) E (d) Cannot be determined
(e) None of these (e) None of these
y
o
u
rs
m
292 IBPS Bank PO/MT CWE Exam 2014

a
h
b
8. Who are immediate neighbours of I? DIRECTIONS (Qs. 16 - 20): This group of questions is based

o
(a) BC (b) BH

o
on a set of conditions. In answering some of the questions, it

b
(c) AH (d) Cannot be determined may be useful to draw a rough diagram. Choose the response

.w
(e) None of these that most accurately and completely answers each question.

o
rd
9. Who is sitting second left of D? A circular field, with inner radius of 10 meters and outer radius

p
(a) G (b) F of 20 meters, was divided into five successive stages for

re
ploughing. The ploughing of each stage was handed over to

s
(c) E (d) J

s
a different farmer.

.c
(e) None of these

o
(i) Farmers are referred to by the following symbols: F1, F2,

m
10. If G and A interchange their positions, then who become F3, F4 and F5.
the immediate neighbours of E? (ii) The points between different stages of project are
(a) G and F (b) Only F referred to by the following symbols: P1, P2, P3, P4, and
(c) Only A (d) J and H P5, not necessarily in the order.
(e) None of these (iii) Farmer F5 was given the work of ploughing stage
starting at point P4.
DIRECTIONS (Qs. 11 - 15): Study the following information
(iv) The stage from point P5 to point P3 was not the first
and answer the questions that follow:-
stage.
Twelve people Abhishek, Binit, Chand, Dhiraj, Eshita, Fatima, (v) Farmer F4 was given the work of the fourth stage.
Garima, Hena, Ishan, Jatin, Kamal and Lalit are sitting around (vi) Stage 3 finished at point P1, and the work of which was
a rectangular table. The following information is known- not given to farmer F1.
The table has 12 chairs numbered from 1 to 12. (vii) Farmer F3 was given work of stage ending at point P5.
6 seats on one side of the table and 6 on the opposite side. 16. Which was the finish point for farmer F2?
The chairs are arranged in such a way that chair number 1 is (a) P1 (b) P2
just opposite to 12, 6 is opposite to 7 and so on- (c) P3 (d) P4
Abhishek is sitting opposite to Kamal who is the only person (e) P5
sitting between Chand and Jatin. Eshita is sitting opposite to 17. Which stage was ploughed by farmer F5?
Ishan who is the only person sitting between Binit and Lalit. (a) First (b) Second
Fatima, sitting at chair number 1, is diagonally opposite to (c) Third (d) Fourth
Chand who is sitting opposite to Dhiraj. (e) Fifth
11. If Garima is sitting opposite to Fatima then who is sitting 18. Which were the starting and finish points of stage 2?
opposite to Hena? (a) P2 and P5 (b) P5 and P3
(a) Lalit (c) P3 and P1 (d) P5 and P4
(b) Binit (e) P3 and P2
(c) Ishan 19. For which farmer was P2 a finishing point?
(d) Uniquely not determined. (a) F1 (b) F2
(e) None of these (c) F3 (d) F4
12. If Lalit is sitting opposite to Hena, then who is sitting (e) F5
opposite to Garima? 20. Which was the starting point for farmer F3?
(a) Eshita or Fatima (b) Jatin or Fatima (a) P2 (b) P3
(c) Jatin or Eshita (d) None of these (c) P4 (d) P1
(e) All of the above (e) None of these
13. How many persons are sitting between Binit and Dhiraj,
if they are on the same side of the table? DIRECTIONS (Qs. 21–25) : In the following question * ,Å
(a) 2 or 3 (b) 1 or 2 , $ , £ and @ are used according to following meaning.
(c) 1 or 3 (d) None of these ‘P*Q’ means, ‘P is neither equal nor smaller than Q’
(e) All of the above ‘PÅQ’ means, ‘P is not smaller than Q’
14. Which one of the following is correct? ‘P$Q’ means, ‘P is neither greater nor smaller than Q’
(a) Lalit is sitting at seat number 12 ‘P£Q’ means, ‘P is neither greater nor equal to Q’
(b) Lalit is sitting at seat number 10 ‘P@Q’ means, ‘P is not greater than Q’
(c) Kamal is sitting at seat number 8 Now according to the folllowing statement if they are true,
(d) Kamal is sitting at seat number 9 judge their Conclusions I, II and III follow definiately true.
(e) None of these 21. Statements
15. Which one of the following is not correct? E@F, OÅF, P@E, P$R
(a) Lalit can be opposite to Jatin. Conclusions
(b) Jatin can be opposite to Hena. I. O $ P II. E Å R III. P £ O
(c) Lalit is adjacent to Chand. (a) Only I is true (b) Only II is true
(d) There are three person sitting between A and F. (c) Either I or II is true (d) Only III is true
(e) None of these (e) None of these
y
o
u
rs
m
IBPS Bank PO/MT CWE Exam 2014 293

a
h
b
22. Statements II. R is the third in height in the ascending order and not

o
A*B, B@C, A$D, D£E as tall as U, P and Q, Q being taller than P but not the

o
b
Conclusions tallest.

.w
I. E * B II. C $ A III. D @ E 29. Who is paternal uncle of P?

o
rd
(a) Only I is true (b) I and II are true
I. P is brother of L, who is daughter of Q, who is sister of

p
(c) Only III is true (d) No one is true
N, who is brother of S.

re
(e) None of these

s
II. M is brother of K, who is husband of L, who is mother

s
23. Statements

.c
of G, who is sister of P.
IÅH, H$T, S£T, S@R

o
30. What is Sudin’s rank in the class of 44 students?

m
Conclusions
I. Ramesh, whose rank is 17th in the class, is ahead of
I. I * T II. I $ T III. S * H
Shyam by 6 ranks, Shyam being 7 ranks ahead of Sudin.
(a) All are true (b) Either I or II is true
II. Suketu is 26 ranks ahead of Sudin and Shyamala is 6 ranks
(c) Only I is true (d) Only II is true
behind Sudin while Savita stands exactly in the middle of
(e) None of these
24. Statements Shyamala and Suketu in ranks, her rank being 17.
S@T, Q$N, T£N, Q*O DIRECTIONS (Qs.31-34) : Study the following information
Conclusions carefully and answer questions that follow :
I. S $ N II. N Å O III. N * O
(a) None is true (b) Either I or III is true The convenience of online shopping is what I like best about
(c) Only I is true (d) Only II is true it. Where else can you shop even at midnight wearing your night
(e) None of these suit? You do not have to wait in a line till the shop assistant is
25. Statements ready to help you with your purchases. It is much better experience
as compared to going to a retail store. – A consumer’s view.
HÅJ, J*K, L$K, K@M
Conclusions 31. Which of the following can be a strong argument in favour
I. K £ M II. L $ J III. H Å L of retail store owners?
(a) I and III are true (b) Only II is true (a) Online shopping portals offer a great deal of discounts
(c) Only III is true (d) None is true which retail stores offer only during the sale season.
(e) None of these (b) One can compare a variety of products online which
cannot be done at retail stores.
DIRECTIONS (Qs. 26-30) : Each of the questions below consists (c) Many online shopping portals offer the ‘cash on
of a questions and two statements numbered I and II given delivery’ feature which is for those who are sceptical
below it. Use the data given to decide whether the data provided about online payments.
in the statements are sufficient to answer the question.
(d) Many consumers prefer shopping at retail stores which
Read both the statements and Give answer are nearer to their houses.
(a) if the data in statement I alone are sufficient to answer
the question, while the data in statement II alone are (e) In online shopping the customer may be deceived as
not sufficient to answer the question. he cannot touch the product he is paying for.
(b) if the data in statement II alone are sufficient to answer 32. Which of the following can be inferred from the given
the question, while the data in statement I alone are information? (An inference is something that is not directly
not sufficient to answer the question. stated but can be inferred from the given information)
(c) if the data either in statement I alone or in statement II (a) One can shop online only at night.
alone are sufficient to answer the question.
(d) if the data in both the statements I and II together are (b) Those who are not comfortable using computers can
not sufficient to answer the question. never enjoy the experience of online shopping.
(e) if the data in both the statements I and II together are (c) All retail stores provide shopping assistants to each
necessary to answer the question. and every customer.
26. Among M, K, B, D and W, who is the youngest? (d) The consumer whose view is presented has shopped
I. B is younger than D. at retailr stores as well as online.
II. W is younger than K but older than M. (e) The consumer whose view is presented does not have
27. What does ‘Ne’ stands for in the code language? any retail stores in her vicinity.
I. ‘Na Ni Nok Ne’ means ‘I will tell you’ and ‘Ni Nok Ne 33. Read the following information carefully and answer the
Nam’ means ‘he will tell you’ in that code language. given question:
II. ‘Ni Ne Mo Nam’ means ‘will he call you’ and ‘Ne Mok Many manufacturing companies are now shifting base to
Sac Ni’ means ‘how will you go’ in that code language. the rural areas of the country as there is a scarcity of space
28. Who amongst P, Q, R, S, T and U is the tallest? in urban areas. Analysts say that this shift will not have a
I. P is taller than R and T but not as tall as U, who is taller huge impact on the prices of the products manufacutured
than Q and S. by these companies as only about 30% consumers live in
urban areas.
y
o
u
rs
m
294 IBPS Bank PO/MT CWE Exam 2014

a
h
b
Which of the following may be consequence of the given DIRECTIONS (Qs. 39 - 44) : Study the following information

o
information?

o
carefully and answer the given questions :

b
(a) The prices of such products will decrease drastically

.w
A word and number arrangement machine when given an
in the urban areas.

o
input line of words and numbers rearranges them following a
(b) People living in urban areas will not be allowed to

rd
particular rule in each step. The following is an illustration of
work in such manufacturing companies.

p
input and rearrangement:

re
(c) These manufacturing companies has set-ups in the

s
urban areas before shifting base. Input : exam 81 56 over down up 16 64

s
.c
(d) Those who had already migrated to the urban areas Step I : down exam 81 56 over up 16 64

o
Step II : down 81 exam 56 over up 16 64

m
will not shift back to rural areas.
(e) The number of people migrated from rural to urban Step III : down 81exam 64 56 over up 16
areas in search of jobs may reduce. Step IV : down 81 exam 64 over 56 up 16
34. Read the following information carefully and answer the Step IV is the last step of the rearrangement of the above
given question: input.
‘Pets are not allowed in the park premises’ - A notice put up As per the rule followed in the above steps, answer the
at the park entrance by the authority that is responsible for following questions.
maintenance of the park. 39. Input : 98 11 64 22 but will an it
Which of the following can be an assumption according to which of the following will be step VI?
the given information? (An assumption is something that (a) step VI can’t be possible because step V will be the
is supposed or taken for granted) last step
(a) At least some people who visit the park have pets. (b) an 98 but 64 it 22 11 will
(b) This is the only park which does not allow pets (c) an 98 but 64 it 22 will 11
(c) People who ignored this notice were fined (d) an 11 but 22 it 64 will 98
(d) There are more than one entrances to the park (e) None of these
(e) Many people have now stopped visiting the park 40. Input: 32 now 20 gift 53 box 62 at
35. In a code language 'PROVIDE' is written as 'MULYFGB', Which of the following will be step IV?
then what will be code for 'BECAUSE' in same languages (a) at 62 box 53 32 now 20 gift
(a) YZHDRVB (b) ZHYDRVB (b) at 62 box 53 gift 32 now 20
(c) YHZDRVB (d) ZYDHVBR (c) at 62 box 53 gift 20 now 32
(e) None of these (d) at 62 53 box 32 now 20 gift
DIRECTIONS (Qs. 36-38) : Study the following information (e) None of these
carefully and answer the questions that follow : 41. Input: pay by 18 36 nose ear 72 54
Which of the following steps will be the last step?
In a building there are thirteen flats on three floors— II, III and IV. (a) Can’t say (b) Five
Five flats are unoccupied. Three managers, two teachers, two (c) Seven (d) Six
lawyers and one doctor occupy the remaining flats. There are at (e) None of these
least three flats on any floor and not more than six flats on any
42. Step III of an input is:
floor. No two persons of the same profession stay on any floor.
damn 96 flag 87 78 14 saint put
On the second floor, out of four flats, one occupant is the lawyer
which of the following steps will be the last but one?
and has only one neighbour. One teacher lives one floor below
the other teacher. The doctor is not the neighbour of any of the (a) Can’t say (b) Four
lawyers. No flat is unoccupied on the third floor. (c) Five (d) Six
(e) None of these
36. How many flats are there on the third floor?
43. Step II of an input is :
(a) Three or Four (b) Four
jug 99 wax sun top 15 31 47
(c) Five (d) Three
which of the following is definitely the input?
(e) None of these
(a) wax sun top 15 31 47 jug 99
37. What is the combination of occupants on the second floor?
(b) wax sun jug 99 top 15 31 47
(a) Lawyer, Manager (b) Teacher, Doctor
(c) wax sun top jug 99 15 31 47
(c) Manager, Doctor (d) Manager, Teacher (d) Cannot be determined
(e) None of these (e) None of these
38. Who among the following is the neighbour of the other 44. Step IV of an input is: Come 95 forward 40 sky 17 over 23.
lawyer? Then which of the following can certainly not be step III?
(a) Manager (a) come 95 forward sky 17 over 23 40
(b) Teacher (b) come 95 forward 17 sky over 23 40
(c) Both the Manager and the Teacher (c) come 95 forward sky 40 17 over 23
(d) Data inadequate (d) Cannot be determined
(e) None of these (e) None of these
y
o
u
rs
m
IBPS Bank PO/MT CWE Exam 2014 295

a
h
(a) RMNBSFEJ (b) BNMRSFEJ

b
DIRECTIONS (Qs. 45 to 48): Study the following information

o
(c) RMNBJEFS (d) TOPDQDCH

o
carefully and answer the questions that follow:

b
(e) None of these

.w
Mr Ghosh recently redecorated his house by coordinating
QUANTITATIVE APTITUDE

o
orange and three other colours for the walls, carpets and

rd
curtains of four different rooms. From the information below,

p
51. Three containers A, B and C are having mixtures of milk

re
determine the colours of the carpet, walls and curtains for
and water in the ratio 1 : 5, 3 : 5 and 5 : 7, respectively. If the

s
each of the room and answer the following questions:

s
.c
(a) Yellow was the only colour used in all the four rooms. capacities of the containers are in the ratio 5 : 4 : 5, then find

o
the ratio of the milk to the water if the mixtures of all the
It was used at least once for walls, carpets and

m
three containers are mixed together.
curtains. (a) 51 : 115 (b) 52 : 115
(b) Three different colours were used in each room but (c) 53 : 115 (d) 54 : 115
only the dining room and the bedroom were decorated (e) None of these
in the same three colours. 52. Groundnut oil is now being sold at ` 27 per kg. During last
(c) The same colour was chosen for the curtains in the month its cost was ` 24 per kg. Find by how much % a
bedroom, the carpet in the living room and the walls family should reduce its consumption, so as to keep the
in the dining room. That colour was not used at all in expenditure same.
the study room. 1 1
(d) The only room with both green and grey in its colour (a) 11 % (b) 11 %
9 11
scheme had carpet of the same colour as in the dining 9 1
room. (c) 11 % (d) 9 %
10 10
(e) Grey was the only colour used exactly twice—both
(e) None of these
times for curtains
53. An ice-cream company makes a popular brand of ice-cream
(f) The study room walls were painted the same colour as
in rectangular shaped bar 6 cm long, 5 cm wide and 2 cm
the living room walls.
thick. To cut the cost, the company has decided to reduce
45. Which of the following rooms had orange curtains and the volume of the bar by 20%, the thickness remaining the
green walls? same, but the length and width will be decreased by the
(a) Dining room (b) Living room same percentage amount. The new length L will satisfy :
(c) Bedroom (d) Study (a) 5.5 < L < 6 (b) 5 < L < 5.5
(c) 4.5 < L < 5 (d) 4 < L < 4.5
(e) None of these
(e) None of these
46. Which of the two rooms had green carpets? 54. A sum of ` 725 is lent in the beginning of a year at a certain
(a) Dining room and bedroom rate of interest. After 8 months, a sum of ` 362.50 more is
(b) Study and living room lent but at the rate twice the former. At the end of the year,
(c) Living room and dining room ` 33.50 is earned as interest from both the loans. What was
the original rate of interest?
(d) Study and dining room
(a) 3.6% (b) 4.5%
(e) None of these (c) 5% (d) 3.46%
47. Which room did not use grey colour at all? (e) None of these
(a) Dining room (b) Cannot say 55. The difference between compound interest and simple
interest on a sum for 2 years at 10% per annum, when the
(c) Study (d) Living room
interest is compounded annually is ` 16. If the interest
(e) None of these
were compounded half-yearly, the difference in two interests
48. The dining room had ___ curtains. would be:
(a) Green (b) yellow (a) ` 24.81 (b) ` 26.90
(c) Orange (d) grey (c) ` 31.61 (d) ` 32.40
(e) None of these
(e) None of these
49. In a certain code language 'how many goals scored' is 56. A person lent out a certain sum on simple interest and the
same sum on compound interest at certain rate of interest
written as '5 3 9 7'; 'many more matches' is written as '9
8 2'; and 'he scored five' is written as '1 6 3 '. How is per annum. He noticed that the ratio between the difference
'goals' written in that code language? of compound interest and simple interest of 3 years and
(a) 5 (b) 7 that of 2 years is 25 : 8. The rate of interest per annum is:
(c) 5 or 7 (d) Data inadequate (a) 10% (b) 11%
(e) None of these 1
50. In a certain code TEMPORAL is written as OLDSMBSP. (c) 12% (d) 12 %
2
How is CONSIDER written in that code?
(e) None of these
y
o
u
rs
m
296 IBPS Bank PO/MT CWE Exam 2014

a
h
b
57. A contract is to be completed in 46 days and 117 men were
Cost break up of Transmission

o
set to work, each working 8 hours a day. After 33 days, 4/7

o
b
of the work is completed. How many additional men may be Others 10%

.w
employed so that the work may be completed in time, each Clutch

o
rd
man now working 9 hours a day ? 30%

p
(a) 80 (b) 81 Gear Box 15%

re
s
(c) 82 (d) 83

s
.c
(e) None of these

o
58. Two pipes A and B can fill a cistern in 30 minutes and 40

m
minutes respectively. Both the pipes are opened. Find when
the second pipe B must be turned off so the cistern may
just be full in 10 minutes. Tyre
2 15%
(a) 26 min (b) 25 min
3
Brake
2 2 30%
(c) 40 min (d) 42 min
3 3
Price of Car = `1, 00, 000
(e) None of these
61. What is the cost of Gear Box?
59. A train leaves station X at 5 a.m. and reaches station Y at 9
a.m. Another train leaves station Y at 7 a.m. and reaches (a) ` 9000 (b) ` 6000
station X at 10: 30 a.m. At what time do the two trains cross (c) `3000 (d) `15,000
each other ? (e) None of these
(a) 7 : 36 am (b) 7 : 56 am
62. What percentage of total cost is contributed by the brake?
(c) 8 : 36 am (d) 8 : 56 am
(a) 5.5% (b) 6.6%
(e) None of these
60. A boat running upstream takes 8 hours 48 minutes to cover (c) 6% (d) 5.4%
a certain distance, while it takes 4 hours to cover the same (e) None of these
distance running downstream. What is the ratio between 63. If the price of tyres goes up by 25%, by what amount
the speed of the boat and speed of water current should be the sale price be increased to maintain the
respectively? amount of profit?
(a) 2 : 1 (b) 3 : 2 (a) ` 750 (b) ` 2250
(c) 8 : 3 (d) Cannot be determined
(c) ` 3750 (d) ` 375
(e) None of these (e) None of these
DIRECTIONS (Qs. 61-65) : Study the pie charts given below 64. If transmission cost increases by 20%, by what amount is
and answer the following questions. the profit reduced (total price of car remains same)?
(a) ` 3000 (b) ` 4000
Price Break up of Car (c) ` 6000 (d) Cannot be determined
(e) None of these
Chasis 15% Transmission
65. What % of sale price is contributed by clutch?
20%
(a) 6% (b) 2%
(c) 3% (d) Cannot be determined
Miscellaneous (e) None of these
25%
DIRECTIONS (Qs. 66 - 70) : Read the following information
and answer the questions that follow.
In a huge Jewellery shop, the electric gadgets being used are 17
30%
tubelights of 40 W each, 14 fans of 80 W each, 16 bulbs of 60 W
Engine cost
each, 11 bulbs of 100 W each, 11 AC’s of 2100 W each, 9 laptops
Margin
of 200 W each and 10 TV’s of 120 W each. In a day, tube lights
10%
and TV’s are used for 8 h but 60 W bulbs are used for 7 h and 100
y
o
u
rs
m
IBPS Bank PO/MT CWE Exam 2014 297

a
h
b
W bulbs are used for 9 h whereas laptops and AC's are used for DIRECTIONS (Qs. 76-80): Study the following graph carefully

o
o
5 h and 9 h respectively. However, fans are used for 11 h. to answer these questions.

b
.w
(Note: 1000 W = 1 unit, 1 month = 30 days). Annual dividend offered by two companies over the years

o
66. What is the total electric energy consumed (in units) by Company A

rd
60 W bulbs in the whole month?

p
Company B

re
(a) 432 (b) 576 24

s
22

s
(c) 67.2 (d) 201.6

.c
20

o
(e) None of these 18

m
16
67. Electricity consumed by all fans is what per cent of energy

Dividend
14
consumed by all the laptops? 12
10
(a) 132.2% (b) 136.88% 8
(c) 122.68 (d) 169.62% 6
4
(e) None of these 2
0
68. Out of the following, which type of gadgets consume more
2009 2010 2011 2012 2013 2014 2015
electricity in the shop?
(a) Fans (b) Tubelights 76. Shri Giridhar invested total amount of ` 25000 in 2009 for
one year in the two companies together and got a total
(c) Laptops (d) TV’s
dividend of ` 3340. What was the amount invested in
(e) None of these
Company A?
69. If one electric unit costs ` 2.70 and power (used by AC's)
(a) ` 12000 (b) ` 9000
unit costs ` 3.70, then what money is paid to the electricity
(c) ` 16000 (d) Cannot be determined
department for one month?
(e) None of these
(a) ` 27368 (b) ` 28683
77. Anuja invested ` 35000 in Company B in 2011. After one
(c) ` 78600 (d) ` 2900
year she transferred the entire amount with dividend to
(e) None of these
Company A in 2012 for one year. What amount will be
70. What is the ratio of consumption of electricity in units by received back by Anuja including dividend?
60W and 100 W bulbs in a month?
(a) ` 49980 (b) ` 49000
(a) 5 : 6 (b) 4 : 5
(c) ` 48300 (d) ` 49563.50
(c) 3 : 4 (d) 2 : 3
(e) None of these
(e) None of these
78. An amount of ` 18000 was invested in Company A in 2012.
DIRECTIONS (Qs.71–75): In each of the following questions After one year the same amount was re-invested for one
two equations are given. You have to solve them and give more year. What was the total dividend received at the end
answer accordingly.
of two years?
(a) If x > y (b) If x < y (a) ` 5805 (b) ` 6300
(c) If x = y (d) If x > y
(e) If x < y (c) ` 6480 (d) ` 6840
(e) None of these
71. I. 2x2 + 5x + 1 = x2 + 2x – 1
II. 2y2 – 8y + 1 = – 1 79. Bhushan invested different amounts in Companies A and B
in 2015 in the ratio of 5 : 8. What will be the ratio between
x2 1 the amounts of dividends received from Companies A and
72. I. +x- = 1
2 2 B respectively?
II. 3y2 – 10y + 8 = y2 + 2y – 10 (a) 2 : 3 (b) 5 : 6
73. I. 4x2 – 20x + 19 = 4x – 1
(c) 3 : 4 (d) Cannot be determined
II. 2y2 = 26y + 84
74. I. y2 + y – 1 = 4 – 2y – y2 (e) None of these
80. In the year 2014, Suraj invested ` 56000 in Company B.
x2 3
II. - x = x-3 How much more or less dividend would he have received
2 2
had the amount been invested in Company A?
75. I. 6x2 + 13x = 12 – x
(a) ` 1640 more (b) ` 1640 less
5y (c) ` 1860 less (d) ` 1680
II. 1 + 2y2 = 2y +
6 (e) None of these
y
o
u
rs
m
298 IBPS Bank PO/MT CWE Exam 2014

a
h
b
DIRECTIONS (Qs. 81 - 85) : Study the following table to answer the given questions.

o
o
b
Production (in crore units) of six companies over the years

.w
o
Years

rd
Company Total
2009 2010 2011 2012 2013 2014

p
re
TP 103 150 105 107 110 132 707

s
ZIR 75 80 83 86 90 91 505

s
.c
A VC 300 300 300 360 370 340 1970

o
CTU 275 280 281 280 285 287 1688

m
PEN 25 30 35 40 42 45 217
SIO 85 87 89 91 92 96 540
Total 863 927 893 964 989 991 5627

81. The production of Company AVC in 2000 is approximately 88. In how many different ways can the letters of the word
what per cent of its average production over the given 'LEADING' be arranged in such a way that the vowels
years? always come together?
(a) 300 (b) 110 (a) 360 (b) 480
(c) 136 (d) 118.25 (c) 720 (d) 5040
(e) None of these (e) None of these
82. For SIO, which year was the per cent increase or 89. In a class, there are 15 boys and 10 girls. Three students are
decrease in production from the previous year the selected at random. The probability that 1 girl and 2 boys
highest? are selected, is:
(a) 2013 (b) 2010 21 25
(c) 2014 (d 2012 (a) (b)
46 117
(e) None of these
83. Which company has less average production in the last 1 3
three years compared to that of first three years? (b) (c)
50 25
(a) No company (b) CTU (e) None of these
(c) ZIR (d) TP 90. Gauri went to the stationery and bought things worth `25,
(e) None of these out of which 30 paise went on sales tax on taxable
84. The total production of the six companies in the first two purchases. If the tax rate was 6%, then what was the cost of
given years is what per cent of that of last two given the tax free items?
years? (round off up to two decimal places) (a) `15 (b) `15.70
(a) 87.08 (b) 104.55 (b) `19.70 (d) `20
(c) 90.40 (d) 10.62 (e) None of these
(e) None of these
85. For ZIR, which of the following is the difference between DIRECTIONS(Qs 91-95): In this type of questions, one term in
production in 2013 and that in 2014? the number series is wrong. Find out the wrong term.
(a) 10,00,00,000 (b) 1,00,00,000 91. 93, 309, 434, 498, 521, 533
(c) 10,00,000 (d) 40,00,000 (a) 309 (b) 434
(e) None of these (c) 498 (d) 521
86. When the price of a radio was reduced by 20%, its sale (e) None of these
increased by 80%. What was the net effect on the sale? 92. 46080, 3840, 384, 48, 24, 2, 1
(a) 44% increase (b) 44% decrease (a) 384 (b) 48
(c) 66% increase (d) 75% increase (c) 24 (d) 2
(e) None of these (e) None of these
87. Two sides of a plot measure 32 metres and 24 metres and 93. 5, 27, 61, 122, 213, 340, 509
the angle between them is a perfect right angle. The other (a) 27 (b) 61
two sides measure 25 metres each and the other three (c) 122 (d) 509
angles. (e) None of these
What is the area of the plot? 94. 11, 5, 20, 12, 40, 26, 74, 54
(a) 768 (b) 534 (a) 5 (b) 20
(c) 696.5 (d) 684 (b) 40 (d) 26
(e) None of these (e) None of these
y
o
u
rs
m
IBPS Bank PO/MT CWE Exam 2014 299

a
h
peak in the mid- 1970s. Policymakers and aid workers turned their

b
95. 1, 3, 10, 21, 64, 129, 356, 777

o
attention to the poor’s other pressing needs, such as health

o
(a) 21 (b) 129

b
care and education. Farming got starved of resources and

.w
(b) 10 (d) 356 investment. By 2004, aid directed at agriculture sank to 3.5%

o
(e) None of these and “Agriculture lost its glitter.” Also, as consumers in high-

rd
DIRECTIONS (96-100): What approximate value will come in growth giants such as China and India became wealthier, they

p
began eating more meat, so grain once used for human

re
place of question mark (?) in the following questions (You are
consumption got diverted to beef up livestock.

s
not expected to calculate the exact value).

s
.c
By early 2008, panicked buying by importing countries and

o
96. 9228.789 – 5021.832 + 1496.989 = ? restrictions slapped on grain exports by some big producers

m
(a) 6500 (b) 6000 helped drive prices upto heights not seen for three decades.
(c) 6300 (d) 5700 Making matters worse, land and resources got reallocated to
(e) 5100 produce cash crops such as biofuels and the result was that
voluminous reserves of grain evaporated. Protestsbroke out
97. 1002 ÷ 49 × 99 – 1299 = ? across the emerging world and fierce food riots toppled
(a) 700 (b) 600 governments.
(c) 900 (d) 250 This spurred global leaders into action. This made them aware
(e) 400 that food security is one of the fundamental issues in the world
98. 29.8% of 260 + 60.01% of 510 –103.57 = ? that has to be dealt with in order to maintain administrative and
political stability. This also spurred the U.S. which traditionally
(a) 450 (b) 320
provisioned food aid from American grain surpluses to help
(c) 210 (d) 280 needy nations, to move towards investing in farm sectors
(e) 350 around the globe to boost productivity. This move helped
99. (21.98)2 – (25.02)2 + (13.03)2 = ? countries become more productive for themselves and be in a
(a) 25 (b) 120 better position to feed their own people.
(c) 10 (d) 65 Africa, which missed out on the first Green Revolution due to
poor policy and limited resources, also witnessed a ‘change’.
(e) 140 Swayed by the success of East Asia, the primary poverty-
fighting method favoured by many policymakers in Africa was
2498 ´ 625 to get farmers off their farms and into modern jobs in factories
100. =?
99 and urban centers. But that strategy proved to be highly
insufficient. Income levels in the countryside badly trailed those
(a) 110 (b) 90
in cities while the FAO estimated that the number of poor going
(c) 200 (d) 160 hungry in 2009 reached an all time high at more than one billion.
(e) 125 In India on the other hand, with only 40% of its farmland
ENGLISH LANGUAGE irrigated, entire economic boom currently underway is held
hostage by the unpredictable monsoon. With much of India’s
DIRECTION (Qs. 101-115): Read the following passage farming areas suffering from drought this year, the government
carefully and answer the questions given below it. Certain will have a tough time meeting its economic growth targets. In
words/phrases have been printed in bold to help you locate a report, Goldman Sachs predicted that if this year too receives
them while answering some of the questions. weak rains, it could cause agriculture to contract by 2% this
fiscal year, making the government’s 7% GDP-growth target look
Governments have traditionally equated economic progress with “a bit rich”. Another green revolution is the need of the hour
steel mills and cement factories. While urban centers thrive and and to make it a reality, the global community still has much
city dwellers get rich, hundreds of millions of farmers remain backbreaking farm work to do.
mired in poverty. However, fears of food shortages, a rethinking 101. What is the author’s main objective in writing the passage
of antipoverty priorities and the crushing recession in 2008 (a) Criticising developed countries for not bolstering
arecausing a dramatic shift in world economic policy in favour economic growth in poor nations
of greater support for agriculture. (b) Analysing the disadvantages of the Green Revolution
The last time when the world’s farmers felt such love was in (c) Persuading experts that a strong economy depends
the 1970s. At that time, as food prices spiked, there was real on industrialization and not agriculture
concern that the world was facing a crisis in which the planet (d) Making a case for the international society to
was simply unable to produce enough grain and meat for an engineer a second Green Revolution
expanding population. Governments across the developing (e) Rationalising the faulty agriculture policies of
world and international aid organisations plowed investment into emerging countries
agriculture in the early 1970s, while technological breakthroughs, 102. Which of the following is an adverse impact of the Green
like high-yield strains of important food crops, boosted Revolution ?
production. The result was the Green Revolution and food (a) Unchecked crop yields resulted in large tracts of land
production exploded. becoming barren
But the Green Revolution became a victim of its own success. (b) Withdrawal of fiscal impetus from agriculture to other
Food prices plunged by some 60% by the late 1980s from their sectors
y
o
u
rs
m
300 IBPS Bank PO/MT CWE Exam 2014

a
h
(c) Farmers began soliciting government subsidies for (d) The GDP as targeted by India was never achieved

b
o
their produce because of losses in agriculture

o
b
(d) Farmers rioted as food prices fell so low that they (e) None of these

.w
could not make ends meet 108. What encouraged African policymakers to focus on urban

o
(e) None of these jobs ?

rd
103. What is the author trying to convey through the phrase (a) Misapprehension that it would alleviate poverty as

p
“making the government’s 7% GDP growth target look “a it did in other countries

re
bit rich”? (b) Rural development outstripped urban development in

s
s
(a) India is unlikely to achieve the targeted growth rate

.c
many parts of Africa
(b) Allocation of funds to agriculture has raised India’s

o
(c) Breaking out of protests in the country and the fear

m
chances of having a high GDP that the government would topple
(c) Agricultural growth has artificially inflated India’s (d) Blind imitation of western models of development
GDP and such growth is not real (e) None of these
(d) India is likely to rave one of the highest GDP growth 109. Which of the following had contributed to exorbitant food
rates prices in 2008 ?
(e) A large portion of India’s GDP is contributed by
(A) Hoarding of food stocks by local wholesalers which
agriculture
inadvertently created a food shortage.
104. Which of the following factors was/were responsible for
the neglect of the farming sector after the green (B) Export of foodgrains was reduced by large producers.
revolution? (C) Diverting resources from cultivation of foodgrains to
(A) Steel and cement sectors generated more revenue for that of more profitable crops.
the government as compared to agriculture. (a) None (b) Only (C)
(B) Large scale protests against favouring agriculture at (c) Only (B) (d) All (A), (B) & (C)
the cost of other important sectors such as education (e) Only (B) & (C)
and healthcare. 110. Which of the following is true about the state of
(C) Attention of policy makers and aid organizations was agriculture in India at present ?
diverted from agriculture to other sectors. (A) Of all the sectors, agriculture needs the highest
(a) None (b) Only (C) allocation of funds.
(c) Only (B) & (C) (d) Only (A) & (B) (B) Contribution of agriculture to India’s GDP this year
(e) All (A), (B) & (C) would depend greatly upon the monsoon rains.
105. What prompted leaders throughout the world to take (C) As India is one of the high-growth countries, it has
action to boost the agriculture sector in 2008? surplus food reserves to export to other nations.
(a) Coercive tactics by the U.S. which restricted food aid (a) Only (A) and (C) (b) Only (C)
to poor nations (c) Only (B) (d) Only (B) and (C)
(b) The realization of the link between food security and (e) None of these
political stability
(c) Awareness that performance in agriculture is DIRECTION (Qs. 111- 113 ): Choose the word/group of words
necessary in order to achieve the targeted GDP which is most similar it meaning to the word printed in bold
(d) Reports that high-growth countries like China and as used in the passage.
India were boosting their agriculture sectors to 111. STARVED
capture the international markets (a) Deprived (b) Disadvantaged
(e) Their desire to influence developing nations to slow (c) Hungry (d) Fasting
down their industrial development. (e) Emaciated
106. What motivated the U.S. to focus on investing in
112. SLAPPED
agriculture across the globe ?
(a) Beaten (b) Imposed
(a) To make developing countries become more reliant
on U.S. aid (c) Withdrawn (d) Avoided
(b) To ensure grain surpluses so that the U.S. had no (e) Persuaded
need to import food 113. PLOWED
(c) To make those countries more self sufficient to whom (a) Cultivated (b) Bulldozed
it previously provided food (c) Recovered (d) Instilled
(d) To establish itself in the market before the high- (e) Withdrew
growth giants such as India and China could DIRECTION (Qs. 114 and 115): Choose the word/phrase
establish themselves which is most opposite in meaning to the word printed in bold
(e) None of these as used in the passage.
107. What impact did the economic recession of 2008 have on
agriculture ? 114. PRESSING
(a) Governments equated economic stability with (a) Unpopular (b) Undemanding
industrial development and shifted away from (c) Unobtrusive (d) Unsuitable
agriculture (e) Unimportant
(b) Lack of implementation of several innovative 115. EVAPORATED
agriculture programmes owing to shortage of funds (a) Absorbed (b) Accelerated
(c) It prompted increased investment and interest in (c) Grew (d) Plunged
agriculture (e) Mismanaged
y
o
u
rs
m
IBPS Bank PO/MT CWE Exam 2014 301

a
h
120. P: Finish specialists recommended a chewing gum

b
DIRECTIONS (Qs. 116-120) : The sentences given in each

o
containing xylitol-a natural sweetener present in birch,

o
question, when properly sequenced, form a coherent

b
paragraph. Each sentence is labelled with a letter. Choose maple, corn and straw-to be used several times a day by

.w
the most logical order of the sentences from amongst the young children.

o
given choices so as to form a coherent paragraph. Q : Chewing gum is a new solution that “may work for

rd
parents whose children suffer from chronic ear

p
116. P : In the past, the customised tailoring units were infections.

re
localised to the township or city and catered exclusively R : An experiment was conducted involving three hundred

s
s
to domestic demand. and six children between two and six years.

.c
Q : Traditionally, Indians preferred custom-made clothing S : After Finish studies showed that xylitol is effective in

o
m
and the concept of ready-to-wear is a relatively recent preventing cavities, a team of researchers decided to
one. investigate its effects on a very similar type of bacteria
R : Consumer awareness of styling issues and the which causes ear infections.
convenience afforded by ready-to-wear helped RMG (a) Q R S P (b) P Q R S
industry make small inroads into the domestic market (c) R Q P S (d) Q P S R
in the 1980s. (e) None of these
S : The customised tailoring outfits have always been a DIRECTIONS (Q. 121-125) : Read each sentence to find out
major source of clothing for domestic market. whether there is any error in it. The error, if any, will be in one
(a) Q R S P (b) Q S P R part of the sentence. The number of that part is the answer. If
(c) R S Q P . (d) S Q P R there is no error, the answer is (e). (Ignore errors of punctuation,
(e) None of these if any.)
117. P : Such a system will help to identify and groom
121. The ongoing merger among /the two companies will/
executives for positions of strategists.
Q : Evaluation of performance is more often than not done (a) (b)
for the purpose of reward or punishment for past have an adverse/impact on consumers. No error
performance. (c) (d) (e)
R : They must become an integral part of the executive 122. It is evident that/the banking sector has underwent/
system’ . (a) (b)
S : Even where the evaluation system is for one’s tremendous changes during/the past two decades. No error
promotion to assume higher responsibilities, it rarely (e)
includes terms that are a key for playing the role of 123. According to the consultant/a more detail analysis of/
strategist effectively, e.g., the skills of playing the role (a) (b)
of change agent and creative problem solving. customer needs / and product pricing is required. No error
(a) S Q P R (b) S R Q P (c) (d) (e)
(c) R S Q P (d) Q S R P 124. Over the next five years / the government needs to invest/
(e) None of these (a) (b)
118. P : Participation involves more than the formal sharing of at less 350 billion dollars/in rural infrastructure. No error
decisions. (c) (d) (e)
Q : Through anticipation individuals or organisations
125. The lack of no funds / has resulted in several / delays in
consider trends and make plans, shielding institutions
from trauma of learning by shock. (a) (b)
R : Innovative learning involves both anticipation and launching our / new product in India. No error
participation. (c) (d) (e)
S : It is an attitude characterised by the cooperation, DIRECTIONS (Qs. 126-130) : Read the following passage care-
dialogue and empathy. fully to give the answer.
(a) Q R S P (b) P Q R S
Regular physical activity provides numerous health
(c) R Q P S (d) S P Q R
benefits — from leaner bodies and lower blood pressure to
(e) None of these improved mental health and cognitive functioning. As the school
119. P : Almost a century ago, when the father of the modem physical education programme promotes physical activity and
automobile industry, Henry Ford, sold the first Model T can teach skills as well as form or change behaviour, it holds an
car, he decided that only the best would do for his important key to influencing health and well-being across the life
customers. span. To improve the fitness of students, we need to rethink the
Q : Today, it is committed to delivering the’ finest quality design and delivery of school-based physical education
with over six million vehicles a year in over 200 programme. Adults in the United States think that information
countries across the world. about health is more important for students to learn than contents
R : And for over 90 years, this philosophy has endured in language, arts, mathematics, science, history or any other
in the Ford Motor company. subject. Despite this high ranking, most schools devote minimal
S : Thus a vehicle is ready for the customers only, if it curriculum time to teaching students how to lead healthy lives.
passes the Ford ‘Zero Defect Programme’. Our first step might be to consider ways to increase curriculum
(a) P Q R S (b) P R Q S time devoted to physical education. In addition, schools need to
(c) R S P Q (d) P R S Q thoughtfully analyse the design and delivery of school physical
(e) None of these education programme to ensure that they are engaging,
y
o
u
rs
m
302 IBPS Bank PO/MT CWE Exam 2014

a
h
developmentally appropriate, inclusive and instructionally 132. (a) sources (b) finances

b
o
powerful. (c) funds (d) manpower

o
b
126. According to this passage, regular physical activity is (e) industries

.w
needed to : 133. (a) exhaustive (b) unexploited

o
(a) control one’s blood pressure

rd
(c) abundant (d) indefinite
(b) lose one’s weight

p
(e) unreliable

re
(c) improve one’s cognitive skill 134. (a) upward (b) drastic

s
s
(d) improve one’s physical as well as mental health (c) negligible (d) incredible

.c
(e) None of these

o
(e) sudden

m
127. In order to tone up the physical education programme : 135. (a) diminishes (b) degenerates
(a) it should be made compulsory at school (c) increases (d) succumbs
(b) an assessment of the existing programme should be (e) stabilizes
made
136. (a) plans (b) attempts
(c) a committee should be set up in every school
(c) attention (d) resources
(d) the programme should be reoriented and implemented
(e) strategy
(e) None of these
137. (a) existing (b) glorious
128. According to American, health education is more important
(c) ancient (d) economic
than teaching :
(e) discouraging
(a) social sciences (b) liberal arts
138. (a) pivotal (b) neutral
(c) any subject (d) natural sciences
(c) insignificant (d) enchanted
(e) None of these
129. The author wants the reoriented physical education (e) vicious
programme to be : 139. (a) increasingly (b) always
(a) given minimal curriculum time (c) gradually (d) deliberately
(b) very comprehensive (e) badly
(c) relevant to the modern society 140. (a) enlighten (b) validate
(d) thoughtful (c) negate (d) underestimate
(e) None of these (e) belittle
130. In order to improve the physical education programme, we
should first of all : COMPUTER KNOWLEDGE
(a) allot more time to the teaching and learning of physical 141. In computer, what are the .xls, .doc. are
activity (a) File names (b) Extensions
(b) decide on the number of activities to be taught (c) Viruses (d) Binary files
(c) employ qualified instructors
(e) None of these
(d) increase the teaching load of instructors
142. What is the shortcut key to insert new document?
(e) None of these
(a) ctrl+a (b) ctrl+n
DIRECTIONS (Qs. 131-140) : In the following passage there (c) ctrl+s (d) ctrl+d
are blanks, each of which has been numbered. These numbers (e) None of these
are printed below the passage and against each five words are
suggested, one of which fits the blank appropriately. Find out 143. What is the full form of LAN?
the appropriate word in each case. (a) Local Area Network
Economic backwardness of a region is 131by the co- (b) Land Area Network
existence of unutilized 132 on the one hand, and 133 natural (c) Long Area Network
resources, on the other. Economic development essentially means (d) Line Area Network
a process of 134 change whereby the real per capita income of a (e) None of these
economy 135 over a period of time. Then, a simple but meaningful 144. Hard copy is a term used to describe...?
question arises; what causes economy development ? Or what (a) Printed output
makes a country developed ? This question has absorbed the
136 of scholars of socio-economic change for decades. Going (b) Writing on a hard board
through the 137 history of developed countries like America, (c) Storing information on the hard disk
Russia and Japan, man is essentially found as 138 in the process (d) All of the above
of economic development. Japan, whose economy was 139 (e) None of these
damaged from the ravages of the Second World War, is the clearest 145. The operating system is also called the ______ between
example of our time to 140 kingdom role in economic development. the user and the computer.
131. (a) developed (b) cured (a) interrelate (b) interface
(c) improved (d) enhanced (c) interference (d) intermediate
(e) characterised (e) None of these
y
o
u
rs
m
IBPS Bank PO/MT CWE Exam 2014 303

a
h
146. If a computer is constantly rebooting itself, what is most 156. Documentation of computer program is important so that

b
o
commonly the problem? (a) users can learn how to use the program

o
b
(a) Insufficient Power-Supply Unit (b) other programmers can know how to maintain the

.w
(b) Bad Sectors on Hard Drive program

o
(c) Processor Overheating (c) the programmer can see why the code is written that

rd
(d) Defective/Bad Memory way while hunting for sources of error

p
re
(e) None of these (d) All of the above

s
147. Networks are monitored by security personnel and (e) None of the above

s
.c
supervised by __________ who set(s) up accounts and 157. A program that enables you to perform calculations

o
passwords for authorized network users. involving rows and columns of numbers is called a

m
(a) IT managers __________.
(b) the government (a) spreadsheet program
(c) network administrators (b) word processor
(d) password administrators (c) graphics package
(e) None of these (d) window
148. What is a benefit of networking your computer with other (e) None of the above
computers? 158. What does the data dictionary identify?
(a) Increase in the computer’s speed (a) Field names (b) Field types
(b) Sharing of cables to cut down on expenses and clutter
(c) Field formates (d) All of the above
(c) You have another computer if yours has a breakdown
(e) None of these
(d) Increase in the speed of the network
159. Which is one function of a database management system
(e) Sharing of resources to cut down on the amount of
equipment needed (DBMS)?
149. In a synchronous modem, the digital-to-analog converter (a) Ensuring usability
transmits signal to the___________. (b) Identifying what a user needs
(a) equalizer (b) modulator (c) Deciding what to do with legacy systems
(c) demodulator (d) terminal (d) Preventing errors arising, while enabling multiple,
(e) None of these simultaneous users
150. The term “host” with respect to the internet, means (e) None of these
__________. 160. A relation (from the relational database model) consists of
(a) A computer that is a stand along computer a set of tuples, which implies that
(b) A computer that is connected to the Internet (a) relational model supports multi-valued attributes
(c) A computer reserved for use by the host whose values can be represented in sets.
(d) A large collection of computers (b) for any two tuples, the values associated with all of
(e) Hyperlink their attributes may be the same.
151. Which of the following operations is safe if an e-mail from (c) for any two tuples, the value associated with one or
an unknown sender is received? more of their attributes must differ.
(a) Open it to know about the sender and answer it. (d) all tuples in particular relation may have different
(b) Delete it after opening it. attributes.
(c) Delete it without opening it. (e) None of these
(d) Open it and try to find who the sender is.
(e) None of these GENERAL AWARENESS WITH
152. Letters, numbers and symbols found on a keyboard are –
(a) Icon (b) Screen REFERENCE TO BANKING
(c) Keys (d) Menu 161. To combat the menace of money laundering, which of the
(e) None of these following financial institutions has introduced the 'Know
153. Capital letters on a keyboard are referred to as – Your Customer' Scheme?
(a) caps lock key (b) grownups (a) IDBI (b) RBI
(c) big guys (d) upper case letters (c) NABARD (d) SIDBI
(e) None of these (e) None of these
154. A symbol on the screen that represents a disk, document 162. Which of the following sectors contributes maximum in
or program that you can select – deciding the growth in income of the states in India?
(a) keys (b) caps (a) Energy (b) Tourism
(c) icon (d) monitor (c) Service (d) Transport
(e) None of these (e) Agriculture
155. To insert a copy of the clipboard contents, whatever was 163. Which of the following is the state where the number of
last cut or copied at the insertion point. people living below poverty line is maximum?
(a) paste (b) stick in (a) Bihar (b) Andhra Pradesh
(c) fit in (d) push in (c) Uttar Pradesh (d) Rajasthan
(e) None of these (e) Odisha
y
o
u
rs
m
304 IBPS Bank PO/MT CWE Exam 2014

a
h
b
164. Who is the person closely associated with operation flood 171. How RBI measured to liquidate the market?

o
programmes and was honoured by Padma Vibhushan (a) By Reverse Repo rate

o
b
recently? (b) By Repo rate

.w
(a) Dr. V. Kurien (b) Dr. M. S. Swaminathan (c) By Cash Reserve Ratio

o
rd
(c) Dr. Amartya Sen (d) Dr. A. P. J. Abdul Kalam (d) By Statutory liquidity ratio

p
(e) None of these (e) None of these

re
165. What is the full form of MTSS? 172. The Bank should comply and intimate the compliance of

s
s
(a) Money Transfer Service Scheme Award to Ombudsman?

.c
o
(b) Money Transparency Service Scheme (a) Within Two Months

m
(c) Market Transfer Service Scheme (b) Within 3 months
(d) Market Tax Service Scheme (c) Within one month
(e) None of these (d) Within one year
166. What is the full form of EFT? (e) None of these
(a) Electric funds transfer 173. Cheque truncation can be done by?
(a) Using MICR data
(b) Electronic finance transaction
(b) Sending cheque by speed post
(c) Electronics Fund Transfer
(c) Using image processing
(d) Emergency fund transfer
(d) Both (a) & (b)
(e) None of these
(e) None of the above
167. RuPay is an Indian domestic card scheme conceived and 174. Which of the following is not insured by the DICGC
launched by (Deposit Insurance and Credit Gurantee Corporation)?
(a) National Payments Corporation of India (NPCI) (a) All Indian commercial Banks
(b) Industrial Finance Corporation of India (IFCI) (b) Foreign Banks branches functioning in India
(c) National Minorities Development & Finance (c) Local Area Banks
Corporation (NDMC) (d) Cooperative Banks
(d) National Handicapped Finance and Development (e) Primary cooperative societies
Corporation (NHFDC) 175. Who is the current Secretary General of UNO?
(e) None of these (a) Ban Ki-moon
168. Which of the following is refered as Fastest mode of (b) Kofi Annan
transaction? (c) U Thant
(a) Transfer funds into different bank's accounts using (d) Trygve Halvdan Lie
NEFT(National Electronic Funds Transfer). (e) None of these
(b) Transfer funds into other bank accounts using RTGS 176. What is the new name given to the Cadbury's India?
(Real Time Gross Settlement). (a) Ferrero Rocher Ltd
(c) Transfer funds into various accounts using IMPS (b) Dante Confections
(Immediate Payment Service). (c) Tootsie Roll Industries
(d) Transfer funds into different account of the same bank (d) Mondelez India Foods Ltd
(e) None of these (e) None of these
169. What is the full for of CBS? 177. Which day would be celebrated as Safe Motherhood Day
(a) Customer Bond Solution as announced by Health Ministry of India?
(b) Core banking System (a) 7th March
(c) Core Banking Solution (b) 8th March
(d) Customer Bond system (c) 9th March
(e) None of these (d) 10 March
170. What do you mean by Customer relationship management (e) None of these
(CRM)? 178. Which Hollywood personality got awarded at IIFA 2014?
I. It is a system for managing a company's interactions (a) John Travolta (b) Arnold Sch
with current and future customers. (c) Vin Diesel (d) Kevin Spacey
II. It is a system for managing a company's interactions (e) None of these
with current and past customers. 179. Which author had received Padmabhushan award 2014?
II. It often involves using technology to organize, (a) Shobha Dey (b) Ruskin Bond
automate, and synchronize sales, marketing, customer (c) Ravinder Singh (d) Chetan Bhagat
service, and technical support. (e) None of these
(a) Only I (b) Only II 180. What is the full form of IBSA ?
(c) Both II and III (d) Both I and III (a) Indonesia, Brazil, South Africa
(e) None of these (b) Italy, Brazil, Sudan
y
o
u
rs
m
IBPS Bank PO/MT CWE Exam 2014 305

a
h
(c) India, Brazil, South Africa (c) Rio de Janeiro, Brazil

b
o
(d) India, Belgium, Saudi Arabia (d) Beijing, China

o
b
(e) None of these (e) None of these

.w
181. Who is the author of the book '2 states' in the same name a 192. Recently, Northern Railways started a smart Card system,

o
rd
film was released reccently? what is the name of the Card?

p
(a) Ravinder Singh (a) Go Bharat Smart Card

re
(b) Drijoy Dutta (b) Go Nation Smart Card

s
s
(c) Amish Patel

.c
(c) Go India Smart Card
(d) Chetan Bhagat

o
(d) Go Railway Smart Card

m
(e) None of these (e) None of these
182. Who is appointed as the new deputy governor of RBI?
193. What is the name of the Pension Scheme of the Unorganised
(a) R Gandhi (b) S S Mudra Sector in India?
(c) Arjit Patel (d) Anil Sinha (a) Aajeevika Scheme
(e) None of these
(b) Swavalamban Scheme
183. Name the committee which is probing the IPL Spot-Fixing?
(c) Indira Awas Yojna
(a) Murali Panel Committee
(d) Varishtha Scheme
(b) Mudgal Panel committee
(e) None of these
(c) Rangaswamy Panel Committee
(d) Rangarajan Panel Committee 194. SEBI extended the time guideline to appoint at least one
women in board of director to ________?
(e) None of these
184. Kudamkulam Nuclear power plant is situated in which state? (a) 31st December 2014
(a) Andhra Pradesh (b) Karnataka (b) 31st March 2015
(c) Odhisha (d) TamilNadu (c) 30th June 2015
(e) None of these (d) 31st December 2015
185. Who is the current Prime Minister of UK? (e) None of these
(a) David Cameroon 195. Which account will not come under RBI’s limitation of ATM
(b) James Cameroon Transaction?
(c) Robert Cameroon (a) Current Deposits/Account
(d) Davis Cameroon (b) Basics Savings Bank Deposit Account
(e) None of these (c) Recurring Deposit/Account
186. Shivasumdaram Hydro Power Project is located in which (d) Fixed Deposit/Account
state? (e) None of these
(a) Andhra Pradesh (b) Karnataka 196. What is the name of BRICS Bank?
(c) Kerala (d) Tamilnadu (a) Federative Republic Bank
(e) None of these (b) New Republic Bank
187. Catolania referendum is related to which country? (c) New Development Bank
(a) Spain (b) Italy (d) New BRICS Bank
(c) Brazil (d) Germany
(e) None of these
(e) None of these
197. What is the capital of Portugal?
188. What is the minimum capital required for foreign bank to
open branch in India? (a) Ankara (b) Tunis
(a) 400 Crore (b) 450 Crore (c) Lisbon (d) Algiers
(c) 500 Crore (d) 550 Crore (e) None of these
(e) None of these 198. What is the currency of Saudi Arabia?
189. Priyanka Chopra in the movie Mary Kom, played the role (a) Dinar (b) Riyal
of? (c) Take (d) Ruble
(a) Gymnastics (b) Singer (e) None of these
(c) Boxer (d) Wrestler 199. Upalappu Srinivas of famous for?
(e) None of these (a) Mandolin (b) Table
190. What is the current Repo Rate? (c) Guitar (d) Sexophone
(a) 7% (b) 7.5 % (e) None of these
(c) 8% (d) 8.5 % 200. Who is the new Chairman & Managing Director of
(e) None of these Allahabad Bank?
191. 2014 Asian Games was held at? (a) S. K. Roy (b) T.C.A Ranganathan
(a) Doha, Qatar (c) Rakesh Sethi (d) U. K. Sinha
(b) Incheon, South Korea (e) Rahul Khullar
y
o
u
rs
m
306 IBPS Bank PO/MT CWE Exam 2014

a
h
b
o
o
b
.w
o
rd
p
re
1-5 : OR

s
1. (c) All possible cases

s
.c
o
Green

m
Toys Pens Papers Black
Books Copies

I. False Yellow
II. False Either
III. False Either
IV. False 3. (e) All possible cases are
OR
Cups
Glasses

Papers Pens
Black
Jugs

Toys Pens

OR

OR Pens
Cups
Black Glasses
Papers Jugs

Toys
Pens
I. False II. False
III. False IV. False
Altrernative Here, special case can be applied for Hence, none follows.
Conclusions I and III. Also for Conclusion II and 4. (d) All possible cases
Conclusion IV.
2. (a) All possible cases YYs
DCs
ECs
ACs

Copies
Yellow
I. False
Green II. True Either
III. False
I. False II. True IV. False
III. False IV. False
Hence, only II follows. Hence, either I or III and II follow.
y
o
u
rs
m
IBPS Bank PO/MT CWE Exam 2014 307

a
h
b
5. (b) All possible cases

o
G or H H or G E J K C

o
b
12 11 10 9 8 7

.w
Newspapers Radios

o
rd
p
Televison

re
1 2 3 4 5 6

s
s
F L or B I B or L A D

.c
OR

o
m
Case II

L or B I L or B J K C
Televison
12 11 10 9 8 7
Newspapers Radios

Magazines 1 2 3 4 5 6
F E G or H H or G A D
11. (d) From the above 2 cases, it follows case (i) and
I. False II. False opposite to Fatima is either Lalit or Binit.
III. True IV. False 12. (b) From the above 2 cases,
Hence, conclusions III follow. In case (i) if Lalit is sitting opposite to Hena then
Fatima is sitting opposite to Garima.
6-10 :
In case (ii) if Lalit is sitting opposite to Hena then
Jatin is sitting opposite to Garima.
13. (c) From the above 2 cases, it follows case (i) and
Left number of persons sitting between Binit and Dhiraj
E is either 1 or 3.
G 14. (c) From the given options only option (c) is correct.
15. (c) From the given options option (c) is in correct.
F
16-20 :
D
Given that there are five stages of the project from stage 1 to
J N stage 5 on the field with starting and ending points from
amongst P1, P2, P3, P4 and P5. Now given information is as
A follows:
W E
H or C (i) Stage 3 finished at P1.
(ii) Stage 4 must have started from point P1.
B
S (iii) Fourth stage work was given to farmer F4; hence, from
I all these outcomes we can conclude that 2nd stage is
H or C from P5 to P3.
Right Hence we can conclude that-
Stage 1- From point P2 to point P5
Stage 2- From point P5 to point P3
Stage 3- From point P3 to point P1
6. (d) 7. (c) 8. (d) 9. (a) 10. (c) Stage 4- From point P1 to point P4
11-15 : Stage 5- From point P4 to point P2
Now it is given that-
Let us denote these 12 students by their 1st letter of name, like Farmer F3 first stage.
Abhishek is A and so on. Farmer F4 fourth stage (from 5th information).
From the given information we can conclude that (C) and (D) Farmer F5 fifth stage (given that work of stage starting at
are at seat numbers 7 and 6, respectively. And (K) is the only point P4 is given to farmer 5).
person between (C) and (J) while (A) is opposite to (K). Hence, Farmer F1 second stage.
(A), (K) and (J) must be at seat numbers 5, 8 and 9. Stage 1- From point P2 to point P5 Farmer F3
respectively. Stage 2- From point P5 to point P3 Farmer F1
Stage 3- From point P3 to point P1 Farmer F2
Then we have following two cases:
Stage 4- From point P1 to point P4 Farmer F4
Case I Stage 5- From point P4 to point P2 Farmer F5
y
o
u
rs
m
308 IBPS Bank PO/MT CWE Exam 2014

a
h
Ne Ni is common in all the four statements. Exact

b
16. (a) Point P1

o
17. (e) From the above result, we get that the fifth stage transformation of Ne can’t be determined.

o
b
was ploughed by farmer F5. 28. (c) From I : P > R, P > T, U > P, U > Q, U > S

.w
18. (b) From the above result we get that the starting and ® U is tallest. [Since U is taller than P, Q & S and P is

o
ending points of stage 2 are P5 and P3. taller than R and T]

rd
19. (e) From the above result, we get that the P2 was the From II : R < U, P & Q...(i); Q > P ... (ii)

p
re
finishing point for farmer F5. From (i) and (ii) R < P < Q < U

s
20. (a) From the above result, we get that the starting Hence U is tallest.

s
.c
point for farmer F3 was P2. 29. (b) From I :

o
m
21-25 : Q(–) (+)N S
P*QÞP>Q
P Å QÞP>Q
P$QÞP=Q P(+)
P£QÞP<Q (–)L

P@QÞP<Q
21. (e) Statements N is maternal uncle of P.
O>F>E>P=R From II :
Conclusions
Maternal uncle
I. O $ P ® O = P (False)
II. E Å R ® E > R (True) L(–) (–)G (–)P
III. P £ O ® P < O (False)
22. (a) Statements
E>D=A>B< C M K
Conclusions (+) (+)

I. E * B ® E > B (True)
II. C $ A ® C = A (False) Paternal uncle
III. D @ E ® D < E (False) M is the paternal uncle of P
23. (b) Statements 30. (c) From I : Ramesh = 17th
I>H=T>S<R \ Shyam = (17 + 6) = 23th
Conclusions Sudin = (23 + 7) = 30th
I. I * T ® I > T From II:
Either
II. I$T®I=T Suketu ¬¾¾ Sudin ¾¾
® Shyamala
25 5

III. S * H ® S > H (False) Suketu ¾¾® Savita ¾¾


® Shyamala
15
24. (e) Statements
S<T<N=Q>O Suketa ¾¾® Savita ¾¾
® Sudin ¾¾® Shyamala
16 8 15
Conclusions So, sudin is 26th in rank in class of 44 students.
I. S $ N ® S = N (False) 31. (e) In online shopping the customer may be deceived as
II. N Å O ® N > O (False) he cannot touch the product he is paying for.
III. N * O ® N > O (True) 32. (d) The customer whose view is presented has shopped
25. (d) Statements at retail stores as well as online.
H > J > K = L, K < M 33. (e) The number of people migrating from rural to urban
areas in search of jobs may reduce.
Conclusions
34. (a) At least some people who visits the park have pets.
I. K £ M ® M > K (False)
35. (c) As,
II. L $ J ® L = J (False)
III. H Å L ® H > L (False) P R O V I D E
26. (d) From I and II: We get
D > B...(i) –3 +3 –3 +3 –3 +3 –3
K > W > M... (ii)
Still, we lack some clue as to whether B or M is the
M U L Y F G B
youngest. Hence, both statements I and II even
together are not sufficient. B E C A U S E
27. (d) From I: Na Ni Nok Ne ®I will tell you ... (i)
Ni Nok Ne Nam ® he will tell you ... (ii) –3 +3 –3 +3 –3 +3 –3
From (i) & (ii) Na ® I and Nam = he Similarly,
From II: Ni Ne Mo Nam ® will he call you ... (iii)
Ne Mok Sac Ni ® how will you go ... (iv) Y H Z D R V B
y
o
u
rs
m
IBPS Bank PO/MT CWE Exam 2014 309

a
h
b
36-38 : step all the words get arranged alphabetically whereas

o
numbers get arranged in descending order.

o
Total Number of flats = 13; Unoccupied flats = 5

b
If any word or number is already arranged in any step, the

.w
Occupied flats = 8 next number or word is arranged.

o
rd
Number of flats on second floor = 4 39. (c) Input : 98 11 64 22 but will an it

p
re
Second floor comprises four flats. One occupant is lawyer Step I : an 98 11 64 22 but will it

s
s
and since he has only one neighbour, this implies that out Step II : an 98 but 11 64 22 will it

.c
o
of four flats on second floor, two are unoccupied. Step III : an 98 but 64 11 22 will it

m
Again, since no flat is unoccupied on the third floor, it Step IV : an 98 but 64 it 11 22 will
implies that there are three unoccupied flats on floor IV.
Step V : an 98 but 64 it 22 11 will
Since there are at least three flats on any floor and no two
Step VI : an 98 but 64 it 22 will 11
same profession stay on any floor and the doctor is not the
neighbour of any lawyer, then floor III comprises only three 40. (a) Input : 32 now 20 gift 53 box 62 at
flats. Thus, floor IV comprises six flats (3 occupied + 3 Step I : at 32 now 20 gift 53 box 62
unoccupied).
Step II : at 62 32 now 20 gift 53 box
Since there are three managers and no two same profession
Step III : at 62 box 32 now 20 gift 53
stay on any floor, therefore, there will be a manager in each
floor. Also there are only two occupant in second floor and Step IV : at 62 box 53 32 now 20 gift
one of them is lawyer, therefore, second occupant should 41. (d) Input : pay by 18 36 nose ear 72 54
be manager. Step I : by pay 18 36 nose ear 72 54
Again, since there are two teachers, there will be a teacher Step II : by 72 pay 18 36 nose ear 54
each on floors III and IV. Again, doctor can’t be neighbour
Step III : by 72 ear pay 18 36 nose 54
of a lawyer. Hence, the doctor and lawyer will not reside on
same floor. Therefore, on floor III – either Doctor or Lawyer Step IV : by 72 ear 54 pay 18 36 nose
then, Step V : by 72 ear 54 nose pay 18 36
on floor IV – either Lawyer or Doctor. Step VI : by 72 ear 54 nose 36 pay 18
Total Occupied Unoccupied 42. (b) Step III : damn 96 flag 87 78 14 saint put
Floor Occupants
Flats flats flats
Step IV : damn 96 flag 87 put 78 14 saint
II 4 2 2 Lawyer,Manager
Step V : damn 96 flag 87put 78 saint 14
Teacher,
Step V is the last step. Therefore, penultimate step is
III 3 3 0 Manager,Lawyer
step IV.
or Doctor
43. (d) Previous steps cannot be determined .
Teacher,
44. (b) ‘17’ cannot be before ‘sky’
IV 6 3 3 Manager,Doctor
or Lawyer
45-48 :
From Statement A :-Yellow was the only colour used in all the
36. (d) Clearly, there are three flats. four rooms. It was used at least once for walls, carpets and
37. (a) From above table that combination is Lawyer & curtains
Manager. From statement B, Dining and bed room have same set of
38. (c) Both the manager and the teacher are the neighbour
colours.
of other lawyer.
39-44 : Walls Carpet Curtain
Here, logic is very simple. It is a case of Arrangement.
Input and following steps give the following information:
In step I the word which comes first according to
alphabetical order rearranges first. Living Room
In the second step the highest among the given numbers Study Room
gets arranged and occupies the place after the word
arranged in step I. From statement C, The same colour was chosen for the
These two steps get repeated alternately. Thus, in the last curtains in the bedroom, the carpet in the living room and the
y
o
u
rs
m
310 IBPS Bank PO/MT CWE Exam 2014

a
h
b
walls in the dining room. And that colour is not used in Study This combination must belongs to either Living or Study

o
o
Room, hence that colour must not be yellow as it is used in rooms.

b
.w
all the four rooms. And must not be grey as it is used only If it belongs to living rooms then

o
rd
twice and that too for curtains (From statement E)
Walls Carpet Curtain

p
re
Walls Carpet Curtain Living Room Green Yellow Grey

s
s
.c
But this combination is ruled out as from condition C the

o
m
common colourcan not be Yellow.
Living Room
Walls Carpet Curtain
Study Room
Living Room Yellow Green Grey
From statement D:- The only room with both green and grey
in its colour scheme had carpet of the same colour as in the But in this case from statement D Dining room will have Walls
dining room. If a room that has green and grey colour then its and carpet with green colour, but it is not possible as repetition
3rdcolour must be Yellow as it is used in all the rooms. That of colour is not allowed.
room must not be Dining and Bed room as they have same set Case (ii) So only possibility is that yellow grey green
of colours. The Carpet of this room must not be of grey colour belongs to study room. And then the common
(From statement E). colour mentioned in statement C is Orange colour,
From statement F:-The study room walls were painted the same hence we have two possibilities-
colour as the living room walls.

Walls
Carpet Curtain Explanation
Walls Carpet Curtain From
Dining Room Dining statement
Orange Yellow Grey
Bed Room
Room D, Curtains
colour is grey
Living Room Bed Room Orange
Study Room The 3rd
colour in
Living Room Green Orange Yellow
Now from D, E and F, we have two cases curtain is
Case (i) Yellow
Study Room Green Yellow Grey
Walls Carpet Curtain
Green /Yellow Yellow/Green Grey This case is ruled out as Dining and bed room has same
combination hence Grey can not be used other than Curtain.

OR

Walls Carpet Curtain


Carpets Green colour is from Statement D, & Yellow must be
Dining Room Orange Green Yellow
present (Step 1)
Bed Room Green Yellow Orange In carpet Yellow must be used

Living Room Yellow Orange Grey (As Grey is used at least twice step 2)
Study Room Yellow Green Grey

45. (c) Using above matrix, we can say Bed room 49. (c) How many goals scored ® 5397 ...(1)
46. (d) Green carpets: Many more matches ® 982 ...(2)
48. (b) Dining room- yellow curtains. He scored five ® 163 ...(3)
y
o
u
rs
m
IBPS Bank PO/MT CWE Exam 2014 311

a
h
b
From (1) & (2) 55. (a) For first year, S.I. = C.I.

o
o
Many ® 9 Now, ` 10 is S.I. on ` 100.

b
.w
From (1) & (3)
æ 100 ö
\ ` 16 is S.I. on ` ç ´ 16 ÷ = ` 160.

o
scored ® 3

rd
è 10 ø

p
\ goals can be 5 or 7.

re
So, S.I. on principal for 1 year at 10% is ` 160

s
T E M P O R A L

s
50. (a)
æ 100 ´ 160 ö

.c
\ Principal = ` ç ÷ = ` 1600.

o
–1 +1
è 10 ´ 1 ø

m
O L D S M B S P
Amount for 2 years compounded half yearly
C O N S I D E R
é æ 5 ö

and –1 +1 = ` ê1600 ´ ç1 + ÷ ú = ` 1944.81.
êë è 100 ø úû
R M N B S F E J
51. (c) Ratio of milk in the containers are, \ C.I. = ` (1944.81 – 1600) = ` 24.81.

1600 ´ 10 ´ 2 ö
1 3 5 5 3 25 S.I. = ` æç ÷ø = ` 320.
5´ : 4´ : 5´ = : : è 100
6 8 12 6 2 12
\ (C.I.) – (S.I.) = ` (344.81 – 320) = ` 24.81.
and the ratio of water in the containers are,
56. (d) Let the principal be ` P and rate of interest be R% per
5 5 7 25 5 35 annum.
5 ´ : 4 ´ : 5´ = : : Differecne of C.I. and S.I. for 3 years
6 8 12 6 2 12

Ratio of mixture of milk and water in the containers é æ R ö


3 ù æ P ´ R ´ 3 ö PR 2 æ 300 + R ö
= ê P ´ ç1 + ÷ - Pú - ç ÷= ç ÷.
êë è 100 ø úû è 100 ø 104 è 100 ø
æ1 3 5 ö æ5 5 7 ö
= ç ´5+ ´4 + ´ 5 ÷: ç ´ 5 + ´ 4 + ´ 5÷
è6 8 12 ø è 6 8 12 ø 2
æ R ö
Difference of C.I. and S.I. for 2 years = P ç ÷
= 106 : 230 = 53 : 115 è 100 ø
27 – 24 100
52. (a) % change in rate = ´ 100 = %
24 8 PR 2 æ 300 + R ö
ç ÷
For fixed expenditure, % change in consumption \ 10 4 è 100 ø = 25 Þ æ 300 + R ö = 25
ç ÷
PR 2 8 è 100 ø 8
% change in rate
= ´100 10 4
100 + % change in rate
100 1
100 / 8 100 1 ÞR= = 12 %.
= ´ 100 = % = 11 % 8 2
é 1ù 9 9
100 ê1+ ú 57. (b) Let x additional men employed.
ë 8û
117 men were supposed to finish the whole work in
53. (b) L × B × 2 = 48 46 × 8 = 368 hours.
Þ L × B = 24
Now, 6 – 6 × 10% = 5.4, 4
But 117 men completed of the work in 33 × 8
5 – 5 × 10% = 4.5 and 7
Therefore, 5.4 × 4.5 = 24.3
Clearly, 5 < L < 5.5 = 264 hours
54. (d) Let the original rate be R%. Then, new rate = (2R)%. \ 117 men could complete the work in 462 hours.
æ 725 ´ R ´1 ö æ 362.50 ´ 2R ´ 1 ö 3
\ç ÷+ç ÷ = 33.50 Now (117 + x) men are supposed to do of the work,
è 100 ø è 100 ´ 3 ø 7
Þ (2175 + 725)R = 33.50 ´ 100 ´ 3 = 10050 working 9 hours a day, in 13 × 9 = 117 hours, so as to
finish the work in time.
10050 i.e. (117 + x) men are supposed to complete the whole
ÞR = = 3.46%
2900
y
o
u
rs
m
312 IBPS Bank PO/MT CWE Exam 2014

a
h
b
7 d = Distance covered downstream in 4 hrs.

o
work in 117 ´ = 273 hours.

o
3

b
æ 4ö 44 11
Þ ç Su ´ 8 ÷ = (Sd ´ 4) Þ Su = 4Sd Þ Sd = Su .

.w
\
(117 + x) × 273 = 117 × 462 è 5ø 5 5

o
rd
Þ (117 + x) × 7 = 3 × 462 \ Required ratio

p
Þ x + 117 = 3 × 66 = 198 Þ x = 81

re
æ Sd + Su ö æ Sd - Su ö æ 16Su 1 ö æ 6Su 1 ö 8 3

s
\ Required number of additional men to finish the =ç ÷:ç ÷=ç ´ ÷:ç ´ ÷= :

s
è 2 ø è 2 ø è 5 2ø è 5 2ø 5 5

.c
work in time = 81.

o
= 8 : 3.

m
58. (a) Let A and B together work for x minutes than amount
æ 1 1 ö 7x
of water filled in the period = x ç + ÷ =
è 30 40 ø 120 1, 00, 000 15
61. (c) Cost of gear box = 20× ´ = 3000
100 100
7x æ 120 - 7x ö
Remaining part = 1 – =ç ÷ 20 ´ 1,00,000 30
120 è 120 ø 62. (c) Cost of brake = ´ = 6000
100 100
120 - 7x
Work done by A in (10 – x) minutes = 6000
120 \ Required percentage = ´ 100 = 6.0%
1,00, 000
7x
= 1-
120
20 ´ 1,00,000 15
63. (a) Price of tyres = ´ = 3000
7x 10 - x 100 100
+ = 1 or 7x + 40 – 4x = 120
120 30
125
3x = 120 – 40 = 80 Increased price of tyres = 3000 ´ = 3750
100
2 \ Price should be increased = 3750 – 3000 = `750
x = 26 min
3
120
59. (b) Let the distance between X and Y be x km. Then, the 64. (b) Increased transmission cost = 20,000 ´ = 24000
100
x 2x \ increase in transmission cost = 24000 – 20000 = `4000
speed of A is km/h and that of B is km/h.
4 7 Here, this increase will reduce the profit by 4000.
2x x km x
65. (a) Price of clutch = 30% of 20,000 = 6,000
km / h km / h
7 X Y 4 6000
\ Required percentage = ´100 = 6%
Relative speeds of the trains 1,00,000
æ x 2x ö 15x 66. (d) Total electric energy consume by 60 W bulb in whole
= ç + ÷= km / h
month = 16 × 60 × 7 × 30 W = 201.6 unit
è 4 7 ø 28
Therefore the distance between the trains at 7 a.m. = 67. (b) Electricity consumed by all fans = 14 × 80 × 11 × 30 W
Electricity consumed by all laptops = 9 × 200 × 5 × 30
x x
x- = km
2 2 14 ´ 80 ´ 11 ´ 30
Required% = ´ 100 = 136.88%
Hence, time taken to cross each other 9 ´ 200 ´ 5 ´ 30

x 68. (a) Electricity consumed by all fans


2 = x ´ 28 = 14 ´ 60 = 56 min = 14 × 80 × 11 × 30 = 369600 W
=
15x 2 15x 15
28 Electricity consumed by all laptops

Thus, both of them meet at 7 : 56 a.m. = 9 × 200 × 5 × 30 = 270000 W


Electricity consumed by all tubelights
60. (c) Let the man's upstream speed be S u kmph and = 17 × 40 × 8 × 30 = 163200 W
downstream speed be Sd kmph. Then,
Electricity consumed by all TV’s
Distance covered upstream in 8 hrs 48 min.
= 17 × 120 × 8 × 30 = 288000 W
y
o
u
rs
m
IBPS Bank PO/MT CWE Exam 2014 313

a
h
b
Electricity consumed by 100 W bulb 73. (b)

o
o
I. 4x2 – 24x + 20 = 0

b
= 11 × 100 × 9 × 30 = 297000 W

.w
x2 – 6x + 5 = 0

o
hence fans consumed more electricity.

rd
x2 – 5x – x + 5 = 0

p
69. (a) One unit cost = ` 2.70.

re
x (x – 5) –1 (x – 5) = 0

s
Power (used by AC’s) Unit cost = ` 3.70

s
(x – 5) (x – 1) = 0

.c
o
Electricity consumed by all equipment (except AC’s) x=5,1

m
= (201.6 + 369.6 + 270 + 163.2 + 288 + 297) unit II. y2 – 13y + 42 = 0

= (1292.4 + 297.00) unit = 1589.4 unit y2 – 7y – 6y + 42 = 0


y (y – 7) – 6 (y – 7) = 0
Cost for these unit = 1589.4 × 2.7 = 4291.38
(y – 7) (y – 6) = 0
Electricity consumed by AC’s
y = 7, 6
= 11 × 2100 × 9 × 30 W = 623700 W = 6237 Unit
Hence, y > x.
Cost for it = 6237 × 3.7 = 23076.9 74. (a)
Total cost = 23076.9 + 4291.38 = ` 27368 I. 2y2 + 3y – 5 = 0

201.6 2 2y2 + 5y – 2y – 5 = 0
70. (d) Required ratio = =
297 3 y (2y + 5) –1 (2y + 5) = 0
71. (b) (2y + 5) (y – 1) = 0
-5
I. 2x2 + 5x + 1 = x2 + 2x – 1 y= ,1
2
x2 + 3x + 2 = 0 II. 2
x – 3x = 2x – 6
x2 + 2x + x + 2 = 0
x2 – 5x + 6 = 0
x (x + 2) + 1 (x – 2) = 0
x2 – 3x – 2x + 6 = 0
(x + 2) (x + 1) = 0
x = –2 , –1 x (x – 3) –2 (x – 3) = 0
II. 2y2 – 8y + 1 = –1 (x – 3) (x – 2) = 0
2y2 – 8y + 2 = 0 x=3,2
y2 – 4y + 1 = 0 Hence, x > y

+4 ± 16 - 4 ´ 1 ´ 1 75. (e)
2 ´1 I. 6x2 + 14x = 12

= 2 ± 12 = 2 ± 2 3 3x2 + 7x – 6 = 0
Hence, y > x (x + 3) (3x – 2) = 0
72. (b) 2
I. x2 + 2x – 1 = 2 x=–3,
3
x2 + 2x – 3 = 0
17
x + 3x – x – 3 = 0 II. 1 + 2y2 = y
6
x (x + 3) – 1 (x + 3) = 0
12y2 – 17y + 6 = 0
(x + 3) (x – 1) = 0
12y2 – 8y – 9y + 6 = 0
x = –3 , 1
4y (3y – 2) – 3 (3y – 2) = 0
II. 2y2 – 12y + 18 = 0
(3y – 2) (4y – 3) = 0
y2 – 6y + 9 = 0
(y – 3)2 = 0 2 3
y= ,
y = 3, 3 3 4
Hence, y > x Hence, x < y
y
o
u
rs
m
314 IBPS Bank PO/MT CWE Exam 2014

a
h
76. (b) Suppose Giridhar invested ` x in company A.

b
is maximum for 2013 to 2014, and production during

o
all these years is almost same. Hence, in the year

o
x ´ 14 (25000 - x) ´13

b
\ + = 3340 2014, SIO registered maximum increase in production

.w
100 100
over the previous year.

o
rd
14 x 13 x 83. (d) Sum of the productions of companies in the first
or, + 3250 - = 3340

p
100 100 three years and the last three years is as follows :

re
s
Company First three Last three

s
x

.c
or, = 90 or,, x = ` 9000. years years
100

o
m
77. (a) Amount of dividend received by Anuja in 2011 from TP 358 349
company B ZIR 238 267
35000 ´ 19 AVC 900 1070
= = ` 6650
100 CTU 836 852
Total amount invested by Anuja in 2012 in Company A PEN 90 127
= 35000 + 6650 = ` 41650 SIO 261 279
120
Reqd amount = 41650 ´ = ` 49980 84. (c) Total production of the six companies in first two
100
given years = 863 + 927 = 1790
æ 20 15 ö Again, total production of the six companies in last
78. (b) Total dividend = 18000 ´ ç + = `6300
è 100 100 ÷ø two given years = 989 + 991 = 1980
5 ´12
79. (c) Reqd ratio = = 3: 4 1790 ´ 100
8 ´10 Therefore, required per cent = = 90.40%
1980
80. (d) From the graph it is obvious that Suraj will get less
dividend in 2014 from company A than from B. 85. (b) The required difference = (91– 92) crore units = 1 ×
Reqd less amount = 3% of 56000 = ` 1680. 10000000 = 10000000 units
81. (b) Production of company AVC in 2012 = 360 crore units
xy (-20 ´ 80)
Average production of AVC over the given years 86. (a) Net effect = x + y + = -20 + 80 +
100 100
1970 = 60 – 16 = 44% increased
=
6
87. (d) 25
360 ´ 6
Hence, required percent = ´ 100 = 109.64%
1970 24
» 110% x 25

82. (c) Apprroximate per cent increase or decrease in y


production from the previous year for SIO are as 32
follows :
(32 - y) 2 + (24 - x ) 2 = 625 ....(1)
2
2010 = ´100 = 2.35% ;
85 x 2 + y 2 = 625 ....(2)

2 ´ 100 Þ (24) 2 + (32) 2 - 64y - 48x = 0 (From (1) & (2))


2011 = = 2.29%
87
Þ 64 y + 48x = 576 + 1024
2 ´ 100 Þ 4y + 3x = 36 + 64 = 100
2012 = = 2.24% ;
89
æ 100 - 3x ö
1 ´100 or y = ç ÷
2013 = = 1.09% è 4 ø
91
2
4 ´100 2 æ 100 - 3x ö
2014 = = 4.35% \ x +ç ÷ = 625 (From (2))
92 è 16 ø

Quicker method : See the difference of produced Þ - 600 x + 16 x 2 + 10000 + 9 x 2 = 625 ´ 16


units between two consecutive years. The difference
y
o
u
rs
m
IBPS Bank PO/MT CWE Exam 2014 315

a
h
b
94. (c) The given sequence is a combination of two series :
Þ 25x 2 - 600 x + 10000 - 625 ´ 16 = 0

o
o
I. 11, 20, 40, 74 and

b
Þ x = 24 and y = 7

.w
II. 5, 12, 26, 54

o
1
\ Area = (24 ´ 25) + ´ 24 ´ 7 = 684

rd
The correct pattern in I is + 9, + 18, + 36,.....
2

p
re
88. (c) The word 'LEADING' has 7 different letters. So, 40 is wrong and must be replaced by (20 + 18) i.e. 38.

s
s
When the vowels EAI are always together, they can 95. (d) The correct pattern is × 2 + 1, × 3 + 1, × 4 + 1, × 3 + 1 + 1,.....

.c
o
be supposed to form one letter. So, 356 is wrong and must be replaced by (129 × 3 + 1)

m
Then, we have to arrange the letters LNDG (EAI). i.e. 388.
Now, 5 (4 + 1 = 5) letters can be arranged in 5! = 120 96. (d) Having a glance at the given options one can find out
ways.
that the two nearest values have a difference of 300.
The vowels (EAI) can be arranged among themselves
So round off the numbers to the nearest ten's values.
in 3! = 6 ways.
9228.789 » 9230; 5021.832 » 5020 and 1496.989 » 1500
\ Required number of ways = (120 × 6) = 720.
89. (a) Let S be the sample space and E be the event of Now the equation will become
selecting 1 girl and 2 boys. 9230 – 5020 + 1500 = ?
Then, n(S) = Number ways of selecting 3 students out
? = 5710
of 25
= 25C3 But the nearest value is 5700.
97. (a) It can be rounded off to the nearest ten's places.
(25 ´ 24 ´ 23)
= = 2300. 1002 » 1000; 49 » 50; 99 » 100 and 1299 » 1300
(3 ´ 2 ´ 1)

n(E) = (10C1 × 15C2) Now the equation will become


1000 ÷ 50 × 100 – 1300 = ?
é (15 ´ 14) ù
= ê10 ´
ë (2 ´ 1) úû 20 × 100 – 1300 = ?
2000 – 1300 = ?
= 1050.
? = 700
n(E) 1050 21
\ P(E) = = = 98. (d) The difference between two nearest values is 70 (210
n(S) 2300 46
and 280). So round off the numbers to the nearest
90. (c) Let the amount taxable purchases be Rs. x. integers. 29.8% of 260 » 30% of 260; 60.01% of
30 510 » 60% of 510 and 103.57 » 104
Then, 6% of x = Now the equation will become
100
30% of 260 + 60% of 510 – 104 = ?
æ 30 100 ö
Þx=ç ´ = 5. 30/100×260+60/100×510 – 104=?
è 100 6 ÷ø
78 + 306 – 104 = ?
\ Cost of tax free items = Rs. [25 - (5 + 0.30)] = `19.70 ? = 384 – 104 = 280
91. (d) The correct pattern is + 63, + 53, + 43, + 33, ..... 99. (a) (21.98)2 » (22)2
So, 521 is wrong and must be replaced by (498 + 33) i.e. (25.02)2 » (25)2
525.
and (13.03)2 » (13)2
92. (c) The correct pattern is ÷ 12, ÷ 10, ÷ 8,÷ 6,..... The equation will becomes
So, 24 is wrong and must be replaced by (48 + 6) i.e. 8. 222 – 252 + 132 = ?
93. (a) The terms of the series are (23 – 3), (33 – 3), (43 – 3), 484 – 625 + 169 = ?
(53 – 3), (63 – 3), (73 – 3), (83–3). 653 – 625 = ?
So, 27 is wrong and must be replaced (33 – 3) i.e. 24. ? = 28 so the nearest value is 25
y
o
u
rs
m
316 IBPS Bank PO/MT CWE Exam 2014

a
h
b
117. (d) Q is the opening sentence as it introduces the
50 ´ 25 1250

o
100. (e) = = 125 subject of evaluation followed by S, which is linked

o
10 10

b
with Q because, it gives conditions attached with

.w
101. (d) the subject of Q.

o
rd
102. (b) 118. (c) R is the opening sentence as it has the subject.

p
There may be a confusion between P and R, but the

re
103. (c)
subject of P- participation, is an object in R, Thus

s
s
104. (b) Only (c)

.c
R will be the just sentence, followed Q and then P,

o
105. (b) The realization of the link between food security and as Q and P explain the objects of R.

m
political stability. 119. (b) P is the opening sentence, followed by R because
106. (c) To make those countries more self sufficient to whom the ‘this’ in R refers to the idea stated in the opening
it previously provided food. sentence and works as a link between them. This
107. (c) It prompted increased investment and interest in will be followed by Q, because the pronoun subject
‘it’ refers to Ford Motor company.
agriculture.
120. (d) Q is the opening sentence, it introduces the subject.
108. (a) Misapprehension that it would alleviate poverty as it This will be followed by P which has a link with Q-
did in other countries. Chewing gum, then will be S which has a link with
109. (e) Only (b) and (c). P-Finish.
110. (c) Only (b) 121. (a) Substitute between for among
111. (a) The meaning of the word Starve (Verb) as used in the 122 (b) Substitute undergone
passage is : keep deprived of : to not give something 123. (b) Substitute detailed
that is needed. 124. (c) Substitute at least
125. (a) Delete no.
Hence, the words starved and deprived are synonyms.
126. (d) According to this passage, regular physical activity
112. (b) The meaning of the word Slap (Verb) as used in the is needed to improve one’s physical as well as mental
passage is : impose : to order especially in a sudden or health
an unfair way, that something must happen or 127. (d) In order to tone up the physical education programme
somebody must do something. the programme should be reoriented and implemented
Hence, the words slapped and imposed are synonyms. 128. (c) According to American, health education is more
important than teaching any subject
113. (a) The meaning of the word Plow (Verb) as used in the
129. (d) The author wants the reoriented physical education
passage is : to invest a large amount of money in a programme to be thoughtful
company or project : to cultivate. 130. (a) In order to improve the physical education programme,
Hence, the words plowed and cultivated are synonyms. we should first of all allot more time to the teaching
and learning of physical activity
114. (b) The meaning of the word Pressing (Adjective) as used
131. (e) 132. (d) 133. (b) 134. (b) 135. (c)
in the passage is : urgent, serous, insistent, needing
136. (c) 137. (d) 138. (a) 139. (e) 140. (b)
to be dealt with immediatley.
141. (b) 142. (b) 143. (a) 144. (a) 145. (b)
Hence, the words pressing and undemanding are
146. (a) 147. (c) 148. (e) 149. (b) 150. (b)
antonyms.
151. (c) 152. (c) 153. (a) 154. (c) 155. (a)
115. (c) The meaning of the word Evaporate (Verb) as used in
156. (d) 157. (a) 158. (d) 159. (d) 160. (c)
the passage is : to disappear, especially by gradually
161. (b) 162. (c) 163. (e) 164. (a) 165. (a)
becoming less and less.
166. (c) 167. (a) 168. (b) 169. (c) 170. (d)
Hence, the words evaporated and grew are antonyms. 171. (b) 172. (c) 173. (d) 174. (e) 175. (a)
116. (b) Q is the opening sentence, it defines the status quo, 176. (d) 177. (b) 178. (a) 179. (b) 180. (c)
followed by S, because S illustrates about customised 181. (d) 182. (a) 183. (b) 184. (d) 185. (a)
tailoring outfit, a subject mentioned in Q ‘custom made 186. (b) 187. (a) 188. (c) 189. (c) 190. (c)
clothing’. Thus will be followed by P, since P explains 191. (b) 192. (c) 193. (b) 194. (b) 195. (b)
further customised tailoring industry. 196. (c) 197. (c) 198. (b) 199. (a) 200. (c)
y
o
u
rs
m
a
h
b
o
SBI MANAGEMENT EXECUTIVE

o
b
.w
EXAM 2014

o
rd
p
Based on Memory

re
s
s
.c
o
m
Give answer (d) if neither course of action I nor II follows.
R EASONING ABILITY Give answer (e) if both courses of action I and II follow.
6. Statement : Drinking water supply to many parts of town is
DIRECTIONS (Qs. 1-5) : Study the following information disrupted due to loss of water because of leakage in pipes
carefully to answer the given questions : supplying water.
Eight persons - A, B, C, D, E, F, G and H are sitting around a Courses of action :
circular table facing outward, but not necessarily in the same I. The government should order an enquiry into the
order. They are at equidistant. D is third to the right of H. C is an matter.
immediate neighbour of both A and H.C is sitting to the left of B.H II. The civic body should set up a fact-finding team to
is second to the right of A. H is second to the left of E. F is not an assess the damage and take effective step.
immediate neighbour of A.
7. Statement: There is an alarming increase in the number of
1. Who is to the immediate left of D ?
people suffering from malaria in many parts of the city.
(a) F (b) E
Courses of action :
(c) G (d) Cannot be determined
I. The municipal corporation has advised all the
(e) None of these
government hospitals to store adequate supply of
2. How many persons are seated between H and F if we go
malaria drugs.
anticlockwise from H to F ?
(a) One (b) Two II. The municipal corporation has urged people to use
mosquito repellants and keep their premises clean.
(c) Three (d) Cannot be determined
(e) None of these 8. Statement : Many people have encroached into the
government property and built their houses and business
3. Which of the following is A's position with respect to D ?
establishments.
(a) Third to the right (b) Third to the left
Courses of action :
(c) Second to the right (d) Fourth to the left
(e) None of these I. The government should take immediate steps to
remove all unauthorised constructions on government
4. Who amongst the following is sitting just opposite to G?
land.
(a) E (b) H
II. All the encroachers should immediately be put behind
(c) C (d) B
bars and also be slapped with a hefty fine.
(e) Either B or E
9. Statement : The meteorological department has predicted
5. Which of the following statements is not true regarding the
seating arrangement ? normal rainfall throughout the country during the current
monsoon.
(a) There are only three persons between G and B
(b) D is sitting exactly between E and F Courses of action :
(c) G is second to the left of D I. The government should reduce the procurement price
(d) B is third to the right of A of foodgrains for the current year.
(e) All are true II. The government should reduce subsidy on fertilizers
for the current year.
DIRECTIONS (Qs. 6 - 10) : In each question below is given a
10. Statement : The number of dropouts in government schools
statement followed by two courses of action numbered I and II. A
has significantly increased in the urban areas over the past
course of action is a step or administrative decision to be taken
for improvement, follow-up or further action in regard to the few years.
problem, policy, etc. On the basis of the information given in the Courses of action :
statement, you have to as some everything in the statement to be I. The government should immediately close down all
true, then decide which of the suggested courses of action logically such schools in the urban areas where the dropout
follow(s) for pursuing. goes beyond 20 per cent.
Give answer (a) if only course of action I follows. II. The parents of all the students who dropped out of
Give answer (b) if only course of action II follows. the government schools in urban areas should be
Give answer (c) if either course of action I or II follows. punished.
y
o
u
rs
m
318 SBI Management Executive Exam 2014

a
h
b
DIRECTIONS (Qs. 11-15) : In each of the questions below are (a) Only I and IV follow (b) Only II, III and IV follow

o
(c) Only III and IV follow (d) Only I, II and III follow

o
given four statements followed by four Conclusions numbered I,

b
II, III and IV. You have to take the given statements to be true (e) Only I, II and IV follow

.w
even if they seem to be at variance from commonly known facts. DIRECTIONS (Qs. 16-20): In the following questions, the symbols

o
rd
Read all the Conclusions and then decide which of the given %, ,$, # and @ are used with the following meaning as
Conclusions logically follows from the given statements

p
illustrated below :

re
disregarding commonly known facts.
'P % Q' means 'P is neither smaller than nor equal to Q'.

s
s
11. Statements :

.c
All pens are books. All books are chairs. 'P ,Q' means 'P is not greater than Q'.

o
'P $ Q' means 'P is not smaller than Q'.

m
Some chairs are desks. Some desks are tables.
Conclusions: 'P # Q' means 'P is neither greater than nor equal to Q'.
I. Some tables are chairs. II. Some desks are pens. 'P @ Q' means 'P is neither greater than nor smaller than Q'.
III. Some chairs are pens. IV. All pens are chairs. 16. Statements :
(a) Only I and II follow (b) Only I and III follow D, R, R # M, M @ K, K % F
(c) Only I and IV follow (d) Only III and IV follow Conclusions :
(e) None of these I. F$M II. K @ R
12. Statements : III. K % R IV. D @ M
Some trains are buses. Some buses are trucks. (a) None is true (b) Only I is true
Some trucks are boats. Some boats are cars. (c) Only II is true (d) Only III is true
Conclusions :
(e) Only IV is true
I. Some trucks are trains. II. Some cars are trucks.
17. Statements :
III. Some boats are buses. IV. Some boats are trains.
(a) None follows (b) Only I and II follow R $ M, B @ A, A % K , K # M
(c) Only III follows (d) Only IV follows Conclusions :
(e) Only III and IV follow I. M %A II. K $ B
13. Statements : III. A % R IV. K # R
All hills are roads. All roads are stones. (a) Only IV is true (b) Only I and IV are true
All stones are jungles. All jungles are rivers. (c) Only II and III are true (d) Only I, III and IV are true
Conclusions : (e) None is true
I. Some rivers are stones. 18. Statements :
II. Some jungles are hills. D # M, M $ R, R @ J , W % J
III. Some stones are hills. Conclusions :
IV. All rivers are jungles.
I. W % R II. M $ J
(a) Only I and II follow (b) Only II and III follow
III. R % D IV. W % M
(c) Only I, II and III follow (d) Only II, III and IV follow
(e) All follow (a) Only II and III are true
14. Statements : (b) Only I and IV are true
Some books are pens. Some pens are glasses. (c) Only I and II are true
Some glasses are plates. Some plates are bottles. (d) Only III and IV are true
Conclusions : (e) All are true
I. Some bottles are books. 19. Statements :
II. Some glasses are books. W @ T, T $ N, N # F, V % F
III. Some plates are glasses. Conclusions :
IV. Some bottles are pens. I. V % N II. W $ N
(a) Only I and II follow III. T $ F IV. V @ N
(b) Only III follows (a) Only I and IV are true (b) Only I and II are true
(c) Only I, II and III follow
(c) Only IV is true (d) Only II, III and IV are true
(d) Only III and IV follow
(e) Only IV follows (e) Only III and IV are true
15. Statements : 20. Statements :
Some petals are flowers. B % K, K # D, D N, N @ T
All flowers are desks. Conclusions :
Some desks are cards. I. N%K II. T $ D
All cards are trains. III. K # B IV. T % K
Conclusions : (a) Only I and II are true
I. Some desks are flowers. (b) Only II and III are true
II. Some desks are petals. (c) Only I, II and IV are true
III. Some petals are cards. (d) Only III and IV are true
IV. Some desks are trains. (e) All are true
y
o
u
rs
m
SBI Management Executive Exam 2014 319

a
h
b
21. How many such digits are there in the number 836257419, (a) Only III is required to answer the question

o
each of which is as far away from the beginning of the (b) Only I and III are required to answer the question

o
b
number as when the digits are rearranged in ascending order (c) Only I or II and III together are required to answer the

.w
within the number? question

o
(a) None (b) One

rd
(d) Only II is required to answer the question
(c) Two (d) Three

p
(e) All I, II and III are not sufficient to answer the question

re
(e) More than three

s
DIRECTIONS (Qs. 26-30) : In each of the questions below is

s
DIRECTIONS (Qs. 22-25) : Each of the questions below consists

.c
given a statement followed by a question. Read the statement

o
of a question and three statements numbered I, II and III given carefully and answer the question that follows :

m
below it. You have to decide whether the data provided in the
26. Statement: A combination of factors have seen the sales
statements are sufficient to answer the question.
volume of the top 25 listed real estate companies almost
22. Who amongst P, Q, R, S, T and V, each securing different halve to about 11.8 million square feet in the quarter ended
marks, secured the second lowest marks ? September 2013. It was 20.73 million square feet in the year
I. R and T secured more marks than P and Q ago period.
II. V secured the highest marks Which of the following may be a probable reason for the'
III. S secured more marks than P but less than Q. slump in the real estate business ?
(a) Only land III (a) Housing is not a problem in India now-a-days and
(b) All I, II and III are required to answer the question every family has got a dwelling unit.
(c) Only II and III (b) The real estate companies have increased the profit
(d) Question cannot be answered even with all I, II and III margin and hence sales have come down.
(e) Only I and II (c) High cost of funds have dried up liquidity for the cash
starved companies, which in turn log jammed the
23. Which village is to the North-East of village R ?
construction activity across India.
I. Village S is to the South-East of village N which is to
(d) Residential and commercial units are being developed
the South-West of village P and village P is to the
at the faster pace in comparison to actual demand.
North of village Q.
(e) The faulty Housing Policy of the Government is an
II. Village T is to the North-West of village Q which is to
impediment to the real estate market.
the south of village P.
27. Statement : Democracy in the sense of majority rule is not
III. Village R which is to the North of village S, lies
what people are seeking. The middle classes in the Ukraine.
between villages N and Q and village N is to the West
Bosnia, Thailand and Venezuela are demanding greater
of village R.
accountability, and are challenging regimes seen as corrupt,
(a) Only I and II (b) Only II and III out of touch and which form obstacles to a better future.
(c) All I, II and III are not sufficient to answer the question Which of the following assumptions is implicit in the above
(d) All I, II and III are required to answer the question statement ?
(e) Only I and III or only II and III are required to answer (An assumption is something supposed or taken for
the question granted).
24. What is the rank of Animesh in a class of 17 students ? (a) The middle classes want a government that is
I. Nirmal who is thirteenth from the bottom is six rank accountable, responsible and effective in moving their
ahead of Bhumika who is two position below Animesh. country further into the modern world.
II. Bhumika is four position ahead of Kamal. (b) The importance of middle classes has been enhanced
III. Bhumika is two position below Animesh and Kamal's in the governance of a democratic country.
rank is 15th. (c) The middle classes are capable of overthrowing a
(a) Only I and III corrupt government.
(b) Only I and II (d) Democracy, in true sense, does not mean the rule of
(c) Only I or II and III together are required to answer the majority which takes into consideration only the
question numbers.
(d) Only II is required to answer the question (e) Except a small number of countries in the world, other
(e) All I, II and III are not sufficient to answer the question coun tries do not favour democr atic form of
25. How is 'them' written in a code language ? government.
I. 'tell them young' is written as 'se me ye' and 'wise young 28. Statement : Should admission to all professional courses
sharp tell' is written as 'me yo na ye' in that code be made on the basis of past academic performances rather
language. than through entrance tests ?
II. 'clever sharp come tomorrow' is written as 'na ki pa lo' Arguments :
and 'bring clever young them' is written as 'ki po se ye' I. Yes. It will be beneficial for those candidates who are
in that code language. unable to bear the expenses of entrance tests.
III. 'clever sharp come them no' is written as 'pa na se ki te' II. Yes. Many deserving candidates securing high marks
and "yellow come sharp run clever no' is written as in their qualifying academic examinations do not
'ki ni pa be te na' in that code language. perform well on such entrance tests.
y
o
u
rs
m
320 SBI Management Executive Exam 2014

a
h
b
III. No. The standard of examinations and assessment 33. How many students do like cricket only ?

o
conducted by different boards and universities are not (a) 7 (b) 6

o
b
comparable and hence there is a need to conduct (c) 4 (d) 5

.w
entrance tests to calibrate them on a common yardstick. (e) None of these

o
Decide which of the arguments is/are 'strong' argument(s)

rd
34. How many students do like football only ?
and which is/are 'weak' argument(s).

p
(a) 4 (b) 3

re
(a) Only I and II are strong (c) 5 (d) 6

s
(b) Only II and III are strong

s
(e) None of these

.c
(c) Only I and III are strong

o
35. How many students do like both volleyball and football but

m
(d) Only III is strong not cricket ?
(e) All I, II and III are strong (a) 4 (b) 6
29. Statement : Although the education system has progressed (c) 5 (d) 7
from the point of view of the number of schools, most of (e) None of these
them are ill equipped and have not achieved excellence in
imparting education. DIRECTIONS (Qs. 36-40): Study the following information
Which of the following inferences can be drawn from the carefully to answer the given questions :
above statement ? Six persons A, B, C. D, E and F are sitting in two cars namely X
(An inference is something which is not directly stated but and Y but not necessarily in the same order. Out of six persons
can be inferred from the given facts). two are driving the cars. There are three persons in each car and
(a) We need not open any more schools in future. one person must be on front seat besides the person who is
driving the car. In Car X, D is neither driving nor sitting on the
(b) In future, we should provide good teachers and
back seat. F is sitting on the back seat in Car Y. C is on the driver's
equipment to these schools. seat but not in the Car X. A is neither driving nor travelling in the
(c) It is necessary to open more schools to accommodate CarY.
the increasing number of students. E is not on the driver's seat in any car.
(d) It is very difficult to manage a large number of schools 36. Who among the following is on the driver's seat in the Car X ?
properly. (a) D (b) F
(e) Any system develops gradually and we should hope (c) B (d) Cannot be determined
for better outcomes. (e) None of these
30. Statement : Satisfaction with co-workers, promotion oppor- 37. Who among the following is sitting on the front seat in the
tunities, the nature of work, and pay goes with high Car Y ?
performance among those with strong growth needs. Among
(a) E (b) A
those with weak growth needs, no such relationship is present
and, in fact, satisfaction with promotion opportunities goes (c) F (d) Cannot be determined
with low performance. (e) None of these
The statement best support the premise that 38. Which of the following groups of three persons are travelling
(a) Satisfaction is an inevitable organisational variable in the Car X ?
(b) Job satisfaction and performance are directly and (a) B, E and F (b) B, D and E
closely related (c) A, C and E (d) A, B and D
(c) Relationship between job satisfaction and performance (e) None of these
is moderated by growth need 39. Which of the following pairs represents the persons sitting
(d) Every organisation has few employees having weak on the front seats in the Car X and Car Y respectively ?
growth need (a) D and F (b) D and E
(e) High performance is essential for organisational (c) B and E (d) A and C
effectiveness. (e) None of these
40. If D shifts his position to the driver's seat in his car, who will
DIRECTIONS (Qs. 31-35): Study the following information shift his position to the driver's seat in other car ?
carefully and answer the given questions :
(a) E (b) F
In a group of students, 21 students like football, 26 students like (c) A (d) B
cricket, 29 students like volleyball, 15 students like cricket and (e) C
volleyball, 14 students like cricket and football, 12 students like
volleyball and football and 8 students like all the three games. DIRECTIONS (41-45): Study the following information carefully
31. How many students are there in the group ? and answer the given questions :
(a) 45 (b) 48 Eight persons A, B, C, D, E, F, G and H are travelling in three
(c) 43 (d) 44 different Cars namely X, Y and Z, but not necessarily in the same
(e) None of these order. There are at least two persons in each Car and each Car has
32. How many students do like volleyball only ? persons of both the sexes. Out of eight persons three are females.
All of them like different colours viz, Red, Green, Yellow, Blue,
(a) 10 (b) 8
Black, White, Grey and Purple but not necessarily in the same
(c) 12 (d) 9 order. One female does not like Grey or White colour. D is travelling
(e) None of these with G in the Car Z. G likes Black colour. The persons who like Red
y
o
u
rs
m
SBI Management Executive Exam 2014 321

a
h
Marks answer (a) if the case is to be referred to the Dean of the

b
and Purple colours, are travelling in the same Car. E does not like

o
Purple colour and he is not travelling with H in the same car. Institution.

o
b
A does not like purple nor Red colour. H is travelling in the Car Y. Marks answer (b) if the case is to be referred to the Director of

.w
B, C and H are females in the group. B likes Blue colour and the Institution.

o
travels with the person who likes Black colour. Red and Green

rd
Marks answer (c) if the candidate is to be selected
colours are liked by female members. The person who is travelling

p
Marks answer (d) if the candidate is not to be selected

re
with H, likes Grey colour. One of the persons travelling in Car Z
Marks answer (e) if the data provided are inadequate to take a

s
likes Yellow colour.

s
decision.

.c
41. In which of the following Cars only two persons are travelling ?

o
(a) Car X (b) Car Y Now read the information provided in each question and mark

m
your answer accordingly.
(c) Car Z (d) Cannot be determined
46. Anup Bhatnagar was born on 12th June 1989. He can deposit
(e) None of these
an amount of ` 4 lakh at the time of admission. He secured
42. The female member who is travelling in the Car X likes which
85 per cent marks in the Graduation and 55 per cent marks in
colour ?
the Group Discussion and Interview. He secured 35 percent
(a) Green (b) Blue marks in the Entrance Test.
(c) Yellow (d) Red
47. Nisha Desai was born on 8th April 1991. She secured
(e) None of these 80 percent marks in the Graduation and 60 percent marks in
43. Who among the following likes Yellow colour ? Group Discussion and Interview. She can pay an amount of
(a) D (b) E ` 4 lakh per annum.
(c) A (d) F 48. Subodh Verma was born on 22nd May 1990. He secured
(e) None of these 50 percent marks in the Graduation and 40 percent marks in
44. Who among the following is/are travelling with female the Entrance Test. He secured 60 percent marks in Group
member C ? Discussion and Interview and can deposit an amount of
(a) E and G (b) A and D ` 4 lakh as fee.
(c) E and F (d) D and E 49. Ravindra Sharma was born on 24th September 1988. He
(e) F and G secured 60 percent marks in the Entrance Test, 60 percent
45. Which of the following combinations of Person-Car-Colour marks in the Graduation and 65 per cent marks in the Group
is not correct ? Discussion and Interview. He can pay ` 3 lakh in the
(a) C X Red (b) F X Purple beginning of the first year.
(c) G Z Black (d) D Z Yellow 50. Rajiv Batra secured 70 percent marks in the Entrance Test
(e) E Y Grey and was born on 5th June 1991. He secured 50 percent
marks in the Graduation and can pay ` 4 lakh at the time of
DIRECTIONS (Qs. 46-50): Study the following information admission. He secured 60 percent marks in the Group
carefully and answer the given questions : Discussion and Interview.
Following are the conditions for selecting candidates for Post
Graduation Diploma in Marketing in an institution.
The candidate must DATA ANALYSIS AND INTERPRETATION
(i) have a Graduation Degree with at least 50 percent marks
DIRECTIONS (Qs. 51-55) : In the following table, marks
(ii) be at least 22 years as on 01.12.2013
obtained by 6 students in 6 different subjects have been given.
(iii) have secured at least 40 percent marks in the Entrance Test Read the table carefully and answer the questions. The marks in
(iv) have secured at least 50 percent marks in the Group Discussion bracket show maximum marks.
and Interview
Marks obtained by 6 students in 6 different subjects
(v) be capable to pay a fee of ` 4 lakh per annum at the time
of admission. Subjects English Maths Physics Sociology Paychology Statistics
In the case of a candidate who fulfills all the conditions EXCEPT Students (150) (150) (150) (150) (150) (150)
(a) at (iii) above but has secured 30 percent marks in the Entrance Tony 86 78 95 105 72 112
Test and 80 per cent marks in the Graduation, his/ her case is to be Vicky 97 102 106 115 86 106
referred to the Dean of the Institution Ravi 115 108 87 99 76 69
(b) at (v) above but can deposit an amount of Rs. 2.5 lakh in the Shobha 68 76 84 72 66 88
beginning of the first year, his/ her case is to be referred to the Pallavi 118 106 126 120 112 98
Director of the Institution. Mahesh 86 99 108 102 78 86
In each question below, details of one candidate are provided. 51. Find the respective ratio of the total marks obtained by
You have to take one of the following courses of actions based
Vicky and Ravi in Maths and that scored by Pallavi and
on the conditions given above and the information provided in
Mahesh in Sociology ?
each question and mark the number of that course of action as
your answer. You are not to assume anything than the information (a) 35 : 37 (b) 37 : 32
provided in each question. All these cases are given to you as on (c) 39 : 59 (d) 29 : 35
01.12.2013. (e) None of these
y
o
u
rs
m
322 SBI Management Executive Exam 2014

a
h
57. The number of candidates recruited for the post of office

b
52. Find the approximate average percentage of marks obtained

o
by all the students in English. assistant operations in company B is approximately what

o
percent of total candidates recruited in that company ?

b
(a) 61 (b) 63

.w
(c) 68 (d) 65 (a) 28% (b) 24%

o
(e) None of these (c) 30% (d) 31%

rd
53. The ratio of total marks scored by Shobha in Sociology and (e) None of these

p
Physics and that scored by Tony in English and Maths is

re
58. The number of candidates recruited for the posts of assistant
(a) 37 : 39 (b) 13 : 23 manager and advertisement office assistant is what percent

s
s
of the candidates recruited for the post of junior office

.c
(c) 39 : 41 (d) 23 : 13

o
(e) None of these administrator and HR officer by the company C ?

m
54. What is the average marks scored by all the students in (a) 115% (b) 120%
Sociology ? (c) 135% (d) 141%
(a) 99.5 (b) 98.5 (e) None of these
(c) 100.5 (d) 102.2 59. The total number of candidates recruited for the post of HR
(e) None of these officers in all the companies is what percent of the total
55. Find the overall percentage of marks obtained by Shobha in candidates recruited by the company A for all posts ?
Physics, Psychology and Statistics. (a) 16% (b) 11%
(a) 45% (b) 48% (c) 12% (d) 14%
(c) 49% (d) 50.2% (e) None of these
(e) None of these 60. What is the respective ratio between the average number of
DIRECTIONS (Qs. 56-60) : In the following multiple graphs, candidates selected for all the posts by company A and
the number of selected candidates for 6 different posts by three company C ?
different companies A, B and C has been shown. Read the graph (a) 113 : 115 (b) 115 : 113
carefully and answer the questions. (c) 113 : 117 (d) 117 : 113
(e) 103 : 105
Company A ; Company B ; Company C DIRECTIONS (Qs. 61-65) : In each of the following questions, a
number series is given. After the series a number is given followed
Number of Selected Candidates in three companies by (a), (b), (c), (d) and (e). You have to complete the series
starting with the number given, following the sequence of the
original series and answer the questions that follow the series.
400
360 350 61. 37 19 20 31.5 65 165
Number of selected candidates

350 21 (a) (b) (c) (d) (e)


330 What will come in the place of (e) ?
300 280 (a) 105 (b) 41
290 260
250 (c) 110 (d) 108
250 220 (e) 116
200 62. 5 6 16 57 244 1245
200 9 (a) (b) (c) (d) (e)
160
150 What will come in the place of (d)?
130
100 80 (a) 366 (b) 364
70 100 (c) 368 (d) 378
50 50 40 (e) 382
10 30 63. 7 5 11 49 335 3005
0 13 (a) (b) (c) (d) (e)
1 2 3 4 5 6
POSTS What will come in the place of (b)?
(a) 31 (b) 27
POST 1 : HR officer (c) 29 (d) 28
POST 2 : IT officer (e) 30
POST 3 : Assistant Manager 64. 12 47 152 467 1412 4247
33 (a) (b) (c) (d) (e)
POST 4 : Advertisement office Assistant
What will come in the place of (d) ?
POST 5 : Office Assistant operatios
(a) 3131 (b) 1133
POST 6 : Junior office Administrator (c) 3311 (d) 3113
56. What is the ratio between the number of all candidates (e) 3123
selected for company A and that selected for the posts of 65. 54 50 84 188 496 1456
assistant managers and junior office administrators in all 42 (a) (b) (c) (d) (e)
three companies ? What will come the in the place of (d) ?
(a) 103 : 107 (b) 102 : 107 (a) 304 (b) 286
(c) 103 : 106 (d) 113 : 117 (c) 293 (d) 281
(e) None of these (e) 301
y
o
u
rs
m
SBI Management Executive Exam 2014 323

a
h
b
DIRECTIONS (Qs. 66-70): Read the following information Give answer (c) if the data either in statement I alone or in statement

o
o
carefully to answer the questions. II alone are sufficient to answer the question.

b
.w
In a store, there are 600 women. The store is a collection of precious Give answer (d) if the data given in both the statements I and II

o
stones namely diamond, ruby and emerald. It is found that 40% of together are not sufficient to answer the question, and

rd
women like diamond, 20% of women like ruby and 10% of women

p
like emerald. 5% of women like diamond and ruby; 3% of women Give answer (e) if the data in both the statements I and II together

re
like ruby and emerald and 4% like emerald and diamond. 2% of are necessary to answer the question.

s
s
women like all three stones.

.c
71. What will be the amount at the end of 2 years, if the interest

o
66. Find the number of women who like both ruby and diamond. is compounded yearly.

m
(a) 30 (b) 32 I. The simple interest on the same sum for a period of
(c) 12 (d) 6
2 years is `400 at the same rate of 5% per annum.
(e) 18
67. Find the number of women who like emerald only. II. The difference between the simple interest and the
(a) 32 (b) 30 compound interest for 2 years at the rate of 5% per
(c) 28 (d) 198 annum is `100.
(e) 40 72. Find the average of five consecutive odd numbers.
68. Find the number of women who like both diamond and I. The difference of fifth number and the first number is
emerald ? 7.
(a) 32 (b) 30
II. The sum of the first two numbers is 5 more than the
(c) 24 (d) 28
fifth number.
(e) 27
73. Find the ratio of the area of the bigger circle and smaller circle.
69. Find the number of women who like diamond only.
(a) 60 (b) 240
(c) 120 (d) 198 0
(e) 84 24 cm
3 cm
70. Find the number of women who like only ruby.
(a) 60 (b) 240 I.The radius of the smaller circle is 24 cm.
(c) 120 (d) 198 II.The difference between the radii of bigger and the
(e) 30 smaller circles is 3 cm.
DIRECTIONS (Qs. 71 - 75) : Each of the questions below consists 74. What is the length of the train ?
of a question and two statements numbered I and II given below I. The train crosses a signal post in 9 seconds.
it. You have to decide whether the data provided in the statements II. If the train with speed x kmph crosses another train
are sufficient to answer the question. Read both the statements 100m long coming from the opposite direction at 60
and kmph in 15 seconds.
Give answer (a) if the data in statement I alone are sufficient to 75. Find the radius of the semi-circle.
answer the question, while the data in statement II alone are not I. The area of semi-circle is equal to the area of the
sufficient to answer the question.
rectangle.
Give answer (b) if the data in statement II alone are sufficient to
answer the question, while the data in statement I alone are not II. The breadth of rectangle is 5 cm less than its length
sufficient to answer the question. and its perimeter is 50 cm.

DIRECTIONS (Qs. 76-80) : In the following table the production of different kinds of toys by a company in different years has been
given. Read the table carefully and answer the questions.
Production of 5-different Toys and Percentage of Defective Toys in Various Years

Toys Type-A Type-B Type-C Type-D Type-E


Years Prod uc % defective Prod uc % defective Prod uc % d efectiv e Prod uc % d efectiv e Prod uct % d efectiv e
tio n to ys tion to ys tion toy s tion toy s io n to ys
2006 18000 06 20000 06 12000 04 22000 07 23000 08
2007 21000 05 15000 05 15000 08 20000 08 18000 06
2008 16000 08 18000 04 17000 05 18000 05 17000 05
2009 22000 09 19000 06 20000 07 24000 06 20000 04
2010 24000 04 21000 09 24000 09 27000 08 24000 08
2011 28000 05 20000 05 28000 05 28000 05 27000 09
2012 26000 07 28000 08 31000 02 30000 05 30000 05
y
o
u
rs
m
324 SBI Management Executive Exam 2014

a
h
76. Find the approximate average number of defect free A, C (a) 26000 (b) 23000

b
o
and E types of toys manufactured in 2007. (c) 24000 (d) 18000

o
b
(a) 16890 (b) 16980 (e) None of these

.w
(c) 16880 (d) 17890 79. In which year was the maximum number of defective toys of

o
type-A manufactured ?

rd
(e) None of these (a) 2010 (b) 2008

p
77. How many defect free C-type of toys were manufactured in 2008?

re
(c) 2012 (d) 2009

s
(a) 16250 (b) 16150 (e) None of these

s
.c
(c) 16350 (d) 16450 80. Find the ratio between the number of defective toys of type A

o
(e) None of these in 2006 and that of defective toys of type-E in 2007 ?

m
(a) 3 : 2 (b) 2 : 3
78. Find the difference between the number of E-type of toys
(c) 1 : 2 (d) 2 : 1
manufactured in 2008 and the total number of A type and
B type of toys manufactured in 2009. (e) 1 : 1

DIRECTIONS (Qs. 81-85) : In the following bar diagram, production of three kinds of vehicles by a company in different years has
been given. Read the bar diagram carefully and answer the questions.
Production of Three Type of Vehicles A, B and C (In Thousands).

80
70
70
Production of Vehicles

65
60 60 60 60
60
(In thousands)

55
50 50
50 45 45 45 45 45 45
40
40 35
30 30 30 30
30
ABC ABC ABC ABC ABC ABC ABC
20
10
0
2006 2007 2008 2009 2010 2011 2012
Years

81. Find the ratio between the number of vehicles of type C (a) 30% (b) 20%
produced in 2012 and that of vehicles A produced in 2006. (c) 32% (d) 28%
(a) 8 : 9 (b) 7 : 9 (e) 26%
(c) 7 : 6 (d) 7 : 8
DIRECTIONS (86-90): In the following pie-charts, the
(e) None of these
percentage wise distribution of candidates who have applied
82. The number of vehicles of type B produced is 2009 is what
percent of the total production of vehicles in 2009? for different subjects in a college and that of selected candidates
(a) 20% (b) 25% has been given. Read the following pie-charts to answer the
(c) 18% (d) 21% questions.
(e) None of these Percentage of Candidates
83. What is the ratio between the total number of vehicles Applied Number of candidates = 88000
produced in 2012 and total production of A-type of vehicles
in the years 2007 and 2008 ?
(a) 7 : 9 (b) 7 : 8 Arts
(c) 7 : 6 (d) 8 : 7 17%
(e) None of these 19%
10% Commerce
84. In which year is the percentage increase in production of Others
type-A vehicles from the previous year is maximum ? Science
(a) 2009 (b) 2007 22%
8%
(c) 2010 (d) 2011 Maths 24%
(e) None of these
English
85. The number of type-C vehicles produced in 2010 is approxi-
mately what percent of total number of vehicles produced
in 2011 ?
y
o
u
rs
m
SBI Management Executive Exam 2014 325

a
h
b
Percentage of Candidates Qualified Number of Shirts and Trousers manufactured by five companies

o
Number of candidates = 14400 M, N, O, P and Q

o
b
.w
o
rd
4% 2000
1840

p
Others 14%

re
Arts 1800 1720

s
10% 1600

Number of shirts and trousers

s
1600

.c
Maths 20%

o
1400

m
Commerce
28% 1120
English 1200
24% 960
Science 1000
800
600
86. What is the difference between the total number of
400
candidates who got selected in Science and the number of
candidates who applied for the same ? 200
(a) 15904 (b) 14904
M N O P Q
(c) 15940 (d) 16940 Companies
(e) None of these
87. What is the sum of the total number of candidates who Ratio of Shirts & Trousers
applied for Arts and the number of candidates who got Companies Shirts Trousers
selected in Maths and English both ? M 5 3
N 24 19
(a) 19432 (b) 20432
O 7 9
(c) 20342 (d) 19432
P 3 5
(e) None of these
Q 8 17
88. What is the ratio between the number of candidates who
91. What is the average of the number of shirts manufactured
qualified in Arts and commerce together and the number of by the companies M, O and Q ?
candidates who qualified in English and Science ? (a) 639 (b) 539
(a) 17 : 25 (b) 17 : 29 (c) 693 (d) 369
(c) 17 : 26 (d) 29 : 17 (e) None of these
(e) None of these 92. The number of shirts manufactured by company P is
(a) 320 (b) 420
89. What percent of candidates qualified in English of the total
(c) 480 (d) 460
candidates applied for the same ? (e) None of these
(a) 15 (b) 16 93. What is the total number of trousers manufactured by
(c) 17 (d) 19 companies N and P ?
(e) 22 (a) 1360 (b) 1260
(c) 1460 (d) 1406
90. Find the average number of candidates who got selected
(e) None of these
for English, Science and Arts. 94. The number of shirts manufactured by company Q is what
(a) 3618 (b) 3682 per cent of its total production ?
(c) 3628 (d) 3268 (a) 25% (b) 28%
(e) 3168 (c) 30% (d) 32%
(e) None of these
DIRECTIONS (Qs. 91-95) : In the following bar diagram number 95. The ratio between the number of shirts manufactured by
of shirts and trousers manufactured by five different company M and that of trousers manufactured by company P is
companies M, N, O, P and Q has been given. The ratio of shirts (a) 9 : 7 (b) 8 : 7
and trousers has been given in the adjoining table. Read both (c) 7 : 8 (d) 5 : 7
the data and answer the questions. (e) 6 : 7
y
o
u
rs
m
326 SBI Management Executive Exam 2014

a
h
b
DIRECTIONS (Qs. 96-100) : In the following table, the number of vehicles passing over a bridge during different time intervals on

o
o
different days of a week is given. Read the table carefully to answer the following questions. Number of Vehicles (In thousands)

b
.w
Time Intervals 8-11 am 11 am - 1 pm 1 pm - 4 pm 4 pm - 7 pm 7 pm - 10 pm

o
rd
Days

p
Monday 12 10 8 11 6

re
Tuesday 15 12 10 12 5

s
s
Wednesday 10 8 6 8 6

.c
Thursday 11 7 7 7 7

o
m
Friday 13 10 8 10 6
Saturday 8 6 7 8 5
96. What is the difference between the total number of vehicles, economy. Indian education sector's market size in Financial Year,
crossing during 7pm-10 pm and the number of vehicles 2012-13 estimated to be USD 71.2 billion is expected to increase to
crossing during 11am-lpm on Tuesday, Thursday and Saturday ? USD 109.8 billion by Financial Year 2015-2016 due to the expected
(a) 8000 (b) 8500 strong demand for quality education. The market grew at a CAGR
(c) 7500 (d) 7800 of 16.5% during Financial Year 2015-2016. Financial Year 2005-
(e) None of these 2012.
97. Find the difference between the number of vehicles crossing Education has been made an important and integral part of the
on Tuesday and Saturday during lpm-4pm and the number national development efforts. The tremendous increase in the
of vehicles crossing on Thursday during lpm-4pm. num-ber of students and of educational institutions has given
(a) 7000 (b) 10000 rise to the term 'education explosion'. No doubt, this has resulted
(c) 24000 (d) 14000 in serious problems such as inadequacy of financial resources
(e) None of these and infrastructure and dilution of personal attention to the
98. What is the percentage decrease in the number of vehicles education and character-formation of the students. Also there is
crossing from time interval 8-11am to 7pm-10pm on the unwanted side-effect of enormous increase in the number of
Wednesday? educated unemployed. However, we cannot overlook the
(a) 45% (b) 38% advantages of education explosion in India. Mere increase in the
(c) 40% (d) 50% percentage of literate people does not indicate a qualitative change
(e) 46% in the educational standards of the people and a substantial
99. Find the average number of vehicles crossing the bridge improvement in manpower resources of India. Unemployment
during 8-11 am. problem in India cannot be blamed on the availability of large
(a) 11056 (b) 12500 masses of educated people in India.
(c) 11050 (d) 11500 Uncertainty and vacillation have marked the government's policy
(e) None of these regarding the medium of education in India. While the
100. Find the total number of vehicles crossing the bridge during government policy in this respect has not changed, a significant
1l am -1 pm on Thursday and Friday. increase in the number of schools-primary and secondary-imparting
(a) 11000 (b) 19500 education through the English medium is a significant
(c) 17500 (d) 19000 development: thousands of nursery schools th at have
mushroomed since the last decade purport to impart education to
(e) 17000
infants through English. This is an unwanted development which
ENGLISH LANGUAGE has been deprecated by educationalists and political leaders.
Regarding the medium of instruction in colleges and universities,
many State Governments have already decided, in principle, to
DIRECTIONS (Qs. 101-110) : Read the following passage switch over to the regional language. However the implementation
carefully and answer the questions given below it. Certain words/ in this respect has remained very slow.
phrases have been printed in bold to help you locate them while
Today virtually every university in India is offering
answering some of the questions.
correspondence courses for different degrees and diplomas.
The Indian education sector is one of the largest sunrise sectors In fact correspondence education has opened new vistas for the
contributing to the country's economic and social growth. The educational system which could not successfully meet the
Indian education system, considered as one of the largest in the challenging problem of providing infrastructure for multitudes of
world, is divided into two major segments of core and non-core new entrants into the portals of higher education. The public
businesses. While, schools and higher education form the core demand for higher education was initially met through evening
group, the non-core business consists of pre-schools, vocational colleges; now correspondence education has come to the rescue
training and coaching classes. The education sector in India is of the worried education administrators.
evolving, led by the emergence of new niche sectors like
101. Which of the following facts is not true regarding the Indian
vocational training, finishing schools, child-skill enhancement
and e-learning. India has emerged as a strong potential market for education sector as per the passage?
investments in training and education sector, due to its favourable (a) It is still in the process of development
demographics (young population) and being a services-driven (b) It is one of the contributors to India's growth
y
o
u
rs
m
SBI Management Executive Exam 2014 327

a
h
b
(c) There has been a recent trend towards the adoption of DIRECTIONS (Qs. 107-108) : Choose the word/group of words

o
regional languages as the medium of instruction

o
which is most similar in meaning to the word/group of words

b
(d) Mushrooming of schools imparting English education printed in bold as used in the passage.

.w
has been appreciated
107. Dilution

o
rd
(e) The number of educated unemployed as increased
(a) Thickening (b) Concentration

p
102. As per the passage, India's education sector has been able

re
(c) Extension (d) Diminution
to attract investments because of

s
(e) Development

s
A. The demographic factor.

.c
108. Emergence

o
B. The Indian economy being services-driven.

m
(a) Disappear (b) Rise
C. Indian democratic governance being an attractive issue.
(c) Abandonment (d) Fall
(a) Only (A) (b) Only (B)
(e) Lessening
(c) Only (C) (d) Both (A) and (B)
(e) Both (B) and (C) DIRECTIONS (Qs. 109-110) Choose the word/group of words
103. Which of the following explains the term 'education which is most opposite in meaning to the word/group of words
explosion' as per the passage? printed in bold as used in the passage.
A. Huge investment in the education sector. 109. Deprecate
B. Pro-active Government policy towards the education sector. (a) Derogate (b) Frown
C. Spurt in the number of students and educational (c) Object (d) Commend
institutions. (e) Disparage
(a) Both (A) and (B) (b) Only (C) 110. Purport
(c) Both (B) and (C) (d) Only (B) (a) Insignificance (b) Connotation
(e) All of the above statements are correct (c) Acceptation (d) Intention
104. Which of the following can be inferred as per the passage? (e) Purpose
A. Increase in literacy levels signifies a qualitative increase DIRECTIONS (Qs. 111-115) : Which of the phrases (a), (b), (c)
in educational attainment of people. and (d) given below should replace the phrase given in bold in
B. Literacy levels are closely related to improvement in the following sentence to make the sentence grammatically
manpower resources. correct. If the sentence is correct as it is and 'No correction is
C. The existence of educated people does not necessarily required', mark (e) as the answer.
contribute to the problem of unemployment. 111. British Airspace has been focusing on build European links.
(a) Only (A) (b) Only (B) (a) focusing on built (b) focusing on forged
(c) Only (C) (d) Both (A) and (B) (c) focusing on forging (d) concentrating to build
(e) All three (e) No correction required
105. As per the passage, which of the following statements 112. The appetite of banks for funds was lost under the onslaught
is/are correct? of the slowdown, corporates refused to borrow even as
(a) Increase in English medium schools is a welcome sign bank deposits flourished.
for the Indian education sector (a) bank deposits flourishing
(b) Increase in English medium schools in India is an (b) bank deposits swelled
insignificant, though wanted development (c) bank deposits were enhanced
(c) Correspondence education has proved to be a panacea in (d) bank deposits flummoxed
terms of educating people without proper infra-structure
(e) No correction required
(d) The implementation of regional languages as medium
113. The 8th century revival of Byzantine learning is an exemplary
of instruction has been quite fast
phenomenon and its economic and military precursors have
(e) The prospects for future growth of India's education yet to be discovered.
sector looks bleak
(a) phenomenon yet to be discovered.
106. Which of the following statements cannot be said to be the
(b) a phenomenon incompletely explained
highlights of the passage?
(c) an inexplicable phenomenon
A. India's education sector is marked by increase in the
number of educated people paralleled by simultaneous (d) an unidentifiable phenomenon
growth in unemployment. (e) No correction required
B. There are both positive and negative aspects of the 114. NASA is all set to start building the world's first spacecraft
education explosion in India. that collected samples from an asteroid in 2018.
C. The Government policy towards education has been (a) that will collect samples
that of clarity marked by sincere efforts. (b) that has collected samples
(a) Only (A) (b) Only (B) (c) that will have collected samples
(c) Only (C) (d) All three (d) who will collect samples
(e) None of these (e) No correction required
y
o
u
rs
m
328 SBI Management Executive Exam 2014

a
h
115. A sculpture by a veteran artist that stands in the lawns of 128. Physicians may soon have____ to help paralysed people

b
o
National Art Gallery was found damaged. move their limbs by by passing the ____ nerves that once

o
b
(a) that stood (b) that had stood controlled their muscles.

.w
(c) that was standing (d) that has stood (a) instruments, detrimental

o
(e) No correction required (b) ways, damaged

rd
(c) reasons, involuntary

p
DIRECTIONS (Qs. 116-120) : Read each sentence to find out

re
whether there is any grammatical error in it. The error if any, (d) impediments, complex

s
(e) instrumental, damaged

s
will be in one part of the sentence, the number of that part is the

.c
answer. If there is no error, mark (e). (Ignore errors of 129. The argument that the need for a looser fiscal policy to____

o
m
punctuation, if any) demand outweighs the need to ____budget deficits is
116. Politicians and actors (a)/are shining on the Indian (b)/ persuasive.
electoral firmament (c)/ for a long time. (d)/ No error (e) (a) assess, minimise (b) outstrip, eliminate
117. Almost all school teachers insist that (a)/ a student's mother (c) stimulate, control (d) restrain, conceal
(b)/ is responsible for the student's conduct (c) / as well as (e) facilitate, hide
his dress. (d)/ No error (e) 130. The Internet is a medium where users have nearly ____
118. In the forthcoming elections (a)/ every man and woman (b)/ choices and ____ constraints about where to go and what
must vote for the candidates (c)/ of their choice. (d)/ No to do.
error (e) (a) unbalanced, nonexistent (b) embarrassing, no
119. It is essential that diseases like tuberculosis (a)/ are detected (c) unlimited, minimal (d) chocking, shocking
and treated (b)/ as early as possible in order to (c)/ assure a (e) limited, maximal.
successful cure. (d)/ No error (e)
120. The Mumbai Police have found (a)/ the body of a man (b)/ DIRECTIONS (Qs. 131 - 135) : In each question below, four
who they believe to be (c)/ the prime suspect in a murder words printed in bold type are given. These are numbered (a),
case. (d)/ No error (e) (b), (c) and (d). One of these words printed in bold may either
be wrongly spelt or inappropriate in the context of the sentence.
DIRECTIONS (Qs. 121-125): In the following questions, a set Find out the word that is inappropriate or wrongly spelt, if any.
of five words is given. Four of the words are related in some way, The number of that word is your answer. If all the words printed
the remaining word is not related to the rest. You have to pick in bold are correctly spelt and appropriate in the context of the
the word which does not fit in the relation. sentence then mark (e) i.e. 'All Correct' as your answer.
121. (a) Disdain (b) Disloyalty 131. People who look (a)/ down on abstract (b)/ art have several
(c) Infidelity (d) Unfaithfulness major arguements (c)/ to support their beliefs. (d)/ All Correct (e)
(e) Unreliability 132. The world Trade Organisation (a)/ was formed (b)/ in the
122. (a) Astute (b) Crafty early 1990s as an oustcome (c)/ of the Uruguay Round
(c) Wily (d) Naive negotiations. (d)/ All Correct (e)
(e) Shrewd 133. Feminists (a)/ have viewed the Patirarchale (b)/ control of
123. (a) Discern (b) Recognise women's bodies as one of the prime issues (c)/ facing the
(c) Discriminate (d) Differentiate contemporary women's movement. (d)/ All Correct (e)
(e) Eviscerate 134. Composed (a)/ with other experimental (b)/ sciences,
astronomy (c)/ has certain limitations. (d)/ All Correct (e)
124. (a) Bewildered (b) Perplexed
135. Toxic effluents (a)/ deplete the level of oxygen in the rivers,
(c) Confused (d) Reserved
endanger (b)/ all aquatic (c)/ life and render water absolute
(e) Flummoxed unfit for human consumption. (d)/ All Correct (e)
125. (a) Pageant (b) Pretense
(c) Exhibition (d) Lurid DIRECTIONS (Qs. 136-140) : In each of the following questions,
five sentences are given that are not in a proper sequence. You
(e) Plush
are required to arrange all the five sentences so that all together
DIRECTIONS (Qs. 126-130): In the following questions, each make a logical paragraph. The correct order of the sentences is
sentence has two blank spaces. From the pairs of words given, your answer.
choose the one that fill the blanks most appropriately. 136. A. Most of the available savings channels do not give
126. Their achievement in the field of literature is described as returns that offset inflation.
____; sometimes it is even called ____. B. The prospect of getting a negative real return is forcing
(a) magnificent, irresponsible investors away from conventional financial savings
(b) insignificant, influential instruments.
(c) significant, paltry (d) unimportant, trivia C. The country needs to boost its financial savings to
(e) significance, trivia step up the overall investment rate.
127. This simplified ____ to the decision-making process is a D. Besides, the seemingly insatiable demand for gold has
must read for anyone ____ important real estate, personal very recently created serious macroeconomic
or professional decisions. problems.
(a) primer, maximizing (b) tract, enacting E. Although the threat of an imbalance has receded,
(c) introduction, under (d) guide, facing ongoing attempts to channel the demand into
(e) guide, enactment productive channels need to be encouraged.
y
o
u
rs
m
SBI Management Executive Exam 2014 329

a
h
(a) DABCE (b) CADEB

b
DIRECTIONS (Qs. 141-150) : In the following passage there

o
(c) ABCDE (d) EDCBA

o
are blanks, each of which has been numbered. These numbers

b
(e) BCEDA are printed below the passage and against each, five words are

.w
137. A. Yet, there appeared to be a contradiction in this, as the suggested, one of which fits the blank appropriately. Find out

o
rd
geographical area occupied by the charismatic cat was the appropriate word in each case.

p
reported to have decreased in some ranges. The fact that the majority of South Asia's population of about

re
B. A fresh exercise to count the country's tigers led by 1.5 billion (141) in villages points towards the need for marketers

s
s
the National Tiger Conservation Authority has now to develop a good understanding of rural markets and the rural

.c
been launched. mar-keting phenomenon. For our purpose, we define rural

o
m
C. The results published in 2010 claimed an improvement marketing (142) any marketing activity in which one dominant
in estimated tiger numbers, at 1,706 individuals participant is from a rural area. This implies that rural marketing
compared to 1,411 in 2006. comprises market-ing of inputs to rural markets as well as marketing
D. What is interesting is that a mere 10 per cent of the of outputs from rural markets to (143) geographical areas.
habitat today hosts 90 percent of the reproducing It is not only the size of the population that makes rural markets in
populations of the big cat. South Asia very (144) for marketers in India. Rural markets (145)
E. This is an important project, given that India hosts the for a similar proportion for consumer durable products too.
most number of tigers in the wild. Banking system, limited availability of credit facilities, and
(a) ABCDE (b) CABED problems of storage infrastructure are additional (146) for
marketers. These challenges need (147) solutions.
(c) EADCB (d) DCEBA
All the South Asian governments accord special (148) to the
(e) BCAED
development of rural areas by addressing issues such as poverty,
138. A. When IT moved into the Valley, educated Blacks found unequal growth, and poor infrastructure. This has led to (149)
opportunities a plenty. improvements in rural literacy levels in the last few years in all the
B. Once, Silicon Valley was the valley of fruits. South Asian countries. In addition, investments by governments
C. Then the White and Asianled organisations began to in improving farm productivity and generating greater employment
prefer South Asians and subtle discrimination had the opportunities in rural areas have made rural markets in South
African Americans moving out. Asia more (150) and prosperous.
D. Then the Hispanics and the Chinese ousted them. 141. (a) Believes (b) Resides
E. Blacks found work in the orchards and canneries and (c) Concentrates (d) Focuses
stores. (e) Amalgamates
(a) CADEB (b) AEBDC 142. (a) Since (b) For
(c) EBCAD (d) BEDAC (c) By (d) As
(e) DACBE (e) While
139. A. Then there is the technique of physical expression 143. (a) Diverse (b) Another
(angika abhinaya) using postures, hand gestures, etc. (c) Different (d) Other
B. A hallmark of Panikkar's presentation is his use of verbal (e) Many
expression (vachika abhinaya) borrowed from Kerala 144. (a) Valid (b) Central
traditions such as Koodiyattam.
(c) Important (d) Trivial
C. His own company, Sopanam, is distinguished by the
(e) Prime
elaborate physical training undergone by all the actors.
145. (a) Responded (b) Counted
D. In relatively short-term production this is not possible.
(c) Accounted (d) Catered
E. In his Hindi presentations too, this vachika approach
(e) Seized
is retained.
146. (a) Attractions (b) Obstacles
(a) BEACD (b) ACDBE
(c) Challenges (d) Pitfalls
(c) EBDCA (d) ADCEB
(e) Diversions
(e) CAEBD
147. (a) Pragmatic (b) Scientific
140. A. Our journey had a destination Thalassery.
(c) Radical (d) Innovative
B. It was a cloudy day and the beaches were crowded.
(e) Practical
C. And that was how I landed in a very tiny hamlet to see
a Theyyam performance for the first time. 148. (a) Fund (b) Budget
D. But I wanted to drive through the villages and we (c) Attention (d) Primacy
promptly got lost. (e) Resources
E. We drove along, turning into smaller villages and 149. (a) Appreciable (b) Revolutionary
hamlets, finding posters of Malayalam stars, while Che (c) Mind-boggling (d) Astonishing
Guevara looked on from every street corner. (e) Surprising
(a) ABCDE (b) BEADC 150. (a) Appropriate (b) Relevant
(c) CD ABC (d) DEBAC (c) Vibrant (d) Challenging
(e) EADCA (e) Dynamic
y
o
u
rs
m
330 SBI Management Executive Exam 2014

a
h
b
o
o
b
.w
o
rd
(1-5):

p
Conclusions:

re
I. False

s
D

s
II. False

.c
E F

o
III. True, conversion of IV conclusion.

m
IV. True
B G 12. (a) None of the conclusion follows.

H A Trains Buses Trucks Boats Car


C
1. (b) E is to the immediate left of D. 13. (c) Rivers
2. (c) Three persons – C, A and G are seated between H Jungles
and F if we go anticlockwise from H to F. Stones
3. (a) A is third to the right of D. Roads
4. (d) B is sitting just opposite to G.
Hills
5. (c) G is second to the right of D.
6. (d) Both of the course of action are not necessary
because ordering an enquiry into the leakage matter
would not solve any purpose and secondly no need
of setting up fact-finding team when cause of
leakage is known to us.
7. (e) Both the course of action I and II should be Conclusions:
followed because sufficient supply of malaria drugs I. True
is necessary to avoid malaria. Secondly use of II. True
mosquito repellant and cleanliness are necessary to III. True
avoid mosquito breeding. IV. False
8. (a) Only I course of action is suitable to follow because So only I, II and III follow.
the government should check all unauthorised
14. (b)
construction on government land. II course of Book
action is too harsh to be suitable. Pen
9. (d) Both the courses of action are not suitable because
when there is no problem, no action is required. Glasses
10. (d) Both the course of action are not practically Plates
possible because government cannot close down all
such schools and also parents of such children Bottles
cannot be punished because there may be genuine
reason for dropouts. No conclusion follows but conclusion III is converse
11. (d) Chairs of third premise so conclusion III is follows
15. (e)
Books Desks
Pens
Desks Flowers
Trains
Tables Cards
Petals
OR
Chairs Conclusions:
I. True, Conversion of the first premise
Books
II. True
Pens III. False
Desks
IV. True
Tables
y
o
u
rs
m
SBI Management Executive Exam 2014 331

a
h
Therefore,

b
(16 - 20): % Þ > ¨ Þ £ $ Þ ³

o
B>K<D£N=T

o
b
# Þ < @ Þ = Conclusions:

.w
(d) D ¨ R Þ D £ R I. N % KÞ N > K : True

o
16.

rd
R#MÞR<M II. T $ D Þ T ³ D : True

p
M@KÞM=K III. K # B Þ K < B : True

re
IV. T % K Þ T > K : True

s
K% F Þ K > F

s
.c
Therefore, 21. (c) 8 3 6 2 5 7 4 1 9

o
D£R<M=K>F

m
1 2 3 4 5 6 7 8 9
Conclusions:
22. (b) From statement I
I. F $ M Þ F ³ M : Not True
R, T > P, Q
II. K @ R Þ K = R : Not True
From statement II
III. K % R Þ K > R : True
V > R, T, P, Q S
IV. D @ M Þ D = M : Not True
From statement III Q > S > P
17. (a) R $ B Þ R ³ B
From statements I and II
B@AÞB=A
V > R, T > P, Q
A%KÞA>K
From all the three statements
K#MÞK<M
V > R, T > Q > S > P
Therefore,
Clearly, S secured the second lowest marks.
R³B=A>K<M
23. (e) From statement I
Conclusions :
P
I. M % A Þ M > A : Not True N
II. K $ B Þ K ³ B : Not True NW NE
III. A% R Þ A > R : Not True N Q W E
IV. K # R Þ K < R : True SW S SE
Only conclusion IV follows.
S
18. (c) D # M Þ D < M There is no information about the village R in the
M$RÞM³R statement I.
R@JÞR=J From statement II
W%JÞW>J T P
Therefore,
D<M³R=J<W
Conclusions : Q
I. W % R Þ W > R : True There is no information about the village R in the
II. M $ J Þ M ³ J : True statement II.
III. R % D Þ R > D : Not True From statement III
IV. W % M Þ W > M : Not True N R Q
Only conclusion I and II follow.
19. (b) W @ T Þ W = T
T$NÞT³N S
N#FÞN<F So Statement I and II donot give any information
V%FÞV>F about village R. So we cannot take any decision
Therefore, even with the statements I and II taken together.
W=T³N<F<V From statements I and III
Conclusions : P
I. V % N Þ V > N : True
II. W $ N Þ W ³ N : True
III. T $ F Þ T ³ F : Not True N R Q
IV. V @ N Þ V = N : Not True
So only conclusion I and II follow.
20. (e) B % K Þ B > K
K#DÞK<D S
It is clear from the diagram that village P is to the
D¨NÞD£N
North-East of village R.
N@T Þ N = T
y
o
u
rs
m
332 SBI Management Executive Exam 2014

a
h
From statements II and III Codes are

b
o
T P them Þ se

o
b
yellow Þ ni or be

.w
run Þ ne or be

o
rd
N R Q Only I and III are required to answer the question.

p
26. (c) Increase in material and labour cost is the main

re
reason for slump i the real estate business which has

s
s
affected net margin severly and higher interest rates

.c
S
have affected cost of funds adversly.

o
It is clear from the diagram that village P is to the

m
North-East of village R. 27. (a) The middle classes expect from the government more
24. (c) From statement I accountability, responsibility and effectiveness in
The position of Nirmal from the top policy implementation so that country as a whole
may progress more and more.
= 17– 13 + 1 = 5th
28. (d) Argument I and II are weak because deserving
5th 9th 11th students cannot be left at the cost of expenses of
4 6
N 6 7 8 A 10 B entrance test. There is not reason for poor
13th performance in entrance test if a students has
12 performed well in academic examination. So argument
The rank of Animesh from the either end is 9th. III seems to be strong because entrance test a
From statement II common yard stick of evaluation.
29. (b) Only statement II can be inferred from the given
B K statement because proper resources like good
From statement III equipment and good teachers would ensure proper
The rank of Kamal = 15th results in imparting education.
Bhumika is two position below Animesh 30. (c) Statement III best support the premise because job
From statements II and III satisfaction and performance of an individual are
9th 11th 15th directly related with growth.
8 2 (31-35) :
A B K
The rank of Animesh is 9th. Let A, B and C be the sets of students who like
Football, Cricket and Volleyball respectively. Then
So either I or II and III together are required to
answer the question. n(A) =21
25. (b) From statement I n(B) = 26
n(C) = 29
tell them young ® se me ye n(A Ç B) = 14
wise young sharp tell ® me yo na ye n(A Ç C) = 12
n(B Ç C) = 15
Codes are
n(A Ç B Ç C) = 8
tell Þ me or ye
\ n(A È B È C)
young Þ me or ye
= [n(A) + n(B) + n(C) + n(A Ç B Ç C)]
them Þ se
– [n(A Ç B) + n (A Ç C) + n (B Ç C)]
From statement II
= [21 + 26 + 29 + 8] – [14 + 12 + 15]
= 84 – 41 = 43
clever sharp come tomorrow ® na ki pa lo
Now, we can draw the complete Venn diagram as
follows:
bring clever young them ® ki po se ye A (Football) B (Cricket)
Codes are
clever Þ ki a b e
From statement III 6 5
3
c
clever sharp come them no —® d 8 f
4 7
pa na se ki te g
10
yellow come sharp run clever no C (Volleyball)
31. (c) There are 43 students in the group.
—® ki ni pa be te na 32. (a) It is clear from the Venn diagram that 10 students like
Volleyball only.
y
o
u
rs
m
SBI Management Executive Exam 2014 333

a
h
33. (d) It is clear from the Venn diagram that 5 students like 46. (a) Anup Bhatnagar satisfies conditions (i), (ii), (a), (iv)

b
o
Cricket only. and (v). Therefore, his case would be referred to the

o
b
34. (b) It is clear from the Venn diagram that 3 students like Dean of the Institution.

.w
Football only. 47. (e) In case of Nisha Desai there is no information about

o
35. (a) It is clear from the Venn diagram that 4 students like the condition (iii).

rd
Football and Volleyball but not Cricket. 48. (c) Subodh Verma satisfies conditions (i), (ii), (iii), (iv)

p
re
(36 – 40) and (v). Therefore, he can be selected.

s
49. (b) Ravindra Sharma satisfies conditions (i), (ii), (iii), (iv)

s
.c
and (b). Therefore, his case would be referred to the
Person Car Sitting position in car

o
Director of the Institution.

m
A X Back-seater 50. (c) Rajiv Batra satisfies all the conditions (i), (ii), (iii), (iv)
B X Driver and (v). Therefore, he can be selected.
C Y Driver 51. (a) Required ratio = (102 + 108) : (120 + 102)
D X Front-seater = 210 : 222 = 35 : 37
E Y Front-seater 52. (b) Total marks obtained in English
= 86 + 97 + 115 + 68 + 118 + 86 = 570
F Y Back-seater
570
\ Required percentage = ´ 100 = 63.2 » 63
36. (c)B is on the driver’s seat in the Car X. 900
37. (a)E is sitting on the front-seat in the Car Y. 53. (c) Required ratio = (72 + 84) : (86 + 78)
38. (d)A, B and D are travelling in the Car X. = 156 : 164 = 39 : 41
39. (b)D is sitting on the front-seat in the Car X and E is 54. (d) Total average marks in Sociology
sitting on the front-seat in the Car Y.
105 + 115 + 99 + 72 + 120 + 102 613
40. (a) D is on the front-seat in the Car X and E is on the = = » 102.2
front seat in Car Y. 6 6
(41-45) : 55. (e) Marks obtained by Shobha in Physics, Psychology
and Statistics
= 84 + 66 + 88 = 238
Person Gender Car Colour
238
A Male Y Grey Required percentage = ´ 100 » 53%
450
B Female Z Blue
56. (a) Total number of candidates selected for company A
C Female X Red = 80 + 40 + 100 + 330 + 280 + 200 = 1030
D Male Z Yellow Assistant Managers + Junior Office Administrators
E Male X While = (100 + 130 + 160) + (200 + 220 + 260)
F Male X Purple = 390 + 680 = 1070
Required ratio : 1030 : 1070 = 103 : 107
G Male Z Black
57. (b) Total candidates selected in company B
H Female Y Green = 10 + 30 + 160 + 360 + 250 + 220 = 1030
41. (b) A and H are travelling in the Car Y. 250
\ Required percentage = ´ 100 » 24%
42. (d) C is the female member in the Car X. She likes Red 1030
colour. 58. (c) Company C
43. (a) D likes Yellow colour. Assistant Manager + Advertisement Office Assistant
44. (c) E and F are travelling with C. = 130 + 290 = 420
45. (e) The combination E-Y-Grey is not correct. Junior Office Administrator + HR Officer
E travels in Car X and he likes white colour. = 260 + 50 = 310
(46-50) : 420
Required percentage = ´ 100 » 135%
310
CRITERIA 59. (d) Total number of HR officers selected
CANDIDATE
(i) (ii) (iii) or (a) (iv) (v) or (b) Ans = 80 + 10 + 50 = 140
Anup ü ü – ü ü ü – 1 140
\ Required percentage = ´ 100 » 14%
Nisha ü ü × – ü ü – 5 1030
Subodh – – 3
60. (e) Total candidates selected :
ü ü ü ü ü
– Company A Þ 1030
Ravindra ü ü ü û – ü 2
Company C Þ 1150
Rajiv ü ü ü – ü ü – 3 Required ratio = 1030 : 1150 = 103 : 115
y
o
u
rs
m
334 SBI Management Executive Exam 2014

a
h
61. (a) The pattern of the given series is : (66 – 70) :

b
o
37 × 0.5 + 0.5 = 18.5 + 0.5 = 19

o
600 ´ 40

b
19 × 1 + 1 = 19 + 1 = 20 Diamond Þ = 240

.w
100
20 × 1.5 + 1.5 = 30 + 1.5 = 31.5

o
31.5 × 2 + 2 = 63 + 2 = 65 600 ´ 20

rd
65 × 2.5 + 2.5 = 162.5 + 2.5 = 165 Ruby Þ = 120

p
100

re
Similarly,

s
600 ´ 10

s
21 × 0.5 + 0.5 = 10.5 + 0.5 = 11 (a) Emerald Þ = 60

.c
11 × 1 + 1 = 11 + 1 = 12 (b) 100

o
m
12 × 1.5 + 1.5 = 18 + 1.5 = 19.5 (c) Diamond + Ruby
19.5 × 2 + 2 = 39 + 2 = 41 (d) 600 ´ 5
Þ = 30
41 × 2.5 + 2.5 = 102.5 + 2.5 = 105 (e) 100
62. (b) The pattern of the given series is : Ruby + Emerald
5 × 1 + 12 = 5 + 1 = 6 600 ´ 3
6 × 2 + 22 = 12 + 4 = 16 Þ = 18
100
16 × 3 + 32 = 48 + 9 = 57 Emerald + Diamond
57 × 4 + 42 = 228 + 16 = 244
244 × 5 + 52 = 1220 + 25 = 1245 600 ´ 4
Þ = 24
Similarly, 100
9 × 1 + 12 = 9 + 1 = 10 (a) Diamond + Emerald + Ruby
11 × 2 + 22 = 22 + 4 = 26 (b) 600 ´ 2
26 × 3 + 32 = 78 + 9 = 87 (c) Þ = 12
100
87 × 4 + 42 = 348 + 16 = 364 (d)
nd Ru
63. (c) The pattern of the given series is : ia mo by
D
7×1–2=7–2=5
5 × 3 – 4 = 15 – 4 = 11 d b
a
11 × 5 – 6 = 55 – 6 = 49
49 × 7 – 8 = 343 – 8 = 335 e
f g
335 × 9 – 10 = 3015 – 10 = 3005
Similarly,
13 × 1 – 2 = 13 – 2 = 11 (a) c

11 × 3 – 4 = 33 – 4 = 29 (b) Emerald
64. (d) The pattern of the given series is : a + d + e + f = 240
12 × 3 + 11 = 36 + 11 = 47 b + d + e + g = 120
47 × 3 + 11 = 141 + 11 = 152 c + f + e + g = 60
152 × 3 + 11 = 456 + 11 = 467 d + e = 30
467 × 3 + 11 = 1401 + 11 = 1412 e + g = 18
1412 × 3 + 11 = 4236 + 11 = 4247 e + f = 24
Similarly, e = 12
33 × 3 + 11 = 99 + 11 = 110 (a) \ f = 24 – 12 = 12
110 × 3 + 11 = 330 + 11 = 341 (b) g = 18 – 12 = 6
341 × 3 + 11 = 1023 + 11 = 1034 (c) d = 30 – 12 = 18
1034 × 3 + 11 = 3102 + 11 = 3113 (d) c = 60 – 12 – 12 – 6 = 30
65. (e) The pattern of the given series is : b = 120 – 30 – 6 = 84
68 × 1 – 8 = 60 a = 240 – 30 – 12 = 198
60 × 1.5 + 14 = 90 + 14 = 104 66. (a) Number of women who like both ruby and diamond
= d + e = 36
104 × 2 – 20 = 208 – 20 = 188
188 × 2.5 + 26 = 470 + 26 = 496 67. (2) Number of women who like emerald only = c = 30
496 × 3 – 32 = 1488 – 32 = 1456 68. (c) Number of women who like both diamond and
emerald = f + e = 24
Similarly
69. (d) Number of women who like diamond only = a = 198
42 × 1 – 8 = 42 – 8 = 34 (a)
70. (e) Number of women who like ruby only = b = 30.
34 × 1.5 + 14 = 51 + 14 = 65 (b)
71. (c) From statement I,
65 × 2 – 20 = 130 – 20 = 110 (c)
S.I. ´100 400 ´100
110 × 2.5 + 26 = 275 + 26 = 301 (d) P= = = ` 4000
R´T 5´ 2
y
o
u
rs
m
SBI Management Executive Exam 2014 335

a
h
Using,

b
50670

o
Required average = = 16890

o
T 3
æ R ö

b
A = P ç1 + ÷ , amount can be determined.

.w
è 100 ø 17000 ´ 95

o
77. (b) Required answer = = 16150

rd
From statement II, 100

p
PR 2 78. (c) Required difference = 22000 + 19000 – 17000 = 24000

re
Difference = 79. (d) Number of defective toys of type – A :

s
(100)2

s
Year 2006

.c
We can find principal and hence amount.

o
18000 ´ 6

m
Data either in statement I alone or in statement II Þ = 1080
alone are sufficient to answer the question. 100
72. (d) From both the statement, we get no result. Year 2008
73. (e) From both the statements, 16000 ´ 8
r1 = 24 cm Þ = 1280
100
r2 – r1= 3 cm
Year 2009
Þ r2 = r1 + 3 = 24 + 3 = 27 cm
22000 ´ 9
pr22 r22 272
81 Þ = 1980
\ Required ratio = = = = 100
2 2 2 64
pr1 r1 24 Year 2012
So data in both the statements together are necessary 26000 ´ 7
to answer the question Þ = 1820
100
74. (e) From both the statements,
So in the year 2009 maximum number of defective
If the length of the train be y metre, toys of type A manufactured
y 5y 18000 ´ 6 18000 ´ 6
Speed of train = m/sec = kmph 80. (e) Required ratio = : =1:1
9 9 ´ 18 100 100
æ 5y ö 15 y + 100 81. (a) Required ratio = 40 : 45 = 8 : 9
Again, ç + 60 ÷ ´ =
è 9 ´ 18 ø 60 ´ 60 1000 30
82. (b) Required percentage = ´ 100
æ 5y ö 15 y + 100 45 + 30 + 45
Þ ç + 60 ÷ ´ = … (i)
è 162 ø 36 10 30
= ´ 100 = 25%
It is to be noted that when a train crosses a pole, the 120
distance covered = length of the train, When it 83. (c) Required ratio = (30 + 35 + 40) : (60 + 30)
crosses another train, = 105 : 90 = 7 : 6
Relative speed × time = sum of lengths of both trains. 84. (d) For vehicles of type A :
Hence, from equation (i), we can determine length of Year, 2007 Þ From 45 thousand to 60 thousand
train.
Year, 2011 Þ From 30 thousand to 70 thousand
75. (e) From statement II,
length = x cm, breadth = (x – 5) cm 50
85. (e) Required percentage = ´ 100 » 26
\ 2(x + x – 5) = 50 195
Þ 2x – 5 = 25 86. (a) Number of candidates who applied for Science
Þ 2x = 25 + 5 = 30 22
Þ x = 15 cm = length = 88000 × = 19360
100
\ Breadth = 15 – 5 = 10cm
Number of candidates selected for Science
From statement I,
24
pr 2 = 14400 × = 3456
= 15 × 10 100
2
Required difference = 19360 – 3456 = 15904
Þ pr2 = 300
87. (b) Number of candidates who applied for Arts
300 17
Þ r= = 88000 × = 14960
p 100
This gives us value of radius, Number of selected candidates in Maths and English
76. (a) Required number of defect free toys
38
21000 ´ 95 15000 ´ 92 18000 ´ 94 = 14400 × = 5472
= + + 100
100 100 100 Required sum = 14960 + 5472 = 20432
= 19950 + 13800 + 16920 = 50670
y
o
u
rs
m
336 SBI Management Executive Exam 2014

a
h
88. (c) Required ratio = (14 + 20) : (28 + 24) = 34 : 52 = 17 : 26 102. (d) It is mentioned in the last few lines of the first

b
o
paragraph that India has emerged as a strong

o
24

b
89. (d) Number of applicants for English = 88000 × potential market for investments in training and

.w
100 education sector due its favorable demographics

o
= 21120 (young population) and being a service driven

rd
economy.
14400 ´ 28

p
re
Number of selected candidates = = 4032 103. (b) The meaning of education explosion is mentioned in
100

s
the second paragraph. It means a tremendous rise in

s
.c
4032 the number of educational institutions and students.
´ 100 » 19

o
Required percentage = 104. (c) Option (c) can be inferred from the last few lines of the

m
21120
second paragraph where it says that unemployment
1 14400 ´ 63 problem in India cannot be blamed on the availability
90. (e) Required average = ´ = 3024
3 100 of large masses of educated people in India.
105. (c) Option (c) can be inferred from the first few lines of
1æ 5 7 8ö the last paragraph where it says that correspondence
91. (a) Required average = ç 960´ +1840´ +1600´ ÷
3è 8 16 25 ø education has opened new vistas for the education
system which could not be met earlier because of the
1 1 challenges of providing necessary infrastructure for it.
= (600 + 805 + 512) = × 1917 = 639
3 3 106. (c) Option (a) and (b) are clearly mentioned in the
3 paragraphs. There is tota contradiction to pint (c) in
92. (b) Required number of shirts = 1120 × = 420 the first few lines of the second last paragraph where
8 it says that uncertainty and vacillation have marked
19 5 the government policy towards medium of education
93. (c) Required number of trousers = 1720´ +1120´ in India.
43 8
107. (d) Dilution means weakening in force, content or value.
= 760 + 700 = 1460 Diminution means reduction in size, content ,
94. (d) Number of shirts manufactured by Company Q importance.
8 108. (b) Emergence means the process of coming into
= 1600 × = 512 existence, prominence, visibility.
25
109. (d) Deprecate means to express disapproval. Commend
512 means to praise, approve.
Required percentage = ´ 100 = 32%
1600 110. (d) Purport as used as a verb means to claim, profess,
Alternatively pretend falsely. Intention in general is clear as expressed.
111. (c) 112. (b)
8 8
Required percentage = ´ 100 = ´ 100 = 32% 113. (c) For the candidate.
8 + 17 25 114. (a) 115. (e)
5 1120 ´ 5 116. (b) Are shining in the.
95. (e) Required ratio = 960 ´ : 117. (c) 118. (d)
8 8
= 96 : 112 = 6 : 7 119. (d) Ensure a successful cure.
96. (a) Required difference 120. (a) The Mumbai police has found.
= [(12 + 7 + 6) – (5 + 7 + 5)] thousand 121. (a) Disdain means disrespect, to consider umworthy of
= (25 – 17) thousand = 8 thousand one's consideration or respect. all the others are
97. (b) Required difference = 10 + 7 – 7 = 10 thousand synonyms meaning unfaithfulness.
122. (d) Naïve means unsophisticatedly simple. All the others
10 - 6 are synonyms meaning clever, shrewd.
98. (c) Percentage decrease = ´ 100 = 40%
10 123. (e) Eviscerate means deprive of essential contents. All
99. (d) Required average the others are synonyms meaning infer.
124. (d) Reserved means to keep to oneself. All the others
æ 12 + 15 + 10 + 11 + 13 + 8 ö are synonyms meaning confused.
= ç ÷ thousand
è 6 ø 125. (d) Lurid means unpleasantly bright in color, shocking,
69 sensational, horrifying. All the other words have
= ´ 1000 = 11500 positive connotations related to beauty pageant or
6 contest.
100. (e) Required answer = 17000 126. (d) 127. (d) 128. (b) 129. (c) 130. (c)
101. (d) It is mentioned in the last few lines of the second 131. (e)
last paragraph that many state governments have 132. (c) Outcome. 133. (b) Patriarchial.
decided in principle to switch over to the regional 134. (a) Combined. 135. (d) Absolutely.
language as the medium of instruction in colleges 136. (c) 137. (b) 138. (d) 139. (a) 130. (b)
and universities. However the implementation in this 141. (b) 142. (d) 143. (d) 144. (c) 145. (c)
regard has been slow. 146. (c) 147. (d) 148. (c) 149. (a) 150. (c)
y
o
u
rs
m
a
h
b
o
SBI SPECIALIST OFFICER ONLINE

o
b
.w
o
EXAM 2014 Based on Memory

rd
p
re
s
s
.c
REASONING ABILITY

o
m
DIRECTIONS (Qs. 1-7) : Study the following information carefully and answer the questions given below :
A word and number arrangement machine when given an input line of words and numbers rearranges them following a particular rule
in each step. The following is an illustration of input and various steps of rearrangement. (All the numbers are two digit numbers).

Input : s creen 31 award 93 b lin d 57 tro op 19


Step I : 93 s creen 31 award b lin d 57 tro op 19
Step II : 93 award s creen 31 b lin d 57 tro op 19
Step III : 93 award 57 s creen 31 blin d tro op 19
Step IV : 93 award 57 b lin d s creen 31 tro op 19
Step V : 93 award 57 b lin d 31 s creen tro op 19
Step VI : 93 award 57 b lin d 31 s creen 19 troo p

And Step VI is the last step of the rearrangement as the desired arrangement is obtained.
As per rules followed in the above steps, find out in each of the questions the appropriate step for the given input.
Input : vital 54 cards 72 help 24 wall 66 lamp 49
1. How many steps would be required to complete the 7. Which word/number would be at the seventh position from
rearrangement ? the right end in the Step VI ?
(a) Eight (b) Six (a) 54 (b) lamp
(c) Seven (d) Nine (c) cards (d) help
(e) Cannot be determined (e) 66
2. How many elements (words/ numbers) are there between 8. What will come at the place of question-mark (?) in the
"lamp" and "66" as they appear in Step V ? following alphabetical series ?
(a) Five (b) Six AG IN PT ?
(c) Four (d) Seven (a) VZ (b) VY
(e) Three (c) WZ (d) UX
3. Which step number is the following output ?
(e) UY
72 cards 66 help 54 lamp vital 24 wall 49
9. What will come at the place of question-mark (?) in the
(a) Step III (b) Step IV
following number series ?
(c) Step V (d) Step VI
12 34125341 2563 4125 6?
(e) There is no such step
(a) 6 (b) 5
4. Which word/number would be to the immediate right of the
(c) 7 (d) 3
fourth element from the right end in Step III ?
help 24 wall 66 lamp 49 (e) 2
(a) help (b) wall 10. Four of the following five are alike in a certain way and
(c) 24 (d) lamp hence form a group. Which is the one that does not belong
(e) 54 to the group ?
5. In the final step "66" is related to "lamp" and "72" is related (a) JN (b) CG
to "help" in a certain way, then "cards" is related to (c) WZ (d) LP
(a) vital (b) 66 (e) FT
(c) 49 (d) wall DIRECTIONS (Qs. 11-17) : Study the following information
(e) 54 carefully and answer the questions given below : P, Q, R, S. T, U,
6. At what position of "wall" would come from the left end in V and W live on different floors in the same building having
the Step V? eight floors numbered one to eight (the ground floor is numbered
(a) Ninth (b) Eighth 1, the floor above it. Number 2 and so on and the topmost floor is
(c) Sixth (d) Seventh
numbered 8).
(e) Fifth
y
o
u
rs
m
338 SBI Specialist Officer Online Exam 2014

a
h
b
There are only two floors between the floors on which P and R (20-21) : Statements

o
live. R lives on an odd numbered floor. There are four floors O = L < T > S; L > I; T < Z

o
b
between the floors on which P and W live. T lives on a floor 20. Conclusions

.w
immediately above the floor on which U lives. There are only two I. I < O II. Z> S

o
floors between the floors on which V and S live. V lives on a floor 21. Conclusions

rd
above the floor of S. Q does not live on a floor immediately above I. Z > I II. S< I

p
re
or immediately below the floor on which R lives. 22. Statements

s
11. On which of the following floors does Q live ? G < K= O>U> P< S

s
.c
(a) Fifth (b) Third Conclusions

o
m
(c) Second (d) Fourth I. G < U II. S> O
(e) Sixth 23. Statements
12. Who amongst the following live on the floors exactly A > L < P < B; P > M
between S and R? Conclusions
(a) V, W and Q (b) V, U and W I. B > M II. A>M
(c) U, V and Q (d) T, U and Q (24-25) : Statements
(e) U, Q and W I > M = P > R; B < P > A
13. Who amongst the following live on the odd numbered floors 24. Conclusions
excluding R ? I. A < I II. I=A
(a) U, W and S (b) Q, V and U 25. Conclusions
(c) U, T and S (d) T, W and S I. A < R II. B<I
(e) None of these
DIRECTIONS (Qs. 26-31) : Study the following information
14. Who amongst the following lives on the topmost floor ?
carefully and answer the questions given below :
(a) T (b) P
(c) V (d) Q In a certain code language,
(e) W (i) "simple game no rules" is written as "ro sa bi ka".
15. On which of the following floors does R live ? (ii) "no game played now" is written as "ka za bi te".
(a) Fifth (b) First (iii) "why no rules given" is written as "ro fo ce ka".
(c) Seventh (d) Third (iv) "now we were given" is written as "ge te fo li".
(e) Either third or fifth 26. What is the code for "now" ?
16. How many persons live between the floors of T and Q ? (a) fo (b) te
(a) Five (b) Three (c) ge (d) ka
(c) Two (d) Either three or four (e) li
(e) Four 27. What is the code for "game" ?
17. Who amongst the following live on the floor immediately (a) za (b) sa
above the floor of W ? (c) ro (d) bi
(a) U (b) R (e) ka
(c) V (d) T 28. Which of the following is coded as 'ce' ?
(e) None of these (a) why (b) no
18. In a certain code language INSTEAD is written as HMRUFBE (c) rules (d) given
and PHRASED is written as OGQBTFE. How will DOUBLES
(e) Cannot be determined
be written in the same code language ?
29. What is the code for "rules" ?
(a) CNTAMFT (b) EPVAMFT
(a) fo (b) sa
(c) EVPCKDR (d) CNTCMFT
(c) ro (d) ka
(e) CNTCKDR
(e) bi
19. Four of the following five are alike in a certain way and
30. Which of the following is coded as 'fo' ?
hence form a group. Which is the one that does not belong
(a) why (b) given
to the group ?
(a) PTSQ (b) UYXV (c) we (d) rules
(c) INMJ (d) KONL (e) were
(e) DHGE 31. Which of the following represents "we played simple" ?
(a) ge sa za (b) za sa li
DIRECTIONS (Qs. 20-25) : In these questions relationship (c) te sa bi (d) li za ro
between different elements is shown in the statements. The (e) Cannot be determined
statements are followed by two conclusions. 32. If it is possible to make only one meaningful English word
Give answer (a) if only Conclusion I is true. with the second, the fifth, the seventh and the ninth letters
Give answer (b) if only Conclusion II is true. of the word SUBSTANTIAL, which of the following will be
Give answer (c) if either Conclusion I or Conclusion II is true. the second letter of that word ? If no such word can be
Give answer (d) if neither Conclusion I nor Conclusion II is true. formed, give '#' as the answer and if more than one such
Give answer (e) if both Conclusions I and II are true. word can be formed, give '@' as the anwer.
y
o
u
rs
m
SBI Specialist Officer Online Exam 2014 339

a
h
b
(a) U (b) T 40. Conclusions:

o
(c) N (d) # I. At least some districts are towns.

o
b
(e) @ II. All towns are villages.

.w
33. How many such pairs of letters are there in the word

o
DIRECTIONS (Qs. 41-47) : Study the following information
CHARGED, each of which has as many letters between them

rd
carefully and answer the questions given below :
in the word as in the English alphabetical series (in both

p
re
forward and backward directions) ? Seven persons - I, J, K, L, M, N and O — are sitting in a circle at

s
(a) None (b) One equidistance but not necessarily in the same order. Some of them

s
.c
(c) Two (d) Three are facing towards the centre and some are facing outside the

o
m
(e) More than three centre. O is facing outside. J sits second to the right of O. N is
34. Each consonant in the word BLARING is replaced with the sitting second to the left of P. P is not an immediate neighbour of
previous letter and each vowel is replaced with next letter in J or O. N is not an immediate neighbour of J. The immediate
the English alphabetical series and the new letters are neighbour of K faces towards the centre. K and P face the same
rearranged alphabetically. Which of the following will be direction. I is sitting second to the right of N. The immediate
the third letter from the right end ? neighbour of M faces outside. I and M face the same direction as
(a) K (b) B that of J.
(c) M (d) J 41. Who amongst the following are not facing the centre ?
(e) F (a) K, N, O and J (b) I, M, J and P
35. B is 7 metres away in the north of A. A moves 9 metres (c) J, M, I, O and N (d) P, K, O and N
towards east from the starting point, takes a right turn and (e) None of these
walks 2 metres, then takes a left turn and moves 1 metre and 42. Which of the following is the J's position with respect to N ?
finally he takes a left turn and moves 9 metres. Now. how far (a) Third to the right (b) Fourth to the right
and in which direction is A from the B? (c) Third to the left (d) Second to the left
(a) 5 metres East (b) 10 metres West (e) Fifth to the left
(c) 5 metres West (d) 10 metres East 43. Which of the following is the K's position with respect to M ?
(e) None of these (a) Third to the left (b) Third to the right
(c) Fourth to the left (d) Second to the right
DIRECTIONS (Qs. 36-40) : In each question below are two or
(e) None of these
three statements followed by two conclusions numbered I and II.
44. Who among the following is sitting exactly between M and K?
You have to take the given statements to be true even if they seem
(a) N (b) P
to be at variance from commonly known facts and then decide
(c) O (d) I
which of the given conclusions logically follows from the given
(e) J
statements disregarding commonly known facts.
45. Which of the following is the P's position with respect to J ?
Give answer (a) if only conclusion I follows. (a) Third to the right (b) Fourth to the left
Give answer (b) if only conclusion II follows. (c) Fourth to the right (d) Third to the left
Give answer (c) if either conclusion I or II follows. (e) Fifth to the left
Give answer (d) if neither conclusion I or II follows. 46. How many persons are sitting between M and O if we move
Give answer (e) if both conclusions I and II follow. clockwise starting from M ?
(36-37) : Statements : (a) None (b) Two
All crops are fields. No yield is a field. (c) Three (d) One
All fields are harvests. (e) Four
36. Conclusions: 47. Which of the following statements is true regarding the
I. No field is a crop. seating arrangement ?
II. All crops being harvest is a possibility. (a) There are only two persons seated between K and I
37. Conclusions: (b) M, K and I are seated adjacent
I. All harvests being yield is a possibility. (c) N, P and J are seated adjacent
II. All harvests are fields. (d) Three persons are facing the centre
38. Statements : (e) One of the neighbours of I is K
Some trades are exports. All businesses are trades. 48. Statement: Many major auto makers will increase the prices
Conclusions: of their vehicles by 7 percent from next month, mainly to
I. At least some businesses are exports. offset higher input and operational costs.
II. All businesses being exports is a possibility. Which of the following arguments would weaken the facts
(39-40) : Statements : given in the statement ?
Some countries are towns. (a) The largest auto maker in the country, Verontee had
All countries are districts. failed to make any profit when prices for its vehicles
All districts are villages. were increased by almost 4 per cent
39. Conclusions: (b) Operational and input costs continue to be 85 per cent
I. At least some towns are villages. of the total costs of the vehicles
II. All countries are villages.
y
o
u
rs
m
340 SBI Specialist Officer Online Exam 2014

a
h
b
(c) Both input and operational costs were gradually 51. How is H related to B ?

o
increased by the government to avoid sudden transit I. H is married to P. P is the mother of T. T is married to D.

o
b
losses to the companies D is the father of B.

.w
(d) At any given time, a price rise of more than 5 per cent II. B is the daughter of T. T Is the sister of N. H is the

o
definitely leads to substantial decrease in sales of the father of N.

rd
vehicles 52. Among five persons D, E, F, G and H each of whom having

p
re
(e) Rather than increasing profit, 7 per cent price rise would different height, who is the second tallest ?

s
bring it at the same level which existed before the rise I. D is taller than only G and E. F is not the tallest.

s
.c
in input/operational costs. II. H is taller than F. G is taller than E but shorter than D.

o
53. In which direction is J with respect to R ?

m
DIRECTIONS (Qs. 49-50) : Study the following information I. R is to the west of P. P is to the south of G. J is to the
carefully and answer the questions given below : west of G. When R, P, G and J are joined by straight
Four Management Institutes have decided to conduct a Common lines, a square is formed.
Admission Test for selecting suitable candidates for the new II. K is to the south of J. H is to the east of K. R is to the
session. north-west of H. When R, K and H are joined by straight
lines, a right angled triangle is formed.
49. Which of the following can be possible consequence(s) of
54. What is the position of K from the right end in a row ?
this decision ?
I. There are even number of students in the row. There
(A) The aspirants would take only one test in a year. are not more than 11 students in the row.
(B) The Common Admission Test would save the II. C stands at the extreme left end of the line. There are
application fee. five students between C and B. O is third to the right of
(C) Now the aspirants will have only one attempt to prove B. K is not at the extreme end. There is only one student
themselves. between O and K.
(D) Only the top most Institute will receive more number 55. On which day of the week did Arvind visit Mumbai ?
of applications if aspirants fill up different forms. I. Arvind visited Mumbai after Monday but before
(E) A large number of aspirants would be selected as there Thursday but not on an odd day of the week.
are more Institutes to accommodate them. II. Arvind visited Mumbai before Friday but after Monday.
(F) It is not possible to allot Institutes to the selected DIRECTIONS (Qs. 56-59) : Study the following information
candidates as per their preferences. carefully and answer the questions given below :
(a) Only (B), (C) and (F) (b) Only (A), (C) and (F)
(c) Only (A), (B) and (E) (d) Only (C), (E) and (F) In a certain code language :
"can we skip" is written as "& 4 2".
(e) Only (B), (C) and (D)
"we skip this" is written as "$ & 2".
50. Which of the following can be the probable cause for this
"that was part" is written as "9 3 5".
decision ? "skip that part" is written as "3 & 9".
(A) To select the most suitable candidates 56. What is the code for 'that' ?
(B) To streamline the selection process (a) 3 (b) 9
(C) To concede the demands of aspirants (c) 5 (d) &
(a) Only (A) and (B) (b) Only (B) and (C) (e) Either 3 or 9
(c) Only (C) (d) Only (A) and (C) 57. Which of the following representes 'we skip this' ?
(e) All (A), (B) and (C) (a) 3 & 4 (b) $ 2 4
(c) $ 2 & (d) & 2 3
DIRECTIONS (Qs. 51-55) : Each of the questions below consists (e) None of these
of a question and two statements numbered I and II given below 58. What is the code for 'skip' ?
it. You have to decide whether the data provided in the statements (a) & (b) $
are sufficient to answer the question. Read both the statements (c) 4 (d) 2
and (e) 9
Give answer (a) if the data in Statement I alone are sufficient to 59. Which of the following represents 'can skip that part' ?
answer the question, while the data in Statement II alone are not (a) 2 & 4 9 (b) 3 4 9 &
sufficient to answer the question. (c) 3 5 & 2 (d) 2 & 9 3
(e) 5 3 $ 9
Give answer (b) if the data in Statement II alone are sufficient to
answer the question, while the data in Statement I alone are not DIRECTIONS (Qs. 60-65): Study the following information
sufficient to answer the question. carefully and anaswer the questions given below :
Give answer (c) if the data in Statement I alone or in Statement II
Eight persons - P, Q, R, S, T, U, V and W — are standing in a
alone are sufficient to answer the question.
straight line at equidistance. Some of them are facing North while
Give answer (d) if the data in both the Statements I and II together
some others are facing South. R is third to the left of W. W is
are not sufficient to answer the question.
facing North. T is third to the right of W. S is second to the left of
Give answer (e) if the data in both the Statements I and II together
Q. Q is not an immediate neighbour of W or R. Both the immediate
are necessary to answer the question.
neighbours of P face South. The immediate neighbour of V faces
y
o
u
rs
m
SBI Specialist Officer Online Exam 2014 341

a
h
b
opposite direction. Immediate neighbours of U face opposite 67. Which of the following two persons visit Mumbai ?

o
direction (If U is facing North, immediate neighbour of U face (a) Q and V (b) R and S

o
b
South and vice-versa). V faces just opposite of Q. U is not an (c) Q and S (d) R and U

.w
immediate neighbour of P and faces opposite direction with (e) Q and U

o
respect to that of P (If U faces South P faces North and vice-vera). 68. T belongs to which department?

rd
T faces the same direction as that of V. (a) Marketing (b) Sales

p
re
60. Who among the following are facing North ? (c) Human Resources (d) Finance

s
(a) P, V, W and Q (b) R. W, U and Q (e) Cannot be determined

s
.c
(c) U, S, T and Q (d) P, W, S and Q 69. Who among the following belongs to the Sales Department?

o
m
(e) None of these (a) Q and Z (b) R and V
61. Who among the following is to immediate left of S ? (c) Q and V (d) R and Z
(a) U (b) T (e) U and Z
(c) V (d) W 70. Which of the following cities is visited by S ?
(e) None of these (a) Mumbai (b) Kolkata
62. How many persons are there between P and U ? (c) Delhi (d) Chennai
(a) Three (b) Four (e) Either Chennai or Delhi
(c) Two (d) One
(e) There is no person ENGLISH LANGUAGE
63. Who among the following are standing on the extreme ends
of the line ? DIRECTIONS (Qs. 71-80) : Which of the phrases at places (a),
(a) P and Q (b) R and Q (b), (c) and (d) given below each sentence should replace the
(c) U and R (d) R and S phrases printed in bold in the following sentences to make the
(e) None of these sentence grammatically correct. If the sentence is correct as it is
64. Who among the following is third to the right of U ? and 'No correction is required', mark (e) as the answer.
(a) P (b) V 71. People in underdeveloped countries are distressing
(c) R (d) Q because of the antagonistic attitude of developed countries.
(e) None of these (a) have been distressing (b) are distressed because
65. Who among the following are the immediate neighbours of (c) are distressed at (d) were distressing by
V? (e) No correction required
(a) R and W (b) P and U 72. He had been behaved impolitely and suffered owing to that.
(c) R and S (d) P and W (a) was behaved (b) had behaved
(e) None of these (c) have been behaved (d) would have behaved
(e) No correction required
DIRECTIONS (Qs. 66-70): Study the following information 73. It has become a commonly practice to talk about women
carefully and answer the questions given below : liberation.
(a) commonly practised talk about
Eight persons — P, Q, R, S, T, U, V and Z – are going to the four (b) common practice to talk about
destinations viz. Chennai. Delhi. Mumbai and Kolkata and they (c) common practice of talking with
represent four different departments viz., Human Resources, Sales, (d) commonly practising to talk about
Finance and Marketing but not necessarily in the same order. At (e) No correction required
least two persons belong to the same department and at least two 74. He is so brisk himself that he cannot tolerate any efficiency.
of them visit the same destination. (a) tolerate hardly any inefficiency
P belongs to the Finance Department and he visits the same city (b) hardly tolerates lethargy
as that of V. V does not belong to the Finance, Marketing or (c) tolerate any haste
Human Resources Department. V does not visit Kolkata or (d) tolerate any delay
Mumbai. R visits Chennai and belongs to the Human Resources (e) No correction required
Department. The person who belongs to the Finance Department 75. No person with a reasonably self-esteem would ever like to
succumb to any pressure.
other than P visits Chennai. Q and V belongs to the same
(a) reasonable self-esteemed
Department. U visits Mumbai and he does not belong to Sales
(b) reasonable self-esteemed
and Marketing Department. The two persons who belong to the (c) reasonably self-esteemed
Marketing Department visit Kolkata. S does not belong to the (d) reasonably a self esteem
Marketing Department. (e) No correction required
66. Which of the following combinations of Person – City – 76. Unless you dedicated to your studies, you cannot expect
Destination is correct ? success.
(a) P – Mumbai – Finance (a) Unless you donot dedicate to
(b) R – Chennai – Human Resources (b) If you dedicate to
(c) T – Kolkata – Marketing (c) In case you didnot dedicate to
(d) S – Chennai – Sales (d) Unless you dedicate to
(e) No correction required
(e) U – Delhi – Marketing
y
o
u
rs
m
342 SBI Specialist Officer Online Exam 2014

a
h
77. The output of our factory was not been satisfactory

b
Four years ago, some of us watched with a mixture of incredulity

o
monitored. and horror as elite discussion of economic policy went completely

o
b
(a) was not being satisfactory off the rails. Over the course of just a few months, influential

.w
(b) has not been satisfactorily people all over the Western world convinced themselves and

o
(c) has not being satisfactorily each other that budget deficits were an existential threat, trumping

rd
(d) is not being satisfactory any and all concern about mass unemployment. The result was a

p
re
(e) No correction required turn to fiscal austerity that deepened and prolonged the economic

s
78. He is firmly believing on democratic principles. crisis, inflicting immense suffering.

s
.c
(a) firm believer for (b) firmly believe in And now it's happening again. Suddenly, it seems as if all

o
(c) firmly believes in (d) firmly belief on

m
the serious people are telling each other that despite high
(e) No correction required unemployment there's hardly any "slack" in labour markets —
79. He persevered and succeeded to face of all the obstacles. as evidenced by a supposed surge in wages — and that the
(a) to face of all the (b) to all the face of Federal Reserve needs to start raising interest rates very soon to
(c) in the face of all (d) at the face of the all
head off the danger of inflation.
(e) No correction required
To be fair, those making the case for monetary tightening
80. He always claims to be superior than everyone in his
are more thoughtful and less overtly political than the archons of
company.
(a) claim to be superior to austerity who drove the last wrong turn in policy. But the advice
(b) claims to be superior to they're giving could be just as destructive.
(c) claimed to superiority over O.K., where is this coming from?
(d) claims superiority for The starting point for this turn in elite opinion is the
(e) No correction required assertion that wages, after stagnating for years, have started to
rise rapidly. And it's true that one popular measure of wages has
DIRECTIONS (Qs. 81-90) : Read each sentence to find out indeed picked up, with an especially large bump last month.
whether there is any grammatical error in it. The error if any, But that bump is probably a snow-related statistical illusion.
will be in one part of the sentence, the number of that part is the As economists at Goldman Sachs have pointed out, average
answer. If there is no error, mark (e). (Ignore errors of wages normally jump in bad weather — not because anyone's
punctuation, if any) wages actually rise, but because the workers idled by snow and
81. They started to scratch (a)/ a few years ago and leveraged storms tend to be less well-paid than those who aren't affected.
(b)/ the mobile services boom (c)/ to become key market Beyond that, we have multiple measures of wages, and only
players. (d)/ No error (e) one of them is showing a notable uptick. It's far from clear that
82. Private banks and financial services firms (a)/ too had a the alleged wage acceleration is even happening.
reasoning quarter (b)/ given an apparent (c) / slowdown in And what's wrong with rising wages, anyway? In the past,
the economy. (d)/ No error (e) wage increases of around 4 percent a year — more than twice the
83. Air India is considering (a)/ selling of its properties (b)/ in current rate — have been consistent with low inflation. And there's
foreign destinations (c)/ because of parting of asset a very good case for raising the Fed's inflation target, which would
monetisation to garner resources. (d)/ No error (e) mean seeking faster wage growth, say 5 percent or 6 percent per
84. The law will prevail over (a)/ a standard (b)/ clause unilateral year. Why? Because even the International Monetary Fund now
(c)/ printed on a bill.(d)/ No error (e) warns against the dangers of "lowflation": too low an inflation
85. Industry experts points out (a)/ that there is an urgent need rate puts the economy at risk of Japanification, of getting caught
(b)/ to overhaul the patent filing (c)/ and generation process. in a trap of economic stagnation and intractable debt.
(d)/ No error (e) Over all, then, while it's possible to argue that we're running
86. A 24-year-old metallurgical engineer (l)/who lost his leg (b)/ out of labour slack, it's also possible to argue the opposite, and
in a road accident, awarded (c)/ ` 30 lakh as a compensation. either way the prudent thing would surely be to wait: Wait until
(d)/ No error (e) there's solid evidence of rising wages, then wait some more until
87. It is imperative to (l)/clear communication (b)/priority areas wage growth is at least back to precrisis levels and preferably
(c)/and a credible road-map for deficit reduction. (d)/ No
higher.
error (e)
Yet for some reason there's a growing drumbeat of demands
88. There are certain aspects for (a)/ modern life that are (b)/ a
that we not wait, that we get ready to raise interest rates right
special challenge (c)/ to traditional table manners. (d)/ No
error (e) away or at least very soon. What's that about?
89. It will be (a)/ a terrible mistake (b)/ for them to ascribe any Part of the answer, is that for some people it's always 1979.
victory (c)/ to their popularity.(d)/ No error (e) That is, they're eternally vigilant against the danger of a runaway
90. Tornadoes ripped through the south-central US (a)/ killing wage-price spiral, and somehow they haven't noticed that nothing
at least 100 people (b)/ and wiped out (c)/ entire like that has happened for decades. Maybe it's a generational
neighbourhoods. (d)/ No error (e) thing. Maybe it's because a 1970s-style crisis fits their ideological
preconceptions, but the phantom menace of stagflation still has
DIRECTIONS (Qs. 91-105): Read the following passage an outsized influence on economic debate.
carefully and answer the questions given below it. Certain words/ Then there's sado-monetarism: the sense, all too common
phrases have been printed in bold to help you locate them while in banking circles, that inflicting pain is ipso facto good. There
answering some of the questions. are some people and institutions — for example, the Basel-based
y
o
u
rs
m
SBI Specialist Officer Online Exam 2014 343

a
h
b
Bank for International Settlements — that always want to see 95. What, according to the passage, sadomonetarism interprets?

o
interest rates go up. Their rationale is ever-changing — it's (a) Inflicting pain is not good.

o
b
commodity prices; no, it's financial stability: no, it's wages — but (b) Inflicting gain is painful.

.w
the recommended policy is always the same. (c) Inflicting higher taxes is good for economic health.

o
Finally, although the current monetary debate isn't as openly (d) Inflicting pain is ipso facto good.

rd
political as the previous fiscal debate, it's hard to escape the (e) None of these

p
re
suspicion that class interests are playing a role. A fair number of 96. How does the writer see the rising wages ?

s
commentators seem oddly upset by the notion of workers getting (a) He sees it as a threat to economy

s
.c
raises, especially while returns to bondholders remain low. It's (b) He sees it as a development to cheer and promote, not

o
a threat to be squashed

m
almost as if they identify with the investor class, and feel uncom-
fortable with anything that brings us close to full employment, (c) He sees it as an obstacle to development
and thereby gives workers more bargaining power. (d) He is of the view that un-employment will grow in-
Whatever the underlying motives, tightening the monetary tensely
screws anytime soon would be a very, very bad idea. We are (e) None of these
slowly, painfully, emerging from the worst slump since the Great 97. Bank for International Settlements is located in
Depression. It wouldn't take much to abort the recovery, and, if (a) Basel (b) Tokyo
that were to happen, we would almost certainly be Japanified. (c) London (d) New York
stuck in a trap that might last decades. (e) Berne
Is wage growth actually taking off? That's far from clear. 98. What is the central idea of the passage ?
(a) Rising wages is a bad omen for development
But if it is, we should see rising wages as a development to cheer
(b) Rising wages is a good omen for development
and promote, not a threat to be squashed with tight money.
(c) Lowering wages is good for a developing economy
91. As mentioned in the passage, at what point the influential
(d) Stagnation is an existing feature of world economy
people of the western world got convinced four years ago ?
(e) None of these
(a) Budget deficits are an existing threat, trumping any
and all concern about mass unemployment DIRECTIONS (Qs. 99-102) : Choose the word/group of words
(b) Budget deficits are non existing threat and there will which is most similar in meaning to the word/group of words
be mass employment soon. printed in bold as used in the passage.
(c) Budget deficits decline rapidly and give sign of
99. Head off
flourishing economy. (a) promote (b) act to prevent
(d) There is good sign of employment, despite long (c) encourage (d) feel headache
standing budget deficits. (e) ameliorate
(e) None of these 100. Uptick
92. Why the serious people seemingly are of the opinion that (a) small increase (b) small decrease
despite high unemployment there is hardly any slack in (c) urchin (d) upwind
labour markets ? (e) downtrend
(a) They have evidenced a supposed surge in wages 101. Phantom
(b) Federal Reserve needs to start raising interest rates (a) real (b) true
(c) A decline in wages has been expected (c) quantitative (d) imaginary
(d) Demand for labourers has increased (e) optimum
(e) None of these 102. Slump
93. According to an economist at Goldman Sachs, average (a) depression (b) incline
wages normally jump in bad weather because (c) increase (d) inflate
(a) workers are paid hand-somely in bad weather (e) insult
(b) the workers idled by snow and storms tend to be less
DIRECTIONS (Qs. 103-105) : Choose the word/group of words
well paid than those who are not affected
which is most opposite in meaning to the word/group of words
(c) Workers in bad weather are humanly treated and paid
printed in bold as used in the passage.
well at home
(d) Workers idled by storm tend to be more well-paid than 103. Slack
those who aren't affected (a) lull (b) lukewarm
(e) None of these (c) quiet period (d) period of activity
94. What does the writer mean by using the term risk of (e) lump
Japanification' ? 104. Austerity
(a) Getting caught in economic inflation (a) simplicity (b) abstinence
(b) Getting encouraging growth like Japan (c) hardness (d) luxury
(c) Getting caught in a trap of economic stagnation and (e) lustre
intractable debt 105. Incredulity
(d) Getting growth rate of 5 per cent or 6 per cent per (a) disbelief (b) belief
annum like Japan. (c) incrustation (d) convenience
(e) None of these (e) credible
y
o
u
rs
m
344 SBI Specialist Officer Online Exam 2014

a
h
b
DIRECTIONS (Qs. 106-115) : In the following passage there 112. (a) deemed (b) deeming

o
(c) dreaming (d) dreamt

o
are blanks, each of which has been numbered. These numbers

b
are printed below the passage and against each, five words are (e) deemful

.w
suggested, one of which fits the blank appropriately. Find out 113. (a) have (b) has

o
(c) has been (d) have been

rd
the appropriate word in each case.
(e) would be

p
re
India with its 600 universities and 35,000 colleges is the third 114. (a) for (b) in

s
(106) education system in the world. But 68 per cent of these (c) with (d) to

s
.c
universities and 73 per cent of colleges are (107) to be medium or (e) by

o
low quality by Indian standard itself. With a severe shortage of

m
115. (a) realise (b) realising
quality higher education institutions, India faces the challenge of (c) understand (d) achieving
educating nearly 30 per cent of its 1.1 billion people. Certainly. (e) realised
India requires many more institutions of higher (108) since nearly
100 million students are going to the higher education market in DIRECTIONS (Qs. 116-120) : Rearrange the following six
the next 10 years. In such a scenario improving the standards of sentences (A), (B), (C), (D), (E) and (F) in the proper sequence
(109) universities and establishing quality institutions should be to form a meaningful paragraph; then answer the questions
our priority. given below them.
Steps should be (110) to reduce the disconnect between (A) A study commissioned by a trade group, the National
higher education institutions and industry requirements. The Association of Software and Service Companies, or
possibilities of private sector contributions in higher education Nasscom, found only one in four engineering graduates to
should be fully exploited. The practice of big business houses be employable.
and philanthropists promoting quality universities in the west (B) India still produces plenty of engineers, nearly 400,000 a
could be (111) in India. But before initiating such private sector year at last count.
participation in higher education the government should ensure (C) The skills gap reflects the narrow availability of high-quality
that there are proper checks and controls on the activities of such college education in India and the galloping pace of country's
entities. Also it is our recent experience that the mushrooming of service-driven economy, which is growing faster than nearly
substandard schools and (112) universities (113) resulted in the all but China's.
deterioration of educational standards in India. (D) The best and most selective universities generate too few
Upgrading the higher education sector especially in terms graduates, and new private colleges are producing graduates
of patents filed and cutting edge research output is essential in of uneven quality.
improving India's global ranking. Our sustainable economic (E) The rest were deficient in the required technical skills,
development depends on human resources. A quality higher fluency in English or ability to work in a team or deliver
education base is essential (114) reap full benefits of our basic oral presentations.
demographic dividend. In its mission to redress inefficiencies in (F) But their competence has become the issue.
higher education, the strategies of the 12th plan focus on technical 116. Which of the following should be the SECOND sentence
education, distance learning, quality research, infrastrucrture, after rearrangement ?
faculty and curriculum content. Wholehearted, co-ordinated (a) F (b) B
efforts of the government departments and the private sector are (c) A (d) C
needed to (115) the plan's educational goals. (e) D
117. Which of the following should be the FIRST sentence after
106. (a) largest (b) large
rearrangement ?
(c) larger (d) bigger
(a) A (b) B
(e) affluent (c) C (d) D
107. (a) founded (b) found (e) E
(c) finding (d) being found 118. Which of the following should be the THIRD sentence after
(e) been founded rearrangement ?
108. (a) learn (b) learning (a) A (b) B
(c) played (d) paid (c) C (d) D
(e) paying (e) F
109. (a) existing (b) existed 119. Which of the following should be the FIFTH sentence after
(c) perished (d) excellent rearrangement ?
(e) exciting (a) A (b) B
(c) C (d) E
110. (a) take (b) took
(e) F
(c) taken (d) stopped 120. Which of the following should be the LAST sentence after
(e) forwarded rearrangement ?
111. (a) repeated (b) replicated (a) A (b) B
(c) copying (d) repeating (c) C (d) D
(e) replicating (e) E
y
o
u
rs
m
SBI Specialist Officer Online Exam 2014 345

a
h
b
o
o
b
.w
o
rd
(1-7) :

p
(11–17):

re
If we examine the pattern of rearrangement carefully. We Floor Number Person

s
would find in the given input and various steps that in 8 P

s
.c
each step one number or one word is getting arranged in 7 T

o
the first step, the highest number is shifting to extreme

m
6 U
left position and in next step the word which comes first
in alphabetical order moves to the second position. 5 R
Pattern in these two steps is repeated till all the numbers 4 V
are arranged in descending order and the words in 3 W
alphabetical order.
2 Q
Input : vital 54 cards 72 help 24 wall 66 lamp 49
Step I : 72 vital 54 cards help 24 wall 66 lamp 49 1 S
Step II : 72 cards vital 54 help 24 wall 66 lamp 49
11. (c) Q lives on the floor number 2.
Step III : 72 cards 66 vital 54 help 24 wall lamp 49
12. (a) V, W and Q live on floors exactly between the
Step IV : 72 cards 66 help vital 54 24 wall lamp 49
floors of S and R.
Step V : 72 cards 66 help 54 vital 24 wall lamp 49
13. (d) T ® Floor Number 7
Step VI : 72 cards 66 help 54 lamp vital 24 wall 49
W ® Floor Number 3
Step VII: 72 cards 66 help 54 lamp 49 vital 24 wall
S ® Floor Number 1
And, Step VII is the last step.
14. (b) P lives on the topmost floor.
1. (c) Seven steps would be required to complete the
15. (a) R lives on the floor number 5.
rearrangement.
16. (e) Four persons U, R, V and W live between the floors
2. (a) There are five elements (help, 54, vital, 24, wall)
of T and Q.
between “66” and “lamp” in the StepV.
17. (c) V lives on the floor immediately above the floor of W.
3. (d) This is Step VI.
4. (b) Fourth element from the right end in the Step III 18. (d) I N S T E A D HMRUFBE
Þ 24 Immediate to the right of “24” Þ wall –1
5. (e) There are two elements between “66” and “lamp" in –1
–1
the final Step. Similarly, there are two elements +1
between ‘72” and “help” in the final Step. +1
So, “cards” would be related to “54”. +1
6. (b) The element “wall” would come at the eighth +1
position from the left end in the Step V.
P HRASE D OGQB T FE
7. (d) The element “help” would be at the seventh position
–1
from the right end in the Step VI. –1
+8 +7
A ¾¾® I ¾¾® P ¾¾® V
+6 –1
8. (b) +1
+7 +6 +5 +1
G ¾¾® N ¾¾® T ¾¾® Y
+1
9. (c) 1 2 3 4 +1
1 2 5 3 4 Therefore.
1 2 5 6 3 4 DOUBLES CNTCMFT
1 2 5 6 7 .... –1
–1
+4 –1
10. (c) J ¾¾® N
+4
+1
C ¾¾® G +1
+4 +1
L ¾¾® P +1
+4
P ¾¾® T +4 –1 –2
19. (c) P ¾¾® T ¾¾ ® S ¾¾® Q
But,
+4 –1 –2
+3 U ¾¾® Y ¾¾ ® X ¾¾® V
W ¾¾® Z
+4 –1 –2
K ¾¾® O ¾¾ ® N ¾¾® L
+4 –1 –2
D ¾¾® H ¾¾ ® G ¾¾® E
y
o
u
rs
m
346 SBI Specialist Officer Online Exam 2014

a
h
now Þ te

b
But, 26. (b)

o
27. (d) game Þ bi

o
+5 –1 –3
I ¾¾® N ¾¾ ® M ¾¾® J

b
28. (a) ce Þ why

.w
(20–21): 29. (c) rules Þ ro

o
O = L £ T > S. 30. (b) fo Þ given

rd
L> I 31. (e) we Þ ‘ge’ or ‘li’

p
T£Z

re
played Þ za

s
O= L£T £Z simple Þ sa

s
.c
Z³T>S So code for ‘we’ is not fixed.

o
O=L>I

m
I<O=L£T>S 32. (c) 1 2 3 4 5 6 7 8 9 10 11
I< O= L£T£Z S U B S T A N T I A L
20. (e) Conclusions: Specified letters Þ U, T, N and I
I. I < O : True Meaningful word Þ UNIT
II. Z > S : True
21. (a) Conclusions:
I. Z > I : True 33. (d) 3 8 1 18 7 5 4
II. S < I : Not True C H A R G E D
22. (d) G £ K= O³ U> P< S
Conclusions:
I. G £ U : Not True So 3 such pairs are there.
II. S > O : Not True 34. (a) B L A R I N G
23. (a) A ³L<P£B –1 –1 +1 –1 +1 –1 –1
P³M
A ³ L<P³M A K B Q J M F
M£P£B Now,
Conclusions:
I. B ³ M : True A B F J K M Q
II. A > M : Not True
(24-25): 3rd from right
I ³M=P>R
B< P³A 35. (d) B
I ³M=P>B
7m

I ³ M= P³A 9m North
A³M=P>R A
B< M= P>R 9m
2m West East
24. (c) Conclusions:
I. A < I: Not True 1m
II. I = A : Not True South
A is either smaller than or equal to I. Required distance = (9 + 1) metres = 10 metres
25. (b) Conclusions: Direction Þ East
I. A £ R : Not True 36 – 37:
II. B < I : True
(26 – 31): Harvest
simple game no rules ® ro sa bi ka Fields
Crops
no game played now ® ka za bi te Yield

Why no rules given ® ro fo ce ka


OR
now we were given ® ge te fo li
Harvest
Codes are
Fields
simple Þ sa now Þ te
game Þ bi why Þ ce Crops
Yield
no Þ ka given Þ fo
rules Þ ro we Þ ge or li
played Þ za were Þ ge or li
y
o
u
rs
m
SBI Specialist Officer Online Exam 2014 347

a
h
b
36. (b) Conclusions : 41. (c) P and K face the centre while N, O, I, J and M face

o
I. False outward.

o
b
II. True 42. (a) J is third to right of N. J is fourth to left of N.

.w
So only conclusion II follows. 43. (d) K is second to the right and fifth to the left of M.

o
44. (b) P is sitting exactly between M and K.

rd
37. (d) Conclusions :
45. (e) P is second to the right and fifth to the left of J.

p
I. False

re
46. (c) Three persons P, K and N are sitting between M and

s
II. False O if we move clockwise from M.

s
.c
So none of the conclusion follows. 47. (a) N and O are seated between K and I.

o
Only two persons, P and K are facing the centre.

m
38. (d) Trades J and O are neighbours of I.
So only statement I is true.
Business 48. (d) Clearly option (4) would contradict the facts given in
Exports statement because profits will reduce alongwith
reduction in sales.
49. (c) Statement (B), (E) and (F) are the possible
consequences because common admission test would
OR
save the money and time of the aspirants. Institutes
have to consider multiple factors for allotment of
Trades seats so statement (F) is also correct.
50. (a) Clearly statement (A) and (B) can be the probable
Business cause of the decision because common admission
Exports test is the most suitable method for selection
process.
51. (c) From statement I
Conclusions : HÛP
I. False (+) (-)
II. False ¯
TÛD
So none of the conclusion follows. ( -) ( + )
¯
(39 – 40): Villages B
Districts Hence, H is the grandfather of B
From statement II
Countries H(+ )
Towns ¯
N « T( -)
¯
B( -)
Hence H is the grandfather of B
39. (e) Conclusions : So data in statement I alone or in statement II alone are
I. True sufficient to answer the question.
II. True 52. (a) From statement I
So both conclusions I and II follow. H > F > D > G , E
40. (a) Conclusions : Clearly, H is the tallest and F is the second tallest.
I. True From statement II
II. False H > F
So only conclusion I follows.
D > G > E
(41 – 47) :
P No answer.
So data in statement I alone are sufficient to answer
M K the question.
53. (c) From statement I
J G North
J NW NE
N
West East
I SE
O R P SW
South
y
o
u
rs
m
348 SBI Specialist Officer Online Exam 2014

a
h
b
It is clear from the diagram that J is to the north of R. 64. (a) P is third to the right of U.

o
From statement II 65. (d) P and W are immediate neighbours of V.

o
b
(66-70):

.w
J Person City Department

o
P Delhi Finance

rd
R

p
Q Mumbai Sales

re
R Chennai Human Resources

s
s
.c
K H S Chennai Finance

o
T Kolkata Marketing

m
It is clear from the diagram that J is to the north of R.
54. (b) From statement I U Mumbai Human Resources
The number of students may be 4, 6, 8 or 10. V Delhi Sales
From statement II Z Kolkata Marketing
C ||||| B K | O 66. (b) The combination R -Chennai - Human Resources is
It is clear that there are 10 students in the row. K is correct.
third from the right.
67. (e) Q and U visit Mumbai.
55. (a) From statement I
68. (a) T belongs to the Marketing Department.
Arvind visited Mumbai on Tuesday or Wednesday.
Wednesday is an odd day. Therefore, Arvind visited 69. (c) Q and V belong to the Sales Department.
Mumbai on Tuesday. 70. (d) S visits Chennai.
From statement II 71. (b) 72. (b) 73. (c) 74. (d)
Arvind visited Mumbai on Tuesday or Wednesday 75. (a) 76. (d) 77. (b) 78. (c)
or Thursday.
(56-59) : 79. (c) 80. (b)
81. (a) They started from scratch.
can we skip & 4 2
82. (b) Too had a reasonabl equarter.
we skip this $ & 2 83. (c)
that was part 9 3 5 84. (c) Clause unilaterally.
85. (a) Industry experts point out.
skip that part 3 & 9 86. (c) In a road accident, was awarded.
Codes are 87. (e)
can Þ 4 was Þ 5
88. (a) Certain aspects of.
we Þ 2 part Þ 9 or 3
skip Þ & 89. (e)
this Þ $ 90. (d) Entire neighbourhood.
that Þ 9 or 3 91. (a) It is mentioned in the first paragrpah.
56. (e) that Þ 3 or 9 92. (a) It is mentioned in the second paragrpah.
57. (c) we Þ 2 93. (b) It is mentioned in the sixth paragrpah.
skip Þ &
94. (c) It is mentioned in the eighth paragrpah.
this Þ $
95. (d) It is mentioned in the twelth paragrpah.
58. (a) skip Þ &
59. (b) can Þ 4 96. (b) It is mentioned in the last paragrpah.
skip Þ & 97. (a) It is mentioned in the twelth paragrpah.
that part Þ 3 or 9 98. (b) It can be clearly inferred from the entire passage
(60-65): especially the last paragraph.
99. (c) 100. (a) 101. (d) 102. (a)
R P V W U S T Q 103. (d) 104. (d) 105. (e) 106. (a)
60. (d) P, W, S and Q face North. 107. (b) 108. (b) 109. (a) 110. (c)
61. (a) U is to the immediate left of S. 111. (b) 112. (a) 113. (b) 114. (d)
62. (c) There are two persons V and W between P and U.
115. (a) 116. (a) 117. (b) 118. (a)
63. (b) R and Q are standing at the extreme ends.
119. (c) 120. (d)
y
o
u
rs
m
a
h
b
o
o
b
.w
SBI PO EXAM 2014

o
rd
p
re
Based on Memory

s
s
.c
o
m
REASONING (HIGH LEVEL) 7. Who among the following is to the immediate left of H?
(a) O (b) J
DIRECTIONS (Qs.1-2) : Study the following information (c) I (d) L
carefully and answer the questions given below : (e) K
8. Four of the following five are alike in a certain way based on
There are five statues - L, M, N, O and P - each of them having the above arrangement and hence form a group. Which one
different height. Statue L is smaller than only statue M. Statue O of the following does not belong to that group?
is smaller than statue N. Statue O is longer than statue P. The (a) N (b) L
height of the tallest statue is 20 feet. The height of the second
(c) O (d) I
smallest statue is 11 feet.
(e) K
1. What Will be the height of statue P?
(a) 13 feet (b) 15 feet 9. Who among the following is exactly between L and J?
(c) 9 feet (d) 12 feet (a) N (b) O
(e) 14 feet (c) H (d) I
2. What Will be the height of the third tallest statue? (e) None
(a) 13 feet (b) 10 feet 10. Who among the following is fourth to the right of J?
(c) 19 feet (d) 9 feet (a) N (b) I
(e) 11 feet (c) H (d) L
3. lf the expressions S = T > O ³ R and P £ O < Z are true, then (e) K
which of the following is not true?
DIRECTIONS (Qs.11-15) : In these questions, relationship
(a) T > O (b) R < S
(c) Z > R (d) P > T between different elements is shown in the statements.
(e) P < Z The statements are followed by two conclusions.
Give answer (a) if only Conclusion I is true.
DIRECTIONS (Qs. 4-10) : Study the following information
carefully and answer the questions given below : Give answer (b) if only Conclusion II is true.
Give answer (c) if either Conclusion I or II is true.
Eight persons - H, I, J, K, L, M, N and O - are standing in a straight
Give answer (d) if neither Conclusion I nor II is true.
line at equidistant. Some of them are facing north while others are
Give answer (e) if both Conclusions I and II are true.
facing south. M is standing third to the right to H. M is standing
at one of the extreme ends. L is standing third to the left to H. The 11. Statements :
immediate neighbours of J face north. N is not an immediate B> C = D³ X; E£X; Z³D
neighbour of H. The persons standing at the extreme ends face Conclusions : I . B > E
the same direction (Both are facing either north or South). The II . Z ³ B
immediate neighbours of H face just opposite direction as that of 12. Statements:
M. The immediate neighbours of O face opposite directions with E > F³G<H£I<J
respect to each other. One of the immediate neighbours of L is K Conclusions : I . G £ E
who is facing north. I is standing between J and M. Not more than II . J ³ F
four persons are facing north. 13. Statements:
4. Who among the following is third to the left of N? K £ L < M > N ³ O; T > M £ P
(a) K (b) J
Conclusions: I . T > K
(c) H (d) I
(e) O II . P > O
5. The immediate neighbours of L are: 14. Statements :
(a) M and N (b) N and O B > O = K ³ L; D > K ³ S
(c) K and N (d) N and H Conclusions : I. O < D
(e) J and H II. S £ L
6. How many persons are standing exactly between I and O? 15. Statements :
(a) Three (b) Four B > O = K ³ L; D > K ³ S
(c) One (d) Two Conclusions : I. L > D
(e) None II. B > S
y
o
u
rs
m
a
h
350 SBI Po Exam 2014

b
o
o
DIRECTIONS (Qs.16-20): Each of the questions below consists I. A man starts walking from point 'A' towards east and

b
after walking 3 metres reaches point 'N', he turns right

.w
of a question and two statements numbered I and II given below
it. You have to decide whether the data provided in the statements and walks 7 metres to reach point 'M'. Then he turns

o
rd
are sufficient to answer the question. Read both the statements right and walks 6 metres to reach point 'O'. He again
turns right and walks 7 metres to reach point 'P'. He,

p
and—

re
then, turns left and walks 2 metres to reach point 'B'.

s
Give answer (a) if the data in Statement I alone are sufficient to II. A man starts walking from point 'A' towards east and

s
.c
answer the question, while the data in Statement II alone are not after walking 3 metres reaches point 'N'. From point 'N'

o
sufficient to answer the question.

m
he walks 7 metres towards south and reaches point
Give answer (b) if the data in Statement II alone are sufficient to 'M'. From point 'M' he walks 6 metres towards west and
answer the question, while the data in Statement I alone are not reaches Point 'O'. From point 'O' he walks 7 metres
sufficient to answer the question. towards north and reaches point 'P'. From point 'P' he
Give answer (c) if the data either in Statement I alone or in walks towards west and reaches point 'B'. The distance
Statement II alone are sufficient to answer the question. between points A and B is 8 metres.
Give answer (d) if the data even in both Statements I and II together 19. How many students scored more than B in a class of 25
are not sufficient to answer the question. students?
Give answer (e) if the data in both Statements I and II together are I. More than four but less than ten students scored more
necessary to answer the question. than that of B. B's rank is an odd number. Seventeen
16. What is the code of 'shine' in a certain code language ? students scored less than D.
I. In that code language shine was peeled off is written II. The rank of C is 16th from the last. B got more marks
as '& # @ 9' and 'no paint but shine' is written as '7 5 # 8'. than C. Only two students are there between B and C.
II. In that code language 'try the new shine' is written as 20. Five persons - A, B, C, D and E - are sitting around a circular
'13 # 0' and 'we try the new' is written as '6013'. table. Some of them are facing towards centre while others
17. How C is related to H? are facing outside.
I. N is son of H. J is mother of Z. N and Z are cousins. C Who is sitting second to the left of A?
is husband of J. I. B is facing outside. C is to the immediate left of B. D is
II. L is father of C. A is mother of D. L is married to A. H is second to the right of C. A is to the immediate left of E.
wife of D. J is wife of C. II. D is to the immediate right of A. Both D and A are
18. In which direction point 'A' is located with respect to point facing towards the centre. D and B are immediate
'B'? neighbours of each other.
DIRECTIONS (Qs.21-27) : Study the following information carefully and answer the questions given below :
A word and number arrangement machine when given an input line of words and numbers rearranges them following a particular rule
in each step. The following is an illustration of input and various steps of rearrangement. (All the numbers are two digit numbers).
Input : sweet 46 nice 36 friend 26 help 96 bright 76 kind 66
Step I : sweet 46 nice 36 friend 26 help bright 76 kind 66 96
Step II : sweet nice 46 36 friend 26 help bright kind 66 76 96
Step III: sweet nice kind 46 36 friend 26 help bright 66 76 96
Step IV: sweet nice kind help 36 friend 26 bright 46 66 76 96
Step V : sweet nice kind help friend 26 bright 36 46 66 76 96
Step VI: sweet nice kind help friend bright 26 36 46 66 76 96
And Step VI is the last step of the rearrangement as the desired arrangement is obtained.
As per rules followed in the above steps, find out in each of the questions the appropriate step for the given input.
Input : arrow 98 paint 58 lamb 38 each 78 great 18 most 48 rent 88
21. Which word/number would be fifth to the left of the sixth 24. Which step number is the following output?
element from the right in the step V? rent paint most arrow 58 lamb 38 each great 18 48 78 88 98
(a) great (b) arrow (a) There is no such step (b) Step II
(c) lamb (d) 38 (c) Step V (d) Step VI
(e) 48 (e) Step III
22. Which of the following represents the position of "58" in 25. Which element (word/number) would be at the eleventh
the step IV? position from the right in the Step III?
(a) Eighth from left (b) Third from right (a) lamb (b) arrow
(c) Ninth from left (d) Eleventh from left (c) 58 (d) 38
(e) Fifth from right (e) each
23. How many elements (words/numbers) are there between 26. Which element (word/number) would be at the sixth position
"most" and "78" as they appear in the Step VI? from the left in the Step VI ?
(a) Eight (b) Seven (a) 18 (b) arrow
(c) Nine (d) Five (c) great (d) each
(e) Four (e) 38
y
o
u
rs
m
a
h
SBI Po Exam 2014 351

b
o
o
27. At which of the following positions "great" would appear 33. Who among the following is sitting fourth to the left of the

b
from the left in the Step V ? person who studies in Standard II?

.w
(a) Fifth (b) Sixth (a) U (b) W

o
rd
(c) Fourth (d) Second (c) X (d) S

p
(e) Third (e) Cannot be determined

re
34. Who among the following is sitting exactly between the

s
DIRECTIONS (Qs. 28-34): Study the following information

s
.c
carefully and answer the questions given below: person who studies in Standard III and W?

o
(a) T (b) U

m
Eight friends -R, S, T, U, V, W, X and Y - are sitting around a
circular table facing the centre, but not necessarily in the same (c) R (d) V
order. Each of the them studies in the different Standards viz, (e) Y
standards I to Standard VIII, but not necessarily in the same order. DIRECTIONS (Qs. 35-40) : Study the following information
T is second to the right of the person who studies in standard VII. carefully and answer the questions given below :
Only one person sits between T and the person who studies in
standard V. X is sitting third to the left of the person who studies A college planned to conduct an education fair. The college asked
in Standard VIII. The person studying in Standard VIII is not an foreign universities to participate in the education fair so that
immediate neighbour of the person studying in Standard VII. T students could know about the foreign universities. Even foreign
does not study in Standard VIII. The person studying in Standard universities can tell which course is good in which university.
VI to the left of U. U does not study in Standard V or Standard The courses offered by these universities are–Social Science,
VIII. The persons studying in Standard VI and VII are immediate Psychology, Medicine, Commerce, Astronomy, Computer Science
neighbours of each other. One of the immediate neighbours of the and Architecture. The fair will start on Monday and will end on
person studying in Standard VIII, studies in Standard II. S and Y Sunday of the same week. Social Science stall will be set up
are immediate neighbours of each other. There is one person immediately before the Medicine stall. Medicine stall will not be
between S and R. V does not study in Standard I. R studies in set up on Tuesday or Thursday. There will be a gap of one day
Standard V. Y does not study in Standard II. W is sitting between between the stalls of Astronomy and Medicine. Architecture stall
the person who study in Standard VII and Standard IV. X is second will be set up immediately after Astronomy stall. Computer Science
to the right of the person studying in Standard I.
stall will be set up on Monday.
28. Who among the following is sitting third to the right of T. 35. Which of the following stalls will be set up on Thursday ?
(a) S (a) Astronomy (b) Psychology
(b) The person studying in Standard II (c) Medicine (d) Architecture
(c) The person studying in Standard V (e) None of these
(d) Y 36. Which of the following stalls will be set up immediately
(e) The person studying in Standard I after the Computer Science stall?
29. Who among the following is sitting just opposite to 'R'? (a) Medicine (b) Commerce
(a) The person studying in Standard VI (c) Psychology (d) Social Science
(b) W (e) None of these
(c) The person studying in Standard V 37. On which of the following days of the week the stall on
(d) S Architecture will be set up?
(e) The person studying in Standard VII (a) Saturday (b) Tuesday
30. Which of the following pairs represents the immediate (c) Thursday (d) Friday
neighbours of 'V'? (e) None of these
(a) R and person studying in Standard VIII 38. Which of the following pairs of stalls will be set up between
(b) S and the person studying in Standard III the stalls on Computer Science and Psychology?
(c) The person studying in Standard V and Y (a) Social Science and Astronomy
(d) Cannot be determined (b) Medicine and Astronomy
(e) None of these (c) Social Science and Medicine
31. What is the position of 'W' with respect to 'S'? (d) Medicine and Commerce
(a) Fourth to the right (b) Fourth to the left (e) None of these
(c) Third to the right (d) Fifth to the right 39. Which of the following stalls will be set up on Friday?
(e) Sixth to the left (a) Commerce (b) Medicine
32. Starting from R, if all the persons are made to sit in the (c) Psychology (d) Architecture
alphabetical order in clockwise direction, the positions of (e) None of these
how many (excluding R) will remain unchanged? 40. Which of the following stalls will be set up on Wednesday?
(a) Two (b) Three (a) Medicine (b) Architecture
(c) Four (d) One (c) Psychology (d) Social Science
(e) None (e) None of these
y
o
u
rs
m
a
h
352 SBI Po Exam 2014

b
o
o
DIRECTIONS (Qs. 41-45) : Below in each question are given (c) Either Conclusion I or Conclusion II follows

b
.w
two statements (A) and (B). These statements may be either (d) There is possibility that some stars are triangles.

o
independent causes or may be effects of independent causes or a (e) Neither Conclusion I nor Conclusion II follows.

rd
common cause. One of these statements may be the effect of the 47. Statements :

p
re
other statement. Read both the statements and decide which of All drums are banjos

s
the following answer choice correctly depicts the relationship Some drums are guitars

s
.c
between these two statements. No banjo is a flute

o
m
Mark answer (a) if statement (A) is the cause and statement (B) Conclusions :
is its effect. I. Some guitars are flutes
Mark answer (b) if statement (B) is the cause and statement (A) is II. No guitar is flute
its effect. (a) Only Conclusion I follows
Mark answer (c) if both the statements (A) and (B) are independent (b) Only Conclusion II follows
causes. (c) There is possibility that some flutes are banjos
Mark answer (d) if both the statements (A) and (B) are effects of (d) Either Conclusion I or Conclusion II follows
independent causes. (e) Both Conclusions I and II follows
Mark answer (e) if both the statements (A) and (B) are effects of 48. Statements :
some common cause. Some pins are needles.
41. (A) A substantial increase in the unhealthy competition All needles are swords
has been observed among the students.
Some swords are knives.
(B) A rise of 23% is reported every year in the cases of Conclusions :
suicides after declaration of grade 10th and 12th
I. All swords being pins is a possibility
examination results.
II. No needle being knife is a possibility.
42. (A) The glaciers at the poles of the earth are melting at a
(a) Only Conclusion I follows
fast rate.
(b) Only Conclusion II follows
(B) In recent times there has been a substantial increase in
(c) Either Conclusion I or Conclusion II follows
the incidents of earthquakes and volcanic eruptions.
(d) Neither Conclusion I nor Conclusion II follows
43. (A) Most of the shopkeepers in the locality closed their
(e) There is possibility that some pins are knives.
shops for the second continuous day.
49. Statements :
(B) Two groups of people living in the locality have been
Some schemes are offers.
fighting with each other with bricks and stones forcing
Some offers are discounts
people to stay indoors.
No discount is a loan
44. (A) The Government has decided to increase the prices of
Conclusions :
LPG gas cylinders with immediate effect.
I. Those offers which are discounts can never be loans.
(B) The Government has decided to increase the prices of
II. Some loans are definitely schemes
kerosene with immediate effect.
(a) Only Conclusion I follows
45. (A) The country 'X' has banned the import of fruit.
(b) There is possibility that all schemes are loans
(B) The intake of that fruit causes disease.
(c) Only Conclusion II follows
DIRECTIONS(Qs. 46-50) : In each question below are given (d) Either Conclusion I or Conclusion II follows
three statements followed by two conclusions numbered I and II. (e) Neither Conclusion II nor Conclusion II follows
You have to assume everything in the statements to be true even 50. Statements :
if they seem to be at variance from commonly known facts and
No car is hotel
then decide which of the given conclusions logically follows
All lodges are hotels
from the information given in the statements.
No lodge is house.
46. Statements : Conclusions :
No star is a cone. I. Some houses are not lodges
Some cones are triangles.
II. No lodge is car
All kites and stars.
(a) There is possibility that all houses are cars
Conclusions :
I. All stars are kites. (b) Only Conclusion I follows
II. At least some triangles are stars. (c) Only Conclusion II follows
(a) Only Conclusion I follows (d) Neither Conclusion I nor Conclusion II follows
(b) Only Conclusion II follows (e) Both Conclusion I and Conclusion II follows
y
o
u
rs
m
a
h
SBI Po Exam 2014 353

b
o
o
DATA ANALYSIS AND INTERPRETATION DIRECTIONS(Qs. 58-63) : In the following multiple bar

b
.w
diagram, the data regarding cost of production and sales revenue

o
DIRECTIONS (Qs. 51-57) : Study the following information of the company XYZ in the given years have been given. Study

rd
carefully and answer the questions. the bar diagram carefully and answer the questions.

p
re
For a room, the rate of painting is ` 3200 per square metre. The rate Sales Revenue Cost of Production

s
s
1000 (In thousand `)

.c
of carpeting per square metre is 120% of that of tiling. The cost of
900

o
decorating the room is 14 times to that of carpeting on the floor. 800 800

m
The cost of electrification is 75% of that of carpeting the floor. 800 750 725

(In thousand `)
The rate of tiling on the floor is 125% of that of painting. The 700
625 600
dimensions of the room are 6m × 6m × 5m. 600 550
500 500 525
51. What is the ratio of the cost of painting the four walls of the 500 450
400 400
room and that of decoration?
(a) 10 : 63 (b) 10 : 61 300
(c) 10 : 21 (d) 21 : 10 200
(e) None of these 100
52. What will be the total cost of decorating the room and tiling 0
2004 2005 2006 2007 2008 2009
the floor when the four walls have also been tiled to a height
of 0.25 metre? YEARS
58. By what per cent is the amount received from the sales in
(a) ` 5287200 (b) ` 2587200 the year 2005 of the company, more than the expenditure on
(c) ` 2588200 (d) ` 2577200 production in the year 2008?
(e) None of these (a) 45% (b) 40%
53. What will be the total cost of painting, carpeting, decoration (c) 48% (d) 49%
and electrification of the room if the dimensions of the room (e) 50%
be 21 m × 42 m × 27m ? 59. In how many years the cost of production is more than the
average cost of production of the given years ?
(a) ` 30888000 (b) ` 8388000 (a) 1 (b) 2
(c) ` 80388000 (d) ` 40888000 (c) 3 (d) 4
(e) None of these (e) None of these
54. What will be the cost of tiling the floor of the room if the 60. In how many years the sales revenue is less than the average
rate of tiling be increased by 75% and the dimensions of the sales revenue of the given years?
(a) 1 (b) 3
room be 51m × 59m × 84m ?
(c) 2 (d) 4
(a) ` 21163000 (b) ` 1263000 (e) None of these
(c) ` 2163000 (d) ` 21063000 61. If the cost of production in 2004 and 2007 be increased by
(e) None of these 25% and 30% respectively, then by what per cent will the
55. If the length of the room be increased by 20%, breadth by total cost in these both years be more than the sales revenue
of the year 2008 ?
32% and height by 12%, then what will be the total cost of
(a) 62.7% (b) 65.7%
painting of the four walls of the room and tiling the floor?
(c) 67.7% (d) 68.5%
(a) ` 76996.80 (b) ` 67996.80 (e) None of these
(c) ` 67796.80 (d) ` 76796.80 62. In which year the company had maximum profit per cent ?
(e) None of these (a) 2008 (b) 2007
56. What will be the total cost of carpeting, decoration, (c) 2006 (d) 2005
(e) 2004
electrification and tiling the floor if the rate of painting be
63. If the sales revenues in the years 2005, 2006 and 2007 increase
doubled?
by 20%, 25% and 30% respectively and the costs of
(a) ` 5771200 (b) ` 5731200 production in the years 2007, 2008 and 2009 increase by
(c) ` 5371200 (d) ` 7571200 20%, 25% and 35% respectively, what will be the difference
(e) None of these between average sales revenue and average cost of
57. What will be the cost of decoration of the room if its cost be production?
increased by 25%? (a) ` 185.59 thousands
(b) ` 188.59 thousands
(a) ` 60480 (b) ` 604800 (c) ` 174.59 thousands
(c) ` 6048000 (d) ` 6448000 (d) ` 200.59 thousands
(e) None of these (e) None of these
y
o
u
rs
m
a
h
354 SBI Po Exam 2014

b
o
o
DIRECTIONS (Qs.64-69) : In the following multiple graphs 69. What was the total production of wheat by these three states

b
in the year 2007? (in quintals)

.w
production of wheat (in quintals) by three states - Bihar, Madhya
(a) 9900 (b) 9700

o
Pradesh and Punjab have been given. Study the following graphs

rd
carefully to answer the questions. (c) 9980 (d) 8800

p
(e) None of these

re
s
Bihar Madhya Pradesh Punjab DIRECTIONS (Qs. 70-75) : In the following bar diagram the

s
4000

.c
3900 3800 3800 number of engineers employed in various companies has been

o
3800 given. Study the bar diagram carefully to answer the questions.

m
3700
3700 3600
3600 3500 Post Graduate Engineers
Production of wheat (in quintals)

3500 3400
3100

3400 3300 Assistant Engineers


3300 3200 Junior Engineers
3200
2900

3100 3000 1000


3000

Number of Engineers
2900 2900 900
2900 2800
2800 2900 2900 800
2700 2800 700 650
2600 2700
600
2500
2400 500
2400 2400 400
2300 400
2200
2100 300
2100
200
2001 2002 2003 2004 2005 2006 2007 100
Years 0
64. If the production of wheat by Madhya Pradesh in the years V W X Y Z
Companies
2003, 2004, 2005 and 2007 increase by 30%, 40%, 45% and
70. The number of post graduate engineers employed in the
40% respectively, what will be the overall percentage increase
company W is what per cent of the total engineers employed
in the production of wheat in the state in the given years?
in that company ?
(a) 22% (b) 25%
1 1
(c) 35% (d) 16% (a) 33 % (b) 30 %
(e) 19% 3 3
65. What was the average production of wheat by all three 1 1
(c) 25 % (d) 36 %
states in the year 2005? (in quintals) 3 3
(e) None of these
1 2 71. What is the average number of junior engineers employed
(a) 2866 (b) 2866
3 3 in all the companies?
(a) 150 (b) 170
2 1
(c) 2688 (d) 2688 (c) 160 (d) 180
3 3 (e) 190
(e) None of these 72. What is the difference between the average number of junior
66. In the given years, what is the average production of wheat engineers and assistant engineers taking all the companies
in Bihar? (in quintals) together?
(a) 3068 (b) 3076 (a) 18 (b) 15
(c) 3086 (d) 3088 (c) 10 (d) 22
(e) None of these (e) 25
67. If the productions of wheat in Bihar in the years 2001, 2002, 73. If the number of assistant engineers employed in all the
2003 and 2004 increase by 20%, 25%, 28% and 35% companies be increased by 37% and the number of post
respectively; what will be the percentage increase in the graduate engineers employed in all the companies b
average production of the state for the given years? decreased by 20%, by what percent will the number of
assistant engineers be less than that of post graduate
(a) 35.7% (b) 38.7%
engineers?
(c) 40.7% (d) 42.5%
(a) 5.6% (b) 7.8%
(e) None of these (c) 8% (d) 9.3%
68. By what per cent is the total production of wheat by three (e) None of these
states in the year 2002, 2003 and 2004 more or less than that 74. If the numbers of all the engineers in the company V, company
in the years 2005, 2006 and 2007? X and company Y be increased by 30%, 35% and 40%
(a) 2.5% (b) 2.6% respectively, what will be the overall percentage increase in
(c) 1.9% (d) 1.09% the number of all engineers of all the companies taken
(e) None of these together?
y
o
u
rs
m
a
h
SBI Po Exam 2014 355

b
o
o
(a) 20% (b) 22% 80. If an increase of 40% is made in the average number of

b
female employees, working in countries III, IV and V, then

.w
(c) 24% (d) 25%
(e) None of these their resulting average number will be what per cent of the

o
rd
75. What is the ratio between the number of assistant engineers average number of female employees?

p
employed in company V and company X? (a) 83% (b) 85%

re
(c) 65% (d) 68%

s
(a) 3 : 5 (b) 3 : 4

s
(e) 69%

.c
(c) 2 : 3 (d) 3 : 2 81. By what per cent is the total number of employees in

o
m
(e) None of these countries V, VI and VII more than the number of male
DIRECTIONS (Qs.76-81) : In the following pie charts the employees working in the countries II, III and IV?
percentage of employees of a company working in 8 different (a) 50.2% (b) 53.6%
countries has been given. Study these pie-charts carefully to (c) 55% (d) 48%
answer the questions. (e) 49%

Total number of employees = 86000 DIRECTIONS (Qs. 82-87) : In the following table, information
regarding publishing books by different publishers has been
given. Study the table carefully and answer the questions.
VIII I
8% 18% Publishing Number of Ratio of Percentage Number of
VII
7% Houses Books Academic of books distributors in
II
VI 12% published and non- distributed publishing
16% academic house
III
V IV 8% books
17% 14%
M 28200 7: 3 81 17
N 32200 5: 9 74 23
Number of male employees = 56000 O 29700 6: 5 92 18
P 31200 8: 5 86 24
VII 11% Q 33800 7: 6 79 25
VI 9% VIII R 35700 11 : 6 82 21
I
7% 15% S 37800 5 : 13 89 24
II 82. What is the difference between the number of academic
V
10%
18% books published by publishing houses M and P ?
III
IV 12% (a) 450 (b) 640
18% (c) 540 (d) 504
(e) None of these
Note: Roman digits show the countries. 83. How many books were given to each distributor by publisher
76. What is the ratio between male employees working in the Q if each publisher gets equal number of books ?
country I and country II respectively? (a) 1806 (b) 1068
(c) 1608 (d) 1308
(a) 3 : 4 (b) 5 : 4
(e) None of these
(c) 5 : 3 (d) 3 : 2 84. What is the average number of non-academic books
(e) 2 : 3 published by publisher R and S?
77. What is the ratio between the number of male employees (a) 18750 (b) 18850
and female employees in country II? (c) 19950 (d) 18950
(a) 70 : 53 (b) 70 : 31 (e) 19990
(c) 70 : 59 (d) 53 : 70 85. What is the total number of books distributed by publishers
(e) None of these O and Q?
78. What is the approximate average number of male employees (a) 26702 (b) 27324
in countries I, II and III ? (c) 55026 (d) 54026
(a) 9670 (b) 6970 (e) None of these
86. If the total number of books published by publishers P, Q
(c) 6907 (d) 6977
and R is increased by 30% and the total number of books
(e) None of these
published by remaining publishers be decreased by 20%,
79. What is the average number of female employees in the what will be the new average of books published by all the
countries IV and VII? publishers ?
(a) 1370 (b) 1070 (a) 33418 (b) 33318
(c) 1570 (d) 1470 (c) 32518 (d) 33618
(e) None of these (e) None of these
y
o
u
rs
m
a
h
356 SBI Po Exam 2014

b
o
o
87. The ratio of the number of books published by publishers P (a) 300 (b) 315

b
and Q is (c) 328 (d) 345

.w
(a) 156 : 139 (b) 156 : 169 (e) 360

o
rd
(c) 3 : 5 (d) 5 : 17
DIRECTIONS (Qs. 96-100) : Study the pie-charts carefully to

p
(e) None of these

re
answer the following questions.

s
DIRECTIONS (Qs. 88-95) : In the following table, the number of

s
Percentage of students enrolled in different activities in a school

.c
candidates appeared and qualified for different posts in an N = 3000

o
institute during various years has been given. Read the table

m
carefully and answer the questions.
Description of appeared - qualified candidates
Years 2001 2002 2003 2004 2005 2006 Dancing Craft
Po s ts A pp . Qu al. A p p . Qu al. A p p. Qual. A p p . Qu al. A p p . Qu al. A p p. Qual. 24% 25%
A 242 157 345 190 330 240 387 309 433 390 483 417
B 387 310 427 280 410 380 417 354 456 412 339 265 Singing Drawing

Sw 16%
C 378 302 411 280 520 415 340 221 381 355 335 290 21% 14%

imm
D 420 364 577 315 487 290 490 480 512 487 410 370
E 392 370 380 316 250 180 430 416 447 423 381 351

ing
F 509 412 480 423 390 322 227 190 329 279 370 337
Note : A pp . - A pp eared Qu al. - Qu alified
88. What is the approximate average number of candidates who Percentage break-up of girls enrolled in these activities out of
qualified for all the posts in the year 2002 ? the total students.
(a) 300 (b) 280 n = 1750
(c) 290 (d) 310
(e) 314
89. What is the approximate average number of candidates who Dancing Craft
appeared for all the posts in the year 2006 ? 20 % 22%
(a) 366 (b) 389
(c) 386 (d) 375 Singing Drawing
(e) 390 28% Sw 14% 16%
90. The difference between the number of candidates who imm
appeared and qualified for the post D in the years 2001, 2002
ing
and 2003 is
(a) 615 (b) 605
(c) 601 (d) 515 96. Number of girls enrolled in Dancing form what per cent of
(e) 505 total number of students in the school? (rounded off to two
91. What approximate percentage of candidates qualified for digits after decimal)
the post of A in the year 2003? (a) 12.35 (b) 14.12
(a) 73% (b) 70% (c) 11.67 (d) 10.08
(c) 68% (d) 69% (e) None of these
(e) 75% 97. How many boys are enrolled in Singing and Craft together?
92. The respective ratio between the number of candidates who (a) 505 (b) 610
qualified for the posts of A and D in the year 2002 is (c) 485 (d) 420
(a) 67 : 38 (b) 38 : 63 (e) None of these
(c) 38 : 67 (d) 19 : 31 98. What is the respective ratio of number of girls enrolled in
(e) None of these Swimming to the number of boys enrolled in Swimming ?
93. In which year maximum number of candidates were selected (1) 47 : 49 (b) 23 : 29
for the post of A? (c) 29 : 23 (d) 49 : 47
(a) 2003 (b) 2004 (e) None of these
(c) 2005 (d) 2006 99. What is the total number of girls enrolled in Swimming and
(e) 2002 Drawing together ?
94. In which year minimum number of candidates qualified for (a) 480 (b) 525
the post of F? (c) 505 (d) 495
(a) 2002 (b) 2003 (e) None of these
(c) 2004 (d) 2006
100. What is the approximate percentage of boys in the school?
(e) 2005
(a) 34 (b) 56
95. The average number of candidates who qualified for all the
(c) 28 (d) 50
posts in the year 2004 is
(e) None of these
y
o
u
rs
m
a
h
SBI Po Exam 2014 357

b
o
o
(d) Assembly, Compilation. Linking
GENERAL AWARENESS, MARKETING

b
(e) Compilation, Linking, Assembly

.w
& COMPUTER 110. Which of the following is the proper removal of any

o
rd
malicious program from a computer System?

p
101. Banks with higher proportion of demand deposits will have (a) Reconfiguration (b) Reboot

re
(a) Low cost money at its disposal (c) Restart (d) Uninstallation

s
s
(b) Low interest money at its disposal (e) Only (b) and (d)

.c
(c) Higher number of current and savings accounts 111. What is the secured messaging system device used by

o
m
(d) High amount of liquid money IDBRT in the field of banking'?
(e) All of the above (a) NEFT (b) SFMS
102. Which commercial bank of India recently became the first to (c) RTGS (d) EFT
launch Mobile Branch with ATM. titled 'Branch on Wheels'? (e) All of the above
(a) State Bank of India (b) Axis Bank 112. Which of the following public sector banks recently became
(c) ICICI Bank (d) ABN Amro Bank the first to launch the Instant Money Transfer (IMF) scheme
(e) Punjab National Bank which is a innovative domestic money remittance facility
103. To which of the following crops does the Rashtriya Krishi from ATMs without an account in the bank?
Bima Yojana (NAIS) provide insurance? (a) State Bank of India (b) Axis Bank
(a) Food crops (cereals and pulses) (c) Punjab National Bank (d) Bank of India
(b) Oilseeds (e) Bank of Baroda
(c) Horticultural crops 113. Which of the following does not come under the second
(d) Commercial crops level of management in an organization?
(e) All of the above (a) Purchase Managers (b) Departmental Heads
104. According to the Reserve Bank of India which of the (c) Supervisors (d) Branch Managers
following contribute to more than three-fourth of the total (e) Finance Managers
Non- Performing Assets (NPAs)? 114. What is the process of copying software programs from
(a) Public Sector bank (b) Foreign Bank secondary storage media to the hard disk called?
(a) Configuration (b) Download
(c) Private Bank (d) Regional Rural Bank
(c) Storage (d) Upload
(e) Co-operatives
(e) Installation
105. Who has been selected for the Gandhi Peace Prize for the
115. In the field of banking, what does CRAR stand for?
year 2013?
(a) Capital Reserve Adequacy Ratio
(a) Chandi Prasad Bhatt (b) Desmond Tutu
(b) Capital to Risk (Weighted) Assets Ratio
(c) Medea Benjamin (d) Julius Nyerere
(c) Capital Reserve Assets Ratio
(e) Gopalkrishna Gandhi
(d) Capital Risk Adequacy Ratio
l06. What is the term for the cheque issued by a person to another
(e) None of these
person in which the issuing date is after a few days (a date 116. Which of the following terms /expressions is used to
which is yet to come)? describe a business unit with a competitive advantage that
(a) Anti-dated cheque (b) Stale cheque will capture some of the extra economic values it creates, no
(c) Post-dated cheque (d) Crossed cheque matter how intense competition is?
(e) Order cheque (a) Consonance (b) Advantage
107. Which of the following characterizes the top level (c) Consistency (d) Feasibility
management of corporates? (e) All of the above
(a) Spend more time in co-ordinating and communicating 117. Which of the following commercial banks of India recently
(b) Executes (implements) the policies and plans opened its branch in Shanghai?
(c) Determine the objectives, policies and plans of the (a) State Bank of India
organisation (b) Axis Bank
(d) Spend more time in directing and controlling (c) ICICI Bank
(e) Coordinate the activities of all the departments (d) Oriental Bank of Commerce
108. Through which of the following ways can devices be (e) Corporation Bank
connected to a computer'? l18. Which of the following come under Policy Framework for
(a) USB (b) Bluetooth Investment Decisions as outlined by the OECD?
(c) CD/DVD Drive (d) RAM (a) Policy coherence
(e) Only (a) and (b) (b) Importance of transparency approach to policy
109. Which of the following is the correct sequence of formulation
compilation? (c) Regular evaluation of policies on investment
(a) Compilation, Assembly, Linking environment
(b) Assembly, Linking, Compilation (d) Corporate governance
(c) Linking, Assembly, Compilation (e) All of the above
y
o
u
rs
m
a
h
358 SBI Po Exam 2014

b
o
o
119. What is the corpus of the National Social Security Fund? 130. President Pranab Mukherjee on the occasion of international

b
Women's Day presented which award to six individuals for

.w
(a) ` 250 crore (b) ` 500 crore
their contribution for the social development and

o
(c) ` 1000 crore (d) ` 1200 crore

rd
empowerment of women in the country?
(e) ` 1500 crore

p
(a) Stree Shakti Puraskar award

re
120. We regularly come across some lines marked on the back of
(b) Nirbhaya Puraskar

s
products. What are they called?

s
(c) Mother Teresa Award

.c
(a) Bar-code (b) Spooler

o
(d) Florence Nightingale Award
(c) Scanner (d) Output reader

m
(e) Ahilyabai Puraskar
(e) Price tag
131. What is the goal in Two Fold Marketing?
121. In the field of computers, which of the following converts
input into output? (a) To attract new customers by promising superior value
(a) Mouse (b) Trackball (b) To keep current customers by delivering satisfaction
(c) Keyboard (d) Bar Code Reader (c) To undertake aggressive marketing campaigns
(e) Light Pen (d) Only (a) and (b)
122. Which application is used for the purposes of basic (e) All of the above
accounting? 132. How can one see the content of a file?
(a) Worksheet (b) Spreadsheet (a) By left clicking with the mouse once
(c) infographic (d) Data sheet (b) By left clicking with the mouse twice
(e) Document (c) By holding the mouse down
123. Which insurance company has launched Asha Kiran, an (d) From the keyboard shortcuts
health insurance cover, for a family with girl children? (e) All of the above
(a) Oriental Insurance comp. Ltd. 133. What is it called when computer connects to other computer
(b) Reliance General insurance and sends information?
(c) United India Insurance Comp. Ltd. (a) LAN (b) Remote Desktop
(d) New India Assurance comp. Ltd. (c) Boot (d) Network
(e) National Insurance Company (e) intemet Protocol
124. Who is the author of 'India at Risk'? 134. How does every bank pay interest on savings?
(a) Stephen Cohen (b) Jaswant Singh (a) Daily (b) Weekly
(c) Tony Alcott (d) Lal Krishna Advani (c) Fortnightly (d) Monthly
(e) Manmohan Singh (e) Quarterly
125. Which of the following was/ were not the subsidiary 135. What is the alphanumeric code on cheques called ?
company of the General Insurance Corporation of India? (a) RTGS (b) MICR
(a) National Insurance Company Limited (c) IFSC (d) NEFT
(b) Life Insurance Corporation (e) CC
(c) New India Assurance Company Limited 136 How can a file be saved in a computer with other name ?
(d) Orienta1 Insurance Company Limited (a) Save command (b) Save as command
(e) United India Insurance Company Limited (c) Place command (d) Duplicate command
126. Which of the following countries will participate in the (e) Export command
NATO 2014 Summit? 137. Which is the new airline company to get licence recently in
(a) Russia (b) China India ?
(c) Germany (d) India (a) Indigo (b) Spice Jet
(e) Australia (c) Air Asia (d) Kingfisher
127. Market Segmentation can be defined as (e) Go Air
(a) Segment, Target, Price 138. How instrument is used by the Reserve Bank of India to
(b) Sales, Target, Product control market liquidity?
(c) Segment, Target, Position (a) Repo rate
(d) Sales, Time, Product (b) Statutory Liquidity Ratio
(e) Service, Sales, Segment
(c) Marginal Adjustment Facility
128. Which of the following provides power back to a computer
(d) Reserve Repo Rate
system even when main current is off?
(e) Only (a) and (d)
(a) UPS (b) Inverter
139. Banking Ombudsman is appointed by
(c) CMOS battery (d) Soft Power
(a) Union Ministry of Finance
(e) Motherhoard Power Connector
(b) Competition Commission of India
129. Which team won the Asia Cup 2014?
(a) India (b) Pakistan (c) Reserve Bank of India
(c) Sri Lanka (d) Bangladesh (d) Indian Banks Association
(e) Both (a) and (c) (e) Securities and Exchange Board of India
y
o
u
rs
m
a
h
SBI Po Exam 2014 359

b
o
o
140. Which state in India has the highest urban population? 150. Which of the following come under the ambit of Customer

b
(a) Maharashtra (b) Delhi relationship management?

.w
(c) Tamil Nadu (d) Karnataka (a) Sales (b) Marketing

o
rd
(e) Gujarat (c) Customer Service (d) Technical support

p
141. What was the reason behind the decision of the Reserve (e) All of the above

re
Bank of India to withdraw currency notes issued before

s
ENGLISH LANGUAGE

s
2005?

.c
(a) To replace them with plastic notes

o
m
(b) to check counterfeit notes
DIRECTIONS (Qs. 151-160) : Read the following passage
(c) To replace mutilated and spoilt notes
carefully and answer the questions given below it. Certain words/
(d) To check illicit notes of Pakistan
phrases have been printed in bold to help you locate them while
(e) None of these
answering some of the questions.
142. Which film was presented the award of Best film at the 61st
National Awards ? India is rushing headlong toward economic success and
(a) Ship of Theseus (b) Bhaag Milkha Bhaag modernisation, counting on high-tech industries such as
(c) Jolly LLB (d) December 1 information technology and biotechnology to propel the nation
(e) Yellow to prosperity. India's recent announcement that it would no longer
143. Which of the following is the fastest mode of transaction? produce unlicensed inexpensive generic pharmaceuticals bowed
(a) RTGS (b) Cheque payment to the realities of the World Trade Organisation while at the same
time challenging the domestic drug industry to compete with the
(c) ATM Cash Withdrawal (d) NEFT
multinational firms. Unfortunately, its weak higher education sector
(e) None of these
constitutes the Achilles' Heel of this strategy. Its systematic
144. A marketing technique where marketer plays a specific role
disinvestment in higher education in recent years has yielded
in a particular segment is called
neither world-class research nor very many highly trained scholars,
(a) Mass Marketing scientists, or managers to sustain high-tech development.
(b) Niche Marketing India's main competitors especially China but also Singapore,
(c) Strategic Marketing Taiwan, and South Korea are investing in large and differentiated
(d) Communication Marketing higher education systems. They are providing access to large
(e) None of these number of students at the bottom of the academic system while at
145. In Marketing Mix 4 P's imply the same time building some research-based universities that are
(a) Product, Price, Place, Promotion able to compete with the world's best institutions. The recent
(b) Product, Price, Policy, Place London Times Higher Education Supplement ranking of the
(c) Place, People, Product, Promotion world's top 200 universities included three in China. three in Hong
(d) People, Price, Profit, Place Kong. Three in South Korea, one in Taiwan, and one in India (an
(e) None of these Indian institute of Technology at number 41- the specific campus
146. Which aspect of a project/ business is/are evaluated in a was not specified). These countries are positioning themselves
SWOT Analysis? for leadership in the knowledge-based economies of the coming
(a) Strengths (b) Weaknesses era.
(c) Opportunities (d) Threats There was a time when countries could achieve economic
(e) All of these success with cheap labour and low-tech manufacturing. Low
147. Which of the following best explains the Delphi Method ? wages still help, but contemporary large-scale development
(a) It is a profit-maximization method requires a sophisticated and at least partly knowledge-based
(b) It is a forecasting method economy. India has chosen that path, but will find a major stumbling
(c) It is a retrenchment method block in its university system.
(d) It is a output enhancement method India has significant advantages in the 21st century
(e) It is a risk analysis method knowledge race. It has a large higher education sector - the third
148. In Market segmentation which among the following is not largest in the world in student numbers, after China and the United
an economic component? States. It uses English as a primary language of higher education
and research. It has a long academic tradition. Academic freedom
(a) Age (b) Gender
is respected. There are a small number of high quality institutions,
(c) Income Level (d) Taxes
departments, and centres that can form the basis of quality sector
(e) Only (a) and (b)
in higher education. The fact that the States, rather than the Central
149. What does Innovation in Marketing refer to ?
Government, exercise major responsibility for higher education
(a) Creating a new idea of thought
creates a rather cumbersome structure, but the system allows for
(b) Discovering new products a variety of policies and approaches.
(c) Modifying an old idea to present needs Yet the weaknesses far out-weigh the strengths. India
(d) Exploring new ways of being competitive educates approximately 10 per cent of its young people in higher
(e) All of the above education compared with more than half in the major industrialised
y
o
u
rs
m
a
h
360 SBI Po Exam 2014

b
o
o
countries and 15 per cent in China. Almost all of the world's India cannot build internationally recognised research-

b
academic systems resemble a pyramid. With a small high quality oriented universities overnight, but the country has the key

.w
tier at the top and a massive sector at the bottom. India has a tiny elements in place to begin and sustain the process. India will need

o
rd
top tier. None of its universities occupies a solid position at the to create a dozen or more universities that can compete

p
top. A few of the best universities have some excellent departments internationally to fully participate in the new world economy.

re
and centres, and there is a small number of outstanding Without these universities, India is destined to remain a scientific

s
s
undergraduate colleges. The University Grants. Commission's backwater.

.c
recent major support of five universities to build on their recognised 151. Which of the following statement(s) is/are correct in the

o
m
strength is a step toward recognising a differentiated academic context of the given passage ?
system – and fostering excellence. At present, the world-class I. India has the third largest higher education sector in
institutions are mainly limited to the Indian Institutes of the world in student numbers.
Technology (IITs), the Indian Institutes of Management (IIMs) II. India is moving rapidly toward economic success and
and perhaps a few others such as the All India Institute of Medical modernisation through high tech industries such as
Sciences and the Tata Institute of Fundamental Research. These information technology and biotechnology to make the
institutions, combined, enroll well under 1 per cent of the student nation to prosperity
population. III. India's systematic disinvestment in higher education
India's colleges and universities, with just a few exceptions, in recent years has yielded world class research and
have become large, under-funded, ungovernable institutions. At many world class trained scholars, scientists to sustain
many of them, politics has intruded into campus life, influencing high-tech development.
academic appointments and decisions across levels. Under- (a) Only I (b) Only II
investment in libraries, information technoloy, laboratories, and (c) Both I and II (d) Both I and Ill
classrooms makes it very difficult to provide top-quality instruction (e) All I, II and III
or engage in cutting-edge research. 152. Which of the following statements in regard to the
The rise in the number of part-time teachers and the freeze
information given in the passage is not true ?
on new full-time appointments in many places have affected morale
(a) The London Times Higher Education Supplement
in the academic profession. The lack of accountability means that
ranking of the world's top 200 universities has included
teaching and research performance is seldom measured. The
system provides few incentives to perform. Bureaucratic inertia three universities of South Korea.
hampers change. Student unrest and occasional faculty agitation (b) India has recently ann ounced not to produce
disrupt operations. Nevertheless, with a semblance of normality, unlicensed inexpensive generic pharmaceuticals that
faculty administrators are able to provide teaching, coordinate will be a challenge for the domestic drug industry to
examinations, and award degrees. compete with the multinational firms.
Even the small top tier of higher education faces serious (c) Contemporary large-scale development requires a
problems. Many IIT graduates, well trained in technology, have sophisticated and at least partly knowledge-based
chosen not to contribute their skills to the burgeoning technology economy.
sector in India. Perhaps half leave the country immediately upon (d) China has the fourth largest higher education sector in
graduation to pursue advanced study abroad – and most do not the world.
return. A stunning 86 per cent of students in science and (e) None of these
technology fields from India who obtain degrees in the United 153. According to the view expressed by the writer in the
States do not return home immediately following their study, passage, what is a step toward recognising a differentiated
another significant group, of about 30 per cent, decides to earn academic system and fostering excellence ?
MBAs in lndia because local salaries are higher – and are lost to (a) The University Grant Commission's recent major
science and technology. A corps of dedicated and able teachers support to five universities to build on their strength.
work at the IITs and IIMs, but the lure of jobs abroad and in the (b) New Education policy of the new government.
private sector make it increasingly difficult to lure the best and (c) Scholarships granted by the Central government for
brightest to the academic profession. research.
Few in India are thinking creatively about higher education. (d) Government policy to open new world class institutions
There is no field of higher education research. Those in government
(e) None of these
as well as academic leaders seem content to do the “same old
154. In writer's opinion which of the following reason(s) is/are
thing.” Academic institutions and systems have become large
responsible for poor higher education in India?
and complex. They need good data, careful analysis, and creative
I. India's colleges and universities, with some exceptions,
ideas. In China, more than two-dozen higher education research
centres, and several government agencies are involved in higher have become large under funded, ungovernable
education policy. institutions.
India has survived with an increasingly mediocre higher II. Politics has intruded into many compuses that
education system for decades. Now as India strives to compete in influences academic appointments and decisions
a globalised economy in areas that require highly trained across levels.
professionals, the quality of higher education becomes III. Under investment in libraries, laboratories, IT and
increasingly important. classrooms hinder cutting edge research.
y
o
u
rs
m
a
h
SBI Po Exam 2014 361

b
o
o
(a) Only I (b) Both 1 and II With such achievements at a time when much of the rest of

b
(c) Both II and III (d) All I. II and III the world struggles with austerity measures and economic

.w
(e) None of these recovery. Asian leaders might be tempted to switch to autopilot.

o
rd
155. Which of the following statements is not true as per the But closer examination of the region’s economic and social

p
given information in the passage ? prospects soon reveals many paradoxes.

re
(a) About fifty per cent of IIT graduates leave India to The world's fastest growing region remains home to nearly

s
s
pursue advanced study abroad. half the world’s extreme poor. While Asia has made tremendous

.c
(b) About 86 per cent of students in science and inroads in the fight against poverty, not enough of the region's

o
m
technology fields from India who obtain degrees in US economic prosperity is reaching its poorest people.
do not return home following their study. In urban areas of China, for example, the Gini coefficient (a
(c) In China more than two-dozen higher education measurement used to calculate inequality) has risen more than
research centres and several government agencies are 35% since 1990. Nearly half a billion Asians still lack access to
involved in higher education policy. safe drinking water and infant mortality in many nations is more
(d) The rise in the number of part-time teachers and the than 10 times higher than the levels seen in developed economics.
freeze on new full-time appointments in many places While "Factory Asia" may be true for manufacturing and
have boosted morale in academic profession in India. information technology services, vast number of its people are
(e) None of these illiterate and unemployed. Its financial sector is underdeveloped,
156. What in your opinion should be an appropriate title of the with many people having no access to simple banking, let alone
given passage ? other financial services. Asia's future prosperity, and the
(a) Poor state of Higher Education in India eradication of extreme poverty, will require much more than simply
(b) Politics in India's Education system high growth. Growth must be accompanied by a narrowing of
(c) Modernisation of Indian Education System inequality.
(d) Higher Education Supplement Ranking It is essential to balance the region's economic expansion
(e) None of these with mor e inclusive policies. Cut off by poor roads,
DIRECTIONS (Qs.157-158) : Choose the word/group of words telecommunications, or government policies that don't allow them
which is most similar in meaning to the word/group of words to easily borrow or save, Asia's poor and vulnerable are watching
printed in bold as used in the passage. the chasm between rich and poor grow ever wider. That gap in
prosperity can aggravate simmering social, economic and political
157. Achi1les' Heel tensions.
(a) weakness (b) strength Asian governments can help stem widening inequality by
(c) acquiescene (d) vulnerable creating better conditions for the private sector to take the lead
(e) strong heel on economic expansion, continuing to promote economic
158. Sustain diversification, and by spending on social services, education
(a) suffer (b) maintain and healthcare, and regional road, sea and air networks that will
(c) swag (d) swallow open more opportunities to more people.
(e) weaken There are areas where western governments can help too.
DIRECTIONS (Qs. 159-160) : Choose the word/ group of words By investing in infrastructure alongside public lenders, they can
which is most opposite in meaning to the word / group of words help attract much larger sums from the private sector. Asia can
printed in bold as used in the passage. also capitalise on financial lessons from the west, particularly
when it comes to setting banking regulations, strengthening
159. Cumbersome regional links, and promoting bonds to better utilise Asian savings.
(a) complicated (b) complex When describing where Asia stands today, it's useful to
(c) simple (d) bulky remember that what we are witnessing is not the emergence of
(e) heavy Asia, but rather the re-emergence of Asia.
160. Differentiated In 1820, Asia accounted for about 60% of total global output,
(a) Distinguished (b) similar with China and India together accounting for nearly half of global
(c) distinct (d) undistinguished GDP. This was followed by nearly two centuries of economic
(e) distraught decline once the western industrial evolution took hold – a trend
DIRECTIONS (Qs. 161-170) : Read the following passage that, since the information age, has been solidly reversed. By
carefully and answer the questions given below it. Certain words/ implementing structural reforms and opening their economics.
phrases have been printed in bold to help you locate them while China and others have rapidly emerged as engines of the global
answering some of the questions. economy.
The recent Asian Development Bank study suggests that
Asia's rapid rise is the most successful story of economic we could see Asia producing over half of global GDP by mid-
development in recent history. Income per capita reached nearly century, and 3 billions Asians would be considered part f the rich
$5.000 in purchasing power parity terms in 2010. Investment rates world, with capita income levels equal to that of Europe today.
averaged 35% of GDP over the decade. The number of people Carefully calibrated government support can help steer
living below the $1.25-a-day poverty line fell by 430 million Asia's economic potential, reducing political risks while opening
between 2005 and 2010.
y
o
u
rs
m
a
h
362 SBI Po Exam 2014

b
o
o
new markets to help move the west beyond the current crisis. In I. By investing in infrastructure alongside public lenders.

b
the long run, an Asian economy built on sustainable growth can

.w
western governments can help attract much larger sums
support greater levels of trade, and generate growth in tourism. from the private sector.

o
rd
Conversely, a weaker Asia presents a host of threats to the west's
II. Asia can capitalise on financial lessons from the west,

p
future growth and prosperity.

re
Particularly when it comes to setting banking
Embracing globalisation and regional co-operation has

s
regulations, strengthening regional links and promoting

s
helped bring developing Asia success. By further strengthening

.c
this process by focusing on greater access and inclusion within bonds to better utilise Asian savings.

o
m
economics, and pursuing sustainable development and improved III. "Factory Asia" can make vast number of its people
governance, an "Asian century" is both plausible and within reach. literate and employed.
But policies that worked when Asia was low-income and (a) Only II (b) Only III
capital scarce are less likely to work today and unlikely to work in (c) Both 1 and III (d) Both II and III
the future. Asia's leaders must devise bold and innovative national
(e) Both I and II
policies while pursuing regional and global co-operation. Long
term prosperity will depend on the intensity of resource use, 165. What suggestions has the author of this passage made that
including water and food, and success in managing the region's can help Asian governments stem widening inequality in
carbon footprint. the region ?
Asia's challenges remain for-midable, and its future I. By creating better conditions for the private sector to
prosperity must be earned. The right policy choices today could take the lead on economic expansion.
indeed make this the "Asian century", but this is far from II. By continuing to promote economic diversification.
preordained.
III. By spending on social services, education and
161. Which of the following statement (s) is/are correct in the
context of the passage? healthcare and regional road, sea and air networks that
I. Asia's per capita income reached approximately $5000 will open more opportunities to more people.
in purchasing power parity terms in 2010. (a) Only I (b) Both II and III
II. The number of people living below the $1.25 a day (c) Both I and II (d) Both I and Ill
poverty line in Asia fell by 430 million between 2005 (e) All I, II and III
and 2010.
166. Which of the following statements is not correct as per the
III. In 1820, Asia accounted for about 60% of total global
given information in the passage ?
output.
(a) Only I (b) Only II (a) According to the Asian Development Bank study. Asia
(c) Both I and II (d) Both II and III can produce over half of global GDP by mid-century.
(e) All I, II and III (b) In 1820. China and India together accounted for nearly
162. What in your opinion should be the appropriate title of the half of global GDP.
given passage? (c) The author opines the necessity to balance the Asian
(a) Asia's Economic Rise and Paradoxes region's economic expansion with more inclusive
(b) Asia's Rise and Europe's Re-emergence policies.
(c) Economic Backwardness of Asia (d) The widening gap between rich and poor people can
(d) Developed and Developing Regions of the world aggravate simmering social, economic and political
(e) None of these tensions.
163. Which of the following is not true as per the given
(e) The world's fastest growing Asian region remains home
information in the passage?
(a) In urban areas of China, the Gini coefficient that is a to more than sixty five per cent of the world's extreme
measurement used to calculate inequality has risen poor.
more than 45% since 1990. DIRECTIONS (Qs. 167-168) : Choose the word/group of words
(b) Nearly half a billion Asians still lack access to safe which is most similar in meaning to the word/ group of words
drinking water. printed in bold as used in the passage.
(c) The infant mortality rate in many Asian nations is more
than 10 times higher than the levels seen in developed 167. Austerity
economics. (a) auscultation (b) decoration
(d) Investment rates in Asia averaged 35 per cent of Gross (c) extravagance (d) spendthrift
Domestic Product over the decade (e) simplicity
(e) Embracing globalisation and regional cooperation has 168. Inroads
helped bring developing Asia success. (a) Noticeable effect (b) making road
164. In the author's opinion what are the areas where western (c) insecurities (d) inquest
government can help Asia boost economic development ? (e) identification
y
o
u
rs
m
a
h
SBI Po Exam 2014 363

b
o
o
DIRECTIONS (Qs. 169-170) : Choose the word/group of words 178. A person who is clean and tidy in how he dresses up

b
commands better–from those around him than those "who

.w
which is most opposite in meaning to the word / group of words
have a–and unkempt appearance—

o
printed in bold as used in the passage.

rd
(a) respectful - slovenly (b) respect - slovenly
169. Aggravate

p
(c) respected - untidy (d) respect - tidy

re
(a) worsen (b) irritate (e) respect - careful

s
(c) aggregate (d) assuage

s
179. Today we have achieved a milestone by completing 60 years

.c
(e) astonish of independence. It's now the time for everyone or every

o
m
170. Simmering Indian to undergo–of the achievements we already made
(a) seething (b) calm and also those that are to be still—
(c) boiling (d) simulating (a) self-introspection-achiver
(e) exhilarating (b) self-examination-achieve
DIRECTIONS (Qs.171-175) : In each of the following questions (c) introspection-achieved
four words are given of which two words are most nearly the (d) search-found
same or opposite in meaning. Find the number of correct letter (e) cross-inspection - made
combination. 180. Education is an essential means of–women with the
knowledge, skills and self-confidence necessary to fully—
171. (A) dominate (B) radical in the development process.
(C) determined (D) monopolise (a) empower- include
(a) A - B (b) B - C (b) empowering - participate
(c) A - D (d) B - D (c) empowered - participating
(e) C - D (d) empowerment - participate
172. (A) critical (B) equitable (e) strengthening - participate
(C) impartial (D) unearth
(a) A - B (b) B - C DIRECTIONS (Qs.181-185) : Rearrange the following six
(c) A - D (d) B - D sentences (A), (B), (C), (D), (E) and (F) in the proper sequence
(e) C - D to form a meaningful paragraph; then answer the questions given
173. (A) shining (B) raise below them.
(C) flourish (D) thrive (A) As a consequence, even if it is plausible that ambient air
(a) A - B (b) B - C pollution plays a role for the onset and increasing frequency
(c) C - D (d) B - D of respiratory allergy, it is not easy to prove this
(e) A - C conclusively.
174. (A) Affable (B) rude (B) Another factor clouding the issue is that laboratory
(C) pacify(D) cajole evaluations do not reflect what happens during natural
(a) A - B (b) B - C exposure when atmospheric pollution mixtures are inhaled.
(c) C - D (d) A - D (C) Interpretation of studies are confounded by the effect of
(e) B - D cigarette smoke. exposure to indoor pollutants and to
175. (A) energetic (B) partial outdoors and indoors allergens.
(C) diffuse (D) dispassionate (D) However, despite evidence of a correlation between the
(a) A - B (b) B - C increasing frequency of respiratory allergy and the
(c) C - D (d) A - D increasing trend in air pollution, the link and interaction is
(e) B - D still speculative.
DIRECTIONS (Qs.176-180) : Pick out the most effective pair of (E) Allergic respiratory diseases such as hay fever and bronchial
words from the given pair of words make the sentences asthma have indeed become more common in the last decades
meaningfully complete. in all industrialized countries and the reasons for this
increase are still debated.
176. Weather officials have __ below-normal rains this year. (F) Several studies have shown the adverse effects of ambient
If the predictions come true, farm output could __ as most air pollution on respiratory health.
of India's farmlands depend on rainwater for irrigation. 181. Which of the following should be the LAST sentence after
(a) forecasted-shrank (b) forecast-shrank rearrangement ?
(c) forecast-shrink (d) predicted-expand (a) A (b) B
(e) predictions-wan (c) C (d) D
177. It is the role of the state to ____ crime and protect people (e) E
and property. If the state is unable to prevent a crime it falls 182. Which of the following should be the FIRST sentence after
upon the state to __ the victim. rearrangement ?
(a) prevent-support (b) preventing-encourage (a) A (b) B
(c) prevent-supporting (d) forbid-discourage (c) C (d) D
(e) forbid-discouraging (e) E
y
o
u
rs
m
a
h
364 SBI Po Exam 2014

b
o
o
183. Which of the following should be the THIRD sentence after Millennium Development Goal 3 is to promote gender equality

b
rearrangement? and empower women. This MDG is critical for (194) poverty and

.w
(a) A (b) B improving prospects for women. But how can women break gender

o
rd
(c) C (d) D based stereotypes to minimise discrimination and (195) gender

p
(e) E based violence when they are trapped in societies with socio-

re
184. Which of the following should be the FOURTH sentence cultural practices which routinely discriminate them from having

s
s
after rearrangement ? equal opportunities in education, health and livelihood? These

.c
(a) A (b) B women are invisible and the (196) in their way prevent them from

o
m
(c) C (d) D accessing the most basic human rights and needs. The outlook is
(e) E bleak.
185. Which of the following should be the SECOND sentence Women make up 70% of the world's working hours and
after rearrangement? (197) only 10% of the world's income and half of what men earn.
(a) A (b) B This leads to greater poverty, slower economic growth and a (198)
(c) C (d) D standard of living. In developing countries, millions of women
(e) E also die each year as a result of gender-based (199). This deep-
rooted gender discrimination creates a bleak outlook for women
DIRECTIONS (Qs.186-190) : Read each sentence to find out
in developing countries. For millions of girls living in poverty, it is
whether there is any grammatical error in it. The error if any,
often those closest to them who work against the child's interests
will be in one part of the sentence, the number of that part is the
and their immediate environment is often (200) and sometimes,
answer. If there is no error, mark (e). (Ignore errors of
down-right harmful. Parents arrange marriages when you are a
punctuation. if any)
child. Neighbours say, if you are a girl, you must limit your activities
186. In the first two months of this fiscal, tractor sales has seen to your home. Friends say, it is OK not to go to school.
(a)/a drop of about five percent (b)/ however, the industry is 191. (a) made (b) make
waiting for the monsoon (c)/ to really arrive at a firm (c) look (d) has made
conclusion about growth prospects for the current year. (e) complete
(d)/No error (e) 192. (a) equality (b) affection
187. Dolphins are truly out of the ordinary because of their (c) inequality (d) support
intelligence (a) / and. among the many creatures that share (e) justice
the earth form (b)/they come closest to humankind in terms 193. (a) vulnerable (b) weakness
of (c)/familial traits, emotions and learining. (d)/ No error (e) (c) vulnerability (d) specific
188. Corruption indulged in by the high and mighty adversely (e) weakest
impacts (a)/ our nation, and in the coming months (b) / we 194. (a) tackle (b) tackling
may see revival of efforts (c)/ to tackle such large scale (c) fight (d) tackled
corruption. (d)/ No error (e) (e) fought
189. It is notable and welcome that the ministry of (a)/ 195. (a) increase (b) support
environmental and forests is to issue approvals online (b)/ (c) weak (d) reduce
in a time bound manner, with clear timelines (c)/in place for
(e) influencing
the various sub-steps along the way. (d)/No error (e)
196. (a) obstacles (b) make
190. To portray (a)/ what a fairness cream does without (b)/ any
(c) pleasantries (d) pebbles
sort of comparison or visual (c)/ references are very difficult.
(e) encouragements
(d)/ No error (e)
197. (a) earns (b) carried
DIRECTIONS (Qs. 191-200) : In the following passage there (c) spend (d) earn
are blanks, each of which has been numbered. These numbers (e) spends
are printed below the passage and against each, five words are 198. (a) slow (b) slower
suggested, one of which fits the blank appropriately. Find out (c) low (d) lowest
the appropriate word in each case. (e) lower
Women (191) up half the world's population and yet represent a 199. (a) violence (b) violent
staggering 70% of the world's poor. We live in a world in which (c) virulent (d) corruption
women living in poverty face gross (192) and injustice from birth (e) violation
to death. From poor education to poor nutrition to (193) and low 200. (a) functional (b) natural
pay employment, the sequence of discrimination that a woman (c) artificial (d) dysfunctional
may suffer during her entire life is unacceptable but all too common. (e) disfunctional
y
o
u
rs
m
a
h
SBI Po Exam 2014 365

b
o
o
b
.w
o
rd
p
1 and 2 : Conclusions:

re
Statue L is smaller than only statue M. Therefore, M is the tallest I. T > K: True

s
s
statue. II. P > O: True

.c
N>O>P 14 - 15 :

o
B>O=K³L

m
Now,
D>K³S
M>L>N>O>P B>O =K³S
D>K=O³L
S £O=K³L
20 feet 11 feet 14. (a) Conclusions:
1. (c) Statue P is the smallest. The height of the second I. O < D: True
smallest statue O is 11 feet. Therefore, the height of the II. S £ L : Not True
statue of P will be less than 11 feet. Hence, (c) is the 15. (d) Conclusions:
correct option. I. L > D: Not True
II. B > S: Not True
2. (a) The tallest statue is 20 feet high. Therefore, the second
16. (c) From Statement I :
tallest statue may be 19 feet, 18 feet ..... high.
Therefore, the third tallest statue will be less than 19 shine was peeled off Þ & # @ 9
feet and more than 11 feet high. no paint but shine Þ 7 5 # 8
3. (d) S = T > O ³ R (Given)
The code for shine is #
P £ O < Z (Given)
From Statement II :
Therefore, S=T >O³P
Z> O³R try the new shine Þ 13 # 0
Hence, P£O³R we try the new Þ 6 0 1 3
P is smaller than T. The code for shine is #
4 - 10 : 17. (c) From statement I :
H C J (–)
NORTH (+)
N Z
(+)
Hence, C is the brother of H.
From Statement II.
K L N O H J I M L A
(+) (–)
SOUTH
J C D
4. (b)
N is facing south. Therefore, J is third to the left of N. (–) (+) (+) (–)
5. (c)
The immediate neighbours of L are K and N. Hence, C is brother-in-law of H.
6. (d)
Two persons - H and J are standing between I and O. 18. (c) From statement I
7. (a)
O is to the immediate left of H. 2m 3m
B P A N
8. (e)
Except K, all others are facing south. K faces north.
9. (b)
O is sitting exactly between L and J.
7m

10. (d)
L is to the fourth of J. 7m
11. B> C= D³X³E
(a)
B>C =D£Z
Conclusions: O M
6m
I. B > E: True
II. Z ³ B: Not True North
12. (d) E > F ³ G < H £ I < J
Conclusions:
West East
I. G £ E : Not True
II. J ³ F : Not True
13. (e) K £ L < M > N ³ O South
K £L<M<T It is clear from the diagram that Point A is to the east of
P³M>N³O Point B.
y
o
u
rs
m
a
h
366 SBI Po Exam 2014

b
o
o
From statement II Input: arrow 98 paint 58 lamb 38 each 78 great 18 most

b
From the information given in statement II, we get the 48 rent 88

.w
same diagram given above. Step I: rent arrow paint 58 lamb 38 each 78 great 18

o
rd
19. (b) From statement I most 48 88 98

p
Five to nine students scored more than that of B. The Step II: rent paint arrow 58 lamb 38 each 78 great 18

re
rank of D is 8th from the top. most 48 88 98

s
s
From statement II Step III: rent paint most arrow 58 lamb 38 each great 18

.c
o
Rank of C from the beginning 48 78 88 98

m
= 25 – 16 + 1 = 10th Step IV: rent paint most lamb arrow 38 each great 18 48
7th 10th 58 78 88 98
Step V: rent paint most lamb great arrow 38 each 18 48
B E
58 78 88 98
Thus, six students scored more than that of B. Step VI: rent paint most lamb great each arrow 18 38 48
20. (e) From statement I 58 78 88 98.
21. (c) 5th to the left of the sixth from the right means 11th
B
from the right.
11th from the right in the Step V Þ lamb
D
22. (d) The position of ‘58’ in Step IV is eleventh from the left
C and fourth from right.
23. (a) There are eight elements (lamb, great, each, arrow, 18,
38, 48, 58) between “most” and “78” in the Step VI.
A/E 24. (e) This is Step III.
A/E
25. (b) 11th from the right in Step III is “arrow”.
From statement II 26. (d) In the Step VI, “each” is at the sixth position from the
left.
B 27. (a) In Step V, “great” is at the fifth position from the left.
28 - 34 :
D IV
C/E T
III I
X W
A
C/E VR U VII
From both the statements

B V Y
II S VI
VIII
D 28. (b) V is sitting third to the right of T. V studies in Standard
VII.
C
29. (e) The person studying in Standard VII is sitting just
opposite to R. U studies in Standard VII.
A
E 30. (a) R and S are immediate neighbours of V. R studies in
Standard V and S studies in Standard VIII.
C is sitting second to the left of A. 31. (c) W is third to the right of S. W is fifth to the left of S.
21 - 27 : After careful analysis of the input and various steps of
rearrangement, it is evident that in each step one word T
and one number are rearranged. The word which comes
S T
last in the dictionary order is moved to the extreme left
X W U
position while the highest number is moved to the
extreme right position in the Step I. In the next step, the
word which comes at the second position in dictionary 32. (d) R R U V
order is placed at the second position from the left and
the second highest number is placed at the second
position from the right. Such procedure is continued till V Y
all the words get rearranged in dictionary order but in Y W
S
reverse manner and all the numbers get rearranged after
X
the words in ascending order.
y
o
u
rs
m
a
h
SBI Po Exam 2014 367

b
o
o
33. (b) V studies in Standard II. W is sitting fourth to the right Conclusions:

b
of V.

.w
I. False
34. (a) X studies on Standard III. T is sitting between X and II. False Either

o
rd
W.

p
35 - 40 : Swords

re
s
s
Pins

.c
Day Course Needles

o
48. (d) Knives

m
Monday Computer Science
Tuesday Social Science
Wednesday Medicine OR
Thursday Psychology
Swords
Friday Astronomy
Saturday Architecture Pins
Needles
Sunday Commerce

35. (b) The stall of Psychology will be set up on Thursday. Knives


36. (d) The stall of Social Science will be set up immediately
after the stall of Computer Science.
Conclusions:
37. (a) The stall of Architecture will be set up on Saturday.
38. (c) The stalls of Social Science and Medicine will be set up I. False
between the stalls of Computer Science and Psychology. II. False
39. (e) The stall of Astronomy will be set up on Friday.
40. (a) The stall of Medicine will be set up on Wednesday. nt
cou Loan
41. (a) Clearly statement (A) is the cause and statement (B) is Dis
its effect.
42. (d) Clearly, both the statements (A) and (B) are effects of 49. (e) Schemes Loan
Offers Discount
independent causes.
43. (b) Clearly statement (B) is the cause and statement (A) is
its effect.
Conclusions:
44. (e) Both the statements (A) and (B) are effects of some
I. False
common cause.
II. False
45. (b) Clearly statement (B) is the cause and statement (A) is
its effect.
Hotels
Cones House Lodges Car
Stars 50. (e)
Triangles
46. (d) Kites Cones Triangles
OR

Conclusions: Hotels
I. False
II. False Lodges Car
From the above diagram it can say that there is
possibility that some stars are triangles.

Banjos Flute House


Drums

47. (d) Guitars Conclusions:


Flute I. True
II. True
y
o
u
rs
m
a
h
368 SBI Po Exam 2014

b
o
o
51 - 57 :
6 ´ 132

b
Rate of painting per sq. metre = ` 3200

.w
Breadth = = 7.92 metre
100
Rate of tiling per square metre

o
rd
3200 ´125 5 ´ 112

p
= Height = = 5.6 metre

re
100 100

s
Area of the four walls

s
= ` 4000

.c
= 2 × height (length + breadth)

o
Rate of carpeting per sq. metre

m
= 2 × 5.6 (7.2 + 7.92)
4000 ´ 120 = 2 × 5.6 × 15.12
= = 169.344 sq. metre
100
51. (a) Area of the four walls of the room Area of the floor = 7.2 × 7.92
= 2 × 5(6 + 6) = 120 sq. metre = 57.024 sq. metre
Painting Þ 3200 × 169.344
\ Expenditure on painting
= ` 541900.8
= ` (120 × 3200)
Tiling Þ 57.024 × 4000
Area of the floor = 6 × 6 = 36 sq. metre
= ` 228096
\ Expenditure on decoration
Total cost = ` (54190.8 + 228096)
= ` (36 × 4800 × 14)
56. (b) Carpeting Þ 6 × 6 × 9600 = ` 345600
\ Required ratio Decoration Þ 36 × 9600 × 14 = ` 4838400
= (120 × 3200) : (36 × 4800 × 14)
= 384000 : 2419200 = 10 : 63 345600 ´ 75
Electrification Þ
52. (b) Area of tiles on the walls 100
= 2 × 0.25 (6 + 6) = 6 sq. metre = ` 259200
\ Required total cost = ` Tiling Þ 6 × 6 × 8000
[(36 + 6) × 4000 + 36 × 4800 × 14] = ` 288000
= 168000 + 2419200 Total cost = 345600 + 4838400 + 259200 + 288000
= ` 2587200 = ` 5731200
53. (c) Area of the four walls of room 57. (c) Expenditure on decoration of the room
= 2 × 27 (21 + 42) = 3402 sq. metre 4838400 ´ 125
Area of the floor =
100
= 21 × 42 = 882 sq. metre
= ` 6048000
Painting Þ (3402 + 882) × 3200
58. (a) Sales revenue
= ` 13708800
= ` 725 thousand
Carpeting Þ 4800 × 882 Cost of production in 2008
= ` 4233600 = Rs. 500 thousand
Decoration Þ 4233600 × 14 \ Required percentage
= ` 59270400
225
4233600 ´ 75 = ´ 100 = 45%
Electrification Þ 500
100 59. (b) Average production cost
= ` 3175200
\ Total expenditure æ 625 + 500 + 450 + 400 + 500 + 525 ö
= ` çè ÷ø thousand
= ` (13708800 + 4233600 + 59270400 + 3175200) 6
= ` 80388000
3000
54. (d) Area of the floor = = ` 500 thousand
6
= 51 × 59 = 3009 sq. metre
Required years Þ 2004 and 2009.
4000 ´ 175 60. (c) Average sales revenue
Rate of tiling =
100
æ 750 + 725 + 550 + 600 + 800 + 800 ö
= ` 7000 = ` çè ÷ø thousand
6
\ Total cost = ` (3009 × 7000)
= ` 21063000 4225
55. (a) New length of the room = = ` 704 thousand
6
6 ´ 120 In years 2006 and 2007 sales revenue is 550 and 600
= = 7.2 metre respectively.
100
y
o
u
rs
m
a
h
SBI Po Exam 2014 369

b
o
o
61. (a) New production cost in the year 2004
500 ´ 125

b
Year 2008 Þ

.w
625 ´ 125 100
=

o
100

rd
= ` 625 thousand
= ` 781.25 thousand

p
525 ´135

re
New production cost in the year 2007 Year 2009 Þ

s
100

s
.c
400 ´ 130 = ` 708.75 thousand
=

o
100

m
Average cost
= ` 520 thousand
\ Total cost = 781.25 + 520 æ 480 + 625 + 708.75 ö
Þ ç ÷ thousand
= ` 1301.25 thousand è 3 ø
\ Required percentage
1813.75
æ 1301.25 - 800 ö =
3
=ç ÷ ´100
è 800 ø = ` 604.58 thousand
Difference
501.25 ´100
= = 62.7% = ` (779.17 – 604.58) thousand
800
= ` 174.59 thousand
62. (a) Percentage profit: 64. (a) Increase in wheat production:
æ 750 - 625 ö 2700 ´ 130
Year 2004 Þ ç ÷ ´100 = 20% Year 2003 Þ = 3510 quintals
è 625 ø 100
æ 750 - 500 ö 2900 ´ 140
Year 2005 Þ ç ÷ ´100 = 45% Year 2004 Þ = 4060 quintals
è 500 ø 100

æ 550 - 450 ö 3000 ´145


Year 2006 Þ ç ÷ ´100 = 22.2% Year 2005 Þ = 4350 quintals
è 450 ø 100

æ 600 - 400 ö 2900 ´ 140


Year 2007 Þ ç ÷ ´100 = 50% Year 2007 Þ
100
= 4060 quintals
è 400 ø
Total earlier production of wheat in Madhya Pradesh
æ 800 - 500 ö = 2400 + 3300 + 2900 + 2900 + 3000 + 2400 + 2800
Year 2008 Þ ç ÷ ´ 100 = 60%
è 500 ø = 19700 quintals
63. (c) Increase in sales revenue: New wheat production
725 ´ 120 = 2400 + 3300 + 3510 + 4060 + 4350 + 2400 + 4060
Year 2005 Þ = ` 870 thousand = 24080 quintals
100
Increase = 24080 – 19700
550 ´ 125 = 4380 quintals
Year 2006 Þ = ` 687.5 thousand Percentage increase
100
600 ´130 4380
= ´ 100 = 22%
Year 2007 Þ = ` 780 thousand 19700
100
Average revenue 65. (b) Required average production

870 + 687.5 + 780 3500 + 3000 + 2100


= =
3 3

2337.5 8600 2
= = = 2866 quintals
3 3 3
= ` 779.17 thousand 66. (c) Average production of Bihar.
Increase in production cost 2800 + 2700 + 2800 + 3800 + 2100 + 3800 + 3600
=
400 ´120 7
Year 2007 Þ
100
21600
= ` 480 thousand =
7
y
o
u
rs
m
a
h
370 SBI Po Exam 2014

b
o
o
67. (b) Increase in wheat production in Bihar: 71. (b) Average number of junior engineers

b
.w
2800 ´ 120 100 + 100 + 150 + 250 + 250 850
Year 2001 Þ = ` 3360 quintals

o
100 = = = 170

rd
5 5

p
re
2700 ´ 125 72. (c) Average number of assistant engineers
Year 2002 Þ = ` 3375 quintals

s
100

s
.c
150 + 300 + 200 + 150 + 100
=

o
2800 ´ 128 5

m
Year 2003 Þ = ` 3584 quintals
100
900
3800 ´135 = = 180
Year 2004 Þ = ` 5130 quintals 5
100
Required difference = 180 – 170 = 10
Total new production = 3360 + 3375 + 3584 + 5130 +
73. (d) In all companies:
2100 + 3800 + 3600
= 29949 quintals Assistant engineers = 150 + 300 + 200 + 150 + 100
Total earlier production = 2800 + 2700 + 2800 + 3800 + = 900
2100 + 3800 + 3600 = 21600 quintals Number after 37% increase
Increase = 29949 – 21600
900 ´137
= 8349 quintals = = 1233
100
8349
Average increase = quintals Postgraduate engineers = 100 + 250 + 400 + 550 + 400 =
7
1700
Required percentage increase
Number after 20% decrease
8349
1700 ´ 80
7 ´100 = = 1360
= 21600 = 38.7% 100
7 \ Required percentage
68. (d) Wheat production by three states: 1360 - 1233
Year 2002 Þ 3300 + 2900 + 2700 = 8900 quintals = ´ 100
1360
Year 2003 Þ 2800 + 2700 + 3700 = 9200 quintals
Year 2004 Þ 2900 + 3100 + 3800 = 9800 quintals 127
Total production in these years = ´ 100 = 9.3
1360
= 8900 + 9200 + 9800 = 27900 quintals
Year 2005 Þ 2100 + 3000 + 3500 = 8600 quintals 74. (b) Increase in the number of engineers:
Year 2006 Þ 2400 + 2900 + 3800 = 9100 quintals
400 ´ 130
Year 2007 Þ 2900 + 3400 + 3600 = 9900 quintals Company V Þ = 520
100
Total production = 8600 + 9100 + 9900
= 27600 quintals
700 ´135
Required percentage Company X Þ = 945
100
27900 - 27600
= ´ 100
27600 950 ´140
= 1.09% Company Y Þ = 1330
100
69. (a) Total wheat production in the year 2007
= (2900 + 3400 + 3600) quintals Total engineers = 520 + 945 + 1330 + 650 + 750
= 9900 quintals = 4195
70. (a) Post graduate engineers employed in company W Total original number of engineers = 400 + 650 + 700 +
= 200 950 + 750 = 3450
200 Percentage increase
\ Required percentage = ´ 100
600
4195 - 3450
= ´ 100 = 22%
100 1 3450
= = 33 %
3 3 = 22%
y
o
u
rs
m
a
h
SBI Po Exam 2014 371

b
o
o
75. (b) Required ratio = 150 : 200 = 3 : 4
86000 - 56000 30000

b
.w
= = = 3750
8 8

o
Country Total No. of
\ Required percentage

rd
employees male

p
3108

re
employees ´ 100 = 83%
=

s
I 15480 8400 3705

s
.c
II 10320 5600 81. (b) Total employees in countries V, VI and VII = 14620 +

o
III 6880 6720 13760 + 6020

m
IV 12040 10080 = 34400
V 14620 10080 Total male employees in countries II, III and IV.
VI 13760 3920 = 5600 + 6720 + 10080
VII 6020 5040 \ Required percentage
VIII 6880 6160
34400 - 22400
= ´ 100
76. (b) Required ratio 22400
= 8400 : 5600 = 15 : 12 = 5 : 4 12000 ´ 100
77. (c) Employees in country II: = = 53.6%
22400
Males = 5600
Females = 10320 – 5600 = 4720 82. (c) Academic books published by publisher M
Required ratio 7
= 5600 : 4720 = 70 : 59 = 28200 ´ = 19740
10
78. (c) Male employees: Academic books published by publisher P
Countries (I + II + III)
= 8400 + 5600 + 6720 = 20720 8
= 31200 ´ = 19200
13
20720
Required average = = 6907 Required difference
3
= 19740 – 19200 = 540
79. (d) Females in country IV 83. (b) Books distributed
= 12040 – 10080 = 1960
Females in country VII 33800 ´ 79
= = 26702
= 6020 – 5040 = 980 100
Required average Books given to each distributor
1960 + 980 2940 26702
= = = 1470 = = 1068
2 2 25
80. (a) New number of female employees: 84. (c) Non-academic books:
Country III
Total employees = 6880 6
Publisher R Þ 35700 × = 12600
Female employees = 6880 – 6720 = 160 17
Country IV 13
Total employees = 12040 Publisher S Þ 37800 × = 27300
18
Female employees = 12040 – 10080 = 1960
Country V 12600 + 27300
Total employees = 14620 Required average =
2
Female employees
Þ 14620 – 10080 = 4540 39900
= = 19950
Total female employees 2
= 160 + 1960 + 4540 85. (d) Required number distributed books
= 6660
29700 ´ 92 33800 ´ 79
= +
6660 100 100
Average = = 2220
3
39900
2220 ´ 140 = = 19950
2
New average = = 3108
100 86. (b) Total books published by Publishers P, Q and R
Earlier average 130
= (31200 + 33800 + 35700) ×
100
y
o
u
rs
m
a
h
372 SBI Po Exam 2014

b
o
o
98. Number of girls enrolled in swimming = 245
130

b
= 100700 × = 130910 Number of boys enrolled in swimming =

.w
100
(480 – 245) = 235

o
Books published by remaining publishers

rd
Required ratio = 245 : 235

p
80 = 49 : 47

re
= (28200 + 32200 + 29700 + 37800) × 99. (b) Total number of girls enrolled in swimming and drawing
100

s
s
together = (245 + 280) = 525

.c
127900 ´ 80

o
= = 102320 100. (e) Number of boys

m
100 = 3000 – 1750 = 1250
\ Required average \ Required percentage
130910 + 102320 1250
= = ´ 100
7 3000
233230 = 41.67 » 42
= = 33318 101. (e) The amount lying in demand deposits can be withdrawn
7
87. (b) Required ratio by the depositors as and when required. This amount
= 312 : 338 is as liquid as the money kept at home. Since the majority
= 156 : 169 of such Demand Deposit accounts are checking and
88. (a) Required average savings accounts, it means that that the bank is getting
190 + 280 + 280 + 315 + 316 + 423 money at low cost, since no interest is padi on the
= current accounts and the interest paid on savings
6
account is usually low.
1804
= » 300 102. (c) ICICI Bank Limited, India’s largest private sector bank,
6 on 26th February 2014, announced the launch of
89. (c) Required average ‘Branch on Wheels’ at Bhubaneshwar in Odisha, as
483 + 339 + 335 + 410 + 381 + 370 part of its financial inclusion plan that aims at providing
=
6 banking services in villages which are so far devoid of
banking facilities. This unique initiative will provide
2318
= » 386 basic banking services to the remote unbanked
6
villages.
90. (d) For the post of D in the years 2001, 2002 and 2003:
Appeared candidates Þ 420 + 577 + 487 = 1484 103. (e) The Government introduced the “National Agricultural
Qualified candidates Þ 364 + 315 + 290 = 969 Insurance Scheme” (NAIS) or “Rashtriya Krishi Bima
Difference = 1484 – 969 = 515 Yojana” (RKBY) in 1999-2000. NAIS envisages
91. (a) Required percentage coverage of all food crops (cereals and pulses),
240 oilseeds, horticultural and commercial crops. It covers
= ´ 100 = 73 all farmers, both loanees and non-loanees, under the
330
scheme.
92. (b) Required ratio
= 190 : 135 = 38 : 63 104. (a) Public sector banks contribute 86 percent of the total
93. (d) It is clear from the table. NPAs in the banking system. Currently, a loan is
Year 2006 Þ 417 classified as sub-standard – the first NPA category – if
94. (c) It is clear from the table year 2004 Þ 190 principal or interest payment is overdue for 9 days.
95. (c) Required average The share of new private sector banks and foreign
309 + 354 + 221 + 480 + 416 + 190 banks in the total NPAs has also gone up.
= 105. (a) Chandi Prasad Bhatt, the noted Gandhian
6
environmentalist and social activist won the Gandhi
1970
= » 328 Peace Prize for the year 2013 on 28 February 2014. Bhatt
6
was one of the pioneers of the Chipko movement for
350 which he was awarded with the Roman Magsaysay
96. (c) Required % = ´ 100 » 1167
3000 Award in 1982.
97. (a) Number of boys enrolled in singing and craft 106. (c) If a cheque bears a date which is yet to come (future
= (630 – 490) + (750 – 385) date) which is yet to come (future date) then it is known
= 140 + 365 as post-dated cheque (PDC). It is a form of a crossed or
= 505 account payee bearer cheque but postdated to meet
y
o
u
rs
m
a
h
SBI Po Exam 2014 373

b
o
o
the said financial obligation at a future date. Such a that is not used to store bulk data, so we use storage

b
.w
cheque cannot be honoured earlier than the date on devices like hard disk, floppy disk, compact disk (C),
the cheque. digital versatile disk (DVD) etc, which are called

o
rd
107. (c) Generally, there are Three Levels of Management, viz., secondary memory.

p
Administrative or Top Level of Management Executive

re
115. (b) CRAR is the acronym for capital to risk weighted assets

s
or Middle Level of Management and Supervisory or ratio, a standard metric to measure balance sheet

s
.c
Lower Level of Management. The Top Level strength of banks. BASEL 1 and BASEL II are global

o
m
Management determines the objectives, policies and capital adequacy rules that prescribe a minimum amount
plans of the organisation. of capital a bank has to hold given the size of its risk
108. (e) A peripheral is a “device that is used to put information weighted assets.
into or get information into or get information out of 116. (b) Advantage refers to best strategies that lead to the
the computer.” USB (Universal Serial Bus) is a serial creation and sustainability of a competitive advantage.
bus standard to interface devices. Similarly, peripherals In order to have that competitive advantage it must
can be connected via Bluetooth. possess a favourable asymmetry (e.g. lower unit cost,
109. (a) Preprocessing is the first pass of any complication. higher buyer willingness to pay for its product) between
Compilation is the second pass. It takes the output of itself and its competitors in the market.
the preprocessor, and the source code, and generates 117. (b) Axis Bank, country’s third largest private lender, opened
assembler source code. Assembly is the third stage of a branch in Shanghai in January 2014 after receiving
compilation which takes the assembly source code and permission from the China Banking Regulatory
produces an assembly listing with offsets. Linking is Commission. With this, it became the first domestic
the final stage of compilation. It takes one or more object
private sector lender to have a branch presence in China.
files or libraries as input and combines them to produce
118. (e) As per the ‘Policy Framework for Investment’ of OECD,
a single (usually executable) file.
there are ten principles to redress weaknesses in the
110. (e) Malicious programs are used to attackers to disrupt
investment environment. Three principles apply
computer operation, gather sensitive information, or
throughout the Fr amework. Policy coherence;
gain access to private computer systems. Normally
importance of a transparent approach to policy
such a program can be removed by simply uninstalling
formulation and implementation; Regular evaluation of
it. However, a reboot will be required to completely
the impact of existing and proposed policies on the
remove it from the system.
investment environment; competition policy, corporate
111. (b) Structured Financial Messaging System (SFMS),
governance, etc. are some of them.
launched in December 2001, is used practically for all
purposes of secure communication within the bank and 119. (c) The Finance Minister, Pranab Mukherjee, while
between banks. The intra-bank part of SFMS can be presenting the Union Budget 2010-11, announced the
used by the banks for secure messaging facility it National Social Security Fund for unorganized sector
provides. workers with an initial allocation of Rs. 1000 crore. This
112. (d) Bank of India, on Monday, in March 2014, launched fund will support schemes for weavers, toddy tappers,
IMT (Instant Money Transfer), which allows cardless rickshaw pullers, bidi worker setc.
cash withdrawal at its IMT-enabled ATMs, a first-of- 120. (a) A barcode is an optical machine-readable
its-kind initiative by any public sector bank. IMT allows representation of data relating to the object to which it
the customer to send money to a receiver only by using is attached. It serves as the Universal Product Code.
the receiver’s mobile number through the bank’s ATM Barcodes originally were scanned by special optical
and retail internet banking facility. scanners called barcode readers. Later, scanners and
113. (c) The Middle Level Management consists of the interpretive software became available on devices
Departmental Heads (HOD), Branch Managers, and the including desktop printers and smartphones.
Junior Executives. The Departmental heads are Finance 121. The devices which are used to input the data and the
Managers, Purchase Managers, etc. The Branch programs in the computer are known as “Input Devices”.
Managers are the head of a branch or local unit. The Keyboard, mouse, track ball and light pen are all input
lower level management consists of the Foremen and devices. Output Devices can produce the final product
the Supervisors. of machine processing into a form usable by humans.
114. (e) The process of copying software programs from For example, Bar code Reader reads bar codes an
secondary storage media to the system’s hard disk is converts them into electric pulses to be processed by
called installation. Computers have primary memory a computer.
y
o
u
rs
m
a
h
374 SBI Po Exam 2014

b
o
o
122. (b) A spreadsheet is an interactive computer application the Stree Shakti Puraskar is a Women’s Award in the

b
.w
program for organization and analysis of data in tabular name of eminent women personalities in Indian history,
form. The program operates on data represented as who are famous for their personal courage and

o
rd
cells of an array, organized in rows and columns. integrity.

p
Microsoft Excel is a spreadsheet application that

re
131. (a) According to ‘Introduction to Marketing Management’

s
features calculation, graphing tools, pivot tables, etc. by Osman Safdar Sarwant, the two fold goal of marketing

s
.c
123. (d) Public sector general insurer, New India Assurance, in is to attract new customers by promising superior value

o
m
March 2014, launched an exclusive health Insurance which also keeping current customers by delivering
cover, New India Asha Kiran, for a family with girl satisfaction. Sarwani outlined a five step model of the
children. The family floater health insurance, gives a marketing process.
50 percent discount for the girl child. It also provides a 132. (b) To open a file, the easiest way is to left click with the
personal accident cover for parents and an occurrence mouse twice. Another way is to right click the mouse
of any accident to parents. and select‘OPEN’.
124. (b) Jaswant Singh, External Affairs and Finance Minister 133. (b) A remote computer is a computer to which a user does
in the previous NDA government released a book not have physical access, but when he or she can
entitled India at Risk – Mistakes, Misconceptions of access or manipulate via some kind of network. Remote
security policy in March 2014. In this book, he has desktop software allows a person to control a computer
discussed th e issues, the conflicts and the from another computer; this allows the user to change
consequences of the security related policy of India. anything on the linear computer, and access all of the
125. (b) General Insurance Corporation of India had four fully file contents.
owned subsidiary companies: National Insurance 134. (a) At present, banks calculate interest paid on money
Company Limited, New India Assurance Company kept in the savings bank account on a daily basis. This
Limited, Oriental Insurance Company Limited and is a departure from the earlier practice of calculating
United India Insurance Company Limited. It is the sole interest on the lowest balance after the 10th of every
reinsurance company in the Indian insurance market month. In 2011, the central bank had decided to give
with over four decades of experience. freedome to commercial banks to fix savings bank
126. (c) The 2014 summit of North Atlantic Treaty Organization deposit rates, the last bastion of the regulated interest-
(NATO) will be held in New Port, Wales, United rate regime.
Kingdom, in September 2014. Russia, China, India and 135. (c) The Indian Financial System Code (IFSC) is an
Australia are not the members of NATO which is an alphanumeric code that uniquely identifies a bank-
international alliances that consists of 28 member states branch participating in the two main electronic funds
from North America and Europe. settlement systems in India: the real time gross
127. (c) Market segmentation pertains to the division of a set settlement (RTGS) and the national electronic funds
of consumers into persons with similar needs and wants. transfer (NEFT) systems.
Market segmentation allows for a better allocation of a 136. (a) When we save a file, we can save it to a folder on our
firm’s finite resources. Due to limited resources, a firm hard disk drive, a network location, disk, DVD, CD, the
must make choices in servicing specific groups of desktop, flash drive, or save as another file format.
consumers. So it can be defined in terms of the STP Save As option is the way to save the data to a different
acronym, meaning Segment, Target and Position. file format.
128. (e) In 1996 PC makers started switching to the ATX 137. (c) The Directorate General of Civil Aviation (DGCA), in
standard which defined a new 20 pin motherboard May 2014, granted an air operator’s permit (AOP) to
power connector. It has a standby 5 volt rail which is AirAsia, paving the way for the airline company to
always on even when the power supply is turned off to launch low-cost services in the country. AirAsia India
provide standby power to the motherboard when the is a joint venture among Malaysia’s AirAsia Berhad,
machine is sleeping. Tata Sons and Telestra Tradeplace.
129. (c) Sri Lanka, on 8 March 2014, beat Pakistan in the final to 138. (e) Repo rate is the rate at which the central bank of a
become Asia Cup champions for the fifth time. The country (RBI in case of India) lends money to
2014 Asia Cup was the twelfth edition of the Asia Cup commercial banks in the event of any shortfall of funds,
cricket tournament. The tournament was held in thereby controlling liquidity. Repo rate is used by
Bangladesh. monetary authorities to control inflation. Repo and
130. (a) President Pranab Mukherjee on the occasion of reverse repo rates form a part of the liquidity adjustment
International Women’s Day on 8 March 2014, awarded facility.
y
o
u
rs
m
a
h
SBI Po Exam 2014 375

b
o
o
139. (c) The Banking Ombudsman Scheme is introduced under forcasting and has certain advantage over another

b
.w
Section 35 A of the Banking Regulation Act, 1949 by structured forecasting approach, prediction markets.
RBI with effect from 1995. The Banking Ombudsman 148. (e) Segmentation is the process of slicing a market for a

o
rd
Scheme was introduced under Section 35A of the particular product or service into a number of different

p
Banking Regulation Act, 1949 by RBI with effect from segments. The segments are usually based on factors

re
s
1995. such as demographics, beliefs or the occassion of use

s
.c
140. (a) According to the 2011 Census, Maharashtra has of the product. Age and gender are demographic

o
m
highest number of people living in urban areas (5.08 components of segmentation.
crores). Maharashtra state has 45.23 per cent of the 149. (a) Innovation is about finding a better way of doing
urban population. With these figures, Maharashtra something. Innovation can be viewed as the application
ranks the third most urbanised states in the country, of better solutions that meet new requirements. In-
behind Tamil Nadu (48.45%) and Kerala (47.72%) articulated needs, or existing market needs. This is
141. (b) The Reserve Bank of India said its move to withdraw accomplished through more effective products,
currency notes printed before 2005 is to check processes, services, technologies, or ideas that are
counterfeit notes and these notes will continue to be readily available to markets.
legal tender even after the July deadline. It clarified 150. (e) Customer relationship management (CRM) is a system
that the move to withdraw has been taken because for managing a company’s interactions with current
they have fewer security features compared to and future customers. It involves using technology to
banknotes printed after 2005. organize, automate and synchronize sales, marketing,
142. (a) Anand Gandhi’s ‘Ship of Theseus’ won the Best Film customer service, and technical support.
award at the 61st National Film Awards. It is a complex 151. (c) 152. (d) 153. (a) 154. (d) 155. (d) 156. (a)
and philosophical film about an unusual photographer, 157. (a) Achilles heel (Noun) = a weak point or fault in
a Jain monk and a stock broker. Rakeysh Omprakash somebody’s character which can be attacked by other
Mehra directed ‘Bhaag Milkha Bhaag’ won the award people.
of the most popular film. 158. (b) Sustain (Verb) = to make something continue for
143. (a) RTGS stands for ‘Real Time Gross Settlement’. RTGS sometime without becoming less; maintain.
is a funds transfer system where transfer of money Look at the sentence:
takes place from one bank to another on a real time
She managed to sustain everyone’s interest until the
basis. This is the fastest mode of funds transfer
end of her speech.
available in India through banking channel.
159. (c) Cumbersome (Adjective) = large and heavy; difficult
144. (b) Niche marketing is concentrating all marketing efforts
to carry; bulky; complex; complicated.
on a small but specific and well defined segment of the
Look at the sentence:
population. A niche market is the subset of the market
Government should ease the cumbersome legal
on which a specific product is focused. The niche
procedures.
market is highly specialized, and aiming to survive
among the competition from numerous super 160. (d) Differentiated (Adjective)
companies. = distinguished; to be the particular thing; important.
145. (a) The Marketing Mix, also known as the 4 P’s of Look at the sentence:
Marketing, is the combination of product, price, place I think grey hair makes you look very differentiated.
(distribution), and promotion. These four P’s are the 161. (e) 162. (a) 163. (a) 164. (e) 165. (e) 166. (e)
parameters that the marketing manager can control, 167. (e) Austerity (Noun) = simplicity; a situation when people
subject to the internal and external constraints of the do not have much money to spend because there are
marketing environment. bad economic conditions.
146. (e) A SWOT analysis is a structure planning method used Look at the sentence:
to evaluate the strengths, weaknesses, opportunities, War was followed by many years of austerity.
and threats involved in a project of in a business 168. (a) Inroads (Noun) = something that is achieved, especially
venture. It can be carried out for a product, place, by reducing the success of something else.
industry or person. Look at the sentence:
147. (b) The Delphi method is a structured communication Tax rises have made some inroads into the country’s
technique, originally developed as a systematic, national debt.
interactive forcasting method which relies on a panel 169. (a) Aggravate (Verb) = to make a bad/unpleasant situation
of experts. It has been widely used for business worse; worsen
y
o
u
rs
m
a
h
376 SBI Po Exam 2014

b
o
o
Assuage (Verb) = to make an unpleasant situation less 174. (a) Affable (Adjective) = pleasant, friendly and easy to

b
.w
severe. talk to; genial.
170. (b) Simmering (Adjective) = to be filled with a strong Rude (Adjective) = discourteous; impolite; showing

o
rd
feeling especially anger which you have difficulty in lack of respect for other people.

p
controlling; seething.

re
Hence, antonymous relationship.

s
Calm = peaceful. 175. (e) Partial (Adjective) = showing or feeling too much

s
.c
Look at the sentence: support for one person, idea etc. in a way that is unfair;

o
m
There was simmering anger among agitators. biased.
171. (c) Dominate (Verb) = to control or have a lot of influence Dispassionate (Adjective) = not influenced by emotion;
over somebody/something; to be the most important impartial.
feature of something. Hence, antonymous relationship.
Monopolise (Verb) = to have or take control of the 176. (c) 177. (a) 178. (b) 179. (c) 180. (b) 181. (a)
larger part of something. 182. (e) 183. (d) 184. (c) 185. (e)
Hence, synonymous relationship. 186. (a) Here subject (tractor sales) is plural. Hence, ..... tractor
172. (b) Equitable (Adjective) = fair and reasonable; treating sales have seen ....... should be used.
everyone in an equal way. 187. (b) It is preposition related error. Hence, .......... that share
Impartial (Adjective) = not supporting one person or the earth with us should be used.
group more than other; unbiased. 189. (b) Here, Noun i.e., environment and forests is ....... should
Hence, synonymous relationship. be used.
173. (c) Flourish (Verb) = to develop quickly and be successful; 190. (d) Here, Infinitive i.e., To portray is subject. Hence,
to grow well; thrive. singular verb i.e. references is very difficult should be
Look at the sentence: used.
Few businesses are flourishing in the present economic 191. (b) 192. (c) 193. (a) 194. (b) 195. (d) 196. (a)
climate. 197. (d) 198. (e) 199. (a) 200. (d)
y
o
u
rs
m
a
h
b
o
IBPS SPECIALIST (I.T.) OFFICER

o
b
.w
o
EXAM 2014 Based on Memory

rd
p
re
s
s
.c
QUANTITATIVE APTITUDE (a) ` 6,49,900/- (b) ` 6,45,900/-

o
m
(c) ` 6,49,400/- (d) ` 6,49,500/-
DIRECTIONS (Qs. 1-5) : Study the following pie-chart and (e) None of these
table carefully and answer the questions given below : 4. The number of mobile phones sold of company A during
July is approximately what percent of the number of mobile
PERCENTAGE WISE DISTRIBUTION OF THE
phones sold of company A during December ?
NUMBER OF MOBILE PHONES SOLD BY
A SHOPKEEPER DURING SIX MONTHS (a) 110 (b) 140
Total number of mobile phones sold = 45,000 (c) 150 (d) 105
(e) 130
5. What is the total number of mobile phones sold of company
December B during August and September together ?
16%
November (a) 10,000 (b) 15,000
July
12% (c) 10,500 (d) 9,500
17%
October (e) None of these
8%
DIRECTIONS (Qs. 6-10) : Answer the following questions based
September August
22% on the given line-graph.
25%
Percentage population growth of five cities in three
five -year intervals from 1998 to 2013.

The respective ratio between the number of mobile phones


sold of company A and company B during six months 90 1998 to 2003 2003 to 2008
2008 to 2013
Month Ratio 90
80
July 8:7 80
70 75
August 4:5 70
%population growth

September 3:2 60
60 65
October 7:5 50
November 7:8 50 55
45 50
December 7:9 40
30 40 40
1. What is the respective ratio between the number of mobile 30 30
phones sold of company B during July and those sold during 20 25
December of the same company ? 10
(a) 119:145 (b) 116:135
0
(c) 119 :135 (d) 119:130
A B C D E
(e) None of these
2. If 35% of the mobile phones sold by company A during
6. If the population of City B was 16 lakh in the year 1998, then
November were sold at a discount, how many mobile phones
what is its population in the year 2013?
of company A during that month were sold without a
discount? (a) 54.161akh (b) 56.28 lakh
(a) 882 (b) 1635 (c) 57.12 lakh (d) 58.6 lakh
(c) 1638 (d) 885 (e) None of these
(e) None of these 7. If the population of City D in 2008 was 13.65 lakh, then what
was its population in the year 1998?
3. If the shopkeeper earned a profit of `433/- on each mobile
phone sold of company B during October, what was his (a) 8.4 lakh (b) 8.2 lakh
total profit earned on the mobile phones of that company (c) 7.8 lakh (d) 7.6 lakh
during the same month ? (e) None of these
y
o
u
rs
m
378 IBPS Specialist (I.T.) Officer Exam 2014

a
h
b
8. If the population of City A and City C was equal to 12 lakh (a) 47.5% (b) 56.25%

o
each in the year 1998, then what is the difference between (c) 62.5% (d) 65%

o
b
the population of City C and the population of City A in the (e) 67.5%

.w
year 2003? 10. If the population of City E and the population of City C in

o
the year 1998 was 20 lakh and 24 lakh respectively, then

rd
(a) 2.4 lakh (b) 1.84 lakh
population of city C is what percentage of the population of

p
(c) 1.62 lakh (d) 1.4 lakh

re
City E in the year 2003?

s
(e) 1.2 lakh (a) 72% (b) 84%

s
.c
9. Population of City D in the year 2008 is what percentage (c) 87.5% (d) 92%

o
m
more than its population in the year 1998? (e) 96%
DIRECTIONS (Qs. 11-15) : Study the table carefully to answer the following questions.
Number of cars (in thousands) manufactured and sold by six companies over the years.

Company ® A B C D E F
Year ¯ M S M S M S M S M S M S
2008 2.58 1.96 1.98 1.62 1.97 1.53 2.46 2.11 2.35 2.16 1.88 1.50
2009 2.34 1.98 2.15 2.00 2.20 2.03 2.46 2.14 2.45 2.20 1.95 1.62 M - Manufactured
2010 2.85 2.05 2.35 1.99 2.18 1.87 2.55 2.23 2.60 2.13 2.25 1.93 S - Sold
2011 2.87 2.11 2.62 2.01 2.25 1.95 2.62 2.30 2.79 2.31 2.39 2.08
2012 2.91 2.22 2.71 2.12 2.68 2.32 2.71 2.19 2.88 2.19 2.58 2.10
2013 2.94 2.25 2.84 2.15 2.86 2.36 2.76 2.28 2.90 2.32 2.67 2.30

11. What is the ratio of the total number of cars manufactured 15. What is the total number of cars sold by Company C in all
by Companies A, B and C together in the year 2009 to that the years together?
of those manufactured by Companies D, E and F together (a) 120600 (b) 14205
(c) 12060 (d) 14050
in the year 2011?
(e) None of these
(a) 164 : 217 (b) 223 : 260
DIRECTIONS (Qs. 16-20) : Study the following information
(c) 260 : 223 (d) 217 : 164 carefully and answer the questions given below it.
(e) None of these
Out of the 15,000 candidates eligible for an Officer’s post in
12. What is the percentage of the number of cars sold by a Public Sector Bank, 450 candidates have prior experience of
Company D in the year 2010 with respect to that of those working in Public Sector banks in rural area only. 25% of the total
number of candidates have prior experience of working in Public
manufactured by it in that year? (rounded off to two digits
Sector Banks in urban areas only. 12% of the total number of
after decimal) candidates have prior experience of working in Private Sector
(a) 87.45 (b) 77.28 Banks in urban areas only. 2% of the total number of candidates
have prior experience of working in Private Sector banks in rural
(c) 92.54 (d) 79.65 areas only. 3,600 candidates have worked in Public sector banks
in both urban and rural areas. 600 candidates have worked in both
(e) None of these
Public and Private Sector Banks in rural areas only. The remaining
13. In which year was the maximum number of cars manufactured candidates have no prior experience of working in the Banking
by all companies together? industry.
16. How many candidates have prior experience of working in
(a) 2009 (b) 2010 rural areas (both Public Sector and Private Sector Banks
(c) 2011 (d) 2012 together)?
(e) None of these (a) 4,350 (b) 4,950
(c) 4,800 (d) 4,900
14. What is the approximate per cent increase in the number of
(e) 4,850
cars sold by Company F in the year 2012 from the previous
year? 17. How many candidates have prior experience of working in
Public Sector Banks (Urban and Rural areas together)?
(a) 13 (b) 19 (a) 12,450 (b) 8,400
(c) 2 (d) 1 (c) 10,050 (d) 10,650
(e) 23 (e) None of these
y
o
u
rs
m
IBPS Specialist (I.T.) Officer Exam 2014 379

a
h
b
18. What is the ratio of the candidates who have a prior 26. A sum of money at simple interest amounts to `5852 in

o
experience of working in Public Sector Banks in rural areas

o
3 years and `7788 in 7 years. What is the rate of interest per

b
only to the candidates who have a prior experience of working

.w
annum?
in Private Sector Banks in rural areas only?

o
(a) 8% (b) 9%
(a) 4 : 3 (b) 3 : 2

rd
(c) 10% (d) 11%

p
(c) 2 : 3 (d) 3 : 4

re
(e) 12%
(e) None of these

s
27. The simple interest accrued on an amount of ` 22,500 at the

s
19. What is the total number of candidates who have worked in

.c
end of four years is ` 10,800. What would be the compound
Private Sector Banks in urban areas?

o
interest accrued on the same amount at the same rate at the

m
(a) 1,800 (b) 2,250
end of two years ?
(c) 4,050 (d) 3,600
(a) ` 16,908 (b) ` 5,724
(e) None of these
(c) ` 28,224 (d) ` 8,586
20. The candidates who have no prior experience of working in
(e) None of these
the banking industry are what per cent of the candidates
who have worked in Public Sector Banks in both urban and 28. The compound interest earned by Suresh on a certain
rural areas together? amount at the end of two years at the rate of 8 p.c.p.a. was
(a) 60.5 (b) 63.5 `1,414.40. What was the total amount that Suresh got back
(c) 62 (d) 64 at the end of two years in the form of principal plus interest
(e) None of these earned?
21. A car travels a distance of 560 km in 9.5 hours partly at a (a) ` 9,414.40 (b) ` 9,914.40
speed of 40 kmh –1 and partly at 160 kmh–1. What is the (c) ` 9,014.40 (d) ` 8,914.40
(e) None of these
distance it travel at the speed of 160 kmh–1?
29. Sonika invested an amount of ` 5800 for 2 years. At what
(a) 120 km (b) 240 km
rate of compound interest will she get an amount of ` 594.50
(c) 320 km (d) 360 km
at the end of two years?
(e) 420 km
(a) 5 pcpa (b) 4 pcpa
22. Two workers A and B working together completed a job in 5
(c) 6 pcpa (d) 8 pcpa
days. If A worked twice as efficiently as he actually did and
(e) None of these
1 30. Mr. ‘X’ invested certain amounts in two different schemes
B worked as efficiently as he actually did, the work would
3 ‘A’ & ‘B’. Scheme ‘A’ offers simple interest at 12 p.c.a. and
have completed in 3 days. Find the time for A to complete Scheme ‘B’ offers compound interest at 10 p.c.p.a. Interest
the job alone. accrued on the amount invested in Scheme A in 2 years was
` 3600 and the total amount invested was ` 35,000. What
1 3 was interest accrued on the amount invested in Scheme
(a) 6 days (b) 5 days
4 4 ‘B’?
(c) 5 days (d) 3 days (a) ` 4,800 (b) ` 4,200
(e) None of these (c) ` 4,000 (d) Cannot be determined
23. A man bought a horse and a carriage for ` 3000. He sold the (e) None of these
horse at a gain of 20% and the carriage at a loss 10%, thereby 31. A rectangular pool 20 metres wide and 60 metres long is
gaining 2% on the whole. Find the cost of the horse. surrounded by a walkway of uniform width. If the total area
(a) ` 1000 (b) ` 1200 of the walkway is 516 square metres, how wide, in metres, is
(c) ` 1500 (d) ` 1700 the walkway?
(e) None of these (a) 43 (b) 4.3
24. Naresh purchased a TV set for `11,250 after getting discount (c) 3 (d) 3.5
(e) None of these
of 10% on the labelled price. He spent `150 on transport
32. The sum of the areas of two circles which touch each other
and `800 on installation. At what price should it be sold so
externally is 153p. If the sum of their radii is 15, find the
that the profit earned would be 15% if no discount was
ratio of the larger to the smaller radius
offered?
(a) 4 (b) 2
(a) `12,937.50 (b) `14,030 (c) 3 (d) None of these
(c) ` 13,450 (d) `15,467.50 (e) None of these
(e) None of these 33. Euclid has a triangle in mind. Its longest side has length 20
25. Prathik sold a music system to Karthik at 20% and Karthik and another of its sides has length 10. Its area is 80. What is
sold it to Swasthik at 40% gain. If Swasthik paid ` 10,500 for the exact length of its third side?
the music system, what amount did Prathik pay for the same?
(a) 260 (b) 250
(a) ` 8,240 (b) ` 7,500
(c) ` 6,250 (d) Cannot be determined (c) 240 (d) 270
(e) None of these (e) None of these
y
o
u
rs
m
380 IBPS Specialist (I.T.) Officer Exam 2014

a
h
b
34. The length, breadth and height of a room are in the ratio 39. How much profit did Mahesh earn on the cost price of an

o
3:2:1. If the breadth and height are halved while the length is article by selling it?

o
b
doubled, then the total area of the four walls of the room A. He got 15% discount on the marked price at the time of

.w
will purchase.

o
(a) remain the same (b) decrease by 13.64%

rd
B. He sold it for ` 3060.
(c) decrease by 15% (d) decrease by 18.75%

p
C. He earned 2% profit on the marked price.

re
(e) decrease by 30%
(a) Only A and B both together are necessary.

s
35. A right circular cone of height h is cut by a plane parallel to

s
.c
the base and at a distance h/3 from the base, then the volumes (b) Only B and C both together are necessary.

o
of the resulting cone and frustum are in the ratio (c) Only A or C and B together are necessary.

m
(a) 1 : 3 (b) 8 : 19 (d) Even A, B and C all together are not sufficient to answer
(c) 1 : 4 (d) 1 : 7 the question.
(e) None of these (e) All three A, B and C together are necessary.
DIRECTIONS (Qs. 36-40) : Each of the questions below consists 40. How much marks did Arun secure in English?
of a question and three statements denoted A, B and C are given A. The average marks obtained by Arun in four subjects
below it. You have to study the questions and all the three included English is 60.
statements and decide whether the question can be answered B. The total marks obtained by him in English and
with any one or two of the statements or all the statements are Mathematics together is 170.
required to answer the question.
C. The total marks obtained by him in Mathematics and
36. What is R’s share of profit in a joint venture? Science together is 180.
A. Q started business investing ` 80,000/- (a) All three A, B and C together are necessary.
B. R joined him after 3 months. (b) Only A and B together are necessary
C. P joined after 4 months with a capital of ` 1,20,000 and (c) Only B and C together are necessary.
got ` 6,000 as his share of profit. (d) Only A and C together are necessary.
(a) Only A and C are required (e) None of these
(b) Only B and C are required 41. What is the value of m which satisfies 3m2 – 21m + 30 < 0 ?
(c) All A, B and C together are required (a) m < 2 or m > 5 (b) m > 2
(d) Even with all A, B and C the answer cannot be arrived (c) 2 < m < 5 (d) m < 5
(e) None of these (e) None of these
37. What is the area of a right angled triangle?
42. If one root of x 2 + px + 12 = 0 is 4, while the equation
A. The perimeter of the triangle is 30 cm.
B. The ratio between the base and the height of the triangle x 2 + px + q = 0 has equal roots, then the value of q is
is 5 : 12.
(a) 49/4 (b) 4/49
C. The area of the triangle is equal to the area of a rectangle
(c) 4 (d) 1/4
of length 10 cms.
(e) None of these
(a) Only B and C together are required
43. Let p and q be the roots of the quadratic equation
(b) Only A and B together are required
(c) Only either A or B and C together are required. x 2 - ( a - 2) x - a - 1 = 0 . What is the minimum possible
(d) Only A and C together are required value of p 2 + q 2 ?
(e) None of these
(a) 0 (b) 3
38. What will be sum of two numbers?
(c) 4 (d) 5
A. Among the two numbers, the bigger number is greater
(e) None of these
than the smaller number by 6.
44. If the roots, x1 and x2, of the quadratic equation x2 – 2x + c
B. 40% of the smaller number is equal to 30% of the bigger = 0 also satisfy the equation 7x2 – 4x1 = 47, then which of
number. the following is true?
C. The ratio between half of the bigger number and (a) c = – 15 (b) x1 = –5, x2 = 3
1 (c) x1 = 4.5, x2 = –2.5 (d) c = 15
rd of the smaller number is 2 : 1.
3 (e) None of these
(a) Only B and C together are necessary 45. If the sum of a number and its square is 182, what is the
(b) Only A and B together are necessary number?
(c) Out of A, B and C any two together are necessary (a) 15 (b) 26
(d) All three A, B and C together are necessary (c) 28 (d) 13
(e) None of these (e) None of these
y
o
u
rs
m
IBPS Specialist (I.T.) Officer Exam 2014 381

a
h
b
DIRECTIONS (Qs. 46-50) : In each of the following questions a 54. P is related to V in the same way as C is related to F. Which

o
of the following is E related to, following the same pattern?

o
number series is given. Only one number is wrong in each series.

b
Find out that wrong number, and taking this wrong number as (a) B (b) D

.w
the first term of the second series formed following the same (c) C (d) A

o
rd
logic, find out the third term of the second series. (e) None of these

p
55. Which of the following is true regarding F?

re
46. 1 2 8 21 88 445

s
(a) 24.5 (b) 25 (a) F sits second to right of C.

s
.c
(c) 25.5 (d) 25 (b) F is not an immediate neighbour of A.

o
(e) None of these (c) F sits third to left of D.

m
47. 6 7 18 63 265 1365 (d) F sits at one of the extreme ends of the line.
(a) 530 (b) 534 (e) F faces V.
(c) 526 (d) 562
(e) None of these DIRECTIONS (Qs. 56-60) : In each of the questions below are
48. 7 23 58 127 269 555 given four statements followed by four conclusions numbered I,
(a) 263 (b) 261 II, III and IV. You have to take the given statements to be true
(c) 299 (d) 286 even if they seem to be at variance with commonly known facts.
(e) None of these Read all the conclusions and then decide which of the given
49. 5 4 9 18 66 195 conclusions logically follows from the given statements
(a) 12 (b) 25 disregarding commonly known facts.
(c) 20 (d) 18 56. Statements: All snakes are eagles.
(e) None of these Some eagles are rabbits.
50. 2 7 28 146 877 6140
All rabbits are birds.
(a) 242 (b) 246
(c) 252 (d) 341 Some birds are animals.
(e) None of these Conclusions: I. Some animals are snakes.
II. Some birds are snakes.
REASONING ABILITY III. Some birds are eagles.
IV. All birds are rabbits.
(a) None follows (b) Only II follows
DIRECTIONS (Qs. 51-55): Study the following information to
(c) Only III follows (d) Both II and III follow
answer the given questions:
(e) None of these
Twelve people are sitting in two parallel rows containing six 57. Statements: Some cameras are calculators.
people each, in such a way that there is an equal distance between
Some calculators are diaries.
adjacent persons, In row 1, P, Q, R, S, T and V are seated and all of
them are facing south. In row 2, A, B, C, D, E and F are seated and All notebooks are diaries.
all of them are facing north. Therefore, in the given seating All diaries are computers.
arrangement each member seated in a row faces another member Conclusions: I. Some notebooks are calculators
of the other row. II. Some calculators are computers.
A sits third to right of D. Neither A nor D sits at extreme III. All notebooks are computers.
ends. T faces D. V does not face A and V does not sit at any of the IV. Some diaries are cameras.
extreme ends. V is not an immediate neighour of T. B sits at on of the (a) None follows (b) Only II follows
extreme ends. Only two people sit between B and E. E does not face (c) Only III follows (d) Both II and III follows
V. Two persons sit between R and Q. R is not an immediate neighbour (e) None of these
of T. C does not face V. P is not an immediate neighbour of R.
58. Statements: All planets are stars.
51. Who amongst the following sit at extreme ends of the rows? All stars are asteroids.
(a) B, E (b) S, T All asteroids are moons.
(c) P, R (d) B, F Some moons are rocks.
(e) None of these Conclusions: I. All asteroids are planets.
52. Who amongst the following faces A? II. All asteroids are stars.
(a) R (b) T III. All moons are stars.
(c) P (d) Q IV. Some rocks are stars.
(e) S (a) None follows
53. How many persons are seated between T and S? (b) Only I follows
(a) One (b) Two (c) Only II follows
(c) Three (d) Four (d) Only either I or II follows
(e) None (e) None of these
y
o
u
rs
m
382 IBPS Specialist (I.T.) Officer Exam 2014

a
h
b
59. Statements: Some bats are toys. 65. What is M’s mother tongue?

o
Some toys are plastics. (a) Bangla (b) Marathi

o
b
Some plastics are mirrors. (c) Telugu (d) Cannot be determined

.w
No mirror is glass. (e) None of these

o
Conclusions: I. Some toys are mirrors.

rd
66. Advertisement: Today’s customers expect high quality.
II. Some plastics are glasses.

p
Every advance in the quality of manufactured products

re
III. Some bats are mirrors.

s
IV. No glass is plastic. raises customer expectations. The company that is satisfied

s
.c
(a) Only III follows with the current quality of its products will soon find that

o
its customers are not. At MegaCorp, meeting of exceeding

m
(b) Only either II or IV follows
(c) Only either I or III follows customer expectations is our goal.
(d) Only either III or IV follows Which of the following must be true on the basis of the
(e) None of these statements in the advertisement above?
60. Statements: All graduates are advocates (a) MegaCorp’s competitors will succeed in attracting
Some advocates are judges. customers only if those competitors adopt
All judges are lawyers. MegaCorp’s goal as their own.
Some lawyers are doctors. (b) A company that does not correctly anticipate the
Conclusions: I. Some doctors are advocates. expectations of its customers is certain to fail in
II. All graduates are judges. advancing the quality of its products.
III. Some doctors are graduates.
(c) MegaCorp’s goal is possible to meet only if continuing
IV. Some lawyers are advocates.
advances in product quality are possible.
(a) None follows
(b) Only I follows (d) If a company becomes satisfied with the quality of its
(c) Only II follows products, then the quality of its products is sure to
(d) Either III or IV follows decline.
(e) None of these (e) None of these
67. ‘A + B’ means ‘A is the son of B’, ‘A – B’ means ‘A is the
DIRECTIONS (Q. 61-65) : Study the following information and
wife of B’. ‘A × B’ means ‘A is the brother of B’, ‘A ¸ B’
answer the questions given below it. means ‘A is the mother of B’, ‘A = B’ means ‘A is the sister
Seven members H, I, J, K, L, M and N are working in different of B’. Which of the following represents P is the maternal-
cities Ahmedabad, Bangalore, Chennai, Hyderabad, Kolkata, Delhi uncle of Q?
and Mumbai, not necessarily in the same order. Each one has a (a) R × P ¸ Q (b) P × R ¸ Q
different mother tongue–Tamil, Kannada, Telugu, Hindi, Marathi, (c) P + R ¸ Q (d) P + R × Q
Punjabi and Bangla, not necessarily in the same order. (e) None of these
J works in Bangalore and his mother tongue is not Tamil or DIRECTIONS (Qs. 68) : Read the following information carefully
Marathi. K’s mother tongue is Punjabi and he works in Ahmedabad. and answer the questions which follow.
L and M do not work in Chennai and none of them has Marathi
(i) If ‘A × B’ means ‘A is the son of B’.
mother tongue. I works in Hyderabad and his mother tongue is
Telugu. The one who works in Delhi had Bangla mother tongue. (ii) If ‘A + B’ means ‘A is the father of B’.
N works in Mumbai and his mother tongue is Hindi. L does not (iii) If ‘A > B’ means ‘A is the daughter of B’.
work in Kolkata. (iv) If ‘A < B’ means ‘A is the wife of B’.
61. What is J’s mother tongue? 68. Which of the following pairs of people represent first cousins
(a) Telugu (b) Hindi with regard to the relations given in the expressions, if it is
(c) Bangla (d) Kannada provided that A is the sister of J:
(e) None of these ‘L > V < J + P’ and ‘S × A < D + F < E + K’
62. Who works in Chennai? (a) LP (b) SP
(a) H (b) L (c) SK (d) SF
(c) M (d) L or M (e) Cannot be determined
(e) None of these 69. A school bus driver starts from the school, drives 2 km
63. Which of the following combinations is correct? towards North, takes a left turn and drives for 5 km. He then
(a) Marathi-I-Hyderabad (b) Tamil-M-Kolkata takes a left turn and drives for 8 km before taking a left turn
(c) Marathi-I-Chennai (d) Punjabi-K-Delhi again and driving for 5 km. The driver finally takes a left turn
(e) None of these and drives 1 km before stopping. How far and towards which
64. Who works in Delhi? direction should the driver drive to reach the school again?
(a) H (b) M (a) 3 km towards North (b) 7 km towards East
(c) L (d) K (c) 6 km towards South (d) 6 km towards West
(e) None of these (e) 5 km towards North
y
o
u
rs
m
IBPS Specialist (I.T.) Officer Exam 2014 383

a
h
74. Statement: H d J, J # N, N @ R, R d W

b
DIRECTIONS (Qs. 70 - 71) : Study the following information to

o
Conclusions

o
answer the given questions:

b
I. W % N II. W % H

.w
In a five letter English word (which may or may not be a
III. R # J IV. R d J

o
meaningful English word), there are two letters between L and P.

rd
S is not placed immediately next to L. There is only one letter (a) Only I is true (b) Only II is true

p
between S and A. S is towards the right of A. S is not placed (c) Only III is true (d) Only IV is true

re
immediately next to E.

s
(e) Either III or IV is true

s
.c
70. Which of the following is correct with respect to the word 75. Statement: B @ D, D d F, F % M, M ¬ N.

o
thus formed?

m
Conclusions
(a) E is at one of the extreme ends of the word. I. B % F II. M d D
(b) P is not placed immediately next to A. III. N % F IV. D d N
(c) There are two letters between A and E in the word thus (a) None is true (b) Only I is true
formed. (c) Only II is true (d) Only III is true
(d) P is placed second to the right of E. (e) Only IV is true
(e) None is correct 76. Statement: F # Z, Z @ H, H % N, N d B
71. Which of the following words will be formed based on the Conclusions
given conditions? I. F @ H II. N % Z
(a) SPAEL (b) PEALS III. B % H IV. B % Z
(c) LEAPS (d) SEPAL (a) I and III are true (b) II, III and IV are true
(e) LAPSE (c) I and II are true (d) I, II and III are true
72. The fare-paying capacity of people who travel on routes (e) None of the above
connecting to small towns is very low. Most successful 77. Statement : M % K, K ¬ W, W d V, V @ N
airlines which operate in such regions have a large number Conclusions
of seats.
I. N ¬ K II. M % W
Which of the following can be inferred from the above
information ? III. K d V IV. V % M
(a) Regional airlines are quite profitable. (a) None is true (b) Only I is true
(b) People from cities are increasingly travelling to small (c) Only II is true (d) Only III is true
towns. (e) Only IV is true
(c) Regional airlines have to charge low fares in order to DIRECTIONS (Qs. 78-82) : Each of the questions below consists
be profitable.
of a questions and two statements numbered I and II given below
(d) The number of people travelling from small towns to it. You have to decide whether the data provided in the statements
cities is massive.
are sufficient to answer the question.
(e) None of these
Read both the statements and — Give answer
DIRECTIONS (Qs. 73-77): In the following questions, the
(a) if the data in statement I alone are sufficient to answer
symbols d, ¬, %, # and @ are used with the following meaning
as illustrated below. the question, while the data in statement II alone are
not sufficient to answer the question.
‘P % Q’ means ‘P is neither greater than nor equal to Q’. (b) if the data in statement II alone are sufficient to answer
‘P d Q’ means ‘P is neither smaller than nor equal to Q’. the question, while the data in statement I alone are
‘P @ Q’ means ‘P is not greater than Q’. not sufficient to answer the question.
‘P ¬ Q’ means ‘P is not smaller than Q’. (c) if the data either in statement I alone or in statement II
‘P # Q’ means ‘P is neither greater than nor smaller than Q’. alone are sufficient to answer the question.
Now, in each of the follwoing questions assuming the given (d) if the data in both the statements I and II together are
statements to be true, find which of the four conclusions I, not sufficient to answer the question.
II, III and IV given below them is/are definitely true and give (e) if the data in both the statements I and II together are
your answer. necessary to answer the question.
73. Statement: R ¬ T, T d M, M % K, K @ V 78. Kiran is older than Manoj and Dilip is older than Neelam.
Conclusions Who among them is the youngest?
I. V d M II. V d T I. Kiran is older than Neelam.
III. M % R IV. K d R II. Manoj is younger than Dilip.
(a) I and II are true (b) I and III are true 79. What is the relation between M and F?
(c) II and IV are true (d) I, III and IV are true I. M has two sons, one of whom is B.
(e) None of these II. The mother of F has two sons D and C.
y
o
u
rs
m
384 IBPS Specialist (I.T.) Officer Exam 2014

a
h
b
80. H is in which direction with respect to V? (b) if not hell to cry may sen

o
I. S is to the south of K, who is to the west of V.

o
(c) sen to if may not hell cry

b
II. M is to the north of H, who is to the east of V.

.w
(d) not hell cry if may sen to
81. What is the shortest distance between Devipur and

o
(e) None of these

rd
Durgapur?
DIRECTIONS (Qs. 88-89) : Study following statements and

p
I. Durgapur is 20 km away from Rampur.

re
answer the questions:
II. Devipur is 15 km away from Rampur.

s
s
Seven students Priya, Ankit, Raman, Sunil, Tony, Deepak and

.c
82. “You must submit your application within 10 days from the
Vicky take a series of tests. No two students get similar marks.

o
date of release of this advertisement.” What is exact date

m
Vicky always scores more than Priya. Priya always scores more
before which the application must be submitted?
than Ankit. Each time either Raman scores the highest and Tony
I. The advertisement was released on 18th February.
gets the least, or alternatively Sunil scores the highest and Deepak
II. It was a leap year. or Ankit scores the least.
DIRECTIONS (Qs. 83-87) : Study the following information 88. If Sunil is ranked sixth and Ankit is ranked fifth, which of the
carefully to answer the questions given below. following can be true?
In a toy exhibition, a machine processes a given input by the (a) Vicky is ranked first or fourth
following rule. Participants are shown one by one till it reaches its (b) Raman is ranked second or third
last step. Following is an illustration of the working of this machine. (c) Tony is ranked fourth or fifth
Input : sui me ato fe zen u no (d) Deepak is ranked third or forth
Step I : fe sui me no ato zen u (e) None of these
Step II : no fe sui u me ato zen 89. If Raman gets the highest, Vicky should be ranked not lower
Step III : u no fe zen sui me ato than:
Step IV : zen u no ato fe sui me (a) Second (b) Third
Step V : ato zen u me no fe sui and so on. (c) Forth (d) Fifth
Now attempt the questions given below. (e) None of these
83. Which of the following steps would read as ‘not you only say 90. Farmers found using chemical fertilizers in the organic-
wise yet are’ for the input ‘say not you are only wise yet’? farming area of their farms would be heavily fined.
Which of the following statements is an assumption implicit
(a) III (b) V
in the given statement ? (An assumption is something
(c) VI (d) VII supposed or taken for granted.)
(e) None of these (a) Chemical fertilisers harm the crop.
84. If the Step V of an input is ‘so cd rom lay is nor it’, which (b) A farm's area for organic and chemical farming is
of the following would be its Step II? different.
(a) is nor it rom lay so cd (b) nor it lay is so cd rom (c) Farmers who do not use chemical fertilizers in the
chemical farming area would be penalized as well.
(c) lay so cd it rom is nor (d) Data inadequate
(d) All farmers undertake both these kinds of farming
(e) None of these (chemical as well as organic) in their farms.
85. If the Step III of an input is ‘lo men chi from yet as know’, (e) Organic fertilizers are banned in the area for chemical
which of the following would be its input? farming.
(a) Data inadequate 91. Small brands are now looking beyond local grocery stores
(b) from lo men know chi yet as and are tying up with Supermarkets such as Big Bazaar to
(c) men chi yet lo as know from pull their business out of troubled waters.
Which of the following can be inferred from the given
(d) chi as know men know from lo
information ? (An inference is something that is not directly
(e) None of these
stated but can be inferred from the given information)
86. Which of the following correctly describes the ‘machine (a) Merchandise of smaller brands would not be available
logic’ in generating various steps based on the given input? at local grocery stores in the near future.
(a) Each step is generated on random basis. (b) Smaller brands cannot compete with bigger ones in a
(b) Words/letters are finally arranged in dictionary order. supermarket set-up.
(c) The seventh letter interchanges with the fourth every time. (c) There is a perception among small brands that sale in a
supermarket is higher than that of small grocery stores.
(d) Data inadequate
(d) Supermarkets generate more revenue by selling
(e) None of these
products of bigger brands as compared to the smaller
87. What will be the step IV for the following input? ones.
Input : may sen to cry if not hell (e) Smaller brands have always had more tie-ups with
(a) cry may sen to if not hell supermarkets as compared to small grocery stores.
y
o
u
rs
m
IBPS Specialist (I.T.) Officer Exam 2014 385

a
h
b
92. There has been a considerable drop in sales of four wheelers immediately preceded by a letter at the odd place in the

o
o
during the past six months when compared to the number English alphabet?

b
of four wheelers sold during this period last year. W2 N1 V9 G2 P4 X6 K7 R1 T 8 L3 H5 Q8 U2 J

.w
Which of the following can the probable cause of the above (a) 3 (b) 5

o
rd
phenomenon? (c) 2 (d) 4

p
(e) None of these

re
(A) The govt. has imposed higher excise duty on four
96. The local board of education found that, because the current

s
wheelers at the beginning of this year.

s
physics curriculum has little direct relevance to today’s

.c
(B) The petrol prices have risen considerably during the world, physics classes attracted few high school students.

o
m
past eight months. So to attract students to physics classes, the board
(C) The rate of interest on home and car loans have been proposed a curriculum that emphasizes principles of physics
rising for the past seven months. involved in producing and analyzing visual images.
Which of the following, if true, provides the strongest
(a) All (A), (B) and (C) (b) (A) and (C) Only
reason to expect that the proposed curriculum will be
(c) (B) and (C) Only (d) (B) Only successful in attracting students?
(e) (A) Only (a) Several of the fundamental principles of physics are
93. Statement: The apex body controlling universities in the involved in producing and analyzing visual images.
country has decided to revise the syllabus of all the technical (b) Knowledge of physics is becoming increasingly
courses to make them focused towards the present needs important in understanding the technology used in
today’s world.
of the industry, thereby making the technical graduates more
(c) Evidence that a large producer of photographic
employable than they are at present.
equipment has donated to the high school could be
Which of the following assumptions is/are implicit in the used in the proposed curriculum.
above statement? (d) In today’s world the production and analysis of visual
(A) Technical colleges affiliated to different universities images is of major importance in communications,
may not welcome the apex body’s decision and may business, and recreation.
continue with the same syllabus as at present. (e) None of these
(B) The industry may welcome the decision of the apex DIRECTIONS (Q. 97) : Study the following arrangement
body and scale up their hiring from these colleges. carefully and answer the question given below:
(C) The Govt. may not allow the apex body to implement
F 4 © J 2 E % M P 5 W 9 @ I Q R 6 U H 3 Z 7« A T B 8 V # G $ Y D
its decision in all the colleges as it may lead to chaos. 97. What should come in place of the question mark (?) in the
(a) None is implicit (b) Only (A) is implicit following series based on the above arrangement?
(c) Only (B) is implicit (d) Only (C) is implicit JEM 59I RU3 ?
(e) Only (A) and (B) are implicit (a) 7 A B (b) 7 A T
94. Statement: Govt. has urged all the citizens to use electronic (c) « 7 8 (d) A B V
media for carrying out their daily activities, whenever (e) None of these
possible, instead of using paper as the manufacture of paper 98. Vikas walked 10 metres towards North, took a left turn and
requires the cutting down of a large number of trees causing walked 15 metres, and again took a left turn and walked 10
severe damage to the ecosystem. metres and stopped walking. Towards which direction was
he facing when he stopped walking?
Which of the following assumptions is/are implicit in the
(a) South (b) South-West
above statement? (c) South-East (d) Cannot be determined
(A) Most people may be capable of using electronic media (e) None of these
to carry out various routines. 99. Cause: Govt has recently decided to hike the procurement
(B) Most people may have access to electronic media for price of paddy for the rabi crops.
carrying out their daily routine activities. Which of the following will be a possible effect of the above
(C) People at large may reject the govt’s appeal and cause?
continue using paper as before. (a) The farmers may be encouraged to cultivate paddy for
(a) Only (A) is implicit the rabi season.
(b) Only (B) is implicit (b) The farmers may switch over to other cash crops in
their paddy fields.
(c) Only (A) and (B) are implicit
(c) There was a drop in production of paddy during Kharif
(d) Only (C) is implicit
season
(e) None of these (d) Govt may not increase the procurement price of paddy
95. In the alpha-numerical sequence/series given below, how during the next Kharif season.
many numbers are there which are (i) immediately followed (e) Govt. will buy paddy from the open market during next
by a letter at the even place in English alphabet and (ii) not few months.
y
o
u
rs
m
386 IBPS Specialist (I.T.) Officer Exam 2014

a
h
b
100. If every third letter from the following English alphabet is 110. Because of his (a)/ lack of interest in the (b)/ affairs of the

o
dropped, which letter will be seventh to the right of eleventh society, the members (c)/ decided to depose the president

o
b
letter from your right? (d)/ No error(e).

.w
ABC DE FG HI JK LM NO PQRS T UV W XYZ DIRECTIONS (Qs. 111-125) : Read the following passage

o
rd
(a) V (b) U carefully and answer the questions given below it. Certain words

p
(c) K (d) I / phrases have been printed in bold to help you locate them

re
(e) None of these while answering some of the questions.

s
s
.c
ENGLISH LANGUAGE For more than three years, Anna Feng didn’t tell her husband

o
that she had sunk nearly half of their savings into the Shanghai

m
stock market. While he thought all their money was safely sitting
DIRECTIONS (Qs. 101-105) : Rearrange the following five
in a bank, the value of the stocks plunged by almost 75%.
sentences (A), (B), (C), (D) and (E) in the proper sequence to form
But over the past couple of months, the Shanghai market has
a meaningful paragraph; then answer the questions given below.
shown signs of life, and Feng, a 56-year-old retiree, has recouped
A. Unfortunately, because of modernisation, much of nature is half her losses. She’s quietly hopeful that may be she’ll make it all
now yielding to towns, roads and industrial areas. back. “Everyone seems to be so optimistic about the markets
B. In a few places, some natural reserves are now being carved now,” she says.
out to avert the danger of destroying nature completely. Around the world, stocks have been on a tear. In Asia, for
C. We should preserve nature to preserve life and beauty. example, the Tokyo TOPIX stock index hit a 14-year high last
D. Man will perish without nature, so modern man should week as a bull run in once-dormant Japan gathered momentum;
continue this struggle to save plants from extinction. Mumbai’s main equity index hit an all-time high in trading early
E A beautiful landscape, full of green vegetation, will not just Friday amid India’s continuing economic boom; and Hong Kong
attract our attention but will fill us with infinite satisfaction. shares reached a five-year high while indices in Singapore, Jakarta
101. Which of the following should be the FIRST sentence after and Sydney set new records. And though stock in Asia, in
rearrangement? particular, are on fire, they are not alone. From Germany to Venezuela
(a) A (b) B to South Africa, equity markets in both mature and emerging markets
(c) C (d) D have moved up sharply this year— and show little sign of slowing.
(e) E The underpinning for stocks' strong performance, global
102. Which of the following should be the SECOND sentence bulls say, is straight-forward. Economic growth continues to be
after rearrangement? strong in places where it has been buoyant for several years (the
(a) A (b) B U.S., China and India) and is finally picking up in places where it
(c) C (d) D had been notably absent—Japan and parts of “old” Europe.
(e) E Moreover, earning and corporate balance sheets around the world
103. Which of the following should be the THIRD sentence after are as healthy as they have been in years. In Japan, corporate
rearrangement? profits have climbed for four straight years and consumer spending
(a) A (b) B is rising briskly on the back of declining unemployment.
(c) C (d) D Economists say that Japan is now in a golden cycle. So, for now,
(e) E is much of the world. “It comes down to very simple
104. Which of the following should be the FOURTH sentence macroeconomics.” says Subir Gokam, an economist at CRISIL,
after rearrangement? India's largest credit-rating firm. “The global economy is growing
(a) A (b) B without much inflationary pressure.”
(c) C (d) D Is anything wrong with this picture? One very big thing,
(e) E warn the skeptics. Interest rates are rising nearly everywhere,
105. Which of the following should be the FIFTH sentence after and if there is one simple adage that many investment advisers
rearrangement? live by, It's this: “When rates are high, stocks will die.” Indeed,
(a) A (b) B one of the most impressive—or scariest— aspects of the current
(c) C (d) D global bull run is that it has come in the teeth of central- bank
(e) E tightening, most importantly by the U.S. Federal Reserve, which
DIRECTIONS (Qs. 106-110) : Read each sentence carefully and could slow growth in the world’s key economic locomotive. The
detect the error which may be in an one of the parts a, b , c or d Fed has increased key short-term interest rate—the so-called Fed
. In case of ‘No error’, (e) will be the answer. funds rate—15 times dating back to June 2004, and is widely
expected to raise it one or twice more over the next few months. A
106. To hit a man (a)/ when he is down (b) / is a contemptuous brief recession and the Sept. 11 terrorist attacks in 2001 spurred a
(c)/ thing to do (d)/ No error(e). prolonged period of very low interst rates. That boosted U.S.
107. The climate on this (a)/ altitude is cool and dry (b)/ but consumption—in particular the ratesensitive housing market—
down in the valley (c)/ it is very sultry(d)/ No error(e). and kept the global economy humming. But long-term rates are
108. Since her mother (a)/ died when she (b)/ was young (c)/ she now beginning to tick upward: last week the U.S. 30- year treasury
did lack maternal are (d)/ No error(e). bond reached 5.04% its highest level since late 2004, and the
109. In 1665 a (a)/ great pestilence caused (b)/ the death of 63000 housing market is cooling off—potentially triggering an economic
persons (c)/ in the city of London (d)/ No error(e). slowdown as homeowners cut their spending.
y
o
u
rs
m
IBPS Specialist (I.T.) Officer Exam 2014 387

a
h
b
111. According to the passage, what makes people hopeful about (d) Markets only in Asian countries have shown upward

o
the markets ? trend

o
b
(a) The descent in the value of stocks (e) None of these

.w
(b) The trend of substantial increase in value of stocks 118. Which of the following is/are the reason(s) for the statement

o
(c) Safety provided by banks to their deposits that ‘Japan is now in golden cycle’ ?

rd
(d) Optimism of the stock market players (A) It is an economic growth without much inflationary

p
re
(e) The interest rates are going up pressure.

s
112. Which of the following is TRUE about the comparison (B) Japan witnesssed a substantial increase in corporate

s
.c
between market indices of Mumbai and Hong Kong on the profits for the last four years.

o
m
one hand and Singapore, Jakarta and Sydney on the other? (C) There are more employment avenues open and
(a) The indices in the former case are increasing while consumer’s spending has increased significantly.
those in the latter are decreasing (a) All the three-(A), (B) and (C)
(b) There is no rerqarkable trend visible between the two (b) (A) and (B) only
sets of indices (c) Either (B) or (C) only
(c) Indices in both the groups of exercises are stable over (d) (A) and (C) only
a period of last five years (e) Either (A) or (C) only
(d) The markets in both the groupd of countries have 119. According to the content of the passage, the spurt in stock
shown upward trend markets appears to be—
(e) None of these (a) a healthy sign of growing world peace
113. What is the impact of increasing long term interest rates? (b) an indication of eradication of global poverty
(A) Demand in housing market is gradually diminishing. (c) a proof of a negligible number of people below poverty
(B) Retardation in economic growth. line
(C) Restrictions imposed by central bank. (d) a transition from underd development to enrichement
(a) (A) only (B)only (e) None of these
(b) (B) and (C) only
(c) (C) only DIRECTIONS (Qs. 120-122) : Which of the following is most
(d) (A) & (C) only OPPOSITE in meaning of the word given in bold as used in the
(e) None of these passage ?
114. In what way did the terrorist attack in the US influence the 120. Briskly
markets ?
(a) vigorously (b) efficiently
(a) It led to a brief recession
(b) It increased long term need for housing (c) hurriedly (d) insignificantly
(c) It helped increase the interest rates on housing (e) slowly
(d) It prolonged the low interest rate regime 121. Boosted
(e) None of these (a) aggravated (b) elevated
115. How are the interest rates associated with the stocks? (c) deflated (d) damaged
(a) In direct proportion
(e) stopped
(b) In inverse proportion
(c) No relation 122. Plunged
(d) Dependent upon the investors (a) fell (b) increased
(e) None of these (c) dropped (d) more
116. Which of the following statements is/are FALSE in the (e) appeared
context of the passage ?
DIRECTIONS (Qs.123-125) : Which of the following is most
(A) Economic growth in US, China, India, Japan and old
European countries started to show a downward trend. nearly the SAME in meaning as the word given in bold as used
(B) Higher interest rates help boost stock value in the passage?
(C) Skeptics firmly believe that economic growth is a boon 123. Buoyant
from all the angles. (a) drawing (b) haphazard
(a) All the three (b) (A) and (B) only (c) upbeat (d) extravagant
(c) (B) and (C) only (d) (A) and (C) only (e) sailing
(e) None of these 124. Spurred
117. How do the stock values in Asian countries compare with (a) shortened (b) widened
that in the other countries of the global ? (c) thronged (d) stimulated
(a) It cannot be inferred on the basic of the context of the (e) escalated
passage 125. Skeptic
(b) There is general decline in the market indices all over (a) disbeliever (b) orthodox
the globe
(c) theist (d) philosoper
(c) Markets show a general rise in the indices all over the
(e) analyst
globe
y
o
u
rs
m
388 IBPS Specialist (I.T.) Officer Exam 2014

a
h
(a) experts have a lot of expectations from this cleaner

b
DIRECTIONS (Qs. 126-135) : In the following passage some of

o
method of producing energy

o
the words have been left out, each of which is indicated by a

b
number. Find the suitable word from the options given against (b) the government is determined to extract maximum out

.w
each number and fill up the blanks with appropriate words to of this technology in the near future

o
rd
make the paragraph meaningful. (c) international lobby has been pressurising the
developing nations to shift their energy production

p
re
Man has been tampering (126) ecosphere for a very (127) from coal to nuclear power.

s
time and is forced to (128) that the environmental resources are

s
(d) the problem of locating adequate numbers of uranium

.c
(129). Environmental problems are (130) social problems. They
reserves to run the reactors is yet to be sorted out

o
begin with people as (131) and end with people as victims.

m
Unplanned uses of resource (132) in the depletion of fossil fuels (e) None of these
and (133) of air and water, Deforestation has led to (134) imbalance 138. _______ experts proposed the idea of a common school
and draining away of national wealth (135) heavy expenditure on system.
oil and power generation. (a) Overlooking the fundamental right of quality education
126. (a) on (b) in of every child in India
(c) with (d) for
(b) Since the curricular requirements of a rural child is
(e) from
different from an urban child
127. (a) short (b) long
(c) small (d) tall (c) Based on the fact that difference in the quality of
(e) high schools acts as a ground for discrimination
128. (a) see (b) look (d) Since a large percentage of Indian children are getting
(c) do (d) recognise free education
(e) realise (e) None of these
129. (a) plenty (b) scarce
139. ______ the soil today is nowhere as rich in native minerals
(c) minute (d) enough
as it used to be some centuries ago.
(e) minimum
130. (a) really (b) coldly (a) As there is a growing consent among farmers regarding
(c) badly (d) happily limiting the use of chemical fertilizers
(e) seriously (b) As the chemical inputs in agriculture improved the yield
131. (a) effect (b) result many folds
(c) cause (d) wisdom (c) Owing to the uninhibited use of chemical inputs in
(e) affect agriculture
132. (a) cause (b) rest
(d) Awareness among farmers regarding the side-effects
(c) consequence (d) result
of chemical farming grew when
(e) get
133. (a) revolution (b) pollution (e) None of these
(c) resolution (d) evolution 140. As allegations of crores of rupees changing hands to permit
(e) solution illegal mining began to fly thick and fast, _____________.
134. (a) ecological (b) biological (a) government ordered an enquiry which exposed a nexus
(c) logical (d) chronological between mine operators and bureaucrats
(e) geographical
(b) it caused great damage to the surrounding ecosystem
135. (a) by (b) in
and the environment in general
(c) out (d) through
(e) from (c) the officials have been irresponsible in failing to bring
it to the notice of the court in time
DIRECTIONS (Qs. 136-140) : Which of the phrases (a), (b), (c)
(d) the powerful mining lobby had bribed the officials to |
and (d) given below each statement should be placed in the
blank space provided so as to make a meaningful and obtain permit for mining on ecologically sensitive land
grammatically correct sentence? If none of the sentences is (e) None of these
appropriate, mark (e) as the answer. DIRECTIONS (Qs. 141-145) : Read the following passage
136. Although information technology has entered the homes carefully and answer the questions given below it. Certain words
offices and hearts of many citizens of India, __________ . are given in bold to help you locate them while answering some
(a) India provides the highest number of IT experts to the of the questions.
world every year
The wakeup call that China represents to India is not limited
(b) many people in rural areas still remain ignorant of its
to its showpiece urban centres or that New Delhi hopes India will
immense benefits
experience the benefits that the Olympic games have brought to
(c) government has done its best by funding research in
Beijing. More pertinent is the comparison of the agricultural sectors
this field appropriately
of the two countries. Why and how has China managed to outstrip
(d) the face of communication in the years to come would
India in agriculture when 25 years ago the two countries were on
change completely from the by gone years
par on most parameters? Both have traditionally been agrarian
(e) None of these
economies and over half their populations continue to depend on
137. While the environment-friendly nuclear energy could make the land for their livelihood. With large populations and histories
a large addition to the energy resources, __________ . of famine, India and China share concern on issues such as food
y
o
u
rs
m
IBPS Specialist (I.T.) Officer Exam 2014 389

a
h
security. However, while India’s agricultural sector is projected to 142. Which of the following is/are area/s in which China has not

b
o
grow by about 2.5 per cent this year – a slide from the previous outdone India?

o
b
year’s growth–, China’s has been steadily growing at between 4 (A) Development of urban infrastructure

.w
per cent and 5 per cent over the last fifteen years. The widest (B) Activities allied to agriculture like animal husbandry

o
divergence between India and China is in the profitable (C) Successful bids for international sporting events

rd
horticultural sector with the production of fruits and vegetables (a) Either (A) or (B) (b) Only (B)

p
in China leaping from 60 million tonnes 1980 compared to India’s

re
(c) Only (A) (d) Both (A) & (C)
55 million tonnes at the same time, to 450 million tonnes in 2003

s
(e) None of these

s
ahead of India’s corresponding 135 million tonnes. China’s added

.c
143. How are Chinese agricultural research facilities governed?
advantage lies in the more diversified composition of its

o
(a) Salaries of staff are linked to performance and this

m
agricultural sector with animal husbandry and fisheries which
account for close to 45 per cent of growth compared to 30 per cent hampers productive research.
for India. (b) Their funding comes from the government alone to
prevent private companies from manipulating the
According to the latest report by the Economic Advisory direction of their research.
Council, the traditional excuses for India’s substandard
(c) A fixed proportion of government grants is allotted to
performance in the farm sector are inadequate since India is placed
favourably when compared to China in terms of quantity of arable be utilised for administrative incidentals which cannot
land, average farm size, farm mechanisation etc. The reasons for be exceeded.
China having outperformed India are threefold: technological (d) Research staff even those on contract are entitled to
improvements accruing from research and development (China increments for good performance.
has over 1,000 R&D centres devoted to agriculture), investment in (e) None of these
rural infrastructure and an increasingly liberalised agricultural 144. What argument has been posed against implementation of
policy moving away from self-sufficiency to leveraging the subsidies?
competitive advantage with a focus on “efficiency as much as (A) Subsidies sacrifice equity for efficiency.
equity”. Investment in rural infrastructure, roads, storage facilities, (B) Subsidies hamper efficient resource utilisation.
marketing facilities are also crucial but government support in (C) Subsidies reduce private sector investment and
India has mainly been through subsidies, not investment. There involvement in agriculture.
has been much debate about subsidies and their utility; the (a) Both (A) & (B) (b) Only (B)
opposing view being that subsidies are against the market reforms (c) Both (B) & (C) (d) Only (A)
and distort the market as well as reduce resource efficiency. In (e) None of these
contrast to the 2,046 applications for the registration of new plant 145. Which of the following cannot be said about Indian
varieties in China over the past few years, data reveals that despite agricultural universities?
India having the largest number of agricultural scientists in the (A) Attendance is poor because of the dwindling funds to
world India’s current research track record is abysmal, equivalent
carry out research.
to what China achieved in the 1980s. Far from developing new
strains, the number of field crop varieties fell by 50 per cent (B) Enrolments of students and qualified staff have fallen
between 1997 and 2001 despite the fact that there was sharp and because of the lack of funds for salaries.
sustained increase in funding. One reason is that majority of the (C) Allotment for research funding by the government is
budget is eaten up by staff salaries with only 3 per cent being non-existent.
allotted for research. In contrast, most agricultural research centres (a) Only (B) (b) Both (A) & (B)
in China must use Central government funding purely for research. (c) Both (B) & (C) (d) All (A), (B) & (C)
Funds relating to salaries and other administrative incidentals (e) None of these
must be generated by the centres themselves. The centres and
scientists are thus encouraged to engage in joint ventures with DIRECTIONS (Qs. 146-150) : Which sentence has almost the
private sector companies to form commercial signoffs from their same meaning with that of the question?
research. In fact, research staff are now being hired on a contract
basis with pay based on performance and salaries raised 146. For more devastating for the author than the loss of his
proportionately for those who perform well. India needs to learn house to fire was the destruction of the sole copy of his
from China’s example and adopt a pragmatic approach if it has to latest novel.
meet its targets of the Eleventh Five Year Plan. (a) In his latest book, the author tells of the pain he
141. What has been the major area of difference in the suffered when he lost his home in a devastating fire.
development of the agricultural sectors of India and China? (b) Even the destruction of his home in the fire was not as
hard for the author to bear as the burning of the
(a) Quantity of arable land in China is far greater than in
manuscript of his new novel.
India.
(c) The only thing the author was able to rescue from the
(b) Food security is not a concern for China as the country flames which destroyed his home was the sole copy of
is basically self-sufficient. his latest book.
(c) China has experienced substantial growth in (d) Even more difficult for the author than writing a single
production in allied agricultural activities like book was seeing it destroyed when his house burnt
horticulture. down.
(d) India’s agricultural sector is too diversified so it is (e) The author was devastated to find his house burnt
difficult to channel funds for development. down with everything in it, among which was the only
(e) None of these manuscript of his latest novel.
y
o
u
rs
m
390 IBPS Specialist (I.T.) Officer Exam 2014

a
h
153. A static partitioned memory management system has a total

b
147. Without his help, the job would have been impossible.

o
(a) Even though he was very helpful, we were still unable of six partitions. If one is allocated to the operating system,

o
b
to complete the project. this will allow a total of _______.

.w
(b) Since he wasn’t able to give us a hand, we didn’t (a) five user jobs (b) six user jobs

o
believe we would be able to finish the job. (c) thirty-two user jobs (d) thirty-six user jobs

rd
(c) There’s no way the task could have been done if he (e) sixty four user jobs

p
154. Which amongst the following is not an advantage of

re
had not provided assistance.
distributed system ?

s
(d) The assignment proved quite easy, even though he

s
.c
refused to give us aid. (a) Reliability (b) Incremental growth

o
(e) Though it wasn’t easy, we managed to accomplish our (c) Resource sharing (d) Low cost

m
task without his assistance. (e) None of these
148. Having arrived at the church, Martin sat down at the front 155. In E-R diagram derived attribute are represented by
(a) Ellipse (b) Dashed Ellipse
listening to the music.
(c) Rectangle (d) Triangle
(a) After martin had arrived at the church, he heard a lovely
(e) None of these
music at the front.
156. It is better to use files than a DBMS when there are
(b) After he had arrived at the church, Martin sat down at (a) stringent real-time requirements
the front and listened to the music. (b) multiple users wish to access the data
(c) In order to sit down at the front and listen to the music (c) complex relationships among data
he arrived at the church early. (d) all of the above
(d) At the church Martin wanted to listen to the music so (e) none of these
he arrived at the church early. 157. Match the following
(e) Martin fount a seat at the front because he came to the List I List II
church early. A. Data link layer 1. The lowest layer whose function
149. Despite the difference in the political ideas, decision were is to activate, deactivate and
taken by consensus. maintain the circuit between DTE
(a) Although there were different political ideas, most of and DCE
the members accepted the idea. B. Physical layer 2. Performs routing and
(b) In spite of the different political ideas, they are used communication
their votes. C. Presentation layer 3. Detection and recovery from
(c) They forced the politicians to take decision together. errors in the transmittd data
(d) Although there were different political ideas, all of the D. Network layer 4. Concerned with for the syntax of
members accepted the ideas. the data
(e) Since there were different ideas, they couldn’t find the Codes
solution. A B C D
150. Problems never seem quite so bad if we can see the funny (a) 3 1 4 2
sides of them. (b) 2 1 4 3
(a) You can solve your problems even if they seem so (c) 4 1 2 3
bad. (d) 2 1 3 4
(b) Although the problems which you face are so bed you (e) 1 2 3 4
can see the funny sides of them. 158. What is the main reason the OSI model was created?
(a) To create a layered model larger than the DoD model.
(c) I we can find something funny with our problems they
(b) So application developers can change only one layer's
never seen so bad.
protocols at a time.
(d) When we confront with problems we must laugh at (c) So different networks could communicate.
them. (d) So Cisco could use the model.
(e) It is very important to look into the problems when (e) None of these
you have them. 159. How many collision domains are created when you segment
a network with a 12-port switch?
PROFESSIONAL KNOWLEDGE (I.T.) (a) 1 (b) 2
(c) 5 (d) 12
151. Which of the following is a service not supported by the
(e) None of these
operating system?
160. If your routing table has a static, a RIP, and an IGRP route to
(a) Protection (b) Accounting
the same network, which route will be used to route packets
(c) Compilation (d) I/O operation by default?
(e) None of these (a) Any available route (b) RIP route
152. FIFO scheduling is (c) Static route (d) IGRP route
(a) pre-emptive scheduling (e) They will all load-balance
(b) Non-pre-emptive scheduling 161. Which protocol does Ping use?
(c) deadline scheduling (a) TCP (b) ARP
(d) fare-share scheduling (c) ICMP (d) BootP
(e) None of these (e) None of these
y
o
u
rs
m
IBPS Specialist (I.T.) Officer Exam 2014 391

a
h
162. You have a system that periodically locks up. You have 172. Multilevel feedback queue scheduling

b
o
ruled out software, and now suspect that it is hardware. (a) allows to select a process and load it to memory for

o
execution

b
What should you do first that could help you narrow it

.w
down to the component at fault? (b) allows to select a process, that are ready to execute

o
(a) rotate the RAM and allows CPU to one of them.

rd
(b) replace the RAM (c) does not allow a process to move between queues

p
(c) replace the level 2 cache SIMM (d) allows processes which are permanently assigned to a

re
(d) disable the CPU cache in CMOS queue on the entry to the system.

s
s
(e) replace the CPU (e) none of these

.c
173. The process is

o
163. What tool is used to test serial and parallel ports?
(a) a program in High Level Language kept on a disk

m
(a) high volt probe
(b) content of main memory
(b) cable scanner (c) a program in execution
(c) loop backs (wrap plugs) (d) a job in secondary memory
(d) sniffer (e) none of these
(e) None of these 174. Software testing techniques are most effective if applied
164. With respect to a network interface card, the term 10/100 immediately after ?
refers to (a) Requirement specification
(a) protocol speed (b) Design
(b) a fiber speed (c) Coding
(c) megabits per seconds (d) Integration
(d) minimum and maximum server speed (e) None of these
(e) None of these 175. The primary job of the operating system is
165. If limited user participation is available, which model is to (a) to manage commands (b) to manage users
be selected? (c) to manage resduces (d) to manage programs
(a) Waterfall model (e) all of the above
(b) Spiral model 176. Which of the following file format supports in windows 7 ?
(c) Iterative enchancement model (a) BSD (b) NTFS
(d) any of the above (c) EXT (d) All of the above
(e) None of these (e) None of these
166. Software Quality is 177. Which of the following is correct?
(a) Conformance to requirements (a) An SQL query automatically eliminates duplicates
(b) An SQL query will not work if there are no indexes on
(b) Fitness for the purpose
the relations
(c) Level of satisfaction
(c) SQL permits attribute names to be repeated in the same
(d) All of the above relation
(e) None of these (d) None of these
167. The purpose of regression testing is to (e) All of the above
(a) increase confidence in the correctness of the modified 178. Index sequential file is made of all of these expect
program (a) primary data storage area
(b) locate errors in the modified program (b) overflow area
(c) preserve the quality and reliability of software (c) hierarchy indices
(d) all of the above (d) address of prime data track.
(e) None of these (e) None of these
168. Errors may be found by outside during 179. The natural join is equal to :
(a) alpha testing (b) white box testing (a) Cartesian product
(c) beta testing (d) Both (a) and (b) (b) Combination of union and cartesian product
(e) None of these (c) Combination of selection and cartesian product
169. The time interval between the time of submission of a process (d) Combination of protection and cartesian product
to the time of completion of a process is known as which of (e) None of these
the following? 180. The disadvantage of “compile and go” loading schemes
(a) waiting time (b) response time (a) a portion of memory is wasted because the core
(c) turn around time (d) none of these occupied by the assembler is unavailable to the object
(e) None of these program
(b) it is necessary to retranslate the user’s program deck
170. E-R modeling technique is a
every time it is run.
(a) top-down approach
(c) it is difficult to handle multiple segments, especially if
(b) bottom-up approach the source programs are in different language and to
(c) left-right approach produce orderly modular programs
(d) both top-down and bottom-up (d) all of the above
(e) none of these (e) None of these
171. Compile time errors do not include 181. Which of the following is used for grouping of characters
(a) lexical error (b) syntactic error into tokens?
(c) semantic error (d) all of the above (a) parser (b) code optimization
(e) None of these (c) code generator (d) scanner
(e) None of these
y
o
u
rs
m
392 IBPS Specialist (I.T.) Officer Exam 2014

a
h
182. Shift reduce parsers are 191. Which of the following is true when describing a unicast

b
o
(a) top-down parsers address?

o
b
(b) bottom-up parsers (a) Packets addressed to a unicast address are delivered

.w
(c) top-down or bottom-up parsers to a single interface.

o
(d) top-up parsers (b) These are your typical publicly routable addresses,

rd
(e) None of these just like a regular publicly routable address in IPv4.

p
183. A top-down parser generates (c) These are like private addresses in IPv4 in that they

re
(a) left most derivation are not meant to be routed.

s
s
(b) right most derivation (d) These addresses are meant for nonrouting purposes,

.c
(c) right most derivation in reverse but they are almost globally unique so it is unlikely

o
m
(d) left most derivation in reverse they will have an address overlap.
(e) None of these (e) None of these
184. Which of the following class of statement usually produces 192. ICMP is the protocol at the Network layer that is used to
no executable code when compiled? send echo requests and replies.
(a) Declaration statements What protocol does PPP use to identify the Network layer
(b) Assignment statements protocol?
(c) Input and output statements (a) NCP (b) ISDN
(d) Structural statements (c) HDLC (d) LCP
(e) None of these (e) None of these
185. The language which are having many types, but the type of 193. Arrange the following in increasing order of their size
every name and expression must be calculable at compile (a) Database < File < Record < Field < Byte < Bit
time is called (b) Bit < Byte < Field < Record < File < Database
(a) strongly typed (b) weakly typed (c) Bit > Byte < Field < Record < Field > File > Database
(c) dynamic typed (d) boldly typed (d) Bit > Byte > Record > Field > File > Database
(e) None of these (e) None of these
186. The linker 194. The keyword used to transfer control from a function back
(a) is the same as loader to the calling function is
(b) is required to create a load module (a) switch (b) goto
(c) user source code as input (c) go back (d) return
(d) is always used before programs are executed (e) None of these
(e) None of these 195. Which of the following special symbol allowed in a variable
187. The task of the lexical analysis phase is name?
(a) to parse the source program into the basic elements or (a) * (asterisk) (b) | (pipeline)
tokens of the language (c) - (hyphen) (d) _ (underscore)
(b) to build a literal table and an identifier table (e) None of these
(c) to build a uniform symbol table 196. In C, if you pass an array as an argument to a function, what
(d) all of the above actually gets passed?
(e) None of these (a) Value of elements in array
188. Which of the following ISO level is more closely related to (b) First element of the array
the physical communications facilities? (c) Base address of the array
(a) Application (b) Session (d) Address of the last element of array
(c) Network (d) Data link (e) None of these
(e) None of these 197. Which of the following statements mentioning the name of
189. In context of OSI or TCP/IP computer network models, which the array begins DOES NOT yield the base address?
of the following is FALSE? 1. When array name is used with the sizeof operator.
2. When array name is operand of the & operator.
(a) Besides span of geographical area, the other major
3. When array name is passed to scanf() function.
difference between LAN and WAN is that the later
4. When array name is passed to printf() function.
uses switching element
(a) 1 (b) 1, 2
(b) A repeater is used just to forward bits from one network
(c) 2 (d) 2, 4
to another one
(e) 1, 2, 3, 4
(c) IP layer is connected oriented layer in TCP/IP
198. Structured programming codes includes ?
(d) A gateway is used to connect incompatible networks
(a) sequencing (b) alteration
(e) None of these (c) iteration (d) multiple exit form loops
190. Consider the OSI standard for LANs, (e) Only A, B and C
(a) the OSI network layer is subdivided into a MAC layer 199. Which of the following is another name for weak entity ?
and a LLC layer (a) Child (b) Owner
(b ) the OSI data link layer is subdivided into an Ethernet (c) Dominant (d) All of the above
layer and a Token ring layer (e) None of these
(c) the OSI data link layer is subdivided into a MAC layer 200. What is nedified when changing the system start-up boot
a LLC layer sequence ?
(d) the OSI physical layer is subdivided in to an Ehternet (a) BIOS/CMOS (b) COWFIG.Sys
layer and a Token Ring layer (c) Autoexec.bat (d) COMMAND.Com
(e) None of these (e) None of these
y
o
u
rs
m
IBPS Specialist (I.T.) Officer Exam 2014 393

a
h
b
o
o
b
.w
o
1-5:

rd
6.69 2.23
= = = 223 : 260

p
Total number Total Number of Total Number of 7.80 2.60

re
of Mobiles Mobiles S old of Mobiles S old of
12. (a) The requried per cent

s
S old Company A Company B

s
.c
Ju ly 7650 4080 3570 2.23
= ´ 100 » 87.45%

o
A u g us t 9900 4400 5500 2.55

m
Septemb er 11250 6750 4500 13. (e) Year Total number of cars
Octo b er 3600 2100 1500 manufactured (in thousands)
No v emb er 5400 2520 2880 1. 2008 13.22
Decemb er 7200 3150 4050
2. 2009 13.55
3. 2010 14.78
1. (c) Number of mobiles sold of company B in July = 3570 4. 2011 15.54
Number of mobiles sold of company B in December 5. 2012 16.47
= 4050 6. 2013 16.97
Required Ratio = 3570 : 4050 = 119 : 135 2.10 - 2.08
14. (d) The required per cent increase = ´ 100
2. (c) Total mobiles sold by companyA during November = 2520 2.10
Total mobiles sold by this company at discount
0.02
= 35% of 2520 = 882 = ´ 100 = 0.95 » 1%
Total mobiles sold by company A without discount 2.10
= 2520 – 882 = 1638 15. (c) The required total
3. (d) Mobile phones sold of company B during October = 1500 = (1.53 + 2.03 + 1.87 + 1.95 + 2.32 + 2.36) × 1000 = 12060
Total profit earned on the mobile phones 16-20 : Distribution of officers in different categories is as
= `(433 × 1500) = ` 6,49,500 follow:
4. (e) Number of mobile phones sold of company Pub. Pub. Pr i. Pr i. Pub.
A during July = 4080 Ru Ur R u Ur ( R u + Ur )
Number of mobile phones sold by company A during 450 3750 300 1800 3600
December = 3150
Pub. + Pr i ( R u ) Pub. + Pr i ( U r )
Required percentage = 4080 ´ 100 = 129.5 » 130% 600 2250
3150 16. (b) 450 + 300 + 3600 + 600 = 4,950
5. (a) Mobile phones sold of company B during August = 5500 17. (d) Total number of candidates.
Mobile phones sold of company B during September = 450 + 3750 + 3600 + 600 + 2250 = 10650
= 4500
Total number of mobile phones = 5500 + 4500 = 10,000 450 3
18. (b) Reqd ratio = = 3: 2
6. (c) P2013 = 16 × 1.4 ×1.5 × 1.7 = 57.12 lakh 300 2
7. (a) Let the population in the years 1998 was x Required ratio = 300 + 450 = 2 : 3
P2008 = 1.25 ´ 1.3 ´ x = 13.65 lakh 19. (c) Required number of candidates working in Private
Sector Banks in Urban Areas only
13.65 = 1800 + 2250 = 4050
x= = 8.4 lakh
1.25 ´ 1.3 20. (e) Number of candidate having no prior experience of
8. (e) PA = 12 × 1.3 = 15.6 lakh, working in banking sector
PC = 12 × 1.4 = 16.8 lakh = 15000 – (450 + 3750 + 300 + 1800 + 3600 + 600 + 2250)
= 15000 – 12750 = 2250
Diff = 16.8 – 15.6 = 1.2 lakh
9. (c) Let the population of City D in the year 1998 be ‘x’. 2250
Req. % = ´ 100 = 15%
\ Its population in 2008 = x × 1.25 × 1.3 = 1.625x 15000
1.625 x - x 21. (b) Let the distance it travelled at the speed of 160 kmh–1
\ Reqd% = ´ 100 = 62.5%
x be ‘x’ km.
10. (e) PE-2003 = 20 × 1.75 = 35 lakh x (560 - x )
PC -2003 = 24 × 1.4 = 33.6 lakh \ + = 9.5
160 40
33.6 x + 4(560 - x)
\ Reqd% = ´ 100 = 96% = 9.5
35 160
11. (b) The required ratio x + 2240 - 4 x = 1520
2.34 + 2.15 + 2.20 \ 3x = 720
=
2.62 + 2.79 + 2.39 \ x = 240 km
y
o
u
rs
m
394 IBPS Specialist (I.T.) Officer Exam 2014

a
h
b
1

o
22. (a) (A + B)’s one day’s work = th work éæ 12 ö 2 ù

o
5 = 22500 êç1 + ÷ - 1ú

b
êëè 100 ø úû

.w
Let A can do job in x days. Then,

o
1 éæ 3 ö2 ù

rd
A’s one day’s work = th work = 22500 ê ç1 + ÷ - 1ú

p
x ê è 25 ø ú
ë û

re
1 1 x-5

s
and B’s one day’s work = – = th work

s
éæ 28 ö 2 ù

.c
5 x 5x æ 784 ö
= 22500 êç ÷ - 1ú = 22500 ç -1
è 625 ÷ø

o
è
ê 25 ø ú

m
æ 1ö 1 ë û
Now , (2)A 's work + ç ÷ B's work = rd work
è 3ø 3 159
= 22500 ´ = ` 5724
625
2 1æ x-5ö 1 25 1
Þ + ç ÷= Þx= = 6 days
x 3 è 5x ø 3 4 4 éæ 8 ö
2 ù
28. (a) Required Amount = P êç1 + ÷ - 1ú = 1414.4
23. (b) Let the C.P. of horse = ` x êëè 100 ø úû
Then the C.P. of carriage = ` (3000 – x)
20% of x – 10% of (3000 – x) = 2% of 3000 P = Principal
x (3000 – x ) é 729 ù
Þ - = 60 Þ Pê - 1 = 1414.4
5 10
ë 625 úû
Þ 2 x - 3000 + x = 600
625
Þ 3x = 3600 Þ x =` 1200 Þ 1414.4 ´ = 8500
104
24. (d) Cost price of TV when discount is not offered
Total amount to be returned
100
= 11250 ´ = `12500 = 8500 + 1414.40 = ` 9914.40
90
Total cost of TV after transport and installation éæ r ö
t ù
= 12500 + 800 + 150 = 13450 29. ê
(a) CI = P ç 1 + ÷ - 1ú
To earn 15% profit, he must sell at êëè 100 ø úû
115
13450 ´ = `15467.50 éæ 2 ù
100 r ö
594.5 = 5800 êçè 100 ÷ø - 1úú
ê 1 +
100 ë û
25. (c) CP for Karthik = 10500 ´ = 7500
140
2
100
594.5 æ r ö
Þ + 1 = ç1 + ÷
\ CP for Prathik = 7500 ´ = ` 6250 5800 è 100 ø
120
26. (d) Let sum = ` A and interest rate = r% 2
6394.5 æ r ö
A´r´3 Þ = ç1 + ÷
A+ = 5852 5800 è 100 ø
100
é 3r ù
æ r ö
\ A ê1 + ú = 5852 … (i) 1.05 = ç 1 + ÷
ë 100 û è 100 ø
A´r´7
A+ = 7788 r
100 Þ 1.05 – 1 = Þ r = 0.05 = 5%
100
é 7r ù 30. (b) Let the amount invested in scheme A be ` x.
\ A ê1 + ú = 7788 … (ii)
ë 100 û
3600 ´ 100
From equations (i) and (ii), r = 11% \ x ´ 12 ´ 2 = 3600 Þ x = = ` 15000
100 24
Interest ×100
27. (b) Rate of simple interest = Total investment = ` 35,000
Principal × time
\ Amount invested in scheme B. = ` (35000 – 15000)
10800 ´100 = ` 20000
= < 12%
22500 ´ 4
éæ R ö
T ù éæ 10 ö
2 ù
é time ù \ C.I = P êç1 + ÷ - 1ú = 2000 ê ç 1 + ÷ - 1ú
Compound interest = Principal êæç1+
rate ö è 100 ø êëè 100 ø úû
÷ – 1ú ëê úû
êëè 100 ø úû
= 20000 (1.21 – 1) = 20000 × 0.21 = ` 4200
y
o
u
rs
m
IBPS Specialist (I.T.) Officer Exam 2014 395

a
h
b
AB = 400 - 256 =12

o
31. (c) xm

o
b
Þ BC = 2

.w
\ x = 4 + 256 = 260

20 m

o
rd
34. (e) Total area of four walls of a room = 2h (l + b)

p
re
Let the height of the room be x.

s
60 m The breadth = 2x and length = 3x

s
.c
o
Let width of the walkway be x metre Area (1) = 2 x (2 x + 3 x ) = 10 x 2

m
Then, (20 + 2x )(60 + 2x ) = 516 + 1200
x æ 2x ö
Area (2) = 2 ´ ç + 3x ´ 2 ÷ = 7 x 2
Þ 1200 + 120 x + 40 x + 4 x 2 = 516 + 1200 2è 2 ø
Þ 4 x 2 + 160 x - 516 = 0 Þ x 2 + 40 x - 129 = 0 \ Area of 4 walls decreases by 30%.

Þ x 2 + 43 x - 3 x - 129 = 0 Þ x = 3, x = -43 35. (b)


2h
\ Width of walkway = 3 metres
3
32. (a) Let the radii be r 1 and r2
h

r1 r2
pr 2 h
Volume of the original cone ' V ' =
3
Height of the smaller cone = 2h / 3
p r12 + p r22 = 153p and radius =
2r
(from similar D’s)
3
r12 + r22 = 153 .....(i) Volume of smaller cone
r1 + r2 = 15 ....(ii) 2
1 æ 2 r ö 2h 8 2 8V
= pç ÷ = pr h =
Squaring equation (ii) on both sides 3 è 3ø 3 81 27
Þ r12 + r22 + 2r1r2 = 225 Volumeof smaller cone 8V / 27 8
= =
Þ 2r1r2 = 72 Þ r1r2 = 36 Volumeof frustum V - 8V / 27 19
If r1 and r2 are roots of equation 36. (d) The question cannot be answered because R’s share
in investment is not given.
then, r 2 - 15r + 36 = 0
37. (b) A Hypotenuse = 52 + 122 = 25 + 144 = 169 = 13
2
Þ r - 12r - 3r + 36 = 0 Þ (r - 12)(r - 3) = 0 Base : Height : Hypotenuse = 5 : 12 : 13
Þ r = 12, 3 B Base + Height + Hypotenuse = 30 cm

r1 5
\ =4 \ Base = ´ 30 = 5 cm
r2 5 + 12 + 13
12
Height = ´ 30 = 12 cm
D 5 + 12 + 13
33. (a)
1 1
Area = ´ base ´ height = ´ 5 ´12 = 30 cm2
20 2 2
38. (b) A x–y=6
x h
B 0.4y = 0.3x
x 4 x y
A 10 C B = ; C. . = 2 :1
y 3 2 3
Let attitude of DADC be h and third side be x
x 3 2 x 4
1 ´ = Þ =
Then, area of triangle = 80 = ´ 10 ´ h Þ h = 16 y 2 1 y 3
2
B and C give the same expression / information
So, using Pythagorous theorem
and hence are equivalent.
y
o
u
rs
m
396 IBPS Specialist (I.T.) Officer Exam 2014

a
h
b
4 44. (a) 7x2 – 4x1 = 47

o
x= y Þ x – y= 6 x1 + x2 = 2

o
3

b
Solving 11x2 = 55

.w
4 y x2 = 5 & x1 = –3
y-y = 6 Þ = 6

o
rd
3 3 \ c = –15

p
45. (d) Let the number be x.

re
4
y = 18 and x = ´18 = 24 Then, x + x2 = 182 Þ x2 + x – 182 = 0

s
3

s
Þ (x + 14)(x – 13) = 0 Þ x = 13.

.c
39. (e) Let the marked price be ` x

o
46. (e) The series is × l + 1, × 2 + 2, × 3 + 3, ... So 8 is wrong.

m
A. cost price = (1 – 0.15)x = ` 0.85x
47. (e) The series is × l + 12, × 2 + 22, × 3 + 32 ....
B. S.P. = ` 3060
C. Profit = 2% of x = 0.02x 48. (b) The series is × 2 + 9, × 2 + 11, × 2 + 13 ....
49. (d) The series is × 1 – 1, × 2 + 2, × 2 – 2, × 3 + 3, ....
0.02x 50. (d) The series is × 3 + 1, × 4 + 1, × 5 + 1, ....
Profit earned on the cost price = ´ 100 » 2.35%
0.85x
0.02x = 3060 – 0.85x 51-55 : P T Q V S R
Row 1
3060
0.87x = 3060 or x =
0.87
3060 Row 2
Actual profit = 0.02x = 0.02 ´ = ` 70.34 C D E F A B
0.87
40. (e) A. Total marks in 4 subjects including English 51. (c) P, R sit at extreme ends of the rows.
= 4 × 60 = 240 52. (e) A faces S.
B. Total marks in English and Maths = 170 53. (b) 2 persons are seated between T and S i.e. Q and V.
C. Total marks in Maths and Science = 180 54. (a) V is at third position in right side of P. F is at third
The question can’t be answered because nothing has position in right side of C in the same way B is at third
been said about the marks in the fourth subject. position in right side of E.
Also, there are four unknowns but only three equations 55. (e) F faces V is true.
can be formed with given data.
F sits third to the right of C.
41. (c) 3m 2 - 21m + 30 < 0
F is immediate neighbour of A.
or m 2 - 7m + 10 < 0 F does not sit at extreme ends of the line.
or m 2 - 5m - 2m + 10 < 0 (factorize) F sits second to the right of D.
or m (m - 5) - 2(m - 5) < 0 56. (c) IV does not follow by converting the third statement.
or (m - 2)(m - 5) < 0 Some eagles are rabbits + All rabbits are birds = 1 + A =
I Some eagles are birds ® conversion ® Some birds
Case I : m - 2 > 0 and m - 5 < 0 are eagles. Hence III follows. All snakes are eagles +
Þ m > 2 and m < 5 Þ 2 < m < 5 Some eagles are birds = A + I = No conclusion. Hence
II and consequently I do not follow.
Case II : m - 2 < 0 and m - 5 > 0 Þ m < 2 and m > 5
57. (d) All notebooks are diaries (A) ® conversion ® Some
nothing common diaries are notebooks (I). Now, Some calculators are
Hence, 2 < m < 5 diaries + Some diaries are notebooks = I + I = No
42. (a) Given x2 + px + 12 = 0
conclusion. Hence I does not follow. Some calculators
Since, x = 4 is the one root of the equation, therefore
are diaries + All diaries are computers = I + A = I = Some
x = 4 will satisfy this equation
calculators are computers. Hence II follows. All notebooks
\ 16 + 4p + 12 = 0 Þ p = -7 are diaries + All diaries are computers = A + A = All
Other quadratic equation becomes x 2 - 7 x + q = 0 notebooks are computers. Hence III follows. Some
cameras are calculators + Some calculators are diaries
(By putting value of p)
= I + I = No conclusion. Hence IV does not follow.
Its roots are equal, so, b2 = 4ac
58. (a) All planets are stars + All stars are asteroids = A + A =
49
Þ 49 = 4q or q = A = All planets are asteroids ® conversion ® Some
4 asteroids are planets (I). Hence I does not follow. Nor
43. (d) Given equation is x 2 - (a - 2)x - a - 1 = 0 does II follow by converting the second statement
Sum of the roots, p + q = a – 2 Again, All stars are asteroids + All asteroids are moons
Product of the roots pq = – a – 1 = A + A = A = All stars are moons ® conversion ®
Now, p2 + q2 = (p+q)2 – 2pq Some moons are stars (I). Hence III does not follow.
= (a – 2)2 + 2 (a + 1) Some moons are rocks (I) ® conversion ® Some rocks
= a 2 + 4 - 4a + 2a + 2 = ( a – 1 )2 + 5 are moons (I) + Some moons are stars = I + I = No
Hence, the minimum value of p2 + q2 will be 5 conclusion. Hence IV does not follow.
y
o
u
rs
m
IBPS Specialist (I.T.) Officer Exam 2014 397

a
h
\

b
59. (b) I-type statements can’t be combined among Required distance = AF = 5 km and required

o
themselves. Hence I and III do not follow. Some plastics direction is North

o
b
are mirrors + No mirror is glass = I + E = O = Some 70-71 : The meaningful english word 'LEAPS' will be formed.

.w
plastics are not glasses. Hence II and IV do not follow. L E A P S

o
However, the two make a complementary I-E pair. Hence Left Right

rd
either II or IV follows. 70. (d) P is placed second to the right of E.

p
re
60. (e) All graduates are advocates + Some advocates are 71. (c) The word ‘LEAPS’ will be formed based on the given

s
judges = A + I = No conclusion. Hence II and conditions.

s
.c
consequently III do not follow. Some advocates are 72. (c) Only this follows by combining the two statements.

o
73-77 : P%QÞP<Q

m
judges + All judges are lawyers = I + A = I = Some
advocates are lawyers ® conversion ® Some lawyers P dQÞ P> Q
are advocates. Hence IV follows. Some advocates are P@QÞP£Q
lawyers + Some lawyers are doctors = I + I = No P¬QÞP³Q
conclusion. Hence I does not follow. P#QÞP=Q
61-65 : Member City Mother tongue
73. (b) R ¬ T Þ R ³ T; T d M Þ T > M; M % K Þ M < K
H Chennai Marathi K@ VÞK£V
I Hyderabad Telugu So, R ³ T > M < K £ V
J Bangalore Kannada Conclusions
K Ahmedabad Punjabi I. V d M Þ V > M (True)
L Delhi Bangla II. V d T Þ V > T (False)
M Kolkata Tamil III. M % R Þ M < R (True)
N Mumbai Hindi IV. K d R Þ K > R (False)
61. (d) 62. (a) 63. (b) 64. (c) 65. (e) Only I and III are true.
66. (c) Mega corp’s goal can be met only if continuous 74. (e) H d J Þ H > J; J # N Þ J = N; N @ R Þ N £ R R d W Þ
improvement in quality is possible. R> W
67. (b) Consider option (b), So, H > J = N £ R > W
P × R ÷ Q, it means that P is the brother of R and R is the Conclusions
mother of Q. So, P is the maternal uncle of Q. I. W % N Þ W < N (False)
68. (b) II. W % H Þ W < H (False)
III. R # J Þ R = J
L > V < J + P IV. R d J Þ R > J
Sister

Only either III or IV is true.


Daughter Wife Father 75. (a) B @ D Þ B £ D; D d F Þ D > F; F % M Þ
C ous

F < M, M ¬ N Þ M ³ N
in

So, B £ D > F < M ³ N


S × A < D + F < E + K Conclusions
I. B % F Þ B < F (False)
Son Wife Father Wife Father II. M d D Þ M > D (False)
Hence, S and P are first cousins. III. N % F Þ N < F (False)
69. (e) According to questions. IV. D d N Þ D > N (False)
AB = 2 km; BC = 5 km; CD = 8 km
So, none of the given conclusions is correct.
DE = 5 km; EF = 1 km; BC = DE = 5 km
76. (c) F # Z Þ F = Z; Z @ H Þ Z £ H; H % N Þ
CD = BE = 8 km
H < N, N d B Þ N > B
BE = EF + AF + AB So, F = Z £ H < N > B
C 5 km B Conclusions
I. F @ H Þ F £ H (True)
left left II. N % Z Þ N > Z (True)
2 km III. B % H Þ B < H (False)
IV. B % Z Þ B < Z (False)
AStarting Only I and II are true.
8 km point 77. (d) M % K Þ M < K; K ¬ W Þ K ³ W; W d V Þ W > V,
V@ NÞV£N
So, M < K ³ W > V £ N
F Conclusions
left left I. N ¬ K Þ N ³ K (False)
1 km
II. M % W Þ M < W (False)
D 5 km E III. K d V Þ K > V (True)
\ AF = BE – (EF + AB) IV. V % M Þ V < M (False)
= 8 – (1 + 2) = 8 – 3 = 5 km Only III is true.
y
o
u
rs
m
398 IBPS Specialist (I.T.) Officer Exam 2014

a
h
83. (c) Input: say not you are only wise yet

b
78. (d) We have been given

o
Kiran > Manoj ...(i) 234567

o
b
Dilip > Neelam ...(ii) Arrangement: not you only say wise yet are

.w
Now, who is the youngest? We need information by Step VI : 2 3 5 1 6 7 4

o
which the above equations can be combined into a 84. (a) Step V: s o cd rom lay is nor it

rd
single equation. 3 5 6 2 7 4 1

p
Step II: 7 4 1 6 2 3 5

re
From I: If Kiran > Neelam then either
is nor it rom lay s o cd

s
Manoj or Neelam will be the youngest.

s
.c
85. (e) Step III: lo men chi from yet as know
From II: If Dilip > Manoj then either Manoj or Neelam

o
6 7 4 5 1 2 3

m
be the youngest. Input: 1 2 3 4 5 6 7
Hence, neither I nor II is sufficient. yet as know chi from lo men
79. (d) From I : M 86. (e) The rule is given above.
87. (b) Input : may sen to cry if not hell
Step 4 : 5 6 7 3 4 1 2
B(+) (+)
Step 4 : if not hell to cry may sen
From II: 88. (d) Raman scores the highest and Tony gets the least.
Vickey always scores more than Priya which in turn
scores more than Ankit. If Sunil is ranked sixth and
Ankit is ranked fifth then considering above (a) and
(c) is ruled out. (b) is also not possible. only (d) is true.
Hence (d) is correct option.
Both the statements I and II are not independently
89. (c) From the same conclusion as in the previous question.
sufficient because statement I does not say about F
If Raman gets the highest then Vickey should not be
and statement II does not say about M. ranked lower than fourth.
80. (b) From I: Hence (c) is the correct option.
N 90. (b) It is clear from the statement that a farm’s areas for
K V
W E organic and chemical farming are different.
91. (c) There is a perception among small brands that sale in a
S S supermarket is higher than that of small grocery stores.
[Information regarding H is absent. Hence, I alone is 92. (a) All the three causes will effect the sales of four
not sufficient] wheelers.
From II: 93. (c) Whenever such a decision is taken, the assumptions
M are that it would be welcome and allowed to implement.
94. (c) The urging of the govt makes sense only when (A)
and (B) are implicit.
95. (b) W2 N 1 V 9 G 2 P 4 X 6 K 7 R 1 T 8 L 3 H 5 Q 8 U 2 J
V H
96. (d) In today’s world the production and analysis of visual
Hence, H is to the east of V images is of major importance in communications,
81. (d) We have no idea about the location of Durgapur, business and recreation.
Devipur and Rampur. Mere distances between 97. (a) The corresponding element of the succeeding term
Durgapur and Rampur and Devipur and Rampur are moves six places forward.
not enough to locate the places. 98. (a) 15 m N
82. (a) Within ten days means before 28th Feb because
10 m

advertisement was released on 18th February.


W E
83-87 : Here the rule followed is: In each step the fourth word
becomes first word and the last word becomes fourth S
word and all other words shift one place rightwards O
except the third, which shifts two place rightwards. 99. (a) The very purpose of hiking the procurement price of a
crop is encouraging the farmers to cultivate it.
In order to make things easier, let us represent the
100. (a) After dropping every third letter, we get
words digitally from 1 to 7.
ABD E GHJ KMNP QST V WYZ
Then we have: (11– 7 =) 4th from the right.
Input: 1 2 3 4 5 6 7 101. (c) 102. (e) 103. (a) 104. (b) 105. (d)
106. (e)
Step I: 4 1 2 7 3 5 6
107. (a) Replace ‘on’ by ‘at’.
Step II: 7 4 1 6 2 3 5 108. (d) Replace ‘did lack’ by ‘lacked’.
Step III: 6 7 4 5 1 2 3 109. (e) 110. (e) 111. (b) 112. (d) 113. (e)
Step IV: 5 6 7 3 4 1 2 114. (d) 115. (b) 116. (a) 117. (c) 118. (c)
119. (d) 120. (e) 121. (d) 122. (b) 123. (c)
Step V: 3 5 6 2 7 4 1 124. (d) 125. (e) 126. (c) 127. (b) 128. (e)
Step VI: 2 3 5 1 6 7 4 129. (b) 130. (a) 131. (c) 132. (d) 133. (b)
y
o
u
rs
m
IBPS Specialist (I.T.) Officer Exam 2014 399

a
h
134. (a) 135. (d) 136. (b) 137. (d) 138. (c) 171. (a) A “compile-time” error is one which prevents your

b
o
139. (c) 140. (a) 141. (c) code from compiling. Compile-time errors are divided

o
b
141. (c) The widest divergence between India and China is the into three categories:

.w
profitable horticultural sector ..... China’s added 1. Lexical: These generally occur when disallowed

o
advantage lies in the more diversified composition of characters are included code (e.g. int #people =

rd
its agricultural sector. 10;).

p
142. (a) In all of these China has outdone India.

re
2. Syntactical: These occur when code is “out of
143. (e) None of these is fully true.

s
order” (e.g. for (int i=0; i++; i<10)).

s
144. (c) There has been much debate about subsidies and their

.c
3. Semantic: These occur when the meaning of code
utility; the opposing view being that subsidies are

o
is unclear (e.g. two variables with the same name).

m
against the market reforms and distort the market as
Note that the exact wording of these errors may vary,
well as reduce resource efficiency.
depending on which development environment using.
145. (d) 146. (b) 147. (c) 148. (b) 149. (d) Errors in a computer program can be classified
150. (c) according to when they are detected and, if they are
151. (c) Protection: The purpose of a computer systems is to detected at compile time, what part of the compiler
allow the user to execute programs. So the operating detects them.
systems provides an environment where the user can (a) A lexical error, detected by the scanner.
conveniently run programs. The user does not have to (b) A syntax error, detected by the parser.
worry about the memory allocation or multitasking or (c) A static semantic error, detected (at compile-time)
anything. These things are taken care of by the by semantic analysis.
operating system. (d) A dynamic semantic error, detected (at run-time)
Accounting: The output of a program may need to be by code generated by the compiler
written into new files or input taken from some files.
In lexical scoping (or lexical scope; also called static
The operating systems provides this service. User gives
scoping or static scope), if a variable name’s scope is a
a command for reading or writing to a file and sees his
her task accomplished. Thus operating systems makes certain function, then its scope is the program text of
it easier for user programs to accomplished their task. the function definition: within that text, the variable
Compilation: A compiler translates the high-level name exists, and is bound to the variable’s value, but
source programs into target programs in machine outside that text, the variable name does not exist. By
languages for the specific hardwares. Once the target contrast, in dynamic scoping (or dynamic scope), if a
program is generated, the user can execute the variable name’s scope is a certain function, then its
program.Compilation is not a service supported by scope is the time-period during which the function is
Operating system. executing: while the function is running, the variable
I/O Operations : Each program requires an input and name exists, and is bound to its variable, but after the
produces output. This involves the use of I/O. The function returns, the variable name does not exist.
operating systems hides the user the details of 172. (d) Multilevel queue scheduling algorithm is used,
underlying hardware for the I/O. All the user sees is processes are permanently assigned to a queue when
that the I/O has been performed without any details. they enter the system. If there are separate queues for
So the operating systems by providing I/O makes it foreground and background processes, processes do
convenient for the users to run programs. not move from one queue to the other, since processes
For efficiently and protection users cannot control I/O do not change their foreground or background nature.
so this service cannot be provided by user-level programs This setup has the advantage of low scheduling
152. (b) FIFO scheduling is Non-pre-emptive scheduling overhead, but it is inflexible. The multilevel feedback-
Process that requests CPU first is allocated the CPU queue scheduling algorithm, in contrast, allows a
first. It is non preemptive and cannot utilize resources process to move between queues.
in parallel.
153. (a) 154. (a) 155. (b) 156. (b) 157. (a) 173. (c) 174. (b) 175. (c) 176. (b)
158. (c) 177. (d) (a) An SQL query automatically eliminates
159. (d) Layer 2 switching creates individual collision domains. duplicates:False
160. (c) Static routes have an administrative distance of 1 by There may be a situation when you have multiple
default. Unless you change this, a static route will duplicate records in a table. While fetching such
always be used over any other found route. IGRP has records, it makes more sense to fetch only unique
an administrative distance of 100, and RIP has an records instead of fetching duplicate records.The
administrative distance of 120, by default. SQL Distinct keyword, which we already have
161. (c) 162. (d) 163. (c) 164. (c) 165. (d) discussed, is used in conjunction with SELECT
167. (d) 168. (b) statement to eliminate all the duplicate records
169. (c) Turnaround time is the time interval between the first and fetching only unique records.
submission of process to the completion of process. Syntax:
170. (a) E-R modeling technique is a top-down approach. It is a The basic syntax of DISTINCT keyword to
graphical technique, which is used to convert the eliminate duplicate records is as follows:
requirement of the system to graphical representation, SELECT DISTINCT column1, column2,.....
columnN
so that it can become well understandable. It also
FROM table_name
provides the framework for designing of database. WHERE [condition]
y
o
u
rs
m
400 IBPS Specialist (I.T.) Officer Exam 2014

a
h
b
(b) An SQL query will not work if there are no indexes 182. (b) Bottom-up parsers.

o
on the relation;False Shift-reduce parsing is the type of bottom-up parsers.

o
b
Indexes are used to police database constraints, 183. (a) Left most derivation.

.w
such as UNIQUE, EXCLUSION, PRIMARY KEY Top-down parsing, always gives left most derivation.

o
and FOREIGN KEY. An index may be declared as

rd
184. (d)
UNIQUE, which creates an implicit constraint on

p
185. (a) The language which are having many types, but the

re
the underlying table. Database systems usually type of every name and expression must be calculable

s
implicitly create an index on a set of columns

s
at compile time is called strongly typed.

.c
declared PRIMARY KEY, and some are capable of

o
186. (b) A compiler reads, analyses and translates code into

m
using an already existing index to police this
either an object file or a list of error messages. A linker
constraint. Many database systems require that
combines one or more object files and possible some
both referencing and referenced sets of columns
library code into either some executable, some library
in a FOREIGN KEY constraint are indexed, thus
or a list of error messages. A loader reads the executable
improving performance of inserts, updates and
code into memory, does some address translation and
deletes to the tables participating in the constraint.
tries to run the program resulting in a running program
Tables are combined in queries by matching the
or an error message (or both).
column or columns in the foreign key . However,
187. (a) The task of the lexical analysis phase is to parse the
creating this index is not required.
source program into the basic elements or tokens of
(c) SQL permits attribute names to be repeated in the
the language. The lexical analyze is the first phase of a
same relation: False
compiler. Its main task is to read input characters and
An attribute value is an attribute name paired with an
produce as output a sequence of tokens that the parser
element of that attribute’s domain, and a tuple is a set
uses for the next phase, the syntax analysis
of attribute values in which no two distinct elements
188. (d) In telecommunication a data link is the means of
have the same name. Thus, in some accounts, a tuple
connecting one location to another for the purpose of
is described as a function, mapping names to values. A
transmitting and receiving digital information. It can
set of attributes in which no two distinct elements have
also refer to a set of electronics assemblies, consisting
the same name is called a heading. A set of tuples
of a transmitter and a receiver (two pieces of data
having the same heading is called a body
terminal equipment) and the interconnecting data
178. (d) Index sequential file is made of all of these except
telecommunication circuit.
address of prime data track. An indexed file system
consists of a pair of files, one holding the data and one 189. (c) The Internet layer is a packet switching network based
storing an index to that data. The index file will store on connectionless communication. Hosts send packets
the addresses of the records stored on the main file. into the network and then the packets travel
179. (d) independently to their destinations since the sender
180. (b) In this scheme, an assembler runs in one portion of (System A) is not directly connected to the receiver
memory and assembled machine instructions and data (System B).
are directly placed into their assigned memory locations 190. (c)
as soon as they are assembled. When an assembly is 191. (a) Packets addressed to a unicast address are delivered
completed, the assembler causes the transfer to the to a single interface. For load balancing, multiple
starting instruction of a program. This is a simple interfaces can use the same address.
solution used by WATFOR FORTRAN compiler and 192. (a) Network Control Protocol is used to help identify the
several other language processors. The compile and Network layer protocol used in the packet.
go loader scheme is shown below. 193. (b)
Disadvantages: 194. (d) The keyword return is used to transfer control from a
1. A portion of memory is wasted because the function back to the calling function.
memory occupied by an assembler is unavailable 195. (d) Variable names in C are made up of letters (upper and
to the object program. lower case) and digits. The underscore character ("_")
2. It is necessary to retranslate the users program is also permitted. Names must not begin with a digit.
every time it is run. 196. (c) The statement 'c' is correct. When we pass an array as
3. It is difficult to handle multiple segments a funtion argument, the base address of the array will
especially if the source programs are in different be passed.
languages (i.e one subroutine in FORTRAN and 197. (b) The statement 1 and 2 does not yield the base address
other in C/C++ and still other in assembly of the array. While the scanf() and printf() yields the
language). This disadvantage makes it difficult to base address of the array.
produce orderly modular programs. 198. (e) These three constructs are sufficient to program any
181. (d) scanner is used for grouping of characters into tokens alogrithm. Moreover, as far as possible single entry
which represents a group of characters forming basic, single exit control constructs are used.
atomic chunk of syntax; of a word. 199. (a) 200. (a)
y
o
u
rs
m
a
h
b
o
CORPORATION BANK SPECIALIST OFFICER

o
b
.w
EXAM 2014

o
rd
p
Based on Memory

re
s
s
.c
o
m
REASONING ABILITY (b) One of the immediate neighbours of F teaches Biology
(c) E is sitting exactly between B and the person who
DIRECTIONS (Qs. 1-6) : Study the following information teaches Mathematics
carefully and answer the given questions : (d) The person who teaches Chemistry is second to the
right of E
Eight people - A. B, C, D, E, F, G and H are sitting around a circular
table facing towards the centre, but not necessarily in the same (e) All are true
order. All of them are at equidistant. Each one of them teaches 6. Four of the following five are alike in a certain way based on
different subjects viz., English, Hindi, Mathematics, Biology, the given sitting arrangement and hence form a group.
Psychology, Physics, Chemistry and Accounts, but not necessarily Which is the one that does not belong to the group?
in the same order. (a) CF (b) GA
The person who teaches Accounts, sits third to the right of G. (c) BD (d) EH
C is an immediate neighbour of G. The person who teaches Mathematics (e) BA
sits second to the left of C. B sits third to the right of H. H teaches
DIRECTIONS (Qs. 7-11): Study the following information
neither Accounts nor Mathematics. Only two persons sit between
carefully and answer the given questions :
C and the person who teaches Physics. A and F are immediate
neighbours of each other. Neither A nor F teaches Accounts. The Twelve persons are sitting in two parallel rows containing six
person who teaches English sits second to the right of A. Two persons each, in such a way that there is an equal distance between
persons sit between D and the person who teaches Hindi. D does adjacent persons. In Row-I, A, B, C, D, E and F are seated (but not
not teach Accounts. The person who teaches Psychology is an necessarily in the same order) and all of them are facing south. In
immediate neighbour of the person who teaches Accounts. The Row-2, P, Q, R, S, T and U are seated (but not necessarily in the
person who teaches Physics sits second to the left of A. One of same order) and all of them are facing north. Therefore, in the
the immediate neighbours of G teaches Chemistry. given sitting arrangement each person seated in a row faces another
person of the other row.
1. Who among the following teaches Chemistry? S sits immediate the right of U. S does not sit at any extreme end.
(a) A (b) H Two persons sit between P and Q. T sits third to the left of U. R is
(c) D (d) G not an immediate neighbour of U.
(e) None of these F sits third to the right of D. C does not sit at any extreme end. One
2. What is the position of B with respect to the person who person sits between B and D. D sits second to the left of C. C is
teaches Psychology? not an immediate neighbour of B and A.
(a) Second to the left (b) Third to the right 7. Who among the following are sitting at the extreme ends of
(c) Third to the left (d) Second to the right any row?
(e) None of these (a) ST (b) EB
(c) FB (d) TP
3. Who among the following sits exactly between the person
(e) DE
who teaches Biology and the person who teaches Physics?
8. What is the position of S with respect to R?
(a) The person who teaches Mathematics
(a) Third to the right (b) Second to the right
(b) E
(c) Third to the left (d) Second to the left
(c) The person who teaches Accounts (e) Fourth to the right
(d) Cannot be determined 9. Four of the following five are alike in a certain way based on
(e) There is no such person their sitting arrangement and hence form a group. Which
4. Which of the following subjects does E teach? one does not belong to the group?
(a) Chemistry (b) Hindi (a) CR (b) BQ
(c) Accounts (d) English (c) FT (d) AS
(e) None of these (e) EU
5. Which of the following statements is true with regard to the 10. What is the position of A with respect to E?
given sitting arrangement? (a) Second to the right (b) Second to the left
(a) The person who teaches Hindi is an immediate (c) Third to the left (d) Third to the right
neighbour of both H and D. (e) Immediate left
y
o
u
rs
m
402 Corporation Bank Specialist Officer Exam 2014

a
h
18. Who is standing second to the right of C ?

b
11. If all the six persons of Row-1 (A, B, C, D, E and F) are asked

o
to sit in an alphabetical order from left to right, the positions (a) F (b) D

o
b
of how many will remain unchanged as compared to their (c) G (d) E

.w
original sitting position? (e) None of these

o
rd
(a) None (b) Two 19. Four of the following five are alike in a certain way based on

p
(c) Three (d) One their positions in the above arrangement and so form a group.

re
(e) More than three Which of the following does not belong to the group ?

s
s
(a) CG (b) GE

.c
DIRECTIONS (Qs. 12-15) : In each of the questions below, three

o
state-ments are given followed by two conclusions numbered I (c) GH (d) DE

m
and II. You have to take the three statements to be true even if (e) FD
they seem to be at variance from the commonly known facts and 20. If all the people are asked to stand in an alphabetical order
then decide which of the given conclusions logically follows from from left to right, the positions of how many will remain
the given statements disregarding the commonly known facts. unchanged ?
Give answer (a) if only conclusion I follows. (a) One (b) Two
Give answer (b) if only conclusion II follows. (c) Three (d) None
Give answer (c) if either conclusion I or II follows. (e) None of these
Give answer (d) if neither conclusion I nor II follows. DIRECTIONS (Qs. 21- 25): In each question below is given a
Give answer (e) if both conclusions I and II follow. statement followed by three courses of action numbered I, II and
12-13 : III. A course of action is a step or administrative decision to be
Statements : taken for improvement, follow-up or further action in regard to
All diamonds are stones. the problem, policy, etc. On the basis of the information given in
All stones are gems. the statement, you have to assume everything in the statement to
be true, then decide which of the suggested courses of action
No gem is a diamond.
logically follow(s) for pursuing.
12. Conclusions :
I. All gems are stones. II. All diamonds are gems. 21. Statement : The major road connecting the two main parts
13. Conclusions : of the city is inundated due to heavy rains during past two
I. No gem is a diamond. II. No diamond is a stone. days.
14. Statements : Courses of action :
No day is night. I. The government should immediately send a relief team
to the affected area.
All nights are noons. No noon is evening.
II. The municipal authority should make immediate effort
Conclusions :
to pump out water from the road.
I. No day is noon. II. No night is evening.
III. The municipal authority should advise the general
15. Statements :
public to stay indoors till the water is cleared.
All jackets are trousers. No trouser is shirt.
(a) Only I follows (b) Only I and II follow
Some shirts are caps.
(c) Only I and III follow (d) Only II and III follow
Conclusions :
(e) All of these
I. Some caps are jackets. II. Some shirts are jackets.
22. Statement: The prices of essential commodities have gone
DIRECTIONS (16-20) : Study following information to answer up substantially during the past few weeks.
the given questions : Courses of action :
Six people C, D, E, F, G and H–are standing in a straight line I. The government should set up an expert committee to
facing north not necessarily in the same order. D is standing study the trend of prices.
second to the right of F. C is standing fourth to the left of H and H II. The government should immediately abolish taxes on
is not standing on the extreme end of the line. E is standing second essential commodities.
to the right of D. III. The government should advise the general public to
16. What is the position of G with respect to E ? refrain from purchasing essential commodities for few
(a) Immediate left (b) Second to the left days.
(c) Third to the left (d) Third to the right (a) None follows (b) Only I follows
(e) None of these (c) Only II follows (d) Only III follows
17. Which of the following pairs represents the people standing (e) Only I and II follow
at the extreme ends of the line? 23. Statement : Large number of students of the local school
(a) FH (b) CE fell ill after having their midday meal provided by the school.
(c) DE (d) CH Courses of action :
(e) None of these I. The government should immediately suspend the
school Principal.
y
o
u
rs
m
Corporation Bank Specialist Officer Exam 2014 403

a
h
b
II. The school authority should initiate an enquiry to find 28. How many such pairs of letters are there in the word

o
out the reason for students' sickness. ADJUSTING each of which has as many letters between

o
b
III. The government should instruct all the schools to them in the word as in the English alphabet ?

.w
suspend their midday meal scheme till further order. (a) None (b) One

o
rd
(a) Only I follows (b) Only II follows (c) Two (d) Three

p
(c) Only III follows (d) Only II and III follow (e) More than three

re
(e) None of these 29. W walked 30 metres towards South, took a left turn and

s
s
24. Statement : Many people living in the slums of western part walked 50 metres, again he took a left turn and walked

.c
o
of the town are diagnosed to be suffering from malaria. 30 metres. How far is he from the starting point ?

m
Courses of action : (a) 80 metres (b) 100 metres
I. The municipal corporation should immediately make (c) 130 metres (d) 50 metres
necessary arrangements to spray mosquito repellent (e) None of these
in the affected area. 30. Cause : A severe cyclonic storm swept away most part of
II. The municipal authority should immediately make the state during the last two days.
necessary arrangements to provide quick medical help Which of the following cannot be a possible effect of the
to the affected people. above cause ?
III. The municipal authority should take steps to shift all (a) Heavy rainfall was reported in most part of the state
the people staying in the slums of western part of the during the last two days.
town to other areas. (b) Many people were rendered homeless as their houses
(a) Only I follows (b) Only II follows were flown away.
(c) Only I and II follow (d) All I, II and III follow (c) The communication system of the state was severely
(e) None of these affected and continues to be out of gear.
25. Statement: Many teachers of the local school have rendered (d) Government has ordered that all the offices and schools
their resignation to the Principal to protest the management's should be kept open.
decision for not implementing revised pay scales. (e) All are possible effects.
Courses of action : 31. Effect : The temple at the religious site wears a deserted
I. The school management should accept the resigna- look with the number of devotees trickling down.
tions and appoint new teachers. Which of the following can be a possible cause of the above
II. The school management should persuade the teach- effect?
ers to continue with an assurance of considering the (a) A structural engineer had visited the temple a month
pay hike issue. back and had declared the structure unsafe.
III. The school management should approach the gov- (b) The temple is facing a drastic depletion of its funds
ernment for guidelines. which had accumulated over the years due to offerings
(a) None follows (b) Only either I or II follows made by devotees.
(c) Only III follows (d) Only I follows (c) The local corporation decided to donate a huge amount
(e) Only II follows of money to the temple for its renovation.
26. Statement: The Government has decided to construct an (d) The village housing the religious site has qualified
eight-lane super highway across the state to facilitate fast priests to perform religious ceremonies.
movement of vehicles. (e) A famous actor recently visited the temple and paid his
Which of the following can be an assumption which is respects to the deity.
implicit in the above statement? 32. Statement: The constable has been recommended for a
(a) The Government has adequate resources to construct suitable reward by his superior in recognition of his sincere
the proposed super highway duty and busting of several gangs of criminals actively
(b) The people of the state may protest against the involved in the loot and incidents of pick pocketing.
Government's decision as their farm land will be taken Which of the following can be a possible assumption of the
over the Government for constructing the highway. above statement ?
(c) The Government may find it difficult to enrol a suitable (a) The superior is certain that the recommendation would
contractor for constructing the highway. be denied.
(d) There is no other highway which can be used for (b) The number of criminals apprehended by this particular
transportation of goods across the state constable was exceptionally high.
(e) None of these (c) The constable desires to be monetarily compensated
27. In a certain code CONQUER is written as MNBRQDT. How for his efforts.
is STEAMER written in that code? (d) The superior wants to set an example for his other
(a) DRSBQDL (b) DSRBLDQ juniors by recommending the reward.
(c) DSRZQDL (d) DSRBQDL (e) Rewards recognising the sincerity and accomplish-
(e) None of these ments of policemen are given.
y
o
u
rs
m
404 Corporation Bank Specialist Officer Exam 2014

a
h
33. Which of the following groups of alphabets should replace

b
In each question below is given details of one candidate. You

o
the blank spaces so that the group of alphabets, given in have to take one of the following courses of actions based on the

o
b
bold, follow a logical pattern from the preceding and the information provided and the conditions and subconditions given

.w
following group of alphabets ? above and mark your answer accordingly. You are not to assume

o
bw __ yza dstuv __ __opqre anything other than the information provided in each question.

rd
(a) x, e, d (b) x, c, f

p
All these cases are given to you as on 01.03.2013.

re
(c) v, e, f (d) x, c, d Mark answer (a) if the candidate is not to be selected.

s
s
(e) x, e, n Mark answer (b) if the candidate is to be selected.

.c
o
DIRECTIONS (Qs. 34): Read the following statements carefully Mark answer (c) if the data are inadequate to take a decision.

m
and answer the questions which follow. Mark answer (d) if the case is to be referred to Vice President -
34. According to a recent government directive, all bank Marketing.
branches in rural areas should be computerized. Mark answer (e) if the case is to be referred to GM-Marketing.
Which of the following statements would weaken the 36. Suresh Mehta has secured 58 per cent marks in graduation.
government's argument ? He was born on 19th May 1979. He has secured 50 per cent
(a) Computerisation of bank branches in urban areas has marks in the selection process. He has been working for the
helped in making their performance more efficient and past seven years in the Marketing division of an organ-
fast. isation after completing his Post Graduation with 62 per
(b) Lack of skilled and qualified manpower has been cent marks.
suitably substituted by computers in banks.
37. Sudha Gopalan has secured 50 per cent marks in both selec-
(c) Non-computerised bank branches in the rural areas
tion process and graduation. She has been working for the
have been proved to be as efficient as their computer-
ized counterparts. past six years in the Marketing division of an organisation
after completing her Post Graduate Diploma in Marketing
(d) The government has introduced a special test for
with 70 per cent marks. She was born on 14th October. 1982.
computer knowledge in all recruitment exams for banks.
(e) Unemployment in the rural areas could be controlled 38. Divya Kohli has been working for the past five years in
by training more and more professionals in computers. Marketing division of an organisation after completing her
35. If 'B × C' means 'B is the daughter of C', 'B + C' means 'B is the Post Graduate Diploma in Marketing with 65 per cent marks.
husband of C' and 'B – C' means 'B is the sister of C' then She has secured 55 per cent marks in graduation and 50 per
what does 'M + N – P × Q' mean ? cent marks in the selection process. She was born on 2nd
(a) M is the brother-in-law of Q April 1979.
(b) M is the uncle of Q 39. Navin Marathe was born on 8th April 1979. He has secured
(c) M is the son-in-law of Q 60 per cent marks in both graduation and Post-Graduate
(d) Q is the mother-in-law of M Degree in Marketing. He has been working for the past six
(e) None of these years in the Marketing division of an organisation after
com-pleting his PG Degree in Marketing. He has secured 50
DIRECTIONS (Qs. 36-40): Study the following information per cent marks in the selection process.
carefully and answer the questions given below :
40. Varun Malhotra was born on 3rd July 1980. He has been
Following are the conditions for selecting Marketing Manager in working as Deputy Marketing Manager in an organisation
an organisation : for the past three years after completing his Post Graduate
The candidate must : Degree in Marketing with 65 per cent marks. He secured 55
(i) be at least 30 years old as on 01.03.2013 per cent marks in both graduation and selection process.
(ii) have secured at least 55 per cent marks in graduation
DIRECTIONS (Qs. 41-45): Study the following information to
(iii) have secured at least 60 per cent marks in Post graduate answer the given questions :
Degree/Diploma in Marketing.
In a certain code 'it is rush hour traffic' is written as 'sa le do mi
(iv) have post qualification work experience of at least five
ru' 'go to school' is written as be no pa', 'one hour to go' is written
years in the Marketing Division of an organisation.
as 'mi fi pa be', 'rush to one' is written as 'fi be sa' and 'traffic is
(v) have secured at least 45 per cent marks in the selection
fine' is written as 'ga ru do'.
process.
In the case of a candidate who satisfies all other conditions 41. Which of the following represents 'school hour go fine' ?
except - (a) pa be fi ga (b) no mi ra pa
(A) at (iv) above, but has post qualification work experience (c) pa no ga mi (d) ga no mi le
of at least two years as Deputy Marketing Manager, (e) None of these
the case is to be referred to GM-Marketing. 42. 'mi fi le' would mean ____
(B) at (ii) above, but has secured at least 65 per cent marks
(a) it one to (b) to rush one
in Post graduate Degree/Diploma in Marketing
Management, the case is to be referred to Vice (c) rush hour it (d) it one hour
President-Marketing. (e) None of these
y
o
u
rs
m
Corporation Bank Specialist Officer Exam 2014 405

a
h
QUANTITATIVE APTITUDE

b
43. What does sa' stand for?

o
o
(a) rush (b) traffic

b
.w
(c) it (d) is DIRECTIONS (Qs. 51-55) : In each of the following number

o
(e) None of these series, a number is wrong. Find out that wrong number.

rd
44. Which of the following may represent 'traffic is for one hour' ? 51. 2 6 15 30 45 43.5 22.5

p
re
(a) fi ye no mi ru (b) fi le do mi ru (a) 6 (b) 30

s
(c) 45 (d) 15

s
(c) fi ye do mi ru (d) fi so do mi ro

.c
(e) 43.5

o
(e) None of these

m
52. 950 661 436 269 146 65 16
45. What does 'do' stand for?
(a) 436 (b) 65
(a) hour (b) 'is' or 'traffic' (c) 269 (d) 661
(c) it (d) is (e) 146
(e) None of these 53. 6.5 11.8 22.4 38.3 59.5 87.3 117.8
DIRECTIONS (Qs. 46- 50) : Study the following information (a) 22.4 (b) 59.5
carefully and answer the given questions. (c) 11.8 (d) 38.3
(e) 87.3
Seven flights namely Jet Air-ways, British Airways, Delta, Quan- 54. 1 2 4 9 23 69 186
tas, Emirates, Lufthansa, Air India are scheduled to fly to London. (a) 2 (b) 9
There is only one flight to London on each of the seven days of (c) 23 (d) 4
the week, starting from Monday and ending on Sunday. (e) 69
Delta flies on Wednesday. Air India flies the day next to British 55. 250 239 216 181 136 75 4
Airways. British Airways does not fly on Monday or Friday. Two (a) 239 (b) 181
airlines fly between the days British Airways and Emirates fly. (c) 75 (d) 216
Emirates does not fly on Sunday. Quantas flies a day before (e) 136
Lufthansa. 56. What will be the difference between the interest accrued on
a sum of ` 4500 at 12% per annum for 2 years and that on a
46. On which of the following days does Jet Airways fly ?
sum of ` 5600 at 9% per annum for 2 years ?
(a) Friday (b) Sunday (a) ` 75 (b) ` 72
(c) Tuesday (d) Thursday (c) ` 69 (d) ` 76
(e) None of these (e) None of these
47. How many flights fly between Lufthansa and Delta ? 3
(a) None (b) One 57. If 50% of a certain number is equal to th of another number,,
4
(c) Two (d) Three what is the ratio between the numbers ?
(e) Five (a) 3 : 2 (b) 2 : 5
48. Which of the following flights flies on Friday ? (c) 5 : 2 (d) 3 : 4
(a) Air India (b) Quantas (e) 4 : 3
58. A 320 metre long train crosses a pole in 16 seconds. It stops
(c) Emirates (d) Lufthansa
five times of duration 18 minutes each. What time will it take
(e) Jet Airways in covering a distance of 576 km ?
49. If Delta postpones its flight to Sunday owing to some
1
technical reasons and all the flights scheduled for Thursday (a) 9 hours (b) 9 hours
to Sunday are now made to take off a day ahead of the 4
schedule, which of the following flights would now fly on 1 1
Friday? (c) 9 hours (d) 8 hours
2 2
(a) Lufthansa (b) Jet Airways (e) None of these
(c) British Airways (d) Air India DIRECTIONS (Qs. 59 - 62): In each of the following questions,
(e) Quantas a question and three statements following it have been given.
50. If Emirates is related to British Airways and Delta is related You are required to study the question and statements and decide
to Quantas in a certain way based upon the given flight that the information given in which statement(s) is necessary to
answer the question ?
schedule, then Jet Airways will be related to which of the
following based upon the same relationship ? 59. What is the principal?
I. The simple interest accrued on that sum at the rate of
(a) Lufthansa (b) Quantas
12% per annum in 2 years is ` 360 less than the
(c) Delta (d) Air India compound interest on the same sum at 12% per annum
(e) None of these in 2 years.
y
o
u
rs
m
406 Corporation Bank Specialist Officer Exam 2014

a
h
b
II. The sum doubles itself in 10 years at 10% per annum

o
rate of simple interest. 60000

o
60000 55000 55000

b
III. The compound interest on the sum in 2 years at the

.w
D 50000
rate of 12% per annum is ` 6360. 50000 D D

Number of Passengers
45000

o
rd
(a) I or III (b) I and III D 40000
40000 D

p
C
(c) I and II (d) II and III

re
C C D
C

s
(e) None of these 30000 C

s
.c
C
60. What will be the area of a 2 metre wide boundary around a B B B

o
20000 B B

m
rectangular field ? B
I. The breadth of the field is one-fourth of its perimeter. 10000 A
A A A A
Area of the field is 144 sq. metre. A
II. The ratio of the length and breadth is respectively 3 : 2. Mumbai Delhi Kolkata Chennai Bangaluru Ahemdabad
III. Area of the field is 216 sq. metre. 64. What is the difference between the number of passengers
(a) I, II and III travelling to all cities by the airlines A and C ?
(b) only I or II and III (a) 5000 (b) 6000
(c) 5500 (d) 6500
(c) I and II (e) 15000
(d) Anyone of the three statements 65. The number of passengers travelling to Delhi and Kolkata
(e) None of these by airline C is what per cent of the number of all passengers
61. What are the marks obtained by Arnab in History ? travelling by the same airline?
(a) 32 (b) 35
I. The average marks obtained by Arnab in History,
(c) 38 (d) 42
Geography and Civics are 65.
(e) 44
II. Marks obtained by Arnab in Geography is 6 more than 66. What is the respective ratio of the number of passengers
that obtained in History. who travelled to Chennai and Bangaluru by airline B and
III. Marks obtained in Geography is as much more than that to Mumbai and Ahmedabad by airline D ?
that in Civics as the marks obtained in Civics is more (a) 6 : 7 (b) 3 : 4
than that in History. (c) 1 : 1 (d) 7 : 6
(a) I and II (b) II and III (e) None of these
(c) I and III (d) I, II and III 67. The number of passengers travelling to Chennai by airline
A in second quarter is 150% of that in first quarter by the
(e) None of these
same airline. The number of passengers in the second quarter
62. What is the speed of train ? by the same airline is 120% of that in the third quarter.
I. Train crosses a signal post in 15 seconds. What is the percentage increase in the number of passengers
II. Train crosses a 250 metre long platform in 27 seconds. in third quarter from that in first quarter ?
III. Train crosses another train running in the same direc- (a) 15% (b) 18%
tion on a parallel track in 32 seconds. (c) 20% (d) 25%
(e) 30%
(a) I and II (b) I and III 68. The number of passengers going to Bangaluru and Kolkata
(c) II and III (d) Any two of the three in first quarter by airline B is what per cent of the number of
(e) None of these passengers going to the same cities in second quarter if
63. What percent of discount is offered ? there be an increase of 30% in the number of passengers
going to Bangaluru from first quarter to second quarter and
I. The profit earned on a article sold for `252 after discount
in that going to Kolkata shows a 40% increase from first to
is `52.
second quarter ?
II. If no discount were allowed, there would have been a (a) 68 (b) 60
profit of `80. (c) 65 (d) 70
III. If there were no discount, there would have been a (e) 74
gain of 40%. 69. What will come in place of the question mark (?) in the
(a) I and II (b) II and either I or III following question?
(c) I and III (d) I and either II or III 4
(?)
(e) None of these 5= 9
36 1
DIRECTIONS (Qs. 64 - 68) : In the following bar diagram, the (?)
5
number of passengers carried to different cities in first quarter of
(a) 342 (b) 361
a year by four airlines A, B, C and D has been given. Study the
following diagram carefully and answer the questions. (c) 324 (d) 18
(e) 4
y
o
u
rs
m
Corporation Bank Specialist Officer Exam 2014 407

a
h
b
DIRECTIONS (Qs. 70 - 74): In each of the following questions DIRECTIONS (Qs. 80-84) : In the following table, populations

o
o
equation I and equation II have been given. You have to solve of 6 cities have been given. Study the following table carefully

b
both of these equations and and answer the questions.

.w
o
Given answer If

rd
(a) x< y Percentage Percentage of

p
Total of adult children below 18

re
(b) x> y
City Population Males years of age

s
(c) x£y

s
.c
(d) x³y A 68300000 45 12

o
m
(e) x = y or no relation between two B 42500000 53 14
can be established. C 71200000 52 13
70. I. 4x2 – 32x + 63 = 0 II. 2y2 – 11y + 15 = 0 D 93500000 49 11
E 54500000 41 15
( )
3
71. I. x3 = 3
216 II. 6y2 = 150 F 81200000 43 16
72. I. 12x2 + 17x + 6 = 0 II. 6y2 + 5y + 1 = 0 * Remaining are adult females
73. 2
I. 20x + 9x + 1 = 0 II. 30y2 + 11y + 1 = 0 80. By what per cent is the number of adult males in cities C and
74. I. x2 + 17x + 72 = 0 II. y2 + 19y + 90 = 0 D more than that of adult females in the same cities ?
75. The average height of 16 students is 142 cm. If the height of (a) 33% (b) 30%
the teacher is included, the average height increases by
(c) 35% (d) 35.9%
1 cm. The height of the teacher is
(e) 40%
(a) 156 cm (b) 159 cm
(c) 158 cm (d) 157 cm 81. The number of children in city F is what per cent of the
number of adult females in the same city ?
(e) 159.5 cm
76. The ratio of the present ages of Mahesh and Ajay is (a) 35 (b) 39
respectively 3 : 2. After 8 years, ratio of their ages will be (c) 42 (d) 43
11 : 8. What will be the present age of Mahesh's son if his (e) 44
age is half of the present age of Ajay ? 82. The difference between the number of adult females in the
(a) 12 years (b) 24 years cities A and D is
(c) 18 years (d) 9 years (a) 8031000 (b) 8301000
(e) None of these (c) 8130000 (d) 8231000
77. Pipe A can fill a tank in 30 minutes while pipe B can fill it in (e) None of these
45 minutes. An other pipe C can empty a full tank in 60
83. What is the adult population in cities B and F ?
minutes. If all three pipes are opened simultaneously, the
empty tank will be filled in (a) 107458000 (b) 104758000
(c) 104458000 (d) 114458000
2 3
(a) hour (b) hour (e) None of these
7 7
84. What is ratio of total population of city D and city E ?
4 5 (a) 187:109 (b) 187:119
(c) hour (d) hour (c) 119 : 187 (d) 129 : 187
7 7
(e) None of these (e) None of these
78. An article was sold for ` 5220 at a loss of 42% of the cost DIRECTIONS (Qs. 85-89) : What approximate value will come
price. What will be the selling price of the article for a profit in place of the question mark (?) in the following questions ?
of 42% ? (You are not required to calculate the exact value).
(a) ` 12580 (b) ` 17280
3 5
(c) ` 12780 (d) ` 15280 85. 4× × 952 – 128 = ?
13 7
(e) None of these
(a) 823 (b) 840
79. Geeta, Shweta and Sita started a business with investment
(c) 835 (d) 839
of ` 4200, ` 8400 and ` 5400 respectively. In the end of the
year, there was a profit of ` 24000. Sita invested 32% of her (e) 845
share in profit in the business. The remaining amount of 3
profit with Sita is 86. 105.01% of 8451 × % of 5006 + 9.999 = ?
7
(a) ` 4986 (b) ` 4689
(a) 8879 (b) 8860
(c) ` 4886 (d) ` 4896
(c) 8850 (d) 8760
(e) None of these (e) None of these
y
o
u
rs
m
408 Corporation Bank Specialist Officer Exam 2014

a
h
103 × 1003 + 999999999 = 10? + 10?

b
87. (a) 49% (b) 48%

o
(a) 6, 9 (b) 9, 9 (c) 45% (d) 50%

o
b
(c) 6, 12 (d) 16, 9 (e) 60%

.w
(e) 6, 18

o
DIRECTIONS (Qs. 95-97) : The following questions are based

rd
88. 21 + 3.7 × 2.9 = ? on the information given below. Study the following information

p
(a) 74 (b) 70 carefully to answer the questions.

re
(c) 27 (d) 32

s
In a school students have options to read only Hindi or only

s
.c
(e) 44 Sanskrit or both Hindi and Sanskrit. Of 175 students in the school

o
89. 22.9782 + 9.002 – ? = 23.001 the ratio of boys and girls is 3 : 4 respectively. 40% of boys opted

m
(a) 9 (b) 8 for Hindi only. 44% of students opted for Sanskrit only. 32% of
(c) 6 (d) 11 total girls opted for both Sanskrit and Hindi. The number of boys
(e) 12 opting for Sanskrit only and that in both Hindi and Sanskrit are in
the ratio 2 : 1 respectively.
DIRECTIONS (Qs. 90-94) : The following questions are based
95. How many boys opted for combo (both Sanskrit and Hindi)
on the following pie chart. In the pie chart populations of 6 states
subjects ?
have been given. The table shows ratio of males and females.
(a) 30 (b) 15
Study the diagrams carefully and answer the questions.
(c) 21 (d) 32
Total Population = 1,50,00,000 (e) None of these
96. What is the ratio between the number of boys who opted
for Hindi only and that of girls who opted for combo
Gujarat
17% Rajasthan subjects?
18% (a) 15 : 16 (b) 10 : 7
(c) 10 : 9 (d) 11 : 12
Karnataka
19% (e) None of these
Uttar Pradesh
22% 97. How many girls opted for Sanskrit only ?
Maharashtra

% (a) 72 (b) 47
oa8 (c) 51 (d) 77
16%

G
(e) None of these
98. In how many different ways can the letters of the word
RUMOUR be arranged ?
State Male : Female
(a) 180 (b) 720
Rajasthan 1 : 7 (c) 30 (d) 90
Maharashtra 5 : 7 (e) None of these
Uttar Pradesh 6 : 5
Goa 5 : 7 DIRECTIONS (Qs. 99-100) : Read the following information to
Karnataka 7 : 8 answer these questions.
Gujarat 9 : 6 There are 9 red, 7 white and 4 black balls in an urn.
90. What is the total number of females in Gujarat and Rajasthan ? 99. Two balls are drawn at random from the urn. What will be
(a) 3482500 (b) 3382500 the probability that both the balls are red ?
(c) 3823500 (d) 3658200 18 9
(e) None of these (a) (b)
95 95
91. What is the respective ratio of populations of Goa and
Maharashtra ? 9 8
(c) (d)
(a) 1 : 2 (b) 2 : 1 10 95
(c) 2 : 3 (d) 3 : 2
5
(e) 4 :3 (e)
92. By what per cent is the population of Rajasthan more than 19
that of Maharashtra ? 100. If two balls are selected at ran-dom what is the probability
(a) 11.5% (b) 10.5% that one ball is white and the other ball is red ?
(c) 12.5% (d) 9% 91 63
(e) 16% (a) (b)
190 190
93. What is the average number of males in Karnataka and Goa?
(a) 815000 (b) 91500 5 4
(c) (d)
(c) 915000 (d) 925000 19 95
(e) 935000
71
94. By what per cent is the population of Goa less than that of (e)
190
Maharashtra ?
y
o
u
rs
m
Corporation Bank Specialist Officer Exam 2014 409

a
h
ENGLISH LANGUAGE

b
Growing integration of our economy with the global

o
economy has increased the susceptibility of the economy to the

o
b
vagaries of global economic forces. Recent global recession

.w
DIRECTIONS (Qs. 101-115): Read the following passage
carefully and answer the questions given below. Certain words/ adversely affected Indian economy and hence the employment

o
scenario in the country as well.

rd
phrases have been given in bold to help you locate them while
101. Which of the following statement(s) is/are true in the context

p
answering some of the questions.

re
of the given passage ?

s
A stated objective of economic policy planning is achievement of I. The stated objective of economic policy planning is

s
.c
high rates of growth of the economy and sustained improvement achievement of high rates of economic growth.

o
in the standards of living of people. A rapid growth in employment

m
II. The other objective is sustained improvement in the
opportunities for all sections of the society, associated with rising standards of living of people.
GDP growth is essential to realize this objective as also to achieve III. Economic policy planning emphasises on growth of
the goal of Inclusive Growth. marketing sector to compete with global trade.
Since India became part of the global economy and (a) Only I and III (b) Only II and III
underwent economic reforms in 1991, its economy grew at a faster (c) Only I and II (d) Only I
rate of nearly 9 per cent per annum till 2010. In the process, India (e) All I, II and III
has become the fourth largest economy in the world. In the last 102. Which factor(s) is/are essential to realise the objective of
two decades, a significant proportion of the population across economic policy planning and achieve the goal of inclusive
the country has reaped the benefits of this economic growth. growth ?
They have become the part of global economy and market, and I. A rapid growth in employment opportunities.
their lives have transformed into one of global citizens with all the II. Rising GDP growth
comforts and luxury in life. III. Increase in foreign trade
Apart from this burgeoning middle class in the country, the (a) Only I and II (b) Only I and III
economic growth seemed to have touched the lives of the poor (c) Only II and III (d) Only III
also. According to the National Sample Survey results, people (e) All I, II and III
living below poverty line have dramatically come down during 103. Which of the following statements is incorrect in the context
the post economic reform era. Many economists question this of the given passage ?
dramatic change in poverty level. The fruits of economic growth (a) India started economic reforms in 1991.
have not benefited everyone uniformly. Some are left behind and (b) India has become the third largest economy in the
some others are not touched by the benefits of economic growth. world.
It is proved globally that the so-called trickle down effect does (c) Modern economy is technology driven, and not
not work in all the societies and India is no exception to this. labour-intensive.
There are various reasons for this uneven development in the (d) Unemployment among females increased by one per
society. cent for urban and rural females.
Modern economy is technology driven and not labour- (e) Uneven development in the society is evident in
intensive. High volume of high quality goods and services are modern economy.
produced with fewer labour hands. In short, the modern economy 104. Growing integration of our economy with the global
is not generating much employment and sometimes it displaces economy has increased the susceptibility of the
and replaces labour with machines and tools . During this period, economy____ .
the unemployment rate remained almost same for rural males and (a) to the vagaries of global economic forces
decreased by just one percentage for urban male. On the other (b) to the complexities of domestic forces
hand, unemployment among females increased by one percentage (c) to black marketing and hoarding
for urban and rural females. During this period, the unemployment (d) to rising prices of essential commodities
rate remained almost same for rural males and decreased by just (e) None of these
one percentage for urban male. On the other hand, unemployment 105. According to the passage, what report do many economists
among females increased by one percentage for urban and rural question ?
females economic development process. Such groups need (a) Dramatic change in poverty level as shown by National
government intervention to ensure that they are not left behind Sample Survey
in the development process and deprived of the benefits because (b) Growth of economy at nearly 9% per annum
they do not have the capacity to be part of the global economy. (c) uneven development in the society
The government needs to develop safety nets for such groups (d) Decreasing inflation
and try to mainstream them in the development process. They (e) None of these
need welfare measures in the form of poverty alleviation 106. What should be the best title of the passage in your opinion?
programmes to ensure that they survive if not prosper in this era (a) Growing India
of economic reform. Further, the poor are not a homogeneous
(b) Economic reforms and uneven development
population and their capacity to survive the economic reform
varied from one group of poor to another. Especially, those who (c) Economic policy planning
are below the poverty line or the poorest among the poor need (d) Technologically driven modern economy
more government help. (e) None of these
y
o
u
rs
m
410 Corporation Bank Specialist Officer Exam 2014

a
h
b
107. Which of the following statement(s) is/are incorrect in the 116. Since most of the (a)/ employees were in (b)/ favour of the

o
context of the passage ? merger (c)/ it was a success. (d)/ No error (e)

o
b
(a) The unemployment rate remained almost same for rural 117. The Reserve Bank of India are (a)/ responsible for ensuring

.w
males during last two decades. (b)/ that the banking sector (c)/ functions effectively.

o
(b) The government needs to develop safety nets for (d)/ No error (e)

rd
deprived section of society

p
118. There is a shortage (a)/ for qualified staff (b)/ in many

re
(c) The poor are a homogeneous population software (c)/ companies in India. (d)/ No error (e)

s
(d) Uneven development is outcome of modem economy

s
119. According to the survey (a)/ more of forty per cent (b)/ of

.c
(e) None of these Indians do not (c)/ have access to banks. (d)/ No error (e)

o
m
108. High volume of high quality goods and services are 120. To improve its efficiency (a)/ all income tax officers (b)/ will
produced with be provided with (c)/ laptops by next year. (d)/ No error (e)
(a) larger labour hands 121. He was not a (a)/ good manager because (b)/ he did not
(b) fewer labour hands i.e. technology know (c)/ what to react in a crisis. (d)/ No error (e)
(c) imported materials 122. Ram will be suspended (a)/ because he opened an account
(d) domestic materials (b)/ without obtaining none (c)/ of the necessary
(e) None of these documents. (d)/ No error (e)
DIRECTIONS (Qs. 109 - 112) : In the following questions, out of 123. Mr. Bajaj has resigned (a)/ from the post of Director (b)/
the five alternatives, choose the one which is most similar in which he holds (c)/ for over ten years. (d)/ No error (e)
meaning to the given word as used in the passage. 124. Inspite of living (a)/ in Kerala for two years (b)/ he yet does
109. Sustained (c)/ not speak Malayalam. (d)/ No error (e)
(a) interrupted (b) uninterrupted 125. Mr. Patil approached (a)/the bank for (b)/ a loan to finance
(c) encouraged (d) unexpected (c)/ his son college education. (d)/ No error (e)
(e) excepttional DIRECTIONS (Qs. 126-130) : Rearrange the following six
110. Realize sentences (A), (B), (C), (D), (E) and (F) in the proper sequence
(a) achieve (b) sell to form a meaningful paragraph; then answer the questions
(c) convert (d) reanimate given below them.
(e) reappear (A) He then hid nearby to watch and see who would remove it.
111. Burgeon (B) He saw a purse full of gold lying in the middle of the road. It
(a) develop slowly (b) run fast was the King's reward to the person who did something
(c) develop rapidly (d) cheat people about the problem.
(e) bargain (C) The king was tired of his subjects only complaining but do-
112. Dramatically ing nothing to solve their problems.
(a) expectedly (b) playing drama (D) A youth on his way to market saw the stone, put down his
(c) change gradually (d) unexpectedly produce and rolled the stone to the side of the road.
(e) None of these (E) Many people passed by but dodged their duty of moving
the stone instead blaming the king for not keeping the
DIRECTIONS (Qs. 113-115) : In the following questions, choose highways clear.
the word which is most opposite in meaning to the given word (F) One day he placed a heavy stone in the middle of the road.
as used in the passage. 126. Which of the following is the SECOND sentence after
113. Ensure rearrangement ?
(a) assure (b) ensue (a) B (b) C
(c) enscone (d) involve (c) D (d) E
(e) jeopardise (e) F
127. Which of the following is the SIXTH (Last) sentence after
114. Susceptibility
rearrangement ?
(a) vulnerability (b) unchangability (a) A (b) B
(c) adaptability (d) sensibility (c) C (d) D
(e) insensitiveness (e) E
115. Adversely 128. Which of the following is the FIRST sentence after
(a) unfavourably (b) favourably rearrangement ?
(c) certainly (d) cruelly (a) A (b) B
(e) upleasantly (c) C (d) D
(e) F
DIRECTIONS (Qs. 116-125) : Read each sentence to find out 129. Which of the following is the THIRD sentence after
whether there is any error in it. The error, if any, will be in one rearrangement ?
part of the sentence. The number of that part is the answer. If (a) A (b) B
there is no error, the answer is (e). (Ignore the errors of (c) C (d) D
punctuation, if any). (e) E
y
o
u
rs
m
Corporation Bank Specialist Officer Exam 2014 411

a
h
b
130. Which of the following is the FIFTH sentence after 137. The criterion for ____ a player should be his recent

o
rearrangement ? performance; but unfortunately, the journalists are ____ to

o
b
(a) B (b) C be carried away by earlier successes.

.w
(c) D (d) E (a) condemning, satisfying

o
rd
(e) F (b) judging, prone

p
(c) revealing, reluctant

re
DIRECTIONS (Qs. 131-135) : Which of the phrases (a), (b), (c)
(d) eager, acclaiming

s
and (d) given below should replace the phrase given in bold in

s
.c
the following sentence to make the sentence grammatically correct (e) criticising, clean

o
? If the sentence is correct as it is and no correction is required, 138. For the last half century he ______ himself to public affairs

m
mark (e) as the answer. ______ taking a holiday.
131. They didn't pay any heed to their superior's instructions : (a) by, committed (b) after, offered
I did neither. (c) devoted, without (d) sacrified, after
(a) I either did (b) Either 1 did not (e) prepared, before
(c) Neither didn't I (d) Nor did I 139. You will see signs of ____ everywhere, which speak well
(e) No correction required for the ____ of these people.
132. Every poet gives voice to his anger and his compassion (a) decoration, senses
through his poems. (b) clear, debris
(a) voiced to give (b) gives voice for (c) beauty, careful
(c) gave voice against (d) voiced at giving (d) industry, prosperity
(e) No correction required (e) repairs, extravaganza
133. The police nabbed a notorious criminal who had been 140. The police arrested Ramesh on a ____ of theft but for lack of
terrorising builders and extorted money from them for the evidence ____ him.
past two years.
(a) crime, imprisoned
(a) was terrorising builders and extorted
(b) punished, complaint
(b) had not been terrorised builders and extorted
(c) left, condition
(c) had been terrorising builders and extorting
(d) tip, absconding
(d) had terrorised builders and extorted
(e) No correction required (e) charge, released
134. Managers frequently encounter situations where they need DIRECTIONS (Qs. 141-150): In the following passage there
to help others solved problems. are blanks, each of which has been numbered. These numbers
(a) to helping others solved are printed below the passage and against each, five words are
(b) help others solved suggested, one of which fits the blank appropriately: Find out
(c) to help others solve the appropriate word in each case.
(d) help others solving The advance of science brings (141) necessary changes in our
(e) No correction required general ideas. The advance of astronomical science, for example,
135. From among various alternatives we should choose the one (142) entirely changed our views regarding man's place in Nature.
which is viable and consumes less time and energy. Before Copernicus it was universally believed that the Universe
was quite a small affair; that the Earth was its centre: that the Sun
(a) is viable and consuming lesser
and the Moon (143) to give light to our world, and that they and
(b) is viability and consumes less
the stars were travelling round the Earth. (144) then there have
(c) being viable and consumes less been many changes in our ideas, until now we know that the
(d) has viable and consuming less Earth travels round the Sun, that the Sun is but one of the millions
(e) No correction required of stars which are scattered in space at distances of millions and
millions of miles ; that all the stars we see (145) only a single star
DIRECTIONS (Qs. 136-140) : In each of the following sentences
family, and that there are millions of other similar star-families
there are two blank spaces. Below each sentence there are five
swimming in space at almost inconceivable distances, but visible
pair of words denoted by numbers 1, 2, 3, 4 and 5. Find out which
through our telescopes as spiral nebulae. We can no longer think
pair of words can be filled up in the blanks in the sentence in the
of man or his home as in any way central, or as being anything
same sequence to make the sentence meaningfully complete.
but very insignificant compared with the Universe as a whole.
136. He objected to the proposal because it was founded on a The advance of biological science has an equally great effect.
____ principle and also was ____ at time. Before the 19th century, it used to be (146) that man was created
(a) faulty, desirable only a few thousand years ago in the same form that he is today,
(b) imerative, reasonable (147) that all other animals and plants existed for his benefit. The
(c) wrong, inconvenient discoveries of geology proved the world to be enormously old,
(d) sound, acceptable and Darwin and his followers showed that man had (148) from an
(e) unconforming, deplorable animal ancestor. Today we know that life has existed on earth for
y
o
u
rs
m
412 Corporation Bank Specialist Officer Exam 2014

a
h
b
over a thousand million years; during that time it had slowly 145. (a) make up (b) making up

o
changed or evolved into many different forms; that man evolved (c) made of (d) forming

o
b
out of an ape-like creature and came on the scene very late in the (e) formed

.w
world's history; that he has changed in various ways during his 146. (a) supposing (b) supposed

o
evolution; and that there is no reason (149) further change and

rd
(c) developing (d) knowing
evolution should not take place. Furthermore, the rest of life does

p
(e) suppose

re
not exist to serve man; man simply happens to be the (150)

s
successful living creature and has been able to use many plants 147. (a) and (b) if

s
.c
and animals for his own ends. (c) only (d) for

o
m
141. (a) about (b) of (e) all
(c) with (d) from 148. (a) evolve (b) evolved
(e) for (c) developing (d) made
142. (a) have (b) has (e) formed
(c) is (d) are 149. (a) that (b) what
(e) has been (c) why (d) when
143. (a) exist (b) exists (e) how
(c) existed (d) existing 150. (a) much (b) more
(e) will (c) most (d) better
144. (a) About (b) Till (e) worst
(c) Before (d) Since
(e) By
y
o
u
rs
m
Corporation Bank Specialist Officer Exam 2014 413

a
h
b
o
o
b
.w
o
1-6 : Accounts 12. (b) Conclusions :

rd
Physics E Psychology I. False

p
re
B D II. True

s
So only conclusion II follows.

s
.c
Mathematics Chemistry 13. (a) Conclusions :

o
F H I. True, Third premise

m
II. False
A G So only conclusion I follows.
Biology English
C 14. (b) Noons
Hindi
1. (b) H teaches Chemistry.
2. (d) D teaches Psychology. B is second to the right of D. Day Night Evening
3. (a) A teaches Biology and B teaches Physics. F, who
teaches Mathematics is exactly between A and B.
4. (c) E teaches Accounts. OR
5. (b) The person who teaches Hindi is C and C is an
immediate neighbour of A and G. Noons
Immediate neighbours of F are A and B. A teaches
Biology. Night
E is sitting exactly between B and D, who teaches
Accounts. Day
H teaches Chemistry and H is second to the left of E.
6. (e) Except in the pair BA, in all others the first person Evening
is second to the right of the second person. In BA, the
first person is second to the left of the second person.
Row- 1 Conclusions :
7-11 :
I. False
RIGHT

II. True
LEFT

So only conclusion II follows.


F C E D A B 15. (d)
T R P U S Q
Trousers
RIGHT
LEFT

Jackets Caps Shirts

Row- 2
7. (c) F and B are sitting at the extreme ends of Row-1. OR
8. (a) S is third to the right of R.
9. (e) Except in EU, in all others one person is just Trousers
opposite to the other person.
10. (b) A is second to the left of E. Jackets Shirts Caps
11. (d)
RIGHT

LEFT

Conclusions :
F C E D A B I. False
F E D C B A II. False
So position of F remains unchanged. So none of the conclusion follows.
12-13 : 16-20 : C F G D H E
RIGHT
LEFT

Gems
Stones Stones Gems
16. (c) G is third to the left of E.
Diamonds OR Diamonds Diamonds 17. (b) C and E are standing at the extreme ends of the line.
18. (c) G is standing second to the right of C.
19. (b) Except in GE, in all others there is only one person
between the two.
y
o
u
rs
m
414 Corporation Bank Specialist Officer Exam 2014

a
h
36-40 :

b
20. (a) C F G D H E

o
o
CRITERIA

b
CANDIDATE

.w
(i) (ii) or B (iii) (iv) or A (v) Ans

o
rd
C D E F G H Suresh ü ü – ü ü – ü 2

p
21. (e) All 3 courses of action should be followed because Sudha – –

re
ü ü ü ü ü 4
sending relief team to the effected area, pumping out

s
Divya û ü – ü ü – ü 1

s
water from the road and advising general public to

.c
stay indoor all would help to tackle the problem. Navin ü ü – ü ü – ü 2

o
m
22. (e) I and II course of action should be followed because –
Varun ü ü ü – ü ü 5
course of action III opposes the need for essential
commodities. 36. (b) Suresh Mehta satisfies all the conditions (i), (ii), (iii),
23. (b) Only course of action II can be persued because (iv) and (v). Therefore, he can be selected.
suspending principal is not a solution of any 37. (d) Sudha Gopalan satisfies the conditions (i), (B), (iii),
problem and suspending midday meal would creat (iv) and (v). Therefore, her case would be referred to
further problems. Vice-president -Marketing.
24. (d) All three courses of action can be followed to 38. (a) Divya Kohli does not satisfy condition (i). Therefore,
overcome malaria problem.
she cannot be selected.
25. (e) Only II course of action can be followed because
school management cannot terminate all teachers 39. (b) Navin Marathe satisfies all the conditions (i), (ii), (iii),
and cannot approach the government for guidelines (iv) and (v). Therefore, he can be selected.
because management has to from its own guidelines. 40. (e) Varun Malhotra satisfies the conditions (i), (ii), (iii),
26. (a) Government decision to construct super highway (A) and (v). Therefore, his case should be referred to
shows that Government has sufficient resources to GM-Marketing.
construct it. 41-45 :
27. (d) C O N Q U E R it is rush hour traffic ® sa le do mi ru
–1 +1 –1
M N B R Q D T go to school ® be no pa
Similarly,
one hour to go ® mi f i pa be
S T E A M E R
–1 +1 –1
rush to one ® fi be sa
D S R B Q D L
traffic is fine ® ga ru do
28. (d)
1 4 10 21 19 20 9 14 7 Codes are
it Þ le go Þ pa
A D J U S T I N G is Þ ru or do to Þ be
rush Þ sa school Þ no
hour Þ mi one Þ fi
N traffic Þ ru or do fine Þ ga
29. (d) 41. (c) school Þ no; hour Þ mi; go Þ pa; fine = ga.
30 m

30 m

W E 42. (d) mi Þ hour; fi Þ one; le Þ it,


43. (a) sa Þ rush
50 m S 44. (e) traffic is Þ ru do; one Þ fi; hour Þ mi
30. (e) All 4 are possible effects because heavy rainfall after The code for ‘for’ may be ’ye’ or ‘so’.
cyclonic storm is obvious and next 3 statements are 45. (b) do Þ is or traffic
the natural after affects of cyclonic storm. 46-50 : Days Flight
31. (a) Option (a) is the only cause because unisafe
structure would cause deserted look in the temple. Monday Emirates
32. (e) Obviously option (e) is an assumption. Tuesday Jet Airways
33. (b) b w x y z a, d s t u v c , f o p q r e Wednesday Delta
34. (c) Statement (3) would weaken the government’s
Thursday British Airways
argument because it negates the importance of
computerisation of bank branches. Friday Air India
35. (c) law Q Saturday Quantas
-in-
Son Sunday Lufthansa

M N P 46. (c) Jet Airways flies on Tuesday.


(+) (–) (–) 47. (d) Three flights fly between Lufthansa and Delta
y
o
u
rs
m
Corporation Bank Specialist Officer Exam 2014 415

a
h
48. (a) Air India flies on Friday

b
Length of train

o
49. (e) Quantas will fly on Friday 58. (c) Speed of train =

o
Time taken in crossing

b
50. (d) There are two flights between Emirates and British

.w
Airways. 320 18

o
51. (e) The pattern is : = = 20 m/sec = 20 ´ = 72 kmph

rd
2×3 =6 16 5

p
6 × 2.5 = 15 Total period of stoppage = 5 × 18

re
15 × 2 = 30

s
3

s
30 × 1.5 = 45 = 90 minutes = hours

.c
2

o
45 × 1 = 45 ¹ 43.5 Total time taken in covering a distance of 576 km.

m
45 × 0.5 = 22.5 æ 576 3 ö
So 43.5 is the wrong term. =ç + ÷ hours = 8 + 3 = 9 1 hours
è 72 2 ø 2 2
52. (c) The pattern is :
59. (a) From statement I.
950 – 661 =289= 172
661 – 436 = 225 = 152 P´ R2
Difference =
436 – 269 = 167 ¹ 132 10000
\ 436 – 267 = 169 = 132 P×12×12
267 – 146 = 121 = 112 Þ 360 =
10000
146 – 65 = 81 = 92
So 269 is the wrong term. 360 × 10000
Þ P= = ` 25000
53. (e) The pattern is : 12×12
6.5 + 5.3 = 11.8 From statement II,
11.8 + 2 × 5.3 = 11.8 + 10.6 = 22.4 Interest = 2P – P = ` P = Principal
22.4 + 3 × 5.3 = 22.4 + 15.9 = 38.3 S.I. × 100 P × 100
38.3 + 4 × 5.3 = 38.3 + 21.2 = 59.5 Principal = = =P
59.5 + 5 × 5.3 = 59.5 + 26.5 = 86 Time× Rate 10×10
From statement III,
¹ 87.3
éæ Rate öTime ù
86 + 6 × 5.3 = 86 + 31.8 = 117.8 C.I. = P êç1+ – 1ú
÷
54. (e) The pattern is : êëè 100 ø úû
1×3–1=2
2×3–2=4 éæ 12 ö2 ù
4×3–3=9 Þ 6360 = P êç 1+ ÷ – 1ú
9 × 3 – 4 = 23 êëè 100 ø úû
23 × 3 – 5 = 69 –5 = 64 ¹ 69 éæ 28 ö2 ù
64 × 3 – 6 = 192 – 6 = 186 Þ 6360 = P êç ÷ – 1ú
25
55. (e) The pattern is : ëêè ø úû
250 – 11 = 239
239 – (11 × 2 + 1) = 239 – 23 = 216 éæ 784 – 625 ö ù = P×159
Þ 6360 = P êç ÷ú
216 – (11 × 3 + 2) = 216 –35 = 181 ëè 625 ø û 625
181 – (11 × 4 + 3) = 181 – 47 = 134 ¹ 136 6360 × 625
Þ P= = ` 25000
134 – (11 × 5 + 4) = 134 – 59 = 75 159
75 – (11 × 6 + 5) = 75 – 71 = 4 So information given in statement I or III is sufficient
Principal ´ Time ´ Rate to answer the question
56. (b) S.I. = Note : You need not calculate during exam. You are
100
required to examine whether requirements of a
4500 ´ 2 ´ 12 5600 ´ 2 ´ 9 formula are satisfied or not.
\ Difference = –
60. (b) From statement I,
100 100
= 1080 – 1008 = ` 72 If Length = x metre and breadth = y metre, then
57. (a) First number = x 1×2
Second number = y y= ( x + y)
4
50 3 Þ 2y = x + y Þ y = x
\ x´ = y´ \ x2 = 144
100 4
x 3 Þ x = 144 = 12 metre
Þ = y´ Area of field = (16)2 – (12)2 = 256 – 144 = 112 sq. metre
2 4
From statements II and III,
x 3 3 Length = 3x metre
Þ = ´2 =
y 4 2 Breadth = 2x metre
y
o
u
rs
m
416 Corporation Bank Specialist Officer Exam 2014

a
h
\ 3x × 2x = 216

b
15000 ´ 100

o
Third quarter Þ = 12500

o
216 120
= 36

b
Þ x2 =

.w
3´ 2 12500 – 10000 250000
Þ x=6 = ´ 100 = = 25

o
10000 10000

rd
\ Length =18 metre, Breadth = 12 metre 68. (e) Passengers of airline B who travelled to Bangaluru.

p
Area of the field with boundary First quarter Þ 10000

re
= (18 + 2 × 2) (12 + 2 × 2)

s
10000 ´130

s
= 22 × 16 = 352 sq. metre Second quarter Þ

.c
= 13000
\ Area of boundary = 352 –216 = 136 sq. m. 100

o
Passengers of airline B who travelled to Kolkata

m
61. (d) From statement I,
H + G + C = 65 × 3 = 195 ...(i) First quarter Þ 10000
From statement II, Second quarter Þ 14000
G=H+6 20000
From statement III, \ Required percentage = ´ 100 » 74
27000
G–C=C–H 4
Þ 2C = G + H
Þ 2C = H + 6 + H = 2H + 6 (?) 5 9
69. (a) =
36 1
Þ C=H+3
\ H + G + C = 195 (?) 5
Þ H + H + 6 + H + 3 = 195 4 1
Þ 3H = 195 – 9 = 186 (?) 5 ´ (?) 5 = 9 ´ 36
Þ
186
Þ H= = 62 4 1
+
3
Þ (?) 5 5 = 324
62. (a) From statements I and II, Þ ? = 324
If the length of the train be x metre, then 70. (b) I. 4x2 – 32x + 63 = 0
x x + 250 Þ 4x2 – 14x – 18x + 63 = 0
Speed of train = = Þ 2x (2x – 7) – 9 (2x – 7) = 0
15 27
Þ (2x – 7) (2x – 9) = 0
x x + 250 7 9
Þ =
5 9 Þ x= or
2 2
Þ 9x = 5x + 1250 II. 2y2 – 11y + 15 = 0
Þ 4x = 1250 Þ 2y2 – 6y – 5y + 15 = 0
1250 Þ 2y (y – 3) – 5 (y – 3) = 0
Þ x= = 312.5 m/sec. Þ (y – 3) (2y – 5) = 0
4
63. (d) From statements I and II, 5
Þ y = 3 or
C.P. of article = 252 – 52 = ` 200 2
Discount = 280 – 252 = ` 28 Clearly, x > y
Marked price = ` 280 1
´3
x 71. (b) I. x3 = (216) 3 = 216
\ 280 ´ = 28 Þ x = 3 216 = 6
100
Þ x = 10% II. 6y2 = 150
Statements I and III will also give same result. 150
64. (a) Passengers of airline A Þ y2 = = 25
6
= (15 +10 + 20 + 10 + 15 + 10) thousands Þ y = 25 = ±5
= 80 thousands
Passengers of airline C Clearly, x > y
= (15 + 15 +15 + 10 + 20 + 10) thousands 72. (a) I. 12x2 + 17x + 6 = 0
= 85 thousands Þ 12x2 + 9x + 8x + 6 = 0
Difference = 85 – 80 = 5 thousands Þ 3x (4x + 3) + 2 (4x + 3) = 0
65. (b) Passengers of airline C who travelled to Delhi and Þ (4x + 3) (3x + 2) = 0
Kolkata = 30 thousands 3 2
Þ x= – or –
30 4 3
Required percentage = × 100 » 35 II. 6y2 + 5y + 1 = 0
85 Þ 6y2 + 2y + 3y + 1 = 0
66. (c) Required ratio = (15 + 10) : (15 + 10) = 25 : 25 = 1 : 1 Þ 2y (3y + 1) + 1 (3y + 1) = 0
67. (d) Passengers of airline A who travelled to Chennai Þ (3y + 1) (2y + 1) = 0
First quarter Þ 10000 1 1
150 Þ y=– or –
Second quarter Þ 10000 ´ = 15000 3 2
100 Clearly, x < y
y
o
u
rs
m
Corporation Bank Specialist Officer Exam 2014 417

a
h
73. (c) I. 20x2 + 9x + 1 = 0

b
79. (d) Share of profit = Geeta : Shweta : Sita

o
Þ 20x2 + 5x + 4x + 1 = 0 = 4200 : 8400 : 5400 = 7 : 14 : 9

o
Þ 5x (4x + 1) + 1 (4x + 1) = 0

b
.w
Þ (4x + 1) (5x + 1) = 0 9
Sita's share = × 24000

o
(7 + 14 + 9)

rd
1 1
Þx=– or –

p
4 5 9

re
II. 30y2 + 11y + 1 = 0 = × 24000 = ` 7200

s
30

s
Þ 30y2 + 6y + 5y + 1 = 0

.c
Þ 6y (5y + 1) + 1 (5y + 1) = 0 Remaining amount after investing 32%

o
= 7200 × (100 – 32)%

m
Þ (5y + 1) (6y + 1) = 0
1 1 7200 ´ 68
Þ y=– or – = = ` 4896
5 6 100
Clearly, x £ y 80. (a) Percentage of females in city C = 100 – 52 – 13 = 35
74. (d) I. x2 + 17x + 72 = 0
52
Þ x2 + 8x + 9x + 72 = 0 Adult males Þ 71200000 × = 37024000
Þ x (x + 8) + 9 (x + 8) = 0 100
Þ (x + 9) (x + 8) = 0
71200000 ´ 35
Þ x = – 9 or –8 Adult females Þ = 24920000
II. y2 + 19y + 90 = 0 100
Þ y2 + 10y + 9y + 90 = 0 Percentage of females in city D = 40
Þ y (y + 10) + 9 (y + 10) = 0
Þ (y + 9) (y + 10) = 0 93500000×49
Adult males Þ = 45815000
Þ y = –9 or –10 100
Clearly, x > y
93500000×40
75 (b) Total height of 16 students= 16 × 142 cm = 2272 cm Adult females Þ = 37400000
Let height of teacher be x. 100
2272 + x Difference = 37024000 + 45815000 – 24920000 –37400000
= 143 = 20519000
17
2272 + x = 2431 20519000
Required percentage = ´ 100 » 33
x = 2431 – 2272 = 159 62320000
Height of teacher is 159 cm
76. (a) Let present ages of Mahesh and Ajay is 3x and 2x 16
81. (b) Required percentage = ´100 » 39
respectively. 41
According to question, 82. (a) Females :
3 x + 8 11 68300000 ´ 43
=
2x + 8 8 City A Þ = 29369000
100
Þ 24x + 64 = 22x + 88
City D Þ 37400000
Þ 2x = 24
Þ x = 12 Difference = 37400000 – 29369000 = 8031000
Present age of Ajay is 2 × 12 = 24 years 83. (b) Adult population :
24 42500000 ´ 86
Present age of Mahesh’s son = = 12 years City B Þ = 36550000
2 100
77. (b) Part of tank filled by all three pipes in 1 minute
81200000 ´ 84
1 1 1 6+ 4 -3 7 City F Þ = 68208000
= + - = = 100
30 45 60 180 180 Total population = 36550000 + 68208000 = 104758000
180 84. (a) Required ratio = 93500000 : 54500000
\ Time taken = minutes = 935 : 545 = 187 : 109
7
180 3 4´3
= = hour 85. (a) ? » × 952 – 129
7 ´ 60 7 12
100 » 952 – 129 » 823
78. (c) C.P. of article = 5220 × (100 - 42) 8450 ´ 105 3
86. (b) ? » - 5006 × + 10
5220 ´100 100 700
= = ` 9000 » 8872 – 21 + 10 » 8861
58
87. (b) 103 × 1003 + 109 = 10? + 10?
9000 ´142 Þ 109 + 109= 10?+ 10? Þ ? = 9
\ Required S.P. = = ` 12780
100
y
o
u
rs
m
418 Corporation Bank Specialist Officer Exam 2014

a
h
(d) ? » 21 + 3.7 × 3 » 21 + 11.1 » 32.1

b
88.
6!

o
89. (a) 23 + 9 – ? = 23 \ Number of arrangements =

o
2!2!

b
Þ ?=9

.w
6×5×4×3× 2×1

o
17 6 =
´

rd
90. (b) Females in Gujarat = 15000000 × = 1020000 2× 2
100 15

p
= 180

re
18 7 99. (a) Total possible outcomes

s
´ = 2362500

s
Females in Rajasthan = 15000000 × = Selection of 2 balls out of (9 + 7 + 4 = 20) balls

.c
100 8

o
20× 19
Total females = 1020000 + 2362500 = 3382500

m
= 20C2 = = 190
91. (a) Required ratio = 8 : 16 = 1 : 2 1×2
Favourable outcomes = Selection of 2 balls out of 9 red
18 - 16 balls
92. (c) Required percentage = ´ 100 = 12.5%
16 9×8
= 9C2 = = 36
1×2
19 7
93. (c) Males in Karnataka = 15000000 × ´ =1330000 36 18
100 15 \ Required probability = =
190 95
8 5 100. (b) Total possible outcomes = 190
Males in Goa = 15000000 × ´ = 500000
100 12 Favourable outcomes = Selection of 1 ball out of 9 red
balls and 1 ball from 7 white balls
1330000 + 500000
Average = = 915000 = 9C1 × 7C1 = 9 × 7 = 63
2
63
\ Required probability =
16 -8 190
94. (d) Percentage decrease = × 100 = 50% 101. (c) It is mentioned in the first paragraph.
16
102. (a) It is mentioned in the first paragraph.
95 – 97 :
103. (b) India has become the fourth largest economy as
3 mentioned in the second paragraph.
Boys Þ 175× = 75 104. (a) It is mentioned in the last paragraph.
7
105. (a) It is mentioned in the third paragraph.
4 106. (b) The passage talks about the status of economic
Girls Þ 175 × = 100
7 development, poverty and unemployemnt as issues.
107. (c) It is mentioned in the second last paragrpah where it
75×40 saya that the poor are not a homogeneous population.
Boys who read only Hindi = = 30
100 108. (b) It is mentioned in the fourth paragraph.
109. (d) 110. (e) 111. (c) 112. (d)
175× 44
Students who read only Sanskrit = = 77 113. (b) 114. (e) 115. (a) 116. (a)
100 117. (a) The Reserve Bank of India is.
100×32 118. (b) Of qualified staff.
Girls reading both Sanskrit and Hindi = = 32 119. (b) More than forty percent.
100 120. (a) To improve their efficiency.
2 121. (d) How to react in a crisis.
Boys who read Sanskrit only = × 45 = 30 122. (c) Without obtaining any.
3
123. (c) Which he held.
Boys who read the combo subjects 124. (c)
= 75 – (30 + 30) = 15 125. (d) His son's college education.
Girls who read Sanskrit only = 77 – 30 = 47 126. (e) 127. (b) 128. (c) 129. (a)
Girls who read only Hindi = 100 – 32 – 47 = 21 130. (c) 131. (d) 132. (e) 133. (a)
95. (b) Boys who read both subjects (combo) = 15 134. (c) 135. (e) 136. (b) 137. (a)
96. (a) Required ratio = 30 : 32 = 15 : 16 138. (c) 139. (d) 140. (e) 141. (a)
97. (b) Girls who read only Sanskrit = 47 142. (b) 143. (c) 144. (d) 145. (a)
98. (a) The word RUMOUR consists of 6 letters in which each 146. (b) 147. (a) 148. (b) 149. (c)
of R and U comes twice. 150. (e)
y
o
u
rs
m
a
h
b
o
IBPS CWE RRB OFFICER SCALE- I

o
b
.w
o
EXAM 2014 Based on Memory

rd
p
re
s
s
.c
o
m
REASONING ABILITY 5. Which competition is exactly preceded by the street play
competition?
1. In a certain code language ‘HORSE’ is written as 71417184, (a) Rock band competition
then the word ‘MONKEY’ is coded as: (b) Group song competition
(a) 11141216425 (b) 12141310424 (c) Debate competition (d) Fash-p competition
(c) 12151411325 (d) 12151210424 (e) None of these
6. Fash-p competition follows which competition?
(e) None of these
(a) Debate competition (b) Street play competition
DIRECTIONS (Qs. 2-6) : Read the following information and (c) Rock band competition (d) All of the above
choose the right alternative in the questions that follow. (e) None of these
During the cultural week of an institute six competitions DIRECTIONS (Qs. 7-11) : Each of the questions below starts
were conducted. The cultural week was inaugurated in the with a few statements, followed by four conclusions numbered 1,
morning of 19th October, Wednesday and continued till 26th 2, 3 and 4. You have to consider every given statement as true,
October. In the span of 8 days six competitions namely debate, even if it does not conform to the accepted facts. Read the
folk dance, fash-p, street play, rock band and group song, were conclusions carefully and then decide which of the conclusion(s)
organized along with various other cultural programs. The logically follow(s) from the given statements, disregarding
information available from the institute is commonly known facts.
(i) Only one competition was held in a day
(ii) Rock band competition was not conducted on the 7. Statements :
closing day I. Some boys are scholars
(iii) Fash-p was conducted on the day prior to debate II. Some teachers are boys
competition III. All scholars are observers
(iv) Group song competition was conducted neither on Conclusions:
Wednesday nor on Saturday 1. Some scholars are boys
(v) None of the competition was conducted on Thursday 2. Some scholars are not boys
and Sunday 3. Some observers are boys
(vi) Street play competition was held on Monday 4. Some teachers are scholars
(vii) There was gap of two days between debate competition (a) 1 and 3 follow
and group song competition (b) 1, 3 and 4 follow
2. The cultural week started with which competition? (c) Either 1 or 2 and 3 follow
(a) Fash-p competition (b) Debate competition (d) All of the above
(c) Street play competition (d) Rock band competition (e) None of these
(e) None of these 8. Statements:
3. How many days gap is there between rock band competition I. All teachers are professors
and group song competition? II. All professors are researchers
(a) Two (b) Three III. All researchers are consultants
(c) Four (d) Five Conclusions:
(e) None of these 1. Some consultants are teachers
4. Which pair of competition was conducted on Wednesday? 2. All professors are consultants
(a) Rock band competition and debate competition 3. Some researchers are teachers
(b) Debate competition and fash-p competition 4. All professors are teachers
(c) Rock band competition and folk dance competition (a) Only 1 and 2 follow (b) Only 1 and 3 follow
(d) All of the above (c) Either 1 or 4 follow (d) All of the above
(e) None of these (e) None of these
y
o
u
rs
m
420 IBPS CWE RRB Officer Scale-I Exam 2014

a
h
13. A is B’s Mother and D is B’s mother.

b
9. Statements:

o
I. Some drivers are technicians T is E’s daughter whose wife is B. how are E and D

o
b
related?
II. All technicians are engineers

.w
(a) Brother (b) Brother-in-law
III. Some engineers are lecturers

o
(c) Uncle (d) Father-in-law

rd
Conclusions: (e) None of these

p
re
1. Some technicians are lectures
DIRECTIONS (Qs. 14-18): Study the following information

s
2. Some lecturers are drivers

s
.c
3. All engineers are technicians carefully and answer the questions given below:.

o
A, B, C, D, E, F, G and H are eight employees of an organization

m
4. Some engineers are drivers
(a) Only 3 follows working in three departments, viz Personnel, Administration
and Marketing with not more than three of them in any
(b) Only 4 follows.
department. Each of them has a different choice of sports from
(c) Only 3 and 4 follows
football, cricket, volleyball, badminton, lawn tennis, basketball,
(d) All of the above hockey and table tennis, not necessarily in the same order.
(e) None of these
10. Statements: D works in Administration and does not like either football or
I. Some barbers are fashion designers cricket. F works in Personnel with only A, who likes table tennis.
II. No fashion designers are businessmen E and H do not work in the same department as D. C likes hockey
III. Some businessmen are traders an d does not work in Mar keting. G does not work in
Conclusions: Administration and does not like either cricket or badminton.
One of those who work in Administration likes football. The
1. No Fashion designers are traders
one who likes volleyball works in Personnel. None of those who
2. Some traders are not fashion designers work in Administration likes either badminton or lawn tennis.
3. Some fashion designers are traders H does not like cricket.
4. Some barbers are not businessmen 14. Which of the following groups of employees work in
(a) Either 1, 2 and 4 or 3, 2 and 4 follow Administration department?
(b) Either 1 and 4 or 3 and 4 follow (a) EGH (b) SAF
(c) Either 1 and 2 or 3 and 2 follow (c) BCD (d) BCD
(d) All of the above (e) Data inadequate
(e) None of thse 15. In which department does E work?
(a) Personnel (b) Marketing
11. Statements: (c) Administration (d) Data inadequate
I. Some boys are scholars (e) None of these
II. Some teachers are boys 16. Which of the following combinations of employee
III. All scholars are observers department favourite sport is correct?
(a) E - Administration - Cricket
Conclusions:
(b) F - Personnel - Lawn Tennis
1. Some scholars are boys (c) H - Marketing - Lawn Tennis
2. Some scholars are not boys (d) B - Administration - Table Tennis
3. Some observers are boys (e) None of these
4. Some teachers are scholars 17. What is E’s favourite sport?
(a) Cricket (b) Badminton
(a) 1 and 3 follow
(c) Basketball (d) Lawn Tennis
(b) 1, 3, and 4 follow (e) None of these
(c) Either 1 or 2 and 3 follow 18. What is G’s favourite sport?
(d) All of the above (a) Cricket (b) Badminton
(e) None of these (c) Basketball (d) Lawn Tennis
12. Immediately after leaving his house, Ratvik turned right (e) None of these
and walked for 40 m. Then he turned left and walked for DIRECTIONS (Qs. 19-23): Each of the questions below
20 mts. Then he again took a left turn and walked for 30 consists of a question and two statements numbered I and II
mts. There he met a friend and turned right to go to the given below it. You have to decide whether the data provided in
coffee shop 20 mts away. After having coffee, he walked the statements are sufficient to answer the question. Read both
back straight for 40 mts in the direction he had come from. the statements and give answer.
How far is he from his house?
(a) 20 m (b) 0 m (a) if the data in Statement I alone are sufficient to answer the
(c) 10 m (d) 40 m question, while the data in Statement II alone are not
(e) None of these sufficient to answer the question;
y
o
u
rs
m
IBPS CWE RRB Officer Scale-I Exam 2014 421

a
h
I b H, E a F, I g F

b
(b) if the data in Statement II alone are sufficient to answer the 26. Statements:

o
question, while the data in Statement I alone are not sufficient Conclusions: I. E a I

o
b
to answer the question; II. H d E

.w
(c) if the data in Statement I alone or in Statement II alone are 27. Statements: V g O, R g V, O b B

o
rd
sufficient to answer the question; Conclusions: I. R g R

p
(d) if the data in both the Statements I and II are not sufficient II. R a B

re
28. Statements: L a U, T g V, O b B.

s
to answer the question;

s
I. T a W

.c
(e) if the data in both the Statements I and II together are Conclusions:

o
necessary to answer the question. II. U g W

m
19. How is P related to Q’? DIRECTIONS (Qs. 29-33): Read the following information
Statements: carefully and answer the questions which follow.
I. Mother of P is daughter of R A, B, C, D, E, F, G and H study in different standards viz 1st,
II. R is sister of Q 2nd, 3rd, 4th, 5th, 6th, 7th and 8th. All of them are seated
around a circular table facing the centre.
20. Among five friends A, B, C, D and E who is the youngest? C sits third to right of H. H studies in std. 8. F sits second to
Statements: left of E. E is not an immediate neighbour of C or H. The one
I. B and D are older than C and E who studies in std. 1 is an immediate neighbour of E. Three
II. C is older than only E people sit between H and the student of std. 7.
H, C and also their immediate neighbours do not study in std.
21. Tower R is in which direction with respect to Q? 2. Only one person sits between the student of std. 2 and G.
Statements: Students of std. 3 and 4 are immediate neighbours of each other.
I. M is to the East of Q and to the North of R. C is neither in std. 4 nor in std. 3. Only one person sits between
A and the student of std 5. A does not study in std. 2 or 3. B
II. L is to the West of R and to the South of Q. does not study in std. 2.
22. How many children are there in the row of children facing 29. Which of the following is true regarding F ?
North? (a) F studies in std. 5
Statements: (b) D and G are immediate neighbours of F
I. Kavitha is twelth from the left end of the row and is (c) One person sits between F and the student of std. 4.
(d) None is true
seventh to the right of Nisha. (e) None of these
II. Nisha is twentieth from the right end of the row. 30. Who amongst the following studies in std. 3 ?
23. How many sons does ‘D’ have ? (a) C (b) F
Statements: (c) G (d) B
(e) None of these
I. R is the only sister of T and M who are sons of D.
31. How many persons sit between D and student of std. 4, when
II. T and M are only brothers of R who is daughter of D. counted from the left hand side of student of std. 4?
(a) None (b) One
DIRECTIONS (Qs. 24-28): In the following questions, the
(c) Two (d) Three
symbols b, g, y, a and d are used with the following meanings.. (e) None of these
P b Q means P is not smaller than Q. 32. Which of the following is true ?
P g Q means P is neither greater than nor smaller than Q. (a) Only one person is sitting between student of std 4
and std. 6.
P y Q means P is not greater than Q.
(b) D studies in std. 2.
P a Q means P is neither smaller than nor equal to Q. (c) The one studying in std. 3 is an immediate neighbour
P d Q means P is neither greater than nor equal to Q. of H
Now in each of the following questions, assuming the given (d) One person sits between H and the student of std.
statements to be ture, find which of the two conclusions I and II 2.
given below them is/are definitely true. Give answer, (e) None of these
(a) if only conclusion I is true; 33. Who amongst the following represent immediate
neighbours of G ?
(b) if only conclusion II is true;
(a) A, B
(c) if either I or II is true; (b) E and the student of std.2.
(d) if neither I nor II is true; and (c) H and the student of std. 6.
(e) if both I and II are true. (d) E, F
24. Statements: M b N, H y Q, B b M (e) None of these
Conclusions: I. H g M DIRECTIONS (Qs. 34-37): Read the following information
II. Q g N carefully and answer these questions.
25. Statements: C a B, L d S, S y C
In a certain coding system,
Conclusions: I. B a S
'816321' means 'the brown dog frightened the cat';
II. C a L
'64851' means 'the frightened cat ran away'.
y
o
u
rs
m
422 IBPS CWE RRB Officer Scale-I Exam 2014

a
h
b
'7621' means 'the cat was brown'; DIRECTIONS (Qs. 46-50): What should come in place of the

o
'341' means 'the dog ran'.

o
question mark (?) in the following questions?.

b
34. What is the code for 'the dog was frightened'?

.w
(a) 5438 (b) 8263 46. 25% of 84 × 24% of 85 = ?

o
(c) 8731 (d) 8560 (a) 424.2 (b) 488.4 (c) 482.8

rd
(e) None of these (d) 428.4 (e) None of these

p
re
35. What is the code for 'frightened' ? 47. 64 × 16 ¸ 256 = (4)(? – 3)

s
(a) 2 (b) 3

s
(a) 1 (b) 4 (c) 5

.c
(c) 6 (d) 8
(d) 3 (e) None of these

o
(e) 5

m
36. What is the code for 'away' ? 48. 25.05 × 123.95 + 388.999 × 15.001 =?
(a) 1 (b) 5
(c) 6 (d) 7 (a) 900 (b) 8950 (c) 8935
(e) 8 (d) 8975 (e) 8995
æ1 5 ö
37. What is the code of 'brown'? 49. 60% of ç ´ 2920 + ´ 2075 ÷ = ?
13 18
(a) 2 (b) 4 (a) 360è (b) 480ø (c) 520
(c) 6 (d) 8 (d) 660 (e) 320
(e) 3 50. 7365 + (5.4)2 + ? = 7437.16
(a) 1894 (b) 1681 (c) 1764
DIRECTIONS (Qs. 38-40): Read the following informations
(d) 2025 (e) None of these
carefully to answer the questions given below.
(i) ‘A + B’ means ‘A is the father of B’. DIRECTIONS (Qs. 51-55): A television manufacturing
company produces four models of televisions viz. TV 29", TV
(ii) ‘A – B’ means ‘A is the wife of B’.
25", TV 21", and TV 17". In the year 2006 the company generated
(iii) ‘A × B’ means ‘A is the brother of B’.
a revenue of Rs. 1 crore by selling 1000 televisions. The cost of
(iv) ‘A ¸ B’ means ‘A is the daughter of B’.
manufacturing, model–wise, is as given below:
38. If P ¸ R + S + Q, which of the following is true?
(a) P is the daughter of Q (b) Q is the aunt of P TV 29" = `4000
TV 25" = `3900
(c) P is the aunt of Q (d) P is the mother of Q
TV 21" = `3500
(e) None of these TV 17" = `3200
39. If P – R + Q, which of the following statements is true? Study the following pie charts to answer these questions
(a) P is the mother of Q (b) Q is the daughter of P Revenue

(c) P is the aunt of Q (d) P is the sister of Q


TV 29",
(e) None of these 5%

40. If P × R ¸ Q, which of the following is true?


(a) P is the uncle of Q (b) P is the father of Q TV 25",
15%
(c) P is the brother of Q (d) P is the son of Q
(e) None of these
TV17",
TV 21",

Quantitative Aptitude
60%
20%

DIRECTIONS (Qs. 41-45): Which is the next number in the


following number series?
41. 780, 681, 592, 513, 444, 385,?
(a) 320 (b) 324 (c) 332 Sales (in units)
(d) 336 (e) 340
42. 6, 17, 55, 196, 825,? TV 29",
(a) 4176 (b) 4212 (c) 4232 10%
(d) 4256 (e) 4281
43. 72, 193, 293, 374, 438, 487,? TV17", TV 25",
(a) 521 (b) 523 (c) 525 40% 25%
(d) 527 (e) 529 `
44. 4,45,368,2583,15504,?
(a) 77521 (b) 77522 (c) 77523
(d) 77524 (e) 77525 TV 21",
45. 180, 271, 352, 423, 484, 535,? 25%
(a) 576 (b) 577 (c) 578
(d) 579 (e) 580
Note: Profit = Sales Value – Manufacturing Cost
y
o
u
rs
m
IBPS CWE RRB Officer Scale-I Exam 2014 423

a
h
51. For which model was the selling price per unit is greater

b
DIRECTIONS (Qs. 61-66): Refer the following table to answer

o
than the average selling price?

o
the questions that follow :

b
(a) TV 21" (b) TV 17"

.w
(c) TV 25" (d) TV 29" Sectorwise Installed Capacity in MW

o
(e) None of these Year Thermal Hydel Nuclear Total

rd
52. What was the profit generated by the TV 29" model 2003 7900 6390 420 14710

p
re
(in ` Lakhs)? 2004 8200 6610 420 15230

s
(a) 5 (b) 4 2005 8900 6780 420 16100

s
.c
(c) 1 (d) 2 2006 9100 6965 640 16705

o
(e) None of these 2007 10150 7530 640 18320

m
53. What was the profit margin of the TV 25" model? 2008 11000 8500 640 20140
(a) 11.4% (b) 53.8% 2009 12000 9200 640 21840
(c) 24.6% (d) 38.2% 2010 13000 9880 640 23520
(e) None of these 2011 15200 10200 800 26200
54. The maximum percentage profit was for the model 2012 16700 10450 800 27950
(a) TV 25" (b) TV 29" 2013 19000 11000 800 30800
(c) TV 21" (d) TV 17" 61. The two years when the capacities were augmented in all
(e) None of these the sectors are
55. Total profit earned by the company in the year 2006 was (a) 2006 and 2011 (b) 2003 and 2004
` ________ million. (c) 2006 and 2009 (d) 2006 and 2013
(a) 6.47 (b) 5.58 (e) None of these
(c) 9.72 (d) 12.70 62. The percentage increase in hydel capacity over that of the
(e) None of these previous year was maximum in
(a) 2010 (b) 2004
DIRECTIONS (Qs. 56-60): Study the following graph carefully (c) 2012 (d) 2008
to answer these questions. (e) None of these
Quantity of Various Items Sold and Price per kg 63. In 2013, the percentage share of nuclear power in the total
power capacity installed was approximately
30 60
Price in (a) 10.0 (b) 2.59
25 50 ` per kg (c) 5.42 (d) 1.11
Quantity sold (e) None of these
20 40 in quintals 64. If the total power generated in thermal units be 40% of the
Price

15 30 installed capacity, in hydel units be 50% of the installed


capacity and in nuclear units be 90% of the installed
10 20 Quantity
capacity, the total power generation in 2011 would be
5 10 (a) 19835 MW (b) 11900 MW
(c) 5015 MW (d) 22100 MW
0 0
A B C D E F (e) None of these
Items 65. In 2003, the percentage share of thermal in total installed
56. If the quantity sold of item D increased by 50% and the capacity was approximately
price reduced by 10%. What was the total value of the (a) 45 (b) 50
quantity sold for item D ? (c) 26 (d) 54
(a) ` 675 (b) ` 6750 (c) `67550 (e) None of these
(d) ` 67500 (e) None of these 66. The growth in the installed thermal capacity between 1991
57. Approximately, what is the average price per kg of items and 2012 was approximately
A, B & C ? (a) 51% (b) 130%
(a) ` 9.50 (b) ` 8 (c) ` 7.50 (c) 92% (d) 103%
(e) None of these
(d) ` 9 (e) ` 10.50
67. A right circular cone and a right circular cylinder have
58. What is the ratio between the total values of quantity sold
equal base and equal height. If the radius of the base and
for items E & F respectively ? the height are in the ratio 5 : 12, then the ratio of the total
(a) 15 : 14 (b) 3 : 2 (c) 5 : 7 surface area of the cylinder to that of the cone is
(d) 7 : 5 (e) None of these (a) 3 : 1 (b) 13 : 9
59. Total value of the quantity sold for item C is what per cent (c) 17 : 9 (d) 34 : 9
of the total value of the quantity sold for item E ? (e) None of these
(a) 111 (b) 85 (c) 90 68. A manufacturer sells a pair of glasses to a wholesale dealer
(d) 87.5 (e) None of these at a profit of 18%. The wholesaler sells the same to a retailer
60. If the price as well as the quantity sold is increased by at a profit of 20%. The retailer in turn sells them to a
20% for item A, what is the total value of quantity sold for customer for ` 30.09, thereby earning a profit of 25%. The
item A? cost price for the manufacturer is
(a) ` 48500 (b) ` 49000 (c) ` 42000 (a) ` 15 (b) ` 16
(d) ` 50400 (e) None of these (c) ` 17 (d) ` 18
(e) None of these
y
o
u
rs
m
424 IBPS CWE RRB Officer Scale-I Exam 2014

a
h
69. Three wheels can complete respectively 60,36,24 revolutions 76. If one cap is picked at random, what is the probability that it

b
o
per minute. There is a red spot on each wheel that touches is either blue or yellow?

o
b
the ground at time zero. After how much time, all these spots 2 1 3

.w
will simultaneously touch the ground again? (a) (b) (c)
9 8

o
(a) 5/2 seconds (b) 5/3 seconds 4

rd
(c) 5 seconds (d) 7.5 seconds 6

p
(d) (e) None of these

re
(e) None of these 11

s
70. A train after travelling 150 km meets with an accident and

s
.c
then proceeds with 3/5 of its former speed and arrives at DIRECTIONS (Qs. 77-80): In the following questions two

o
its destination 8 h late. Had the accident occurred 360 km equations numbered I and II are given. You have to solve both

m
further, it would have reached the destination 4 h late. the equations and Give Answer if
What is the total distance travelled by the train? (a) x > y (b) x > y
(a) 840 km (b) 960 km (c) x < y (d) x < y
(c) 870 km (d) 1100 km (e) x = y or the relationship cannot be established.
(e) None of these 77. I. 20x2 – x – 12 = 0
71. Two friends P and Q started a business investing in the II. 20y2 + 27y + 9 = 0
ratio of 5 : 6. R joined them after six months investing an 78. I. x2 – 218 = 106
amount equal to that of Q’s. At the end of the year, 20% II. y2 – 37y + 342 = 0
profit was earned which was equal to `98,000. What was
the amount invested by R? 7 5
79. I. + = x
(a) `1,05,000 (b) `1,75,000 x x
(c) `2,10,000 (d) Data inadequate
II. y 2 -
(12 )5/2
=0
(e) None of these y
72. A can build up a wall in 8 days while B can break it in 3 80. I.
days. A has worked for 4 days and then B joined to work 361x + 16 = 0
with A for another 2 days only. In how many days will A II. 441y + 4 = 0
alone build up the remaining part of wall?

(a) 13
1
days (b) 7
1
days
ENGLISH LANGUAGE
3 3
DIRECTIONS (Qs. 81-85): Read the following passage and
1 answer the questions that follows:
(c) 6 days (d) 7 days
3
One of the creation’s great mysteries is the formation and
(e) None of these
73. In a family, a couple has a son and daughter. The age of existence of the rocky planets, such as our Earth. Recently,
the father is three times that of his daughter and the age of scientists who peered for 13 days across 1500 light years of
the son is half of his mother. The wife is nine years younger universe believe that they may have the answer to this question.
to her husband and the brother is seven years older than They used a sophisticated orbiting X-ray telescope, called
his sister. What is the age of the mother? Chandra, to study 1400 young stars in Orion Nebula and
(a) 40 years (b) 45 years (c) 50 years identified 27 stars that behave very much as the Sun may once
(d) 60 years (e) 65 years have done, some 4.6 billion years ago. These young stars are
surrounded by discs from which planets could condense and they
DIRECTIONS (Qs. 74-76): Study the following information
tend to erupt in vast flares far greater than anything now observed
carefully to answer the questions that follow. A box contains 2
in the mature Sun. These flares could be an evidence of a process
blue caps, 4 red caps, 5 green caps and 1 yellow cap:
that would clear the way for small rocky planets to form.
74. If four caps are picked at random, what is the probability Scientists peer back to a time when Venus, Earth and Mars might
that none is green ?
have first formed from stony cinders whirling around a young
7 5 7 star. So, it is said by a scientist that they do not have time machine
(a) (b) (c)
99 99 12 to see back, but the next big thing is to observe the sun-like stars
5 in Orion Nebula.
(d) (e) None of these
12 81. Scientists now believe that can now disclose the candid
75. If three caps are picked at random, what is the probability creation's great mystery, which is
that two are red and one is green ?
(a) the formation and existence of planets like Earth
9 6 1 (b) the formation of stars with planets orbiting around them
(a) (b) (c)
22 19 6 (c) the appearance of the Sun some 4.6 billion around them
3 (d) the appearance of planets from discs
(d) (e) None of these (e) None of these
22
y
o
u
rs
m
IBPS CWE RRB Officer Scale-I Exam 2014 425

a
h
92. (a) blessed (b) enjoys

b
82. ‘Chandra’ is the name of

o
(a) a noon (c) endows (d) prevails

o
b
(b) a star (e) occurs

.w
(c) a telescope 93. (a) aid (b) jeopardise

o
(c) promotes (d) endure

rd
(d) a newly invented time machine
(e) cater

p
(e) None of these

re
94. (a) acute (b) utilising
83. Young stars are surrounded by discs from which

s
(c) restricting (d) inspiring

s
.c
(a) satellites formed (b) planets condensed (e) increased

o
(c) eruption took place (d) universe was created 95. (a) both (b) besides

m
(e) None of these (c) combining (d) participating
84. Though the Orion Nebula has a number of stars, only 27 stars (e) a
(a) might have formed from stony cinders
DIRECTIONS (Qs. 96-100): Read each of the following
(b) tend to erupt in vast flares
sentences, some part of the sentence is given in bold. Below
(c) seem to have planets like Earth each sentence is given five ways of phrasing the bold given part.
(d) are found to be too young to be surrounded by discs Select the answer from among the choices which produces the
(e) None of these most effective sentence, one that is clear and exact. If there is
85. According to a scientist, the next best thing is no error in the bold given part (e) is the answer.
(a) to travel in space to have a close look at the stas
(b) to travel to Venus and Mars to see how they were formed 96. That was the record that make me sit up and take notice
(c) to look at the stars in Orion that behave like the Sun of Neil Hannon.
(d) to take a look at the planets in Orion, that behave like Earth (a) made me sit up and take notice
(e) None of these (b) had made me sit up and take notice
DIRECTIONS (Qs. 86-95): In the following passage there are (c) has made me sit up and take notice
blanks, each of which as been numbered. These numbers are (d) sit up and take notice
printed below the passage and against each, five words/phrases (e) No correction required
are suggested, one of which fits the blank appropriately. Find 97. These mistakes catch up with him when he ran for
out the appropriate word/ phrase in each case. President.
There is a considerable amount of research about the factors (a) caught up in (b) caught up with
that make a company innovate. So is it possible to create an (c) caught up on (d) caught up at
environment (86 ) to innovation? This is a particularly pertinent
(e) No correction required
(87) for India today. Massive problems in health, education etc.
(88) be solved using a conventional approach but (89) creative 98. When I told her about breaking the window, she goes
and innovative solutions that can ensure radical change and (90). through the roof.
There are several factors in India’s (91). Few countries have (a) she went through the roof
the rich diversity that India or its large, young population (92). (b) she went via the roof
While these (93) innovation policy interventions certain
(c) she went through roof
additional steps are also required. These include (94) investment
in research and development by (95) the government and the (d) she went with roof
private sector, easy transfer of technology from the academic (e) No correction required
world etc. To fulfill its promise of being prosperous and to be 99. A preventive programme for the break out of epidemic
at the forefront, India must be innovative. in quake-hit areas must be introduced.
86. (a) stimuli (b) conducive (a) A preventive programme before the break out of
(c) incentive (d) facilitated (b) A preventive programme for the breaking out of
(e) impetus (c) A preventive programme for the out break of
87. (a) objective (b) controversy (d) A preventive programme before the out break of
(c) doubt (d) question (e) No correction required
(e) inference 100. Last but not the least, I would like to thank all the
88. (a) cannot (b) possibly workers without whose co-operation, the rally wouldn’t
(c) should (d) never be a success.
(e) must (a) Last but not least, I would like to thank all the worker
89. (a) necessary (b) apply without their co-operation.
(c) need (d) consider (b) Last but not the least, I would like to thank all the
(e) requires worker without their co-operation,
90. (a) quantity (b) advantages (c) Last but not least, I would like to thank all the workers
(c) increase (d) chaos without whose co-operation,
(e) growth
(d) Last but not the least, I would like to thank all those
91. (a) challenges (b) praises
workers without their co-operation,
(c) favour (d) leverage
(e) No correction required
(e) esteem
y
o
u
rs
m
426 IBPS CWE RRB Officer Scale-I Exam 2014

a
h
only by external en vironmen tal pressures for

b
DIRECTIONS (Qs. 101-105): The sentences given in each

o
organizational change.

o
question, when properly sequenced, form a coherent paragraph.

b
Each sentence is labelled with a letter. Choose the most logical D. These young, skilled and educated workers bring with

.w
them demands for a voice in the determination of their
order of the sentences from amongst the given choices so as to

o
own organizational destiny-a chance to participate in

rd
form a coherent paragraph.
those decisions which affect their organizational lives.

p
101. P : Such a national policy will surely divide and never

re
(a) ABDC (b) BADC

s
unite the pople. (c) ABCD (d) BCDA

s
.c
Q : Infact, it suites the purpose of the politicians, they (e) None of these

o
can drag the people into submission by appealing to

m
DIRECTIONS (Qs. 106-110) : Read the following passage
them in the name of religion.
carefully and answer the questions given below it. Certain words/
R : In order to inculcate, the unquestioning belief, they
phrases are printed in bold to help you to locate them while
condemn the other states which do not follow their
religion. answering some of the questions.
S : The emergence of the theocratic states, where all types We have inherited the tradition of secrecy about the budget from
of crimes are committed in the name of religion of Britain where also the system has been strongly attacked by emi-
the Middle Ages is dangerous. nent economists and political scientists including Peter Jay. Sir
(a) P Q R S (b) S Q R P Richard Clarke, who was the originating genius of nearly every
(c) S Q P R (d) R SP Q important development in the British budgeting techniques dur-
(e) None of these ing the last two decades, has spoken out about the abuse of bud-
102. A. You would be very surprised indeed to find it hot. get secrecy: “The problems of long-term tax policy should surely
B. Cold, of course. be debated openly with the facts on the table. In my opinion, all
C. And yet that was what I found when I visited North governments should have just the same duty to publish their
Island, the northern part of New Zealand. expenditure policy. Indeed, this obligation to publish taxation
D. When you go bathing in a river or a pond, do you policy is really essential for the control of public expenditure in
expect the water to be hot or cold? order to get realistic taxation implications.” Realising that de-
(a) ACBD (b) CABD mocracy flourishes best on the principles of open government,
(c) ACDB (d) DBAC more and more democracies are having an open public debate
(e) None of these on budget proposals before introducing the appropriate Bill in
103. A. Because, if the manager’s subordinates are inefficient the legislature. In the United States the budget is conveyed in a
and ineffective and are not helped to increase their message by the President to the Congress, which comes well in
advance of the — date when the Bill is introduced in the Con-
efficiency and effectiveness, the task may not be
gress. In Finland the Parliament and the people are already dis-
achieved.
cussing in June the tentative budget proposals which are to be
B. This must be just as true as the responsibility for
introduced in the Finnish Parliament in September. Every bud-
achieving his prescribed tasks. .
get contains a cartload of figures in black and white - but the
C. If it is achieved it is at too great a cost, or at the risk dark figures represent the myriad lights and shades of India’s
of other effects, many of which are less obvious. life, the contrasting tones of poverty and wealth, and of bread so
D. It is often and that one of the prime responsibilities of dear and flesh and blood so cheap, the deep tints of adventure
a manager is the training and development of his staff. and enterprise and man’s ageless struggle for a brighter morn-
(a) ADBC (b) CABD ing. The Union budget should not be an annual scourge but a
(c) BDAC (d) ‘DBAC part of presentation of annual accounts of a partnership between
(e) None of these the Government and the people. That partnership would work
104. A. Modern research, however, has proved thatinvaders much better when the nonsensical secrecy is replaced by open-
even before the Aryans poured into this land. ness and public consultations, resulting in fair laws and the
B. It was thought that they came to a country which was people’s acceptance of their moral duty to pay.
Uncivilized and barbarian. 106. How do the British economists and political scientists re-
C. They had evolved a civilization higher than that of act to budget secrecy? They are
the Aryan hordes who came in their wake.
(a) in favour of having a mix of secrecy and openness.
D. Till recently the Aryans were regarded as the earliest
(b) indifferent to the budgeting techniques and taxation policies.
invaders of the land.
(a) ABCD (b) BCAD (c) very critical about maintenance of budget secrecy.
(c) BDAC (d) DBAC (d) advocates of not disclosing in advance the budget contents.
(e) None of these (e) None of these
105. A. Organizations today are becoming increasingly 107. The author thinks that openness in budget is essential as it
populated by youthful, highly skilled, highly educated leads to
workers. (a) prevention of tax implications
B. Two of the most prevalent and provocative (b) people’s reluctance to accept their moral duties
organizational dynamics of our time are the themes (c) exaggerated revelation of the strengths and weak-
of participation and change. nesses of economy
C. These demands for participation are creating pressures (d) making our country on par with Finland
for internal organizational change which are matched (e) None of these
y
o
u
rs
m
IBPS CWE RRB Officer Scale-I Exam 2014 427

a
h
b
108. The author seems to be in favour of DIRECTIONS (Qs. 115-117) : Choose the word which is most

o
(a) maintaining secrecy of budget

o
nearly the SAME in meaning to the word printed in bold as used

b
(b) judicious blend of secrecy and openness in the passage.

.w
(c) transparency in budget proposals

o
115. SCOURGE

rd
(d) replacement of public constitution by secrecy
(a) ritual (b) presentation

p
(e) None of these

re
109. The secrecy of the budget is maintained by all of the follow- (c) whip (d) compromise

s
s
ing countries except (e) remedy

.c
A Finland 116. MYRIAD

o
m
B India (a) adequate (b) functional
C United States (c) incompatible (d) abundant
(a) Only A (b) Only B (e) excellent
(c) Only C (d) A and C 117. DUTY
(e) B and C (a) obligation (b) imposition
110. Which of the following statements is definitely TRUE in the (c) tax-liability (d) function
context of the passage?
(e) job
(a) The British Government has been religiously main
taining budget secrecy. DIRECTIONS (Qs. 118-120) : Choose the word/phrase which
(b) Budget secrecy is likely to lead to corrupt practices. is most OPPOSITE in meaning to the word printed in bold as
(c) Consulting unjustifiable taxes with public helps make used in the passage.
them accept those taxes. 118. FLOURISHES
(d) There should be no control on public expenditure in (a) disappears (b) degenerates
democratic condition. (c) vanishes (d) blooms
(e) None of these (e) opens
111. Sir Richard Clarke seems to deserve the credit for 119. DEBATED
(a) transformation in the British budgetary techniques. (a) questioned severely
(b) maintenance of secrecy of the British budget. (b) opposed strongly
(c) detection of abuse of transparency in budget.
(c) accepted unconditionally
(d) bringing down the tax load on British people.
(d) discussed frankly
(e) None of these
(e) implemented forcibly
112. From the contents of the passage, it can be inferred that
120. IMPORTANT
the author is
(a) major (b) uncountable
(a) authoritarian in his approach.
(c) significant (d) unscheduled
(b) a democratic person.
(e) trivial
(c) unaware of India’s recent economic developments.
(d) a conservative person.
(e) None of these
COMPUTER KNOWLEDGE
113. Which of the following statement(s) is/are definitely false 121. Which of the following is not valid version of MS Office?
in the context of the passage? (a) Office XP (b) Office Vista
A Transparency helps unscrupulous elements to resort (c) Office 2007 (d) All of the above
to corrupt practices. (e) None of these
B Open approach of Government is a sign of healthy 122. You cannot close MS Word application by
democracy.
(a) Choosing File menu then Exit submenu
C People’s acceptance of their moral duties can best be
(b) Press Alt+F4
achieved through openness and public consultations.
(a) Only A (b) Only B (c) Click X button on title bar
(c) Only C (d) A and B (d) From File menu choose Close submenu
(e) B and C (e) None of these
114. For making the budget realistic, the Government should 123. The key F12 opens a
(a) refrain from making public the proposed provisions (a) Save As dialog box (b) Open dialog box
before finalisation. (c) Save dialog box (d) Close dialog box
(b) discuss it secretly within themselves. (e) None of these
(c) encourage the public to send in their suggestions. 124. What is the short cut key to open the Open dialog box?
(d) consult the public, defend their own plans and accept (a) F12 (b) Shift F12
public suggestions. (c) Alt + F12 (d) Ctrl + F12
(e) None of these (e) None of these
y
o
u
rs
m
428 IBPS CWE RRB Officer Scale-I Exam 2014

a
h
136. A co-processor

b
125. Where can you find the horizontal split bar on MS Word screen?

o
(a) On the left of horizontal scroll bar (a) Is relatively easy to support in software

o
b
(b) On the right of horizontal scroll bar (b) Causes all processor to function equally

.w
(c) On the top of vertical scroll bar (c) Works with any application

o
(d) On the bottom of vertical scroll bar (d) Is quite common in modern computer

rd
(e) None of these (e) None of these

p
re
126. Which of the following is not available on the Ruler of MS 137. A Microsoft Windows is ….. a(n)

s
Word screen? (a) Operating system (b) Graphic program

s
.c
(c) Word Processing (d) Database program
(a) Tab stop box (b) Left Indent

o
(e) None of these

m
(c) Right Indent (d) Center Indent
138. Which of the following is program group?
(e) All of them are available on ruler (a) Accessories (b) Paint
127. What is place to the left of horizontal scroll bar? (c) Word (d) All of the above
(a) Tab stop buttons (b) View buttons (e) None of these
(c) Split buttons (d) Indicators 139. Which is not application software?
(e) None of above (a) Windows NT (b) Page Maker
128. Which file starts MS Word? (c) WinWord XP (d) Photoshop
(a) Winword.exe (b) Word.exe (e) None of these
(c) Msword.exe (d) Word2003.exe 140. Which of the following is an example of a real time
(e) None of these operating system?
129. How many ways you can save a document? (a) Lynx (b) MS DOS
(a) 3 (b) 4 (c) Windows XP (d) Process Control
(c) 5 (d) 6 (e) None of these
(e) None of these 141. Computer Network is
130. If you want to keep track of different editions of a document (a) Collection of hardware componets and computer
which features will you use? (b) Interconnected by communication cahannels
(a) Editions (B) Versions (c) Sharing of resource and informaton
(c) Track Change (d) All of the above (d) All of the above
(e) None of these
(e) None of these
142. DHCP Stand for
131. If there are multiple recycle bin for a hard disk
(a) Dynamic host control protocol
(a) you can set different size for each recycle bin (b) Dynamic Host Configuration Protocol
(b) you can choose which recycle bin to use to store your (c) Dynamic Hyper Control Protocol
deleted files (d) Dynamic Hyper Configuration Protocol
(c) You can make any one of them default recycle bin (e) None of these
(d) All of the above 143. IPV4 Address is
(e) None of these (a) 8 Bit (b) 16 Bit
132. Identify false statement (c) 32 Bit (d) 64 Bit
(a) You can find deleted files in recycle bin (e) None of these
(b) You can restore any files in recycle bin if you ever need 144. ADSL Stand for
(c) You can increase free space of disk by sending files in (a) Asymmetric Dual Subscriber Line
recycle bin (b) Asymmetric Digital System Line
(d) You can right click and choose Empty Recycle Bin to (c) Asymmetric Dual System Line
clean it at once (d) Asymmetric Digital Subscriber Line
(e) None of these (e) None of these
133. If the displayed system time and date is wrong, you can 145. The Last Address of IP Address Represent
reset it using (a) Unicast Address (b) Network Address
(a) Write (b) Calendar (c) Broadcast Address (d) All of Above
(e) None of these
(c) Write file (d) Control panel
146. How many bits are there in the Ethernet Address?
(e) None of these
(a) 64 Bits (b) 48 Bits
134. You should save your computer from? (c) 32 Bits (d) 16 Bits
(a) Viruses (b) Time bombs (e) None of these
(c) Worms (d) All of the above 147. Which of following is not the networking devices?
(e) None of these (a) Getewasys (b) Linus
135. World Wide Web is being standard by (c) Routers (d) Firewalls
(a) Worldwide corporation (e) None of these
(b) W3C 148. Which of the following can be software?
(c) World Wide Consortium (a) Router (b) Firewalls
(d) World Wide Web Standard (c) Gateway (d) Modems
(e) None of these (e) None of these
y
o
u
rs
m
IBPS CWE RRB Officer Scale-I Exam 2014 429

a
h
b
149. What is the the address size of IPv6?
GENERAL AWARENESS

o
(a) 32 Bit (b) 64 Bit

o
b
(c) 128 Bit (d) 256 Bit 161. Which state in India has highest rural population?

.w
(e) None of these (a) UP (b) MP

o
rd
150. Which of the following IP Address class is Multicast? (c) Haryana (d) Bihar
(e) None of these

p
(a) Class A (b) Class B

re
(c) Class C (d) Class D 162. Which is the currency of Finland?

s
(a) Pound (b) Euro

s
(e) None of these

.c
(c) Dollar (d) Yen

o
151. If a web page's URL includes ________, the page secure. (e) None of these

m
(a) https (b) serial interface protocol 163. RBI imposed a limit of ATM transactions. Which cities
(c) cookie (d) flat-file does come under this?
(e) None of these (a) Pune and Raipur (b) Pune and Baroda
152. In a computer, a _______ Can perform both input and (c) Chandigarh and Raipur (d) Mumbai and Delhi
output functions. (e) None of these
(a) microphone (b) trackball 164. “The Vijay Mallya Story” book has been written by
(a) MB Krishan (b) K Giriprakash
(c) touch screen (d) RAM
(c) Balkrishan Acharya (d) Pankaj Singh
(e) None of these (e) None of these
153. What does MICR mean? 165. RBI license has been given to which micro-finance
(a) Magnetic Idle Cassette Recorder company?
(b) Magnetic Ink Codes reader (a) NAFED (b) IFFCO
(c) Magnetic Ink Cases Reader (c) Bandhan (d) IFCI
(d) Magnetic Ink Character Reader (e) None of these
(e) Magnetic Ink Code Reader 166. Which country is largest wind power producer?
(a) China (b) Japan
154. Which of the following is not an output device? (c) Brazil (d) Cuba
(a) Monitor (b) Printer (e) None of these
(c) Mouse (d) Speaker 167. Who has been appointed as chairman of Dronacharya
(e) None of these Award selection committee.
155. VGA stands for: (a) Ravi Shastri (b) Bishan Singh Bedi
(a) Video Graphics Array (c) Ajitpal Singh (d) Kapil Dev
(b) Visual Graphics Array (e) None of these
(c) Volatile Graphics Array 168. Who is the present chairman of ISRO
(a) K.Radha Krishnan (b) Vikram Sarabhai
(d) Video Graphics Adapter (c) Madhavan Nair (d) Satish Dawan
(e) Visual Gallery Adapter (e) None of these
156. The 'add or remove program' utility can be found in: 169. Headquarters of NABARD is located in
(a) Control Panel (b) CPU (a) Jaipur (b) Mumbai
(c) Desktop (d) Search Engine (c) Mysore (d) New Delhi
(e) My Computer (e) None of these
157. Which of the following is spreadsheet software? 170. Find the Public Sector Bank from the below list
(a) MS Excel (b) MS Word (a) AXIS Bank (b) UCO Bank
(c) HDFC Bank (d) ICICI Bank
(c) Power Point (d) Adobe Acrobat (e) None of these
(e) My Computer 171. Who received Rajeev Khel Ratna award for the year 2013?
158. What does 'ATM' stand for? (a) Ronjan Sodhi (b) Vijay Kumar
(a) Automatic Teller Machine (c) Sushel Kumar (d) Saina Nehwal
(b) Automated Teller Machine (e) None of these
(c) Augmented Teller Machine 172. In previous Axis Bank is called as_______
(d) Automatic Telling (a) UTI Bank (b) IBDI Bank
(e) Automated Transfer Machine (c) ICICI Bank (d) HDFC Bank
(e) None of these
159. Which one of the following in the first program of 173. Devdas book was written by_____
Microsoft? (a) Sharat Chandra Chattopadya
(a) Windows XP (b) Windows 98 (b) Chetan Bhagat
(c) MS DOS (d) Windows 7 (c) Rabindranath Tagore
(e) Windows 97 (d) Koutilya
160. DOT MATRIX is a kind of: (e) None of these
(a) Scanner (b) Operating System 174. Who received best actress Oscar Award for 2013?
(c) Software (d) Printer (a) Lisa Westcott (b) Jennifer Lawrence
(e) Monitorans (c) Anne Hathaway (d) None of these
(e) None of these
y
o
u
rs
m
430 IBPS CWE RRB Officer Scale-I Exam 2014

a
h
175. In which country Summer Olympics 2020 will be held? 189. Where will 2015 BRICS Summit will be held?

b
o
(a) England (b) Russia (a) Durban, South Africa (b) New Delhi, India

o
b
(c) Germany (d) Japan (c) Sanya, China (d) Ufa, Russia

.w
(e) None of these (e) None of these

o
176. Roger Federes is related to which game? 190. Ken-Betwa river link will benefit which two states?

rd
(a) Tennis (b) Badminton (a) UP & Rajasthan (b) UP & MP

p
(c) Polo (d) Squash (c) UP & Hariyana (d) MP & Bihar

re
(e) None of these (e) None of these

s
s
.c
177. Which bank has launched first equated EMI on debit cards? 191. Who got fields Medal by International Mathematical Union

o
(a) ICICI (b) HDFC (IMU)?

m
(c) SBI (d) Indian (a) Cedric Villani (b) Elon Lindenstrass
(e) None of these (c) S. Smirnov (d) Manjul Bhargava
178. Which Irish author created the character Dracula? (e) None of these
(a) James Joyce (b) Oscar Wilder 192. How much capital is to be infused in banks to meet Basel-
(c) Bram Stoker (d) John Banville III norms by 2018 as per union budget?
(e) None of these (a) `2.50 lakh crore (b) `2.60 lakh crore
179. What is the full form of E-BAAT? (c) `2.40 lakh crore (d) `2.30 lakh crore
(a) Elected Banking Awareness and Technology (e) None of these
(b) Electronic Budget Awareness and Technology 193. What is the full form of AIMA?
(c) Electronic Banking Awareness and Technology (a) All India Management Association
(d) Electronic Banking Awareness and Transaction (b) All India Monetary Association
(e) None of these (c) All India Money Association
180. Where is first skill Development centre (d) All India Managing Association
(a) Lucknow, Uttar Pradesh (e) None of these
(b) Nasik, Uttar Pradesh 194. Which of the following feature of RTGS is not true?
(c) Ujjain, Madhya Pradesh (a) It is used for high-value transactions
(d) Jaipur, Rajasthan (b) It is used when payments need to be settled urgently
(e) None of these (c) It is usually operated by a country’s central bank
181. World diabetes day is celebrated on (d) It is used for small transactions
(a) 10th December (b) 14th January (e) None of these
(c) 14th November (d) 10th November 195. In which state Rongali Bihu is celebrated?
(e) None of these (a) Odisha (b) Assam
182. Dipika Pallikal and Joshana Chinappa are related to (c) Manipur (d) Bihar
(a) Polo (b) Tennis
(e) None of these
(c) Squash (d) Bedminton
196. Which company has established 1200 megawatt Lanco
(e) None of these
power plant in Vdup ai?
183. The Nobel Prize 2013 in Physics went to Brit Peter Higgs and
(a) JSW Energy (b) TNA
(a) Serge Haroche (b) David Wineland
(c) Adani Group (d) Jindal Brothers
(c) Adam G. Riess (d) Francois Englert
(e) None of these
(e) None of these
184. Who is the head of Governance of Boards of Banks in 197. Which is the capital of Australia?
India? (a) Vienna (b) Canberra
(a) P. J. Nayak (b) Ajay Shankar (c) Cairo (d) Baku
(c) H. Devaraj (d) Hari Gautam (e) None of these
(e) None of these 198. What is full form of NEET?
185. Commercial paper (CP) is issued in the form of (a) National Evaluation Fund Transfer
(a) credit ratings (b) credit fund (b) National Electronic Funds Transfer
(c) promissory notes (d) securities (c) National Electronic Financial Transation
(e) None of these (d) National Electronic Fund Transaction
186. Which insurance company will insure under ‘varishtha (e) None of these
Pension Bima Yojana’? 199. Which committee permits NBFCS to work as Business
(a) United India (b) LIC correspondents of banks?
(c) National Insurance (d) New India Assurance (a) Ujit Patel Committee
(e) None of these (b) Deepak Mohanty Committee
187. Which country will host 7th IBSA Summit? (c) Nachiket Mor Committee
(a) Japan (b) China (d) Vijay Kelkar Committee
(c) India (d) Brazil (e) None of these
(e) None of these 200. A bank can be started with out the permission of RBI in
188. BKS lyengar who passed away recently was a renowned which part of India?
(a) Music teacher (b) Sanskrit teacher (a) South eastern part (b) North eastern part
(c) Yoga teacher (d) Philosophy teacher (c) North western part (d) South western part
(e) None of these (e) None of these
y
o
u
rs
m
IBPS CWE RRB Officer Scale-I Exam 2014 431

a
h
b
o
o
b
.w
o
rd
1. (b) Here every alphabet is substituted by one less than its and teachers is not a null set.

p
numerical equivalent. Hence this is incorrect.

re
i.e. A = 0, B = 2 – 1 = 1, C= 3 – 1 = 2 etc. Hence option (a)

s
s
.c
MONKEY = 12141310424
8. (d)

o
Hence option (b) Consultants

m
2–6: Researchers
Given information can be summarised as below Professors
Rock band – 26 Oct closing day (Wednesday) (×)’
Fash – p one day prior to debate competition group song – Teachers
(Wednesday) (×) (Saturday) (×)
2 days gap- between debete and group song.
Date Day
19 October W ednes day Rock band
20 October Thurs day
× Conclusion 1 : (True)
21 October Friday Fash-P Conclusion 2 : (True)
22 October Saturday Debate Conclusion 3 : (True)
23 October Sunday
× Conclusion 4 : (False)
24 October M onday Street play 9. (b) I and II conclude that some drivers are engineers or
25 October Tues day Grou p s ong some engineers are drivers.
26 October W ednes day Folk dance None of the other combination of sentences draws a
conclusion.
2. (d) 3. (d) 4. (c) 5. (c) 6. (c) ALTERNATIVE
7. (a) Some boys are scholars:
Let us draw a Venn diagram to represent the given
There are two possible deductions from this statement statements.
(i) All scholars are boys (i.e. the set of scholars is a subset E
of boys)
(ii) Some scholars are boys (i.e. the intersection of the set
D L
of scholars and boys is not a null set)
Some teachers are boys
There are two possible deductions from this statement T
(i) All boys are teachers (i.e. the set of boys is a subset of
teachers) So we see that only 4 (some engineers are drivers) follows.
(ii) Some boys are teachers (i.e. the intersection of the set 10. (d) I and II conclude that some barbers are not
of teachers and boys is not a null set) businessmen. i.e. conclusion 4.
All scholars are observers II and III conclude that some traders (who are
There are two possible deductions businessmen) are not fashion designer, i.e.
(i) All observers are scholars (i.e. the set of observers conclusion 2.
and scholars are identical) Nothing can be said about 1 and 3.
(ii) Some observers are scholars (i.e. the set of observers So only 2 and 4 follows.
is a subset of scholars) 11. (a) Some boys are scholars:
Now Analyze the conclusion: There are two possible deductions from this statement.
(1) Some scholars are boys All scholars are boys (i.e. the set of scholars is a subset
This is true of boys)
(2) This may or may not be true. Some scholars are boys (i.e. the intersection of the
(3) We know that either all or some observers are scholars sets of scholars and boys is not a null set)
and the intersection of the sets of scholars and boys is Some teachers are boys
not a null set. There are two possible deductions from this statement.
Hence this is true All boys are teachers (i.e. the set of boys is a subset
(4) The intersection of set of boys and scholars in not a of teachers)
null set and the intersection of set of boys and teachers Some boys are teachers (i.e. the intersection of the
is not a null set. sets of teachers and boys is not a null set)
All scholars are observers.
But this doesn’t mean that the intersection of boys
There are two possible deductions from this statement.
y
o
u
rs
m
432 IBPS CWE RRB Officer Scale-I Exam 2014

a
h
All observers are scholars (i.e. the set of observers

b
14. (c) 15. (b) 16. (e) 17. (a) 18. (d)

o
and scholars are identical) 19. (d) P’s sex can’t be determined even after joining the two

o
b
Some observers are scholars (i.e. the set of observers statements.

.w
is a subset of scholars) 20. (b) From II : C is older than only E. It means E is the

o
Let us analyze the conclusions: youngest.

rd
Some scholars are boys. 21. (c) From I :

p
re
This is true for both the cases that have been mentioned
Q M

s
above.

s
.c
Some scholars are not boys. So, R is to the south-east of Q.

o
This may or may not be true because there are chances

m
that all scholars may also be boys. R
Some observers are boys. From II :
We know that either all or some observers are schol- Q
ars (i.e. the set of observers is definitely a subset of
the set of scholars) and the intersection of the sets of So, R is to the south-east of Q.
scholars and boys is not a null set. L R
This means that some observers will always be boys. 22. (e) I + II : No of children in the row = 20 + 6 – 1 = 25
Hence conclusion 3 is true.
23. (c) From I : D has two sons.
Some teachers are scholars.
From II : D has two sons – T and M.
The intersection of set of boys and scholars is not a
null set and the intersection of set of boys and teach- 24. (d) M| ³ |N and H | £ | Q and Q | ³ | M. We cannot get a
ers is not a null set. relationship between H, Q and M.
But this doesn’t mean that the intersection of boys 25. (b) C | > | B, L | < | S and S | £ | C. There is no relationship
between B and S. But since L | < | S | £ |C, then C | >
and teachers will also be a null set.
| L is true .
There may or may not be some teachers who are scholars.
26. (e) 1| ³ | H, E | > | F and I | = | F. We get E | >| I, hence I
Thus this statement is incorrect. is wrong, also E | > | I | ³ | H hence E | > | H.
12. (c) The problem can be illustrated with the help of dia- 27. (c) V | = | O, R | ³ | V and O | ³ | B. Hence R | ³ | V | = | O
gram as below : | ³ |B. Either I or II is true.
House 28. (d) L | > | U, T | = | L and U | £ | W. We cannot get a
Coffee relationship between T and W hence I is wrong. II
10m shop may or may not be true.
20m
29-33 :
40m
40m
30m

20m
So, from the diagram we can see that he is 10 m away
from his house. 29. (d) None is true
So, option (c) is correct. 30. (d) B
13. (b) A is B’s mother and D is B’s brother So, D and B 31. (d) Three (H, G, E)
are A’s children. B is E’s wife. 32. (b) D studies in std. 2.
33. (c) H and the student of std. 6.
Hence, E and D are brother in law.
34-37 :
In the first statements, the code digit '1' as well as the word 'the'
14 - 18 : Employee Department Favourite sport occurs twice. So, '1' is the code for 'the'.
In the first and fourth statements, the common code digits are '1' and
A Personel Table Tennis
'3', and the common words are 'the' and 'dog'. Since '1' stands for
B Administration Foot ball 'the', so '3' is the code for 'dog'.
Thus, in the fourth statement, '4' is the code for 'ran'.
C Administration Hockey In the second and third statements the common code digits are '1'
D Administration Basketball and '6' and the common words are 'the' and 'cat'. Since 1 stands for
the. So, b is the code for 'cat.
E Marketing Cricket In the first and third statements, the common code digits are '1', '6'
F Personnel Volley ball and '2' and the common words are 'the', 'brown' and 'cat'. Since '1'
and '6' stand for 'the' and 'cat' respectively, so '2' is the code for
G Marketing Lawn tennis 'brown'.
H Marketing Badminton
Thus, in the third statement, '7' is the code for 'was'.
In the first and second statements, the common code digits are '1', '6'
y
o
u
rs
m
IBPS CWE RRB Officer Scale-I Exam 2014 433

a
h
and '8' and the common words are 'the', 'cat' and 'frightened'.

b
5 ´10000000 ´100

o
Since '1' and '6' stand for 'the' and 'cat' respectively, so '8' is
= 5000

o
the code for 'frightened'. Selling price of TV 29" =
100 ´100 ´1000

b
.w
Thus, in the second statement, '5' is the code for 'away'.
34. (c) 35. (d) 36. (b) 37. (a) Hence, selling price of TV 17´´ was greater than the

o
rd
38-40 : average selling price.

p
52. (c) Selling price of TV 29´´

re
Symbol –– Relation

s
s
+ ® Father 5 ´10000000 ´100

.c
= = 5, 000
100 ´ 10 ´1000

o
– ® Wife

m
´ ® Brother Cost price of a TV 29´´ = 4000
Profit = 5000 – 4,000 = 1,000
¸ ® Daughter
Total profit generated by the TV 29 ´´
38. (c) R(+)
1, 000 ´10 ´ 1000
= = 1, 00, 000
P(–) S(+) 100
Aunt 53. (b) Selling price of a TV 25´´
Q
39. (a) P = R 15 ´ 10000000 ´ 100
= = 6, 000
(–) (+) 100 ´ 25 ´ 1000
Mother Cost price of a TV 25´´ = 3,900
Q Profit per unit = 6,000 – 3,900 = 2,100
40. (d) Q
Son 2,100 ´100
Profit % = = 53.84 %
3,900
P R(–)
(+) 54. (d) % Profit on 25´´ = 53.84 %
1, 000 ´100
41. (d) Series is –99, –89, –79, –69, –59. % Profit on 29´´ = = 25%
42. (a) Series is × 1 + 11, × 2 + 21, × 3 + 31, × 4 + 41. 4, 000
43. (b) Series is + 112, + 102, + 92, + 82.
44. (e) Series is × 9 + 9, × 8 + 8, × 7 + 7, × 6 + 6. 4, 500 ´ 100
% Profit on 21´´ = = 128.57 %
45. (a) Series is +91, +81, +71, +61. 35, 00
46. (d) 25% of 84 × 24% of 85 = ?
21 × 204 = ? 11,800 ´100
% Profit on 17" = = 368.75 %
428.4 = ? 3, 200
64 ´ 16
= 4( ) = 4 = 4(? – 3) = ? – 3 = 1
?- 3 Hence, the maximum percentage profit was for the
47. (b)
256 model TV 17´´.
48. (c) 25 × 124 + 389 × 15 = 3100 + 5835 = 8935
55. (a) Total profit earned by the company
60 æ 2920 10375 ö
49. (b) ?= ´ç + ÷ = Total sales value – Manufacturing cost
100 è 13 18 ø
60 60 ´ 800 = 1,00,00,000 –{(4,000× 100) + (3900 × 250) + (3500
» ´ (225 + 575) ; = 480 × 250 ) + (3200 × 400)}
100 100
50. (e) 7365 + 29.16 + ? = 7437.16 = 1,00,00,000 –35,30,000 = 6.47 millions
? = 473.16 – 7394.16 56. (d) Total value of the quantity sold for item D
? = 43 = 1849 40 ´ 150 12.5 ´ 90
= ´ ´ 100 = 60 × 11.25 × 100
10000000 100 100
51. (b) Average selling price = = 10,000 = ` 67500
1000
57. (e) Average price per kg. of items A, B and C
20 ´10000000 ´100 (17.5 + 10 + 7.5 ) 35
Selling price of TV 21" = = = = ` 11.667 » 10.50 (approx)
100 ´ 25 ´1000 3 3
= 8,000 58. (a) Total value of quantity sold for item E = 15 × 25 × 100
= 37500
60 ´ 10000000 ´ 100 Total value of quantity sold for item F
Selling price of TV 17" =
100 ´ 40 ´ 1000 = 10 × 35 × 100 = 35000
= 15,000 Required ratio = 37500 : 35000 = 15 : 14
59. (e) Total value of the quantity sold for item C
15 ´10000000 ´ 100 = 45 × 100 × 7.50 = 33750
Selling price of TV 25" = = 6000 Total value of the quantity sold for item E
100 ´ 25 ´1000
y
o
u
rs
m
434 IBPS CWE RRB Officer Scale-I Exam 2014

a
h
= 22.5 × 100 × 15 = 33750

b
2pr ´ h + 2pr 2

o
33750

o
\ Required percentage = ´ 100 = 100% =

b
33750
pr r 2 + h 2 + pr

.w
60. (d) Required price
120 17.5 ´ 120

o
= 20 ´ 100 ´ ´ = 2400 × 21 = ` 50400 2h + 2r 24k + 10k

rd
100 100 = +
25k 2 + 144k 2 + 5k

p
61. (a) The two years in which the capacities were augmented r2 + h 2 + r

re
in all the sectors are 2006 and 2011 because in these

s
34k 34k 17

s
years there is a increment in all of them. =

.c
= = or 17 : 9
13k + 5k 18k 9

o
62. (d) % Increase in Hydel capacity over that of the previous

m
years is in all the years is as follows : 68. (c) Let the cost price of manufactures is = P

6610 – 6390 18 59P


in 2004 = ´ 100 = 3.44% Selling price of manufacturer = P + P × =
6390 100 50

6780 – 6610 59 P 59 P 20
in 2005 = ´ 100 = 2.57% Wholesaler selling price = + ´
6610 50 50 100

6965 – 6780 59 P 59 P 354 P


in 2006 = ´ 100 = 2.72% = + =
6780 50 250 250

7530 – 6965 354 P 354 P 25


in 2007 = ´ 100 = 8.1% Retailer selling price = + ´
6965 250 250 100
8500 – 7530 354 P 177 P 805 P
in 2008 = ´ 100 = 12.88% = + =
7530 250 500 500
9200 – 8500 805 P
in 2009 = ´ 100 = 8.2% Now, = 30.09
8500 500
9880 – 9200 Þ P = 17
in 2010 = ´ 100 = 7.39%
9200
Shortcut
10200 – 9880
in 2011 = ´ 100 = 3.2% æ100 100 100 ö
9880 P =çç ´ ´ ´30.09 ÷÷=17
10450 – 10200 è 118 120 125 ø
in 2012 = ´ 100 = 2.4%
10200
69. (c) A makes 1 rev. per sec
11000 –10450
in 2013 = ´ 100 = 5.2%
10450 B makes 6 rev per sec
From above it is obvious that the % increase is 10
maximum in 1995.
4
63. (b) % Share of nuclear power in the total power capacity C makes rev. per sec
10
800
in 2013 = ´ 100 = 2.597.
30800 In other words A, B and C take 1 , 5 & 5 seconds to
64. (b) Total power generated 3 2
40 50 90 complete one revolution.
= ´ 15200 + ´ 10200 + ´ 800
100 100 100
5 5 L.C.M. of 1,5,5
= 6080 + 5100 + 720 = 11900 MW L.C.M of 1, & = =5
65. (d) In 2003, the % share of thermal in total installed 3 2 H.C.F. of 1,3,2
capacity Hence, after every 5 seconds the red spots on all the
7900 three wheels touch the ground
= ´ 100 = 53.70 » 54
14710 70. (c) Let the total distance to be travelled = x km
66. (d) %Growth in installed thermal capacity between 2004 Speed of train = v km/h
and 2012 and time taken = t hr.
16700 – 8200 150 x - 150
= ´ 100 = 103.65% + = (t + 8)
8200 v æ 3v ö
67. (c) Let the radius of the base are 5k and 12k respectively ç ÷
è 5ø
Total surface area of the cylinder .....(1)
\
Total surface area of the cone
y
o
u
rs
m
IBPS CWE RRB Officer Scale-I Exam 2014 435

a
h
b
510 x - 510 1

o
+ = (t + 4) = th wall

o
v 3 12

b
v

.w
5 1 11
Remaining wall = 1 - = th

o
.....(2) 12 12

rd
Eq (2) – Eq (1)

p
1

re
510 150 x - 510 x - 150 Now, th wall is built up by A in one day..
- + - =-4

s
8

s
v v 3 3v

.c
v

o
5 5 11 11 1

m
\ th wall is built up by A in 8 ´ = 7 days .
360 360 ´ 5 12 12 3
- = - 4 Þ v = 60 km/hr..
v 3v 73. (d) Let the mother’s age be y years.
x
t= \ The age of father = (y + 9) years
60
Put in eqn (1) y
The age of son = years
2
150 x - 150 æ x ö æy ö
+ = ç + 8÷ The age of daughter = ç 2 - 7 ÷ years
60 3 ´ 60 è 60 ø è ø
5
Now according to the given condition,
5 x - 150 x æ yö
+ = +8 (y + 9) = 3ç 2 - 7 ÷
2 36 60 è ø
3 y - 42
x - 150 x 5 11 Þ y+9=
- =8- = 2
36 60 2 2 Þ 2y + 18 = 3y – 42
10 x - 1500 - 6 x 11
= Þ y = 60 years
360 2
74. (a) Total number of caps = 12
360 ´11
Þ 4x– 1500 = = 1980 Þ 4x = 3480 Total result n(S) = 12C4.
2
12! 12 ´ 11´ 10 ´ 9 ´ 8!
n (S ) =
x=
3480
km = 870 km 4! ´ 12! - 4 = 4 ´ 3 ´ 2 ´1 ´ 8! = 5 × 99
4 n(E1) = Out of 5 caps, number of ways to not pick a
71. (c) Let the total investment be `x. green cap = 5C0.
n(E2) = Out of 7 caps, number of ways to pick 4 caps
Then, 20% of x = 98000 7! 7 ´ 6 ´ 5 ´ 4 ´ 3!
= 7C4. = = = 35
4!´ 7!- 4 4 ´ 3 ´ 2 ´1 ´ 3!
æ 98000 ´ 100 ö n ( E1 ) n ( E 2 ) 1 ´ 35 7
Þx=ç ÷ = 490000. p (E ) = = =
è 20 ø n (s ) 5 ´ 99 99
75. (d) Total number of caps = 12
Let the capitals of P, Q and R be `5x, `6x and `6x 12! 12 ´ 11´ 10 ´ 9!
n (S) = 12C3 = = = 220
respectively. Then, 3! ´ 12! - 3 3 ´ 2 ´ 1´ 9!
n(E1) = Out of 4 red caps, number of ways to pick 2 caps
(5x × 12) + (6x × 12) + (6x × 6) = 490000 × 12 = 4C2
4! 4 ´ 3 ´ 2 ´1
= = =6.
2!´ 4!– 2 2 ´1´ 2 ´1
n(E 2 ) = Out of 5 green caps, number of ways to
æ 490000 ´ 12 ö pick one cap = 5 C1 = 5
Þ 168x = 490000 ´12 Þ x = ç ÷ = 35000.
è 168 ø n ( E1 ) ´ n ( E 2 ) 6´5 3
p (E) = = =
n (S ) 220 22
\ R’s investment = 6x = `(6 × 35000) = `210000. 76. (b) Total number of caps = 12
1 n (S) = 12C1 = 12
72. (b) A’s one day’s work = th work Out of (2 blue + 1 yellow) caps number of ways to
8 pick one cap n(E) = 3C1 = 3
n (E)
B’s one day’s work = 1 rd work Required probability p ( E ) = n (S) = 12 = 4
3 1
3
77. (b) I. 20x2 – x – 12 = 0
1 1 Þ 20x2 – 16x – 15x – 12 = 0
\ A’s 4 day’s work = 4 ´ = nd work
8 2 Þ 4x – (5x – 4) + 3 (5x – 4) = 0
Þ (5x – 4) (4x + 3) = 0
1 æ1ö æ1ö Þ 5x – 4 = 0 or 4x + 3= 0
\ In next two days, total wall = + 2ç ÷ - 2ç ÷
2 è8ø è 3ø
Þ x = 4 or -3
5 4
y
o
u
rs
m
436 IBPS CWE RRB Officer Scale-I Exam 2014

a
h
II. 20y2 + 27y + 9 = 0 94. (e) increased

b
o
Þ 20y2 + 15y + 12y + 9 = 0 95. (a) both

o
Þ

b
5y (4y + 3) + 3 (4y + 3) = 0 96. (a) 97. (b) 98. (a)

.w
Þ (5y + 3) (4y + 3) = 0 99. (d) Corrects the misuse of phrasal verb for noun. ‘Breakout’

o
Þ y = -3 or -3 is a phrasal verb meaning ‘to start or appear suddenly’

rd
while ‘ out break’ is a noun meaning ‘sudden

p
5 4

re
Clearly, x > y appearence. A preposition or an article is used before

s
a noun, not a verb. Use preposition ‘before’ in place

s
.c
78. (d) I. x2 = 106 + 218 = 324 of ‘for’ because a preventive measure is taken before

o
the appearance of epidemic.

m
\ x = 324 = ±18 100. (c) Corrects the idiomatic error. ‘Last but not least’ is an idiom
II. y2 – 37y + 342 = 0 which means ‘last but not less important than others’.
Þ y2 – 18y – 19y + 342 = 0 101. (b) S is the opening sentence which introduces the subject
followed by Q, the pronoun subject of Q replaces the
Þ y (y – 18) – 19 (y – 18) = 0
noun subject of S. This will be followed by R because
Þ (y – 19) (y – 18) = 0 the ‘they’ in R refers to the ‘politicians’ in S.
Þ y = 19 or 18 102. (d) ‘B’ is answering question asked in ‘D’ ‘A’ is hinting
towards an alternate situation and ‘C’ is giving
79. (e) I. 7 5 alternate example.
+ = x
x x 103. (d) ‘D’ is stating a fact. ‘B’ is supporting the argument in
‘D’, ‘A’ is highlighting what may happen if situation
Þ 7+5= x´ x in ‘D’ is not achieved. ‘C’ is indicating towards that
Þ x = 12 eventuality.
II. y 2 - ( )
5/2 104. (d) ‘D’ is stating a perception. ‘B’ is adding information
12
=0 to this perception. ‘A’ is proof against this perception.
y ‘C’ is a corollary of A.
2+
1 105. (b) ‘B’ is a statement on a trend. ‘A’ is cause of that
Þ y 2 - (12) (5/2)
=0 change. ‘D’ is the result of that cause. ‘C’ is the
ultimate result.
5/2 5/2
Þ y = 12 106. (c) Eminent British economists and political scientists
Þ y = 12 have strongly attacked the tradition of budget secrecy.
80. (c) I. 19x + 4 = 0 107. (e) It leads to the control of public expenditure in order to
Þ19x = – 4 set realistic taxation implications.
108. (b) He has presented the example of both, the open budget
-4
Þ x= system and the secret budget system, practised by
19 various countries and has looked into all their aspects.
II. 21y + 4 = 0 109. (d) 110. (e)
-4 111. (a) Sir Richard Clarke was the originating genius of
Þ y= nearly every important development in the British
21 budgeting techniques during the last two decades.
81. (a) The passage entirely talks about the formation and 112. (b)
existence of planets like Earth. Now, the scientists claim 113. (a) The statement goes against the idea of the passage.
that they can disclose the candid creation of the universe. 114. (d) An open public debate on budget proposals should be
82. (c) The given passage talks about the telescope named held before introducing the appropriate bill.
‘Chandra’, to study 1400 young stars in Orion Nebula. 115. (c) 116. (d) 117. (a) 118. (b) 119. (c)
83. (b) The passage mentions that the young stars are surrounded 120. (e) 121. (b) 122. (d) 123. (a) 124. (d)
by discs from which planets could condense and they tend 125. (c) 126. (d) 127. (b) 128. (a) 129. (a)
to erupt in vast flares. 130. (b) 131. (a) 132. (c) 133. (d) 134. (d)
84. (b) The telescope ‘Chandra’ has discovered that the Orion 135. (b) 136. (a) 137. (a) 138. (a) 139. (a)
Nebula has a number of stars, only 27 stars tend to erupt 140. (d) 141. (d) 142. (b) 143. (c) 144. (d)
in vast flares.
145. (c) 146. (b) 147. (b) 148. (b) 149. (c)
85. (c) Scientists with the help of telescope ‘Chandra’ hoped that
150. (d) 151. (a) 152. (c) 153. (e) 154. (c)
the stars, in the Orion may behave like the Sun. As these
155. (a) 156. (a) 157. (a) 158. (b) 159. (c)
young stars are surrounded by discs from which planets
160. (d) 161. (a) 162. (b) 163. (a) 164. (b)
could condense and they tend to erupt in vast flares.
165. (c) 166. (a) 167. (c) 168. (a) 169. (b)
86. (b) conducive
170. (b) 171. (a) 172. (a) 173. (a) 174. (b)
87. (d) question 175. (d) 176. (a) 177. (a) 178. (c) 179. (c)
88. (a) cannot 180. (d) 181. (c) 182. (c) 183. (d) 184. (a)
89. (c) need 185. (c) 186. (b) 187. (c) 188. (c) 189. (d)
90. (e) growth 190. (b) 191. (d) 192. (c) 193. (a) 194. (d)
91. (c) favour 195. (b) 196. (c) 197. (b) 198. (b) 199. (c)
92. (b) enjoys 200. (b)
93. (a) aid
y
o
u
rs
m
a
h
b
o
IBPS SPECIALIST (I.T.) OFFICER

o
b
.w
EXAM 2015 Based on Memory

o
rd
p
re
s
s
.c
REASONING ABILITY

o
DIRECTIONS (Qs. 6-10): In the following questions, the

m
symbols are used as follows
A© B means ‘A is greater than B’.
DIRECTIONS (Qs. 1-5) : Study the following information and
answer the given questions. A word arrangement machine, when A © B means ‘A is either greater than or equal to B’.
=
given an input line of words, rearrange them following a A = B means ‘A is equal to B’.
particular rule in each step. The following is an illustration of A @ B means ‘A is smaller than B.’
input and the steps of rearrangement. A @ B means ‘A is either smaller than or equal to B’.
=
Input Go for to though by easy To Access at
Now in each of the following questions, assuming the three
Step I Access go for to though by easy To at
statements to be true, state which of the two conclusions I
Step II Access at go for to though by easy To
and II given below them is definitely ture.
Step III Access at by go for to though easy To
Given Anwser (a) if only conclusion I is true;
Step IV Access at by easy go for to though To
Given Answer (b) if only conclusion II is true;
Step V Access at by easy for go to though To
Given Answer (c) if either I or II is true;
Step VI Access at by easy for go though to To
Given Answer (d) if neither I nor II is true
Step VII Access at by easy for go though To to
Given Answer (e) if both I and II are true.
(and step VII is the last step for this input). As per the rules
followed in the above steps, find out in the following questions 6. Statements Q @ R, R @ M, M @ D
=
the appropriate step for the given output. Conclusions I. D © R II. D © Q
1. Input Together over series on feast the so which of the 7. Statements M @ K, K © R, R © P
following steps will be the last but one? Conclusions I. P @ K II. P @ M
(a) II (b) III 8. Statements T © M, M = P, P © R
(c) IV (d) V Conclusions I. R @ T II. T © R
(e) None of these 9. Statements P @ Q, Q © K, K @ M
2. Input Every and peer to an for which of the following Conclusions I. M = Q II. M © Q
steps would be ‘an and every for peer to?
10. Statements P © F, M @ F, F © N
(a) II (b) IV =
Conclusions I. M © P II. N @ P
(c) V (d) III
11. Pointing to a girl, Arun said, ‘She is the daughter of my
(e) None of these
grandfather’s only son.’ How is the girl related to Arun?
3. The step II of an input is an follows: ‘and Do pet to an
(a) Daughter (b) Sister
that’.
(c) Cousin sister (d) Data inadequate
Which of the following would definitely be the input?
(e) None of these
(a) Do and pet to an that
(b) Do pet to and that on DIRECTIONS (Qs. 12-16) : Study the following information
(c) Do on pet to and that to answer the given questions.
(d) Cannot be determined Twelve card viz Mercedes, Swift, Santro, Accord, Innova, Polo,
(e) None of these Punto, Figo, Civic, City, Ferrari and Landrover are parked in
4. Input ‘Over Go for through at are’. two parallel rows containing six cars each,, in such a way that
Which steps will be the last step of the above input? there is an equal distance between adjacent cars. The cars parked
(a) II (b) VI in row-1 are parked in such a manner that the driver seated in
(c) IV (d) VII the cars would face the South. In row-2, cars are parked in such
(e) III a manner that the drivers seated in these card would face the
5. Input ‘Story for around on was he at’. North. Therefore, in the given parking arrangement each car
Which of the following will be step IV for the given input? parked in a row faces another car of the other row.
(a) around at He for story on was (i) Mercedes being a big car must be parked at one of the
(b) around at for He story on was extreme ends. Mercedes is parked second to the left of
(c) around at for He on story was Santro. Santro faces Figo.
(d) around at for He on was story (ii) Punto and Innova are parked immediately next to each
(e) None of these other. Neither is parked next to Figo or Santro.
y
o
u
rs
m
438 IBPS Specialist (I.T.) Officer Exam 2015

a
h
b
(iii) Civic is parked in such a manner that its driver when 20. A river flows West to East and on the way turns left and

o
seated in the parked car would face South. Civic is parked goes in a semi-circle round a hillock and then turns left at

o
b
third to the left of Polo. right angles. In which direction is the river finally flowing?

.w
(iv) City is parked in such a manner that it faces Ferrari. Ferrari (a) East (b) West

o
is parked second to the left of Landrover. (c) North (d) South

rd
Landrover faces North and is not parked next to Figo. (e) None of these

p
re
(v) Swift is parked third to the right of Ferrari and faces Innova.
DIRECTIONS (Qs. 21-25) : Study the following information

s
(Note Left and right parking directions are with reference

s
to answer the given questions.

.c
to the driver as if the driver is seated in the car.)

o
m
12. How many cars are parked between Santro and Civic? Twelve people are sitting in two parallel rows containing six
(a) One (b) Two people each, in such a way that there is an equal distance between
(c) Three (d) Four adjacent persons. In rows - 1 A, B, C, D, E and F are seated and
(e) Both cars are parked in different rows all of them are facing South. In row - 2 P, Q, R, S, T and V are
13. Polo is related to Ferrari in the same way as Innova is seated and all of them are facing North. Therefore, in the given
related to Accord. To which of the following is city related seating arrangement each member seated in a row faces another
to, following the same pattern ? member of the other row.
(a) Mercedes (b) Innova V sits third to right of S. S faces F and F does not sit at at
(c) Santro (d) Punto any of the extreme ends of the line. D sits third to right of C. R
(e) Landrover faces C. The one facing E sits third to right of P. B and P do not
14. Which of the following cars are parked at extreme ends of sit at the extreme ends of the line. T is not an immediate
the rows ? neighbour of V and A is not an immediate neighbour of C.
(a) Landrover, Punto (b) Innova, Polo 21. Who amongst the following faces D?
(c) Polo, Accord (d) Landrover, Innova (a) T (b) P
(e) Ferrari, Swift (c) Q (d) R
15. Which of the following card faces Polo? (e) None of these
(a) Mercedes (b) Civic 22. Who amongst the following represent the people sitting
(c) Landrover (d) Punto at extreme ends of the rows?
(e) Accord (a) R, F (b) T, A
16. Which of the following is true regarding Figo? (c) D, R (d) C, Q
(a) A driver seated in the parked Figo would face North (e) S, A
(b) Swift is parked second to the right of the car facing 23. Four of the following five are alike in a certain way and
Figo thus form a group. Which is the one that does not belong
(c) Punto and Ferrari are parked immediately next to to that group?
Figo (a) B–T (b) A–Q
(d) Mercedes and Accord are parked at the extreme ends (c) C–S (d) F–P
of the row in which figo is parked (e) D–R
(e) None is true 24. Four of the following five are alike in a certain way and
thus form a group. Which is the one that does not belong
DIRECTIONS (Qs. 17-18) : Following question are based on
to that group?
the information given below
(a) D (b) S
(i) ‘P × Q’ means ‘P is the brother of Q’.
(ii) ‘P ¸ Q’ means ‘P is the sister of Q’. (c) V (d) T
(iii) ‘P + Q’ means ‘P is the father of Q’. (e) A
(iv) ‘P – Q’ means ‘P is the mother of Q’. 25. How many persons are seated between R and T?
(a) One (b) Two
17. Which of the following represents ‘M is nephew of N’? (c) Three (d) Four
(a) N – K + M (b) N × K ¸ M ÷ T (e) None of these
(c) N ¸ K × M (d) N ¸ K + M × T
(e) None of these DIRECTIONS (Qs. 26-30) : Study the following information
18. How is T related to D in the expression: H + T ¸ R – D? carefully and answer the given questions.
(a) Nephew (b) Niece Representatives from eight different Banks viz., A, B, C, D, E,
(c) Nephew or Niece (d) Data inadequate F, G and H are sitting around a circular table facing the centre
(e) None of these but not necessarily in the same order. Each one of them is from
19. Vijay started walking towards South. After walking 15m, a different Bank viz. UCO Bank, Oriental Bank of Commerce,
he turned to the left and walked 15 m. He again turned to Bank of Maharashtra, Canara Bank, Syndicate Bank, Punjab
his left and walked 15 m. How far is he from his original National Bank, Bank of India and Dena Bank.
position and in which direction? F sits second to right of the representative from Canara
(a) 15 m, North (b) 15 m, South Bank. Representative from Bank of India is an immediate
(c) 30 m, East (d) 15 m, West neighbour of the representative from Canara Bank.
(e) None of these Representative from Bank of India is an immediate neighbour
y
o
u
rs
m
IBPS Specialist (I.T.) Officer Exam 2015 439

a
h
b
of the representative from Canara Bank. Two people sit between Give answer (a) If only conclusion I follows

o
the representative of Bank of India and B. C and E are immediate (b) If only conclusion II follows

o
b
neighbours of each other. Neither C nor E is an immediate (c) If either conclusion I or conclusion II follows

.w
neighbour of either B or the representative from Canara Bank. (d) If neither conclusion I nor conclusion II follows

o
Representative from Bank of Maharashtra sits second to Right

rd
(e) If both conclusion I and conclusion II follow
of D. D is neither the representative of Canara Bank nor Bank

p
Statements (Qs. 31 to 33) : All stars are bottles. Some bottles

re
of India. G and the representative from UCO Bank are are paper. No paper is a calendar.

s
immediate neighbours of each other. B is not the representative

s
31. Conclusions

.c
of UCO Bank. Only one person sits between C and the

o
I. Atleast some calendars are bottles.

m
representative from Oriental Bank of Commerce.
II. No calendar is a star.
H sits third to left of the representative from Dena Bank.
Representative from Punjab National Bank sits second to left of 32. Conclusions
the representative from Syndicate Bank. I. All stars being papers is a possibility.
26. Who amongst the following sit exactly between B and the II. No calendar is a bottle
representative from Bank of India? 33. Conclusions
(a) A and the representative from UCO Bank I. All calendars being stars is a possibility.
(b) F and G II. Atleast some bottles are stars.
(c) H and the representative from Bank of Maharashtra Statements (Qs. 34 to 35) : Some pencils are blankets. All
(d) H and G blankets are erasers.
(e) Representative from Syndicate Bank and Oriental 34. Conclusions
Bank of Commerce I. Atleast some pencils are erasers
27. Who amongst the following is the representative from II. All erasers being pencils is a possibillity
Oriental Bank of Commerce? 35. Conclusions
(a) A (b) C I. No eraser is a pencil.
(c) H (d) G II. All blankets being pencils is a possibility.
(e) D
28. Four of the following five are alike in a certain way based DIRECTIONS (Qs. 36-38) : In making decisions about
on the given arrangement and thus form a group. Which important questions, it is desirable to distinguish between
is the one that does not belong to that group? ‘strong’ argument and ‘weak’ argument. A ‘strong’ argument
(a) H-UCO Bank must be both important and directly related to the question. A
(b) A - Canara Bank ‘weak argument may not be directly related to the question and
(c) D - Bank of Maharashtra may be of minor importance or may be related to the trivial
(d) E - Syndicate Bank aspect of the question. Each question below is followed by two
arguments, numberd I and II. You have to decide which of the
(e) F - Punjab National Bank
arguments is ‘strong’ and which is ‘weak’
29. Who amongst the following sits second to left of B?
(a) C Give answer
(b) H (a) if only argument I is strong
(c) The representative from Canara Bank (b) if only argument II is strong
(d) The representative from Punjab National Bank (c) if either I or II strong
(e) G (d) if neither I nor II is strong and
30. Which of the following is true with respect to the given (e) if both I and II are strong.
seating arrangement? 36. Statement Keeping in consideration the longivitity of life
(a) B is the representative from Bank of Maharashtra in India, should the age limit for retirement in government
(b) C sits second to right of H jobs be increased?
(c) The representative from Dena Bank sits to the Argument
immediate left of the representative from UCO Bank I. Yes, other countries have decided so long before.
(d) A sits second to right of the representative from Bank
II. Yes, it is the actual demand of lakhs of employees.
of India
37. Statement Should the admission to professional courses
(e) The representatives from Bank of Maharashtra and
in India be given only on merit without any concession to
Syndicate Bank are immediate neighbours of each
any particular group of students?
other
Argument
DIRECTIONS (Qs. 31-35) : In each of the question sets below I. Yes, this will improve the quality of the professionals
are tow/three statements followed by two conclusions numbered as they will be able to complete the courses
I and II. You have to take the given statements to be true even if successfully.
they seem to be at variance from commonly kown facts and II. No, this will keep large number of socially and
then decide which of the given conclusions logically follows economically backward students out of the reach of
from the given statements disregarding commonly known facts. the professional courses.
y
o
u
rs
m
440 IBPS Specialist (I.T.) Officer Exam 2015

a
h
b
38. Statement Should private companies be allowed to operate (C) The government should restrict the issue of fresh licences

o
air services in India? to factories and automobiles.

o
b
Argument (D) Cancer, heart attacks, brain strokes, tuberculosis are the

.w
I. Yes, Indian Private Managements usually give better major disease which are rapidly increasing in industrial

o
services. cities.

rd
II. No, we should not ape the western countries. (E) All types of pollutants are very harmful for health.

p
re
(F) Excessive growth of industries has increased the pollution
DIRECTIONS (Qs. 39 to 40) : In each of the questions, below

s
s
level in the city.

.c
is given a statement followed by two courses of action. Course
43. Which of the following among (A), (B), (C) and (D) can

o
of action is a step for administrative decision to be taken for

m
improvement, follow up or further action in regard to the be an immediate course of action for the Government?
problem, policy etc. On the basis of the information given in (a) Only (A) (b) Only (B)
statement, you have to assume everything in the statement to (c) Only (C) (d) Only (D)
be true, then decide which of the given suggested courses of (e) All of these
action is/are logically worth pursuing. 44. Which of the following among (A), (B), (C) and (D) can
not be an immediate course of action for the government?
Give your answer as (a) Only (A) (b) Only (B)
(a) if only I follows (c) Both (A) and (B) (d) Only (D)
(b) if only II follows (e) None of these
(c) if either I or II follows 45. Which of the following (A), (B), (D) and (E) may be the
(d) if neither I nor II follows effect of increment in the pollution level in the city?
(e) if both I and II follow (a) Only (A) (b) Only (B)
39. Statement The central Bureau of Investigation receives (c) Only (D) (d) Only (E)
the complaint of an officer taking bribe to do the duty he is (e) None of these
supposed to.
DIRECTIONS (Qs. 46-50) : Study the following information
Courses of Action
carefully and answer the questions given below:
I. CBI should wait for some more complaints about the
officer to be sure about the matter. Eight persons, A, B, C, D, E, F, G and H, are sitting around a
II. CBI should try to catch the officer red-handed and rectangular table in such a way that two persons sit on each of
then take a strict action against him the four sides of the table facing the centre. Persons sitting on
40. Statement There has been significant drop in the water opposite sides are exactly opposite to each other.
level of all the lakes supplying water to the city. D faces North and sits exactly opposite H. E is on the
Courses of Action immediate left of H. A and G sit on the same side. G is exactly
I. The water supply authority should impose a partial opposite B, who is on the immediate right of C. A is next to the
cut in supply to tackle the situation. left of D.
II. The government should appeal to all residents through 46. Who is sitting opposite A?
mass media for minimal use of water. (a) G (b) D
41. In a certain code ‘CURATIVE’ is written as ‘BSVDDUHS’ (c) E (d) A
How is ‘STEAMING’ written in that code? (e) None of these
(a) BFUTFMHL (b) TUFBFMHL 47. Who is next to E in clockwise direction?
(c) BFUTLHMF (d) BFUTHOJN (a) G (b) B
(e) All of these (c) F (d) A or F
42. Four the following five are alike in a certain way and so (e) None of these
form a group. Which is the one that does not belong to 48. Which of the following pairs of persons has both the
that group? persons sitting on the same side with first person sitting to
(a) X-ray (b) Telephone the right of second person?
(c) Computer (d) Radio (a) DF (b) CB
(e) Television (c) FC (d) AG
DIRECTIONS (Qs. 43-45) : Study the following information (e) None of these
carefully and answer the given questions. 49. Who is sitting opposite E?
Any further increase in the population level in the city by way of (a) D (b) A
industrial effluents and automobile exhaustions would pose a (c) F (d) A or D
severe threat to the inhabitants. (e) None of these
(A) All the factories in the city should immediately be closed 50. Which of the following statements is definitely true?
down. (a) A is facing North (b) E is sitting opposite F
(B) The automobiles should not be allowed to ply on the road (c) F is the left of G (d) C is to the left of A.
for more than four hours a day. (e) None of these
y
o
u
rs
m
IBPS Specialist (I.T.) Officer Exam 2015 441

a
h
b
QUANTITATIVE APTITUDE (a) 16.56 km/h. (b) 17.89 km/h.

o
(c) 26.67 km/h. (d) 35 km/h.

o
b
51. If the cost price of 15 articles be equal to the selling price (e) None of these

.w
of 20 articles, then find the loss% in the transaction.

o
DIRECTIONS (Qs. 61-65) : What will come in place of

rd
(a) 16% (b) 20% question mark (?) in the following questions ?

p
(c) 25% (d) 26%

re
(e) None of these 61. 6, 7, 16, 51, 208, ?

s
(a) 1036 (b) 1042

s
52. A toy is in the shape of a hemisphere surmounted by a

.c
cone. If radius of base of the cone is 3 cm and height is (c) 1048 (d) 1056

o
m
4cm. The total surface area of the toy is : (e) None of these
(a) 33 p cm 2 (b) 42 p cm2 62. 18, 48, 100, 180, 294, 448, ?
(c) 66 p cm 2 (d) 56 p cm2 (a) 979 (b) 586
(e) None of these (c) 732 (d) 648
53. Two trains of length 110 m and 90 m are running on (e) None of these
parrallel lines in the same direction with a speed of 45km/h 63. 289, 361, 529, 841, 961,?
and 50 km/h respectively. In what time will they pass other? (a) 2209 (b) 2136
(a) 136 s (b) 138 s (c) 1849 (d) 1681
(c) 142 s (d) 146 s (e) 1369
(e) None of these 64. 8, 31, 122, 485, 1936, 7739, ?
54. A man rows 25 km downstream and 20 km upstream (a) 30460 (b) 30720
taking 5 h each time. What is the velocity of the current? (c) 30840 (d) 30950
(a) 1 km/h (b) 2.5 km/h (e) None of these
(c) 3.5 km/h (d) 4.5 km/h
65. 8484, 4248, 2112, 1074, 513, 286.5, ?
(e) None of these
(a) 136.25 (b) 161.25
55. A works twice as fast as B. If B can complete a work in 24
(c) 107.25 (d) 112.25
days independently, the number of days in which A and B
can together finish the work is (e) None of these
(a) 12 days (b) 9 days DIRECTIONS (Qs. 66-70) : Study the following information
(c) 8 days (d) 6 days to answer the given questions.
(e) None of these
56. A, B and C enter into a partnership by investings `15000, Percentage of Different Types of Employees
` 21000 and `27000 respectively. At the end of 8 months, in an Organization
B receives `1260 as his share. Find the share of A. Total Employees = 7000
(a) `600 (b) ` 1620
(c) `1080 (d) ` 720 officer II Steno
(e) None of these 11% 10%
57. The average weight of 25 students is 16 kg. The average officer I Assistant
weight of the first 12 students is 14 kg and of the last 12 8% 15%
students is 17 kg. Find the weight of the thirteenth student. clerk II
(a) 29 kg (b) 22 kg 20 % supervisor
(c) 27 kg (d) 24 kg 19%
(e) None of these clerk I
58. What will be the difference between simple interest and 19 %
compound interest at 4% per annum on a sum of `5000
after 3 yr? Employee Out of these Percent of
(a) ` 24.32 (b) ` 28.56 Direct promotees
(c) ` 32.48 (d) ` 36.18 Steno 30 70
(e) None of these
Assistant 40 60
59. Pure milk costs `20 per litre. After adding water the
milkman sells the mixture at the rate of `18 per litre, Supervisor 50 50
thereby making a profit of 25%. In what ratio does he mix Clerk I 90 10
the two? Clerk II 30 70
(a) 18 : 7 (b) 18 : 5
Officer I 90 10
(c) 12 : 7 (d) 12 : 5
(e) None of these Officer II 70 30
60. A car driver travels from the planes to a hill station. Which 66. What is the difference in direct recruit and promotee
are 200 km apart at an average speed of 40 km/h. In the assistants?
return trip he covers the same distance at an average speed (a) 210 (b) 280
of 20 km/h. The average speed of the car over the entire (c) 180 (d) 110
distance of 400 km is (e) None of these
y
o
u
rs
m
442 IBPS Specialist (I.T.) Officer Exam 2015

a
h
67. The promotee clerk I is approximately what per cent of

b
74. Expenditure of company ‘B’ in years 2009 and 2010 were

o
that of direct recruit clerk I? in the ratio of 5 : 7 respectively. What was the respective

o
b
(a) 10 (b) 9 ratio of their incomes?

.w
(c) 11 (d) 10.50 (a) 10 : 13 (b) 8 : 13

o
(e) 9.75 (c) 13 : 14 (d0 11 : 14

rd
68. How many employees are supervisor?
(e) None of these

p
(a) 1050 (b) 1019

re
(c) 1109 (d) 1290 75. Total expenditure of companies A and B together in 2014

s
was `13.5 lac. What was the total income of the two

s
(e) None of these

.c
69. How many total direct recruits among all types of companies (in ` lac) in that year?

o
m
employees are there? (a) 19.757 (b) 20.25
(a) 4000 (b) 3885 (c) 19.75 (d) Cannot be determined
(c) 3000 (d) 3115 (e) None of these
(e) None of these
70. Which type of employees has maximum number of direct DIRECTIONS (Qs. 76-80) : The following questions are based
recruits? on the stacked bar graph given below:
(a) Clerk I and Officer I (b) Officer I Sales of Various precious Stones in India for
(c) Clerk I (d) Clerk II
the Period of 2009-2010 to 2013-2014
(e) None of these
1600
DIRECTIONS (Qs. 71-75) : Study the following graph to 1400 Bezel
answer these questions. 1200
1000 Opal
Percent Profit Earned by Two Companies (in kg)
800 Ruby
A and B over the Years 600
Profit = Income - Expenditure Emerald
400
Profit 200 T opaz
Percent profit = ×100 0
Expenditure 2009- 2010- 2011- 2012- 2013-
70 Company A 10 11 12 13 14
Company B 76. What is the total sales of Ruby as a per cent of the total
60 sales of precious stones for the given period?
Percent profit earned

50 (a) 17.3% (b) 19.23%


(c) 23.1% (d) Cannot be determined
40 (e) None of these
30 77. By what percent is the average annual sales of Emerald
for the given period more than the sales of Opal in 2012-
20 13?
10 (a) 120% (b) 50%
(c) 25% (d) 40%
0 (e) None of these
2009 2010 2011 2012 2013 2014
71. If the income of company ‘A’ in 2011 was `142500 what 78. For how many years is the sales of Bezel as a percentage
was its expenditure in that year? of the total sales of precious stones less than that of Topaz?
(a) `105000 (b) `95500 (a) one (b) two
(c) `99500 (d) `105555 (c) three (d) four
(e) None of these
(e) None of these
72. Expenditure of company ‘B’ in 2012 was 90% of its
expenditure in 2011. Income of company ‘B’ in 2012 was 79. If the sales of Topaz increased from 2008-09 to 2014-15
what percent of its income in 2011? by 25% and increased from 2013-2014 to 2014-15 by 50%,
2 then what is the difference between the sales of Topaz in
(a) 130.5 (b) 96 2008-09 and that in 2014-15?
3
1 (a) 50000 tonnes (b) 100000 tonnes
(c) 121.5 (d) 99 (c) 140000 tonnes (d) 160000 tonnes
3
(e) None of these (e) None of these
73. If the expenditure of company ‘A’ in 2010 was `70 lac 80. Which of the given precious stones experienced the highest
and income of company A in 2010 was equal to its percentage growth in the sales in any year over that of the
expenditure in 2011. What was the total income (in lac `) previous year for the period 2010-11 to 2013-14?
of the company A in 2010 and 2011 together?
(a) 175 (b) 131.25 (a) Topaz (b) Emerald
(c) 218.75 (d) Cannot be determined (c) Ruby (d) Bezel
(e) None of these (e) None of these
y
o
u
rs
m
IBPS Specialist (I.T.) Officer Exam 2015 443

a
h
b
DIRECTIONS (Qs. 81-85): Study the following information 87. What is the average monthly income of a man if he saves

o
` 85000 during a year?

o
carefully to answer the questions given below it.

b
A. The average monthly expenditure for the first 4

.w
A survey conducted on 1800 villages shows that 25% of the months is ` 18000.

o
total villages have only adequate water supply. 15% of the total

rd
B. The average monthly expenditure for the next 8
number have proper supply of electricity only. 7% of the total

p
months is ` 21000.

re
number of villages have only proper education facilities. 12?%
88. What is the salary of D, in a group A, B, C, D, E and F

s
of the total number of villages have tele-communication services

s
whose average salary is ` 45000?

.c
only. 16% of the total number of villages have proper health

o
care services only. 6% of the total number of villages have A. Total salaries of A and F is ` 88900.

m
adequate water as well as supply of electricity. 8% of the total B. Total salaries of B and C is ` 95200.
number of villages have adequate supply of water, electricity as 89. What is the temperature on Thursday?
well as health care services. 5% of the total number of villages A. Average temperature for Monday, Tuesday and
have proper supply of electricity, tele-communication services Wednesday is 34°C.
as well as health care services and 6% of the total number of B. Average temperature for Tuesday, Wednesday and
villages have all the facilities. Thursday is 38°C.
81. How many villages in all have adequate water supply ? 90. The average of 12 numbers is 18. Find the new average
(a) 702 (b) 450 when
(c) 594 (d) 810 A. The average of first 10 numbers is 20.
(e) None of these
B. Each number is multiplied by 6.
82. How many villages in all have adequate supply of water
as well as electricity? DIRECTIONS (Qs. 91-93) : What approximate value should
(a) 360 (b) 108 come in place of the question mark (?) in the following questions?
(c) 720 (d) 972 (Note that you are not expected to calculate the exact value)
(e) None of these
91. 195.994 ¸ 13.995 ¸ 2.5 = ?
83. How many villages in all do not have proper supply of
(a) 5.15 (b) 5.9
electricity?
(a) 720 (b) 850 (c) 5.75 (d) 5.1
(c) 920 (d) 1080 (e) 5.6
(e) None of these 92. 441.441 + 256.256 = ?
84. How many villages have only proper education facilities? (a) 37 (b) 36
(a) 108 (b) 126 (c) 34 (d) 31
(c) 234 (d) 216
(e) 30
(e) None of these
93. 68.721 – 12.815 – 8.409 – 1.152 = ?
85. How many villages have all the facilities?
(a) 62.915 (b) 67.115
(a) 90 (b) 126
(c) 144 (d) 106 (c) 58.295 (d) 52.715
(e) None of these (e) 46.345

DIRECTIONS (Qs. 86-90) : Each of the question below DIRECTIONS (Qs. 94 and 95) : What should come in place
consists of a question and two statements marked A and B given of question mark (?) in the following questions?
below it. You have to decide whether the data provided in the
94. 6 × 0.6 × 0.06 × 0.006 × 60 = ?
statements are sufficient to answer the question. Read both the
(a) 77.76 (b) 7.776
statements and give answer.
(c) 0.7776 (d) 0.07776
(a) if the data in statement A alone is sufficient to answer the (e) 0.00776
question.
(b) if the data in statement B alone is sufficient to answer the 4.5 ´ 3.7 + 4.5 ´ 6.3
95. =?
question. 1.5 ´ 8.4 - 1.5 ´ 7.4
(c) if the data either in statement A alone or in statement B (a) 24 (b) 28
alone is sufficient to answer the question
(c) 34 (d) 32
(d) if the data given in both statements A and B together are
(e) 30
not sufficient to answer the question.
(e) if the data in both statements A and B together are DIRECTIONS (Qs. 96-100) : In each of these questions two
necessary to answer the question. equations are given. You have to solve these equations and Give
86. What is the average age of children in the class? answer.
A. Age of the teacher is as many yrs as the number of
children. (a) if x < y (b) if x > y
B. Average age increased by 2 yr, if the teachers age is (c) if x = y (d) if x > y
also included. (e) if x < y
y
o
u
rs
m
444 IBPS Specialist (I.T.) Officer Exam 2015

a
h
b
96. I. x2 – 6x = 7 very selective, it will automatically reduce the number of

o
entrants. This should apply particularly to new colleges, many

o
II. 2y2 + 13y + 15 = 0

b
of which are little more than degree factories. Only then can

.w
97. I. 3x2 – 7x + 2 0
the authorities hope to bring down the teacher student ratio to

o
II. 2y2 – 11y + 15 = 0 manageable proportions what is more, teacher should be given

rd
p
98. I. 10x2 – 7x + 1 = 0 refresher courses every summer vacation to brush up their

re
knowledge. Besides, if college managements increase the library
II. 35y2 – 12y + 1 = 0

s
s
budget it will help both staff and students a great deal. At the

.c
99. I. 4x2 = 25 same time, however, it will be unfair to deny college employers

o
m
II. 2y2 – 13y + 21 = 0 students a great deal. At the same time, however, it will be
100. I. 3x2 + 7x = 6 unfair to deny college deny college education to thousands of
young men and women unless employers stop insisting join
II. 6(2y2 + 1) = 17y degress even for clerical jobs. For a start, why can’t the
Government disqualify graduates from securing certain jobs say
ENGLISH LANGUAGE class III and IV posts? Once the degrees are declined from jobs,
at least in some important departments, it will make many young
DIRECTIONS (Qs. 101-106) : Read the following passage people think two before joining college.
carefully and answer the questions given below it. Certain 101. The author’s attitude is
words/pharases are given in bold to help you locate them while (a) cynical (b) optimistic
answering some of the questions. (c) critical (d) conservative
Frustration is global cancer. It has spared no country. In some (e) constructive
countries, frustration exists because these countries are populated 102. What, according to the passage, would be the granting
by “have-notes”. In other countries, which are populated by autonomy to some colleges?
‘haves’, frustration is among them also because they do have. (a) The teacher-students ratio will come down
As far as India is concerned its education system is breeding (b) Disparity between city and suburban colleges will
more and more frustration among both students and teachers. increase
And yet, no one sees what can be done to make it more (c) Colleges will multiply
meaningful. Many reforms have been discussed at length but (d) Some colleges would start selling degress.
have come to nothing for one reason or another. The authorities (e) All of the above
are peddling the idea of autonomous colleges as a means of 103. Which of the following is the most significant feature of
toning to teaching standards. They argue that by allowing certain the scheme of autonomous colleges according to the
colleges to introduce their own courses, hold seminar, and above passage?
all, evolve their own method of assessing students, students (a) They can introduce new courses
will get a far better deal, Indeed, they make out they such colleges (b) They can hold seminars
will have free hand in nearly everything except granting degrees. (c) They can evolve their method of assessment
In theory, all this sounds attractive enough. But there is little to (d) They can award degrees
show that the managements concerned are keen on such reforms. (e) They can make admissions without restricitions
Even today, nothing prevents a college from inviting quest 104. The author does not believe that
speakers on the specialist subjects or holding courses in English (a) colleges have the capacity to develop courses
for vernacular students. But not a single one of these who are (b) colleges have the expertise for assessment
now clamoring for autonomy has bothered to do so. It is no (c) some colleges are only degree factories
secret that colleges which may be freed to an extend from (d) colleges managements are really keen on reforms
university control are highly elitist. In Mumbai, for example, (e) some colleges are elitist
for example there is a big cultural gulf between city colleges 105. To which of the following does the author give precedence?
and suburban colleges. If some of the former are now given a
(a) Increasing library facilities
degree of autonomy, it will only heighten this disparity.
(b) Holding refresher courses for teachers
The answer to the vexing problem of declining standards
(c) Abolishing reservation for teachers
in higher education does not lie in encouraging the growth to
model institutions but in improving overall standards. This is (d) Stopping opening of new colleges
of course easier said than done. Since more and more young (e) Instituting entrance examinations
people are seeking degrees, the only solution, however 106. Frustration in the world is result of which of the following
unpalatable it may sound, is drastically to reduce the number of (A) Large number d of Haves
those who are admitted to colleges. Even though there is political (B) Large number of Have-nots
pressure on many state governments to build new colleges and (C) Concerous cells
to reserve more seats for backward classes, it will be sheer folly (a) Only (A) (b) Only (B)
to expand such facilities recklessly without giving any thought (c) Only (C) (d) (A) and (B)
to the quality of education imparted. If admissions are made (e) None of these
y
o
u
rs
m
IBPS Specialist (I.T.) Officer Exam 2015 445

a
h
113. Which of the following should be the SIXTH sentence

b
DIRECTIONS (Qs. 107 and 108) : Choose the word which is

o
after rearrangement?

o
most nearly the same in meaning to the word given bold used

b
in the passage. (a) E (b) G

.w
(c) C (d) B

o
107. Vexing (e) A

rd
(a) Annoying (b) Recurring 114. Which of the following should be the SEVENTH (Last)

p
re
(c) Irresolvable (d) Complex sentence after rearrangement?

s
(e) Dangerous (a) A (b) B

s
.c
108. Evolve (c) F (d) D

o
(a) Introduce (b) Start

m
(e) None of these
(c) Develop (d) Abandon 115. Which of the following should be the FIRST sentence
(e) Establish after rearrangement?
DIRECTIONS (Qs. 109 and 110) : Choose the word is Most (a) A (b) B
Opposite in meaning to the word given in bold as used in the (c) C (d) D
passage. (e) E
109. Folly DIRECTIONS (Qs. 116-125) : In the following passage, there
(a) Right (b) Exact are blanks, each of which has been numbered. These numbers
(c) Mistake (d) Action are printed below the passage and against each number five
(e) Wisdom words are suggested, one of which fits the blank appropriately.
110. Heighten Find out the appropriate word in each case.
(a) Widen (b) Decrease If you’ve ever watched a documentary where a cheeta his [116]
(c) Strengthen (d) Dissolve a gazelle for its next meal, you know what agility is. As the two
(e) Disappear animals [117] in a sixty-mile an hour life-and-death race, agility
DIRECTIONS (Qs. 111-115) : Rearrange the following seven and speed determine how the [118] the end. Whether the gazelle
sentences (A), (B), (C), (D), (E), (F) and (G) in the proper can [119] the cheetah or the cheetah can feed her cubs will be
sequence to form a meaningful paragraph; then answer the [120] by which animal is faster and more agile. Both animals
have skeletal and muscle structures that enable them to be [121]
questions given below them.
swift and nimble. Southwest Airlines, like the cheetah and the
A. But seriously, how much would you pay to know what gazelle, is lean and muscular and has quick [122] its agility in
thoughts are swimming around in someone else’s head? quikcly [123] people and reconfiguring resources comes in part
B. In most fictional movies thus, the idea of reading minds its [124] communication style. and its open communication is
of seeing the private intentions of another, and the partly a function of its lean structure and [125] code of conduct.
possibility of intervening in those plans & has always been 116. (a) proceeding (b) leading
highly attractive. (c) pursuing (d) racing
(e) investigating
C. Such fantastical questions have long been the bread and
117. (a) bind (b) engage
butter of fiction.
(c) conflict (d) interlock
D. Today, more than four centuries since the phrase, “A penny (e) pledge
for your thoughts?”, was first recorded, inflationary 118. (a) debate (b) controversy
accounting makes that ancient penny worth more than (c) life (d) confrontation
$40. (e) fun
E. The going rate for a “thought”- a probe into the thinking 119. (a) refrain (b) void
of another was once quite bargain (c) quash (d) please
F. And if you could really know their treeth fulness haw much (e) baffle
more would you pay? 120. (a) determined (b) ended
G. Even with the sliding value of the dollar, this still seems (c) claimed (d) defined
quite a bargain. (e) intended
121. (a) incredibly (b) excitedly
111. Which of the following should be the SECOND sentence
(c) ironically (b) expectedly
after rearrangement?
(e) believably
(a) F (b) D 122. (a) Stimulation (b) actions
(c) C (d) G (c) positions (d) manifestation
(e) A (e) reflexes
112. Which of the following should be the FOURTH sentence 123. (a) rendering (b) mobilizing
after rearrangement? (c) serving (d) perparing
(e) changing
(a) A (b) B
124. (a) smooth (b) complex
(c) G (d) D (c) defined (d) streamlined
(e) F (e) resistant
y
o
u
rs
m
446 IBPS Specialist (I.T.) Officer Exam 2015

a
h
b
125. (a) lively (b) lax 139. I want to joint (a)/ lecturership (b)/because I think it is a

o
(c) informal (d) non-existent (c)/peaceful proffession. (d)/ No error (e)

o
b
(e) vague 140. We must wait (a)/ for quiter time before (b)/the claim of

.w
civilization (c)/can override over the claims of the party

o
DIRECTIONS (Qs. 126-130) : Which of the phrases (1), (2),

rd
spirit. (d)/ No error (e)
(3) and (4) given below should replace the phrase given in bold

p
DIRECTIONS (Qs. 141-145) : Choose the correct alternatives.

re
type in the following sentence to make the sentence

s
grammatically meaningful and correct? If the sentence is correct

s
141. The _____ of the Minister's statement cannot be verified

.c
as it is and no correction required, make (5) as the answer. by people who have no access to official records.

o
m
126. The rationale for the use of various measures were known (a) veracity (b) verbosity
to people since long. (c) ambiguity (d) validity
(a) measure was being known (e) None of these
(b) measure had known 142. The more your action and thought are allied and ______
(c) measure had been known the happier you grow.
(d) measure to be known (a) diverget (b) unravelled
(c) integrated (d) invincible
(e) No correction required
(e) None of these
127. The inmates of prison refused water stating that they are
143. The Hubble Space Telescope will search for planets around
no hunger strike.
the stars, a key to the_____ extraterrestrial life, and examine
(a) they are under (b) we have been under
interstellar dust and gases out of which stars are born.
(c) we would be at (d) they were on
(a) perception (b) discovery
(e) No correction required (c) enquiry (d) quest
128. The differences could resolve through negotiations. (e) None of these
(a) should resolve (b) should have resolved 144. He knew everything better than anybody else, and it was
(c) could be resolved (d) could have resolved an affront to his ______ vanity that you should disagree
(e) No correction required with him.
129. The construction work was in full swing. (a) overstrung (b) overweening
(a) in fully swinging (b) fully swinging (c) overwhelming (d) overwrought
(c) swinging in full (d) at full swinging (e) None of these
(e) No correction required 145. One major ______ between the Election Commission and
130. What does it matter most is what you talk to other rather the Union Government related to the powers of the former
than how you do so. in respect of the deployment of central police forces at
(a) What matters (b) What matters it places where are elections is held.
(c) How does it matter (d) What it matters (a) irritant (b) conflict
(e) No correction required (c) pain (d) culprit
(e) None of these
DIRECTIONS (Qs. 131-140) : Read each sentence to find out
whether there is any error in it. The error, if any, will be in one DIRECTIONS (Qs. 146-150) : In each question, an incomplete
part of the sentence, the name of the part is the answer. If there statement followed by fillers is given. Pick out the best one
is No error, the answer is (e). which can complete incomplete stem correctly and meaningfully.

131. This is the wristwatch (a)/ which my uncle (b)/brought it 146. In order to help the company attain its goal of enhancing
(c)/for you. (d)/ No error (e) profit, all the employees ________
132. “Harish is comparatively better (a)/ today and we hope (a) urged the management to grant paid leave
that (b)/he will recover soon”, (c)/said Ramesh. (d)/ No (b) appealed the management to implement new welfare
error (e) schemes
133. When he lent me (a)/ some money, he asked (b)/to return (c) voluntarily offered to work overtime with lucrative
it back (c)/within a week. (d)/ No error (e) compensation
134. He doesn’t hardly know (a)/ about the real factors (b)/that (d) voluntarily offered to render additional services in
have created (c)/so many problems. (d)/ No error (e) lieu of nothing
(e) decided to enhance production at the cost of quality
135. He reimbursed back (a)/ the money which (b)/I spent (c)/
of the product
on his medical treatment. (d)/ No error (e)
147. His behaviour is so unpredictable that he ______
136. My friend asked (a)/ me if there was any place (b)/in the
(a) never depends upon others for getting his work done
(c)/compartment for him. (d)/ No error (e)
(b) is seldom trusted by others
137. All his family members (a)/ are (b)/social (c)/and
(c) always finds it difficult to keep his word
cooperative. (d)/ No error (e)
(d) always insists on getting the work completed on time
138. According to me (a)/ every student should (b)/go through (e) seldom trusts others as far as the work schedule is
the (c)/notes given by the professors. (d)/ No error (e) concerned
y
o
u
rs
m
IBPS Specialist (I.T.) Officer Exam 2015 447

a
h
b
148. Although initial investigations pointed towards him _____ 156. What is the valid host range the IP address 172.16.10.22

o
(a) the preceding events corroborated his involvement 255.255.255.240 is a part of?

o
b
in the crime (a) 172.16.10.20 through 172.16.10.22

.w
(b) the additional information confirmed his guilt (b) 172.16.10.1 through 172.16.10.255

o
(c) the subsequent events established that he was guilt (c) 172.16.10.16 through 172.16.10.23

rd
(d) the subsequent events proved that he was innocent (d) 172.16.10.17 through 172.16.10.31

p
re
(e) he gave an open confession of his crime (e) 172.16.10.17 through 172.16.10.30

s
149. The weather outside was extremely pleasant and hence 157. Which of the following are invalid SQL language

s
.c
we decided to ________ elements?

o
(a) utilise our time in watching the television (a) colon

m
(b) refrain from going out for a morning walk (b) Queries which retrieve data based on specific criteria.
(c) enjoy a morning ride in the open (c) Expressions which can produce either scalar values
(d) employ this rare opportunity for writing letters or tables consisting of columns and rows of data.
(e) remain seated in our rooms in the bungalow (d) Statements which may have a persistent effect on
150. With great efforts his son succeeded in convincing him schemas and data, or which may control transactions,
not to donate his entire wealth to an orphanage _____ program flow, connections, sessions, or diagnostics.
(a) and lead the life of a wealthy merchant (e) None of these
(b) but to a home for the forsaken children 158. Which protocol working at the Transport layer provides a
(c) and make an orphan of himself connectionless service between hosts?
(d) as the orphanage needed a lot of donations (a) IP (b) ARP
(e) as the orphanage had been set up by him (c) TCP (d) UDP
(e) None of these
PROFESSIONAL KNOWLEDGE 159. Which of the following categories of the data integrity
RDBMS will enforce specific business rules:
151. The number of tuples in a relation is called its ________ (a) Entity Integrity
While the number of attributes in a relation is called it's (b) Domain Integrity
_______ (c) Referential integrity
(a) Degree, Cardinality (b) Cardinality, Degree (d) User-Defined Integrity
(c) Rows, Columns (d) Columns, Rows (e) None of these
(e) None of these 160. SQL is all but:
152. Which of the following is false for a query? (a) SQL is an ANSI standard computer language
(a) It is an expression which on evaluation returns results (b) SQL allows you to access a database
from the DBMS. (c) SQL allows you to execute queries against a database
(b) It is a command given to the DBMS (d) SQL does not allow you to update records in a
(c) DBMS might support some kind of command to database
execute a query (e) None of these
(d) queries make the database useful
161. Disk scheduling includes deciding
(e) None of these
(a) which should be accessed next
153. What are the valid responses of a RDBMS?
(b) order in which disk access requests must be serviced
(a) Completion codes
(c) the physical location of the file
(b) Messages
(d) the logical location of the file
(c) Results of the queries
(e) None of these
(d) all of the above
162. Which among following scheduling algorithms give
(e) None of these
minimum average waiting time
154. The GROUP BY clause is not:
(a) used to combine, or group, rows with related values (a) FCFS (b) SJF
into elements of a smaller set of rows. (c) Round robin (d) On priority
(b) often used in conjunction with SQL aggregate (e) None of these
functions 163. What is not true about COMMIT and ROLLBACK?
(c) used to preserve duplicate rows in a result set. (a) interact with areas such as transaction control and
(d) used to eliminate duplicate rows from a result set. locking.
(e) None of these (b) COMMIT alone cannot make data permanent.
155. The parameter which gives probability of the transport (c) both terminate any open transaction and release any
layer itself spontaneously termination 2 a connection due locks held on data.
to internal problem is called? (d) both maintain consistency of database.
(a) Protection (e) None of these
(b) resilience 164. Which of the following is a function of e-mail system?
(c) option negotiation (a) Composition (b) Transfer
(d) transfer failure (c) Reporting (d) All of above
(e) None of these (e) None of these
y
o
u
rs
m
448 IBPS Specialist (I.T.) Officer Exam 2015

a
h
b
165. Mobile computers and persoanl digital assistant (PDAs) 175. A ________ Does not have a distinguishing attribute if

o
are the examples of ? its own and mostly are dependent entities, which are part

o
b
(a) Radio broadcasting of some another entity.

.w
(b) Wireless network (a) Weak entity

o
(b) Strong entity

rd
(c) Geosynchronous
(c) Non attributes entity

p
(d) LAN

re
(e) None of these (d) Dependent entity

s
(e) None of these

s
166. The XOR operator + is?

.c
(a) commutative 176. ________ is the complex search criteria in the where

o
clause.

m
(b) associative
(c) distributive over AND operator (a) Sub string (b) Drop Table
(d) (a) and (b) (c) Predict (d) Predicate
(e) None of these (e) None of these
167. An identifier in C? 177. The term module in the design phase refers to?
(a) is a name of thing such as variable and function (a) Functions (b) Procedures
(b) is made up of letters numerals and the underscore (c) Sub programs (d) All of the above
(c) can contain both uppercase and lowercase letters (e) None of these
(d) All of above 178. Switch is a Device of ________ Layer of OSI Model.
(e) None of above (a) Network Layer (b) Data Link Layer
168. Milestones are used to ? (c) Application Layer (d) Session Layer
(a) Know the cost of the project (e) None of these
(b) Know the status of the project 179. The number of binary trees with 3 nodes which when
(c) Know the user expectations traversed in post order gives the sequence A, B, C is?
(d) None of the above (a) 3 (b) 9
(e) None of these (c) 7 (d) 5
169. The average search time of hashing with linear probing (e) None of these
will be less if the load factor ? 180. Which database level is closest to the users?
(a) is far less that one (a) External (b) Internal
(b) equals one (c) Physical (d) Conceptual
(c) is far greater than one (e) None of these
(d) none of above 181. Which are the two ways in which entities can participate
(e) None of these in a relationship?
170. Negative numbers can’t be represented in ? (a) Passive and active
(a) signed magnitude form (b) Total and partial
(b) 1’s complement form (c) Simple and Complex
(c) 2’s complement form (d) All of the above
(d) None of above (e) None of these
(e) None of these 182. Which among these is incorrect for relations?
171. Sending a file from your personal computer’s primary (i) degree-number of tuples
memory or disk to another computer is called? (ii) relation-table
(a) Uploading (b) hang on (iii) tuple-row
(c) logging on (d) downloading (iv) attribute-column
(e) None of these (a) only (i) (b) (i) and (ii)
172. The memory cell of a dynamic RAM is simpler and smaller (c) (i) and (iii) (d) (iii) and (iv)
that the memory cell of a _______ RAM? (e) None of these
(a) volatile (b) semiconductor
183. What is meant by type in RDBMS?
(c) static (d) bipolar
(a) Domain. (b) Range.
(e) None of above
(c) Named set of values. (d) Both (a) and (c)
173. The normal form that is not necessarily dependency
(e) None of these
preserving is
184. A database schema includes the following information
(a) 2NF (b) 3NF
(c) BCNF (d) 4NF except:
(e) None of these (a) interface for user to interact
174. Fifth Normal form is concerned with (b) Format for storage representation
(a) Functional dependency (c) Integrity parameters such as physically authorization
(b) Multivalued dependency and backup policies.
(c) Join dependency (d) Characteristics of data items such as entities and
(d) Domain key attributes
(e) None of these (e) None of these
y
o
u
rs
m
IBPS Specialist (I.T.) Officer Exam 2015 449

a
h
b
185. Which of the following is not true? (b) Stdin, stdout, stderr

o
(a) External view:highest level of abstraction as seen by (c) All of above

o
b
a user (d) Key words, screen, stderr

.w
(b) Conceptual view:sum total of Data Base Management (e) All of above

o
rd
System user's views. 193. The index register in a digital computer is used for?

p
(c) Internal level :middle level of abstraction (a) Pointing to the stack address

re
(d) Internal level:how the data are physically stored. (b) Indirect addressing

s
s
(e) None of these (c) Keeping track of number of times a loop is executed

.c
186. Which of the following is true about WHERE clause? (d) Address modification

o
m
(a) does not include a comparison predicate, (e) None of these
(b) no restriction on the number of rows 194. The HAVING clause:
(c) eliminates all rows from the result set where the (a) includes a comparison predicate used to eliminate rows
comparison predicate does not evaluate to True after the GROUP BY clause is applied to the result set
(d) The WHERE clause is applied after the GROUP BY (b) it acts on the results of the GROUP BY clause,
clause. (c) aggregate functions can be used in the HAVING
clause predicate
(e) None of these
(d) all of the above
187. What protocol is used to find the hardware address of a
(e) None of these
local device? 195. What are the functions of DCL?
(a) RARP (b) ARP (a) handling the authorization aspects of data.
(c) IP (d) ICMP (b) permitting the user to control who has access to see
(e) BootP or manipulate data within the database.
188. Establishing a virtual connection is functionally equivalent (c) both (a) and (b)
to? (d) (a) only
(a) Connecting as virtual memory (e) None of these
(b) Physically connecting a DTE and DCE 196. Which of these are the activities done by RDBMS?
(c) Placing a telephone call prior to a conversation (a) take note of authorisations
(d) Placing a modem prior to a conversation (b) take note of constraints
(e) None of above (c) Update variables according to authorisations and
constraints
189. Which of the following is a variation of Frequency
(d) all of the above
Division multiplexing (FDM) ?
(e) None of these
(a) Time division Multiplexing (TDM) 197. Which of the following are the properties of entities?
(b) Pulse Code Multiplexing (PCM) (a) Groups (b) Table
(c) Wavelength Division Multiplexing (WDM) (c) Attributes (d) Switchboards
(d) None of above (e) None of these
(e) All of the these 198. ________ is preferred method for enforcing data integrity
190. The binary equivalent of the Gray code 11100 is ______ ? (a) Constraints (b) Stored Procedure
(a) 10111 (b) 00111 (c) Triggers (d) Cursors
(c) 01011 (d) 10101 (e) None of these
(e) None of these 199. ________ data type can store unstructured data
191. Star Topology is Based On a Central Device that can be (a) RAW (b) CHAR
________? (c) NUMERIC (d) VARCHAR
(e) None of these
(a) HUB (b) Switch
200. A DBMS responds to commands given by
(c) Only (a) (d) Both (a) and (b)
(a) general purpose application programs
(e) None of these (b) Custom developed application programs
192. Any program is C has access to three standard files? (c) users
(a) Standard input file, standard output file, standard (d) all of the above
error file (e) None of these
y
o
u
rs
m
450 IBPS Specialist (I.T.) Officer Exam 2015

a
h
b
o
o
b
.w
o
1. (d) Step I feast Together over series on the so 13. (c) Car parked opposite to Polo is Mercedes and Ferrari is

rd
to the immediate right of Mercedes. Similarly, Ferrari

p
Step II feast on Together over series the so

re
is parked opposite to city and Santro is to the
Step III feast on over Together series the so

s
immediate right of Ferrari.

s
.c
Step IV feast on over series Together the so

o
14. (c) Polo and Accord are parked at the extreme ends of
Step V feast on over series so Together the

m
the rows.
Step VI feast on over series so the Together
15. (a) Mercedes faces Polo.
2. (d) Using the above rule, we observe that ‘are and every
for peer to’ will be the III and last step for the given 16. (b) Swift is parked record to the right of the car facing
input. Figo i.e, Santro.
3. (d) For the given step we cannot definitely find out the 17. (d) N ¸ K + M × T
input because the position of the words in input
cannot be determined.
4. (c) Step I are over Go for through at N K
Step II are at over Go for through Sister
Step III are at for over Go through Father
Step IV are at for Go over through T
M Brother
5. (b) The step IV for the given input will be ‘around at for
He story on was’. Nephew
(Qs. 6-10) :
6. (e) Here Q < R < M £ D or D ³ M > R > Q
Hence, M is Nephew of N.
Now, I. D © Q Þ D > R (True)
18. (e) H + T ¸ R – D
II. D © Q Þ D > Q (True)
7. (a) Here M < K > R > P
H
Now, I. P @ K Þ P < K (True)
II. P @ M Þ P < M (False) Father
8. (e) Here T > M = P > R
T Sister
Now, I. R @ T Þ R < T (True) R
II. T © R Þ T > R (True)
Mother
9. (d) Here P < Q > K < M
Aunt D
Now, I. M = Q Þ M = Q (False)
II. M © Q Þ M > Q (False)
Hence, T is aunt of D.
10. (b) Here P ³ F > N, M < F
19. (e) Follow the given movements,
Now, I. M © P Þ M > P (False)
II. N @ P Þ N > P (True) Starting Final
11. (b) Girl is the daughter of Arun’s father. Hence, she is point point
Arun’s sister.
(Qs. 12-16) :
15 m 15 m
Polo City Figo Civic Innova Punto towards
south

left 15 m
Mercedes Ferrari Santro Landrover Swift Accord
Hence, he is 15 m to the East from the starting point.
12. (e) Both cars are parked in different rows.
y
o
u
rs
m
IBPS Specialist (I.T.) Officer Exam 2015 451

a
h
b
20. (a) Follow the given movements, 31. (d) I. Atleast some calenders are bottles. (False)

o
o
II. No calender is star (False)

b
.w
Hence, neither conclusion I nor conclusion II is true.

o
32. (d) I. All stars being papers is a possibility. ( False)

rd
S

p
R II. No calender is a bottle. (False)

re
Hence, neither conclusion I nor II follows.

s
P Q

s
.c
33. (b) I. All calendars being stars is a possibility (False)

o
Hence, river is finally flowing to East.

m
II. Atleast some bottles are stars (True)
(Qs. 21-25) :
Hence, only conclusion II follows.
D B F C E A (Qs. 34 to 35) :

Pencils Blankets
Erasers
T P S R Q V
34. (a) I. Atleast some pencils are erasers. (True)
21. (a) T faces D.
22. (b) T, A are sitting at the extreme ends of the row. II. All erasers being pencils is a possibility. (False)
23. (e) Except D-R all others are diagonally opposite to each hence, only conclusion I Nor II follows.
other. 35. (d) I. No eraser is a pencil. (False)
24. (b) Except S all others are sitting at extreme ends of the
II. All blankets being pencils is a possibility.(False)
rows.
25. (b) There are two persons (P and S) between T and R. hence, neither conclusion I nor II follows.
(Qs. 26-30) : 36. (d) Both the arguments are weak. Other countries have
different conditions. And in India population of youth
Punjab National Bank is increasing so, for the benefits of youth age limit
(E) for retirement should not be increased in government
Oriental Bank (C)Dena Bank jobs.
(D)
of Commerce 37. (b) Our country seaks to support educationally and
economically backward classes for their overall
Syndicate (B) (F) UCO Bank growth and development. Therefore, argument I is
Bank centre not strong, in Indian context.
38. (a) First argument is strong as it is based upon an
(G) Bank of India established fact. Second is based upon example, hence
(A)
Bank of it is a weak argument.
(H)
Maharashtra 39. (b) Only II follows because one complaint is enough for
Canara Bank
a wrong doing. This should be confirmed by catching
26. (c) H and the representative of Bank of Maharashtra. the officer red-handed and then strict action taken
27. (e) D is the representative of Oriental Bank of Commerce. against him.
28. (b) Except A – Canara Bank, all have the wrong bank.
40. (e) The situation can be tackled by periodic cuts in supply
29. (d) The representative from Punjab National Bank sits and urging people to conserve water. So, both the
second to the left of B. courses follow.
30. (e) The representatives from Bank of Maharashtra and
Syndicate Bank are immediate neighbours of each 41. (a) C B
other. U +1 S
(Qs. 31-33) : V
R
A D
Bottles Bottles
or T D
Stars Papers Calender Stars Papers
I +1 U
Calender V H
E S
y
o
u
rs
m
452 IBPS Specialist (I.T.) Officer Exam 2015

a
h
b
According to question
Similarly S

o
B

o
T +1 F 25 110 + 90

b
=

.w
18 t
E U

o
200 ´18

rd
A T
Þ t= = 144 s

p
25

re
M F

s
I +1 M 25

s
.c
54. (e) Speed downstream = km/h = 5 km/h
5

o
N H

m
L 25
G Speed upstream = km/h = 4 km/h
5
42. (a) All except X-ray are electronic media.
1
43. (c) 44. (a) 45. (c) 46. (e) 47. (b) Velocity of the current = (5 – 4) km/h = 0.5 km/h
48. (d) 49. (c) 50. (b) 2
55. (c)
H E
(46-50) : (days) x 2x (days)

G B
A B
A C x efficiency
2x
(The no. of days are indirectly proportional to the
D F efficiency)
51. (c) 15 ´ CP = 20 ´ SP Now Given that,
2x = 24
SP 15 x = 12 days.
Þ =
CP 20
12 ´ 24
A and B finish the work together = = 8 days
SP 15 36
-1 = -1
CP 20 56. (e) Ratio of capitals of A, B and C
= 15000 : 21000 : 27000
SP - CP 15 - 20 =5:7:9
Þ =
CP 20 Let the total profit be ` x.
5 7 x
= Loss = Then B’s share = ` x=`
20 21 3

5 x
Loss percentage = ´ 100 = 25% But = 1260
20 3
Þ x = 3780
52. (a)
57. (e) Sum of the weight of 25 students
4 cm
= (25 × 16)kg
3 cm Sum of the weight of first 12 students
= (14 × 12)kg = 168 kg
Total surface area of toy Sum of the weight of last 12 students
= area of hemisphere + area of cone = (17 × 12) kg = 204 kg
= 2pr2 + prl \ Weight of the thirteenth student
= (400 – 168 – 204) kg = 28 kg
= 2 ´ p ´ (32 ) + p´ 3 ´ 5 58. (a) Difference in rate of interest at 4%
= 18p + 15p = 33p æ 16 + 16 + 16 4 ´ 4 ´ 4 ö
= ç4 + 4+ 4 + + ÷ - 3´ 4
ç (100)2 ÷ø
(\ l = )
(3) 2 - (4) 2 = 25 = 5cm è
48 64
100

53. (e) Relative speed = (50 – 45) km/h = 5 km/hr = + = 0.48 + 0.0064 = 0.4864%
100 (100)2
5 25 5000 ´ 0.4864
Þ 5´ m/s = m/sec Difference in amount = = ` 24.32.
18 18 100 ´ 10000
y
o
u
rs
m
IBPS Specialist (I.T.) Officer Exam 2015 453

a
h
b
Number of direct recruits = 90% of 1330
æ 100 ö

o
59. (a) Mean cost price = ` ç ´ 18÷ = `14.4 per litre

o
è 125 ø 90 ´ 1330

b
= = 1197

.w
By the rule of alligation 100

o
CP of 1 L milk CP of 1 L water Now, number of promotees = 1330 – 1197 = 133

rd
p
20 0 133 13300

re
Hence, required percentage = ´ 100 = = 11%
1197 1200

s
Mean Price

s
.c
14.4 68. (e) Number of supervisors = 17% of 7000

o
m
14.4 5.6 17
= ´ 7000 = 1190
Required rate = 144 : 56 = 18 : 7 100
60. (c) Average speed of entire distance of 400 km 69. (b) Type of employees Number
total distance 10 30
= Steno ´ ´ 7000 = 210
total time 100 100
15 40
200 + 200 400 400 Assistant ´ ´ 7000 = 420
= = = = 26.67km / h 100 100
200 200 5 + 10 15
+ 17 50
40 20 Supervisor ´ ´ 7000 = 595
100 100
61. (b) The pattern is
19 90
× 1 + 1, × 2 + 2, × 3 + 3, × 4 + 4, × 5 + 5,.... Clerk I ´ ´ 7000 = 1197
100 100
So, the missing term is (208 × 5) + 5 = 1045
20 3
Clerk II ´ ´ 7000 = 420
648 100 100
62. (d) 18 48 100 180 294 448
90 8
Officer I ´ ´ 7000 = 504
100 100
+30 +52 +80 +114 +154 +200 11 70
Officer II ´ ´ 7000 = 539
100 100
22 28 34 40 46 Total direct recruitment
= 210 + 420 + 595 + 1197 + 420 + 54 + 539 = 3885
6 6 6 6 70. (c) Clerk I has maximum number of direct recruits.
71. (e) Let the expenditure of company A be
63. (e) The given pattern is 172, 192, 232, 292, 312, 372, .... 100
(ie, square of prime numbers) = 142500 ´ = ` 95000
150
So, the missing term is 372 = 1369 72. (b) Let the expenditure of company B in 2011 be `100.
64. (d) The given pattern is × 4 – 1, × 4 – 2, 4 × – 3, × 4 –
4, × 4 – 5, × 4 – 6, .... ( I1 - 100) ´ 100 = 35
Then
So, the missing term is (7739 × 4) – 6 = 30950 100
65. (c) The given pattern is Þ I1 = 100 + 35 = `135
Now, expenditure in 2012 = `9
8484 4248 2112 1074 513 286.5 107.25
( I2 - 90) ´ 100 = 45
Now,
÷ 12 + 6 ÷ 12 – 1 2 ÷ 12 + 1 8 ÷ 12 – 2 4 ÷ 12 + 3 0 ÷ 12 – 36 90
Þ 405 – 10I2 + 900 = 0
66. (a) Total number of assistants = 15% of 7000
1305
15 Þ I2 = = `130.5
= ´ 7000 = 1050 10
100
130.5 ´ 100 2
Number of direct recruits = 40% of 1050 Hence, required % = = 96 %
135 3
40 73. (c) Expenditure of company A = `70 lac
´ 1050 = 420 Then, income of company A in 2010
100
Now, number of promotee assistants 125
= ´ 70 = `87.5 lac
= 1050 – 420 = 630 100
Hence, required difference = 630 – 420 = 210 Expenditure of company A in 2011
67. (c) Number of clerk I = 19% of 7000 150
= 87.5 × = `131.25 lac
100
19 Hence, required total = `(87.5 + 131.25) lac
= × 7000 = 1330
100 = ` 218.75 lac
y
o
u
rs
m
454 IBPS Specialist (I.T.) Officer Exam 2015

a
h
b
74. (a) Given that, 8

o
Have proper water supply, electric supply ´ 1800 = 144

o
Exp. of B in 2009 5 100
=

b
and health care services

.w
Exp. of B in 2010 7
5

o
100 ´ 1800 = 90

rd
Have proper electric supply tele-
Income of B in 2009 ´ 100
140 = 5 communication and health care services

p
=

re
100 7 6
Income of B in 2010 ´

s
´ 1800 = 108

s
130 Have all the facilities

.c
100

o
IB2009 5 ´ 140 10 81. (d) Number of villages with adequate water supply

m
= = =
IB2010 7 ´ 130 13 = 450 + 108 + 144 + 108 = 810
75. (d) Let the expenditure of company ‘A’ be `a 82. (a) Number of villages with adequate supply of water
Then, expenditure of company B = `(135.5 – a) lac and electricity
( I1 - a ) ´ 100 = 45 = 108 + 144 + 108 = 360
Now, [company A] ...(i) 83. (d) Number of villages with proper supply of electricity
a
= 270 + 108 + 144 + 90 + 108 = 720
ëé I2 - (13.5 - a ) ûù ´ 100 = 50 Number of villages not having proper electric supply
and [company B] ...(ii)
(13.5 - a ) = 1800 – 720 = 1080
Here, we have three unknowns but two equations. 84. (b) Number of villages with only proper education
Hence, I 1 + I2 connot be determined. facilities = 126
76. (a) Total sales of Ruby 85. (e) Number of villages with all the facilities = 108
= 200 + 100 + 200 + 200 + 200 = 900
86. (d) Let the number of children be x ...(i)
900 From statement A we get, teacher’s age = x
Now, required % = ´ 100 = 17.3%
5200 From statement B we get, Average age of (x + 1)
1400 persons = (x + 2)
77. (d) Avg. annual sales of Emerald = = 2800 \ Teacher’s age = (x + 2) (x + 1) = x2
5
= x2 + 3x + 2 – x2 = 3x + 2 ...(ii)
280 - 200 From (i) and (ii) also we cannot find the average age
\ Required % = ´ 100 = 40%
200 of the children.
78. (c) It is clear from the chart. 87. (e) Savings of the man = ` 85000 ...(i)
79. (c) Sales of Topaz in 2008-09 From statement A we get
200000 Monthly expenditure of the man for the first 4 months
= = 1600000 tonnes
1.25 From statement B we get
Sales of topaz in 2014 – 2015 = 200000 × 1.5 = ` (18000 × 4) = ` 72000 ...(ii)
= 300000 tonnes = Monthly expenditure of the man for the next 8
Now, required increase = 300000 – 160000 months
= 140000 tonnes = ` (21000 × 8) = ` 168000 ...(iii)
80. (a) Highest % growth can be measured for Topaz.
From (i), (ii) and (iii), we can find the income and
Calculation for Questions (81 to 85) :
his average monthly income.
Facilities No. of Villagers 88. (a) A + B + C + D + E + F = (45000 × 6) = 270000
...(i)
25
Have only adequate water supply ´ 1800 = 450 From statement A, we get
100
A + F = 88900 ...(ii)
15 From statement B, we get
Have only proper electric supply ´ 1800 = 270
100 B + C = 95200 ...(iii)
7 From (i), (ii) and (iii) also we cannot find D, because
Have only proper education facilities ´ 1800 = 126
100 the salary of E is not given.
89. (d) From A
12
Have only telecommunication services ´ 1800 = 216
100 Monday + Tuesday + Wednesday = 34°c ´ 3 = 102°c
16 From B
Have only health care services ´ 1800 = 288
100 Tuesday + Wednesday + Thursday = 38°c ´ 3 = 114°c
6 Here, statment A and statment B together are not
Have water and electric supply ´ 1800 = 108
100 sufficient to answer the question.
y
o
u
rs
m
IBPS Specialist (I.T.) Officer Exam 2015 455

a
h
b
90. (b) From statement A, we get II. 2y2 – 13y + 21 = 0

o
Sum of the first 10 numbers = 10 × 20 = 200 ...(i) or, 2y2 – 6y – 7y + 21 = 0

o
b
Statement A is not sufficient to get the required or, (y – 3) (2y –7) = 0

.w
answer. or, y = 3, 7/2

o
Hence, y > x

rd
From statement B, we get new average
100. (e) I. 3x2 + 7x – 6 = 0

p
= (18 × 6) = 108 ...(ii)

re
or, 3x2 + 9 x – 2x – 6 = 0

s
Statement B alone is sufficient to get the required or, (x + 3) (3x – 2) = 0

s
.c
answer. or, x = – 3, 2/3

o
m
196 1 II. 6(2y2 + 1) = 17y
91. (e) ´ = 5.6 or, 12y2 + 6 – 17y = 0
14 2.5
or, 12y2 – 9y – 8y + 6 = 0
92. (a) ? = 441.441 + 256.256 or, (4y – 3) (3y – 2) = 0
or, y = 3/4, 2/3
= 441 + 256 Hence, y ³ x
= ( 21 + 16 ) » 37 101. (e) 102. (b) 103. (c) 104. (d) 105. (c)
93. (e) 68 – 12 – 8 – 1 = 47 106. (e) 107. (d) 108. (c) 109. (a) 110. (b)
111. (b) 112. (a) 113. (c) 114. (b) 115. (e)
6 6 6 77760
94. (d) ? = 6 ´ ´ ´ ´ 60 = 116. (c) 117. (b) 118. (d) 119. (e) 120. (a)
10 100 1000 1000000
121. (a) 122. (e) 123. (b) 124. (a) 125. (c)
7776 126. (c) 127. (e) 128. (c) 129. (e) 130. (a)
= = 0.07776
100000 131. (c) Delete ‘to’. It is superfluous
132. (a) Comparatively better is superfluous Harish is
4.5 ´ 3.7 + 4.5 ´ 6.3 4.5 ´ ( 3.7 + 6.3)
95. (e) = comparatively good or Harish is better.
1.5 ´ 8.4 - 1.5 ´ 7.4 1.5 ´ (8.4 - 7.4 ) 133. (c) Use of back with return is supperfluous.
134. (a) Use of not with hardly/scarcely is superfluous.
4.5 ´10
= = 30 135. (a) Use of back with reimburse/return is superfluous.
1.5 ´ 1 136. (b) Change place to room.
96. (b) I. x2 – 6x = 7 137. (a) write ‘All the members of his family’.
or, x2 – 6x – 7 = 0 138. (a) Replace ‘According to me’ by ‘In my opinion’.
or, (x – 7) (x + 1) = 0 139. (b) Replace ‘lectureship’ by lecturership’.
or, x = 7, – 1 140. (d) Use of over after override is superfluous.
II. 2y2 + 13y + 15 = 0 141. (a) 142. (c) 143. (d) 144. (a) 145. (b)
or, 2y2 + 3y + 10y + 15 = 0
146. (d) 147. (b) 148. (d) 149. (c) 150. (c)
or, (2y + 3) (y + 5) = 0 or,
151. (b) 152. (b) 153. (d) 154. (c)
y = –3/2, –5
155. (b) Resilience is the ability to provide and maintain an
Hence, x > y
acceptable level of service in the face of faults and
97. (a) I 3x2 – 7x + 2 = 0
challenges to normal operation.
or, 3x2 – 6x – x + 2 = 0
156. (e) First start by using the 256 mask, which in this case is
or, (x – 2) (3x – 1) = 0
256-240 =16. The first subnet is 16; the second subnet
or, x = 2, 1/3
is 32. This hast must be in the 16 subnet; the broadcast
II. 2y2 – 11y + 15 = 0
address is 31 and the valid hast range is 17-30.
or, 2y2 – 6y – 5y+ 15 = 0
157. (a)
or, (2y – 5) (y – 3) = 0
158. (d) User Datagram Protocol is used at the Transport layer
or, y = 5/2, 3
to provide a connectionless service.
Hence, y > x
159. (d) 160. (d) 161. (b) 162. (b) 163. (b)
98. (d) I. 10x2 – 7x + 1 = 0
or, 10x2 – 5x – 2x + 1 = 0 164. (d)
or, (2x – 1) (5x – 1) = 0 165. (b) Persoanl Digital Assistant also known as palmtop
or, x = 1/2, 1/5 computers is a mobile device that functions as the
II. 35y2 – 12y + 1= 0 personal information manager, mostly access internet
or, 35y2 – 7y – 5y + 1 = 0 through Wi-fi or wireless wide area network.
or, (5y – 1) (7y – 1) = 0 166. (d) As A + B = B + A and A + ( B+C) = (A+B) + C Hence
or, y = 1/5, 1/ 7 it is commutative and associative.
Hence, x y 167. (d)
99. (a) I. 4x2 = 25 168. (a) Milestones are used to measure the process or status
or, x2 = 25/4, or x = ± 5/2 of the project
y
o
u
rs
m
456 IBPS Specialist (I.T.) Officer Exam 2015

a
h
b
169. (a) Load factor is the ratio number of records that are 187. (b) Address Resolution Protocol (ARP) is used to find

o
currently present and the total number of records that the hardware address from a known IP address.

o
b
can be present. If the load factor is less, free space 188. (c)

.w
will be more. This means probability of collision is 189. (c) The WDM is commonly applied on the optical carrier

o
less. So the search time will be less. while FDM is applied on the radio carrier. Since

rd
wavelength and frequency are tied together through

p
170. (d)

re
171. (a) Uploading refers to sending data from local system a simple directly inverse relationship, the two terms

s
actually describe the same concept.

s
to remote system such as server or client.

.c
190. (a) The rule for changing the Gray code to binary is that

o
172. (c) 173. (a) 174. (c) 175. (a) 176. (d)

m
first bit remains the same and the next bit is obtained
177. (d) All the terms have the same meaning. by adding the first LSB of binary to the second LSB
178. (b) Switches Operate On Th Second Layer of OSI Model of Gray code and so on ... So the answer of the
That is Data Link Layer. question is 10111.
179. (d) Five trees are 191. (d) Hub and Switch are used in Star Type Networks.
192. (b)
193. (d) An index register in a computer’s CPU is a processor
register used for modifying operand addresses during
the run of a program, typically for doing vector/array
operations.
180. (a) 181. (b) 182. (a) 183. (d) 184. (a) 194. (d) 195. (c) 196. (d) 197. (c) 198. (a)
185. (c) 186. (c) 199. (a) 200. (d)
y
o
u
rs
m
a
h
b
o
IBPS CWE-RRB OFFICER SCALE-I

o
b
.w
o
EXAM 2015 Based on Memory

rd
p
re
s
s
.c
REASONING ABILITY DIRECTIONS (Qs. 8-10) : The following questions are based

o
m
on the alphabetical series given below :
1. The positions of the first and the fourth letters of the word
PLANET are interchanged, similarly, the positions of the C LRT BQ S MAPD I NF J K GYX
second and fifth letters and third and sixth letters are 8. Four of the following five are alike in a certain way and
so form a group. Which is the one that does not belong
interchanged. In the new arrangement thus formed, how
to the group ?
many letters are there between the letter which is second
from the right and the letter which is fourth from the left (a) LBT (b) SPA
according to the English alphabetical order ? (c) IJF (d) PID
(a) None (b) One (e) BMS
(c) Two (d) Three 9. If ‘CT’ is related to ‘RQ’ and ‘AI’ is related to ‘DF’ in
(e) Four a certain way, to which of the following is ‘SP’ related
2. The positions of how many alphabets will remain to, following the same pattern ?
unchanged if each of the alphabets in the word WORTHY (a) MD (b) DN
is arranged in alphabetical order from left to right ? (c) AD (d) AI
(a) None (b) One (e) DF
(c) Two (d) Three 10. What will come in place of the question mark in the
(e) More than three following series ?
3. How many such pairs of letters are there in the word CR LB TM SI ?
REASON, each of which has as many letters between them (a) PK (b) DK
in the word (in both forward and backward directions) as (c) DG (d) NX
they have between them in the English alphabetical series? (e) PG
(a) None (b) One
(c) Two (d) Three DIRECTIONS (Qs. 11-15) : Study the following information
(e) More than three and answer the questions given below:
4. ‘Writing is related to Pen’ in the same way as ____ ? Eight people— S, R, N, L, M, T, O and P are sitting in a circle
(a) ‘Singing’ is related to ‘Music’ facing the centre. All eight belong to different professions —
(b) ‘Stitching’ is related to ‘Needle’ reporter, doctor, cricketer, teacher, accountant, shopkeeper,
(c) ‘Dancing’ is related to ‘Exercise’ painter and supervisor. They are not necessarily seated in the
(d) ‘Carpenter’ is related to ‘tools’ mentioned order.
(e) ‘Paper’ is related to ‘book’ M is sitting third to the left of O. The doctor is to the
5. If it is possible to make only one meaningful word with immediate right of M and M is not a reporter. R is sitting fourth
the second, third, eighth and ninth letters of the word to the right of P. Neither R nor P is an immediate neighbour
‘CONFLICTED’, which would be the second letter of the of M. T is a teacher and is sitting third to the right of the doctor.
word from the left ? If more than one such word can be The shopkeeper is sitting second to the left of the teacher. The
formed, give ‘A’ as the answer. If no such word can be painter is sitting second to the left of M. S the cricketer is sitting
formed, give ‘Z’ as your answer. exactly between T and P. The accountant is sitting second to
(a) A (b) T the right of the cricketer. N is sitting third to the left of T.
(c) O (d) N 11. Who amongst the following is a reporter ?
(e) Z (a) O (b) L
6. In which of the following expressions will the expression (c) N (d) R
P < M be definitely true ? (e) None of these
(a) M <R > P > S (b) M > S = P < F 12. What is S’s position with respect to R ?
(c) Q < M < F = P (d) P = A < R < M (a) Third to the right (b) Second to the right
(e) None of these (c) Third to the left (d) Second to the left
7. In a case of 42 children, Joseph’s rank is sixteenth from (e) Fourth to the right
the top. Kevin is seven ranks below Joseph. What is Kevin’s 13. How many people are sitting between P and N when
rank from the bottom ? counted in an anti clockwise direction from N ?
(a) 22nd (b) 20th (a) One (b) Two
(c) 19th (d) 23rd (c) Three (d) Four
(e) 25th (e) None
y
o
u
rs
m
458 IBPS CWE-RRB Officer Scale-I Exam 2015

a
h
14. Four of the following five pairs are like in a certain way

b
23. Which of the following is/are required to establish that

o
based on their positions in the above arrangmeent and so W is the daughter of R ?

o
b
form a group. What of the following does not belong to (a) No extra information is required as the relation can

.w
the group ? be established from the given information

o
(a) Teacher-Painter (b) Supervisor-Shopkeeper (b) R has only three children, one son and two daughters

rd
(c) Cricketer-Reporter (d) Doctor-Accountant (c) Q has only the grandson

p
re
(e) Shopkeeper-Doctor (d) Y has only two children, a son and a daughter

s
15. Which one of the following statements is false according (e) Either (b) or (c)

s
.c
to the above mentioned arrangement ?

o
(a) N is to the immediate right of the supervisor DIRECTIONS (Qs. 24-29) : In these questions, relationship

m
(b) The cricketer is third to the right of shopkeeper between different elements is shown in the statements. These
(c) The doctor is sitting exactly between the supervisor statements are followed by two conclusion.
and the shopkeeper Mark answer (a) if only conclusion I follows
(d) L is neither a teacher nor a supervisor Mark answer (b) if only conclusion II follows.
(e) There are only three people between S and N
Mark answer (c) if either conclusion I or II follows
DIRECTIONS (Qs. 16-20) : Study the following information Mark answer (d) if neither conclusion I nor II follwos
and answer the questions that follow : Mark answer (e) if both conclusions I and II follows
24. Statement :
In a certain code language, ‘hope to see you’ is coded as ‘re
so na di’, ‘please come to see the party’ is coded as ‘fi ge na A ³ B £ C, C < D, D > F
di ke zo’, ‘hope to come’ is coded as ‘di so ge’ and ‘see you Conclusions :
the party’ is coded as ‘re fi zo na’. I. F > B II. A > D
16. How is ‘please’ coded in the given code language ? 25. Statement :
(a) di (b) ke X > Y ³ Z, Q = Y, P > X
(c) fi (d) na Conclusions :
(e) None of these I. Z < P II. P > Q
17. What does the code ‘so’ stand for in the given code 26. Statement :
language ? L ³ I, H > I ³ J, K < J
(a) hope (b) come Conslusions :
(c) see (d) to I. H > L II. L > K
(e) None of these 27. Statement :
18. How is ‘party’ coded in the given code language ? O ³ P = Q, R < P, S < Q
(a) Either ‘re’ or ‘fi ’ (b) Either ‘zo’ of ‘na’ Conclusions :
(c) Either ‘zo’ of ‘fi ’ (d) Either ‘zo’ or ‘ge’
I. R < S II. O > S
(e) ‘Either ‘ke’ or ‘fi ’
19. How will ‘please see you’ be coded in the given code 28. Statement :
language ? D ³ E > F = G, E = H < J
(a) re na ke (b) so re na Conclusions :
(c) zo re na (d) na di ke I. J > D II. G < J
(e) ke re ge 29. Statement :
20. Which one of the following will be coded as ‘so di re’ in J ³ R > Z, R > F < W, B > J
the given code language ? Conclusions :
(a) you see hope (b) hope you please I. J > F II. B > W
(c) hope you come (d) the hope to
DIRECTIONS (Qs. 30-35) : K, L, M, P, Q, R, S and T are
(e) you hope to
sitting around a square table in such a way that four of them
DIRECTIONS (Qs. 21-23) : Read the following information sit at four corners of the square while four sit in the middle
carefully and answer the questions which follow : of each of the four sides. The ones who sit at the four corners
P is son of Q. Q is mother of R. R is wife of T. T is father of face outside while those who sit in the middle of the sides face
V. V is brother of W. Y is mother of T. the centre of the table.
21. Which of the following is true based upon the relationships P sits third to the right of S. S faces the centre. Q sits third
given above ? to the left of M. M does not sit in the middle of the sides. Only
(a) W is grand-daughter of Y one person sits between Q and R. R is not an immediate
(b) R is sister of P neighbour of M. T faces the centre. K is not an immediate
(c) V is son of Q neighbour of R.
(d) V is brother-in-law of Y 30. What is position of M with respect to L ?
(e) None is true. (a) Third to the right
22. How is T related to P ? (b) M and L sit diagonally opposite to each other
(a) Son-in-law (b) Brother (c) Second to the right.
(c) Father-in-law (d) Brother-in-law (d) Second to the left
(e) Cannot be determined (e) Fifth to the right
y
o
u
rs
m
IBPS CWE-RRB Officer Scale-I Exam 2015 459

a
h
b
31. Who sits exactly between Q and R ? Conclusions :

o
(a) T (b) P I. Some jungles are tyres.

o
b
(c) K (d) M II. Some tubes are rains.

.w
(e) S and K III. Some jungles are rains.

o
rd
32. Which of the following pairs represents the persons seated IV. Some tubes are flowers.

p
in the middle of the sides who face each other ? (a) Only I, II and III follow

re
(a) S, Q (b) K, L (b) Only II, III and IV follow

s
s
(c) M, P (d) R, T

.c
(c) Only I, III and IV follow

o
(e) T, Q (d) All follow

m
33. Who amongst the following sits between R and K when (e) None of these
counted in anti-clockwise direction from K ? 38. Statements :
(a) No one sits between R and K as R and K are
All desks are chairs.
immediate neighbours of each other
All chairs are tables.
(b) S, P and L (c) P and Q
All tables are boxes.
(d) L and R
All boxes are trunks.
(e) M, S and T
Conclusions :
34. If K is made to face the opposite direction, who would
sit to his immediate right ? I. Some trunks are boxes.
(a) R (b) Q II. All chairs are boxes.
(c) P (d) T III. Some boxes are desks.
(e) S IV. All desks are trunks.
35. Four of the following five are alike in a certain way and (a) Only I, II and III follow
so form a group. Which is the one that does not belong (b) Only I, II and IV follow
to that group ? (c) Only II, III and IV follow
(a) L (b) M (d) All follow
(c) K (d) P (e) None of these
(e) R 39. Statements :
DIRECTIONS (Qs. 36-40) : In each question below are four Some birds are goats.
statements followed by four conclusions numbered I, II, III and Some goats are horses.
IV. You have to take the four given statements to be true even Some horses are lions.
if they seem to be at variance from commonly known facts and Some lions are tigers.
then decide which of the given conclusions logically follows Conclusions :
from the four given statements disregarding commonly known I. Some tigers are goats.
facts. Then decide which of the answers (1), (2), (3), (4) and II. No tiger is goat.
(5) is correct and indicate it on the answersheet. III. Some lions are birds.
36. Statements : IV. No lion is bird.
All belts are rollers. (a) Only either I or II follows
Some rollers are wheels. (b) Only either III or IV follows
All wheels are mats. (c) Only either I or II and either III or IV follow
Some mats are cars. (d) Only I and II follow
Conclusions : (e) None of these
I. Some mats are rollers. 40. Statements :
II. Some mats are belts. All papers are bottles.
III. Some cars are rollers. All bottles are cups.
IV. Some rollers are belts. Some cups are jugs.
(a) Only I and II follow Some jugs are plates.
(b) Only I, III and IV follow Conclusions :
I. Some plates are cups.
(c) Only I and IV follow
II. Some plates are bottles.
(d) Only II, III and IV follow
III. Some cups are papers.
(e) None of these
IV. Some bottles are papers.
37. Statements :
(a) Only III and IV follow
Some tyres are rains. (b) Only I and II follow
Some rains are flowers. (c) Only I and III follow
All flowers are jungles. (d) Only II and IV follow
All jungles are tubes. (e) None of these
y
o
u
rs
m
460 IBPS CWE-RRB Officer Scale-I Exam 2015

a
h
QUANTITATIVE APTITUDE

b
50. 117.95 × 8.017 × 4.98 = ?

o
(a) 4670 (b) 4780

o
b
(c) 4840 (d) 4720

.w
DIRECTIONS (Qs. 41-45) : In each of these questions a number
(e) 4800
series is given. Below the series one number is given followed

o
51. A milkman bought 15 kg of milk and mixed 3 kg of water

rd
by (a), (b), (c), (d) and (e). You have to complete this series
in it. If the price per kg of the mixture becomes ` 22, what

p
following the same logic as in the original series and answer

re
is cost price of the milk per kg?
the question that follows.

s
(a) ` 28.00 (b) ` 26.40

s
.c
41. 5 9 25 91 414 2282.5 (c) ` 24.00 (d) ` 22.00

o
3 (a) (b) (c) (d) (e)

m
(e) None of these
What will come in place of (c)? 52. A person invested some amount at the rate of 12% simple
(a) 63.25 (b) 63.75 interest and the remaining at 10%. He received yearly an
(c) 64.25 (d) 64.75 interest of ` 130. Had he interchanged the amounts
(e) None of these invested, he would have received an interest of ` 134. How
42. 15 9 8 12 36 170 much money did he invest at different rates?
19 (a) (b) (c) (d) (e) (a) ` 500 at the rate of 10%, ` 800 at the rate of 12%
What will come in place of (b)? (b) ` 700 at the rate of 10%, ` 600 at the rate of 12%
(a) 18 (b) 16 (c) ` 800 at the rate of 10%, ` 400 at the rate of 12%
(c) 22 (d) 24
(d) ` 700 at the rate of 10%, ` 500 at the rate of 12%
(e) None of these
(e) None of these
43. 7 6 10 27 104 515
53. The marked price of a machine is ` 18000. By selling it at
9 (a) (b) (c) (d) (e)
a discount of 20%, the loss is 4%. What is the cost price of
What will come in place of (d)?
the machine?
(a) 152 (b) 156
(c) 108 (d) 112 (a) ` 10000 (b) ` 12000
(e) None of these (c) ` 14000 (d) ` 15000
44. 6 16 57 244 1245 7506 (e) None of these
4 (a) (b) (c) (d) (e) 54. A train travels at a speed of 40 km/h and another train at a
What will come in place of (d)? speed of 20 m/s. What is the ratio of speed of the first train
(a) 985 (b) 980 to that of the second train ?
(c) 1004 (d) 1015 (a) 2 : 1 (b) 5 : 9
(e) None of these (c) 5 : 3 (d) 9 : 5
45. 8 9 20 63 256 1285 (e) None of these
5 (a) (b) (c) (d) (e) 55. A is thrice as efficient as B and hence completes a work in
What will come in place of (e)? 40 days less than the number of days taken by B. What
(a) 945 (b) 895 will be the number of days taken by both of them when
(c) 925 (d) 845 working together ?
(e) None of these (a) 22.5 days (b) 15 days
(c) 20 days (d) 18 days
DIRECTIONS (Qs. 46-50) : What approximate value will come (e) None of these
in place of the question mark (?) in the following questions? 56. A person goes to a market between 4 p.m. and 5 p.m. When
(You are not expected to calculate exact value.) he comes back, he finds that the hour hand and minute
46. (32.51)2 – (17.45)2 = ? hand have interchanged their positions. For how much time
(a) 780 (b) 850 (approximately) was he out of his house?
(c) 680 (d) 820 (a) 55.25 minutes (b) 55.30 minutes
(e) 750 (c) 55.34 minutes (d) 55.38 minutes
47. 88.25% of 450 = ? % of 530 (e) None of these
(a) 70 (b) 68 57. There are 48 cricket balls, 72 hockey balls and 84 tennis
(c) 75 (d) 80 balls and they have to be arranged in several rows in such
(e) 65 a way that every row contains the same number balls of
balls of one type. What is the minimum number of rows
48. 898 × (12.005)2 + ? = 5000 required for this to happen ?
(a) 680 (b) 720 (a) 12 (b) 16
(c) 750 (d) 620 (c) 17 (d) 19
(e) 630 (e) None of these
49. 3745 ÷ 24.05 × 17.98 = ? 58. If 1 is subtracted from the numerator of a fraction it becomes
(a) 2860 (b) 2800 (1/3) and if 5 is added to the denominator the fraction
(c) 2760 (d) 2720 becomes (1/4). Which fraction shall result, if 1 is subtracted
(e) 2840 from the numerator and 5 is added to the denominator ?
y
o
u
rs
m
IBPS CWE-RRB Officer Scale-I Exam 2015 461

a
h
b
5 7 (a) ` 28,939.25 (b) ` 29,838.75

o
(a) (b) (c) ` 31,748.25 (d) ` 31,738.75

o
12 23

b
(e) None of these

.w
1 2
(c) (d) 64. Abhishek invested an amount of ` 45,000 for two years

o
8 3 with Company B under Scheme III which offers compound

rd
(e) None of these
interest and Jeevan invested equal amount for two years

p
59. The price of an article is ` 25. After two successive cuts by

re
with Company C under Scheme IV which offers simple
the same percentage, the price becomes ` 20.25. If each

s
interest. Who earned more interest and how much?

s
.c
time the cut was x%, then
(a) Abhishek, ` 1,875 (b) Jeevan, ` 1,875

o
(a) x = 9 (b) x = 10

m
(c) Abhishek, ` 1,962 (d) Jeevan, ` 1,962
(c) x = 11 (d) x = 11.5 (e) None of these
(e) None of these 65. Mr. Lal investesd ` 30,000 in Company A under Scheme
60. A person could save 10% of his income. But 2 years later, II which offers simple interest and ` 48,000 in Company
when his income increased by 20%, he could save the same D under Scheme II which offers compound interest. What
amount only as before. By how much percentage has his will be the total amount of interest earned by Mr. Lal in
expenditure increased? two years?
2 1 (a) ` 14728.80 (b) ` 17428.50
(a) 22 % (b) 23 % (c) ` 14,827.70 (d) ` 16,728.20
9 3
(e) None of these
2 2 DIRECTIONS (Qs. 66-70) : Study the following graph carefully
(c) 24 % (d) 25 %
9 9 to answer the questions.
(e) None of these
Quantity of Various Items Sold and Price Per Kg.
DIRECTIONS (Qs. 61-65) : Study the following table carefully 800 80
to answer the questions. 700 70
Quantity Sold (In kg.)

600 60
Rate of Interest (P.C.P.A) of Fered by Five companies

Price ( ` . per kg.)


500 50
on Deposits Under Different Schemes
400 40
Company A B C D E 30
300
Scheme
200 20
I 8.5 9.0 8.0 8.5 9.0 100 10
II 9.5 8.5 9.0 9.0 8.5 0 0
Wheat Rice Sugar Groundnut Cumin Corriander
III 8.0 8.0 7.5 8.5 8.5 Item
Price
IV 10.0 9.5 10.5 9.5 10.0 Quantity
66. What is the average price per kg of wheat and rice sold?
61. Mr. X deposited an amount in Scheme II with Company C (rounded off to nearest integer)
for two years. After that he withdrew the amount and (a) 32 (b) 34
reinvested only the principal amount in Scheme IV of (c) 36 (d) 35
Company B for two years. Total amount of simple interest (e) 31
accrued from the two schemes is ` 14, 800. What was the 67. If cumin is sold at a 10% discount, the quantity sold
principal amount? doubles. What will be the cost of total quantity of cumin
(a) ` 48,000 (b) ` 42,000 sold on discount?
(c) ` 40,000 (d) Cannot be determined (a) ` 52,600 (b) ` 28,800
(e) None of these (c) ` 32,000 (d) ` 57,600
62. Company E offers compound interest under Scheme I and (e) None of these
Company A offers simple interest under Scheme IV. What 68. Out of the total quantity of sugar sold, half the quantity is
will be the difference between the interest earned under sold at the given rate while half the quantity is sold on a
the Scheme I of Company E and Scheme IV of Company price increased by 20%. What is the total cost of entire
A respectively in two years on an amount of ` 1.2 lakhs? quantity of sugar sold?
(a) ` 1,428 (b) ` 1,328 (a) ` 23,100 (b) ` 23,800
(c) ` 1,528 (d) ` 1,548 (c) ` 22,400 (d) ` 23,400
(e) None of these (e) None of these
63. Company D offers compound interest under Scheme II and 69. If the quantities sold of Groundnut and Cumin are
simple interest under Scheme IV. Abhijit invested ` 25,000 interchanged, what will be the total cost of quantities sold
with this company under Scheme IV and after one year for these two items together?
switched over to Scheme II along with the interest for one (a) ` 62,000 (b) ` 60000
more year. What is the total amount he will get at the end (c) ` 65,000 (d) ` 63,000
of two years? (e) None of these
y
o
u
rs
m
462 IBPS CWE-RRB Officer Scale-I Exam 2015

a
h
b
70. If the quantity of corriander sold is increased by 200% and (C) Many doctors now-a-days use it for therapeutic

o
the price per kg. is reduced by 8%, what will be the total purposes which are based upon the findings of this

o
b
cost of corriander sold? research.

.w
(a) ` 38, 460 (b) ` 36,480 (D) The research has demonstrated that listening to

o
(c) ` 38,640 (d) ` 36,840 classical music increases the rate of nervous

rd
(e) None of these connections in brain.

p
re
(a) BDCA (b) DBAC
DIRECTIONS (Qs. 71-75) : In the following questions two

s
s
(c) ACBD (d) BDAC

.c
equations numbered I and II are given. You have to solve both
(e) BCAD

o
the questions and

m
82. (A) This right however comes along with the responsibility
Give answer (a) if x > y which the press is forced to work with
Give answer (b) if x ³ y (B) It means that the pressmen have every right to expose
Give answer (c) if x < y matters related to public and national interest.
Give answer (d) if x £ y (C) Such responsibilities include not broadcasting matters
Give answer (e) if x = y or the relationship cannot be established related to national security and other sensitive issues.
71. I. x2 + 5x + 6 = 0 (D) Freedom of press is symbolic of democracy.
II. y2 + 3y + 2 = 0 (a) DBCA (b) DBAC
72. I. x2 – 10x + 24 = 0 (c) ABCD (d) BDAC
II. y2 – 9y + 20 = 0 (e) BCDA
73. I. (x)2 = 961 II. y = 961 83. (A) A popular example of this damage is the Taj Mahal
whose marble has been corroded due to acid rain.
74. I. x2 – 72 = x II. y2 = 64
(B) It changes the acidic content of water bodies thus
75. I. x2 – 463 = 321
affecting the lives of the living organisms of this
II. y2 – 421 = 308
habitat.
DIRECTIONS (Qs. 76-80) : What should come in place of (C) Acid rain has been a major factor responsible for the
the questions mark (?) in the following questions? degradation of the environment.
76. 8.5 × (80 × 1.5 × 2.5) ÷ 4 = ? (D) Along with affecting the living beings, it is also
responsible for the corrosion of several heritage
(a) 637.5 (b) 620.5
buildings thus causing irreparable damages to them.
(c) 544.22 (d) 647.5
(a) CBAD (b) CDBA
(e) None of these
(c) BADC (d) BDAC
77. 23% of 2500 - 22% of 500 = 930 ÷ ?
(e) CBDA
(a) 2.5 (b) 2
84. (A) Along with missing their education these children also
(c) 3 (d) 1.5
face life threatening dangers by working in hazardous
(e) None of these
chemical factories.
78. 5312.15 - 1318.82 - 2321.43 = ?
(B) Child labour has been interfering with the education
(a) 1681.9 (b) 1571.9
of millions of children across India.
(c) 2671.7 (d) 1671.9
(C) However, there is a dire need to reinforce these laws
(e) None of these strictly throughout the country.
79. (333.33 + 33.33 + 3333.34) ÷ 50 = ?
(D) Many laws have been framed in order to curb this
(a) 74 (b) 78 evil.
(c) 82 (d) 84 (a) BADC (b) CBAD
(e) None of these (c) DBAC (d) BDAC
80. 3% of 3500 + 5% of 700 = ?% of 1400 (e) BACD
(a) 12% (b) 15% 85. (A) Their invention has been proved to be a boon to the
(c) 10% (d) 8% society since many diseases caused by the microbes
(e) None of these could be cured by these antibiotics.
ENGLISH LANGUAGE (B) The scientists therefore face a continuous challenge
to keep inventing newer drugs to counter this problem.
(C) A major hurdle in the use of antibiotics, however, is
DIRECTIONS (Qs. 81-85) : In each of the following questions, that the microbes develop resistance to the antibiotics
rearrange the given sentences (A), (B), (C) and (D) and indicate thus rendering these ineffective.
the correct sequence in which these sentences must appear so (D) Antibiotics are chemical substances produced by
as to make a meaningful paragraph. microbes which are capable of inhibiting the growth
81. (A) By increasing these connections the intelligence is of other microbes.
also increased. (a) BACD (b) CBAD
(B) Classical music has been proved to be very helpful in (c) DABC (d) DACB
child development. (e) ABDC
y
o
u
rs
m
IBPS CWE-RRB Officer Scale-I Exam 2015 463

a
h
b
DIRECTIONS (Qs. 86-95) : In the following passage there DIRECTIONS (Qs. 96-100) : In each of the following sentences,

o
o
are blanks, each of which has been numbered. These numbers an idiomatic expression or a proverb is highlighted. Select the

b
are printed below the passage and against each, five words/ alternative which best describes its use in the sentence.

.w
phrases are suggested, one of which fits the blank appropriately.

o
96. Although Ravi felt lonely in his fight against the wrong,

rd
Find out the appropriate word/phrase in each case. he decided to take the bull by the horns.

p
re
The barter system for getting goods and services (86) back many (a) Ravi decided to convince others of his stance

s
centuries. In most cultures the barter system was used before (b) Ravi decided to give up his fight

s
.c
money was (87). People who had specific items or services would
(c) Ravi decided to put forward all his courage and

o
(88) these with others for the things they needed. Good

m
strength
negotiation was the (89) to making good trades. While the barter
system (90) based on basic needs, today the barter system (d) Ravi decided to apologize for his actions
continues to thrive. The barter system transcends the monetary (e) None of these
system. The barter system is making a (91) today. What makes 97. As the situation got out of control, the speaker of the
the barter system even better today than ever before is that it can parliament tried to put oil over troubled waters.
now be done globally. In the past bartering was simply done (a) The speaker tried to calm the situation
with those that were located nearby. Today the barter system can (b) The speaker tried to take advantage of the situation
be used in a much more (92) way than ever before yet it carries
(c) The speaker jumped into the fight
with it the same basic motivation -the need for something that
you don’t have and the excess of something that someone else (d) The speaker remained indifferent to the situation
wants. (e) None of these
The barter system is enjoying (93) interest today. Bartering 98. The researcher tried his best to explain the importance of
allows you to get the things you need without having to (94) his new discovery to the panel members but soon realized
additional money. Instead you can use the things you no longer that he was casting pearls before a swine.
need or want to get the things you do need. There are swap (a) He realised that the panel members would take his
markets and online auctions that (95) you to sell or trade your suggestions seriously
items or to purchase items that you want. Negotiation takes place (b) He realised that he disclosed the information to a
just like it did hundreds of years ago. greedy panel
86. (a) discovered (b) dates
(c) He realised that he had come to the right group of
(c) began (d) started
people
(e) initiated
(d) He realised that the panel members can not appreciate
87. (a) bought (b) imagined
his findings
(c) began (d) emerged
(e) invented (e) None of these
88. (a) buy (b) sell 99. The chairman had to eat a humble pie after the workers
(c) exchange (d) give decided to go on strike to protest against the biased
(e) return promotions.
89. (a) important (b) essential (a) The chairman politely refused to take the promotion
(c) result (d) key orders back
(e) intention (b) The chairman gave up his pride and apologized for
90. (a) originated (b) stood the mistake
(c) generated (d) created (c) The chairman had to face defeat by a group of ordinary
(e) produced workers
91. (a) issue (b) comeback (d) The chairman’s career was tinted due to the allegations
(c) withdrawal (d) recall of biased promotions
(e) fading (e) None of these
92. (a) primitive (b) appreciated 100. The bankrupt organisation has been handed over to its new
(c) promoted (d) sophisticated owners, lock, stock and barrels.
(e) better (a) Only a few departments of the bankrupt organisations
93. (a) diminishing (b) revival have been taken over by the new owners
(c) perishing (d) declining
(b) The new owners have bought the bankrupt
(e) renewed
organisation for a very nominal amount
94. (a) expend (b) exchanging
(c) The owners have bo ught the bankrupt organiszation
(c) expand (d) consume
in a hurry
(e) cost
95. (a) insist (b) force (d) The new owners have completely taken over the
(c) allow (d) lure bankrupt organisation
(e) constraint (e) None of these
y
o
u
rs
m
464 IBPS CWE-RRB Officer Scale-I Exam 2015

a
h
b
DIRECTIONS (Qs. 101-105) : Pick out the most effective pair of radically gives rise to a culture where even the smallest change is

o
heralded as a break through.

o
words from the given pair of words to make the sentence /

b
Indian corporate leaders have done well standing up to

.w
sentences meaningfully complete.
global giants as their companies have grown in size and market

o
101. Many people take spirituality very seriously and ______ share. To be successful in international markets they need to be

rd
about those who don't, worrying about them and ______ distinct-distinct products, processes, technologies, business

p
re
them to believe. models and ogranisations. The bottom line will be lnnovation.

s
(a) think, criticising Creativity workshops are organized to channel people to think

s
.c
(b) pride, appraising differently. There are fantastic ideas being generated all the time

o
(c) rationalise, enabling but no industry breakthrough. Simply because of gravity-a

m
(d) wonder, prodding regressive force exerted by a mindset.
(e) ponder, venturing Thinking has therefore to happen at three levels : idea,
102. Unless new reserves are found soon, the world's supply of frame and paradigm. From a narrow focus on either product or
coal is being ______ in such a way that with demand process innovation organizations need to look at innovating the
continuing to grow at present rates, reserves will be ______ whole ecosystem of the organization. Many a time waiting for a
by the year 2050 hundred percent solution before going to the market the
(a) consumed, completed oranization forgets that it could end up waiting forever. Moreover
(b) depleted, exhausted sometimes organizations are too focused on today to see
(c) reduced, argument tomorrow. Since management mandates are short-term, sowing
(d) burnt, destroyed the seed for a revenue stream today and leaving its benefits to be
(e) utilized, perished reaped by a successor doesn't appeal to today's business leader.
103. If you are a ______ you tend to respond to stressful This is a serious hurdle to innovation. Establishing a function
situations, in a calm, secure, steady and ______ way. called innovation management or training employees through
(a) resilient, rational creativity workshops will have few benefits unless each frontline
(b) obdurate, manageable employee is empowered to share his innovative ideas with
(c) propitious, stable management. What happens to this system when the person
(d) delectable, flexible driving the change leaves the organization ? The approach to
(e) supportive, positive innovation hence needs to be system driven rather than people
104. Management can be defined as the process of ______ driven. In thirty years India can be the largest world economy
organizational goals by working with and through human save China and the US. However as companies grow there exists
and non-human resources to ______ improve value added a resemblance in their products, services,promotions, processes
to the world. and pricing and so on. There remains only one escape from this
(a) getting, deliberately trap.
(b) managing, purposefully 106. The main idea of the passage is :
(c) targeting, critically (a) India has tremendous potential
(d) realizing, dialectically (b) Indian companies need to expand to international
(e) reaching, continuously markets
105. If you are an introvert, you ______ to prefer working alone (c) the more things change the more they temain the
and, if possible, you will ______ towards projects where same.
you can work by yourself or with as few people as possible. (d) creative thinking is the key to india's success
(a) like, depart
(e) None of these
(b) advocate, move
107. Which of the following is true in the context of the passage?
(c) tend, gravitate
(d) express, attract (a) Following tried and tested methods will enable a
(e) feel, follow company to continue being successful globally
(b) A crisis is the optimal time for a company to make
DIRECTIONS (Qs. 106-110) : Read the following passage changes
carefully and answer the questions given below. Certain words (c) Generating new opportunities for the future should
in the passage have been printed in bold to help you locate be the focus of innovation
them when answering some of the questions. (d) Empowering employees to voice their opinions causes
Can the last fifteen years be called the most successful decade disharmony in the company
and a half in Indian history and will the next fifteen be equally (e) Innovation management is the key to growth for
successful ? Consider a culture where independent thinking is Indian companies
not encouraged. Or take the example of traditional family run 108. Which of the following is a cause for 'gravity' ?
business with vast resistance to change or a whole nation who (A) Mindset (B) Socialization
believes that breakthrough ideas can be generated abroad but (C) Limited focus
never at home. Partly responsible is socialization from early years (a) Only (A) (b) Only (B)
we are taught not to question our elders but at workplaces this (c) Only (C) (d) All of these
creates a hurdle for new thinking. Being unable to change (e) Both (A) and (B)
y
o
u
rs
m
IBPS CWE-RRB Officer Scale-I Exam 2015 465

a
h
b
109. Which of the following is NOT TRUE in the context of the (a) (F) (b) (D)

o
passage ? (c) (E) (d) (A)

o
b
(a) A change in mindset can help promot innovation (e) (C)

.w
(b) The author believes India has tremendous potential 117. Which is THIRD sentence of the passage ?

o
(c) India suffers because of the belief the breakthrough (a) (A) (b) (D)

rd
ideas can only be generated abroad (c) (B) (d) (C)

p
re
(d) Innovation automatically happens if a company (e) (E)

s
subscribes to the latest management strategies 118. Which is the SECOND sentence of the passage ?

s
.c
(e) All are true (a) (B) (b) (D)

o
(c) (F) (d) (C)

m
110. An appropriate title for the passage would be
(a) Industry Problems (e) (E)
(b) Creativity : Boon or Curse 119. Which is the LAST (SIXTH) sentence of the pasage ?
(c) What Ails Innovation (a) (C) (b) (B)
(d) India : Next Superpower ? (c) (F) (d) (D)
(e) None of these (e) (E)
120. Which is the FIRST sentence of the passage ?
DIRECTIONS (Qs. 111-113) : Pick out the word which is closest (a) (A) (b) (D)
in meaning to the word printed in bold as used in the passage. (c) (C) (d) (B)
111. Paradigm (e) (E)
(a) category (b) difficulty
(c) hypothesis (d) foundation COMPUTER APTITUDE
(e) None of these
112. Distinct 121. Which of the following operating system does not
(a) infrequent (b) abundant implement the multitasking truly?
(c) complex (d) cautious (a) Windows 98 (b) Windows NT
(e) None of these (c) Windows XP (d) MS DOS
113. Heralded (e) None of these
(a) chastised (b) appealed
122. Which of the following windows version support 64 bit
(c) challenged (d) proclaimed
processor?
(e) None of these
(a) Windows 98 (b) Windows 2000
DIRECTIONS (Qs. 114-115) : Pick out the word which is most (c) Windows XP (d) Windows 95
opposite in meaning to the word printed in bold as used in the (e) None of these
passage. 123. Which of the following Operating System does not
114. Regressive implement multitasking truly?
(a) customary (b) aggravating (a) Windows 98 (b) Windows NT
(c) uniform (d) natural (c) Windows XP (d) MS DOS
(e) advanced (e) None of these
115. Vast
124. What program runs first after computer is booted and
(a) diverse (b) deliberate
(c) minute (d) valid loading GUI?
(e) extensive (a) Desktop Manager (b) File Manager
(c) Windows Explorer (d) Authentication
DIRECTIONS (Qs. 116-120) : Rearrange the following six (e) None of these
sentences (A), (B), (C), (D), (E) and (F) in the proper sequence
125. Which of the following operating system do you choose to
to form a meaningful paragraph : then answer the questions given
implement a client server network?
below them.
(a) MS DOS (b) Windows
(A) Development of drought resistance could benefit large (c) Windows 98 (d) Windows 2000
numbers of farmers.
(e) None of these
(B) Hence the human race has no choice but to adapt to these
126. My Computer was introduced from
impacts.
(a) Windows 3.1 (b) Windows 3.11
(C) India has to be concerned about climatic changes.
(c) Windows 95 (d) Windows 98
(D) This impact can run into decades and centuries.
(e) None of these
(E) Environment day is thus an important occasion to assess
127. Which of the following does not support more than one
the past and our future.
program at a time?
(F) Since there is a possibility of adverse impact on agriculture
(a) DOS (b) Linux
which could deter growth.
(c) Windows (d) Unix
116. Which is the FIFTH sentence of the passage after the
rearrangement ? (e) None of these
y
o
u
rs
m
466 IBPS CWE-RRB Officer Scale-I Exam 2015

a
h
b
128. Which of the following is not an operating system? 138. Which of the following is not a font style?

o
(a) Bold (b) Italics

o
(a) DOS (b) Linux

b
(c) Regular (d) Superscript

.w
(c) Windows (d) Oracle
(e) None of these

o
(e) None of these

rd
129. Which of the following is not a multitasking operating 139. Which of the following terms is used for the from in which

p
the word will appear on the page?

re
system?

s
(a) Windows (b) Linux (a) Text formatting (b) Character formatting

s
.c
(c) Win NT (d) DOS (c) Point size (d) Type face

o
(e) None of these

m
(e) None of these
130. You should choose Sleep option when 140. A(n) _____ allows you to access your e-mail from
anywhere.
(a) The computer is tired after working for the whole day
(a) forum (b) webmail interface
(b) You are leaving for a very short time and want to
(c) message board (d) weblog
resume you work shortly
(e) None of these
(c) When computer gets hanged frequently. Let it sleep
141. The following are included in the world programme ______
for some time
Title Bar, Ribbon, Status Bar, Views, Document workspace.
(d) You finish working and going to bed
(a) Window (b) Browser
(e) None of these (c) Explorer (d) Website
131. Background color or effects applied on a document is not (e) None of these
visible in
142. Your business has contracted with another company to have
(a) Web layout view (b) Print Layout view them host and run an application for your company over
(c) Reading View (d) Print Preview the Internet. The company providing this service to your
(e) None of these business is called an
132. What is a portion of a document in which you set certain (a) Internet service provider
page formatting options? (b) Internet access provider
(a) Page (b) Document (c) Application service provider
(c) Section (d) Page Setup (d) Outsource agency
(e) None of these (e) None of these
133. Borders can be applied to 143. Which of the following would you find on Linkedin?
(a) Cells (b) Paragraph (a) Games (b) Connections
(c) Text (d) All of above (c) Chat (d) Applications
(e) None of these (e) None of these
134. Which of the following is not a type of page margin? 144. ________ is a technique that is used to send more than
(a) Left (b) Right one call over a single line.
(c) Center (d) Top (a) Digital transmission (b) Infrared transmission
(e) None of these (c) Digitizing (d) Multiplexing
(e) None of these
135. What is gutter margin?
145. The search companion can
(a) Margin that is added to the left margin when printing
(a) locate all files containing a specified phrase
(b) Margin that is added to right margin when printing
(b) restrict its search to a specified set of folders
(c) Margin that is added to the binding side of page when
(c) locate all files containing a specified phrase and
printing
restrict its search to a specified set of folders
(d) Margin that is added to the outside of the page when
(d) cannot locate all files containing a specified phrase
printing
or restrict its search to a specified set of folders
(e) None of these (e) None of these
136. Portrait and Landscape are 146. Which of the following cannot be part of an e-mail address?
(a) Page Orientation (b) Paper Size (a) Period (.) (b) At sign (@)
(c) Page Layout (d) All of above (c) Space() (d) Underscore (_)
(e) None of these (e) None of these
137. If you need to change the typeface of a document, which 147. Which of the following must be contained in a URL?
menu will you choose? (a) A protocol identifier (b) The letters www.
(a) Edit (b) View (c) The unique registered domain name
(c) Format (d) Tools (d) www.and the unique registered domain name
(e) None of these (e) None of these
y
o
u
rs
m
IBPS CWE-RRB Officer Scale-I Exam 2015 467

a
h
b
148. Which of the following information systems focuses on 157. What is the difference between a CD-ROM and a CD-RW?

o
making manufacturing processes more efficient and of (a) They are the same just two different terms used by

o
b
higher quality? different manufacturers

.w
(a) Computer aided manufacturing (b) A CD-ROM can be written to and a CD-RW cannot

o
rd
(b) Computer integrated manufacturing (c) A CD-RW can be written to, but a CD-ROM can only

p
(c) Computer aided software engineering be read from

re
(d) Computer aided system engineering (d) A CD-ROM holds more information than a CD-RW

s
s
(e) None of these

.c
(e) None of these

o
149. A mistake in an algorithm that causes incorrect results is 158. What is the term for how words will appear on a page?

m
called a
(a) Text formatting (b) Character formatting
(a) logical error (b) syntax error
(c) Point size (d) Typeface
(c) procedural error (d) compiler error
(e) None of these (e) None of these
150. A device for changing the connection on a connector to a 159. The process of a computer receiving information from a
different configuration is server on the Internet is known as
(a) a converter (b) a competent (a) Pulling (b) Pushing
(c) an attachment (d) an adapter (c) Downloading (d) Transfering
(e) None of these (e) None of these
151. To navigate to a new web page for which you know the 160. _______ can be done through Digital Banking ?
URL, type that URL in the browser's ______ and press Enter (a) Mobile Phone (b) Internet
(a) Address bar (b) Domain bar (c) Telephone (d) All of the above
(c) Address button (d) Name button (e) None of these
(e) None of these
152. The CPU also called the ______ when talking about PCs, GENERAL AWARENESS
does the vast majority of the processing for a computer.
(a) Macroprocessor (b) RAM 161. Which day is observed as World Vegetarian Day?
(c) Memory system (d) microprocessors (a) 1st November (b) 1st October
(c) 1st December (d) 10th October
(e) None of these
(e) None of these
153. A computer's type, processor and operating system define
its ______ 162. Punjab National Bank is an Indian Financial Services
company based in which city?
(a) Brand (b) Size
(a) New Delhi (b) Mumbai
(c) Platform (d) Speed
(c) Chandigarh (d) Jaipur
(e) None of these
(e) None of these
154. A kiosk
163. Which bank launched “Dhanchayat” an Educational film,
(a) is data organised and presented in a manner that has
to raise awareness on the dangers of borrowing money from
additional value beyond t he value of the data itself
unorganized sources?
(b) combines microscopic electronic components on a
(a) ICICI (b) SBI
single integrated circuit that processes bits according
to software instructions (c) HDFC (d) PNB
(c) is a computer station that provides the public with (e) None of these
specific and useful information and service 164. What does stand for “S” in RTGS?
(d) describes a computer's type, processor, and operating (a) Security (b) Services
system (c) Settlement (d) System
(e) None of these (e) None of these
155. The part of the CPU that accesses and decodes programme 165. Who is the brand ambassador of Renault?
instructions, and coordinates the flow of data among (a) Ranbir Singh (b) Aamir Khan
various system components is the (c) Salman Khan (d) Ranbir Kapoor
(a) ALU (b) Control unit (e) None of these
(c) Megabertz (d) Motherboard 166. Which sport does PV Sindhu relates to?
(e) None of these (a) Tennis (b) Badminton
156. Computer programs are written in a high level (c) Polo (d) Squash
programming language, however, the human readable (e) None of these
version of a program is called 167. DICGC, Act is related to which sector?
(a) cache (b) instruction set (a) Banking (b) Agricultural
(c) source code (d) word size (c) Industrial (d) Sport
(e) None of these (e) None of these
y
o
u
rs
m
468 IBPS CWE-RRB Officer Scale-I Exam 2015

a
h
b
168. World Information Technology Summit 2018 is going to be (a) Kolkata (b) Siliguri

o
held in (c) Goa (d) Benguluru

o
b
(a) New Delhi (b) Mumbai (e) None of these

.w
(c) Bangaluru (d) Hyderbad 180. Head Quarters of UNESCO is located in?

o
rd
(e) None of these (a) Geneva (b) New York

p
169. Which state is India’s largest state by area? (c) Paris (d) London

re
s
(a) Madhya Pradesh (b) Uttar Pradesh (e) None of these

s
.c
(c) Rajasthan (d) Maharashtra 181. Ujani Dam is located on which river?

o
m
(e) None of these (a) Pravara River (b) Bhima River
170. Which is the first Asian country to become an Associate (c) Mutha River (d) Walayar River
Member state of CERN as India’s request for membership (e) None of these
is still pending? 182. What is the currency of Argentina?
(a) Sri Lanka (b) Bangladesh (a) Real (b) Peso
(c) Pakistan (d) Nepal (c) Taka (d) Euro
(e) None of these (e) None of these
171. What is the percentage of RRB sponsor bank contribution? 183. One Night at the call center was written by?
(a) 75% (b) 35% (a) Arundhati Roy
(c) 45% (d) 50% (b) R.K. Narayanan
(e) None of these (c) Rabindranath Tagore
172. KVP is a fixed income, long term and risk free government- (d) Chetan Bhagat
run product. The minimum investment amount required (e) None of these
to start with is `1,000. What is the maximum limit? 184. Which day is celebrated as Anti terrorism day?
(a) `20,000 (b) `50,000 (a) May 27 (b) May 21
(c) `1,00,000 (d) No maximum limit (c) May 24 (d) May 26
(e) None of these (e) None of these
173. What is the currency of Thailand? 185. SAARC headoffice is located in ?
(a) Peso (b) Taka (a) Newyork (b) Kathamandu
(c) Euro (d) Thai Baht (c) Geneva (d) New Delhi
(e) None of these (e) None of these
174. Jerusalem is the capital of ____ 186. National Maritime day is celebrated on?
(a) Israel (b) Italy (a) April 7th (b) April 5th
(c) Iran (d) Iraq (c) April 6th (d) April 8th
(e) None of these (e) None of these
175. Vijender Singh belongs to ? 187. Winter Olympics 2018 is going to be held in ?
(a) Boxing (b) Shooting (a) Russia (b) Brazil
(c) Cricket (d) Badminton (c) South Korea (d) Japan
(e) None of these (e) None of these
176. Who is the chairman of SEBI? 188. Big Apple is the nick name for which city?
(a) Rajeev Sinha (b) Anand Sinha (a) Los Angeles (a) New York
(c) U.K. Sinha (d) Rajendra Sinha (a) Portland (a) Dallas
(e) None of these (e) None of these
177. Guravayur Temple is located in?
189. Bank to every Indian is a tag line for which bank?
(a) Guwahati (b) Delhi
(a) ICICI Bank (b) SBI
(c) Bhubaneswar (d) Kerala
(c) IOB (d) HDFC Bank
(e) None of these
(e) None of these
178. Kumari Selja belongs to ?
190. Which Temple complex will be renovated by Indian
(a) Minister of Commerce and Industry
assistance in Nepal?
(b) Minister of Social Justice and Empowerment
(a) Shiva Temple
(c) Minister of External Affairs
(b) Pashupatinath Temple
(d) Minister of Rural Development
(c) Honuman Temple
(e) None of these
(d) Dakshinaawatar Temple
179. Netaji Subhash Chandra Bose International Airport is
(e) None of these
located in ?
y
o
u
rs
m
IBPS CWE-RRB Officer Scale-I Exam 2015 469

a
h
b
191. The form ‘ITR-1’ is a form for using (d) Allocation of more funds to provide education to

o
(a) Income Tax Challan children who are 6–14 years of age

o
b
(b) Excise Duty (e) None of these

.w
(c) Customs Duty 196. ‘Green House Gases’ are responsible mainly for —

o
rd
(a) Growth of Livestock population
(d) Income Tax Return

p
(b) Volume of foreign trade

re
(e) None of these
(c) Quality of agro products in a country

s
192. In economic terms, the total market value of all final goods

s
(d) Industrial Development of a nation

.c
and services produced in a country in a given year is

o
(e) Global warming

m
known as –––––
197. ‘IAEA’ is an organization which functions in the area of
(a) GNI (b) GDP
(a) Prevention of Money Laundering
(c) Inflation (d) PPP
(b) Control of Foreign Direct Investment
(e) Wealth of a nation
(c) Nuclear Energy
193. Which of the following is one of the indicators of Human
(d) World Trade
Development Index (HDI) ?
(e) Promoting International understanding among the
(a) Life expectancy at birth
nations.
(b) Total cost of the agricultural land with a family
198. Which of the following organisations is established
(c) Total area of the agricultural land with a family
mainly to promote Micro, Small and Medium industrial
(d) Availability of perennial source of water for irrigation
sector in India ?
(e) Nature of employment, casual/permanent or
(a) NABARD (b) SIDBI
semipermanent
(c) IDBI Bank (d) EXIM Bank
194. In India, the National Income is measured on the basis of
(e) All of these

(a) Flow of goods only 199. Which of the following Acts helps a bank in its day to
(b) Number of employed youth only day activities ?
(c) Volume of tax collected per annum (a) Competition Act
(d) Earning of people working in PSUs and Government (b) Negotiable Instruments Ac
jobs (c) Hindu Marriage Act
(e) All of these (d) Hindu Succession Act
195. Which one of the following a measure of Fiscal Reforms, (e) NRI Act
has Government of India adopted ? 200. What does the letter ‘M’ denote in the abbreviated term
(a) Bringing down fiscal deficit to a particular level by ‘MRTP’, an act which was applicable in India till recently?
an Act of Law (a) Money (b) Moral
(b) Fixing higher rate of growth (c) Minimum (d) Monopolies
(c) Allocation of more funds to job oriented schemes (e) Market
y
o
u
rs
m
470 IBPS CWE-RRB Officer Scale-I Exam 2015

a
h
b
o
o
b
.w
o
P 11. (a) O is a reporter.

rd
1. (d) L A N E T
12. (a) S is third to the right of R.

p
re
N E T P L A 13. (d) Four persons - R, O, T and S.

s
s
L M N O P 14. (e) Shopkeeper and Doctor are immediate neighbours.

.c
15. (c) The doctor is sitting exactly between the Supervisor

o
2. (e) W O R T H Y

m
and Shopkeeper.
H O R T W Y (16 - 20)
you re
hope to see so na di
3. (b) 18 5 1 19 15 14
R E A S O N please come to see the party fi ge na di ke zo
4. (b) Pen is used to write something. Similarly, stitching
is done by needle. hope to come di so ge
5. (a) 1 2 3 4 5 6 7 8 9 10
C O N F L I C T E D See you the party re fi zo na
Meaningful words Þ Note, Tone
6. (d) P=A<R<MÞP< M Codes are :
7. (b) Kevin’s rank from the top Þ 16 + 7 = 23rd hope Þ so please Þ ke
Kevin’s rank from the bottom Þ 42 – 23 + 1 = 20th to Þ di come Þ ge
+3 –1 see Þ na the Þ fi or zo
8. (d) L ¾¾® B ¾¾® T
you Þ re party Þ fi or zo
+3 –1
S ¾¾® P ¾¾® A 16. (b) please Þ ke
+3 –1
17. (a) so Þ hope
I ¾¾® J ¾¾® F 18. (c) party Þ fi/zo
+2 –1 19. (a) please Þ ke; see Þ na: you Þ re
P ¾¾® I ¾¾® D
20. (e) so Þ hope; di Þ to: re Þ you
+3 –1
B ¾¾® M ¾¾® S (21-23) Q(–) Y(–)
So the term PID does not follow the pattern.
+2 +2 P R(–) T(+)
9. (d) C ¾¾® R A ¾¾® D (+)

+2 +2
T ¾¾® Q I ¾¾® F V (+) W
Similarly, (?)
+2 21. (b) The sex of W is not given.
S ¾¾® A R is sister of P.
+2 V is grandson of Q.
P ¾¾® I
10. (c) So AI follows same pattern. V is grandson of Y.
22. (d) T is brother-in-law of P.
+1 +2 +3 +4
C ¾¾® L ¾¾® T ¾¾® S ¾¾® D 23. (b) If R has one son and two daughters, then W is the
+2 +3 +4 +5 daughter of R.
R ¾¾® B ¾¾® M ¾¾® I ¾¾® G 24. (d) A³B£C<D>F
(11 - 15): Sitting arrangement Conclusions
Supervisor I. F > B : Not True
Doctor M II. A > D : Not True
Accountant
N L 25. (e) P> X>Y= Q> Z
Conclusions
Sh

I. Z < P : True
P Painter
op

R II. P > Q : True


kee

26. (e) L > I; H > I ³ J > K


per

S Conclusions
O
Reporter Cricketer I. H > L : True
T II. L > K : True
Teacher
y
o
u
rs
m
IBPS CWE-RRB Officer Scale-I Exam 2015 471

a
h
b
27. (b) O>P=Q> R: O> P= Q> S Conclusions : I. (False)

o
Conclusions II. (True)

o
b
I. R < S : Not True III. (True)

.w
II. O > S : True IV. (True)

o
28. (b) D > E = H > F = G 38. (d)

rd
Trunks
D> E =H< J

p
Boxes

re
Conclusions Tables

s
I. J > D : Not True

s
Chairs

.c
II. G < J : True Desks

o
m
29. (a) B > J > R > Z
B> J> R>F<W
Conclusions
I. J > F : True Conclusions : I. (True)
II. B > W : Not True II. (True)
(30 - 35) : K S M III. (True)
IV. (True)
So all conclusions I, II, III and IV follow.
Q T

Tigers
Horse

Lions
39. (c)

ats
s
rd
Go
Bi
P R L Conclusions :
30. (d) M is second to the left of L×2.
31. (b) P sits exactly between Q and R. I. (False)
Either
32. (e) Q faces T and both are sitting in the middle of the sides. II. (False) Complementary Pair
III. (False)
33. (c) P and Q. Either
IV. (False)
34. (b) Q would sit to the immediate right of K.
35. (e) Except R, all others are seated at the corners. Cups
40. (a)
Bottles
36. (c) Rollers
rs

Jugs Plates
pe

Mats
Pa

Belts
els Cars
Whe
Or

Or Cups
Bottles
rs

Jugs
pe

Rollers
Pa

Mats Plates
Belts
els Cars
Whe Conclusions : I. (False)
II. (False)
III. (True) Conversion of the first
Conclusions : I. (True)
conclusion
II. (False)
IV. (True) Conversion of the first
III. (False)
premise
IV. (True)
41. (d) The pattern of the given series is :
So only conclusion I and IV follow.
5 × 1.5 + 1.5 = 7.5 + 1.5 = 9
37. (b) Tubes
9 × 2.5 + 2.5 = 22.5 + 2.5 = 25
Jungles
25 × 3.5 + 3.5 = 87.5 + 3.5 = 91
s
er
ow

Rains Tyres 91 × 4.5 + 4.5 = 409.5 + 4.5


Fl

= 414
Similarly,
Or (a) Þ 3 × 1.5 + 1.5
Tubes = 4.5 + 1.5 = 6
Jungles (b) Þ 6 × 2.5 + 2.5
s
er

= 15 + 2.5 = 17.5
ow

Rains
Fl

(c) Þ 17.5 × 3.5 + 3.5


Tyre
s = 61.25 + 3.5 = 64.75
y
o
u
rs
m
472 IBPS CWE-RRB Officer Scale-I Exam 2015

a
h
50. (d) ? » 118 × 8 × 5 » 4720

b
42. (b) The pattern of the given series is:

o
15 × 1 – 1 × 6 = 15 – 6 = 9 51. (b) Let cost price of milk ` x per kg.

o
b
9 × 2 – 2 × 5 = 18 – 10 = 8 Price of 15kg of milk = ` 15x.

.w
8 × 3 – 3 × 4 = 24 – 12 = 12 Now, mix 3kg of water, therefore quantity of mixture

o
= (15 + 3) kg = 18 kg

rd
12 × 4 – 4 × 3 = 48 – 12 = 36
So, price of mixture is `22 per kg

p
36 × 5 – 5 × 2 = 180 –10 = 170

re
Similarly, According to question.

s
15x = 22 × 18

s
(a) Þ 19 × 1 – 1 × 6 = 19 – 6 = 13

.c
22 ´ 18

o
(b) Þ 13 × 2 – 2 × 5 = 26 – 10 = 16 x=

m
43. (a) The pattern of the given series is : 15
7 ×1 – 1= 6 132
6 × 2 – 2 = 10 = = 26.40
5
10 × 3 – 3 = 27
27 × 4 – 4 = 104 Alternate Method :
Let CP of milk be ` x per kg.
104 × 5 – 5 = 515
By Alligation method
Similarly, price of milk price of water
(a) Þ 9 × 1 – 1 = 8 x 0
(b) Þ 8 × 2 – 2 = 14
(c) Þ 14 × 3 – 3 = 39 22
(c) Þ 39 × 4 – 4 = 152 22 x – 22
44. (e) The pattern of the given series is:
6 × 2 + 22 = 12 + 4 = 16 \ 22 : (x – 22) = 15 : 3
16 × 3 + 32 = 48 + 9 = 57 22 15
57 × 4 + 42 = 228 + 16 = 244 Þ =
x - 22 3
Similarly,
(a) Þ 4 × 2 + 22 = 8 + 4 = 12 22
Þ = 5
(b) Þ 12 × 3 + 32 = 36 + 9 = 45 x - 22
(c) Þ 45 × 4 + 42 = 180 + 16 = 196 Þ 22 = 5x – 110
Þ 22 = 132
(d) Þ 196 × 5 + 52 = 980 + 25 = 1005
\ x = ` 26.40
45. (c) The pattern of the given series is: 52. (d) Let the person invest ` x and y at two different rates
8 ×1 + 1= 9 63 × 4 + 4 = 256 12% and 14% respectively.
9 × 2 + 2 = 20
20 × 3 + 3 = 63 x ´12 ´ 1 y ´10 ´ 1 æ P ´ R ´T ö
\ + = 130 çQ SI = ÷
Similarly, (a) Þ 5 × 1 + 1 = 6 100 100 è 100 ø
(b) Þ 6 × 2 + 2 = 14 Þ 12x + 10y = 13000 ...(i)
(c) Þ 14 × 3 + 3 = 45 After inter changing invested amount.
(d) Þ 45 × 4 + 4 = 184 y ´ 12 ´1 x ´10 ´1
+ = 134
(e) Þ 184 × 5 + 5 = 925 100 100
46. (e) ? = (32.51 + 17.45) (32.51 – 17.45) Þ 12y + 10x = 13400 ...(ii)
= 49.96 × 15.06 On solving equations (i) and (ii), we get
» 50 × 15 » 750 x = ` 500 and y = ` 700
53. (d) Given marked price of machine = ` 18000
450 ´ 88 530 ´ ?
47. (c) » 20
100 100 \ Discount = × 18000 = ` 3600
100
450 ´ 88 \ SP = 18000 – 3600 = ` 14400
Þ?» » 75
530 If loss of 4%, then

48. (a) 900 ´ (12)2 + ? » 5000 100 ´ SP 100 ´ 14400


CP = =
100 - r 100 - 4
[898 » 900; 12.005 » 12]
Þ 30 × 144 + ? » 5000 100 ´ 14400
Þ ? » 5000 – 4320 » 680 = = ` 15000
96
3745 54. (b) Speed of a train = 40 km/h
49. (b) ?» ´ 18 » 2808.75
24 5
= 40 ´ m/s
\ Required answer = 2800 18
y
o
u
rs
m
IBPS CWE-RRB Officer Scale-I Exam 2015 473

a
h
60. (a) Let income be ` 100

b
Speed of another train = 20 m/s

o
90

o
Speed of first train
Expenditure amount = 100 ´

b
\ Required ratio = = ` 90
100

.w
Speed of second train

o
120
Now, income increased by 20% = 100 ´

rd
5 = ` 120
40 ´ 10 5 100
18 = 2 ´ 5 =

p
= = or 5 : 9

re
18 9 Expenditure amount = (120 – 10) = ` 110
20 18

s
Increase in expenditure = 110 – 90 = ` 20

s
55. (b) Let efficiency of B is x

.c
20

o
Increase in % of expenditure = ´ 100

m
90
x 3x 200 2
Days = = 22 %
9 9
A B 61. (c) Let the principal be ` x, Interest accrued from
3x x Efficiency
x ´ 2´9 9x
Days are inversaly proportional to efficiency. company C = = `
100 50
According to question Interest accrued from company
Þ 3x – x = 40
Þ 2x = 40 9.5 x
B= `
\ x = 20 50
Time taken by A = x days = 20 days
9.5 x 9 x
Time taken by B = 3x days = 3 × 20 = 60 days \ + = 14800
50 50
20 ´ 60 Þ 18.5x = 14800 × 50
Time taken by A and B together = = 15 days
80
14800 ´ 50
56. (d) Let us assume that he was out of house for ‘t’ min. Þ x= = ` 40000
18.5
So angle formed by min. hand = 6 × t
Angle formed by hour hand = 0.5 × t 62. (a) CI earned under the scheme I of company E
Now, 0.5 × t + 6 × t = 360 éæ T ù éæ 2 ù
R ö 9 ö
Þ 6.5 t = 360 = P êç 1 + ÷ –1ú = 120000 êç1 + ÷ – 1ú
êëè 100 ø úû êëè 100 ø úû
360
t= = 55.38 min = 120000 × 0.1881 = ` 22572
6.5
CI earned under the scheme IV of the company A
57. (c) HCF of 48, 72 and 84 = 22 × 3 = 12
120000 ´ 2 ´ 10
48 72 84 = = ` 24000
minimum number of rows = + + 100
12 12 12
Difference
= 4 + 6 + 7 = 17
= ` (24000 – 22572) = ` 1428
58. (c) Let the numerator and denominator of a fraction are
63. (b) SI earned under the scheme IV of company D
x and y, respectively,
According to question, 25000 ´ 9.5
= = ` 2375
x -1 1 100
= Þ 3 x - 3 = y Þ 3 x - y = 3...(i ) Amount = ` (25000 + 2375) = ` 27375
y 3
Total amount under scheme II
x 1
and = Þ 4x - y = 5 ...(ii ) æ 9 ö æ 27375 ´ 109 ö
y+5 4 = ` 27375 ç 1 + ÷ = ` ç ÷
è 100 ø è 100 ø
On solving eqs. (i) and (ii), we get
= ` 29838.75
x = 2 and y = 3
éæ 8 ö
2 ù
x -1 2 -1 1 64. (d) CI got by Abhishek = 45000 êç1 + ÷ – 1ú
\ Required fraction = = = êëè 100 ø úû
y +5 3+5 8
59. (b) According to the question, = ` (45000 × 0.1664) = ` 7488
Interest got by Jeevan
æ 100 - x öæ 100 - x ö
Þ 25 ´ ç ÷ç ÷ = 20.25
è 100 øè 100 ø æ 45000 ´ 10.5 ´ 2 ö
=` ç ÷ = ` 9450
202500 è 100 ø
Þ (100 - x )2 = Þ (100 - x ) = 8100
2
\ Difference
25
= ` (9450 – 7488) = ` 1962
Þ 100 - x = 90 Jeevan earned more interest.
\ x = 10
y
o
u
rs
m
474 IBPS CWE-RRB Officer Scale-I Exam 2015

a
h
b
65. (a) Interest earned under scheme II of company A
II. y = 961 = ± 31

o
o
æ 30000 ´ 9.5 ´ 2 ö \ x=y

b
=` ç ÷ = ` 5700

.w
è 100 ø 74. (b) I. x2 – x – 72 = 0

o
CI earned under scheme II of company D Þ x2 – 9x + 8x –72 = 0

rd
Þ x(x – 9) +8 (x – 9) = 0

p
éæ 2 ù Þ (x + 8) (x – 9) = 0

re
9 ö
= 48000 êç1 + ÷ – 1ú

s
êëè 100 ø \ x = –8 or 9

s
úû

.c
II. y2 = 64

o
= ` (48000 × 0.1881) = ` 9028.8

m
Þ y = 64 = ± 8
\ Total interest earned by Mr. Lal
= ` (5700 + 9028.8) = ` 14728.8 \x³y
66. (b) Required average price 75. (e) I. x2 = 463 + 321 = 784

750 ´ 25 + 600 ´ 45 \ x = 784 = ± 28


= æ 18750 + 27000 ö
=` ç ÷ per kg II. y2 = 308 + 421 = 729
1350 è 1350 ø
» ` 34 per kg \ y = 729 = ± 27
67. (d) Required cost price Relationship between x and y cannot be established.
æ 90 ö 76. (a) 8.5 × (80 × 1.5 × 2.5) ÷ 4 ® 8.5 × (300) ÷ 4 = 637.5
= ` ç 800 ´ ´ 80 ÷ 77. (b) 23% of 2500 – 22% of 500 = 930 ÷ ?
è 100 ø 575 – 110 = 930 ÷ ?
= ` 57600 ?= 2
68. (a) Total cost of entire quantity of sugar = (350 × 78. (d) 5312.15 – 1318.82 – 2321.43 =1671.9
120 79. (a) (333.33 + 33.33 + 3333.34) ÷ 50 ? 3700 ÷ 50 = 74
30 + 350 × 30 ) 80. (c) 3% of 3500 + 5% of 700 = ?% of 1400
100
105 + 35 = ? % of 1400
= ` (10500 + 12600) = ` 23100
? = 10%
69. (e) Required cost
81. (d) 82. (b) 83. (e) 84. (a)
= ` (500 × 80 + 400 × 60)
85. (d) 86. (b) 87. (e) 88. (c)
= ` 64000
89. (d) 90. (a) 91. (b) 92. (d)
70. (c) Total cost of corriander sold
93. (e) 94. (a) 95. (c) 96. (c)
æ 92 ö 97. (a) 98. (d) 99. (b) 100. (d)
= ` ç 600 ´ ´ 70 ÷ = ` 38640
è 100 ø 101. (d) 102. (b) 103. (a) 104. (e)
71. (d) I. x + 5x + 6 = 0
2 105. (c) 106. (d) 107. (c) 108. (a)
Þ x2 + 2x + 3x + 6 = 0 109. (d) 110. (b) 111. (e) 112. (d)
Þ x(x + 2) + 3 (x + 2) = 0 113. (d) 114. (e) 115. (c) 116. (d)
Þ (x + 3) (x + 2) = 0 117. (b) 118. (c) 119. (e) 120. (c)
\ x = –3 or –2 121. (d) 122. (a) 123. (d) 124. (d)
II. y2 + 3y + 2=0 125. (d) 126. (c) 127. (a) 128. (d)
Þ y2 + 2y + y + 2 = 0 129. (d) 130. (b) 131. (d) 132. (c)
Þ y(y + 2) + 1 (y + 2) = 0 133. (d) 134. (c) 135. (c) 136. (a)
Þ (y + 1) (y + 2) = 0 137. (c) 138. (d) 139. (a) 140. (b)
\ y = –1 or –2 141. (a) 142. (d) 143. (b) 144. (d)
\ x<y 145. (a) 146. (c) 147. (d) 148. (a)
72. (b) I. x2 – 10x + 24 = 0 149. (b) 150. (d) 151. (a) 152. (d)
Þ x2 – 6x – 4x + 24 = 0 153. (d) 154. (c) 155. (b) 156. (c)
Þ x(x – 6) – 4 (x – 6) = 0 157. (c) 158. (a) 159. (c) 160. (a)
Þ (x – 4) (x – 6) = 0 161. (b) 162. (a) 163. (c) 164. (c)
\ x = 4 or 6 165. (d) 166. (b) 167. (a) 168. (d)
II. y2 – 9y + 20 = 0
169. (c) 170. (c) 171. (b) 172. (d)
Þ y2 – 5y – 4y + 20 = 0
173. (d) 174. (a) 175. (a) 176. (c)
Þ y(y – 5) – 4 (y – 5) = 0
177. (d) 178. (b) 179. (a) 180. (c)
Þ (y – 4) (y – 5) = 0
181. (b) 182. (b) 183. (d) 184. (b)
\ y = 4 or 5
\ x>y 185. (b) 186. (b) 187. (c) 188. (b)
73. (e) I. x2 = 961 189. (b) 190. (b) 191. (d) 192. (b)
193. (a) 194. (e) 195. (a) 196. (e)
Þ x = ± 961 = ± 31 197. (c) 198. (b) 199. (a) 200. (d)
y
o
u
rs
m
a
h
b
o
IBPS PO Prelim Exam 2015

o
b
.w
o
rd
p
Based on Memory

re
s
s
.c
Reasoning Ability 7. If another student J who is taller than E but shorter than G

o
m
is added to the group, which of the following will be
definitely true?
DIRECTIONS (Qs. 1-5) : Study the following information
carefully and answer the questions given below. (a) C and J are of same height
(b) J is shorter than D
P, Q, R, S, T, V and W are seven students of a school. Each of (c) J is shorter than H
them studies in different standard from Standard IV to Standard X (d) J is taller than A
not necessarily in the same order. Each of them has favourite subject (e) None of these
from English, Science, History,Geogra-phy, Mathematics, Hindi and 8. Which of the following will definitely be the third from
Sanskrit not necessarily in the same order. top when the eight students are arranged in descending
Q studies in VII Standard and does not like either order of height?
Mathematics or Geography. R likes English and does not study (a) B (b) F
either in V or in IX. T studies in VIII Standard and likes Hindi. (c) G (d) B or G
The one who likes Science studies in X Standard. S studies in (e) Cannot be determined
IV Standard. W likes Sanskrit. P does not study in X Standard. 9. How many of them are definitely shorter than F ?
The one who likes Geography studies in V Standard. (a) Three (b) Four
1. In which standard does W study? (c) Five (d) Data inadequate
(a) VII (b) IX (e) None of these
(c) X (d) Data inadequate 10. Which of the following is redundant to answer all the above
(e) None of these questions?
2. Which subject does P like? (a) (ii) only (b) (ii) and (iii) only
(a) Geography (b) Mathematics (c) (iii) and (iv) only (d) (i) and (v) only
(c) English (d) History (e) All are necessary to answer the above questions
(e) None of these
3. Which subject does S like? DIRECTIONS (Qs. 11-15) : Study the following information
(a) History (b) Geography carefully and answer the given questions :
(c) Mathematics (d) Data inadequate In a certain code language :
(e) None of these "demand and supply market" is written as "pa ni de re"
4. In which standard does P study? "market needs more demand" is written as "de ja ni fe"
(a) IV (b) VII
"supply demand is related" is written as "le de re ab"
(c) IX (d) X
(e) None of these "more related to economics" is written as "ka ha ab ja"
5. Which of the following combinations of student-standard- 11. What is the code for 'economics' ?
subject is correct? (a) ab (b) ka
(a) T -VIII -Mathematics (b) W -VII -Sanskrit (c) ha (d) ja
(c) Q -VII -Geography (d) V -X -Science (e) Either 'ka' or 'ha'
(e) None of these 12. Which of the following represents 'supply related market' ?
DIRECTIONS (Qs. 6-10) : Study the following information (a) ab ni de (b) ni re ab
carefully and answer the questions given below: (c) pa ni re (d) de ab ni
(i) A, B, C, D, E, F, G and H are eight students each having a (e) None of these
different height. 13. What is the code for 'more' ?
(ii) D is shorter than A but taller than G. (a) fe (b) ni
(iii) E is taller than H but shorter than C. (c) de (d) ja
(iv) B is shorter than D but taller than F. (e) Cannot be determined
(v) C is shorter than G.
14. Which of the following may represent "market needs more
(vi) G is not as tall as F.
customers" ?
6. Which of the following is definitely false?
(a) fe ja ni sa (b) ja ni de ab
(a) G is shorter than F (b) C is shorter than F
(c) F is taller than C (d) B is taller than E (c) ni ja ka pa (d) pa ni fe re
(e) All are true (e) le re ni ja
y
o
u
rs
m
476 IBPS PO Prelim Exam 2015

a
h
b
15. What is the code for 'needs' ? 24. Boys and girls are sitting in a row in audience in front of a

o
(a) ni (b) fe stage which faces North. Rani is fifth to the left of Sunita

o
b
(c) pa (d) le who is eighth to the right of Nishant. How many children

.w
(e) None of these are there between Rani and Nishant ?

o
(a) One (b) Two

rd
DIRECTIONS (Qs. 16-20) : In these questions relationship (c) Four (d) Cannot be determined

p
re
between different elements is shown in the statements. The (e) None of these

s
statements are followed by two conclusions.

s
25. Pointing to a photograph Shubha said, "he is the only

.c
Give answer (a) if only conclusion I is true. grandson of my mother's father". How is the man in

o
m
Give answer (b) if only conclusion II is true. photograph related to Shuhha?
Give answer (c) if only conclusion I or II is true. (a) Cousin (b) Brother
(c) Uncle (d) Cannot be determined
Give answer (d) if neither conclusion I nor II is true.
(e) None of these
Give answer (e) if both conclusion I and II are true.
26. Four of the following five are alike in a certain way and so
16. Statements :
form a group which is the one that does not belong to that
E ³ F = G; I = T; T £ G group ?
Conclusions : (a) 29 (b) 85
I. I < E II. I = E (c) 147 (d) 125
17. Statements : (e) 53
G > H < T; I > F; H > J 27. Which of the following has the same relationship as that
Conclusions : of Money : Wealth ?
I. J < G II. F < H (a) Pity : Kindness (b) Cruel : Anger
18. Statements : (c) Wise : Education (d) Pride : Humility
V > W < X; X < Y; Z > X (e) None of these
Conclusions : DIRECTIONS (Qs. 28–32) : Study the following information
I. Z > V II. Y > W carefully and answer the given questions.
19. Statements :
Eight family members A, B, C, D, E, F, G and H are sitting
M > N > P; O > P; S < P
around a circular table, facing the centre but not necessarily
Conclusions : in the same order.
I. S < M II. O < M • F, the wife of D is sitting third to right of C.
20. Statements : • A is the son of H. A is sitting second to left of D. D is
A > E > F; G > F; M > A not an immediate neighbour of either F or C. No male
Conclusions : is an immediate neighbour of D.
I. M > E II. G < A • G sits second to left of D’s son. Only two persons sit
21. Four of the following five are alike in a certain way and so between H and A’s brother. Neither C nor D is the brother
form a group. Which is the one that does not belong to the of A.
group ? • D’s son and the wife of D’s son are immediate neighbours
(a) Paper (b) Pencil of each other.
(c) Pen (d) Marker • F is the mother of H. F is not an immediate neighbour
(e) Highlighter of B and G.
22. In a row of children facing North, Shweta is fifteenth from • G is the sister of E.
the left and Jyoti is third to the left of Shweta. Ram who is 28. Who amongst the following is D’s son ?
seventh to the right of Jyoti is fifth from the right end of (a) E (b) G
the row. What is Shweta's position from the right end ? (c) A (d) B
(a) 12th (b) 10th (e) Cannot be determined
29. Who sits second to the left of G ?
(c) 8th (d) 9th
(a) A’s brother (b) G’s mother
(e) None of these
(c) D (d) B’s father
23. If it is possible to make only one meaningful word from
(e) A’s aunt
the first, the fifth, the seventh, the eighth and the eleventh
30. How many people sit between A and his brother ?
letters of the word DEPARTMENTAL, first letter of the (a) None (b) One
word is your answer. If more than one such word can be (c) Two (d) Three
formed, your answer is 'X' and if no such word can be (e) Four
formed, your answer is 'Y' 31. Who amongst the following sits exactly between H and F ?
(a) A (b) D (a) D’s wife (b) D’s son
(c) R (d) X (c) C (d) B
(e) Y (e) A
y
o
u
rs
m
IBPS PO Prelim Exam 2015 477

a
h
b
32. Who amongst the following is the brother of A ?
Quantitative Aptitude

o
(a) E (b) G

o
b
(c) A (d) B DIRECTIONS (Qs. 36-40): Find the missing term.

.w
(e) Cannot be determined

o
36. 0.5, 1.5, 5, 8,76,?

rd
DIRECTIONS (Qs. 33-35) : In each question below are two/ (a) 380 (b) 385

p
(c) 390 (d) 395

re
three statements followed by two conclusions numbered I and
(e) None of these

s
II. You have to take the two/three given statements to be true

s
37. 65, 72, 86, 114 ?

.c
even if they seem to be at variance from commonly known facts

o
(a) 160 (b) 165

m
and then decide which of the given conclusions logically (c) 170 (d) 175
follows from the given statements disregarding commonly (e) None of these
known facts. 38. 63, 31, 15, 7, 3 ?
Give answer (a) if only conclusion I follows. (a) 0 (b) 1
(c) 2 (d) 3
Give answer (b) if only conclusion II follows.
(e) None of these
Give answer (c) if either conclusion I or conclusion II follows. 39. 13. 70, 71, 76, ?, 81, 86, 70, 91
Give answer (d) if neither conclusion I nor conclusion II (a) 70 (b) 71
follows. (c) 80 (d) 96
Give answer (e) if both conclusion I and conclusion II follow. (e) None of these
(33-34): 40. 8, 43, 11, 41, ?, 39, 17
Statements : (a) 8 (b) 14
(c) 43 (d) 44
Some colours are paints.
(e) None of these
All colours are varnishes.
No varnish is dye. DIRECTIONS (Qs. 41-45): In each of these questions, two
33. Conclusions : equations (I) and (II) are given. You have to solve both the
I. No paint is dye. equations and give answer
II. All paints being varnishes is a possibility. (a) if x > y (b) if x < y
34. Conclusions: (c) if x ³ y (d) if x £ y
I. Some varnishes are paints. (e) if x = y or relation cannot be established between 'x' and 'y'.
41. I. 8x + y =10
II. No dye is colour.
II. 4x + 2y = 13
35. Statements : 42. I. (x + 3) (y + 2) = 12
All squares are triangles. II. 2xy + 4x + 5y = 11
No triangle is circle. 43. I. (3x – 2)/y = (3x + 6)/(y + 16)
All circles are rectangles. II. (x + 2)/(y + 4) = (x + 5)/(Y + 10)
35. Conclusions : 44. I. x2 + 20x + 4 = 50 – 25x
I. No rectangle is square. II. y2 – 10y – 24 = 0
45. I. (x2 – 10x + 16)/(x2 – 12x + 24) = 2/3
II. All rectangles being square is a possibility.
II. y2 – y – 20 = 0
DIRECTIONS (Qs. 46-50) : Study the given table carefully to answer the following questions.
Field Shape Side Base Height Radius Cost of flooring Cost of fencing
Name (in m) (in m) (in m) (in m) (in Rs. per sq. (in Rs. per m)
metre)
A Triangle 16 12 50 20
B Rectangle 10 × 20 30 15
C Square 15 40 18
D Parallelogram 20 12 60 25
E Circle 10 45 22
46. What is the cost of flooring of A? (a) `180 (b) `120
(c) `240 (d) `360
(a) `4000 (b) `4600 (e) `480
(c) `4800 (d) `5000 48. What is the ratio of the cost of flooring to that of fencing
of field D?
(e) `4400 (a) 4 : 1 (b) 6 : 1
47. What is the difference between the cost of fencing of C (c) 8 : 1 (d) 9 : 1
and that of B? (e) 5 : 1
y
o
u
rs
m
478 IBPS PO Prelim Exam 2015

a
h
b
49. The cost of fencing of field E is approximately what percent 55. What is the ratio of the number of Hindu soldiers in Bihar

o
of the cost of flooring of field C? regiment to the number of non-Hindu soldiers in Jat

o
b
(a) 10.5% (b) 19.46% regiment?

.w
(c) 18.71% (d) 15.36% (a) 11 : 10 (b) 12 : 11

o
(e) 13.82%

rd
(c) 13 : 12 (d) 14 : 13
50. The cost of fencing of field C is what percent of the cost of

p
(e) 15 : 14

re
fencing of field D? 56. If the compound interest on an amount of ` 29000 in two

s
(a) 87.54% (b) 67.5%

s
years is ` 9352.5, what is the rate of interest?

.c
(c) 72.13% (d) 54.36% (a) 11 (b) 9

o
m
(e) 46.5% (c) 15 (d) 18
DIRECTIONS (Qs. 51-58) : Study the following table and pie (e) None of these
chart carefully to answer the given questions. 57. Three friends A, B and C start running around a circular
stadium and complete a single round in 8, 18 and 15
The table shows the ratio of Hindu religion soldiers to soldiers
seconds respectively. After how many minutes will they
of other religions
meet again at the starting point for the first time?
Name of regiment Hindu Other religions (a) 12 (b) 6
Jat regiment 4 1 (c) 8 (d) 15
Sikh regiment 3 5 (e) 18
58. The perimeter of a square is equal to the radius of a circle
Madras regiment 2 1 having area 39424 sq cm, what is the area of square?
Maratha regiment 3 2 (a) 1225 sq cm (b) 441 sq cm
Bihar regiment 5 3 (c) 784 sq cm (d) Can't say
(e) None of these
Percentage of various regiments in the Army DIRECTIONS (Qs. 59-61) : Study the following information
Bihar carefully to answer the questions that follow-
18% regiment
Maratha 12% A committee of five members is to be formed out of 5 Males, 6
regiment Females and 3 Children. In how many different ways can it be
15% done if-?
Madras 35% 59. The committee should consist of 2 Males, 2 Females and 1
regiment Jat Child?
20% regiment (a) 450 (b) 225
Sikh (c) 55 (d) 90
regiment
(e) None of these
60. The committee should include all the 3 Childs?
Total number of soldiers in the army = 10000
(a) 90 (b) 180
51. What is the number of Hindu soldiers in Jat regiment? (c) 21 (d) 55
(a) 2600 (b) 2700 (e) None of these
(c) 3200 (d) 2800 61. Thirty men can complete a work in 36 days. In how many
days can 18 men complete the same piece of work?
(e) 2350
(a) 48 (b) 36
52. What is the difference between Hindu soldiers in Madras
(c) 60 (d) 72
regiment and soldiers of other religions in Bihar regiment?
(e) None of these
(a) 485 (b) 550
62. Ram spends 50% of his monthly income on household
(c) 520 (d) 510
items, 20% of his monthly income on buying clothes, 5%
(e) 490 of his monthly income on medicines and saves remaining
53. The number of Hindu soldiers in Sikh regiment is what ` 11,250. What is Ram's monthly income?
percent of the number of other soldiers in Maratha (a) ` 38,200 (b) ` 34,000
regiment? (c) ` 41,600 (d) ` 45,000
(a) 97.12% (b) 99.56% (e) None of these
(c) 102% (d) 104.16% 63. The number obtained by interchanging the two digits of a
(e) 25% two digit number is lesser than the original number by 54.
54. In which regiment is the number of non-Hindu soldiers If the sum of the two digits of the number is 12, then what
the maximum? is the original number?
(a) Maratha regiment (b) Sikh regiment (a) 28 (b) 39
(c) Madras regiment (d) Jat regiment (c) 82 (d) Can't say
(e) Bihar regiment (e) None of these
y
o
u
rs
m
IBPS PO Prelim Exam 2015 479

a
h
64. At present Geeta is eight times her daughter's age. Eight 71. Which of the following would be the SECOND sentence?

b
o
years from now, the ratio of the ages of Geeta and her (a) A (b) C

o
b
daughter will be 10 : 3 respectively. What is Geeta's present (c) D (d) E

.w
age ? (e) F

o
(a) 32 years (b) 40 years 72. Which of the following would be the FOURTH sentence?

rd
(c) 36 years (d) Can't say (a) A (b) B

p
re
(e) None of these (c) C (d) E

s
(e) F

s
65. In how many different ways can 4 boys and 3 girls be

.c
73. Which of the following would be the FIFTH sentence ?
arranged in a row such that all the boys stand together and

o
(a) A (b) B

m
all the girls stand together ?
(c) C (d) D
(a) 75 (b) 576 (e) F
(c) 288 (d) 24 74. Which of the following would be the FIRST sentence ?
(e) None of these (a) B (b) C
(c) D (d) E
DIRECTIONS (Qs. 66-70) : What approximate value will come
(e) F
in place of question mark (?) inthe following questions (You are
75. Which of the following would be the SIXTH (LAST)
not expected to calculate the exact value).
sentence ?
66. 9228.789 – 5021.832 + 1496.989 = ? (a) A (b) B
(a) 6500 (b) 6000 (c) C (d) D
(c) 6300 (d) 5700 (e) E
(e) 5100 DIRECTIONS (Qs. 76-80) : Read each sentence to find out
67. 1002 ÷ 49 × 99 – 1299 = ? whether there is any grammatical error in it or a wrong word
(a) 700 (b) 600 has been used. The error, if any, will be in one part of the
(c) 900 (d) 250 sentence which has been printed in bold and has been
(e) 400 numbered (a), (b), (c) or (d). The number of that part is the
68. 29.8% of 260 + 60.01% of 510 - 103.57 = ? answer. If there is no error, the answer is (e) i.e. ‘No error’.
(a) 450 (b) 320 (Ignore the errors of punctuation, if any.)
(c) 210 (d) 280 76. The convergence of (a)/Indian accounting standards with
(e) 350 International Financial Reporting Standards (IFRS)
69. (21.98)2 – (25.02)2 + (13.03)2 = ? beginning (b)/in April is expecting to (c)/ see power
(a) 25 (b) 120 companies struggling with (d)/ significant first-time
(c) 10 (d) 65 adoption impact. No error (e)
77. Researchers at (a)/ the Indian Institute of Science (IISc),
(e) 140
Bangalore, are mapping (b)/ India’s solar hot spots-where
70. 24.98 ´ 625 ´ 99 = ? round-the-year (c)/ sunlight makes it viable of (d)/
(a) 110 (b) 90 companies to set up solar power plants. No error (e).
(c) 200 (d) 160 78. Though their qualifications span a diverse (a)/ range,
there is an equal (b)/ number of graduates and those who
(e) 125
have just completed School, each set (c)/ making up (d)/
ENGLISH LANGUAGE close to 30% of these households. No error (e)
79. As if (a)/ the most dangerous moment for any dictatorship
is when (b)/ it starts to (c)/ reform, North Korea looks
DIRECTIONS (Qs. 71-75) : Rearrange the following six
ready to turn that truism on its head. (d)/ No error (e)
sentences (A), (B), (C), (D), (E) and (F) in the proper sequence
80. It so happens (a)/ that this happy campy ritual is their
to form a meaningful paragraph; then answer the questions way of life (b)/ and one into which (c)/ they don’t
given below them. particularly welcome (d)/ voyeuristic intrusions. No
(A) Two of the best-performing major economies in 2014 error (e)
were China and Brazil, with growth estimated at 7.5%
DIRECTIONS (Qs. 81-90) : Read the following passage
and 10.5% resepectively.
carefully and answer the questions given below it. Certain words/
(B) Despite that limp growth, major US stock market indexes
phrases are printed in bold to help you to locate them while
are up between 11% and 20% for the year.
answering some of the questions.
(C) Even knowing where economies are headed sometimes it
is of no help to an investor. The outside world has pat answers concerning extremely
(D) It is hard to anticipate the direction of financial markets. impoverished countries, especially those in Africa. Everything
(E) But as of December, stock markets of both nations were comes back, again and again, to corruption and mis-rule. Western
in the red for the year. officials argue that Africa simply needs to behave itself better, to
(F) By contrast, the US economy is likely to have expanded allow market forces to operate without interference by corrupt
at only about 2.6% for the year. rulers. Yet the critics of African governance have it wrong.
y
o
u
rs
m
480 IBPS PO Prelim Exam 2015

a
h
b
Politics simply can't explain Africa's prolonged economic crisis. (b) definite information about what is happening in under

o
The claim that Africa's corruption is the basic source of the developed countries

o
b
problem does not withstand serious scrutiny. During the past (c) stopped extending any financial aid to under developed

.w
decade I witnessed how relatively well-governed countries in countries

o
Africa, such as Ghana, Malawi, Mali and Senegal, failed to

rd
(d) misconceptions about the aid given to the poor nations
prosper, whereas societies in Asia perceived to have extensive

p
by developed countries

re
corruption, such as Bangladesh, Indonesia and Pakistan, enjoyed
(e) None of these

s
rapid economic growth.

s
.c
82. According to the Westerners the solution to eradicate
What is the explanation? Every situation of extreme poverty

o
poverty of African nations lies in

m
around the world contains some of its own unique causes, which
need to be diagnosed as a doctor would a patient. For example, (a) corruption
Africa is burdened with malaria like no other part of the world, (b) improving their own national behaviour
simply because it is unlucky in providing the perfect conditions (c) mis-rule
for that disease; high temperatures, plenty of breeding sites and (d) prolonged economic crisis
particular species of malaria-transmitting mosquitoes that prefer
(e) None of these
to bite humans rather than cattle.
Another myth is that the developed world already gives plenty 83. The author has given the example of Bangladesh, Indonesia
of aid to the world's poor. Former U.S. Secretary of the Treasury, and Pakistan in support of his argument that
Paul O'Neil expressed a common frustration when he remarked (a) corruption is the major culprit in the way of prosperity
about aid for Africa : "We've spent trillions of dollars on these (b) mis-governance hampers the prosperity of nations
problems and we have damn near nothing to show for it". O'Neil (c) despite rampant corruption, nations may prosper
was no foe of foreign aid. Indeed, he wanted to fix the system so (d) developed nations arrogantly neglect under developed
that more U.S. aid could be justified. But he was wrong to believe countries.
that vast flows of aid to Africa had been squandered. President
Bush said in a press conference in April 2004 that as "the greatest (e) None of these
power on the face of the earth, we have an obligation to help the 84. The author has mentioned Ghana as a country with
spread of freedom. We have an obligation to feed the hungry". (a) reasonably good-governance
Yet how does the U.S. fulfill its obligation? U.S. aid to farmers (b) corrupt leadership
in poor countries to help them grow more food runs at around (c) plenty of natural resources
$200 million per year, far less than $1 per person per year for
(d) rapid economic growth
the hundreds of millions of people living in subsistence farm
households. (e) None of these
From the world as a whole, the amount of aid per African per 85. The cases of malaria in Africa are mainly due to
year is really very small, just $30 per sub-Saharan African in A. high temperature.
2002. Of that modest amount, almost $5 was actually for B. climatic conditions conducive for breeding.
consultants from the donor countries, more than $3 was for C. malaria carries liking for human blood in preference
emergency aid, about $4 went for servicing Africa's debts and to that of cattle.
$5 was for debt-relief operations. The rest, about $12, went to (a) None of these (b) Only B and C
Africa. Since the "money down the drain" argument is heard
(c) Only A and C (d) Only A and B
most frequently in the U.S., it's worth looking at the same
calculations for U.S. aid alone. In 2002, the U.S. gave $3 per (e) All the three
sub-Saharan African. Taking out the parts for U.S. consultants DIRECTIONS (Qs. 86-88) : Choose the word/group of words
and technical cooperation, food and other emergency aid, which is most nearly the same in meaning to the word/group of
administrative costs and debt relief, the aid per African came to words printed in bold as used in the passage.
grand total of 6 cents.
The U.S. has promised repeatedly over the decades, as a signatory 86. OBLIGATION
to global agreements like the Monterrey Consensus of 2002, to (a) lip sympathy (b) true sympathy
give a much larger proportion of its annual output, specifically (c) self pity
upto 0.7% of GNP, to official development assistance. The U.S. (d) conditional responsibility
failure to follow through has no political fallout domestically, of (e) moral binding
course, because not one in a million U.S. citizens even knows of 87. SQUANDER
statements like the Monterrey Consensus. But no one should
(a) use economically (b) spend wastefully
underestimate the salience that it has around the world. Spin as
American might about their nation's generosity, the poor (c) siphon judiciously (d) donate generously
countries are fully aware of what the U.S. is not doing. (e) None of these
81. The passage seems to emphasize that the outside world 88. MODEST
has (a) humble (b) sufficient
(a) correct understanding about the reasonable aid (c) meagre (d) sober
provided by the USA to the poor countries (e) unpretentious
y
o
u
rs
m
IBPS PO Prelim Exam 2015 481

a
h
b
DIRECTIONS (Qs. 89-90) : Choose the word/group of words 91. (a) Developed (b) Cured

o
(c) Improved (d) Enhanced

o
which is most opposite in meaning of the word given in bold as

b
(e) Characterized

.w
used in the passage.
92. (a) Source (b) Finance

o
89. MYTH (c) Funds (d) Manpower

rd
(a) reality (b) mystery (e) Industries

p
re
(c) misery (d) misconception 93. (a) Exhaustive (b) Unexploited

s
s
(e) exaggeration (c) Abundant (d) Indefinite

.c
o
90. EXTENSIVE (e) Unreliable

m
(a) intensive (b) abominable 94. (a) Upward (b) Drastic
(c) inherent (d) rampant (c) Negligible (d) Incredible
(e) negligible (e) Sudden
95. (a) Diminishes (b) Degenerates
DIRECTIONS (Qs. 91-100) : sIn the following passage there (c) Increases (d) Succumbs
are blanks, each of which has been numbered. These numbers (e) Stabilizes
are printed below the passage and against each five words are
96. (a) Plans (b) Attempts
suggested, one of which fit the blank appropriately. Find out
(c) Attention (d) Resources
the appropriate word in each case.
(e) Strategy
Economic backwardness of a region is (91) by the coexistence 97. (a) Existing (b) Glorious
of unutilized (92) on the one hand and (93) natural resources on (c) Ancient (d) Economic
the other. Economic development essentially means a process of (e) Discouraging
(94) change whereby the real per capita income of an economy
98. (a) Pivotal (b) Neutral
(95) over a period of time. Then a simple but meaningful ques-
(c) Insignificant (d) Enchanted
tion arises; what causes economic development? Or what makes
(e) Vicious
a country developed? This question has absorbed the (96) of
scholars of socio-economic changes for decades. Going through 99. (a) Increasingly (b) Always
the (97) history of developed countries like America, Russia and (c) Gradually (d) Deliberately
Japan, man is essentially found as (98) in the process of eco- (e) Badly
nomic development. Japan, whose economy was (99) damaged 100. (a) Enlighten (b) Validate
from the ravages of the 2nd world War, is the example of our time (c) Negate (d) Underestimate
to (100) kingdom role in economic development. (e) Belittle
y
o
u
rs
m
482 IBPS PO Prelim Exam 2015

a
h
b
o
o
b
.w
o
(1-5): 13. (d) The code for more is ‘ja’

rd
14. (a) market Þ ni

p
Student Standard Favourite

re
needs Þ fe

s
Subject more Þ ja

s
.c
P V Geography The code for customers may be ‘sa;.

o
m
Q VII History 15. (b) The code for ‘needs’ is fe.
16. (c) E ³ F = G ³ T = I
R VI English
Conclusions :
S IV Mathematics I. I < E : Not True
T VIII Hindi II. I = E : Not True
V X Science I is either smaller than or equal to E. Therefore, either
W IX Sanskrit conclusion I or II follows.
17. (a) G > H < T
1. (b) W studies in Standard IX. I>F
2. (a) P likes Geography. H>J
3. (c) S likes Mathematics.
G>H>J
4. (e) P studies in Standard V.
J<H<T
5. (d) The Combination V-X-Science is correct.
(6 - 10): Conclusions :
(i) A > D > G (ii) C > E > H I. J < G : True
(iii) D > B > F (iv) G > C II. F < H : Not True
(v) F > G So only conclusion I follows.
From all the statements 18. (b) V > W < X < Y
A>D>B>F>G>C>E>H V>W< X<Z
6. (e) All statements are true. Conclusions :
7. (b) According to question I. Z > V : Not True
A>D>B>F>G>C>E>H II. Y > W : True
J So only conclusion II is true.
It is not clear whether C and J are of same height, 19. (a) M > N > P
But, J is definitely shorter than D. O>P
8. (a) A > D > B > F........ S<P
M>N>P<O
3rd from top M>N>P>S
9. (b) G, C, E and H are shorter than F. O>P>S
10. (e) All the statements are necessary to answers the Conclusions :
question.
I. S < M : True
(11 to 15): II. O < M : Not True
demand and supply market pa ni de re 20. (a) M > A > E > F < G
market needs more demand de ja ni fe Conclusions :
I. M > E : True
supply demand is related le de re ab II. G < A : Not True
more 21. (a) Except Paper, all others are writing materials. We
related to economics ka ha ab ja
write or mark something on paper.
Hence, codes are : 15th 19th
demand Þ de needs Þ fe J S R
22. (d)
and Þ pa more Þ ja 5th
supply Þ re is Þ le
Total number of children in the row = 19 + 5 – 1 = 23
market Þ ni related Þ a b
to Þ ka or ha economics Þ ka or ha Shweta’s position from right end
11. (e) The codes for economics is either ‘ka or ‘ha’. = 23 –15 + 1 = 9th
12. (b) supply Þ re 1 2 3 4 5 6
23. (b)
related Þ ab
market Þ ni D E P A R T
y
o
u
rs
m
IBPS PO Prelim Exam 2015 483

a
h
Or

b
7 8 9 10 11 12

o
triangles

o
rectangle

b
M E N T A L

.w
Specified letters Þ D, R, M, E, A

o
Meaningful word Þ DREAM

rd
p
24. (b) .... N R S .....

re
So there are two persons between Rani and Nishant.

s
s
25. (b) Father circle squares

.c
35. (d) Conclusion: I. (False)

o
Grandson II. (False)

m
Mother
(–) 36. (b) 0.5 × 1 + 1 = 1.5
Shubha Man 1.5 × 2 + 2 = 5
(+) 5 × 3 + 3 = 18
Brother 18 × 4+4= 76
26. (d) 125 is perfect cube of 5. 76 × 5+5= 385
27. (a) A person can get wealth from money in the same 37. (c) 65 + 7; 72 + 14; 86 + 28 ; 114+ 56= 170
way a person can get kindness from pity. 38. (b) (63-1) / 2; ( 31-1)/2 ; (15-1)/2; (7-1)/2; (3-1)/2 =1
(28 – 32) : Female 39. (a) In this series, 5 is added to the previous number; the
Male F number 70 is inserted as every third number.
A E Male 40. (b) This is a simple alternating addition and subtraction
series. The first series begins with 8 and adds 3; the
second begins with 43 and subtracts 2.
H B Male 41. (b) From both equation
Female x = 7/12, y = 16/3
y>x
D C 42. (e) xy + 3y + 2x + 6 =12
Male Female 2xy + 6y + 4x = 12 ...(i)
G 2xy + 5y + 4x = 11 ...(ii)
Female From equation (i) and (ii)
F is the wife of D.
y=1
B is the son of D and F.
From equation (i)
H is the daughter D and F.
x=1
C is the wife B. x=y
A’s brother is E. 43. (b) (3x – 2)/y = (3x + 6)/(y + 16)
28. (d) B is the son of D 48x – 8y = 32 ...(i)
29. (b) H is mother of A, E And E and G. (x + 2)/(y + 4) = (x + 5)/(y + 10)
H sits second to left of G. y = 2x ...(ii)
30. (b) One person sits between A and his brother E. From equation (i) & (ii)
31. (e) A sits exactly between H and F. x = 1, y = 2
32. (a) E is the brother of A. y>x
(33-34) : varnishes varnishes 44. (b) From the given equation
x = 1, – 46
& y = – 4, 6
colours colours y>x
Or
paints 45. (a) From 1st equation
x2 – 6x = 0
paints x = 0,6
dye dye
From 2nd equation
33. (a) Conclusion : I (True) (y + 4) (y – 5)
II (False) y = – 4, 5
So only conclusion I follows. x>y
34. (e) Conclusion : I (True) 46. (c) A is a triangle
II (True) So, area of A = 1/2 × 16 × 12 = 96 sqm
So both the conclusions follow. So, cost of flooring of A = 96 × 50 = ` 4800
triangles 47. (a) Perimeter of B = 2 (10 + 20) = 60 m
35. rectangle So, cost of fencing of B = 60 × 15 = 900
Perimeter of C = 4 × 15 = 60 m
So, cost of fencing of C = 60 × 18 = ` 1080
So, required difference = 1080 – 900 = ` 180
48. (d) Area of D = Base × Height = 20 × 12 = 240 mtr sq
So, cost of flooring of D= 240 × 60 = ` 14400
circle squares
y
o
u
rs
m
484 IBPS PO Prelim Exam 2015

a
h
Perimeter of D = 2 (20 + 12) = 64 m 58. (c) R2 = 39424

b
o
So, cost of fencing of D = 64 × 25 = ` 1600 R = 112

o
b
So, required ratio = 14400 : 1600 = 9 : 1 Perimeter of square = 4a = 112

.w
49. (d) Perimeter of E = 2pr = 2 × 22/7 × 10 = 440/7 m Side of square = 112/4 = 28

o
Cost of fencing of E = 440/7 × 22 = ` 1382.85 Area of square = 282 = 784 cm2.

rd
Area of C = 15 × 15= 225 mtr square 59. (a) Number of ways = 5C2 × 6C2 ×3C1 = 450

p
re
So, cost of flooring of C = 225 × 40 = ` 9000 60. (d) Number of ways = 11C2 × 3C3 = 55.

s
So, required % = 1382.85 × 100 / 9000 61. (c) Required number of days = (30 × 36)/18 = 60

s
.c
= 15.36% of flooring cost of C. 62. (d) Let total income of Ram be x. Then

o
50. (b) Fencing cost of C = ` 1080 (100 – 50 – 20 – 5)% of x = 11250

m
Fencing cost of D = ` 1600 x = 45000.
Required % = 1080/1600 × 100 = 67.5% 63. (e) Let the number be xy
51. (d) Number of soldiers in Jat regiment = 10000 × 35% = (10x + y) – (10y + x) = 54
3500 x – y = 6 And x + y = 12
Number of Hindu soldiers in Jat regiment = 3500 × Solving the equations we get x = 9 and y = 3
4 So the number is 93.
= 2800 64. (a) Let the age of Geeta's daughter be x. Then Geeta's
5
52. (b) Number of Hindu soldiers in Madras regiment = age is 8x.
(8x + 8)/(x + 8) = 10/3
2
10000 × 15% × = 1000 x=4
3 Geeta's present age = 8x = 32 years.
Number of soldiers of other religions in Bihar 65. (c) Required number of ways = 4! × 3! × 2! = 288.
3 66. (d) 9228.789 ~ 9230; 5021.832~5020 and 1496.989 ~ 1500
regiment = 10000 × 12% × = 450
8 Now the equation will become
3 9230 – 5020 + 1500 = ?
So, difference = 1000 – = 450 = 550 ? = 5710
8
53. (d) Number of Hindu soldiers in Sikh regiment = 10000 But the nearest value is 5700.
3 [Note: Even rounding of the numbers to nearest
× 20% × = 750 hundred places gives the same
8 67. (a) 1002 ~1000; 49~ 50; 99~ 100 and 1299~ 1300
Number of soldiers of other religions in Maratha
Now the equation will become
2 1000 ÷ 50 × 100 – 1300 = ?
regiment = 10000 × 18% × = 720 20 × 100 – 1300 = ?
5
So, required % = = 104.16% 2000 – 1300 = ?
54. (a) Number of non-Hindu soldiers in Jat regiment ? = 700
= 3500 – 2800 = 700 68. (d) The difference between two nearest values is 70 (210
and 280). So round off the numbers to the nearest
5
Similary in Sikh regiment = 10000 × 20% × = 125 integers.
8 29.8% of 260 ~ 30% of 260; 60.01% of 510~ 60% of
1 510 and 103.57 ~104
In Madras regiment = 10000 × 15% × = 500
3 Now the equation will become
2 30% of 260 + 60% of 510 – 104 = ?
In Maratha regiment = 10000 × 18% × = 720 30/100 × 260 + 60/100 × 510 – 104=?
5
78 + 306 – 104 = ?
3
In Bihar regiment = 10000 × 12% × = 450 ? = 384 – 104 = 280
8 69. (a) (21.98)2 = (22)2
In Maratha regiment the number of non-Hindu (25.02)2 = (25)2
soldiers is the maximum. and (13.03)2 = (13)2
55. (e) Number of Hindu soldiers in Bihar regiment = 10000 The equation will becomes
5 222 – 252 + 132 = ?
× 12% × = 750
8 484 – 625 + 169 = ?
Number of non-Hindu soldiers in Jat regiment = 700 653 – 625 = ?
So, required ratio = 750 : 700 = 15 : 14 ? = 28 so the nearest value is 25
56. (c) P = 29000 CI = 9352.5 N = 2 years A = P + I = 38,352.50 70. (e) 24.98 ´ 6.25 ´ 99 = ?
Substituting the values in 5 × 2.5 × 10 = 125
æ 71. (b) 72. (d) 73. (e) 74. (c) 75. (b)
R ö÷
n

A = P ççç1 ∗ ÷ 76. (a) 77. (d) 78. (d) 79. (d) 80. (c)
è 100 ø÷ 81. (d) 82. (b) 83. (c) 84. (a) 85. (e)
Solving we get R = 15%. 86. (e) 87. (b) 88. (a) 89. (a) 90. (e)
57. (b) The required time will be the LCM of 8, 18 and 15 91. (e) 92. (d) 93. (b) 94. (b) 95. (c)
which is 360 sec or 6 minutes. 96. (c) 97. (d) 98. (a) 99. (e) 100. (b)
y
o
u
rs
m
a
h
b
o
o
IBPS PO Main Exam 2015

b
.w
o
rd
p
Based on Memory

re
s
s
.c
o
m
REASONING ABILITY II. Shardha sits second to the right of Tania. Vimla is
an immediate neighbour of Shradha.
DIRECTIONS (Qs. 1-5) : Each of the questions below DIRECTIONS (Qs. 6-10) : Study the following information
cionsists of a question and two statements numbered I and II to answer the given questions
given below it. You have to decide whether the data provided A word and number arrangement machine when given an
in the statements are sufficient to answer the question. Read input line of words and numbers rearranges them
both the statements and____ following a particular rule. The following is an illustration
Give answer (a) If the data in statement I alone are of input and rearrangement. (Single digit numbers are
sufficient to answer the questions, while the data in preceded by a zero. All other numbers are two digit
statement II alone are not sufficient to answer the numbers)
question. Input : Good 18 to raise 02 12 money 28 for 57 charity
Give answer (b) If the data in statement II alone are 09.
sufficient to answer the question, while the data in Step I : to good 18 raise 02 12 money 28 for charity 09
statement I alone are not sufficient to answer the question. 57.
Given answer (c) If the data either in statement I alone Step II : to raise good 18 02 12 money for charity 09 28
or in statement II alone are sufficient to answer the 57.
question. Step III : to raise money good 02 12 for
Give answer (d) If the data even in both statement I and charity 09 18 28 57.
II together are not sufficient to answer the question. Step IV :to raise money good 02 for charity 09 12 18 28
Give answer (e) If the data in both statements I and II 57.
together are necessary to answer the question Step V : to raise money good for charity 02 09 12 18 28
1. How is ‘party’ coded in the language ? 57.
I. ‘going to a party’ is coded as ‘la fa qu tu’ and ‘for Step V : is the last Step of the arrangement of the above
a party’ is coded as ‘fa me tu’. input as the intended arrangement is obtained.
II. ‘start the party’ is coded as ‘tu co ra’ and ‘going to DIRECTIONS (Qs. 6-7) : These questions are based on the
start’ is coded as ‘qu co la’. following input:
2. On which day of the week is Priya definitely travelling ?
I. Amita correctly remembers that Priya is travelling Input : always 19 give 21 84 for 62 14 worthy cause
after Tuesday but before Saturday. Rohit correctly 6. Which of the following would be step III for the above
remembers that Priya is travelling before Friday but input ?
after Monday. (a) worthy give for always 19 14 cause 84 62 21.
II. Priya does not travel on a Friday. (b) worthy give for always 14 19 cause 21 62 84.
3. How is P related to A ? (c) always give for worthy 19 14 cause 21 62 84.
I. A is the daughter of M and the sister of S. (d) worthy give for always 19 14 cause 21 62 84.
II. K is the son of P and the husband of M. (e) always give for cause 19 14 worthy 21 62 84.
4. Four movies are screened from Monday to Thursday, (one 7. How many steps would be needed to complete the
on each day starting on Monday and ending on Thursday) arrangement for the above input ?
viz - Crash, Social Network, Ice Age and Notting Hill. (a) VI (b) V
On which day is the movie Crash screened ? (c) IV (d) VII
I. Social Network is screened on the last day. (e) None of these
II. Neither Ice Age nor Notting Hill are screened on
DIRECTIONS (Qs. 8-10) : These questions are based on the
Monday. following input :
5. Who sits to the immediate right of Ayesha ?
I. Four friends Shradha, Tania, Vimla and Ayesha are Input : 50 62 tips on 67 how can 42 stay young 17 89
sitting around a circular table facing the centre. forever 03.
y
o
u
rs
m
486 IBPS PO Main Exam 2015

a
h
(e) The western companies which earlier outsourced to

b
8. The following stands for which step of the arrangement ?

o
young tips stay 50 on how can 42 17 forever 03 62 67 India would benefit financially as offices would not

o
b
89. have to be set overseas.

.w
(a) Step III (b) Step V 13. Which of the following can be inferred? (An infernece

o
(c) Step VI (d) Step IV is something which is not directly stated but can be

rd
(e) None of these inferred from the given facts)

p
re
9. Which of the words/numbers below would be at the fifth (a) Unemployment in India is not as severe a problem

s
position (from the right end) in Step V of the input ? as that in the west

s
.c
(a) forever (b) 42 (b) Employees working in the back offices and call

o
(c) 50 (d) young centres in the west earn much more than their

m
(e) None of these counterparts in India
10. Which of the following would be the last step for the (c) Developing countries such as China and India do not
input? outsource their back office jobs at all to other
(a) young tips stay on how forever can 03 17 42 50 62 countries
67 89. (d) Countries which do not outsource jobs do not face
(b) young tips stay on how forever can 89 67 62 50 42 the problem of unemployment
17 03. (5) One of the main reasons for high unemployment rate
(c) can forever how on stay tips forever 89 67 62 50 42 in India is its clean of call centres and back offices
17 03. which undertake outsourced work from the west
(d) young tips stay on how forever can 03 17 42 50 67
62 89. DIRECTIONS (Qs. 14-19) : Read the following information
(e) can forever how on stay tips young 03 17 42 50 62 carefully and answer the questions that follow.
67 89. (K, L, M, P, Q, R, S and T are sitting around a square table
DIRECTIONS (Qs. 11-13) : Read the followign information in such a way that four of them sit at four corners of the square
carefully and answer the quesitons that follow. while four sit in the middle of each of the four sides. The ones
who sit at the four corners face outside while those who sit in
Ruling governments in the west are being punished by the the middle of the sides face the centre of the table.)
voters for ever rising unemployment rates. Their parliament is
abuzz with campaigns, marked by criticism of India as an P sits third to the right of S. S faces the centre. Q sits
outsourcing hub. India is seen by many in the west as a land third to the left of M. M does not sit in the middle of the
of call centres and back offices with cheap labour that cost sides. Only one person sits between Q and R. R is not an
people in the west, their jobs. immediate neighbour of M. T faces the centre. K is not
11. Which of the following statements would weaken the an immediate neighbour of R.
argument given in the passage ? 14. What is position of M with respect to L ?
(a) Outsourced jobs do not require highly skilled and (a) Third to the right
qualified employees. (b) M and L sit diagonally opposite to each other
(b) Nearly 34% of the unemployed people would secure (c) Second to the right.
jobs in the west if outsourced jobs were in sourced (d) Second to the left
by their organisations (e) Fifth to the right
(c) After suffering heavy losses in the elections the 15. Who sits exactly between Q and R ?
governments in the west are expected to change their (a) T (b) P
decision on outsourcing to India. (c) K (d) M
(d) Outsourcing, a dynamic, two-way relationship has (e) S and K
created jobs and growth in India as well as the west. 16. Which of the following pairs represents the persons seated
(e) Although outsourcing to India allowed many in the middle of the sides who face each other ?
companies in west to focus on their core operations, (a) S, Q (b) K, L
they heavily compromised the quality and the (c) M, P (d) R, T
standards of their back office jobs. (e) T, Q
12. Which of the following can be a possible repercussion 17. Who amongst the following sits between R and K when
of the opposition to outsourcing in the west ? counted in anti-clockwise direction from K ?
(a) Jobs which are currently outsourced to India would (a) No one sits between R and K as R and K are
be transferred to another country which in all immediate neighbours of each other
probability would be China. (b) S, P and
(b) Call centres and back offices employees in India (c) P and Q
would expect a salary at par with their western (d) L and R
counterparts. (e) M, S and T
(c) In-sourcing of jobs by the west would render 18. If K is made to face the opposite direction, who would
thousands of Indians unemployed. sit to his immediate right ?
(d) If in-sourced in the west itself, the companies would (a) R (b) Q
not be able to employ professionals and experts of (c) P (d) T
same quality as available in India. (e) S
y
o
u
rs
m
IBPS PO Main Exam 2015 487

a
h
b
19. Four of the following five are alike in a certain way and Arguments :

o
so form a group. Which is the one that does not belong I. Yes, this will help government tide over the problem

o
b
to that group ? of inadequate generation of electricity.

.w
(a) L (b) M II. No, every citizen has right to consume electricity as

o
(c) K (d) P per their requirement as they pay for using electricity.

rd
(e) R III. No, the Government does not have the machinery to

p
re
put such a restriction on use of electricity.
DIRECTIONS (Qs. 20-23) : Read the following information

s
(a) Only I is strong

s
.c
and five statements given below it carefully and answer the
(b) Only II is strong

o
questions which follow.

m
(c) Only I and II are strong
Exodus from rural areas to the urban hubs in search of (d) Only II and III are strong
job opportunities has now declined to nearly twenty-six (e) All I, II and III are strong
percent of what it was at the turn of 21st century. 25. Statement : Many patients suffering from malaria were
(A) Since the last decade, the rural economy has administered anti-malarial drug for a week. Some of them
transformed itself into a bankable, profit making and did not respond to the traditional drug and their condition
commercially viable venture. deteriorated after four days.
(B) Job opportunities differ in urban and rural areas Which of the following would weaken the findings
(C) The load on infrastructure and resources in the urban mentioned in the above statement?
areas which had remained unmanageable for a long (a) Those patients who responded to the traditional drugs
time has been eased a little since the last decade. and recovered were needed to be given additional
(D) This trend of reverse migration which was seen only doses as they reported relapse of symptoms.
in developed countries till now has entered the (b) The mosquitoes carrying malaria are found to be
scenes of developing nations as well resistant to traditional malarial drugs.
(E) According to a recent report more than eighty (c) Majority of the patients suffering from malaria
percent of the professionals having roots in rural responded well to the traditional malarial drugs and
areas prefer to work in urban cities rahter than their recovered from the illness.
home villages. (d) Many drug companies have stopped manufacturing
20. Which of the statements numbered (A), (B), (C), (D) and traditional malarial drugs.
(E) mentioned above respresents an effect of the given (e) None of these
information most appropriately ? 26. Statement : The cost of manufacturing cars in state A is
(a) B (b) C thirty per cent less than the cost of manufacturing cars in
(c) D (d) C state B. Even after transportation fee for the differential
(e) Either D or E distances of states A and B and the interstate taxes, it is
21. Which of the statements numbered (A), (B), (C), (D) and cheaper to manufacture cars in state B than state A for
(E) mentioned above represents a cause of the given selling these cars in State C.
information most appropriately ? Which of the following supports the conclusion drawn in
(a) E (b) D the above statement?
(c) C (d) B (a) The cost of transportation from state A to state C is
(e) A more than thirty per cent of the production cost.
22. Which of the statements numbered (A), (B), (C), (D) and (b) The production costs of cars in state B are lower in
(E) mentioned above represents an assumption most comparison to state A.
appropriately ? (An assumption is something supposed or (c) Only Entry tax at state C is more for the products
taken for granted) originated in state A
(a) A (b) B (d) Entry tax at state C is more for the products origi-
(c) C (d) Both A and C nated in state B
(e) D (e) The total of transportation cost of cars from state B to
23. Which of the statements numbered (A), (B), (C), (D) and state C and entry tax of cars at state C is less than
(E) mentioned above would weaken / contradict the facts thirty per cent of the production cost of cars in state
presented in the paragraph ?
B.
(a) A (b) B
27. Statment : Many people are of the opinion that use of cell
(c) C (d) D
phones in offices for personal use should totally be banned.
(e) E
It has been found in a research study that there was sig-
DIRECTIONS (Qs. 24) : Question below is followed by three nificant drop in output of employees in the organisation
arguments numbered I, II and III. You have to decide which of where use of cell phones was liberally allowed vis-a-vis
the argument(s) is/are “strong” and indicate your answer those organizations where use of cell phones are banned
accordingly. for making personal calls.
24. Should the Government restrict use of electricity for each Which of the following contradicts the findings in the
household depending upon the requirement? above statement?
y
o
u
rs
m
488 IBPS PO Main Exam 2015

a
h
b
(a) People spend more time on talking while using cell III. Some cats are flowers.

o
phone for personal calls. IV. Some buses are tigers.

o
b
(b) Use of cell phones has become common in all the (a) None follows

.w
organizations. (b) Only I and II follow

o
(c) In the organization where employees were allowed to (c) Only III and IV follow

rd
use cell phones for making personal calls the

p
(d) Only I and IV follow

re
employees are found to be more motivated to carry (e) Only II and III follow

s
out their duties than those working in other

s
31. Statements :

.c
organizations. All fans are rooms.

o
m
(d) Many organizations who provide cell phones to their No room is green.
employees for making official calls advise them to Some windows are green.
refrain from making personal calls during office hours Conclusions :
(e) None of these I. Some windows are fans.
28. Statement : Some of the country’s largest food beverage II. Some windows are rooms.
and pharma companies may be forced to import sugar III. Some fans are green.
directly as the government plans to improve stock limits IV. No green is fan.
on industrial consumers to ease the shortage in grocery (a) Only I follows
shops and cool down prices that are at a 28 month high. (b) Only Ill follows
Which of the following substantiates the views expressed (c) Only IV follows
in the above statement? (d) Only II and IV follow
(a) Food, beverage and pharma companies were not (e) All follow
allowed in the past to import sugar. 32. Statement:
(b) Sugar prices have been fluctuating for the past seven Some tablets are rains.
months. All dogs are rains.
(c) Government does not have authority to restrict All rains are chairs.
purchase of sugar from the open market. Conclusion:
(d) Import of sugar for commercial use will help lowering I. Some chairs are tablets.
down the sugar price in the retail market. II. All dogs are chairs.
(e) None of these III. Some tablets are dogs.
IV. Some tablets are chairs.
DIRECTIONS (Qs. 29-33) : In each of the questions below (a ) All follow
are given three statements followed by four conclusions
(b) Only I, II and III follow
numbered I, II, III and IV. You have to take the given statements
(c) Only II, III and IV follow
to be true even if they seem to be at variance from commonly
(d) Only Ill and IV follow
known facts. Read all the conclusions and then decide which of
the given conclusions logically follows from the given statements (e) None of these
disregarding commonly known facts. 33. Statement:
No man is sky.
29. Statement: No sky is road.
Some boxes are trees. Some men are roads.
Some trees are horses. Conclusions:
All horses are fruits. I. No road is man.
Conclusions: II. No road is sky.
I. Some fruits are boxes. III. Some skies are men.
II. Some fruits are trees.
IV. All roads are men.
III. Some horses are boxes.
(a) None follows
IV No fruit is box
(b) Only I follows
(a) None follows
(c) Only I and III follow
(b) Only either II or IV follows
(d) Only II and III follow
(c) Only either I or IV and II follow
(e) None of these
(d) Only either I or Ill and IV follow
(e) None of these DIRECTIONS (Qs. 34 -38) : Study the following information
30. Statement: carefully to answer these questions.
All flowers are buses. Seven students Ashwin, Devika, Baljit, Chandrakant,
Some buses are cats. Urmila, Nagesh and Pranjali have taken admissions for MBA
All cats are tigers. with specialization in HR or Finance or Marketing. Each one
Conclusions: has got admission in different institutes M, J, K, L, R, T, F not
I. Some tigers are buses. necessarily in the same order. At least two have opted for each
II. Some tigers are flowers. of the specializations.
y
o
u
rs
m
IBPS PO Main Exam 2015 489

a
h
b
Devika has opted for Marketing but not in Institute J or T. 41. At which of the following stations do B and D get in ?

o
Chandrakant has taken admission for HR in Institute K. The (a) I (b) Base station

o
b
one who studies in Institute F does not study Finance. Nagesh (c) III (d) Data inadequate

.w
studies the same specialization as that of Devika in Institute R. (e) None of these

o
Ashwin does not study in Institute L or T. Baljit studies HR in 42. After how many stations does E get down?

rd
Institute J. Pranjali studies in Institute F and does not study (a) One (b) Two

p
re
marketing. (c) Three (d) Four

s
34. Which of the following combinations of institute and (e) Five

s
.c
specialization is true for Urmila ? 43. E gets down after how many stations at which F gets down?

o
m
(a) L - Finance (b) L - Marketing (a) Next station (b) Two
(c) T- Marketing (d) T - Finance (c) Three (d) Four
(e) None of these (e) None of these
35. Devika studies in which institute ?
DIRECTIONS (Qs. 44-46) : Study the following arrangement
(a) L (b) T
and answer the questions given below :
(c) F (d) M
(e) None of these R4 T M 7 W% J 9 5 I # 1 PB 2 TA3 D $ 6 E N F 8 UH
36. Which of the following groups represents the students @
studying HR ? 44. Which of the following is sixth to the right of the fourteenth
(a) Baljit and Chandrakant from the right end ?
(b) Baljit, Chandrakant and Pranjali (a) 5 (b) 6
(c) Ashwin, Baljit and Chandrakant (c) I (d) $
(d) Urmila and Chandrakant (e) None of these
(e) None of these 45. How many such consonants are there in the above
37. Which of the following combinations of institute, student arrangement, each of which is immediately followed by a
and specialization is correct ? symbol but not immediately preceded by another consonant ?
(a) R - N - HR (a) None (b) One
(b) T - N - Marketing (c) Two (d) Three
(c) L - B - HR (e) None of these
(d) M - D- Marketing 46. Which of the following is the eighth to the left of the
(e) None of these
sixteenth frorn the left end ?
38. What is the specialization of Ashwin ?
(a) J (b) E
(a) Marketing
(c) % (d) 6
(b) HR
(e) None of these
(c) Finance
(d) Marketing or HR 47. Consider the following sentences :
(e) Finance or Marketing "Now-a-days the FMCG companies woo their customers
or buyers by selling their goods or products in small sachets.
DIRECTIONS (Qs. 39-43) : Study the following information This has increased the sale of the product".
carefully and answer the questions given below : Which of the following may be the appropriate reason for
A, B, C, D, E, F and G are seven persons who travel to office the given facts ?
everyday by a particular train which stops at five stations-I, 11, (a) People generally want to buy any products in small
III, IV and V respectively after it leaves base station. Three among quantity
them get in the train at the base station. D gets down at the next (b) FMCG companies save cost of packaging and so their
station at which F gets clown. B does not get down either with A profit is maximised
or E. G alone gets in at station III and gets down with C after (c) It is easy to store any products in sachets so buyer
one station. A travels between only two stations and gets down prefer them
at station V. None of them gets in at station II. C gets in with F
(d) It is possible to assess the quality of products without
but does not get in with either B or D. E gets in with two others
wasting much money when a sachets is bought
and gets down alone after D, B and D work in the same office
and they get down together at station III. None of them gets (e) Generally people consume the whole quality of product
down at station I. contained in the sachets at once and so people buy a
39. At which station does E get down ? large number of sachets at one time.
(a) II (b) III DIRECTIONS (Qs. 48-50) : Study the following information
(c) IV (d) Data inadequate carefully to answer the given questions
(e) None of these
40. At which station do C and F get in ? Seven persons – A, B, C, D, K, L and N – are seated in a straight
(a) I (b) II line facing north in ascending order of their salaries. N earns
(c) III (d) Data inadequate more than L and D. N earns more than A but he does not earn
(e) None of these the highest. A earns more than L. The person who earns the
y
o
u
rs
m
490 IBPS PO Main Exam 2015

a
h
b
second highest receives a salary of ` 35,000 while the third lowest II. Ratio between present ages of Suchitra and her mother

o
earner receives ` 23,000. K earns less than L but more than D. is 2 : 3 respectively.

o
b
C earns `18,000. III. Four years hence the ratio between Suchitra’s age and

.w
her son’s age will be 13 : 24 respectively.
48. Who among the following earn(s) more than ` 23,000 but

o
rd
less than ` 35,000 ? (a) Only II (b) Only III
(c) Either I or II only (d) Either II or III only

p
(a) Only A (b) Only L

re
(e) None of these
(c) A and L (d) K and L

s
54. What is Neeta’s share in the profit earned at the

s
.c
(e) A and N end of 2 years in a joint business run by Neeta, Seeta

o
49. Who among the following may earn ` 21,000 ? and Geeta?

m
(a) K (b) D I. Neeta invested ` 85,000/ to start the business.
II. Seeta and Geeta joined Neeta’s business after six
(c) L (d) C
months investing amounts in the ratio of 3 : 5
(e) N respectively.
50. Who among the following earns more than K but less than III. Total amount invested by Seeta and Geeta is ` 2.3
A? lakhs
(a) C (b) N (a) Only II
(c) D (d) L (b) Only III
(c) Only either II or III
(e) B (d) Information in all three statements is required for
answering the question.
QUANTITATIVE APTITUDE (e) Question cannot be answered even with the
information in all three statements.
DIRECTIONS (Qs. 51-55) : In these questions, a question is 55. What is the labelled price of the article?
given followed by information in three statements. You have to I. Cost price of the article is ` 500/.
consider the information in all the three statements and decide II. Selling price after offering 5% discount on the labelled
price is ` 608/.
the information in which of the statement(s) is not necessarily
III. Profit earned would have been 28% if no discount
required to answer the question and therefore can be dispensed was offered.
with. Indicate your answer accordingly. (a) Only I (b) Only III
51. How many students from Institute ‘A’ got placement? (c) Only II & III (d) Only I and III
I. Number of students studying in Institutes A & B are (e) Only I and II
in the ratio of 3 : 4 respectively. DIRECTIONS (Qs. 56-60) : Study the following graph carefully
II. Number of students who got placement from Institute to answer these questions.
B is 120% of the number of students who got Quantity of various Items Sold and Price Per Kg.
placement from Institute A.
III. 80% of the students studying in Institute B got 800 80
placement. 700 70
Quantity Sold (In kg.)

(a) None of the statements can be dispensed with 600 60

Price (Rs. per kg.)


(b) Only I 500 50
(c) Only II 400 40
(d) Anyone of the three 300 30
(e) Question cannot be answered even with the 200 20
information in all three statements 100 10
52. What is the monthly income of Mr. X? 0 0
Wheat Rice Sugar Groundnut Cumin Corriander
I. Mr. X spends 85% of his income on various items
Price Item
and remaining amount is saved.
Quantity
II. Monthly saving of Mr. X are ` 4,500/.
III. Out of the total money spent by Mr. X in a month,
56. What is the average price per kg of wheat and rice sold?
one-fifth is spent on food and remaining amount of `
(rounded off to nearest integer)
20, 400 on other items.
(a) 32 (b) 34
(a) Only II (c) 36 (d) 35
(b) Only III (e) 31
(c) Only either II or III 57. If cumin is sold at a 10% discount, the quantity sold
(d) Question cannot be answered even with the doubles. What will be the cost of total quantity of cumin
information in all three statements sold on discount?
(e) None of these (a) ` 52,600/ (b) ` 28,800/
53. What is Suchitra’s present age? (c) ` 32,000/ (d) ` 57,600/
I. Suchitra’s present age is double the age of her son. (e) None of these
y
o
u
rs
m
IBPS PO Main Exam 2015 491

a
h
b
58. Out of the total quantity of sugar sold, half the quantity is DIRECTIONS (Qs. 66-70): Study the following Table carefully

o
sold at the given rate while half the quantity is sold on a

o
and answer the question given below :

b
price increased by 20%. What is the total cost of entire

.w
quantity of sugar sold?

o
MARKS OBTAINED BY DIFFERENT STUDENTS

rd
(a) ` 23,100/ (b) ` 23,800/
IN DIFFERENT SUBJECTS

p
(c) ` 22,400/ (d) ` 23,400/

re
(e) None of these

s
SUBJECTS

s
.c
59. If the quantities sold of Groundnut and Cumin are (Maximum Marks)
Students

o
Hindi English Maths S.St. Science Sanskrit Phy.
interchanged, what will be the total cost of quantities sold

m
(100) (100) (100) (100) (75) (50) Edu
for these two items together? (75)
(a) ` 62,000/ (b) ` 60000/ Anupama 85 95 87 87 65 35 71
(c) ` 65,000/ (d) ` 63,000/ Bhaskar 72 97 55 77 62 41 64
Charu 64 78 74 63 55 25 53
(e) None of these
Deepak 65 62 69 81 70 40 50
60. If the quantity of corriander sold is increased by 200% and
Garima 92 82 81 79 49 30 61
the price per kg. is reduced by 8%, what will be the total Vishal 55 70 65 69 44 28 30
cost of corriander sold?
(a) ` 38, 460/ (b) ` 36,480/ 66. How many students have scored the lowest marks in two
(c) ` 38,640/ (d) ` 36,840/ or more subjects ?
(e) None of these (a) 2 (b) 3
(c) 1 (d) 0
DIRECTIONS (Qs. 61-65): Study the following information (e) 4
carefully to answer these questions : 67. Who has scored the highest marks in all the subjects
For an examination consisting of three subjects-Maths, Physics together ?
(a) Deepak (b) Charu
and Chemistry, 280 students appeared. When the results were
(c) Anupama (d) Garima
declared, 185 students had passed in Maths, 210 had passed
(e) Bhaskar
in Physics and 222 had passed in Chemistry. 68. What is the percentage of Deepak's marks (upto two digits
All those except 5 students who passed in Maths, passed in after decimal) in all the subjects together ?
Physics. (a) 88.63 (b) 77.38
All those except 10 students who passed in Maths, passed in (c) 67.83 (d) 62.83
Chemistry. (e) 72.83
47 students failed in all the three subjects. 69. Marks obtained by Charu in Hindi are what percent of
200 students who passed in Physics also passed in Chemistry. marks (upto two digits after decimal) obtained by Anupama
61. How many students passed in Chemistry only ? in the same subject ?
(a) 18 (b) 21 (a) 75.92 (b) 78.38
(c) 77.29 (d) 75.29
(c) 25 (d) 29
(e) 72.83
(e) None of these
70. What are the average marks obtained by all the students
62. How many students passed in all the three subjects ? together in Science ?
(a) 185 (b) 175 (a) 55.75 (b) 57.5
(c) 170 (d) 171 (c) 60 (d) 59.5
(e) Cannot be determined (e) 58
63. How many students failed in Physics and Maths ?
(a) 65 (b) 18 DIRECTIONS (Qs. 71-75): What should come in place of the
question mark (?) in the following number series ?
(c) 58 (d) 47
(e) Cannot be determined 71. 104 109 99 114 94 ?
64. How many students passed in Maths but failed in both (a) 69 (b) 124
Physics and Chemistry ? (c) 120 (d) 78
(a) 5 (b) 10 (e) None of these
(c) 15 (d) 1 72. 980 392 156.8 ? 25.088 10.0352
(a) 65.04 (b) 60.28
(e) Cannot be determined
(c) 62.72 (d) 63.85
65. A student has to pass in at least two subjects to get (e) None of these
promoted. How many students get promoted ? 73. 14 16 35 109 441 ?
(a) 180 (b) 213 (a) 2651 (b) 2205
(c) 200 (d) 185 (c) 2315 (d) 2211
(e) Cannot be determined (e) None of these
y
o
u
rs
m
492 IBPS PO Main Exam 2015

a
h
74. 1331 2197 4913 6859 ? 24389

b
84. I. 3p + 2q – 58 = 0

o
(a) 12167 (b) 13824 II. 4q + 4p = 92

o
b
(c) 9261 (d) 15625 85. I. 3p2 + 17p + 10 = 0

.w
(e) None of these II. 10q2 + 9q + 2 = 0

o
75.. 3600 725 150 35 12 ?

rd
(a) 8 (b) 7.4 DIRECTIONS (Qs.86-90): In the following graph the number

p
re
(c) 10.5 (d) 10 of laptops manufactured by six different companies in the years

s
(e) None of these 2013 and 2014 has been given. Read the graph carefully and

s
.c
76. If the numerator of a fraction is increased by 350% and answer the questions.

o
the denominator of the fraction is increased by 300% the

m
Number of laptops (in thousands)
9
resultant fraction is . What is the original fraction ? manufactured by 6 different companies
22
3 5 Samsung
(a) (b)
4 12
7 4 40
(c) (d)
9 11 35
(e) None of these
30
77. The length of rectangular plot is thrice its breadth. If the Dell Apple
25
area of the rectangular plot is 6075 sq. metres,what is its
length? 20

(a) 145 metres (b) 130 metres 15

(c) 75 metres (d) 45 metres 10

(e) None of these 5


78. Ms. Sujata invests 7% i.e. ` 2170 of her monthly salary in
0
mutual funds. Later she invests 18% of her monthly salary 5
10 5
in recurring deposits also, she invests 6% of her salary on 15
20 10
NSC's. What is the total annual amount invested by 25
30 15
Ms. Sujata ?
35
(a) ` 1,25,320 (b) ` 1,13,520 40 20
HP
Lenovo
(c) ` 1,35,120 (d) ` 1,15,320 25
(e) None of these 30
79. In how many different ways can the letters of the word 35
'PRIDE' be arranged ? 40
(a) 60 (b) 120
Abascus
(c) 15 (d) 360
2014 2013
(e) None of these
86. The respective ratio between the number of laptops
80. The ages of Samir and Tanuj are in the ratio of 8 : 15 years
manufactured by Lenovo in 2013 and that by Abascus in
respectively. After 9 years the ratio of their ages will be 11 :
2014 is
18. What is the difference in years between their ages ?
(a) 24 years (b) 20 years (a) 8 : 7 (b) 7 : 8
(c) 33 years (d) 21 years (c) 3 : 5 (d) 5 : 3
(e) None of these (e) None of these
87. What is the average number of laptops (in thousands)
DIRECTIONS (Qs. 81-85): For the two given equations
manufactured by all companies taken together in 2013 ?
I and II.
(a) 22 (b) 22.5
Give answer (1) if p is greater than q. (c) 32.5 (d) 23.5
Give answer (2) if p is smaller than q. (e) 27.5
Give answer (3) if p is equal q. 88. What is the percentage increase in production of laptops
Give answer (4) if p is either equal to or greater than q. by HP in 2014 in comparison to that in 2013 ?
Give answer (5) if p is either equal or smaller than q.
(a) 125 (b) 100
81. I. p2 + 5p + 6 = 0
II. q2 + 3q + 2 = 0 (c) 150 (d) 250
82. I. p2 = 4 (e) None of these
II. q2 + 4q = – 4 89. The difference between the number of laptops manufactured
83. I. p2 + p = 56 by Apple, Lenovo and Samsung in 2013 and that by Dell,
II. q2 – 17q + 72 = 0 HP and Abascus in 2014 is
y
o
u
rs
m
IBPS PO Main Exam 2015 493

a
h
b
(a) 5500 (b) 4550 99. If tax on a commodity is reduced by 10%, total revenue

o
(c) 3550 (d) 4500 remains unchanged. What is the percentage increase in its

o
b
(e) 5000 consumption?

.w
90. In 2014, which company manufactured the maximum number

o
1

rd
of laptops? (a) 11 % (b) 20%
9

p
(a) Abascus (b) Lenovo

re
1

s
(c) Dell (d) Samsung
(c) 10% (d) 9 %

s
.c
(e) HP 11

o
(e) None of these

m
DIRECTIONS (Qs. 91-95): What value will come in place of
100. The denominators of two fractions are 5 and 7 respectively.
the question mark (?) in the following questions.
41
1 The sum of these fractions is . On interchanging the
91. 7072 ÷ (16% of 884) = 30 × 1 of (? ÷ 39) 35
12
(a) 60 (b) 62 43
numerators, their sum becomes . The fractions are
(c) 65 (d) 55 35
(e) 67 2 4 3 4
92. (562.5 × 6)6 ÷ (135 ÷ 9)10 ÷ (37.5 × 6)7 = (3.75 × 4)? – 6 (a) and (b) and
5 7 5 7
(a) 0 (b) 2
(c) 3 (d) 4 4 2 3 5
(c) and (d) and
(e) 5 5 7 5 7
93. 1478.4 ÷ 56 + 66.8 × 57 = (? × 3) + (34 × 34.5 ) (e) None of these
(a) 785 (b) 887
(c) 889 (d) 989
(e) 885 ENGLISH LANGUAGE
94. (13 + 2 5) 2 = ? 5 + 189 DIRECTIONS (Qs. 101-115): Read the following passage
(a) 26 (b) 25 based on an Interview to answer the given questions based on
(c) 52 (d) 130 it. Certain words are printed in bold to help you locate them
(e) None of these while answering some of the questions.
95. 0.2 × 1.1 + 0.6 × 0.009 = ? – 313.06 A spate of farmer sucides linked to harassment by
(a) 353.2184 (b) 353.2854 recovery agents employed by Micro Finance Institutions
(c) 331.54 (d) 313.2854 (MFIs) in Andhra Pradesh spurred the state government
(e) 331.2854 to bring in regulation to protect consumer interests. But,
96. Sujata scored 2240 marks in an examination that is 128 marks while the Bill has brought into sharp focus the need for
more than the minimum passing percentage of 64%. What is consumer protection, it tries to micro-manage MFI
the percentage of marks obtained by Meena if she scores operations and in the process it could scuttle some of the
907 marks less than Sujata? crucial benefits that MFls bring to farmers, says the
(a) 35 (b) 40 author of Microfinance India, State of the Sector Report
(c) 45 (d) 36 2010. In an interview he points out that prudent
(e) 48 regulation can ensure the original goal of the MFIs–social
97. There are 8 brown balls, 4 orange balls and 5 black balls in a uplift of the poor.
bag. Five balls are chosen at random. What is the probability Do you feel the AP Bill to regulate MFIs is well thought
of their being 2 brown balls, 1 orange ball and 2 black balls? out ? Does it ensure fairness to the borrowers and the
long-term health of the sector ?
191 180 The AP bill has brought into sharp focus the need for
(a) (b)
1547 1547 customer protection in four critical areas First is pricing.
280 189 Second is tender’s liability — whether the Lender can
(c) (d) give too much loan without assessing the customer’s
1547 1547
(e) None of these ability to pay. Third is the structure of loan repayment –
98. The ratio of the salaries of A and B is 8 : 9. If A's salary is whether you can ask money on a weekly basis from people
increased by 50% and B's salary is reduced by 25%, their who don’t produce weekly incomes. Fourth is the
ratio becomes 16 : 9. What is the salary of A ? practices that attend to how you deal with defaults.
But the Act should have looked at the positive benefits
(a) ` 22000 (b) ` 28500
that institutions could bring in, and where they need to
(c) ` 37000 (d) Cannot be determined
be regulated in the interests of the customers. It should
(e) None of these have brought only those features in.
y
o
u
rs
m
494 IBPS PO Main Exam 2015

a
h
b
Say, you want the recovery practices to be consistent certain amount of time. MFIs went to markets perhaps

o
with what the customer can really manage. If the customer because of the need to grow too big too fast. The

o
b
is aggrieved and complains that somebody is harassing government thought they were making profit off the poor,

.w
him, then those complaints should be investigated by the and that’s an indirect reasons why they decided to clamp

o
District Rural Development Authority. down on MFIs. If you say an MFI won’t go to capital

rd
Instead what the Bill says is that MFIs cannot go to the market, then it will keep political compulsions under

p
re
customer’s premises to ask for recovery and that all rein.

s
transactions will be done in the Panchayat office. With 101. Which of the following best explains ‘Structure of loan

s
.c
great difficulty, MFIs brought services to the door of repayment’ in the context of the first question asked to

o
the author ?

m
people. It is such a relief for the customers not to be
spending time out going to banks or Panchayat offices, (a) Higher interest rate
which could be 10 km away in some cases. A facility (b) Payment on weekly basis
which has brought some relief to people is being shut. (c) Giving loan without assessing ability to pay
(d) Method of dealing with defaults
Moreover, you are practically telling the MFI where it
(e) Total amount of loan
should do business and how it should do it.
102. The author is of the view that _____
Social responsibilities were in-built when the MFIs
(a) the bill to regulate MFIs is not needed
were first conceived. If MFIs go for profit with loose
(b) the bill neglects the interests of the customers
regulations, how are they different from moneylenders? (c) the positive aspects of MFIs should also be considered.
Even among moneylenders there are very good people (d) most of the MFIs are not good.
who take care of the customer’s circumstance, and there (e) MFIs must be told what and where they should do
are really bad ones. A large number of the MFIs are good business
and there are some who are coercive because of the kind 103. One of the distinct positive feature of MFIs is that _____.
of prices and processes they have adopted. But (a) they brought services to the door of people
Moneylenders never got this organised. They did not have (b) they dealt with defaulters very firmly
such a large footprint. An MFI brought in organisations, (c) they provided adequate customer protection
it mobilized the equity, it brought in commercial funding. (d) they are governed by the local people
It invested in systems. It appointed a large number of (e) they have highly flexible repayment plan
people. But some of then exacted a much higher price 104. What is the difference between MFIs and moneylenders ?
than they should have. They wanted to break even very (a) There is no difference.
fast and greed did take over in some cases. (b) A large number of money lenders are good whereas
Are the for-profit MFIs the only ones harassing people only a few MFIs are good
for recoveries ? (c) Money lenders gave credit at lower rate of interest
Some not-for-profit outfits have also adopted the same than that of MFIs
kind of recovery methods. That may be because you have (d) MFIs adopted a structure and put a process in place,
to show that you are very efficient in your recovery which was not the case with moneylenders
methods and that your portfoilo is of a very high quality (e) Moneylender appointed large number of local people
if you want to get commercial funding from a bank. as against more outside people in MFIs
In fact, among for-profits there are many who have 105. Which of the following is positive outcome of the AP Bill
sensible recovery practices. Some have fortnightly recovery, to regulate MFIs ?
(a) The banks have started this service in remote areas
some have monthly recovery. So we have differing
(b) It highlighted some areas of customer protection
practices. We just describe a few dominant ones and
(c) It highlighted the bad practices being followed by
assume every for-profit MFI operates like that. moneylenders
How can you introduce regulations to ensure social (d) MFIs is invested in systems and broguth in
upliftment in a sector that is moving towards for -profit commercial funding.
models ? (e) It will help convert MFIs into small banks
I am not really concerned whether someone wants to make 106. The author is recommending ____.
a profit or not. The bottom-line for me is customer (a) Not-for profit MFIs
protection. The first area is fair practices. Are you telling (b) For-profit MFIs
your customers how the loan is structured ? Are you being (c) Stoppage of commercial funding to MFIs
transparent about your performance ? There should also (d) Customer satisfaction irrespective of ‘Not-for profit’
be a lender’s lilability attached to what you do. Suppose or ‘for profit’ MFIs
you lend excessively to a customer without assessing their (e) Public sector promoted MFIs
ability to service the loan, you have to take the hit. 107. Why did MFIs go to the equity markers ?
Then there’s the question of limiting returns. You can say (a) To repay the loan
that an MFI cannot have a return on assets more than X, (b) To lower interest rate
a return on equity of more than Y. Then suppose there (c) There were political compulsions
is a privately promoted MFI, there should be a regulation (d) To become a public sector institution
to ensure the MFI cannot access equity markets till a (e) To grow very fast
y
o
u
rs
m
IBPS PO Main Exam 2015 495

a
h
b
108. Which of the following has not been indicated as one of to form a meaningful paragraph; then answer the questions given

o
the features of air practices for customer protection ? below them.

o
b
(a) Providing information about loan structuring.

.w
(A) There are a number of item in the atomic energy programme
(b) MFIs should also be held liable for some of their
which are being made indigenously.

o
actions

rd
(B) Given the overall energy situation in India, the use of
(c) Not to raise money from capital market

p
re
(d) MFIs should also inform public about their own nuclear power in some measure is inescapable even while

s
performance also thermal and hydro power continue to be the dominant

s
.c
(e) To provide credit as per the rational assessment of elements.

o
m
their ability to service the loan (C) However, commercial aspects of exploiting nuclear
109. Which of the following could possibly be most plausible capabilities, especially for power-generation programmes,
reason for banning recovery by going to customer’s have been recently given high priority.
premises ? (D) Atomic energy programmes have been subject to severe
(a) To protect the family members restrictions for every obvious reason as the Department of
(b) To protect the customer from harassment and
Atomic energy is becoming self-reliant in areas in which
coercion
only a few countrices have such capability.
(c) To reduce the undue expenses of MFIs is resulting
in lower interest rates. (E) Even to meet these nuclear power requirements, India
(d) To account systematically the money recovered in the critically requires a commercia level power-generation
books of accounts capability, with its commensurate safety and nuclear waste
(e) To keep Panchayat office out of these transactions management arrangements.
(F) Thus, in Indian context energy security is also crucial,
DIRECTIONS (Qs. 110-113) : Choose the word which is most
nearly the same in meaning as the word/group of words printed perhaps much more than it is for the U.S.A.. because India
in bold. imports a good part of its crude oil requirements, paying
for it with precious foreign exchange.
110. manage
116. Which of the following will be the FIFTH sentence after
(a) afford (b) assess
(c) thrust (d) administer rearrangement ?
(e) use (a) (A) (b) (B)
111. exacted (c) (C) (d) (D)
(a) perfected (b) demanded (e) (E)
(c) estimated (d) corrected 117. Which of the following will be the THIRD sentence after
(e) accurate rearrangement ?
112. scuttle (a) (A) (b) (B)
(a) delay (b) mix (c) (C) (d) (D)
(c) shuffle (d) destroy
(e) (E)
(e) smoothen
118. Which of the following will be the SECOND sentence after
113. spurred
(a) agitated (b) instigated rearrangement ?
(c) reflected (d) disapproved (a) (A) (b) (B)
(e) prompted (c) (C) (d) (D)
(e) (E)
DIRECTIONS (Qs. 114-115) : Choose the word or group of
119. Which of the following will be the FIRST sentence after
words which is most opposite in meaning of the word printed
in bold. rearrangement ?
(a) (A) (b) (B)
114. under rein
(c) (C) (d) (D)
(a) under wrap
(e) (E)
(b) without target
120. Which of the following will be the FOURTH sentence after
(c) let loose
rearrangement ?
(d) no cloud
(a) (A) (b) (B)
(e) under cloud
(c) (C) (d) (D)
115. coercive
(e) (E)
(a) gentle (b) promoting
(c) progressive (d) natural DIRECTIONS (Qs. 121-125) : In each question below, two
(e) opinionated sentences or parts of sentences are given with two blank spaces
( ____ ) ( _____ ) between them. From among the three
DIRECTIONS (Qs. 116-120) : Rearrange are following six parts / sentences denoted by (A), (B) and (C) given below each,
sentences (A), (B), (C), (D), (E) and (F) in the proper sequence find out which two can fill up the blanks in proper sequence
y
o
u
rs
m
496 IBPS PO Main Exam 2015

a
h
b
(i.e. the order in which these are given in the answer options) DIRECTIONS (Qs. 126-130): Read each sentence given below

o
to make the sentence/ group of sentences meaningfully

o
and find out whether there is an error in it. The error if any

b
complete. will be one of the sentence which are marked as A, B, C and

.w
121. Four years ago, I had a major surgery. (____) . (____) . D. If there is no error, the answer will be (E) i.e. No error.

o
rd
I was too ill. (Ignore the errors of punctuation, if any)

p
(A) It left me disabled.

re
126. The low learnings levels is due to the fact (a)/ that the
(B) My attempts to go back to work could not succeed.

s
state spends 87% of its budget (b)/ on salaries of its

s
.c
(C) I had fully recovered except for some minor teachers (c)/ and not on infrastructure development for

o
weakness. students (e)/ No error (e).

m
(a) (B) and (A) only (b) (C) and (A) only 127. Recent incidents of tigers straying have brought to focus
(c) (B) and (C) only (d) (A) and (B) only (a)/ the lack of proper regulatory mechanism and powers
(e) (A) and (C) only with the forest department (b)/ to take action against the
122. Based on scientific research, scientists conclude that (___ ) resorts (c)/ mushroom in forest fringes (d)/ No error (e).
(____), they will live even more than a hundred years. 128. The beauty of the palace comes alive (a)/ When over a
(A) keep themselves active all through lakh bulbs (b)/ is switched on between 7pm and 7.45pm
(B) exercise more, ensure proper vitamin intake, (c)/ on specific days (d)/ No error (e)
(C) human species, if regulate their diet, 129. In view of the rising complaints (a)/ of unscrupulous
(a) Only (A) and (B) (b) Only (A) and (C) financial institutes duping people with luring them (b)/
(c) Only (C) and (A) (d) Only (C) and (B) with handsome returns on their investment, the police
(e) Either C and (A) or (C) and (B) have appealed (c)/ to the citizens to stay away from such
123. The Central Government’s plans to (_____) (_____) companies (d)/ No error (e)
despite the fact that the opposition parties’ united front 130. More and more cab drivers are approaching the regional
had mobilized support of general public at the outskirts transport office (RTO) (a)/ to obtain identity cards (b)/
of Mumbai. after the transport office intensified action against errant
(A) defy the curfew imposed to prevent the proposed drivers (c)/ in the last couple of months (d)/ No error (e).
rally from protesting against the hike in petrol prices DIRECTIONS (Qs. 131-135): Fill in the blank with most
(B) curb the rally called for by the opposition succeeded appropriate words from the given options.
due to ruling party’s an nouncement to
(C) cancel all the trains entering Mumbai, Tibet ______ up images of a mystic land. Snow-capped
(a) (A) and (B) only (b) (B) and (A) only mountain peaks pierce the blue sky and fierce chilly winds
sweep the rolling grasslands. Maroon-robed Buddhist monks
(c) (B) and (C) only (d) (A) and (C) only
pray in remote monasteries and _______ horsemen pound the
(e) Either (B) and (C) or (A) and (C)
rugged earth. People in this high plateau perform punishing
124. The economic gloom was exacerbated yesterday by Greek
rituals like prostrating hundreds of miles in tattered clothes on
Prime Minister’s surprise announcement (_____)(____), pilgrimage. Spirits, spells and flying apparitions are part of the
and vote might put the torturously conceived package in Tibetan world. In short, Tibet remains an exotica. Such images
jeopardy. are largely the result of books by Western travellers and
(A) that was struck last week explorers in the last century, which helped in keeping the
(B) that he would recommend the Parliament to strike mystique alive. And when the Communist rulers took over Tibet
down the obnoxious deal in the 1950s and began _______ Chinese language and culture
(C) that his country would hold a referendum on the on the people, Tibet’s own history started to ______ in the
European debt deal background. Thus, the only books available in English to
(a) (C) and (A) only Tsering Wangmo Dhompa as a young girl growing up in India
(b) (B) and (A) only and Nepal as a refugee ________ those written by Westerners,
(c) (A) and (C) only and so she came to view the country as a forbidden land, a place
(d) Either (C) and (A) or (B) and (A) where fantasy and fable collaborated against a dramatic
(e) None of these backdrop of mountains, black magic and people with strange
125. The world’s most powerful nations were warned (____) customs and appearances.
(___) and trigger mass social unrest. 131. (a) makes (b) conjures
(A) that could cost millions of jobs around the globe (c) puts (d) toil
(B) that international economy was on the brink of a (e) appoints
deep new economic crisis 132. (a) sturdy (b) wobbly
(C) that the only solution was pumping a huge amount (c) handsome (d) herculean
of money In the economy (e) beautiful
(a) (C) and (A) only (b) (B) and (A) only 133. (a) implementing (b) evading
(c) (B) and (C) only (d) (C) and (B) only (c) imposing (d) experimenting
(e) None of these (e) all of these
y
o
u
rs
m
IBPS PO Main Exam 2015 497

a
h
134. (a) amplify (b) Stretch

b
139. Which of the following is most OPPOSITE in meaning of

o
(c) die (d) recede the word given in bold as used in the passage? INFLUX

o
b
(e) increase (a) Invasion (b) Enlargement

.w
135. (a) are (b) have been (c) Advance (d) Incline

o
rd
(c) was (d) were (e) Ebb

p
(e) are 140. Which of the following is/are true according to the

re
passage?

s
DIRECTIONS (Qs. 136-140): Read the passage carefully and

s
(a) With each passing day, banking is becoming

.c
answer the questions given below it. Certain words/ phrases

o
narrower.

m
have been given in bold to help you locate them while (b) Smart users today have round-the-clock access to
answering some of the questions. their bank accounts and carry their back in their
Mobile technology is transforming the global banking and pockets.
payment industry by providing added convenience to existing (c) Only limited customers can avail all services of
bank customers in developed markets, and by offering new bank.
services to the unbanked customers in emerging markets. While (d) All of the above
consumers and governments are keen to adopt mobile (e) None of these
technology for government-to-person (G2P) payments,
intermediaries are creating barriers as they end up losing a good COMPUTER KNOWLEDGE
source of income.
Any new product for the G2P market needs to consider the 141. When sending an e-mail, the … line describe the contents
incentives and motivations of all parties involved in the current of the message
value chain. As banks, mobile network operators (MNOs), (a) Subject (b) To
NGOs and for-profit firms build new services to seize the (c) Contents (d) CC
opportunity to reach the large unbanked population, they must (e) None of these
take time to understand the needs of customers. Even within 142. All the deleted files go to
a country, there are distinct differences in customer needs in (a) Recycle bin (b) Task bar
urban and rural areas, and across segments. This has the (c) Tool bar (d) My computer
potential to unlock a large untapped market. This opportunity (e) None of these
has attracted several new players ranging from MNOs and 143. You organise files by storing them in
start-ups to companies from adjacent industries such as retail, (a) Archives (b) Folders
each trying its own business model to succeed in this new (c) Indexes (d) Lists
world. The influx of so many players and services has created (e) None of these
confusion for customers, lack of coordination among players 144. A website address is a unique name that identifies a
and limited scale for a single company. History tells us that after specific ___ on the web
the initial stage of confusion, the dust eventually settles down (a) Web browser (b) Website
as a few winners emerge. (c) PDA d) Link (e) None of these
136. What does the author mean by ‘unbanked customer’? 145. _________ are specially designed computer chips that
(a) Not having access to the services of a bank. reside inside o ther devices, such as your car or your
(b) A person who buys goods or services from a electronic thermostat
financial institution. (a) Servers (b) Embedded computers
(c) Robotic computers (d) Mainframes
(c) A customer of a specified kind with whom one has
(e) None of these
to deal.
146. Which of the following places the common data elements
(d) Both 2 and 3 in order from smallest to largest?
(e) Other than given options. (a) Byte, Bit, Character, file, record, field, database
137. Which of the following is possibly the most appropriate (b) Character, record, field, file, database
title for the passage? (c) Character, field, record, file, database \
(a) Progress on banking (d) Bit, byte, character, record, field, file, database
(b) Banking in the future (e) None of these
(c) Mobile banking 147. A disk's content that is recorded at the time of
manufacture and that cannot be changed or erased by the
(d) Integration between e-commerce firms and banks user is
(e) Instant banking (a) Memory only (b) Write only
138. Which of the following is most SIMILAR in meaning of (c) Read only (d) Run only
the word given in bold as used in the passage? (e) None of these
UNTAPPED 148. An error in a computer program
(a) Final (b) Fresh (a) Crash (b) Power Failure
(c) Concluding (d) Latest (c) Bug (d) Virus
(e) Last (e) None of these
y
o
u
rs
m
498 IBPS PO Main Exam 2015

a
h
b
149. Distributed processing involves 158. The simultaneous processing of two or more programs by

o
(a) solving computer component problems from a multiple processors is

o
b
different computer (a) multiprogramming (b) multitasking

.w
(b) solving computing problems by breaking them into (c) time-sharing (d) multiprocessing

o
rd
smaller parts that are separately processed by (e) None of these

p
different computers 159. The secret code that restricts entry to some programs

re
(c) allowing users to share files on a network

s
(a) Password (b) Passport

s
(d) allowing users to access network resources away

.c
(c) Entry code (d) Access code

o
from the office
(e) None of these

m
(e) None of these
160. Computers use the ________ number system to store data
150. The operating system determines the manner in which all and perform calculations.
of the following occurs except
(a) Binary (b) Octal
(a) user creation of a document
(c) Decimal (d) Hexadecimal
(b) user interaction with the processor
(e) None of these
(c) printer output
(d) data displayed on the monitor
(e) None of these GENERAL AWARENESS
151. Office LANs that are spread geographically apart on a (With to special referance to Banking)
large scale can be connected using a corporate 161. Which is the highest populated state of India?
(a) CAN (b) LAN
(a) Madhya Pradesh (b) Uttar Pradesh
(c) DAN (d) WAN
(e) None of these (c) Bihar (d) Maharashtra
152. You can keep your personal files/folders in (e) Haryana
(a) My folder (b) My documents 162. Who is the head of the panel formed RBI to look into
(c) My files (d) My text parameters for urban co-operative banks recently?
(e) None of these (a) Bimal Jalan (b) R Gandhi
153. A directory within a directory is called
(c) Urijit Patel (d) YV Reddy
(a) Mini Directory (b) Junior Directory
(c) Part Directory (d) Sub Directory (e) None of these
(e) None of these 163. Which banks have been designated as Domestic
154. A compiler translates a program written in a high-level Systemically Important Banks (D-SIBs)?
language into (a) ICICI& SBI (b) PNB & SBI
(a) Machine language (b) An algorithm
(c) HDFC & SBI (d) ICICI & PNB
(c) A debugged program (d) Java
(e) None of these (e) None of these
155. A ___________ is a unique name that you give to a file 164. Bhakti Sharma, who is in news recently associated with
of information which sports?
(a) Device letter (b) Folders (a) Swimming (b) Tennis
(c) File name (d) File name extension
(c) Badminton (d) Volley Ball
(e) None of these
(e) None of these
156. Hardware includes
(a) all devices used to input data into a computer 165. Consider the following statements:
(b) sets of instructions that a computer runs or executes A. Yog Guru Baba Ramdev was appointed as state
(c) the computer and all the devices connected to it that brand ambassador of Haryana Government
are used to input and output data B. It was announced by Minister of Sports and Health
(d) all devices involved in processing information of Haryana, Anil Vij
including the central processing unit, memory and
storage C. Haryana Govt took this decision as it want to
(e) None of these promote yoga and ayurveda in Haryana State
157. A ______ contains specific rules and words t hat express Which of the above statement is/are True
the logical steps of an algorithm (a) Only C (b) Both A and C
(a) Programming language (b) Syntax error
(c) Both A and B (d) All A, B, C
(c) Programming structure (d) Logic chart
(e) None of these (e) None of these
y
o
u
rs
m
IBPS PO Main Exam 2015 499

a
h
b
166. Frank Tyson was a cricketer from which country? (c) Ministry of Finance

o
o
(a) New Zeland (b) England (d) RBI

b
.w
(e) Interest Rate Commission of India
(c) West Indies (d) South Africa
176. Where is the Head-quarter of Indian Bank?

o
rd
(e) None of these (a) Chennai (b) Mumbai

p
167. What is the full form of NBFC as used in the financial (c) Delhi (d) Bengaluru

re
sector?

s
(e) Hyderabad

s
.c
(a) New Banking Finance Company 177. Many a times, we read in newspaper that the RBI has

o
(b) National Banking & Finance Corporation revised certain rates/ratios under LAF. What is full form

m
(c) Non Business Fund Company of LAF?
(a) Legal Adjustment Facility
(d) New Business Finance & Credit
(b) Liquidity Adjustment Facility
(e) None of these
(c) Longterm Achievement Facility
168. Which of the following fund transfer mechanisms, can be (d) Legal Adjustment Formality
moved from one bank to another and where transaction (e) None of these
is settled instantly without being bunched with any other
178. What is KVP lock period?
transaction?
(a) 36 Months (b) 48 Months
(a) RTGS (b) NEFT (c) 24Months (d) 30 Months
(c) TT (d) EFT (e) 40 Months
(e) MT 179. Which of the following statement is true about the
169. Lima is the capital of Competition Commission?
(a) Brazil (b) Peru 1. The Competition has been established to prevent
practices which do not support healthy business
(c) Ecuador (d) Colombia
competition.
(e) None of these 2. The Commission will have five members including
170. Where is Buxa National Park located? the chairman
(a) Odisha (b) West Bengal 3. The Commission has to ensure that the financial
(c) Madhya Pradesh (d) Bihar operation of any business entity does not get
(e) Rajasthan concentrated in the hands of few people.
(a) Only 1 (b) Only 2
171. English Stephen Constantine has been appointed as the
(c) Only 3 (d) All of them
head coach of Indian men's ____________ team.
(e) None of them
(a) Hockey (b) Cricket
180. What is the cost of Delhi National Memorial Cost?
(c) Football (d) Badminton
(a) 100 crore (b) 1000 crore
(e) None of these (c) 500 crore (d) 250 crore
172. Banking ombudsmen is appointed by (e) None of these
(a) SBI (b) Indian Government 181. RuPay is launched by
(c) RBI (d) President (a) RBI (b) Finance Ministry
(e) Prime minister (c) SBI (d) NPCI
173. Which committee framed the RRB Act? (e) None of these
(a) Narsimham Committee 182. The __________ committee on minimum alternate tax
(b) Ashok Mehta Committee (MAT) has recommended to the finance ministry that the
levy shouldn't be imposed for the period preceding Apr
(c) Sachar Committee 1, 2015.
(d) Shah Nawaz Committee (a) Altamas Kabir (b) HL Dattu
(e) None of these (c) AP Shah (d) Rajendra Mal Lodha
174. Ravindra Jain who passed recently was a (e) Mukul Mudgal
(a) Politician (b) Singer 183. India got its first private bank in 11 years as the Bandhan
(c) Author (d) Music Composer Bank commenced operations on 23 Aug with 501
(e) None of these branches. What is the punch line of the bank?
(a) Apna Bhala, Aapki Bhalai
175. Base rate is the rate below which no bank can allow their
(b) Aapka Bhala, Sabki Bhalai
lending to anyone. Who set up this base rate for banks?
(c) Banking the Unbanked
(a) Individual Banks Boards (Correct Answer)
(d) Khayal Aapka
(b) Ministry of Commerce
(e) None of these
y
o
u
rs
m
500 IBPS PO Main Exam 2015

a
h
b
184. Where is BIS (Bank for International Settlements) located? 194. Which film got most of the awards in BAFTA Awards

o
(a) Switzerland (b) USA 2015?

o
b
(c) UK (d) India (a) Birdman

.w
(e) None of these (b) The Grand Budapest Hotel

o
rd
185. Who won the 100m gold at World Championships 2015 (c) The Imitation Game

p
held in Beijing on 23 Aug?
(d) The Theory of Every Thing

re
(a) Su Bingtian (b) Usain Bolt

s
(e) Boyhood

s
(c) Andre de Grassse (d) Trayvon Bromell

.c
195. What is India's ranking in World Press Freedom Index

o
(e) Justin Gatlin

m
(WPFI) 2015?
186. Where is the headquarters of Central bank of India
located? (a) 96th (b) 121st
(a) New Delhi (b) Mumbai (c) 136th (d) 145th
(c) Pune (d) Bangalore (e) 130th
(e) None of these 196. Which committee was constituted for recommendations
187. Which IT firm in India has unveiled three new service on bringing reforms in central public sector enterprises
lines called Aikido, a combination of three words - Ai, (CPSEs)?
Ki and Do, which in Japanese mean combining, spirit and
(a) Roongta committee
path/way?
(a) Wipro (b) TCS (b) Mathur Committee
(c) Infosys (d) Tech Mahindra (c) Vishwanathan committee
(e) HCL (d) Madhava Menon committee
188. The __________ committee on minimum alternate tax (e) None of these
(MAT) has recommended to the finance ministry that the 197. Who has been conferred with Khushwant Singh Memorial
levy shouldn't be imposed for the period preceding Apr Prize for Poetry?
1, 2015.
(a) Keki N. Daruwalla
(a) Altamas Kabir (b) HL Dattu
(c) AP Shah (d) Rajendra Mal Lodha (b) Joy Goswami
(e) Mukul Mudgal (c) Ranjit Hoskote
189. Bad advances of a bank are called (d) Arundhathi Subramaniam
(a) standard accounts (e) None of these
(b) book debt 198. –––––––– signed 50:50 Joint Venture for TV Commerce
(c) non performing assets with DEN Network
(d) out of order account (a) Flipkart (b) Snapdeal
(e) overdrawn accounts (c) Myntra (d) Ebay
190. 'FLASHremit' service to offer an instant bank transfer (e) Olx
service to the Indians residing in the Gulf nation has been
launched by which bank in association with UAE 199. Many a times, we read about Special Drawing Right
exchange? (SDR) in newspapers. As per its definition, SDR is a
(a) HDFC bank (b) SBI monetary unit of the reserve assets of which of the
(c) Axis bank (d) ICICI bank following organizations/agencies?
(e) None of these (a) World Bank
191. Betla national park is located at? (b) International Monetary Funds (IMF)
(a) Jharkhand (b) West Bengal (c) Asian Development Bank
(c) Madhya Pradesh (d) Kerala (d) Reserve Bank of India
(e) None of these (e) None of these
192. Federal Reserve is the Central Bank of which country? 200. Nationalizations of banks aimed at all of the following
(a) France (b) China except
(c) Germany (d)United States of America
(a) Provision of adequate credit for agriculture, SME &
(e) None of these
Exports
193. Who is the author of the book "India Shastra: Reflection
(b) Removal of control by a few capitalists
of the Nation in Our Time"?
(c) provision of credit to big industries only (Correct
(a) Arvind Kejriwal (b) Manmohan Singh
(c) Shashi Tharoor (d) LK Advani Answer)
(e) none of these (d) Access of banking to masses
(e) Encouragement of a new class of entrepreneurs
y
o
u
rs
m
IBPS PO Main Exam 2015 501

a
h
b
o
o
b
.w
o
1. (e) order but in reverse order while the numbers are

rd
From statement I rearranged in descending order from the right.

p
re
(6-7) :
going to a party ® la fa qu tu

s
Input : always 19 give 21 84 for 62 14 worthy cause

s
.c
Step I : worthy always 19 give 21 for 62 14 cause 84

o
For a party fa me tu

m
Step II : worthy give always 19 21 for 14 cause 62 84
from statement II
Step III : worthy give for always 19 14 cause 21 62 84
start the party tu Co ra Step IV : worthy give for cause always 14 19 21 62 84
going to start qu Co la
6. (d) Option (d) is the Step III.
From both the statements 7. (c) Four steps are needed to complete the arrangement.
The code for party is tu (8-10) :
2. (d) From statement I Input : 50 62 tips on 67 how can 42 stay young 17 89
According to Amita, Priya is travelling on forever 03
Wednesday. Thursday or Friday. Step I : young 50 62 tips on 67 how can 42 stay 17 forever
According to Rohit, Priya is travelling on Tuesday, 03 89
Wednesday or Thursday. Step II : young tips 50 62 on how can 42 stay 17 forever
From both the statements 03 67 89
Priya is travelling on Wednesday or Thursday. Step III : young tips stay 50 on how can 42 17 forever
3. (d) From both the statements 03 62 67 89
P(?) Step IV : young tips stay on how can 42 17 forever 03
50 62 67 89
(–) M K(+) Step V : young tips stay on how forever can 17 03 42 50
62 67 89
S A(–)
Step VI : young tips stay on how for ever can 03 17 42
The sex of P is not known 50 62 67 89
The data in both statements are not sufficient to 8. (a) It is Step III.
answer the question. 9. (b) 42 is at the fifth position from the right end in Step
4. (e) From both the statements V.
Monday Þ Crash 10. (a) Option (a) is the last step.
Tuesday Þ lce Age/Notting Hill 11. (e) Statement (e) weakens the argument in the passage
Wednesday Þ Ice Age/Notting Hill because heavy compromise on quality and standards
Thursday Þ Social Network of their back office jobs. contradicts the argument.
So data in both statements are sufficient to answer 12. (c) Outsourcing from the West would sender thousands
the question. of Indians unemployed.
5. (d) From both the statements 13. (c) Statement (c) can be easily inferred from the facts
given is passage.
Shraddha
L(Out) R(In)
P(Out)
Vimla/ Vimla/
Ayesha Ayesha

Tania T(In) Q(In)


So data in both statements are not sufficient to answer
the question.
(6-10):
After careful analysis of the given input and various M(Out) K(Out)
S (In)
steps of arrangement it is evident that in each step
one word and one number are rearranged. The
words are rearranged from the left in alphabetical 14. (d) M is second to the left of L.
15. (b) P sits exactly between Q and R.
y
o
u
rs
m
502 IBPS PO Main Exam 2015

a
h
16. (e) Q faces T and both are sitting in the middle of the sides.

b
o
17. (c) P and Q. Rooms

o
b
18. (b) Q would sit to the immediate right of K.

.w
Fans Green Windows
19. (e) Except R, all others are seated at the corners. 31. (c)

o
20. (b) Statement (C) is the effect.

rd
21. (e) Statement (A) is the cause.

p
re
22. (e) Statement (D) is an assumption. Or

s
s
23. (e) Statement (E) contradicts the facts stated in the

.c
Rooms
paragraph.

o
m
24. (c) Shortage of electricity can only lee faced with saving Fans
in electricity. So argument I seems to be strong. Windows Green
Argument II is also strong because every citizen is
paying every single unit of electricity so he/she has
right to consume it. Argument III seems to be weak Conclusions:
because government can earily manage restricted use I. (False)
of electricity. II. (False)
25. (c) Statement (c) would weaken the findings that some III. (False)
of the patients did not respond to the traditional drug IV. (True)
meant for malria. So only conclusion IV follows.
26. (e) Data given is conclusion (e) is in favour of findings 32. (e)
given in the statement.
27. (c) Statement (c) is just opposites of findings given in Chairs Chairs
Rains Rains
statement because the organisations where employees
Dogs Dogs
are allowed to use cell phones their work-efficiency Tablets Or Tablets
is reduced considerably.
28. (d) If sugar is not supplied to largest food beverage and
pharma companies the stock of sugar will improve Conclusions:
and prices will come down. I. (True)
II. (True)
III. (False)
29. (c) Fruits Or Fruits
IV. (True)
Horses Horses Trees So only conclusion I, II and IV follows.
s
ee Boxes xe
s
Tr Bo 33. (e) Men Roads Sky

Conclusions: Conclusions:
I. (False) I. (False)
II. (True) II. (True) Conversion of the second premise
Either
III. (False) III. (False)
IV. (False) IV. (False)

30. (e) Buses Or Buses


Tigers
Tigers (34-38) :
Flowers Cats Cats
Students Institute Specialization
Flowers
Ashwin M Finance
Devika L Marketing
Baljit J HR
Conclusions: Chandrakant K HR
I. (True) Urmila T Finance
II. (False) Nagesh R Marketing
III. (False) Pranjali F HR
IV. (True) 34. (d) 35. (a) 36. (b)
So only conclusion I & IV follows. 37. (e) 38. (c)
y
o
u
rs
m
IBPS PO Main Exam 2015 503

a
h
b
(39-43) : From statements I and III,

o
Person Boarding station Leaving station

o
4 85

b
A IV V x´ ´ = 20400

.w
5 100
B Base or I III

o
rd
C Base or I V 20400 ´ 5 ´ 100
Þx=

p
D Base or I III 4 ´ 85

re
s
E Base IV = ` 30000

s
.c
F Base or I II 53. (a) From Statements I and III, Let Suchitra’s son’s

o
m
G Base or I V present age be x years.
39. (c) 40. (d) 41. (d) \ Suchitra’s present age
42 (c) 43 (b) = 2x years
44. (b) Sixth to the right of 14th from the right end means 8th After 4 years,
from right.
2 x + 4 13
........D $ 6 E N F 8 U H @ =
x + 4 24
We can get the required answer by this relation.
8th from right
So statement II is not required.
45. (d) No Consonant Consonant Symbol 54. (d) From all three statements, Seeta’s investment
Such combinations are : æ3 ö
= ` ç ´ 2.5 ÷ lakh
7W% , 3D$ , UH @ è 8 ø
46. (a) 8th to the left of 16th from left means 8th from the = ` 93750
left. Geeta’s investment
= ` 156250
R 4 T M 7 W % J 9 5 I ........
Ratio of Neeta’s, Seeta’s and Geeta’s profit
= 85000 × 24 : 93750 × 18 : 156250 × 18
8th from left We can get the share of Neeta if total profit is given.
47. (e) Obviously, option (e) justifies the given fact. People
So data given in all three statements is required to
generally prefer small quantity of product at a time so
answer the question.
the product in form of sachet is more popular in
55. (d) Let the marked price of the article be ` x.
consumers. Consumers thinks it is better to consume
the ingredients at once. So sale of product also From statement II.
increases. 95 ´ x
(48 – 50): = 608
100
` 23,000

` 35,000
` 18,000

608 ´100
Þx= = ` 640
95
C D K L A N B From statements I and III,
North 128 ´ 500
48. (c) A and L earn more than ` 23,000 but less than ` Marked price =
35,000. 100
49. (a) D earns more than ` 18, 000 but less than ` 23,000. = ` 640
So, D may earn ` 21,000. So statement II is sufficient to give the answer.
50. (d) L earns more than K but less than A. 56. (b) Required average price
51. (e) From statement I, 750 ´ 25 + 600 ´ 45
Let the number of students in the intitutes A and B =
1350
be 3x and 4x respectively. However we get no
conclusive answer by using data given in all the æ 18750 + 27000 ö
=` ç ÷ per kg
statements. è 1350 ø
52. (c) From statements I and II, Let Mr. X’s monthly » ` 34 per kg
income = ` x. Then 57. (d) Required cost price
15 ´ x æ 90 ö
= 4500
100 = ` ç 800 ´ ´ 80 ÷
è 100 ø
4500 ´100 = ` 57600
Þx= = ` 30000
15
y
o
u
rs
m
504 IBPS PO Main Exam 2015

a
h
58. (a) Total cost of entire quantity of sugar = (350 × 30

b
63. (a) Number of students who failed in Physics and

o
120 Maths = 47 + 18 = 65

o
+ 350 × 30 )

b
100 64. (d) Number of students who passed in Maths but

.w
= ` (10500 + 12600) = ` 23100 failed in both Physics and Chemistry = 1

o
rd
59. (e) Required cost 65. (b) Required answer

p
= ` (500 × 80 + 400 × 60) =x + y+ z+ w

re
9 + 29 + 4 + 171 = 213

s
= ` 64000

s
66. (a) Charu has scored minimum marks both in Sanskrit

.c
60. (c) Total cost of corriander sold
and Social Studies.

o
m
æ 92 ö Vishal has scored minimum marks in Science,
= ` ç 600 ´ ´ 70 ÷
è 100 ø Physical Education and Hindi.
= ` 38640 67. (c) Total marks obtained by :
(61-65): Deepak ® 65 + 62 + 69 + 81 + 70 + 40 + 50 = 437
Total Number of students = 280 Charu ® 64 + 78 + 74 + 63 + 55 + 25 + 53 = 412
NUmber of students who passed Maths = 185 Anupama ® 85 + 95 + 87 + 87 + 65 + 35 + 71 = 525
Number of students who passed Physics = 210 Garima ® 92 + 82 + 81 + 79 + 49 + 30 + 61 = 474
Number of students who passed Chemistry = 222 Bhaskar ® 72 + 97 + 55 + 77 + 62 + 41 + 64 = 468
Students who passed in Maths and physics = 180 68. (e) Required percentage
Students who passed in Maths and Chemistry = 175 437
Students who passed in Physics and Chemistry = = ´ 100 = 72.83
600
200 69. (d) Marks obtained by Charu in Hindi = 64
Number of students who passed in atleast one Marks obtained by Anupama in Hindi = 85
subject = 280 – 47 = 233 Required percentage
64
M x P = ´ 100 = 75.29
85
w 70. (b) Average marks obtained in Science
y z
65 + 62 + 55 + 70 + 49 + 44
=
C 6
345
= = 57.5
x + w = 180 ....(i) 6
w + z = 200 ....(ii) 71. (e) The given series is based on the following pattern :
w + y = 175 ....(iii) 104 109 99 114 94 119
M + x + w + y = 185 ....(iv)
P + x + w + z = 210 ....(v) +5 –10 +15 –20 +25
C + y + w + z = 222 ....(vi) Hence, 119 will come in place of the question mark.
M + P + C + y + x + z + w = 233 ....(vii) 72. (c) The given series is based on the following pattern :
By (iv) + (v) + (vi) – (vii), 980 392 156.8 62.72 25.088 10.0352
x + y + z + 2w = 384 ....(viii)
by (i) + (ii) + (iii), ÷2.5 ÷2.5 ÷2.5 ÷2.5 ÷2.5
Hence, 62.72 will come in place of the question mark.
x + y + z + 3w = 555 ....(ix)
73. (d) The given series is based on the following pattern :
By (ix) – (viii),
w = 171 14 16 35 109 441 2211
From equation (i),
×1+2 ×2+3 ×3+4 ×4+5 ×5+6
x = 9
Hence, 2211 will come in place of the question mark.
From equation (ii), 74. (a) The given series is based on the following pattern :
z = 200 – 171 = 29 Numbers are cubes of consecutive prime numbers.
From equation (iii), i.e.
y = 175 – 171 = 4 113 = 1331
M = 185 – 4 – 9 – 171 = 1 133 = 2197
P = 210 – 9 – 171 – 29 = 1 173 = 4913
C = 222 – 4 – 29 – 171 = 18 193 = 6859
61. (a) Number of students passed in Chemistry only = 18 233 = 12167
62. (d) Number of students who have passed in all three
293 = 24389
subjects = 171
Hence. 12167 will come in place of the question mark.
y
o
u
rs
m
IBPS PO Main Exam 2015 505

a
h
82. (d) I. Þ p = ± 2

b
75. (b) The given series is based on the following pattern :

o
II. Þ q2 + 2q + 2q + 4 = 0

o
3600 725 150 35 12 7.4

b
Þ q (q + 2) + 2 (q + 2) = 0

.w
÷5+5 ÷5+5 ÷5+5 ÷5+5 ÷5+5 Þ (q + 2) + (q + 2) = 0

o
Hence. 7.4 will come in place of the question mark. Þ q=–2

rd
p
a Obviously p > q

re
76. (d) Let the original fraction is . 83. (b) I. p2 + p – 56 = 0
b

s
s
Þ p2 + 8p – 7p – 56 = 0

.c
According to question,
Þ p (p + 8) –7 (p + 8) = 0

o
350

m
a+ ´a Þ (p + 8) (p – 7) = 0
100 9
= Þ p = 7 or – 8
300 22
b+ ´b II. q2 – 8q – 9q + 72 = 0
100 Þ q (q – 8) – 9 (q – 8) = 0
4.5a 9 Þ (q – 8) (q – 9) = 0
Þ =
4b 22 Þ q = 8 or 9
a 9 4 4 Obviously p < q
Þ = ´ = 84. (a) We have,
b 22 4.5 11
77. (e) Let breadth of rectangular plot is b cm length of 3p + 2q = 58 … (i)
rectangular plot, l = 3b 4p + 4q = 92
Þ 2p + 2q = 46 … (ii)
l × b = 6075
By (i), (ii) we get p = 12
Þ 3b2 = 6075
From (i), 3 × 12 + 2q = 58
Þ b2 = 2025 Þ 2q = 58 – 36 = 22
Þ b = 45 Þ q = 11
Length of plot = 3×45 cm = 135 cm Hence, p > q
78. (d) Let her monthly salary be ` x. 85. (b) I. Þ 3p2 + 15p + 2p + 10 = 0
According to the question, Þ 3p (p + 5) + 2(p + 5) = 0
7 Þ (p + 5) (3p + 2) = 0
´ x = 2170
100 2
p = – 5 or –
2170 ´ 100 3
Þ x= = `31000
7 II. Þ 10q2 + 5q + 4q + 2 = 0
Total monthly investment = (18 + 6 + 7)% of 31000 Þ 5q (2q + 1) + 2 (2q + 1) = 0
31 Þ (2q + 1) (5q + 2) = 0
= × 31000 = 9610
100 1 2
Total annual investment = 12 × 9610 = ` 115320 Þ q= – or –
79. (b) ‘PRIDE’ has five different letters. 2 5
So, it can be arranged in 5 ! = 120 ways Hence, p < q
80. (d) Let present ages of Samir and Tanuj are 8x and 15x 86. (a) Required ratio = 40 : 35 = 8 : 7
years respectively. 87. (b) Required average
Difference between their ages = 15x – 8x = 7x æ 15 + 25 + 30 + 40 + 15 + 10 ö
Ratio of ages after 9 years, = ç ÷ thousand
è 6 ø
8 x + 9 11
= 135
15 x + 9 18 = = 22.5 thousand
6
Þ 144x + 162 = 165x + 99
88. (c) Required percentage increase
Þ 21x = 63 Þ x = 3
25 - 10
Difference between their ages = 7x = 21 years = ´ 100 = 150
81. (e) I. Þ p2 + 3p + 2p + 6 = 0 10
89. (e) Laptops manufactured by Apple, Lenovo and Samsung
Þ p (p + 3) + 2(p + 3) = 0
in 2013 = 15 + 40 + 25 = 80 thousand
Þ (p + 3)(p + 2) = 0 Laptops manufactured by Dell, HP and Abascus in
Þ p = 2 or – 3 2014 Required = 15 + 25 + 35 = 75 thousand
II. Þ q2 + q + 2q + 2 = 0 Difference = 5000
Þ q (q + 1) + 2 (q + 1) = 0 90. (a) Abascus Þ 35000
Þ (q + 1) + (q + 2) = 0
æ 884 ´16 ö 13 ?
Þ q = – 1 or – 2 91. (a) 7072 ÷ ç ÷ = 30 ´ ´
è 100 ø 12 39
Obviously p £ q
y
o
u
rs
m
506 IBPS PO Main Exam 2015

a
h
b
5´? Hence, A’s salary cannot be determined.

o
Þ 7072 ÷ 141.44 =

o
6 99. (a) Percentage increase in the consumption

b
.w
Þ 50 × 6 = 5 × ? 10 100 1
= ´ 100 = = 11 %

o
50 ´ 6 100 - 10 9 9

rd
Þ ?= = 60

p
5 3 4 21 + 20 41 4 3 28 + 15 43

re
100. (b) + = = and + = =
92. (a) (3375)6 ÷ (15)10 ÷ (225)7 = (15)? – 6 5 7 35 35 5 7 35 35

s
s
Þ ((15)3)6 ÷ (15)10 ÷ (152)7 = (15)? – 6

.c
o
101. (b) 102. (c) 103. (a) 104. (d) 105. (b)
Þ 1518–10–14 = (15)?–6

m
106. (d) 107. (e) 108. (d) 109. (b) 110. (a)
Þ –6=?–6 111. (b) 112. (d) 113. (e) 114. (c) 115. (a)
Þ ?=0 116. (a) 117. (b) 118. (c) 119. (d) 120. (e)
1478.4
121. (d) 122. (e) 123. (c) 124. (a) 125. (b)
93. (b) + 66.8 × 57 = ? × 3 + 34 × 34.5 126. (e)
56
127. (d) ‘Mushrooming’ should be used-which would serve
Þ 26.4 + 3807.6 = ? × 3 + 1173 as an adjective.
Þ 3834 - 1173 = ? × 3 128. (c) ‘Are’ should replace ‘is’- verb should agree with
Þ ? × 3 = 2661 ‘bulbs’
Þ ? = 887 129. (b) ‘By’ should replace ‘with’ - which means ‘by the
way of’
(13 + 2 5 )
2
94. (c) =?× 5 + 189 130. (e)
Þ 169 + 20 + 2 × 13 × 2 5 = ? × 131. (b) ‘Make up’ would mean create by altering or
5 + 189
modifying such as ‘making up a story.’ ‘Puts up’ is
Þ 189 + 52 × 5 = ? × 5 + 189 also an incorrect expression here as it means ‘stays’
Þ ? = 52 ‘Toil up’ is again incorrect option as it means to put
95. (d) 0.22 + 0.0054 = ? – 313.06 hard work into. Option (b) ‘conjures’ is the right
Þ 0.2254 + 313.06 = ? answer which means ‘to recall.’
Þ ? = 313.2854 132. (a) ‘Sturdy’ means ‘with strong built up’, ‘wobbly’
96. (b) If total maximum marks be x, then, means ‘unsteady’, ‘herculean’ means ‘having great
x ´ 64 strength.’, ‘handsome’ means ‘good looking.’ Option
= 2240 – 128 = 2112 (a) is the most appropriate here as a horseman is
100
generally visualized as the one having strong built
2112 ´ 100
Þ ?= = 3300 up.
64
133. (c) Since the sentence talks about communist rulers
Marks obtained by Meena = 2240 – 907 = 1333 ‘taking over’, option (c) ‘imposing’ which means ‘to
1333 thrust.’
Required percentage = × 100 » 40 134. (d) ‘Amplify’ and ‘stretch’ both mean an increase. ‘Die’
3300
97. (c) Total possible outcomes = 17C5 would be a little extreme word here. ‘Recede’ which
means ‘to move back’ matches up with the word
17 ´ 16 ´ 15 ´ 14 ´ 13
= = 6188 background used in the sentence.
1´ 2 ´ 3 ´ 4 ´ 5
135. (d) The correct tense form here would be option (d) i.e.;
Total favourable outcomes = 8C2 × 4C1 × 5C2
‘were.’
8´ 7 5´ 4
= ´ 4´ = 28 × 4 × 10 = 1120 136. (a) 137. (c) 138. (b) 139. (e) 140. (e)
1´ 2 1´ 2 141. (b) 142. (a) 143. (b) 144. (d) 145. (b)
1120 280 146. (c) 147. (c) 148. (d) 149. (b) 150. (b)
Required probability = =
6188 1547 151. (d) 152. (b) 153. (d) 154. (a) 155. (d)
98. (d) A’s salary = ` 8x and B’s salary = ` 9x 156. (d) 157. (a) 158. (a) 159. (a) 160. (a)
8 x ´150% 16 161. (b) 162. (b) 163. (a) 164. (a) 165. (d)
=
9 x ´ 75% 9 166. (b) 167. (b) 168. (a) 169. (b) 170. (b)
171. (c) 172. (c) 173. (a) 174. (d) 175. (a)
8 x ´ 150 16
Þ = 176. (a) 177. (a) 178. (d) 179. (d) 180. (c)
9 x ´ 75 9
181. (d) 182. (c) 183. (b) 184. (a) 185. (b)
12 x 16 48 16 186. (b) 187. (c) 188. (c) 189. (c) 190. (d)
Þ = Þ =
27 x 9 27 9 191. (a) 192. (d) 193. (c) 194. (e) 195. (c)
4 196. (a) 197. (d) 198. (b) 199. (b) 200. (c)
y
o
u
rs
m
a
h
b
o
o
b
.w
o
SBI BANK PO PRELIM EXAM 2015

rd
p
re
s
s
Based on Memory

.c
o
m
REASONING ABILITY 9. A man is facing west. He turns 45 degree in the clockwise
direction and then another 180 degree in the same direction
1. How many such pairs of letters are there in the word and then 270 degree in the anticlockwise direction. Find
SHOULDER each of which has as many letters between them which direction he is facing now?
in the word as in the English alphabet? (a) South-West (b) West
(a) None (b) One (c) South (d) East-South
(c) Two (d) Three (e) None of these
(e) More than three
DIRECTIONS (Qs. 10 - 15) : Each of the questions are given
2. If it is possible to make only one meaningful English word
four statements followed by four conclusions numbered I, II, III
with the first, the fifth, the seventh and the tenth letters of
the word STREAMLINE, using each letter once in each IV. You have to take the given statements to be true even it they
world, which of the following is the third letter of that word? seem to be at variance from commonly known facts Read all the
If no such word can be made, give ‘X’ as the answer and if conclusions and then decide which of the given conclusions
more than one such word can be made, give ‘Y’ as the answer. logically follows from the given statements disregarding
(a) L (b) E commonly known facts.
(c) S (d) X 10. Statements : All drums are tubes.
(e) Y Some tubes are pipes.
3. Four of the following five are alike in a certain way and so form No pipe is stick.
a group. Which is the one that does not belong to that group? Some sticks are rubbers.
(a) Nitrogen (b) Hydrogen Conclusions I : Some rubbers are tubes.
(c) Methane (d) Neon Conclusions II : Some sticks are drums.
(e) Helium Conclusions III : Some pipes are drums.
4. In a certain code BREAKING is written BFSCFMHJ, How is Conclusions IV : Some sticks are tubes.
MOTHERLY written in that code?
(a) None follows (b) Only I follows
(a) IUPNZMSF (b) IUPNXKQD
(c) Only I and II follow (d) Only III follows
(c) IUPNFSMZ (d) GSNLZMSF
(e) None of these
(e) None of these
11. Statements : Some pens are rooms.
5. Among P, Q, R, S and T Each having a different height, Q is
All rooms are walls
taller than S. T is shorter than P. R is taller than Q but shorter
than T. Who among them in the tallest? Some walls are bricks
(a) S (b) P All bricks are slates.
(c) R (d) Data inadequate Conclusions I : Some slates are walls.
(e) None of these Conclusions II : Some walls are pens.
6. In a row of 40 children, R is 11th from the right and there are Conclusions III : Some bricks are rooms.
15 children between R and M. What is M’s position from the Conclusions IV : Some slates are rooms.
left and of the row? (a) Only I and III follow (b) Only II and III follows
(a) 14th (b) 15th (c) Only I and II follow (d) Only III and IV follow
(c) 13th (d) Can’t be determined (e) None of these
(e) None of these 12. Statements : Some chairs are pencils.
7. In a certain code language ‘how many are there’ is written as Some pencils are bottles.
‘ka na ta da’ and ‘many are welcome here’ is written as ‘na pa Some bottles are bags.
ni ka’. How is ‘how’ written in that code language? Some bags are books.
(a) ta (b) da Conclusions I : Some books are pencils.
(c) ta or da (d) Data inadequate Conclusions II : Some bottles are chairs.
(e) None of these Conclusions III : No book is pencil.
8. If ‘R’ denotes ‘¸’, ‘T’ denotes ‘–’, ‘M’ denotes ‘+’ and ‘W’ Conclusions IV : Some bags are chairs.
denotes ‘×’, then (a) Only I follows
27 T 15 R 3 W 4 M 6 = ? (b) Only either I and III follows
(a) 7 (b) 13 (c) Only III follows
(c) ‘ –’ 23 (d) 1 (d) Only IV follows
(e) None of these (e) None of these
y
o
u
rs
m
a
h
508 SBI Bank PO Prelim Exam 2015

b
o
o
13. Statements : Some roads are buses. Volleyball, Badminton, Basketball and Carom, not necessarily in

b
All buses are trains. the same order, Each of them also has a specific choice of color

.w
Some trains are trucks. from- Blue, Red, Green, Yellow, Grey, Black and White, not

o
All trucks are kites..

rd
necessarily in the same order.
Conclusions I : Some trucks are roads.

p
Conclusions II : Some kites are buses. R likes Green and his favorite sport is Badminton. V’s choice

re
Conclusions III : Some trains are roads. of color is neither Red nor Black. T’S favorite sport is neither

s
s
Conclusions IV : Some kites are trains. Table tennis nor Basketball. The one who likes Blue does not like

.c
(a) None follows (b) Only I follows Carom. The one who likes Volleyball does not like Yellow and

o
(c) Only II follows (d) Only III follows

m
Grey. Q’s favorite sport is Lawn Tennis and he likes Black. S likes
(e) None of these
14. Statements : All beads are rings. White. W likes Basketball. P likes Volleyball. T likes Blue. The
All rings are bangles. one who likes Basketball does not like Grey.
All bangles are tyres. 21. What is V’s choice color?
All tyres are pendants. (a) Black (b) Grey
Conclusions I : Some pendants are beads.
Conclusions II : Some tyres are rings. (c) Yellow (d) Data inadequate
Conclusions III : Some bangles are beads. (e) None of these
Conclusions IV : Some pendants are rings. 22. What is T’s favorite sport?
(a) Only I and II follow (b) Only I, II and III follows (a) Basketball (b) Volleyball
(c) Only II, III and IV follow (d) Only I, III and IV follow (c) Chess (d) Data inadequate
(e) All follow
15. Statements : Some desks are fruits. (e) None of these
All fruits are flowers. 23. Whose favourite sport is Carrom?
No flower is branch. (a) S (b) R
Some branches are roots. (c) W (d) Data inadequate
Conclusions I : Some roots are flowers. (e) None of these
Conclusions II : No branches are desks.
Conclusions III : Some flowers are desks. 24. Whose favourite sport is basketball?
Conclusions IV : Some desks are branches. (a) S (b) T
(a) Only either II and IV follows (c) W (d) R
(b) Only III follows (e) None of these
(c) Only either II or IV and III follow.
(d) Only I and II follow 25. What is W’s choice of colour?
(e) None of these (a) Green (b) White
(c) Black (d) Data inadequate
DIRECTIONS (Qs. 16 to 20): Read the following information (e) None of these
carefully and answer the questions given below:
A, B, C, D, E, F, G and H are sitting around a circle facing the DIRECTIONS (Qs. 26 to 30): Read the following information
centre. B is 2nd to the right of D who is 3rd to the right of F. C is 2nd and answer the questions based on it.
to the left of A who is 2nd to the left of F. G is 3rd to right of E.
P @ Q means P is either greater than or equal to Q
16. In which of the following combination is the 1st person sitting
between the 2nd and the 3rd persons? P + Q means P is either smaller than or equal to Q
(a) GCD (b) FGH P % Q means P is greater than Q
(c) EFH (d) ABE P × Q means P is smaller than Q
(e) None of these P $ Q means P is neither greater than nor smaller than Q.
17. Who is 3rd to the right of H? Now in each of the following questions assuming the given
(a) G (b) D
(c) C (d) Data inadequate statement to be true, find which of the two conditions I and II
(e) None of these given below them is/are definately true? Give answer.
18. Who is to the immediate right of A? (a) If only conclusion I is true
(a) B (b) E (b) If only conclusion II is true
(c) F (d) Data inadequate (c) If either I or II is true
(e) None of these
19. What is H’s position with respect to B? (d) If neither I or II is true
(a) 5th to the right (b) 3rd to the left (e) If both I and II is true
(c) 5 to the left
th
(d) 3rd to the right 26. Statements : M @ R, R % T, T $ K
(e) 4th to the left Conclusion : (I) K × M, (II) T × M
20. Who is to the immediate left of G? 27. Statements : H % J, B + J, B @ F
(a) H (b) F
Conclusion : (I) F $ J, (II) J % F
(c) D (d) Data inadequate
(e) None of these 28. Statements : D $ M, M % W, W @ R
Conclusion : (I) R×D, (II) W + D
DIRECTIONS (Qs. 21 to 25): Study the following information 29. Statements : A + N, N×V, V$J
carefully and answer the questions given below: Conclusion : (I) J @ N, (II) A + V
P, Q, R, S, T, V and W are seven members of a club. Each of 30. Statements : K×T, T @ B, B + M
them has a favorite sport from-Chess, Table Tennis, Lawn Tennis, Conclusion : (I) M % T, (II) K + B
y
o
u
rs
m
a
h
SBI Bank PO Prelim Exam 2015 509

b
o
o
DIRECTIONS (Qs. 31 to 35): Read the following information 40. Ms. Pooja Pushpan invests 13% of her monthly salary, i.e.

b
`, 8554 in Mediclaim Policies, Later she invests 23% of her

.w
carefully and answer the questions given below it:
monthly salary on Child. Education Policies; also she

o
In a family, there are six members A, B, C, D, E and F. A and B are

rd
invests another 8% of her monthly salary on Mutual
married couple, A being the male member. D is the only son of C,

p
Funds. What is the total annual amount invested by Ms.

re
who is the brother of A. E is the sister of D. B is the daughter-
Pooja Pushpan?

s
inlaw of F, whose husband has died.

s
.c
(a) ` 28952 (b) ` 43428
31. How is F related to A?

o
(c) ` 347424 (d) ` 173712

m
(a) Mother (b) Sister-in-law
(c) Sister (d) Mother-in-low (e) None of these
(e) None of these 41. The profit earned after selling an article for `878 is the
32. How is E related to C? same as loss incurred after selling the article for `636.
(a) Sister (b) Daughter What is the cost price of the article?
(c) Cousin (d) Aunt (a) ` 797 (b) ` 787
(e) Mother (c) ` 767 (d) ` 757
33. Who is C to B? (e) None of these
(a) Brother (b) Brother-in-law
42. In a class of 240 students, each student got sweets got
(c) Nephew (d) Son-in-law
(e) None of these sweets that are 15% of the total number of students.
34. How many male members are there in the family? How many sweets were there?
(a) One (b) Two (a) 3000 (b) 3125
(c) Three (d) Four (b) 8640 (d) Cannot be determined
(e) Five (e) None of these
35. How is F related to C? 43. Sonika spent `45, 760 on the interior decoration for her
(a) Mother-in-law (b) Sister-in-law home, `7896 on buying air conditioner and the remaining
(c) Mother (d) Aunt
28% of the total amount she had as cash with her. What
(e) Sister
was the total amount?
(a) `98540 (b) `102300
QUANTITATIVE APTITUDE (b) `134560 (d) Cannot be determined
36. The simple interest accrued on an amount of `20,000 at (e) None of these
the end of three years is `7,200. What would be the 44. The ages of Khushi and Jagriti are in the ratio of 5 : 8
compound interest accrued on the same amount at the respectively. After 8 years the ratio of their ages will be
same rate in the same period? 3 : 4 what is the difference in their ages?
(a) ` 8098.56 (b) ` 8246.16 (a) 16 years (b) 8 years
(c) ` 8112.86 (d) ` 8342.36 (c) 10 years (d) 12 years
(e) None of these (e) None of these
37. If the numerator of a fraction is increased by 300% and the
45. In how many different ways can the letters of the word
denominator is increased by 500%, the resultant fraction
‘PUNCTUAL’ be arranged?
5
is . What was the original fraction? (a) 64 (b) 40320
12
(c) 960 (d) 20160
8 5 (e) None of these
(a) (b)
5 11
12 5 DIRECTIONS (Qs. 46-50) : What will come in place of the
(c) (d) question mark (?) in the following number series.
5 7
(e) None of these 46. 93 95 99 ? 110 121 134
38. A sum of money is divided among A, B, C and D in the
(a) 104 (b) 96
ratio of 3 : 5 : 9 : 13 respectively. If the share of C is `.
2412 more than the share of A, then what is the total (c) 82 (d) 103
amount of money of B and D together? (e) None of these
(a) ` 4422 (b) ` 7236 47. 8 12 18 27 40.5 60.75?
(c) ` 6030 (d) ` 4824 (a) 104.125 (b) 121.125
(e) None of these (c) 96.125 (d) 83.125
39. The average age of 80 girls was 20 years, the average age (e) None of these
of 20 of them was 22 years and that of another 20 was 24
years. Find the average age of the remaining girls. 48. 4 7 11 18 29 ? 76 123
(a) 17 years (b) 19 years (a) 59 (b) 38
(c) 21 years (d) 15 years (c) 46 (d) 53
(e) None of these (e) None of these
y
o
u
rs
m
a
h
510 SBI Bank PO Prelim Exam 2015

b
o
o
49. 3 10 ? 172 885 5346 37471 299832 Production of two companies A & B over the years (Production

b
in Lakh units)

.w
(a) 39 (b) 27
(c) 24 (d) 34

o
14

rd
(e) None of these A

p
50. 15 22 56 183 ? 3755 22542 13 B

re
(a) 709 (b) 698

s
12

s
(c) 748 (d) 800

.c
(e) None of these 11

o
Production in Lakh units

m
10
DIRECTIONS (Qs. 51-55) : What approximate value will come
in place of the question mark (?) in the following questions? 9
8
51. 2959.85 ÷ 16.001 – 34.99 = ?
(a) 160 (b) 150 7
(c) 140 (d) 180 6
(e) 170
5
52. (1702 ÷ 68) × 136.05 = ?
(a) 3500 (b) 3550 4
(c) 3450 (d) 3400 3
(e) 3525
2
53. 2950 ÷ 12.25 + 160 = ?
(a) 440 (b) 350 1
(c) 380 (d) 360 0
(e) 400
54. 25.05% of 2845 + 14.95 × 2400 = ?
2009

2012
2008

2010

2013
2011

2014

2015
(a) 36,700 (b) 36,500
(c) 35,800 (d) 35,600
Years
(e) 36,200
61. For Company A, what is the percent decrease in production
55. (186 × 270.99) ÷ 40 = ?
from 2008 to 2009?
(a) 1160 (b) 1200 (a) 75 (b) 50
(c) 1300 (d) 1180 (c) 35 (d) 10
(e) 1260 (e) None of these
DIRECTIONS (Qs. 56-60) : In each of the following questions 62. In 2004, the production of Company B is approximately
two equations are given. You have to solve them and what per cent of that of in 2013?
(a) 60 (b) 157
Give answer (a) if p < q; (c) 192 (d) 50
Give answer (b) if p > q; (e) 92
Give answer (c) if p £ q; 63. For Company A, in which year is the percentage increase/
Give answer (d) if p ³ q; decrease in the production from the previous year the
Give answer (e) if p = q; highest?
(a) 2014 (b) 2005
56. I. p2 – 7p = – 12
(c) 2012 (d) 2008
II. q2 – 3q + 2 = 0 (e) None of these
64. What is the difference in the total production of the two
57. I. 12p2 – 7q = –1
Companies for the given years?
II. 6q2 – 7q + 2 = 0 (a) 27,00,000 (b) 31,00,000
(c) 2,70,000 (d) 3,10,000
58. I. p2 + 12p + 35 = 0 (e) None of these
II. 2q2 + 22q + 56 = 0 65. Which of the following is the closest average production
in lakh units of Company B for the given years?
59. I. p2 – 8p + 15 = 0 (a) 4.1 (b) 3.5
II. q2 – 5q = – 6 (c) 4.3 (d) 3.75
(e) 3.9
60. I. 2p2 + 20p + 50 = 0
DIRECTIONS (Qs. 66-70) : Study the following information to
II. q2 = 25 answer the given questions.
DIRECTIONS (Qs. 61 - 65) : Study the following graph to Percentage of students in various courses (A, B, C, D, E, F) and
answer the given questions. Percentage of girls out of these.
y
o
u
rs
m
a
h
SBI Bank PO Prelim Exam 2015 511

b
o
o
Total students : 1200 (D) Some have high birth rate and still have a high death rate.

b
(E) Compared to this is Europe the growth rate is low.

.w
(800 girls + 400 boys)
(F) The developing countries show the most rapid growth rate.

o
Percentage in various courses 71. Which will the LAST sentence in the Para?

rd
(a) A (b) B

p
re
(c) C (d) D
F

s
(e) E

s
13% A

.c
20%
72. Which will be the FIRST sentence in the Para?

o
E (a) E (b) D

m
12% (c) C (d) B
B (e) A
D 73. Which will come at SECOND place in the para?
35% C 15% (a) A (b) B
5% (c) C (d) D
(e) E
Total Girls : 800 74. Which sentence will come at FIFTH place in the Para?
Percentage of Girls in courses (a) F (b) D
(c) B (d) A
(e) C
F 75. Which will be the THIRD sentence in the Para?
14% A (a) B (b) A
30% (c) D (d) F
E (e) E
14%
B DIRECTIONS (Qs. 76 to 80): Fill in the blanks with appropriate
D
C
10% word:-
30%
2% Let children learn to judge their own work. A child ___(76)___ to
talk does not learn ___(77)___ being corrected all the time
66. For course D what is the respec-tive ratio of boys and girls? ___(78)___corrected too much, he will ___(79)___ talking. He
(a) 3 : 4 (b) 4 : 5 notices a thousand times a day the difference between the
___(80)___ he uses and the language those around him use.
(c) 3 : 5 (d) 5 : 6 76. (a) endeavouring (b) learning
(c) experimenting (d) experiencing
(e) None of these (e) preparing
67. For which pair of courses is the number of boys the same? 77. (a) In (b) on
(a) E & F (b) A & D (c) By (d) to
(c) C & F (d) B & D (e) From
(e) None of these 78. (a) unless (b) being
68. For course, E, the number of girls is how much per cent (c) until (d) if
more than the boys for course E? (e) s o
79. (a) stop (b) halt
(a) 250 (b) 350 (c) avoid (d) shun
(c) 150 (d) 80 (e) giveup
(e) None of these 80. (a) speech (b) language
69. For which course is the number of boys the minimum? (c) talk (d) skill
(a) E (b) F (e) virtue
(c) C (d) A
(e) None of these DIRECTIONS (Qs. 81 to 85): Five alternative a, b, c, d and e are
70. How many girls are in course C ? given under each sentence, you are required to select the most
(a) 44 (b) 16 suitable alternative to fill in the blank/blanks in the sentence to
(c) 40 (d) 160 make it meaningful.
(e) None of these 81. Intelligence is an _____ part of one’s success.
(a) inseparable (b) inimitable
ENGLISH LANGUAGE (c) indivisible
(e) None of these
(d) indispensable
82. Anjana impressed the interviewer with her concise, ____
DIRECTIONS (Qs. 71 to 75): Given below are six sentences i.e. answers.
A, B, C, D, E and F, which have been presented in a wrong order. (a) allusive (b) revealing
Arrange them in order to form a meaningful paragraph and (c) pertinent (d) referential
then answer the questions given below. (e) None of these
83. The coach asked the players to _____ with his ideology or
(A) Some others, like the European nations, have a low leave the team.
birth rate and a low death rate. (a) counter (b) align
(B) Many others have a high birth rate with a low death rate. (c) favour (d) separate
(C) Different countries show different patterns of growth. (e) None of these
y
o
u
rs
m
a
h
512 SBI Bank PO Prelim Exam 2015

b
o
o
84. The seminar helped _____ the students on the harmful effects 92. The phrase lavish with his hospitatlity signifies

b
of smoking and alcohol. (a) miserliness in dealing with his friends.

.w
(a) educate (b) learn (b) considerate ness in spending on guests and strangers.

o
(c) teach (d) insist (c) extravagance in entertaining guests.

rd
(e) None of these (d) indifference in treating his friends and relatives.

p
re
85. Rajeev was _____ legal aid to fight his extradition. (e) none of these

s
(a) offered (b) granted 93. We understand from the passage that

s
.c
(c) allowed (d) awarded (a) all mean people are wealthy

o
(e) None of these (b) wealthy people are invariably successful.

m
(c) carefulness generally leads to failure.
DIRECTIONS (Qs. 86 to 90): Read each sentence to find out (d) thrift may lead to success.
whether there is any grammatical error in it. The error, if any (e) none of these
will be in one part of the sentence. The letter of that part is the 94. It seems that low paid people should
answer. If there is no error, the answer is ‘D’. (Ignore the errors (a) not pay their bills promptly.
of punctuation, if any) (b) not keep their creditors waiting.
86. “The Patient is comparatively better (a)/ today and I hope (b)/ (c) borrow money to meet their essential needs
that he will recover soon” (c)/ said the doctor (d)/ No error (e) (d) feel guilty if they overspend
87. All the members (a)/ of the committee are (b)/ kindly (e) none of these
requested to appear (c)/ in the next morning (d)/ No error (e). 95. The word paradox means:
88. This is the new (a)/ book “One Night @ the call centre” (a) statement based on facts.
authored by Chetan Bhagat which my father (b)/ bought it (b) that which brings out the inner meaning.
(c)/ for you (d)/ No error (e) (c) that which is contrary to received opinion.
89. The passing marks (a)/ in Economics is thirty three (b)/ but (d) statement based on the popular opinion.
(e) none of these
he has secured (c)/ just twenty-nine (d)/ No error
96. How does the housewife, described by the writer, feel when
90. As my neighbourers (a)/ are very co-operative (b)/ I do not
she saves money?
(c)/ feel any difficultly in living in this locality (d)/ No error (e)
(a) is content to be so thrifty.
DIRECTIONS (Qs. 91 to 100): Read the following passage (b) wishes life were less burdensome.
carefully and answer the questions given below it. (c) is still troubled by a sense of guilt.
(d) wishes she could sometimes be extravagant.
It is a strange that, according to his position in life, an
(e) none of these
extravagant man is admired or despised. A successful
97. Which of the following is opposite in meaning to the word
businessman does nothing to increase his popularity by applauded in the passage?
being careful with his money. He is expected to display his (a) Humiliated (b) Decried
success, to have smart car, an expensive life, and to be lavish (c) Cherished (d) Suppressed
with his hospitality. If he is not so, he is considered mean (e) None of these
and his reputation in business may even suffer in 98. The statement she is able to face the milkman with equanimity
consequence. The paradox remains that if he had not been implies that:
careful with his money in the first place, he would never (a) she is not upset as she has been paying the milkman
have achieved his present wealth. Among the two income his dues regularly.
groups, a different set of values exists. (b) she loses her nerve at the sight of the milkman who
The yaung clerk who makes his wife a present of a new always demands his dues.
dress when the hadn’t paid his houserent condemned as (c) she manages to keep cool as she has to pay the milkman
extravagant. Carefulness with money to the point of who always demands his dues.
meanness is applauded as a virtue. Nothing in his life is (d) she remains composed and confident as she knows
considered more worthy than paying his bills. The ideal wife that she can handle the milkman tactfully.
for such a man separaters her housekeeping money into (e) none of these
joyless little piles- so much for rent, for food, for the children’s 99. As far as money is concerned, we get the impression that
shoes; she is able to face the milkman with equanimity and the writer:
never knows the guilt of buying something she can’t able to (a) is incapable of saving anything
face the milkman with equanimity and never knows the guilt (b) is never inclined to be extravagant
of buying something she can’t really afford. As for myself, I (c) would like to be considered extravagant
fall into neither of these categories. If I have money to spare (d) doesn’t often have any money to save
I can be extravagant, but when, as is usually the case, I am (e) none of these
hard up, then I am then meanest man imaginable. 100. Which of the following would be the most suitable title for
91. In the opinion of the writer, a successful businessman: the passage?
(a) is more popular if he appears to be doing nothing. (a) Extravagance leads to poverty
(b) should not bother about his popularity. (b) Miserly habits of the poor.
(c) must be extravagant before achieving success. (c) Extravagance in the life of the rice and the poor.
(d) is expected to have expensive tastes. (d) Extravagance is always condemnable.
(e) None of these (e) None of these
y
o
u
rs
m
a
h
SBI Bank PO Prelim Exam 2015 513

b
o
o
b
.w
o
rd
p
re
s
1. (b) moved 180 degree clockwise, now he will be facing OC. From

s
.c
2. (e) Meaningful words: here he moved 270 degree anticlockwise, Finally he is facing

o
m
SALE, SEAL OD, which is South west.
1 2 3 4 5 6 7 8 9 10
S T R E A M L I N E
3. (c) Among these, only Methane is compound gas. 10. (a) Tubes

Drums Pipes Sticks Rubbers


4. (b) B R E A K I N G

+1 or

B F S C F M H J
Tubes

MO T H E R L Y Drums Pipes

+1 –1
Sticks Rubbers
I U P N X K Q D

5. (b) P>T>R> Q>S


\ P is the tallest Conclusion I : (False)
6. (a) Conclusion II : (False)
7. (c) how many are there ® ka na ta da (i) Conclusion III : (False)
many are welcome here ® na pi ni ka (ii) Conclusion IV : (False)
From equations (i) and (ii) many are ® na ka \ None follows
how ® ta or da
8. (b) Given arrangement = 27 T 15 R 3 W 4 M 6 11. (c) Walls Slates
According to question, letters converted into
mathematical symbols Rooms Bricks
= 27 – 15 ¸ 3 × 4 + 6 = 27 – 5 × 4 + 6
= 27 – 20 + 6 = 33 – 20 = 13 Pens

9. (a) B Conclusion I : (True)


Conclusion II : (True)
270° Conclusion III : (False)
Starting Conclusion IV : (False)
point 180°
45°
A
O Chairs Pencils Bottles Bags Books
12. (b)
End point
C
Conclusion I : (False)
D
Conclusion II : (False) Either I and III follows.
The man firstly faces the direction OA. On moving 45 degree Conclusion III : (False)
clockwise [Please check carefully always if clockwise or Conclusion IV : (False)
anticlockwise], he faces the direction OB. Now again he
y
o
u
rs
m
a
h
514 SBI Bank PO Prelim Exam 2015

b
o
o
13. (e) 16-20 :

b
Trains Kites

.w
o
Buses Trucks B

rd
p
A C

re
Roads

s
s
.c
E D

o
m
Conclusion I : (False)
Conclusion II : (False) G
F
Conclusion III : (True) H
Conclusion IV : (True)
16. (d) 17. (c) 18. (b) 19. (e) 20. (a)
14. (e)
Pendants 21-25 :
Tyres
Bangles Members Sports Colors
Rings P Volleyball Red
Beads Q Lawn-tennis Black
R Badminton Green
S Carrom/Table tennis White
T Chess Blue
V Carrom/Table tennis Grey

Conclusion I : (True) W Basketball Yellow

Conclusion II : (True) 21. (b) 22. (c) 23. (d) 24. (c) 25. (e)
Conclusion III : (True) 26. (e) M³R> T=K
Conclusion IV : (True) Conclusion I : K < M (True)
Hence, All four follows. Conclusion II : T < M (True)
Hence, both conclusions are true.
27. (c) M> J³B ³F
15. (c)
Flowers Conclusion I : F = J
Fruits Either
Branches Roots Conclusion II: J > J
Desks Hence, either I or II is true.
28. (a) D= M>W³R
or
Conclusion I : R < D (True)
Flowers Conclusion II : W £ D (False)
Fruits Hence, only conclusion I is true.
Desks
29. (d) A £ N < V = J

Roots Branches Conclusion I : J ³ N (False)


Conclusion II: A £ V (False)
Hence, both conclusions are false.
30. (d) K < T ³ B £ M
Conclusion I : (False)
Conclusion I : M > T (False)
Conclusion II : (False)
Conclusion II : K £ B (False)
Conclusion III : (True)
Hence, both conclusions are false.
Conclusion IV : (True)
y
o
u
rs
m
a
h
SBI Bank PO Prelim Exam 2015 515

b
o
o
31-35 : Þ x = 2412 × 5 = `12060

b
\ Amount received by B and D together

.w
F (–)

o
æ (5 + 13) ö
daughter-in-law

rd
=`ç
ç ´ 12060 ÷
÷
è 30

p
ø

re
C A B = ` 7236

s
s
(+) (+) (–) 39. (a) Total age of remaining 40 girls

.c
= (80 × 20 – 20 × 22 – 20 × 24) years

o
m
= (1600 – 440 – 480) years
D E = 680 years
(+) (–) \ Required average age
680
31. (a) 32. (b) 33. (b) 34. (c) 35. (c) = = 17 years
40
S.I.´100 40. (c) Let Ms. Pooja Pushpan’s monthly salary = ` . x
36. (a) Rate = Pr incipal ´ Time
According to the question,
7200 ´100 13% of the x = ` 8554
= = 12% per annum
20000 ´ 3 æ 8554 ´100 ö
Þ x = `ç
ç ÷
÷
éæ T ù è 13 ø
R ö
\ C.I. = P êêçç1 + ú
÷ - 1ú
÷
100 ø = ` 65800
ëêè úû
Total monthly investment in percentage
éæ
12 ö
3 ù = 13 + 23 + 8 = 44
= 20000 êêçç1 + ú
÷÷ - 1ú \ Total monthly investment
êëè 100 ø úû
= 44% of `65800
= 20000 [(1.12)3 – 1]
= 20000 × (1.404928 – 1) æ 44 ´ 65800 ö
= ` çç ÷÷
= `8098.56 è 100 ø
x = ` 28952
37. (e) Let the original fraction be y . \ Total annual investment
= `(12 × 28952)
According to the question,
= ` 347424
x ´ 400 5 41. (d) Let the C.P. of the article be ` x.
=
y ´ 600 12 According to the question,
x 5 6 5 878 – x = x – 636
Þ = ´ = Þ 2x = 878 + 636 = 1514
y 12 4 8
38. (b) Let the original sum be `x. 1514
Þx= =` 757
Sum of the Ratios 2
= 3 + 5 + 9 + 13 = 30 42. (c) Number of sweets received by each student
9x = 15% of 240
\ C 's share =`
30 15 ´ 240
= = 36
3x 100
=`
10 \ Total number of sweets
= 240 × 36 = 8640
3x
A’s share = ` 43. (b) Let the total amount be ` x.
30
The amount spent = 100 – 28 = 72%
x \ 72% of x
=`
10 = ` (45760 + 27896)
According to the question,
72 ´ x
3x x Þ = 73656
- = 2412 100
10 10
73656 ´100
2x Þx=
Þ = 2412 72
10
= `102300
y
o
u
rs
m
a
h
516 SBI Bank PO Prelim Exam 2015

b
o
o
44. (e) Let the present ages of Khushi and Jagriti be 5x and 183 × 4 + 4 × 4 = 748

b
8x years respectively.

.w
After 8 years, 748 × 5 + 5 × 3 = 3755

o
3755 × 6 + 6 × 2 = 22542

rd
5x + 8 3 Hence, 748 will replace the question mark.

p
=

re
8x + 8 4 51. (b) ? = 2959.85 ÷ 16.001 – 34.99

s
Þ 24x + 24 = 20x + 32 » 2960 ÷ 16 – 35

s
.c
Þ 4x = 32 – 24 = 8 2960

o
» - 35 »185 - 35

m
8 16
Þ x= =2
4
»150
45. (d) The word PUNCTUAL consists of 8 letters in which 52. (d) ? = (1702 ÷ 68) × 136.05
the letter ‘U’ comes twice.
1700
\ Number of arrangements. » ´136
68
8! 8 ´ 7 ´ 6 ´ 5 ´ 4 ´ 3 ´ 2 ´1
= = = 20160 » 3400
2! 1´ 2
46. (d) The given number series is based on the following 2950
53. (e) ?= + 160
pattern: 12.25
93 + 2 = 95 2950
95 + 3 = 98 » + 160 » 405.8
12
98 + 5 = 103 Clearly 12,25 » 12 < 12.25
103 + 7 = 110 Hence, 2950 ÷ 12 will give larger quotient.
110 + 11 = 121 \ Our answer should be 405.
121 + 13 = 134 54. (a) ? = 25.05% 2845 + 14.95 × 2400
Hence, 103 will replace the question mark. 25
47. (e) The given number series is based on the following » ´ 2845 + 15 ´ 2400
100
pattern:
» 711.25 + 36000
8 × 1.5 = 12
» 36711.25 » 36700
12 × 1.5 = 18
186 ´ 271
18 × 1.5 = 27 55. (e) ? » »1260.15
27 × 1.5 = 40.5 40
40.5 × 1.5 = 60.75 »1260
\ ? = 60.75 × 1.5 = 91.125 56. (b) (I) p2 – 7p = – 12
Hence, 91.125 will replace the question mark. Þ p2 – 4p + 12 = 0
48. (e) The given number series is based on the following Þ p2 – 4p – 3p + 12 = 0
pattern:
4 + 7 = 11 Þ p (p – 4) –3 (p –4) = 0
11 + 7 = 18 Þ (p – 4) (p – 3) = 0
18 + 11 = 29
Þ p = 3 or 4
\ ? = 29 + 18 = 47
(II) q2 – 3q + 2 = 0
Hence, 47 will replace the question mark.
49. (a) The given number series is based on the following Þ q2 – 2q – 9 + 2 = 0
pattern: Þ q (q – 2) –1 (q – 2) = 0
3 × 2 + 22 = 10
Þ (q – 2) (q – 1) = 0
10 × 3 + 32 = 39
39 × 4 + 42 = 172 Þ q = 1 or 2
172 × 5 + 52 = 885 Obviously p > q
885 × 6 + 62 = 5346
57. (a) (I) 12p2 – 7p = – 1
Hence, 39 will replace the question mark.
50. (c) The given number series is based on the following Þ 12p2 – 7p + 1 = 0
pattern : Þ 12p2 – 4p – 3p + 1 = 0
15 × 1 + 1 × 7 = 22
22 × 2 + 2 × 6 = 56 Þ 4p (3p –1) –1 (3p –1) = 0
56 × 3 + 3 × 5 = 183 Þ (3p –1) (4p – 1) = 0
y
o
u
rs
m
a
h
SBI Bank PO Prelim Exam 2015 517

b
o
o
1 1 Þ q = ±5

b
Þp= or

.w
4 3 Obviously, p £ q,

o
2

rd
(II) 6q – 7q + 2 = 0 61. (c) Production of Company A

p
2

re
Þ 6q – 4q – 3q + 2 = 0 in 2008 = 4 lakh units

s
s
Þ 2q (3q – 2) –1 (3q – 2) = 0 in 2009 = 3 lakh units

.c
o
Þ (3q – 2) (2q – 1) = 0 Decrease = 4 – 3 = 1 lakh units

m
2 1 \ % decrease
Þq = or
3 2
1
= ´100 = 25%
Obviously, p £ q 4
58. (c) (I) p2 + 12p + 35 = 0 62. (b) Production of Company B
2
Þ p + 7p + 5p + 35 = 0 in 2013 = 7 lakh units
Þ p (p + 7) + 5 (p + 7) = 0 in 2014 = 11 lakh units.
Þ (p + 7) (p + 5) = 0 \ Required percentage
Þ p = – 5 or – 7
11
= ´100 = 157.14 »157%
(II) 2q2 + 22q + 56 = 0 7
Þ 2q2 + 14q + 8q + 56 = 0 63. (d) It is obvious from the graph.
Þ 2q (q + 7) + 8 (q + 7) = 0 64. (a) Total production of Company A
Þ (q + 7) (2q + 8) = 0 = (4 + 3 + 8 + 8 + 8 + 7 + 8 + 12) = 58 lakh units
Þ q = – 7 or – 4 = 5800000
Obviously, p £ q Total production of Company B
2
59. (d) (I) p – 8p + 15 = 0 = (1 + 1 + 1 + 2 + 3 + 5 + 7 + 11) = 31 lakh units
2
Þ p – 3p – 5p + 15 = 0 = 3100000
Þ p (p – 3) – 5 (p – 3) = 0 \ Required difference
Þ (p – 3) (p – 5) = 0 = (58 – 31) × 100000
Þ p = 3 or 5 = 2700000
2
(II) q – 5q = – 6 65. (e) Required average production of Company B
2
Þ q – 5q + 6 = 0 31
2
= lakh units
Þ q – 3q – 2q + 6 = 0 8

Þ q (q – 3) – 2 (q – 3) = 0 = 3.875 » 3.9 lakh units


Þ q (q – 3) (q – 2) = 0 66. (a) For course D
Þ q = 3 or 2 No. of girls = 30% of 800
Obviously, p ³ q. 30 ´ 800
= = 240
2 100
60. (c) (I) 2p + 20p + 50 = 0
Þ p 2 + 10p + 25 = 0 No. of students
2
Þ (p + 5) = 0 1
= 35 ´ ´1200 = 420
Þp+5=0 100

Þp=–5 \ No. of boys = 420 – 240 = 180


(II) q2 = 25 \ Required ratio = 180 : 240 = 3 : 4
y
o
u
rs
m
a
h
518 SBI Bank PO Prelim Exam 2015

b
o
o
67. (c) Number of boys 76. (b) 77. (c) 78. (d) 79. (a) 80. (b)

b
81. (d) indispensable means essential, and fits the blank.

.w
in course E = 12% of 1200 – 14% of 800 = 144 – 112
82. (d) pertinent means “relevant or applicable to the matter

o
= 32

rd
at hand”. So it fits the blank.

p
in course F = 13% of 1200 – 14% of 800 = 156 – 112 83. (b) align means to “adjust or support”, and fits the blank.

re
= 44 84. (a) The correct answer here is educate. While the other

s
s
.c
in course A = 20% of 1200 – 30% of 800 = 240 – 240 words have similar meaning, they do not make the

o
=0 sentence grammatically correct.

m
85. (b) Legal aid is “granted”. So, it fits the blank.
in course D = 35% of 1200 – 30% of 800 = 420 – 240
86. (a) Either you have to remove “comparatively” or convert
= 180
“better” to “good” in the sentence. The meaning of
in course C = 5% of 1200 – 2% of 800 = 60 – 16 = “comparatively” is - to evaluate anything using
44 comparison estimate by comparison, etc. hence you
Obviously pair C and F is our answer. should use either “better” or “comparatively good” in
order to make the sentence correct.
68. (a) For Course E, 87. (c) “Kindly” will not be used here; because “kindly” and
No. of girls = 14% of 800 “requested” are never used simultaneously “kindly”
and “please” are used in Active Voice to denote
= 112
“request”; while denote “kindly” and “please” in
No. of boys = 32 Passive Voice, “Requested” is generally used.
\ Required percentage 88. (c) The use of “it” is superfluous here, as the usage of the
Object of “bought”- the book: is correct in the
80 sentence.
= ´100 = 250
32 89. (a) Instead of “passing marks” you should use “pass
marks”, because “passing marks” is not correct
69. (d) 90. (a) You should use “neighbours” in stead of
70. (b) No. of girls in course C “neighbourers” because, there is no word in English
like “neighbourers”. The meaning of “neighbours” is
= 2% of 800 = 16
people who live or are situated nearby.
71-75 : The Correct order is:- C D A B F E 91. (d) 92. (c) 93. (d) 94. (d) 95. (c) 96. (a)
71. (e) 72. (c) 73. (d) 74. (a) 75. (b) 97. (b) 98. (a) 99. (d) 100. (c)
y
o
u
rs
m
a
h
b
o
o
b
.w
o
SBI BANK PO MAIN EXAM 2015

rd
p
re
s
s
Based on Memory

.c
o
m
REASONING ABILITY 8. Statements
S³ M < Y = Z > F > T
Conclusions
DIRECTIONS (Qs. 1-3): Study the following information to
I. S > F II. Y> T
answer the given questions.
In a certain code ‘colours of the sky’ is written as ‘ki la fa so’, DIRECTIONS (Qs. 9-13): Study the following information
‘rainbow colours’ is written as ‘ro ki’ and ‘sky high rocket’ is carefully and answer the given questions.
written as ‘la pe jo’ and ‘the rocket world’ is written as ‘pe so ne’. (i) A, B, C, D, E, F, G and H are sitting around a circle facing
1. Which of the following is the code for ‘colours sky high’? the centre but not necessarily in the same order.
(a) ro jo la (b) fa la jo (ii) B sits second to left of H’s husband. No female is an
(c) la ki jo (d) ki jo la immediate neighbour of B.
(e) fa ki jo (iii) D’s daughter sits second to right of F, F is the sister of G. F
2. Which of the following will/may represent ‘the’? is not an immediate neightbour of H’s husband.
(a) Only fa (b) Only la (iv) Only one person sits between A and F. A is the father of G.
(c) Only so (d) Either (a) or (b)
H’s brother D sits to the immediate left of H’s mother. Only
(e) Either (a) or (c)
one person sits between H’s mother and E.
3. What does ‘pe’ represent in the code?
(v) Only one person sits between H and G. G is the mother of
(a) colours (b) sky
(c) high (d) rainbow C. G is not an immediate neighbour of E.
(e) rocket 9. What is position of A with respect of his mother-in-law?
(a) Immediate left (b) Third to the right
DIRECTIONS (Qs. 4-8): In these questions, relationship (c) Third to the left (d) Second to the right
between different elements is shown in the statements. These (e) Fourth to the left
statements are followed by two conclusions. 10. What is the position of A with respect to his mother-in-
Give answer grandchild?
(1) If only conclusion I follows (a) Immediate Right (b) Third to the right
(2) If only conclusion II follows (c) Third to the left (d) Second to the right
(3) If either conclusion I nor II follows (e) Fourth to the left
(4) If neither conclusion I nor II follows 11. Four of the following five are alike in a certain way based
(5) If both conclusion I and II follows on the given information and so form a group. Which is
4. Statements the one that does not belong to that group?
W³D < M < P < A = F (a) F (b) C
Conclusions (c) E (d) H
I. F > D II. P < W (e) G
5. Statements 12. Which of the following is true with respect to the given
H³ M > F < A = B > S seating arrangement?
Conclusions (a) C is the cousin of E
I. H > B II. F < S (b) H and H’s husband are immediate neighbours of each
6. Statements other
B>T>Q>R=F (c) No female is an immediate neighbour of C
Conclusions (d) H sits third to left of her daughter
I. Q ³ F II. T > F (e) B is the mother of H
7. Statements 13. Who sits to the immediate left of C?
S = R ³ Q, P < Q (a) F’s grandmother (b) G’s son
Conclusions (c) D’s mother-in-law (d) A
I. S ³ P II. R > P (e) G
y
o
u
rs
m
a
h
520 SBI Bank PO Main Exam 2015

b
o
o
DIRECTIONS (Qs. 14-18): In each question given four Input : gate 20 86 just not 71 for 67 38 bake sun 55

b
Step I : bake gate 20 just not 71 for 67 38 sun 55 86

.w
statements are followed by three Conclusions I, II and III. You
Step II : for bake gate 20 just not 67 38 sun 55 86 71

o
have to take the four given statements to be true even if they

rd
seem to be tat variance from the commonly known facts. Read Step III : gate for bake 20 just not 38 sun 55 86 71 67

p
the conclusions and decide which logically follows from the four Step IV : just gate for bake 20 not 38 sun 55 86 71 67

re
given statements disregarding commonly known facts. Step V : not just gate for bake 20 sun 86 71 67 55 38

s
s
Step VI : sun not just gate for bake 86 71 67 55 38 20

.c
Qs. No. 14-15

o
Steps VI is the last step of the arrangement the above input.

m
Statements All footballers are music lover.
As per the rule followed in the above steps, find out in each of
All footballers are dancer.
the following questions the appropriate step for the given input.
No dancer is cricketer.
No cricketer is player. Input: 31 rise gem 15 92 47 aim big 25 does 56 not 85 63 with
14. Conclusions I. Dancer can be player being a possibility moon
II. All those dancer are footballers are also 19. How many steps will be required to complete the
music lovers. rearrangement?
III. Any cricketer can never be player. (a) Eight (b) Six
(a) None follow (b) All follow (c) Seven (d) Five
(c) Only I follows (d) Both II and III follow (e) None of these
(e) None of these 20. Which words numbers would be at 7th position from the
15. Conclusions I. All cricketer can be music lover. left in step IV?
II. Some player are footballers. (a) rise (b) aim
III. At least some footballers are cricketer. (c) big (d) 15
(a) Only I follows (b) I and III follow
(e) 47
(c) II and III follow (d) None follows
21. Which step number is the following output?
(e) None of these
Qs. No. 16-18 rise not moon gem does big aim 15 with 92 85 63 56 47 31 25
Statements All boys are intelligent. (a) Step V (b) Step VII
Very few girls are intelligent. (c) Step IV (d) Step VIII
None girl is leader. (e) There is no such step
Some professor are leader as 22. Which of the following represents the position of ‘92’ in
well as boys. step VI ?
16. Conclusions I. 36% of girls are intelligent. (a) Ninth from the left (b) Fifth from the right
II. 2% girls are intelligent. (c) Sixth from the right (d) Ninth from the right
III. Some boys can never be a girl.
(e) Seventh from the left
(a) Only II follows (b) Only III follows
(c) Both I and II follow (d) None follows 23. Which words numbers would be at 5th position from the
(e) All follow right in the last step?
17. Conclusions I. All leaders are professors. (a) gem (b) 63
II. There is a possibility that every girl can (c) 56 (d) 85
be professor. (e) does
III. All professor can be leaders.
(a) None follows DIRECTIONS (Qs. 24-28) : Each of the questions below
(b) Only II follows consists of a question and two statements numbered I and II
(c) Both II and III follows given below it. You have to decide whether the data provided in
(d) Only III follows the statements are sufficient to answer the question. Read both
(e) None of these the statements and and answer the questions.
18. Conclusions I. Atleast some professors are girls Give answer (a) if the data in statement I alone is sufficient to
II. No professors is intelligent. answer the question, while the data in statement II alone are not
III. 5% professors are leader.
sufficient to answer the question.
(a) Only I follows (b) Only II follows
Give answer (b) if the data in statement II alone are sufficient to
(c) Only III follows (d) Both II and III follows
(e) None of these answer the question, while the data in statement I alone are not
sufficient to answer the question.
DIRECTIONS (Qs. 19-23): Study the following information Give answer (c) if the data either in statement I alone or in
carefully and answer the given question: statement II alone are sufficient to answer the question; and
A word and number arrangement machine when given an input Give answer (d) if the data given in both the statements I and II
line of words and numbers rearrange them following a particular together are not sufficient to answer the question; and
rule in each step. The following is an illustration of input and Give answer (e) if the data in both the statements I and II together
rearrangement. (All the numbers are two-digit numbers). are necessary to answer the question.
y
o
u
rs
m
a
h
SBI Bank PO Main Exam 2015 521

b
o
o
24. How is J related to K? (c) Shorter people suffer from inferiority complex

b
I. J’s father P is the brother of N. N is K’s wife. throughout them life

.w
II. J is the son of P. P is the brother of N. N is K’s wife. (d) Taller people are found to be more positive in

o
rd
25. On which floor of the building does G stay? (the building approach to daily chores of lifes.

p
has five floor 1, 2, 3, 4, 5.) (e) None of the above

re
I. Only the even-numbered floors are occupied and G 31. Statement

s
s
Many patients suffering form malaria were administered

.c
does not stay on the second floor.
anti-malarial drug for a week. Some of them did not respond

o
II. G does not stay on an odd-numbered floor.

m
to the traditional drug and their condition deteriorated after
26. How many days did Raju take to complete his assignment? four days.
I. Mohit correctly remembers that Raju took more than Which of the following would weaken the findings
3 days less than 9 days to complete his assignment. mentioned in the above statement?
II. Mina correctly remembers that Raju took more than (a) Those patients who responded to the traditional drugs
7 days less than 11 days to complete his assignment. and recovered were needed to be given additional does
27. How is the word ‘GATES’ coded in the code language? as they reported relapse of symptoms
I. ‘BRICK’ is coded as ‘LDJSC’ and ‘PIN’ is coded as (b) The mosquitoes carrying malaria are found to be
‘OJQ’ resistant to traditional malarial drugs
(c) Majority of the patients suffering from malaria
II. ‘WATER’ is coded as ‘SFUBX’ and ‘DISH’ is coded responded well to the traditional malarial drugs and
as ‘ITJE’ recovered from the illness
28. Among A, B, C and D, which school has the highest number (d) Many drug companies have stopped manufacturing
of students. traditional malarial drugs
I. School A has fewer students than school D. (e) None of the above
II. School C has fewer students than school D. 32. Statement
29. In Swartkans territory, archaeologists discovered charred The cost of manufacturing cars in state A is 30% less than
bone fragments dating back one million years. Analysis of the cost of manufacturing cars in state B. Even after
transportation fee for the differential distances of states A
the fragments, which came from a variety of animals,
and B and the interstate taxes, it is cheaper to manufacture
showed that they had been heated to temperatures no higher
cars in state B than state A for selling these cars in state C.
than those produced in experimental campfires made from
Which of the following supports the conclusion drawn in
branches of white, stinkwood, the most common tree around the above statement?
Swartkans. (a) The cost of transportation from state A to state C is
Which of the following, if true, would, together with the more than 30% of the production cost
information above, provide the best basis for the claim that (b) The production costs of cars in state B are lower in
the charred bone fragments are evidence of the use of fire comparison to state A
by early hominids? (c) Only Entry Tax at state C is more for the products
(a) The white stinkwood tree is used for building material originated in state A
by the present day inhabitants of Swartkans (d) Entry Tax at state C is more for the products originated
(b) Forest fires can heat wood to a range of temperatures in state B
(e) The total of transportation cost of cars from state B
that occur in campfires
to state C and Entry Tax of cars at state C is less than
(c) The bone fragments were fitted together by the 30% of the production cost of cars in state B
archaeologists to form the complete skeletons of 33. Statement
several animals. There was a slow decline in number of patients with flu
(d) The bone fragments were found in several distinct like symptoms visiting various health facilities in the city
layers of limestone that contained primitive cutting during the last fortnight.
tools known to have been used by early hominids Which of the following substantiates the facts mentioned
(e) None of the above in the above statement?
30. Statement (a) Majority of the people suffering from flu visit the
health facilities in the city
It has been reported in a recent research report that taller
(b) There has been a continuous increase in the sale of
people are found to be happier than those who are relatively medicines for curing flu in the city limits
shorter. (c) People have started visiting the crowed places like
Which of the following contradicts the above finding? malls and cinema balls during the last fortnight after
(a) People are generally truthful when they respond to cautioned gap of one month.
the questions of such research studies (d) There is a sudden increase in the number of deaths
(b) Those people who have a comfortable lifestyle are caused by flu like symptoms followed by respiratory
found to be happier than those who lead a strenuous complications.
lifestyle irrespective of their height (e) None of the above
y
o
u
rs
m
a
h
522 SBI Bank PO Main Exam 2015

b
o
o
34. Statement 40. The Chess player is a:

b
It is reported that during the last fortnight there has been (a) Power engineer (b) Mechanical engineer

.w
three cases of near miss situation involving two aircrafts (c) Design engineer (d) Quality inspector

o
rd
over the busy city airport. In all these cases both the aircrafts (e) None of these

p
came perilously close to each other and could avoid collision

re
as the pilots acted just in time. DIRECTIONS (Qs. 41-45): Study the following information

s
Courses of action carefully and answer the given questions.

s
.c
I. The pilots of all the six aircrafts involved in these Eight Childhood friends A, B, C, D, E, F, G and H are sitting

o
incidents should immediately be derostered.

m
around a circular table facing the centre, but not necessarily in
II. Some flights should be diverted to other airports for
the same order. Each of them have completed their respected
the next few months to decongest the sky over the
city airport. courses viz MBA, MCA, B.Tech, CA, CS, B.Com, Finance and
III. Air traffic controllers of the city airport should be M.Tech.
sent for refresher courses in batches to prepare • There are two persons between those who study MCA and
themselves to handle such pressure situation. MBA and neither of them sit opposite A.
(a) Only 1 follows • C and G are immediate neighbours and neither of them
(b) Only II follows
(c) Only III follows studies M.Tech or MBA.
(d) II and III follows • A, who studies Finance, sits second to the right of one who
(e) None of these studies M.Tech.
DIRECTIONS (Qs. 35-40) : Read the following information • B and E neither study M.Tech or nor sit adjacent to A.
and answer the questions that follow : • E studies MBA and sits second to the left of G, who studies
In a Public Sector Undertaking Township, there are five executives CA.
– Ambrish, Amit, Rohit, Manu and Tarun. Two of them play • C does not study MCA.
Cricket while the other three play different games viz. Football,
Tennis and Chess. One Cricket player and a Chess player stay in • The immediate neighbour of E faces C, who sits third to the
the third flat, whereas the other three stay in different flats, i.e. left of the one who studies Finance.
2nd, 4th and 5th. Two of these five players are mechanical engineers • F does not study M.Tech.
while the other three are quality inspector, design engineer and • Both D and H do not study MCA and B.Tech.
power engineer respectively. The chess player is the oldest in
age while one of the cricket players, who plays at the national • B studies B.Com while D studies CS.
level, is the youngest in age. The age of the other cricket player, • B is not an immediate neighbour of A.
who plays at the regional level, lies between the football player 41. MCA is related to B.Com in a certain way and MBA is
and the chess player. Manu is a regional level player and stays in
related to M.Tech in the same way. Which of the following
the 3rd flat while Tarun is a quality inspector and stays in the
is related to B.Tech following the same pattern?
5th flat. The football player is a design engineer and stays in the
2nd Flat. Amit is a power engineer and plays Chess while Ambrish (a) CA (b) Finance
is the mechanical engineer and plays Cricket at the national level. (c) CS (d) can’t be determined
35. Who stays in the 4th flat? (e) none of these
(a) Ambrish (b) Amit 42. Who among the following studies MCA?
(c) Rohit (d) Manu (a) F (b) H
(e) None of these (c) E (d) can’t be determined
36. What does Tarun play? (e) none of these
(a) Chess (b) Football 43. Who among the following sits third to the right of F?
(c) Cricket (d) Tennis
(a) E (b) G
(e) None of these
37. Who plays football? (c) C (d) B
(a) Ambrish (b) Amit (e) none of these
(c) Rohit (d) Manu 44. Which of the following statements is/are true about E?
(e) None of these (a) E is sitting opposite H.
38. Agewise, who among the following lies between Manu and (b) E studies M.Tech
Tarun? (c) E is not an immediate neighbour of B
(a) Quality inspector (b) Mechanical engineer (d) All are true
(c) Power engineer (d) Design engineer
(e) none of these
(e) None of these
45. Which of the following courses is studied by C?
39. Who stay in the same flat?
(a) Ambrish and Amit (b) Maim and Tarun (a) CA (b) B.Tech
(c) Amit and Manu (d) Rohit and Tarun (c) CS (d) MBA
(e) None of these (e) None of these
y
o
u
rs
m
a
h
SBI Bank PO Main Exam 2015 523

b
o
o
DIRECTIONS (Qs. 46–49): Read the following information (a) 16707 (b) 16686

b
(c) 16531 (d) 16668

.w
carefully to answer the question:
(e) None of these

o
P X Q means “P is sister of Q”

rd
52. The total number of women liking RR forms approximately
P ÷ Q means “ P is mother of Q”

p
P + Q means “ P is brother of Q” what per cent of the total number of women liking all the

re
P – Q means “ P is father of Q” teams?

s
s
(a) 11 (b) 15

.c
46. Which of the following represent W is grandfather of H?
(c) 20 (d) 20

o
(a) W + T - H (b) W ÷ T – H

m
(c) W × T + H (d) W ÷ T + H (e) None of these
(e) None of these 53. What is the total number of children liking CSK?
47. Which of the following represent “M is nephew of R”? (a) 14085 (b) 16389
(a) M ÷ T – R (b) R ÷ T – M (c) 20031 (d) 14850
(c) R × T ÷ M × J (d) R ÷ T – M + J (e) None of these
(e) None of these 54. What is the ratio of total number of men liking DD to those
48. How T is related to S “W ÷ T – H + V – S”? liking RR?
(a) sister (b) mother
(c) aunt (d) uncle (a) 69 : 49 (b) 7 : 5
(e) None of these (c) 208 : 147 (d) 70 : 52
49. The expression means “S ÷ T – H × V – N”? (e) None of these
(a) S is grandmother of N 55. The number of men liking DC forms forms what per cent
(b) S is great grandmother of N of those liking RCB? (rounded off to two digits after
(c) S is mother of V decimal).
(d) N is grand son of S (a) 21.05% (b) 475.04%
(e) None of these (c) 25.56% (d) 25%
50. Statement: Exporters in the capital are alleging that
(e) None of these
commercial banks are violating a Reserve Bank of India
directive to operate a post shipment export credit DIRECTIONS (Qs. 56-60) : Study the following table carefully
denominated in foreign currency at international rates from and answers the questions given below it.
January this year.
Courses of Action: Statistics of the Quantity produced and quantity Sold by
I. The officers concerned in the commercial banks are Aishwarya Co Limited for the Five Months. The Stock at the
to be suspended. Beginning of June is 75 units.
II. The RBI should be asked to stop giving such directives Months Quantify produced Quantity sold
to commercial banks. June 300 350
(a) Only I follows (b) Only II follows July 250 275
(c) Either I or II follows (d) Neither I nor II follows August 200 175
(e) Both I and II follow September 200 200
October 325 300
DATA ANALYSIS AND INTERPRETATION 56. During which month, the company had the least stock to
began with?
DIRECTIONS (Qs. 51-55) : Study the table carefully to answer (a) June (b) July
the questions that follow. (c) August (d) October
Number of people Liking Eight Eifferent Teams in IPL-5 and (e) None of these
the percentages of Men, Women and Children Liking these 57. What was the stock at the beginning of September?
Tearms (a) 35 (b) 15
(c) 25 (d) 20
Teams Total number Percentage of
(e) None of these
of people Men Women Children
58. What should be the minimum stock at the beginnings of
CSK 45525 20 44 36 June, if the company has to supply smoothly for all the five
DD 36800 39 33 28 months?
DC 56340 45 30 25 (a) 45 units (b) 35 units
MI 62350 38 28 34 (c) 25 units (d) 30 units
RR 48300 21 44 35 (e) None of these
RCB 35580 15 35 50 59. During which month did the company have the highest
KXI 56250 24 36 40 sales?
KKR 64000 16 54 30 (a) June (b) July
51. What is the approximate average number of women liking (c) August (d) October
all the teams? (e) None of these
y
o
u
rs
m
a
h
524 SBI Bank PO Main Exam 2015

b
o
o
60. What is the stock at the beginning of July? Unit Price

b
(a) 50 (b) 25

.w
(c) 100 (d) 75 16

o
14 14 15
14

rd
(e) None of these 12

p
12 11
12

re
DIRECTIONS (Qs. 61-65) : The circle graph given here shows 10 10

s
the spending for a country on various sports during a particular 10 10 10

s
8

.c
year. Study the graph carefully and answer the questions given 8
6

o
m
below it. 4
2
0

2004

2005

2006

2007

2008

2009

2010

2012

2013

2014
2011
Cricket Hockey
81° 63°
Football Revenue — Profit
Basket Ball 54°
50° 1600
1400
Others 1400
Tennis 1200 1100
31° 1200
45° Golf
1000 900
36° 800
800 900 700
600 700
400
400 300 400 200
61. What per cent of total expenditure is spent on tennis? 200
200 100 150 150 100 300
0 0
1 1 0
(a) 12 % (b) 22 %
2004

2005

2006

2007

2008

2009

2010

2012

2013

2014
2011
2 2
(c) 25% (d) 45%
(e) 40% 66. In which year per unit cost in highest?
62. How much per cent more is spent on hockey than that on (a) 2006 (b) 2005
golf? (c) 2009 (d) 2011
(a) 27% (b) 35% (e) 2012
67. What is the approximate average quantity sold during the
(c) 37.5% (d) 75%
period 2004-2014?
(e) None of these (a) 64 units (b) 70 units
63. If the total amount spend on sports during the year be (c) 77 units (d) 81 units
`18000000, then the amount spent on basketball exceeds (e) 87 units
on Tennis by 68. If volatility of a variable during 2000-2014 is defined as
(a) `250000 (b) `360000
Maximum value – Minimum value
(c) `375000 (d) `410000 , then which of the
Average value
(e) `30000 following is true?
64. How many per cent less is spent on football than that on (a) Price per unit has highest volatility
Cricket? (b) Cost per unit has highest volatility
2 (c) Total profit has highest volatility
(a) 22 % (b) 27%
9 (d) Revenue has highest volatility
1 1 (e) None of the above
(c) 33 % (d) 37 % 69. If the price per unit decrease by 20% during 2004-2008
3 7
and cost per unit increase by 20% during 2009-2014, then
(e) 29% during how many number of years there is loss?
65. If the total amount spent on sports during the year was `2 (a) 3 yr (b) 4 yr
crore, the amount spent on cricket and hockey together was (c) 5 yr (d) 7 yr
(a) `800000 (b) `8000000 (e) None of these
(c) `12000000 (d) `16000000 70. If the price per unit decrease by 20% during 2000-2004
(e) None of these and cost per unit increase by 20% during 2005-2010, then
the cumulative profit for the entire period 2000-2010
DIRECTIONS (Qs. 66-70): Answer the questions based on the decrease by
following two graphs, assuming that there is not fixed cost (a) `1650 (b) `1550
component and all the units produced are sold in the same (c) `1300 (d) `1250
year. (e) Data inadequate
y
o
u
rs
m
a
h
SBI Bank PO Main Exam 2015 525

b
o
o
DIRECTIONS (Qs. 71-75) : Study the following graph carefully in B.Tech. (computer science). 18% of boys are in B.Tech.

b
(Mechanical). Girls in B.Tech. (civil) are 30% of the girls in B.

.w
to answer the question that follow.
Tech. (computer science). 15% of boys are in B. Tech.

o
Number of foreign tourists who visited three

rd
different places in five different days (electronics). Boys in B. Teach (Computer Science) are 50% of

p
the girls in the same 15% of girls are in B. Tech. (aerospace).

re
Golden Temple Wagha Border Taj Mahal
The ratio of boys to girls in B.Tech. (civil) is 3:1 respectively.

s
300

s
.c
24% of the total numbers of students are in B.Tech. (electronics).

o
250 The ratio of boys to girls in B.Tech. (aerospace) is 12:5,

m
Number of tourists

200 respectively.
76. What is the total number of students enrolled in B.Tech.
150 (mechanical)?
100
(a) 1062 (b) 1530
(c) 1584 (d) 1728
50 (e) 1800
0
77. Number of girls enrolled in B.Tech. (electronics) forms
Monday Tuesday Wednesday Thursday Friday approximately. What per cent of total number of students
Days in the college?
71. What is the average number foreign tourists who visited (a) 7% (b) 13%
Wagha Border on all the days together? (c) 15% (d) 22%
(a) 184 (b) 172 (e) 24%
(c) 186 (d) 174 78. What is the total number of girls enrolled in B.Tech. (nano
(e) None of these technology)?
72. Total number of foreign tourists who visited all three places (a) 144 (b) 306
on Tuesday together is what per cent of the total number (c) 365 (d) 480
foreign tourists who visited Wagha Border on Monday and (e) 522
Wednesday together? 79. Number of boys enrolled in B. Tech. (aerospace) forms,
(a) 168% (b) 124% what per cent of the total number of girls enrolled in B.Tech.
(c) 112% (d) 224% (computer science)?
(d) 212% (a) 187.5% (b) 200%
73. On which day was the total number of foreign tourists who (c) 212.5% (d) 225%
visited all the three places together second lowest?
(e) 232.5%
(a) Monday (b) Tuesday
80. What is the total number of boys enrolled in B. Tech. (civil)?
(c) Wednesday (d) Thursday
(a) 240 (b) 432
(e) Friday
(c) 630 (d) 756
74. What is the respective ratio between the number of foreign
(e) 810
tourists who visited Taj Mahal on Thursday and the number
of foreign tourists who visited Golden Temple on Friday? DIRECTIONS (Qs. 81-85): In the following questions three
(a) 14:9 (b) 8:13 equations numbered I, II and III are given. You have to solve
(c) 13:8 (d) 13:7 all the equations either together or separately, or two together
(e) None of these and one separately, or by any other method and—
75. On which day was the different between the number of Give answer If
foreign tourists who visited Taj Mahal and Golden Temple, (a) x < y = z (b) x < y < z
second highest? (c) x < y > z (d) x = y > z
(a) Monday (b) Tuesday (e) x = y = z or if none of the above relationship is established
(c) Wednesday (d) Thursday 81. I. 7x + 6y + 4z = 122
(e) Friday II. 4x + 5y + 3z = 88
DIRECTIONS (Qs. 76-80): Study the information carefully to III. 9x + 2y + z = 78
answer the following questions. 82. I. 7x + 6y = 110
II. 4x + 3y = 59
There are 7200 students in an engineering college. The ratio of III. x + z = 15
boys of girls is 7:5, respectively. All the students are enrolled in
six different specialization viz., B.Tech. (electronics), B.Tech. 83. I. x= é(36)1/ 2 ´ (1296)1/ 4 ù
(Computer science), B.Tech. (Mechanical), B. Tech. (Aerospace), ë û
B.Tech. (nano technology), B.Tech. (civil). 22% of the total II. 2y + 3z = 33
students are in B. Tech. (nano technology). 16% of the girls are III. 6y + 5z = 71
y
o
u
rs
m
a
h
526 SBI Bank PO Main Exam 2015

b
o
o
84. I. 8x + 7y = 135 92. 46 9 13.5 20.25 30.375?

b
II. 5x + 6y = 99

.w
(a) 40.25 (b) 45.5625
III. 9y + 8z = 121

o
(c) 42.7525 (d) 48.5625

rd
85. I. (x + y)3 = 1331 (e) None of these

p
II. x– y+ z= 0

re
93. 400 240 144 86.4 51.84 31.104 ?
III. xy = 28

s
s
(a) 19.2466 (b) 17.2244

.c
DIRECTIONS (Qs. 86-90) : Each of the questions below (c) 16.8824 (d) 18.6625

o
m
consists of a question and two statements numbered I and II (e) None of these
given below it. You have to decide whether the data provided in
94. 9 4.5 4.5 6.75 13.5 33.75?
the statements are sufficient to answer the question. Read the
(a) 101.25 (b) 103.75
question and both the statements and -
(c) 99.75 (d) 105.50
Give answer (a) if the data in statement I alone are (e) None of these
sufficient to answer the question, while the data in statement II
95. 705 728 774 843 935 1050 ?
alone are not sufficient to answer the question.
(a) 1190 (b) 1180
Give answer (b) if the data in statement II alone are
sufficient to answer the question, while the data in statement I (c) 1185 (d) 1187
alone are not sufficient to answer the question. (e) None of these
Give answer (c) if the data either in statement I alone or DIRECTIONS (Qs. 96-97) : Read the information given below
in statement II alone are sufficient to answer the question. and answer the questions that follow :
Give answer (d) if the data even in both the statements I
and II together are not sufficient to answer the question. The tournament for ABC Cup is arranged as per the following
Give answer (e) if the data in both the statements I and II rules: in the beginning 16 teams are entered and divided in 2
together are necessary to answer the question. groups of 8 teams each where the team in any group plays exactly
once with all the teams in the same group. At the end of this
86. Train 'A' running at a certain speed crosses another train 'B'
round top four teams from each group advance to the next round
running at a certain speed in the opposite direction in 12
in which two teams play each other and the losing team goes out
seconds. What is the length of train 'B'?
of the tournament. The rules of the tournament are such that
I. The length of both the trains together is 450 metres.
every match can result only in a win or a loss and not in a tie.
II. Train 'A' is slower than train 'B'.
The winner in the first round takes one point from the win and
87. Area of a rectangle is equal to the area of a right angled
the loser gets zero. In case of tie on a position the rules are very
triangle. What is the length of the rectangle ?
complex and include a series of deciding measures.
I. The base of the triangle is 40 cms.
II. The height of the triangle is 50 cms. 96. A red light flashes 3 times per minute and a green light
88. What was the total compound interest on a sum after three flashes 5 times in two minutes at regular intervals. If both
years ? lights start flashing at the time, how many times do they
I. The interest after one year was ` 100/- and the sum flash together in each hour?
was ` 1,000/-. (a) 30 (b) 24
II. The difference between simple and compound interest (c) 20 (d) 60
on a sum of ` 1,000/- at the end of two years was ` 10/-. (e) None of these
89. What is the two digit number where the digit at the unit 97. Ashish is given Rs. 158 in one rupee denominations. He
place is smaller ? has been asked to allocate them into a number of bags such
I. The difference between the two digits is 5. that any amount required between Re. 1 and Rs. 158 can
II. The sum of the two digits is 7. be given by handing out a certain number of bags without
90. What is the speed of the boat in still water ? opening them. What is the minimum number of bags
I. It takes 2 hours to cover distance between A and B required?
downstream. (a) 11 (b) 12
II. It takes 4 hours to cover distance between A and B (c) 13 (d) 15
upstreams. (e) None of these
DIRECTIONS (Qs. 91-95) : What should come in place of
DIRECTIONS (Qs. 98-100) : These questions are based on the
question mark (17) in the following number series ?
information and graph given below.
91. 13 14 30 93 376 1885 ? These questions are based on the price fluctuations of four
(a) 10818 (b) 10316 commodities - arhar, pepper, sugar, and gold during February -
(c) 11316 (d) 11318 july 1999 as described in the figures below :
(e) None of these
y
o
u
rs
m
a
h
SBI Bank PO Main Exam 2015 527

b
o
o
Arhar (Friday quotations) Pepper (Friday quotations)

b
.w
2500 20000

Price (Rs. per quintal)

Price (Rs. per quintal)

o
2300 19500

rd
2100 19000

p
1900 18500

re
s
1700 18000

s
.c
1500 17500

o
1300 17000

m
F1 F4 M3 A2 A5 M3 JN2 JL1 F1 F4 M3 A2 A5 M3 JN2 JL1
Weeks (1999) Weeks (1999)

Sugar (Friday quotations) Gold (Friday quotations)


1520 4300
Price (Rs. per quintal)

Price (Rs. per quintal)


1500 4200
1480 4100
1460 4000
1440 3900
1420 3800
1400 3700
F1 F4 M3 A2 A5 M3 JN2 JL1 F1 F4 M3 A2 A5 M3 JN2 JL1
Weeks (1999) Weeks (1999)

98. Price change of a commodity is defined as the absolute 101. She asked me (a)/ Where I was going to (b)/ and what I
difference in ending and beginning prices expressed as a had done (c)/ the previous day. (d)/ No error (e)
percentage of the beginning. What is the commodity with 102. Yesterday in the night (a)/ he came (b)/ by bus (c)/ and was
the highest price changes disturbed. (d)/ No error (e)
(a) Arhar (b) Pepper 103. Within two hours (a)/ we will approach (b)/ near Agra (c)/
(c) Sugar (d) Gold by car. (d)/ No error (e)
(e) None of these 104. Throughout the whole year (a)/ there was (b)/ not a single
99. Price volatility (PV) of a commodity is defined as follows day (c)/ without any incidence of violence. (d)/ No error
PV = (highest price during the period- lowest price during (e)
the period)/ average price during the period and
105. The P.M. said (a)/ that it was his decision (b)/ and that
Highest price + lowest price + nobody (c)/ could get it changed. (d)/ No error (e)
ending price + beginning price DIRECTIONS (Qs. 106-110): In each of the following
Average price =
4 sentences, there are two blank spaces. Below each sentence there
What is the commodity with the lowest price volatility? are five pairs of words denoted by numbers (1), (2), (3), (4) and
(a) Arhar (b) Pepper (5). Find out which pair of words can be filled up in the blanks
(c) Sugar (d) Gold in the sentences in the same sequence to make the sentence
(e) None of these grammatically correct.
100. Mr. X, a fund manager with an investment company 106. The ___________ you work, the ___________ for your
invested 25% of his funds in each of the four commodities prosperity.
at the beginning of the period. He sold the commodities at (a) more, best (b) least, best
the end of the period. His investments in the commodities
(c) harder, better (d) decent, brightest
resulted in
(a) 17% profit (b) 5.5% loss (e) better, brighter
(c) no profit, no loss (d) 5.4% profit 107. ___________ you need a duplicate ration card, you must
(e) None of these submit the ___________ of your residence.
(a) Should, proof (b) If, numbers
ENGLISH LANGUAGE (c) Had guarantee (d) Do, number
(e) Would, document
DIRECTIONS (Qs. 101-105): Read each sentence to find out 108. It was my ___________ that ___________ to the serious
whether there is any error in it. The error, if any, will be in one problem.
part of the sentence, the name of the part is the answer. If there (a) desire, brought (b) negligence, led
is no error, the answer is (e). (c) fault, lauded (d) mistake, subjected
(e) decision, put
y
o
u
rs
m
a
h
528 SBI Bank PO Main Exam 2015

b
o
o
109. Because he was ___________ he left the party earlier and Delinking of jobs 116 degrees is one of the salient 117 of our

b
___________ home. education 118. There has been a steep 119 in 120 in the academic

.w
(a) tired, brought (b) precarious, approached field in recent years. There is a 121 of degree holders in the

o
rd
(c) preoccupied, sent (d) ill, contacted country. As a result, university degrees have lost their value and

p
(e) unwell, returned charm while the number of students in colleges and universities

re
110. An impartial person ___________ others without any of the country has been steadily 122. Consequently, thousands of

s
s
___________ . graduates and postgradutes come out of these institutions and

.c
(a) likes, reservation (b) judges, bias stand in 123 waiting to get some 124 jobs 125 in the country.

o
m
(c) blames, prudence (d) praises, point 116.(a) to (b) with
(e) wishes, malice (c) from (d) by
(e) None of these
DIRECTIONS (Qs. 111-115): Which of the phrases (a), (b), (c)
and (d) given below each sentence should replace the phrase 117.(a) factors (b) features
printed in bold in the sentence to make it grametically correct ? (c) reasons (d) methods
If the sentence is correct as it is given and no correction is (e) None of these
required mark (e) as the answer. 118.(a) process (b) system
(c) procedures (d) policy
111. The Governor has a good collection of paintings adoring
the walls of the Raj Bhavan. (e) None of these
(a) painting adoring the wall 119.(a) fall (b) rise
(b) painting adoring the walls (c) down (d) decrease
(c) painting sticking the walls (e) None of these
(d) painting adorning the walls 120.(a) evaluation (b) assessment
(e) No correction required (c) result (d) competence
112. The principals and teachers at the selected English (e) None of these
Medium School were contacted. 121.(a) flood (b) growth
(a) of the selected English Medium (c) increase (d) slope
(b) at the select English Medium (e) None of these
(c) upon the selected English Medium 122.(a) falling (b) diminishing
(d) of the section English Medium (c) rising (d) growing
(e) No correction required (e) None of these
113. Besides these norms, the data was also analyzed to form 123.(a) lines (b) long
the sub groups.
(c) rows (d) queues
(a) the data had analyzed also
(e) None of these
(b) the data were also concluded
(c) the data were also analyzed 124.(a) managerial (b) nice
(d) an data were analyzed also (c) prestigious (d) available
(e) No correction required (e) None of these
114. He does not know the Mount Everest is the Most highest 125.(a) vacate (b) posted
mountain peak. (c) created (d) available
(a) is a more highest mountain (e) None of these
(b) is the most high mountain DIRECTIONS (Qs. 126-130): There are six sentences marked
(c) is the higher mountain S1, S6, P, Q, R, S. The positions of S1 and S6 are fixed as the first
(d) is the highest mountain and last sentence of the passage. You are required to choose one
(e) No correction required of the five alternatives given below every passage which would
115. Walking in the morning in open air is being observed to be most logical sequence of the sentences in the passage.
be better for the health of the people who live in congestion.
(a) was observer the best 126. S1 : There are numerous kinds of superstitions in different
(b) hand been proved far better parts of the country.
(c) has been observed good S6 : A dog’s howling predicts death– this is a typical
(d) was proved far better superstition.
(e) No correction required P : But people go on respecting it through force of blind
custom.
DIRECTIONS (Qs. 116-125): In the following passage there Q : Most of them have a bearing on ‘luck’–good or bad.
are blanks, each of which has been numbered. These numbers R : Superstitions usually hae their origin in fear and
are printed below the passage and against each, four words are ignorance.
suggested, one of which fits the blank appropriately. Find out S : Nobody remembers now how a superstition first started
the appropriate word in each case. in remote ages.
y
o
u
rs
m
a
h
SBI Bank PO Main Exam 2015 529

b
o
o
The proper sequence should be Q : He returned to teaching is 1945 and gave it up in 1962,

b
(a) QPRS (b) RSPQ and is now a full-time writer.

.w
(c) RSQP (d) QSPR R : In 1939, he married and started teaching at Bishop

o
rd
(e) PRSQ Wordsworth’s School in Salisbury.

p
127. S1 : A spider’s web, after a shower of rain, is a very S : At first his novels were not accepted.

re
beautiful thing. The proper sequence should be

s
s
S6 : They are also feared because their bites may have (a) RPQS (b) RPSQ

.c
unpleasant effects like a rash on the skin. (c) SRPQ (d) SQPR

o
m
P : This party explains why spiders are thoroughly (e) RQPS
disliked.
Q : But no poet has ever sung of the beauty of the spiders, DIRECTIONS (Qs. 131-150): Read the following information
for most spiders are not beautiful. and answer the questions that follows:
R : On the contrary, most of them are rather unattractive, PASSAGE-1
if not ugly! True, it is the function of the army to maintain law and order in
S : Poets have sung about the beauty of the spider’s webs, abnormal times. But in normal times there is another force that
comparing the water drops on them to ropes of pearls. compels citizens to obey the laws and to act with due regard to
The proper sequence should be the rights of others. The force also protects the lives and the
(a) SPQR (b) QSRP properties of law abiding men. Laws are made to secure the
(c) QRSP (d) SQRP personal safety of its subjects and to prevent murder and crimes
(e) PQRS of violence. The are made to secure the property of the citizens
128. S1 : Unhappiness and discontent spring not only form against theft and damage to protect the rights of communities
poverty. and castes to carry out their customs and ceremonies, so long as
S6 : We suffer from sickness of spirit and hence we should they do not conflict with the rights of other. Now the good citizen,
discover our roots in the internal. of his own free will obey these laws and he takes care that
P : Man is a strange creature fundamentally different from everything he does is done with due regard to the rights and
other animals. well-being of others. But the bad citizen is only restrained from
Q : If they are undeveloped and unsatisfied, he may have breaking these laws by fear of the consequence of his actions.
all the comforts of the wealth, but will still feel that
And the necessary steps to compel the bad citizen to act as a
life is not worthwhile.
good citizen are taken by this force. The supreme control of law
R : He has far horizons invariable hopes, spiritual powers.
and order in a state is in the hands of a Minister who is responsible
S : What is missing our age is the soul, there is nothing
to the State Assembly and acts through the Inspector General of
wrong with the body.
Police.
The proper sequence should be
131. The expression ‘customs and ceremonies’ means
(a) PRQS (b) SPRQ
(a) fairs and festivals.
(c) SPQR (d) PRSQ
(b) habits and traditions.
(e) QSRP
(c) usual practices and religious rites.
129. S1 : On vacation in Tangier, Morocco, my friend and I sat
down at a street cafe. (d) superstitions and formalities.
S6 : Finally a man walked over to me and whispered, (e) None of these
“ Hey buddy .... This guy is your waiter and he wants 132. A suitable title for the passage would be :
your order.” (a) the function of the army.
P : At one point, he bent over with a big smile, showing (b) laws and the people’s rights.
me a single gold tooth and dingy fez. (c) the fear of the law and citizen’s security.
Q : Soon I felt the presence of someone standing alongside (d) the functions of the police.
me. (e) None of these
R : But this one wouldn’t budge. 133. Which of the following is not implied in the passage?
S : We had been cautioned about beggars and were told (a) Law protects those who respect it.
to ignore them. (b) Law ensures people’s religious and social rights
The proper sequence should be absolutely and unconditionally.
(a) SQRP (b) SQPR (c) A criminal is deterred from committing cirmes only
(c) QSRP (d) QSPR for fear of the law.
(e) PRQS (d) The forces of law help to transform irresponsible
130. S1 : In 1934, William Holding published a small volume citizens into responsibile ones.
of peoms. (e) None of these
S6 : But Lord of the flies which came out in 1954 was 134. According to the writer, which one of the following is not
welcomed as ‘a most absorbing and instructive tale’. the responsibility of the police?
P : During the World War II (1939-45) he joined the Royal (a) To protect the privileges of all citizens.
Navy and was present at the sinking of the Bismarck. (b) To check violent activities of citizens.
y
o
u
rs
m
a
h
530 SBI Bank PO Main Exam 2015

b
o
o
(c) To ensure peace among citizens by safeguarding (c) A Minister and a responsible State, Assembly exercise

b
individual rights. direct authority in matters pertaining to law and order.

.w
(d) To maintain peace during extraordinary circumstances. (d) The Inspector General of Police is responsible to the

o
rd
(e) None of these State Assembly for maintaining law and order.

p
135. Which of the following reflects the main thrust of the (e) None of thesess

re
passage. PASSAGE - 2

s
s
(a) It deals with the importance of the army in maintaining A recent report in News Week says that in American colleges,

.c
students of Asian origin outperform not only the minority group

o
law and other.

m
(b) It highlights role of the police as superior to that of students but the majority whites as well. Many of these students
the army. must be of Indian origin, and their achievement is something we
(c) It discusses the roles of the army and the police in can be proud of. It is unlikely that these talented youngsters will
different circumstances. come back to India and that is the familiar brain drain problem.
(d) It points to the responsibility of the Minister and the However recent statements by the nation’s policy-makers indicate
Inspector General of Police. that the perception of this issue is changing. ‘Brain bank’ and
(e) None of thesess not ‘brain drain’ is the more appropriate idea, they suggest since
136. “They are made to secure the property of citizens against the expertise of indians abroad is only deposited in other places
theft and damage”, means that the law : and not lost.
This may be so, but this brain bank, like most other banks,
(a) helps in recovering the stolen property of the citizens.
is one that primarily serves customers in its neighborhood. The
(b) assist the citizens whose property has been stolen or
skills of the Asians now excelling in America’s colleges will
destroyed.
mainly help the U.S.A.. No matter how significant, what non-
(c) initiate process against offenders of law.
resident Indians do for India and what their counterparts do for
(d) safeguard people’s possessions against being stolen other Asian lands is only a by-product.
or lost. But it is also necessary to ask, or be remained, why Indians
(e) None of thesess study fruitfully when abroad. The Asians whose accomplishments
137. Out of the following which one has the opposite meaning News Week records would have probably has a very different
to the word ‘restrained’ in the passage? tale if they had studied in India. In America they found elbow
(a) Promoted (b) Accelerated room, books and facilities not available and not likely to be
(c) Intruded (d) Inhibited available here. The need to prove themselves in their new country
(e) None of thesess and the competition of an international standard they faced there
138. Which one of the following statement is implied in the must have cured mental and physical laziness. But other things
passage? helping them in America can be obtained here if we achieve a
(a) Peaceful citizens seldom violate the law, but bad change is social attitudes, specially towards youth.
citizens have to be restrained by the police. We need to learn to value individuals and their unique
(b) Criminals, who flout the law, are seldom brought to qualities more than conformity and respectability. We need to
book. learn the language of encouragement to add to our skill in flattery.
(c) The police hardly succeed in converting bad citizens 141. Among the many groups of students in American colleges,
into good citizens. Asian students.
(d) The police check the citizens, whether they are good (a) are often written about in magazines like News Week.
or bad, and stop them from violating the law. (b) are most successful academically
(e) None of thesess (c) have proved that they are as good as the whites.
139. Which of the following statement expresses most accurately (d) have only a minority status like the blacks.
the idea contained in the first sentence? (e) None of these
(a) It is the job of the army to ensure internal peace at all 142. The student of Asian origin in America include :
times. (a) a fair number from India.
(b) It is the police that should always enforce law and (b) a small group from India.
order in the country. (c) persons from India who are very proud.
(c) Army and the police ensure people’s security through (d) Indians who are the most hard working of all.
combined operations. (e) None of thesess
(d) It is in exceptional circumstances that the army has to 143. In general, the talented young Indians studying in America:
ensure peace in the country. (a) have a reputation for being hard working
(e) None of thesess (b) have the opportunity to contr ibute to India’s
140. The last sentence of the passage implies that development.
(a) The Inspector General of Police is the sole authority (c) can solve the brain drain problem because of recent
in matters of law and order. changes in policy.
(b) In every State maintenance of public peace is under (d) will not return to pursue their careers in India.
the overall control of the responsible Minister. (e) None of thesess
y
o
u
rs
m
a
h
SBI Bank PO Main Exam 2015 531

b
o
o
144. There is talk now of the ‘brain bank’ This idea :
GENERAL AWARENESS

b
(a) is a solution to the brain drain problem

.w
(b) is a new problem caused parly by the brain drain 151. PM Narendra Modi launched which bank in In dia?

o
rd
(c) is a new way of looking at the role of qualified Indians (a) HDFC (b) ICICI

p
living abroad. (c) SBI (d) PNB

re
(d) is based on a plan to utilize foreign exchange

s
(e) Indian

s
remittances to stimulate research and development.

.c
152. LIC ( Life Corporation of India) started business in which
(e) None of thesess

o
country?

m
145. The brain bank has limitations like all banks in the sense
(a) Nepal (b) Myanmar
that:
(a) a bank’s services go mainly to those near it. (c) Bangladesh (d) Bhutan
(b) small neighbourhoods banks are not visible in this age (e) Sri Lanka
of multinationals. 153. Who is the CEAT cricketer of the year 2015?
(c) only what is deposited can be withdrawn and utilized. (a) M S Dhoni (b) Rahul Dravid
(d) no one can be forced to put his assets in a bank. (c) Virat Kholi (d) Kumar Sangkkara
(e) None of thesess (e) None of these
146. The author feels that what non-resident Indians do for India: 154. Where is Jim Corbett National Park is located?
(a) will have many useful side effects. (a) Uttar Pradesh (b) Odisha
(b) will not be their main interest and concern. (c) Bihar (d) Madhya Pradesh
(c) can benefit other Asian countries as a by-product. (e) Uttarakhand
(d) can American colleges be of service to the world 155. On which river Omkareswar dam is located?
community. (a) Tapti (b) Ravi
(e) None of thesess (c) Ganga (d) Narmada
147. The performance of Indians when they go to study in the (e) Satluj
West : 156. Which country recently launched the first ATM with face
(a) shows the fruits of hardwork done by school teachers recognition?
in India. (a) China (b) Nepal
(b) should remind us that knowledge and wisdom are not (c) India (d) Sri Lanka
limited by the boundaries of race and nation.
(e) Bangladesh
(c) is better than people in the West expect of non-whites.
157. What is Full Form of BIOS?
(d) is better than what it would have been if they had
studied in India. (a) Basic Investment/ Output System
(e) None of thesess (b) Basic In/ out System
148. The high level of competition faced by Asian students in (c) Basic Input/ Output System
America. (d) Basic Input/ Output Service
(a) helps them overcome their lazy habits. (e) None of these
(b) makes them lazy since the facilities there are good. 158. Agartala is the capital of which state ?
(c) makes them worried about failing. (a) Tripura (b) Assam
(d) helps them prove that they are as good as whites. (c) Mizoram (d) Odisha
(e) None of thesess (e) Nagaland
149. The author feels that some of the conditions other than the 159. Volkswagen Group and its subsidiaries, is a ________
level of facilities that make the West attractive. multinational automotive manufacturing company.
(a) are available in India but young people do not (a) British (b) American
appreciate them. (c) Chinese (d) German
(b) can never be found here because we believe in (e) Japanese
conformity. 160. In the Balance of Payment (BOP), the transfer of payment
(c) can be created if our attitudes and values change. is included in which of the following ?
(d) can also give respectability to our traditions and (a) Current Account
customs.
(b) Capital Account
(e) None of thesess
(c) Savings Accounts under Govt. Schemes
150. One of the ways of making the situation in India better
would be (d) Official Reserve Accounts
(a) to eliminate flattery from public life. (e) All of them
(b) to distinguish between conformity and respectability. 161. Who has been selected as head coach of India U-19?
(c) to give appreciation and not be tightfisted. (a) Rahul Dravid (b) Sachin Tendulkar
(d) to encourage people and no merely flatter them. (c) Saurabh Gaguly (d) Virendra Sehwag
(e) None of thesess (e) None of these
y
o
u
rs
m
a
h
532 SBI Bank PO Main Exam 2015

b
o
o
162. Where is Katasraj temple located? (a) NABARD (b) SEBI

b
(a) India (b) Pakistan (c) SIDBI (d) RBI

.w
(c) Afganistan (d) Nepal (e) Govt. of India

o
rd
(e) Bangladesh 174. Who is the author of the book 'Odyssey of My Life'?

p
163. Which of the following is not a part of Money Market? (a) Karan Singh (b) Kwasi Kwarteng

re
(a) Treasury Bill (b) Commercial Papers (c) James Astill (d) Shivraj V Patil

s
s
(c) Mutual Funds (d) Certificate of Deposits (e) None of these

.c
(e) None of these 175. World Photography day is observed on :

o
m
164. The Nobel Prize in Chemistry 2014 was awarded _______. (a) 19th August (b) 22nd August
(a) Eric Betzig (b) Stefan W. Hell (c) 20th August (d) 17th August
(c) William E.Moerner (d) All of the above (e) 16th August
(e) None of these 176. Which of the statement on Islamic banking is incorrect?
165. Land Border Agreement between India and Bangladesh was (A) Islamic banking is only for Muslims
the _________. (B) Islamic banking activity do not allow paying and
(a) 120th amendment (b) 100th amendment receiving of interest
(c) 117th amendment (d) 112th amendment (C) Islamic banking promotes the principle of profit
(e) None of these sharing between banking institution and depositor
166. The sixty-eighth session of the World Health Assembly (a) Only A (b) Only B
(WHA)took place in ________ 18-26 May 2015. (c) Only C (d) Both B and C
(a) Geneva (b) Rome (e) None of these
(c) Paris (d) Washington DC 177. The difference between the outflow and inflow of foreign
(e) None of these currency is known as ___
167. What term is used for maximum capital which the company (A) Foreign Exchange Reserves
can raise in its life time? (B) Current Account Deficit
(a) Authorized Capital (b) Registered Capital (C) Fiscal Deficit
(c) Nominal Capital (d) All of them (D) Balance of Payments
(e) None of these (a) Only A (b) Only B
168. Which state government approved mandatory Global (c) Only C (d) Only D
Positioning System (GPS) in all public transport vehicles (e) None of these
from June 2015? 178. NABARD (National Bank for Agriculture and Rural
(a) Delhi (b) Madhya Pradesh Development) established JLG to extract the productive
(c) Odisha (d) Maharashtra capability from rural population. JLG stands for ?
(e) Punjab (a) Junior Link Group (b) Join Life Group
169. China signed a currency swap deal with which country? (c) Joint Line Group (d) Joint Liability Group
(a) Sri Lanka (b) Russia (e) None of these
(c) India (d) Chile 179. What is MZM in money market?
(e) None of these (a) Money with highest return
170. The Concept of 'Universal Banking' was implemented in (b) Money with zero maturity
India on the recommendations of: (c) Money with sovereign guarantee
(a) Abid Hussai Committee (d) Monetary base or total currency
(b) R H Khan Committee (e) None of these
(c) S Padmanabhan Committee 180. A person who used his or expertise to gain access to other
(d) YH Malegam Committee peoples computers to get information illegally or do damage
(e) None of these is
171. Who among the following is the head of the committee (a) Hacker (b) Analyst
formed by SEBI to frame a single set of guidelines for all (c) Instant messenger (d) Programmer
types of foreign investors? (e) Spammer
(a) Prabhat Kumar (b) T.R. Prasad 181. To access properties of an object, the mouse technique to
(c) K.M. Chandrashekhar (d) Kamal Pande use is ________
(e) None of these (a) dragging (b) dropping
172. Largest shareholder (in percentage shareholding) of a (c) right-clicking (d) shift-clicking
nationalized bank is. (e) None of these
(a) RBI (b) NABARD 182. Computers used the _______ number systems to store data
(c) LICI (d) Govt. of India and perform calculations
(e) IBA (a) binary (b) octal
173. Which of the following organization finance the Rural (c) decimal (d) hexadecimal
Infrastructure Development Fund? (e) None of these
y
o
u
rs
m
a
h
SBI Bank PO Main Exam 2015 533

b
o
o
183. The default view in Excel is ________ View. 192. A marketing technique where marketer plays a specific role

b
.w
(a) Work (b) Auto in a particular segment is called -
(c) Normal (d) Roman

o
(a) Mass Marketing

rd
(e) None of these (b) Niche marketing

p
184. In word you can force a page break _______

re
(c) Strategic Marketing

s
(a) by positioning your cursor at the appropriate place (d) Communication Marketing

s
.c
and pressing the F1 key (e) None of these

o
(b) By positioning your cursor at the appropriate place

m
193. In Marketing Mix 4 P's Imply -
and pressing Ctrl + Enter (a) Product, Price, Place, Promotion
(c) By using the Insert/section break (b) Product, Price, Policy, Place
(d) By changing the font size of your documents
(c) Place, People, Product, Promotion
(e) None of these
(d) All of the above
185. Specialized programs that assist users in locating
(e) None of these
information on the Web are called
194. Good marketing strategy envisages good and proper ____.
(a) Information engines (b) locator engines
(a) Product distribution (b) Networking of branches
(c) web browsers (d) resource locators
(c) High Pricing (d) Placement of counter staff
(e) search engines
186. Compiling creates a(n) (e) Relationship management
(a) error-free program (b) program specification 195. Marketing in banks has been necessitated due to
(c) subroutine (d) algorithm (a) globalisation
(e) executable program (b) excess staff
187. Wha is the name given to those applications that combine (c) nationalisation of bank
text, sound, graphics, motion video, and/or animation? (d) complacency among the staff
(a) motionware (b) anigraphics (e) poor customer service
(c) videoscapes (d) multimedia 196. Innovation in marketing is same as-
(e) maxomedia (a) Abbreviation
188. How are the World Wide Web (WWW) and the internet (b) Communication
different? (c) Creativity
(a) They aren’t different; they are two different names for (d) Aspiration
the same thing. (e) Research work
(b) The internet is text only, the WWW incorporates 197. Effective Selling Skills depends on-
multimedia. (a) Number of languages known to the DSA
(c) The internet is primarilyused by businesses; the WWW (b) Data on marketing staff
can be used by home users. (c) Information regarding IT market
(d) The WWW is faster than the more archaic internet. (d) Knowledge of related market
(e) None of these
(e) Ability to talk fast
189. Which ports connects special types of music instruments
198. The first CNG train is launched on -
to sound cards?
(a) Delhi-Agra Section
(a) BUS (b) CPU
(b) Rohtak-Chandigarh Section
(c) USB (d) MIDI
(c) Delhi-Chandigarh Section
(e) MINI
(d) Rewari-Rohtak Section
190. The long term objective of marketing is-
(a) Customer satisfaction (e) None of these
(b) Profit maximization 199. Name the Malaysian woman who has become the world's
(c) Cost cutting oldest women(and also the oldest living person, presently).
(d) Profit maximization with customer satisfaction (a) Nurul Yeoh (b) Yati Musdi
(e) None of these (c) Michele Chia (d) Celine Choo
191. Zero-based budgeting (ZBB) means- (e) None of these
(a) A tool of marketing cost 200. Which among the following is the theme of the World
(b) a tool for financial analysis Health Day-2015 ?
(c) each year, budgeting starts from a scratch. (a) Food Safety (b) Aging and Health
(d) a certain percentage of sales (c) Water Safety (d) Small Bite Bit Thread
(e) Both 1 & 2. (e) None of these
y
o
u
rs
m
a
h
534 SBI Bank PO Main Exam 2015

b
o
o
b
.w
o
rd
(1-3) : 9. (d) A’s mother-in-law is B and A is second to the right

p
re
Colours of the sky = ki la fa so ...(i) of B.

s
rainbow colours = ro ki ...(ii) 10. (a) A’s grandchild is C and A is immediate right of C.

s
.c
[from (i) and (ii)] 11. (c) Except C all others are women.

o
\ Colours = ki..... 12. (e) B is the mother of H which is true with respect to

m
sky high rocket = la pe jo ...(iii) the given sitting arrangement.
from (i) and (iii), sky = la 13. (a) F’s grandmother sits to the immediate left of C.
the rocket world = pe so ne (14-15) :
from (i) and (iv), the = so ...(iv) According to given information.
and from (iii) and (iv), rocket = pe
1. (e) colours sky high = ki la jo
2. (c) ‘the’ represents only ‘so’. Dancer Footballer Music lover
3. (e) ‘pe’ represents ‘rocket’.
(4-8) :
Cricketer Player
4. (a) Statements W ³ D < M < P < A = F
Conclusions 14. (b) Possibility case
I. F > D ® It follows.
II. P < W ® It does not follows.
So, only conclusion I follows. Player Dancer Footballer Music lover
5. (d) Statements H ³ M > F < A = B > S
Conclusions
I. H > B ® It does not follow. Cricketer
II. F < S ® It does not follow. Hence, all of the conclusion follows.
Neither Conclusion I nor II follows. 15. (a) Possibility case
6. (b) Statements B > T > Q > R = F
Conclusions Music lover
I. Q ³ F Þ It does not follow because Q > F only.. Dancer Footballer
Cricketer
II. T > F Þ It follows.
Only Conclusion II follows.
7. (b) Statements S = R ³ Q, P < Q Player
\ S = R³ Q > P Hence, only Conclusion I follows.
Conclusions (16-18) :
I. S ³ P Þ It does not follow because S ³ Q and Q > P.. According to given information,
II. R > P Þ It follows.
Only conclusion II follows.
8. (b) Statements S ³ M < Y = Z > F > T Intelligent Boys Professor
Conclusions
I. S > F Þ It does not follow. Girls Leader
II. Y > T Þ It follows. 16. (c) Hence, Conclusions I and II follow.
Only conclusion II follows. 17. (c) Possibility case
(9-13) : Eight persons sitting arrangements areas as follows.
II
(D's Daughter)
Professor
(G's Uncle or E

Boys
(A's Wife D's Girls
Maternal-uncle) D
+ –
H Sister) Intelligent
Leader
(H's Mother A's – –
III
Mother-in-law B F (A's and H
F's Grandmother) Daughter
(+) (–)
G's Sister)
C G Boys Professor
(G's Son A's +
(C's Mother A
GrandChild) A and H Daughter) Intelligent
Girls Leader
(H's Husband Hence, Conclusions II and III follow.
G's Father)
y
o
u
rs
m
a
h
SBI Bank PO Main Exam 2015 535

b
o
o
18. (c) Hence, only Conclusions III follows. argument. If these bone fragments were found in con-

b
(19-23) : After careful analysis of the given input and various junction with some other evidence of the presence of

.w
steps of rearrangement it is evident that in each step one word early hominids, then the evidence from the Swartkans

o
and one number are rearranged. location could be used to support the claim that early

rd
The word are rearranged from left in alphabetical order and the hominids used fire.

p
re
numbers are rearranged from the right in descending order but Hence, statement (4) properly indentifies evidence that

s
in the final step the word get rearranged in alphabetical order in links early hominids to these bone fragments and so

s
.c
reserves manner and number appear in descending order. strengthens the argument.

o
Input: 31 rise gem 15 92 47 aim big 25 does 56 not 85 63 with 30. (b) People having a comfortable lifestyle are found to be

m
moon happier than leading strenuous lifestyle by their height
Step I : aim 31 rise gem 15 47 big 25 does 56 not 85 63 on most contradicts the findings.
with moon 92 31. (c) Clearly, option (c) weaken the findings because suf-
Step II : big aim 31 rise gem 15 47 25 does 56 not 63 with fering from malaria respond well to the traditional
moon 92 85 malaria drugs, while in statement not seems so.
Step III : does big aim 31 rise gem 15 47 25 56 not with 32. (a) The cost of transportation from state A to state C is
moon 92 85 63 more than 30% of the production cost can be directly
Step IV : gem does big aim 31 rise 15 47 25 not with moon drawn from the statement.
92 85 63 56 33. (c) After a cautioned gap of one month people have
Step V : moon gem does big aim 31 rise 15 25 not with 92 started visiting crowded places most substantiates the
85 63 56 47 facts.
Step VI : not moon gem does big aim rise 15 25 with 92 85 34. (c) The air traffic controllers of the city should be sent to
63 56 47 31 refresher courses so that they can improve them self
Step VII : rise not moon gem does big aim 15 with 92 85 63 in handling such situation is the most suitable course
56 47 31 25 of action.
Step VIII : with rise not moon gem does big aim 92 85 63 56 (35 - 40) :
47 31 25 15 We can conclude the following result:
19. (a)
20. (d) Flat No. Agewise decreasing Qualification Name
21. (b)
22. (c) (3) Chess player Power Engineer Amit
23. (c)
(3) Cricket (Regional) Mech. Engineer Manu
24. (b) I. P « N ƒ K +
(+ ) b (–) (2) Football Design Engineer Rohit
J
Using I only, we cannot determine if J is the nephew (5) Tennis Quality Inspector Tarun
or niece of K.
(4) Cricket (National) Mech. Engineer Ambrish
II. P « N ƒ K ∗
(∗ ) b (, )

J 35. (a)
(∗ ) 36. (d)
J is the nephew of K. 37. (c)
25. (a) I. Using I only, we can determine that G stays 38. (d)
on 4th floor. 39. (c)
26. (e) 40. (a)
I. Raju can take 4 to 8 days to complete the work. (41-45) :
II. Raju can take 8 to 10 days to complete the work. Us-
ing both the statements together we can determine A (Finance)
that Raju took 8 days to complete the work.
27. (c) D (CS) F (MCA)
28. (d) Even by using both the statements together we can-
not determine whether B has highest number of stu-
dent or D.
29. (d) Which additional piece of information would
E (MBA) H (M.TECH)
strengthen the argument? The information that
strengthens this argument will be about the subjects
of the argument, not about tangential issues. In this
case, the argument is about early hominids’ use of
fire. Any physical evidence, that links the early homi- B (B.com) C (B.TECH)
nids to the charred bone fragments strengthens the
G (CA)
y
o
u
rs
m
a
h
536 SBI Bank PO Main Exam 2015

b
o
o
41. (c) 42. (a) 43. (a) 44. (a) 45. (b)
æ 45 ö 1

b
46. (e) 47. (e) =ç ´100 ÷ % = 12 %

.w
48. (e) T is grandfather. è 360 ø 2

o
49. (b) S is great grandmother of N. 62. (d) Degree value of expenditure on hockey = 63°

rd
50. (d) The statement mentions that the commercial banks Degree value of expenditure on golf = 36°

p
re
violate a directive issued by the RBI. The remedy is 63 - 36

s
only to make the banks implement the Act. So, none \ Required percentage = ´ 100 = 75%

s
.c
of the courses follows. 36

o
51. (e) Number of women liking 63. (a) Amount spent on basketball exceeds that on tennis

m
CSK = 44% of 45525 = 20031 by
DD = 33% of 36800 = 12144 æ (50 - 45) ö
DC = 30% 0f 56340 = 16902 = `ç ´ 18000000 ÷ = `250000
è 360 ø
MI = 28% of 62350 = 17458
RR = 44% of 48300 = 21252 64. (c) Degree value of the expenditure
RCB = 35% of 35580 = 12453 on football = 54°
KXI = 36% of 56250 = 20250 on cricket = 81°
KKR = 54% of 64000 = 34560 81 - 54
Total = 155050 \ Required percentage = ´ 100
81
155050
\ Required average = =19381.25 ; 19381 27
=
1
´ 100 = 31 %
8 18 3
52. (e) Required percentage 65. (b) Degree value of expenditure on cricket and hockey
Number of women liking RR = (81 + 63)° = 144°
= ´ 100
Number of women liking all teams 144
\ Required amount spent on them =` ´ 2 crore
= 21252/155050 × 100 = 13.7% 360
53. (b) Required number = 45525×36/100 = 16389 = 0.8 crore = `8000000
54. (c) Required ratio = (36800 × 39) : (48300 × 21) = 208 66. (b) Suppose x units are produced each year.
: 147 So, in the year 2006,
55. (c) Number of men liking DC = 45% 56340 = 25353 Total revenue = 1200
Number of men liking RCB = 15% of 355080 = 5337 Þ 12 × x = 1200
25353 Þ x = 100
\ Required percentage = ´ 100 = 475.04% Prifit = 300
5337
\ Cost price = 1200 – 300 = 900
(56-60) :
Stock at the beginning of 900
\ Cost per unit = =9
June = 75 units 100
July = 75 – (350 – 300) = 75 – 50 = 25 units In the year 2005,
August = 25 – (275 – 250) = 25 – 25 = 0 unit Total revenue = 1400
September = 200 – 175 = 25 units Þ 14 × x = 1400
October = 25 – (325 – 300) = 50 units x = 100
November = 25 + (325 – 300) = 50 units Profit = 100
From the above calculation, we get the following table \ Cost price = 1400 – 100 = 1300
Months Quantity Quantity Stock of
Produced sold the beginning 1300
\ Cost per unit = = 13
June 300 350 75 100
July 250 275 25 In the year 2009,
August 200 175 0 Total revenue = 400
September 200 200 25 Þ 8 × x = 400
October 325 300 25 Þ x = 50
56. (c) From the above table, it is clear that in the beginning Profit = 150
of August, the company had the least stock. \ Cost price = 400 – 150 = 250
57. (c) The stock at the beginning of September was 25 250
units. \ Cost per unit = =5
58. (e) Required minimum stock should be 75 units. If we 50
had the stock at the beginnings less than 75 units, In the year 2011,
the company would not have been able to supply in Total revenue = 700
August. Þ 14 × x = 700
59. (a) Þ x = 50
60. (b) The stock at the beginnings of July was 25 units. Profit = 200
61. (a) Percentage of money spent on tennis \ Cost price = 700 – 200 = 500
y
o
u
rs
m
a
h
SBI Bank PO Main Exam 2015 537

b
o
o
500 69. Year New revenue Total cost

b
\ Cost per unit = = 10

.w
50 = Old revenue – Profit

o
Thus, per unit cost is highest in the year 2005. 2004 80% of 700 – 560 700

rd
2005 80% of 1400 = 1120 1300
67. (b) Years Unit price Revenue Total units

p
2006 80% of 1200 = 960 900

re
Revenue 2007 80% of 900 = 720 750

s
=

s
.c
Unit price 2008 80% of 1100 = 880 1100

o
2009 400 120% of 250 = 300
2000 10 700 70

m
2001 14 1400 100 2010 200 120% of 100 = 120
2002 12 1200 100 2011 700 120% of 500 = 600
2003 12 900 75 2012 600 120% of 600 = 720
2004 11 1100 100 2013 800 120% of 400 = 480
2005 8 400 50 2014 900 120% of 600 = 720
2006 10 200 20 In a year when total cost is more than new revenue then
2007 14 700 50 in that year there is loss.
2008 10 600 60 These years are 2004, 2005, 2006, 2007 and 2008
2009 10 800 80 So, there are total 5 yr.
2010 15 900 60 70. (b) Total decrease in revenues
Total = 765 = 20% of (700 + 400 + 1200 + 900 + 1100) = 1060
Total increase in cost
765
\ Average units < ; 70 units = 20% of (250 + 100 + 500 + 600 + 400 + 600) =
11 490
68. (c) Checking option (a), Decrease in cumulative profit
15 - 8 77 = Total decrease in revenues + Total increase in cost
Volatility per unit = = = 0.611 = 1060 + 490 = `1550
126 126
71. (b) Total number of foreign tourists who visited Wagha
11 Border on all the days together
Checking of option (b), = 130 + 190 + 120 + 200 + 220 = 860
Total cost = Revenue – Profit
= (700 + 1400 + 1200+ 900 + 1100 + 400 + 200 860
Therefore, required average = = 172
+ 700 + 600 + 800+ 900) – (0 + 100 + 300 + 150 5
+ 0 + 150 + 100 + 200 + 0 + 400 + 300) 72. (e) Total number of tourists whos visited all the three
= 8900 – 1700 = 7200 places on Tuesday = 70 + 190 + 270 = 530
7200 Total number of foreign tourists who visited Wagha
Average cost per unit = Border on Monday and Wednesday together
11 = 130 + 120 = 250
(1400 - 100) - (200 - 100) 530
Volatility cost per unit = \ Required percentage = ´ 100% = 212%
7200 250
1300 - 100 73. (c) Total number of tourists who visited all the three
= ´ 11 = 1.833
7200 places on Monday = 90 + 130 + 200 = 420
Checking option (c), on Tuesday = 70 + 190 + 270 = 530
on Wednesday = 120 + 150 + 180 = 450
1700 on Thursday = 200 + 230 + 260 = 690
Average profit =
11 on Friday = 140 + 190 + 220 = 550
400 - 0 44 Thus, the number of tourists was second lowest on
\ Profit volatility = = = 2.588 Wednesday.
1700 17
74. (d) Number of tourists who visited Taj Mahal on
11 Thursday = 260
Checking option (d), Number of thourists who visited Golden Temple on
8600 Friday = 140
Average revenue = Therefore, required ratio = 260 : 140 = 13 : 7
11
75. (a) Difference between the number of foreign tourists
1400 - 200 who visited Taj Mahal and Golden Temple
\ Revenue volatility =
8600 on Monday = 200 – 90 = 110
11 on Tuesday = 270 – 70 = 200
on Wednesday = 180 – 150 = 30
132
= = 1.535 on Thursday = 260 – 230 = 30
86 on Friday = 190 – 140 = 50
\ Total profit has highest volatility. Thus, the difference is second highest on Monday.
y
o
u
rs
m
a
h
538 SBI Bank PO Main Exam 2015

b
o
o
(76-80) : Number of girls enrolled in B.Tech. (computer

b
Number of students in the college = 7200 science) = 480

.w
7

o
1080
Number of boys = ´ 7200 = 4200 \ Required percentage = ´ 100 % = 225%

rd
12 480

p
re
5 80. (b) Total number of boys enrolled in B.Tech. (civil) =
Number of girls = ´ 7200 = 3000

s
432

s
12

.c
81. (a) 7x + 6y + 4z = 122 ... (i)
Number of student in B. Tech. (nano technology)

o
4x + 5y + 3z = 88 ... (ii)

m
= 22% of 7200 = 1584 9x + 2y + z = 78 ... (iii)
Number of girls in B. Tech. (computer science) By equation (iii) × 3 – equation (ii),
= 16% of 3000 = 480
Number of boys in B.Tech. (mechanical) = 18% of 4200 27x + 6y + 3z = 234
= 756 4 x + 5 y + 3z = 88
Number of girls in B.Tech. (civil) = 30% of 480 = 144 - - - - ...(iv)
Number of boys in B.Tech. (electronics) = 15% of 4200 23x + y = 146
= 630 By equation (iii) × 4 – equation
Number of boys in B.Tech. (computer science) = 50% of (i),
480 = 240
Number of girls in B.Tech. (aerospace) = 15% of 3000 = 36 x + 8 y + 4 z = 312
4500 7 x + 6 y + 4 z = 122
- - - - ...(v)
3
Number of boys in B.Tech. (civil) = ´ 144 = 432 29 x + 2 y = 190
1
Number of students in B.Tech (electronics) = 24% of 7200 By equation (iv) × 2 – equation
= 1728 (v),
\ Number of girls in B.Tech. (electronics) = 1728 – 630
=1098 46 x + 2 y = 292
29 x + 2 y = 190
12 - - - ...(iv)
Number of boys in B.Tech. (aerospace) = ´ 450 = 1080
5 17 x = 102
Number of boys in B.Tech. (nano technology) = Remaining
Number of boys = 4200–(756+630+240+432+1080) Þ x = 6
= 4200–3138=1062 From equation (iv),
\ Number of girls in B.Tech. (nano technology) 23 × 3 + y = 146
=1584 – 1062 = 522
Þ y = 146 – 138 = 8
Number of girls in B.Tech. (mechanical) = Remaining
Number of girls = 3000 – (480 + 144 + 450 + 1098 + From equation (iii),
522) = 3000 – 2694 = 306 9 × 6 + 2 × 8 + z = 78
Tabular form of above information is shown below. Þ 54 + 16 – z = 78
Subjects Number of Number of Þ z = 78 – 70 = 8
boys girls Clearly, x < y = z
B.Tech. (computer science) 240 480
82. (c)
B.Tech. (mechanical) 756 306
B.Tech. (civil) 432 144 By equation (II) × 2 – equation (I)
B.Tech. (electronics) 630 1098
B.Tech. (aerospace) 1080 450 Number Floor Person
B.Tech. (nano technology) 1062 522
6 Fifth Floor P
Total 4200 3000
5 Fourth Floor T
76. (a) From the table, it is clear that total number of 4 Third Floor V
students enrolled in B.Tech. (meachanical)
77. (c) Number of girls enrolled in B.Tech. (electronics) = 3 Second Floor S
1098 2 First Floor R
1098 1 Ground Floor Q
\ Required percentage = ´ 100%
7200
=15.25% ; 15% From equation (I),
78. (e) Number of girls enrolled in B.Tech. (nano technology) 7 × 8 + 6y = 110
= 522 Þ 6y = 110 – 56 = 54
79. (d) Number of boys enrolled in B.Tech. (aerospace) =
Þ y = 9
1080
y
o
u
rs
m
a
h
SBI Bank PO Main Exam 2015 539

b
o
o
From equation (iii), 86. (d) Let S1 and S2 be the speed of train A and B respectively

b
8 + z = 15 Þ z = 7

.w
Time taken by both the trains in crossing each other.

o
Clearly, x < y > z
450

rd
= 12, S1 + S2 = 37.5

p
1 1 S1 + S2

re
83. (b) I. x = (36) ´ (1296) = 6 ´ 6 = ±6

s
2 4 S1 and S2 can have so many values. Both statement I

s
.c
By equation II × 3 – equation I and II are not sufficient to find speed of train B.

o
87. (d) Area of rectangle = Area of triangle.

m
6 y + 9 z = 99 From the information given in both the statements,
6 y + 5 z = 71
- - - we can find area of triangle or area of rectangle. For
Þ z = 9 finding length, breadth is required, which is not
4z = 28 known.
From equation II, 88. (c) From the statement I.
2y + 3 × 7 = 33 100 ´100
Þ 2y = 33 – 21 = 12 r= = 10%
1000
Þ y = 6
Thus we have,
x £ y< z P = ` 1000, r = 10%, t = 3 years
84. (d) By equation 1 × 5 – II × 8 Hence, C.I. can be determined
40 x + 35 y = 675 From the statement II.
40 x + 48 y = 792
- - - 1000 ´ r ´ 2
S.I. = = 20r
- 13 y = -117 1000

Þ y = 9 éæ r ö
2 ù
From equation I, C.I.= 1000 ç 100 ÷ - 1ú
ê 1 +
êëè ø úû
8x + 7 × 9 = 135
\ C.I. – S.I.
Þ 8x = 135 – 63 = 72
Þ x = 9 é 200r + r 2 ù
From equation III, = 1000 ê 10000 ú - 20r
êë úû
9 × 9 + 8z = 121
Þ 2000r + r2 – 200r = 100
Þ 8z = 121 – 81 = 40 Þ r = 10
Þ z = 5 Hence, C.I. can be determined
Clearly, x = y > z 89. (e) Let the unit’s digit be x and ten’s digit be y and x < y.
85. (e) I. (x + y)3 = 1331
\ Number = 10y + x
Þ x + y = 11
From statement I,
Þ y = 11 – x
y–x=5 ... (i)
From equation III,
From statement II,
x (11 – x) = 28
y+x=7 ... (ii)
Þ 11x – x2 = 28
From (i) and (ii), x, y can be calculated and two digit
Þ x2 – 11x + 28 = 0 number can be found.
Þ x2 – 7x – 4x + 28 = 0 90. (d) Let the distance between A and B be z km.
Þ x (x – 7) – 4 (x – 7) = 0 Again, let speed of boat in still water be x kmph and
Þ (x – 7) (x – 4) = 0 that of stream be y kmph.
Þ x = 7, 4 \ Rate downstream = (x + y) kmph
From equation I Rate upstream = (x – y) kmph
y = 4, 7 From statement I,
From equation II
7 –4 +z =0 Þz =– 3 z
=2 ... (i)
4 –7 +z =0 Þz = 3 x+ y
y
o
u
rs
m
a
h
540 SBI Bank PO Main Exam 2015

b
o
o
From statement II, 19250 - 18500

b
Pepper = = 4%

.w
z 18500
=4

o
x- y
... (ii)

rd
1435 - 1440
= –0.3%

p
we have two equations and three variables, therefore Sugar =

re
1440
both equations are not sufficient.

s
s
3840 - 4240

.c
91. (c) The given number series is based on the following Gold = = 9.43%

o
pattern : 4240

m
13 × 1 + 1 = 14 99. (c) Price Volatility is defined in the question PV
14 × 2 + 2 = 30
H.P. - L.P.
30 × 3 + 3 = 93 =
A.P.
93 × 4 + 4 = 376
H.P. L.P. A.P. PV
376 × 5 + 5 = 1885
Arhar 2300 1500 1912.50 0.42
\ ? = 1885 × 6 + 6 = 11316 Pepper 19500 17400 18622.50 0.112
Hence, number 11316 will replace the question mark. Sugar 1500 1410 1446.25 0.06
Gold 4300 3800 4045 0.124
4 6 9 13.5 20.25 30.375 45.5625
So the lowest price volatility is for sugar.
92. (b)
×1.5 ×1.5 ×1.5 ×1.5 ×1.5 ×1.5 100. (d) Price change which we have calculated previously is
nothing but profit percent or loss percent depending
400 240 144 86.4 51.84 31.104 18.6624 upon the sign. In the first two we have profit percentage
93. (d) which is 26.5 + 4% = 29.5% and the loss percentage
× 0.6 × 0.6 × 0.6 × 0.6 × 0.6 × 0.6 is
9 4.5 4.5 6.75 13.5 33.75 101.25 0.3 + 9.4 = 9.7% then net profit % comes out to be
20.8% and then the average of which gives i.e.
94. (a)
× 0.5 ×1 × 1.5 ×2 × 2.5 ×3 20.8
= 5.2% so the closest answer is (d).
In each term in the series added by 1, 2, 3, ... and 4
then multiplied by 23 101. (b) Delete ‘to’. It is superfluous.
95. (e) 705 + 1 × 23 = 728 102. (a) Replace ‘Yesterday in the night’ by last night.
728 + 2 × 23 = 774 103. (c) Replace ‘will approach’ with ‘will be approaching’.
774 + 3 × 23 = 843 104. (a) Use of whole is superfluous.
843 + 4 × 23 = 935 105. (c) Use of that is superfluous.
935 + 5 × 23 = 1050 106. (c) 107. (a) 108. (b) 109. (e) 110. (b)
\ ? = 1050 + 6 × 23 111. (d) 112. (b) 113. (c) 114. (d) 115. (e)
= 1050 + 138 = 1188 116. (c) 117. (b) 118. (b) 119. (a) 120. (d)
96. (a) Red light flashes 3 times / min. i.e after every 121. (a) 122. (c) 123. (d) 124. (c) 125. (d)
20 seconds. 126. (d) 127. (d) 128. (a) 129. (c) 130. (a)
Green light flashes 5 times in 2 min. i.e. after every 131. (c) 132. (d) 133. (b) 134. (c) 135. (c)
24 seconds.
136. (d) 137. (b) 138. (d) 139. (d) 140. (b)
So, they flash together after every 2 minutes
141. (c) 142. (a) 143. (d) 144. (c) 145. (a)
= 120 seconds 146. (b) 147. (d) 148. (d) 149. (c) 150. (c)
(L.C.M. of 20 & 24 = 120) 151. (b) 152. (c) 153. (d) 154. (e) 155. (d)
Hence, in 1 hour they flash together 60/2 = 30 times 156. (a) 157. (c) 158. (a) 159. (d) 160. (a)
97. (e) Minimum number of bags we have to allocate them in 161. (a) 162. (b) 163. (c) 164. (d) 165. (b)
such a way that we get all the numbers
166. (a) 167. (d) 168. (a) 169. (d) 170. (b)
i.e., 1 + 2 + 4 + 8 + 16 + 32 + 64 + 128
171. (c) 172. (d) 173. (a) 174. (d) 175. (a)
Hence, minimum no. of bags required is 8; having
176. (a) 177. (b) 178. (d) 179. (b) 180. (a)
number of coins 20 , 21 , 2 2 ,...., 27 181. (c) 182. (a) 183. (c) 184. (b) 185. (e)
98. (a) Price change of each commodity is as follows : 186. (e) 187. (d) 188. (b) 189. (d) 190. (d)
191. (c) 192. (b) 193. (a) 194. (e) 195. (a)
2150 - 1700
Arhar = = 26.5% 196. (c) 197. (d) 198. (d) 199. (b) 200. (a)
1700
y
o
u
rs
m
a
h
b
o
IBPS SPECIALIST (I.T.) OFFICER EXAM

o
b
.w
o
Held on 15 Feb, 2016

rd
Based on Memory

p
re
s
s
.c
Reasoning Ability some are facing North. T faces South. Only three people sit to the

o
m
left of T. Only two people sit between T and Q. U sits third to the
right of W. W is neither an immediate neighbour of T nor Q. W
DIRECTIONS (Qs. 1-5) : Study the given information carefully
and answer the given questions. does not sit at any of the extreme ends of the line. Both the
immediate neighbours of S face North. S is not an immediate
L, M, N, O, P, Q, R and S are sitting around a circular table at equal neighbour of W. Only one person sit between Sand P. P faces
distances between each other, but not necessarily in the same same direction as W. Immediate neighbours of Q face opposite
order. Some of the people are facing the centre while some face directions (i.e. if one neighbour faces North then the other South
outside (i.e. in a direction opposite to the centre) . and vice-versa.) Persons sitting at extreme ends face opposite
O sits third to the right of L. Both Q and L face the same
directions. R faces a direction opposite to S.
direction. Only three people sit between P and M. P is neither an
6. How many persons are seated between T and P?
immediate neighbour of L nor O. M faces outside. R sits to the
(a) Four (b) More than four
immediate left of M. Q sits to the immediate right of N. Neither L
nor Q is an immediate neighbour of N. Both the immediate (c) One (d) Three
neighbours of M face the opposite directions. S sits second to (e) Two
the right of R. Both R and Q face the same direction as S. P face a 7. Which of the following pairs represents persons seated at
direction opposite to that of N. the two extreme ends of the line?
1. How many people in the given arrangement face the centre? (a) R, T (b) P, V
(a) One (b) Four (c) P, U (d) Q, R
(c) Five (c) Three (e) R, S
(e) Two 8. Which of the following is true about V with respect to the
2. What is N's position with respect to P? given arrangement?
(a) Immediate left (b) Third to the right (a) Only two people sit between V and Q.
(c) Fourth to the right (d) Second to the left (b) Only one person sits to the right of V.
(e) Immediate right (c) None of the given options is true.
3. Four of the following five are alike in a certain way based on (d) U is one of the immediate neighbours of V.
the given seating arrangement and so form a group. Which
(e) R sits second to the right of V.
is the one that does not belong to that group?
9. Who amongst the following sits to immediate left of R?
(a) L (b) S
(a) T (b) P
(c) N (d) Q
(e) R (c) W (d) Q
4. Which of the following is true regarding Q as per the given (e) S
arrangement? 10. Four of the following five are alike in a certain way based on
(a) Only three people sit between Q and R. the given arrangement and so form a group. Which is the
(b) Q sits second to right of O. one that does not belong to that group?
(c) Q faces outside. (a) R (b) U
(d) None of the given statements is true. (c) P (d) W
(e) Q sits second to the left of P. (e) None of these
5. Who amongst the following sits exactly between M and the
one who sits second to the left of P? DIRECTIONS (Qs. 11-15) : Study the following information to
answer the given questions.
(a) L (b) N
(c) R (d) Q Eight people A, B, C, D, E, F, G and H live on separate floor of an
(e) O 8-Floor building but not necessarily in the same order. Ground
floor is numbered 1, first floor is numbered 2 and so on until the
DIRECTIONS (Qs. 6-10): Read the following information to
topmost floor is numbered eight.
answer the given questions. Only two people live below the floor on which G lives. Only
P, Q, R, S, T, U, V and W are seated in a straight line but not one person lives between G and A. H lives on an odd numbered
necessarly in the same order. Some of them are facing South while floor but not on floor number 7.
y
o
u
rs
542

m
IBPS : Specialist (I.T.) Officer Exam

a
h
Only two people live between H and B. B does not live on 18. As per the given arrangement Q is related to Chinese and R

b
the topmost floor. is related to Japanese in a certain way. To which of the

o
o
A does not live on the lowermost floor. following is P related to in the same way?

b
.w
C lives immediately below D. Neither C nor E lives on floor (a) Sanskrit
number 6.

o
(b) Other than those given as options

rd
11. How many people live between the floors on which A and D (c) Spanish

p
live?

re
(d) English
(a) Three (b) More than three

s
(e) French

s
(c) None (d) Two

.c
19. Four of the following five form a group as per the given

o
(e) One
arrangement. Which of the following that does not belong

m
12. Who lives on the floor immediately below G?
to that group?
(a) F (b) E
(a) Q-Tuesday (b) N-Thursday
(c) D (d) B
(c) S-Sunday (d) M-Saturday
(e) C
13. On which of the following floor numbers does D live? (e) P-Friday
(a) 4 (b) 1 20. On which of the following days does P have a class?
(c) 8 (d) 5 (a) Monday (b) Thursday
(e) 7 (c) Wednesday (d) Tuesday
14. 'Which of the following is true 'with respect to F as per the (e) Sunday
given arrangement? DIRECTIONS (Qs. 21-25): In these questions, two/three
(a) Only three people live between F and B. statements followed by two conclusions numbered I and II have
(b) Only three people live above F. been given. You have to take the given statements to be true even
(c) F lives on the floor number 6. if they seem to be at variance from commonly known facts and
(d) None of the given options is true. then decide which of the given conclusions logically follows
(e) F lives immediately above D. from the given statements disregarding commonly known facts.
15. Who among the following lives on floor number 5?
(a) D (b) C Give answer
(c) E (d) B (a) Only conclusion I is true.
(e) A (b) Only conclusion II is true.
(c) Both conclusions are true.
DIRECTIONS (Qs. 16-20) : Study the following information and
(d) Neither conclusion I nor II is true.
answer the questions.
(e) Either conclusion I or II is true.
Seven friends namely, M, N, O, P, Q, R and S attend different 21. Statements All apartments are houses.
language classes namely, French, English, Spanish, Sanskrit, No apartment is a motel.
Chinese, German and Japanese, not necessarily in the same order, Conclusions I. Some houses being motels is a possibility.
from Monday to Sunday (of the same week).
II. No house is a motel.
M attends a class on Friday. Only two people attend classes
22. Statements All plants are trees.
between M and the one who is learning Sanskrit. R attends a
Some trees are weeds.
class immediately before P. Neither R nor P is learning Sanskrit.
Only one person attends a class between R and the one who is All weeds are shrubs.
learning Spanish. The one who is learning Spanish deos not have Conclusions I. No shrub is a plant.
a class on Monday. N attends a class immediately before the one II. All weeds being trees is a possibility.
who is learning French. M is not learning French. Only one person 23. Statements Some drinks are juices.
has a class between R and Q. O is learning German. Only two All juices are beverages.
people have classes between O and the one who is learning No beverage is a solid.
Chinese. Q is not learning Japanese. Conclusions I. No juice is a solid.
16. Which of the following represent those who have language II. No drink is a solid.
classes immediately before and immediately after N? 24. Statements Some drinks are juices.
(a) P, M (b) Q, M All juices are beverages.
(c) Q, R
No beverage is a solid.
(d) Other than those given as options
Conclusions I. Some drinks are beverages.
(e) R, P
17. Which of the following language is Q learning? II. All drinks are beverages.
(a) Other than those given as options 25. Statements All plants are trees.
(b) Chinese Some trees are weeds.
(c) English All weeds are shrubs.
(d) Spanish Conclusions I. At least some trees are shrubs.
(e) Sanskrit II. All plants being shrubs is a possibility.
y
o
u
rs
543

m
IBPS : Specialist (I.T.) Officer Exam

a
h
DIRECTIONS (Qs. 26-30): Study the given information carefully 32. Statements L < E = A > P; Y > E > R

b
Conclusions I. Y > L II. A > R

o
and answer the questions.

o
33. Statements D < S > L > U; Q < S

b
When a word and number arrangement machine is given an input

.w
Conclusions I. Q < D II. U > Q
line of words and numbers, it arranges them following a particular

o
34. Statements L < E = A > P; Y > E > R

rd
rule. The following is an illustration of input and rearrangement. Conclusions I. P > R II. A < Y

p
(All the numbers are two-digit numbers) 35. Statements N < A = T > Z; R > T; Z < S

re
Input 46 span role 62 79 into main 13 39 deal

s
Conclusions I. R = Z II. Z < R

s
Step I 79 46 role 62 into main 1339 deal span

.c
DIRECTIONS (Qs. 36 & 37): Read the given informations and

o
Step II 62 79 46 into main 1339 deal span role

m
Step III 46 62 79 into 13 39 deal span role main answer the question.
Step IV 39 46 62 79 13 deal span role main into 36. Company X- one of the largest automobile manufacturing
Step V 13 39 46 62 79 span role main into deal companies of the country has started its own dedicated
Step V is the last step of the above arrangement as the Colour Design Department which decides which colour will
intended arrangement is obtained. be given to which model of colour.
As per the rules followed in the given steps, find the appropriate Which of the following may not be a reason for starting this
steps for the given input. colour dedicated department by the said company?
Input 11 east 54 vent kind 35 over 27 71 bowl (a) Although people usually come with a clear idea about
26. How many elements are there between '35' and 'kind' in the the basic features in a car within a stipulated budget,
last step? colour is the trial factor which influences their decision
(a) Two (b) Four prior to purchase.
(c) One (d) Three (b) It has been recently suggested by experts that the
(e) None depth and brightness of colour can be used to
27. What is the position of 'bowl' from the right of '71' in the accelerate the shape and size of a car so that it appeals
second step? to prospective customers.
(a) Seventh (b) Fifth (c) Two years ago a similar department was started by
(c) Sixth (d) Second company R, one of the biggest competitors of company
(e) Third X.
28. Which of the following represent the two consecutive (d) As per market research done by the company many
elements to the immediate right of 'over' in the second last customers have chosen cars of competitor companies
step? only because the colour of the cars of company X did
(a) bowl, vent (b) 11, bowl not match their expectations.
(c) vent, kind (d) 71,11 (e) As per recent research the colour of the car is an
(e) kind, east important aspect in the customer is likely to choose a
car looking at its colour at first sight.
29. Which is the third element to the left of the sixth element
from the left element in the first step? 37. Some preschools in city Y hav recently discontinued the
physical safety training for toddlers which was compulsorily
(a) over (b) 35
provided to staff members for three months after joining.
(c) bowl (d) east
Which of the following may be a reason for the decision
(e) 27
taken by some of the schools in city Y?
30. In step II, which element appears exactly between '27' and
(a) Even after employing trained staff members and
'over'?
providing further training to them, a few parents in
(a) Only 11 (b) Only 71 city Y do not send their children to preschools as
(c) Both kind and 71 (d) Only kind they are worried about the safety of their children.
(e) Both bowl and vent (b) A manual of safety is followed by all preschools of
DIRECTIONS (Qs. 31-35): In these questions, relationship city Y.
between different elements is shown in the statements. The (c) Some teachers, who have been associated with these
statements are followed by conclusions. Study the conclusions preschools for long, have suggested that the training
based on the given statements and select the appropriate answer. which was provided to the institute was a little different
as compared to the training provided.
(a) Only conclusion I is true.
(d) All preschools in city Y have recently started
(b) Only conclusion II is true. employing only those people who have a certification
(c) Both conclusions are true. in physical safety of toddlers and providing additional
(d) Neither conclusion I nor II is true. training simply leads to duplication of what they have
(e) Either conclusion I or II is true. already learnt.
31. Statements N < A = T > Z; R > T; Z < S (e) There is no authority in city Y to regulate whether the
Conclusions I. R > N II. S > Z staff members of schools are provided training.
y
o
u
rs
544

m
IBPS : Specialist (I.T.) Officer Exam

a
h
38. The agriculture experts of country X experimented the usage 41. As per the given information, how is J related to K?

b
of genetically-modified cotton plants to manufacture clothes. (a) Son (b) Son-in-law

o
o
After the post-manufacture analysis, it was decided to (c) Niece (d) Nephew

b
.w
discontinue the use of such modified plants and grow only (e) Daughter-in-law
the natural ones.

o
42. As per the given information, how is R related to Y?

rd
Which of the following statements (if taken to be true) best (a) Nephew (b) Cannot be determined

p
supports the decision of the agricultural exports? (c) Uncle (d) Niece

re
(a) The trend of polyester clothes has dominated the

s
(e) Aunt

s
markets of country X since the past two years but

.c
43. If L is the sister of B, then how is B related to J?
now the demand for cotton clothes has gone up again.

o
(a) ster-in-Iaw (b) Cannot be determined

m
(b) The weather conditions of country X is such that the
(c) Brother (d) Brother-in-law
residents of the country have to resort to only woollen
clothes. (e) Uncle
(c) The high rate of pest attack in chemically grown plants DIRECTIONS (44-48): Study the following information and
as compared to the natural plants makes it unfeasible
answer the questions given below.
and uneconomic to grow.
(d) Other few genetically grown plants have proved to be In a conference 8 people Anshu, Ankit, Kapil, Sachin, Jatin, Nikhil,
successful in country X. Sneha and Rohit from different cities Haldia, Patna, Gaya, Kochi,
(e) Growing of genetically modified plants does not Mumbai, Delhi, Shimla and Pune not necessarily in same order
necessitate the use of humus-rich soil and hence is sitting around a rectangular table.
easy to grow. Three persons are sitting on each longer side and each on
DIRECTIONS (Qs. 39-40): Read the following information and the smaller sides. Sachin is sitting second to the right of the person
the sentences A, B, C, D, E and F given below it carefully and who is from Pune. Jatin is sitting third to the left of the person
answer the given questions. who is from Delhi. Nikhil and Sneha are sitting opposite each
other. Kapil is sitting diagonally opposite the person from Shimla.
Town ABC has seen very few takers. 48% fiats are lying unsold
Ankit is sitting opposite the person who is from Haldia. The
ever since its construction. Prices should be cut by 20% per sq
feet in order to match the ongoing market price of ` 6800 per sq person from Kochi is sitting second to the right of the person
feet. This would bring the buyers back and ideal with this slump from Mumbai and second to the left of Anshu, who is not sitting
in sales. near the person who is from Shimla. Sneha is sitting on the smaller
A. Market price of the flats are controlled by the government side and to the right of the person who is from Shimla. Persons
which cannot be altered by independent authorities. from Mumbai and Kochi are not on the same side of the table. The
B. All present flats are being sold at more than more ` 8000 per person from Kochi is sitting third to the right of the person from
sq feet. Pune, who is not sitting diagonally opposite the person from
C. The township ABC is located far from the main city and has Patna. The person from kochi is third to the left of Kapil. Nikhil
poor transport facilities for the same. sits second to the left of Rohit. The person from Patna sits opposite
D. In order to recover the investments made by the builders, to the person from Mumbai.
they must make a profit of minimum ` 10000 per sq feet per 44. Who is sitting third to the right of the person from Delhi?
flat sold. (a) Jatin (b) Person from Mumbai
E. No buyer is willing to pay a price more than the ongoing (c) Anshu (d) Person from Shimla
market rate. (e) None of these
F. There have been numerous complaints about poor 45. Who is definitely sitting diagonally opposite to Sachin?
construction material and apathy of builders towards the (a) Anshu (b) Kapil
same by people already residing in the township. (c) Person from Patna (d) Person from Gaya
39. Which of the given statements weakens the promise that
(e) Can't be determined
lack of buyers is only because of the price 0 the flats?
46. According to the sitting arrangement what will come in place
(a) Only D (b) A and B
of question mark?
(c) Only A (d) A and E
(e) C and F Sachin : Kochi :: Kapil : ?
40. Which of the following may prove that 30% price cut may (a) Delhi (b) Mumbai
not be a feasible step to be taken by the builders? (c) Gaya (d) Pune
(a) Only D (b) A and B (e) None of these
(c) Only E (d) E and B 47. From which city does Anshu belong?
(e) Only B (a) Patna (b) Haldia
DIRECTIONS (Qs. 41-43): Study the following information and (c) Mumbai (d) Pune
answer the given questions. (e) Can't be determined
48. Which of the following combinations is definitely correct?
Y is the sister of J. L is the wife of J. L has only one son R. K is the (a) Rohit -Pune (b) Sachin - Mumbai
mother of L. K is married to D. D has only one son and only one (c) Anshu-Gaya (d) Sneha - Kochi
daughter. (e) None of these
y
o
u
rs
545

m
IBPS : Specialist (I.T.) Officer Exam

a
h
DIRECTIONS (Qs. 49-50): Read the given information carefully 58. 17 19 42 132 ? 2690

b
(a) 532 (b) 536

o
and answer the given question.

o
(c) 538 (d) 546

b
Ram is 9 m to the south of Ramsey. Ramsey is 5 m to the east of

.w
(e) None of these
Ramandeep. Ramandeep is 4 m to the north of Raman. Ram Singh

o
59. 25 29 67 217 ? 4501

rd
is 3 m west of Raman. Ramanand is 7 m south of Ram Singh. (a) 885 (b) 887

p
RamKunj is 8 m east of Ramanand.

re
(c) 889 (d) 891
49. If Ramada is 5 m to the west of Ram, then what is the distance

s
(e) None of these

s
between Raman and Ramada?

.c
60. 21 38 59 84 113 ?

o
(a) 8 m (b) 9 m

m
(c) 5 m (d) 2 m (a) 138 (b) 140
(e) 6 m (c) 142 (d) 146
50. How far and in which direction is Ramsey from Ramkunj? (e) None of these
(a) 11 m to the south (b) 7 m to the north DIRECTIONS (Qs. 61-65) : In the given questions, two equations
(c) 11 m to the north (d) 7 m to the south numbered I and /I are given. Solve both the equations and mark
(e) 11 m to the west the appropriate answer.

Quantitative Aptitude (a) x> y (b) x > y (c) x < Y


(d) Relationship between x and y cannot be determined
DIRECTIONS (Qs. 51-55): What approximate value will come (e) x<y
in place of question mark (?) in the given questions? (You are 61. I. 6x2 + 25x + 24 = 0 II. 12y2 + 13y + 3 = 0
not expected to calculate exact value.) 62. 2
I. 12x – x – 1= 0 II. 20y2 – 41y + 20 = 0
63. 2
I. 10x + 33x + 27 = 0 II. 5y2 + 19y + 18 = 0
51. 105.27% of 1200.11 + 11.80% of 2360.85 = 21.99% of ? +
1420.99 64. I. 15x2 – 29x – 14 = 0 II. 6y2 – 5y – 25 = 0
65. 2
I. 3x – 22x + 7 = 0 II. y2 – 20y + 91 = 0
(a) 500 (b) 240
(c) 310 (d) 550 DIRECTIONS (Qs. 66-70): In each of the given questions, one
(e) 960 questions and two statements numbered I and II are given. you
52. 0.98% of 7824 + 4842 ¸ 119.46 – ? = 78 have to decide whether the data given in both statements are
(a) 30 (b) 60 sufficient to answer the question or not. Read both the satements
(c) 40 (d) 50 and give answer
(e) 70 (a) if the data in statement I alone are sufficient to answer the
53. (41.992 – 18.042) – ? = 13.112 –138.99 question, while the data in statement II alone are not sufficient
(a) 4004 (b) 1200 to answer the question.
(c) 1720 (d) 8432 (b) if the data in statement II alone are sufficient to answer the
(e) 1410 question, while the data in statement I alone are not sufficient
54. 24.962 / (34.11 + 20.05) + 67.96 + 89.11 = ? to answer the question.
(a) 884 (b) 546 (c) if the data either in statement I alone or in statement II alone
(c) 252 (d) 424 are sufficient to answer the question.
(e) 170 (d) if the data in both statements I and II together are not
sufficient to answer the question.
55. (2025.11) × (256.04) + (399.95) × (?) = 33.98
(e) if the data in both the statements I and II together are
× 40.11 necessary to answer the question.
(a) 1682 (b) 1024 66. By how many years is Rasika younger than her brother
(c) 1582 (d) 678 Sunil?
(e) 1884 I. Ratio between Rasika's present age and Sunil's age
DIRECTIONS (Qs. 56-60): What should come in place of question after four years is 5 : 7 respectively.
mark (?) in the fol/owing number series? II. Ratio between Rasika's age four years ago and Sunil's
present age is 2 : 3 respectively.
56. 13 13 19 43 103 ? 67. What is the quantity of milk in 80 litres of mixture of milk and
(a) 221 (b) 227 water?
(c) 223 (d) 217 I. If 8 litres of mixture is replaced by equal quantity of
(e) None of these water the ratio of milk and water in the mixture becomes
57. 27 13 12 16.5? 75 27 : 13 respectively.
(a) 30.5 (b) 31.5 II. If 16 litres of mixture is replaced by equal quantity of
(c) 37.5 (d) 39.5 milk, the ratio of milk and water in the mixture becomes
(e) None of these 4 : 1 respectively.
y
o
u
rs
546

m
IBPS : Specialist (I.T.) Officer Exam

a
h
68. Neeraj invested certain amount in schemes A and B for 2 75. The radius of a cylinder is 5 m more than its height. If the

b
curved surface area of the cylinder is 792 m2. What is the

o
ears in the ratio of 3 : 5 respectively. The schemes A and B

o
offer compound interest compound annually and simple interest volume of the cylinder? (in m3)

b
.w
respectively. What is the amount invested in scheme A? (a) 5712 (b) 5244
(c) 5544 (d) 5306

o
I. Rate of interest offered by scheme A is 20% per annum

rd
and the rate of interest offered by scheme B is 25% (e) 5462

p
less than that offered by scheme A. 76. A dealer marked the price of an item 40% above the cost

re
II. Amount of interest accrued from scheme B is more price. Once he gave successive discounts of 20% and 25%

s
s
than the amount of interest accrued from scheme A by to a particular customer. As a result, he incurred a loss of

.c
` 448. At what price did he sell the item to the mentioned

o
` 900.

m
69. Two friends X and Y start running towards each other at the customer?
same time from points A and B respectively and meet after (a) ` 2416 (b) ` 2352
135 minutes. At what speed is X running? (c) ` 2268 (d) ` 2152
I. Point B is 45 km away from point A and speed of X is (e) ` 2578
150% of the speed of Y. 77. The respective ratio between Parul's present age and Rohit's
II. Distance covered by Y was 18 km. present age is 7: 5. The sum of their ages 5 years from now
70. What is the cost of painting two adjacent walls of a hall will be 94. After how many years, Rohit's age will be equal to
having no door or window at `450 per m2? Parul's present age?
I. Length and breadth are in the ratio of 3 : 2 respectively. (a) 21 (b) 7
II. Perimeter of the hall is 50 m and height is one-fourth of (c) 14 (d) 18
the perimeter. (e) 24
71. The speed of the boat in still water is 5 times the speed of 78. 35 kg of a type of sandal powder (type A) which costs ` 614
the current. It takes 1.1 hours to row to point B from point A per kg was mixed with certain amount of another type of
downstream. The distance between point A and point B is sandal powder (type B), which costs ` 695 per kg. Then the
13.2 km. How much distance will it cover in 312 minutes mixture was sold at ` 767 per kg and 18% profit was gained.
upstream? What was the amount of type B sandal powder in the
(a) 43.2 km (b) 48 km mixture?
(c) 41.6 km (d) 44.8 km
(a) 24 kg (b) 28 kg
(e) 40 km
(c) 32 kg (d) 36 kg
72. 24 men can complete a piece of work in 15 days. 2 days after
the 24 men started working, 4 men left the work. How many (e) 20 kg
more days will the remaining men now take to complete the 79. Ashok left from place A (towards place B) at 8 am and Rahul
remaining work? left from place B (towards place A) at 10 am. The distance
between place A and place B is 637 km. If Ashok and Rahul
3 4 are travelling at a uniform speed of 39 km/h and 47 km/h
(a) 15 (b) 16
5 5 respectively, at what time will they meet?
2 4 (a) 5 : 30 pm (b) 4 : 30 pm
(c) 11 (d) 10 (c) 5 : 00 pm (d) 4 : 00 pm
5 5
(e) 3 : 30 pm
1 80. A started with an investment of ` 28000. After 2 months, B
(e) 14
5 joins with ` 20000 and after another two months C joins with
73. In a primary school, the average weight of male students is ` 18000. At the end of 10th month from start of the business,
65.9 kg and the average weight of female students is 57 kg. if B withdraws ` 2000 and C withdraws ` 2000 what is the
If the average weight of all the students (both male and respective ratio in which profit should be distributed among
female) is 60.3 kg and the number of male students in the A, B and C at the end of the year?
school is 66, then what is the number of female students in (a) 12 : 7 : 5 (b) 12 : 9 : 5
the school? (c) 12 : 6 : 3 (d) 14 : 7 : 5
(a) 154 (b) 162 (e) 11 : 9 : 7
(c) 168 (d) 180 81. The metal to be used for covering a cylinder having external
(e) 112 radius 5 cm, height 21 cm and thickness 1 cm is to be cast
74. Shashi had a sum of money. Two-third of the total money he from a cylinder. What should be the height of the cylinder
invested in scheme A for 6 years and rest of the money he of radius 3 cm from which this casting can be done?
invested in scheme B for 2 years. Scheme A offers simple (a) 12 cm (b) 39 cm
interest at a rate of 12% per annum and scheme B offers (c) 21 cm (d) 20 cm
compound interest (compounded annually) at a rate of 10% (e) 18 cm
per annum. If the total sum obtained from both the schemes 82. A, B and C can alone complete a work in 15, 25 and 30 days
is ` 2805, what was the total amount invested by him in respectively. A and B started the work and after some days A
scheme A and scheme B together?
is replaced by C. Now the work is completed in a further of
(a) ` 1500 (b) ` 5100
(c) ` 1000 (d) ` 2000 4
6 days. How much of the total work did B did?
(e) ` 1464 11
y
o
u
rs
547

m
IBPS : Specialist (I.T.) Officer Exam

a
h
4 4 Company P 150 190 220 160 240 310

b
(a) (b)

o
11 15 Company Q 130 100 160 120 180 210

o
b
5 5 86. What is the difference between total number of vehicles

.w
(c) (d) manufactured by company P in 2011, 2012 and 2014 together
12 11

o
and company Q in 2012, 2013 and 2014 together? (in

rd
2
thousands)

p
(e)

re
13 (a) 120 (b) 210

s
83. An article is marked at `18,000. A trader bought it at succes- (c) 100 (d) 270

s
.c
sive discounts of 25% and 10% respectively. He spent `1,350
(e) 180

o
on its transportation to his shop and then sold the article

m
for `15,000. What is trader's profit% in the whole transac- 87. What is the average number of vehicles manufactured by
tion? company Q over six years? (in thousands)
(a) 170 (b) 150
2 (c) 90 (d) 60
(a) 16 % (b) 28%
3 (e) 130
1 88. What is the percentage decrease in number of vehicles
(c) 30% (d) 11 % manufactured by company from 2011 to 2012?
9
(e) 20% 3 3
84. A person lent out certain sum on simple interest and the (a) 45 % (b) 33 %
11 11
same sum on compound interest at a certain rate of interest
per annum. He noticed that the ratio between the difference 6 3
of compound interest and simple interest of 3 years and 2 (c) 26 % (d) 27 %
19 11
years is 16 : 49. The rate of interest per annum is
1 4
(a) 9% (b) 8 % (e) 33 %
7 11
89. Out of the number of vehicles manufactured by company
1
(c) 13 % (d) 7% Pin 2013, 15000 pieces were found defective and out of the
3 number of vehicles manufactured by company Q in 2014,
1 10000 pieces were found defective. What is the respective
(e) 6 % ratio of non-defective vehicles manufactured by company
4
85. A box contains 2 blue, 3 green and 5 red balls. If three balls P in 2013 and Q in the 2014?
are drawn at random, what is the probability that all balls are (a) 9 : 8 (b) 11 : 4
different in color? (c) 3 : 8 (d) 5 : 8
3 1 (e) 7 : 4
(a) (b) 90. In year 2015, there was an increase of 30% in number of
10 4
vehicles manufactured by company P as compared to
3 4 vehicles manufactured by same company in the year 2010.
(c) (d)
7 11 What is the total number of vehicles manufactured by the
same company in the year 2015?
2
(e) (a) 247 (b) 297
9 (c) 211 (d) 310
DIRECTIONS (Qs. 86-90): Study the following graph and (e) 283
answer the given questions. DIRECTIONS (Qs. 91-95): Refer to the pie-chart and the table
Number of Vehicles Manufactured By and answer the given questions.
Two Companies during Six Years (in thousands) Distribution of Total Number of Cellular Phones (Both Nokia
350 and Samsung) Sold by Six Stores in October
Number of Vehicles Manufactured

300
U 12% P 14%
250
200
Q 10%
150 T 20%
100 R 16%
50
S 28%
0
2009 2010 2011 2012 2013 2014
y
o
u
rs
548

m
IBPS : Specialist (I.T.) Officer Exam

a
h
96. What was the profit of all countries together in year 2015 if

b
Respective Ratio of Number of Nokia Cellular Phone the total imports of all the countries together was ` 385

o
o
Store Sold to the Number of Samsung Cellular Phones Sold crore?

b
(a) 125 (b) 160

.w
P 4:3
(c) 280 (d) 240

o
Q 3:1

rd
(e) 200

p
R 5:4 97. If the respective ratio of export to import in country S and

re
S 7:6 country U is 1 : 2 and 4 : 1 in the year 2011, then what is the

s
s
1:4 total imports of country U and S together in that particular

.c
T
year? (in ` crore)

o
U 11 : 10

m
(a) 52 (b) 22
91. What is the average number of Nokia cellular phones sold (c) 36 (d) 96
by stores P, R, S and T together? (e) 44
(a) 1007 (b) 1048 98. If the export of country P in the year 2016 is 20% more than
(c) 3908 (d) 1006 the total exports of country Q in 2014 and export of country
(e) 996 T in 2013 together, then what was the profit of P in the year
92. Number of Nokia cellular phones sold by store R is what 2016 if its imports were ` 92 crore for that year? (in ` crore)
percent more than the total number of Samsung cellular (a) 10 (b) 58
phones sold by stores P and Q together? (c) 22 (d) 46
(e) 34
1 5 3 99. By what percent the average export of country T over all the
(a) 23 % (b) 19 % (c) 20 %
17 17 17 given years more than the average export of country R over
all the given years?
11 24
(d) 17 % (e) 4 %
17 119 7 1
(a) 13 % (b) 9 %
93. What is the central angle corresponding to total number of 11 11
cellular phones (both Nokia and Samsung) sold by store S? 5 7
(a) 99.2° (b) 93.6° (c) 13 % (d) 4 %
(c) 100.8° (d) 97.4° 7 11
(e) 101.2° 1
94. What is the respective ratio between number of Nokia (e) 12 %
cellular phones sold by store S and total number of Samsung 7
cellular phones sold by stores T and U together? 100. What is the percent increase in the exports of all the countries
(a) 43 : 72 (b) 49 : 76 together in the year 2012 to 2014? (Rounded off to two digits
(c) 43 : 76 (d) 49 : 72 after decimal?
(e) None of these (a) 88.99 (b) 72.39
95. Total number of cellular phones (both Nokia and Samsung)
sold by stores Q increased by 15% from October to (c) 38.89 (d) 62.89
November and total number of cellular phones (both Nokia (e) 40.60
and Samsung) sold by store T increased by 5% from October
to November. What was the total number of cellular phones English language
sold by stores Q and T together in November?
(a) 3540 (b) 3720 DIRECTIONS (Qs. 101-110) : Read the followling passage
(c) 3640 (d) 3420 carefully and answer the given questions. Certain words/phrases
(e) 3880
have been given in bold to help you locate them while answering
DIRECTIONS (Qs. 96-100): Study the table and answer the some of the questions.
given questions.
As the government prepare to empty its filling candidates and
Total Exports of Six Countries over heads for the hot and dusty plans to solicit votes. It is visibly
Five Years (in ` crore) exuding optimism about the economy. According to its non-elected
Year representatives, all the lead indicators seem to be showing signs
2011 2012 2013 2014 2015 of a revival with the first glimmer of some incipient growth pushing
Country
through the enveloping groom. Steel, cement, auto, fast moving
P 20 40 60 45 90
consumer goods (such as soaps and detergents), food items,
Q 30 25 15 50 100 beverages, volume’ of goods moved by the railways have all
R 50 55 70 90 65 shown some improvement in January after having shrunk in the
S 45 60 20 15 25 previous two months.
But before we start congratulating the government for its
T 60 50 55 100 110 excellent economic management, let’s hit the pause button for a
U 24 40 60 75 120 moment. How much of the Indian economy’s resilience is owed to
governmental intervention? Ok, is there a strategy at all? One of
Note: Profit = Exports - Imports
y
o
u
rs
549

m
IBPS : Specialist (I.T.) Officer Exam

a
h
the economy’s mainstays for over a decade has been service. 102. Which of the following is most nearly the opposite in meaning

b
This contributes to over 50% of the country’s GDP and has been to the word 'Exuding' as used in the passage?

o
o
providing enormous growth impulse over the past few years. If (a) Excluding (b) Displaying

b
.w
you were to listen to the government representatives, it would (c) Percolating (d) Closing
seem as if they had foreseen the coming age of services and had

o
(e) Concealing

rd
designed that structure.The truth is somewhat different. There 103. Which of the following is most nearly the same in meaning

p
are many reasons behind the extraordinary growth of services. to the word 'Apposite' as used in the passage?

re
One of the reasons is the kind of elaborate rent-seeking structures

s
(a) Appropriate (b) Opposite

s
erected by the government in the manufacturing sector. Any

.c
(c) Further (d) Soft

o
person wanting to set-up a manufacturing facility in India still has (e) Believable

m
to fill a large number of outstretched palms, making the operations 104. According to the author, one of the reasons why India's
costly from day one. economy managed to sail through the economic downturn
Here's another unique aspect of the economy for which was
politicians routinely take credit. One of the saving graces for the A. There was demand for Indian goods from domestic
Indian economy during this episode of the downturn is the safety consumers even when internationally it had declined.
net expected to be provided by Indian consumers, even as the
B. The politicians had foreseen the trouble and could
international economy winds down and eschews consumption of
prepare the country to efficiently deal with it.
goods made in India.
This has had a deleterious impact on Indian exports, leading C. It was least affected by the economic downturn.
many exporters to scale down their operations and restructure (a) Only B (b) Only C
their businesses. Fortunately, for the planners and the (c) A and B (d) Only A
administrators, the impact of the global slowdown is likely to be (e) A and C
cushioned, to a large extent, by the gigantic Indian domestic 105. Which of the following is the central theme of the passage?
market, which will continue consuming and providing the growth (a) Social contract-A Must in Every Country
push to the economy. (b) Hurdles Created by the government for the Common
Again, it's not as if some wise person in government woke Man
up one morning and presciently decreed that hence forth the (c) Government's Campaign to Once Again Regain
country would focus only on the domestic markets. The Confidence of the Masses
government has always felt that exports should be the apposite (d) Economic Growth-A Result of Natural Forces rather
strategy for economic growth, just like some of the other emerging than the government
countries. (e) Service Industry-The Backbone of Every Economy
Guess what? Exporters also have to manufacture and that is across the Globe
quite an endurance test in India. Plus, the intricate structure built 106. The author attributes the improvement (or maintenance of)
around promoting exports also worked as a huge deterrent. The the economic condition of the country to
government also did not quite see exports as an alternative, viable A. The government, as it could foresee a dwindle in the
economic growth model till the Southeast Asian success story
economy and could take suitable measures to tackle it.
burst on to the scene. Hence, till then exports did not quite get the
B. Appropriate policies to push people to spend more.
required push. So, no grand design here too.
C. The Economy for it is surviving on its own either
Unlike in USA and various other European economies, where
the government provides unemployment benefits as part of their through consumption or saving.
social contract, Indians have to fend for themselves. In the current (a) Only B (b) Only C
downturn, for example, many Indians-especially in the urban and (c) A and B (d) Only A
semi-urban settlements-are wary of spending because of (e) A and C
uncertainties surrounding their jobs. This has impacted 107. Which of the following is most nearly the same in meaning
consumption but, conversely, is bound to improve the savings to the word 'Eschews' as used in the passage?
rate. (a) Faces (b) Avoids
The credit, therefore, should go to the Indian citisen who, (c) Accepts (d) Bestows
despite the varous hurdles and inconveniences, is using his (e) Ridicules
ingenuity to improve his lot at all times. This collective strength 108. Which of the following is most nearly the opposite in meaning
has not been forged by some steely policy push, but has to the word 'Deleterious' as used in the passage?
developed by default, almost in line' with Charles Darvins’s theory (a) Positive (b) Harmful
of survival. (c) Disastrous (d) Zilch
101. According to the author, one of the reasons behind services (e) Additional
flourishing in our country is 109. Which of the following correctly explains the phrase 'Wary
(a) financial support provided by the government of’ as used in the passage regarding spending behaviour of
(b) funding by theele citizens?
(c) availability of a pool of experts from developed
(a) Careless (2) Miserly
countries in this sector
(c) Upset about (d) Cautious about
(d) availability of young working population
(e) Other than those given as options (e) Fearless in
y
o
u
rs
550

m
IBPS : Specialist (I.T.) Officer Exam

a
h
110. Which of the following is not true in the context of the (a) After weeks of talks

b
passage? (b) the two parties failed to resolving

o
o
(a) Traditionally the government did not believe exports (c) their differences and the makers eventually

b
to be providing a boost to the economy.

.w
(d) decided to look for an alternative
(b) Most unemployed Indians today reap the benefits (e) No error

o
rd
provided under social contracts. 118. The Minister said that the State was facing/an acute financial

p
(c) The author is a firm believer of the fact that the crunch and feared that/the government might be compelled

re
government has played the smallest role in supporting to take/loans to pay the salaries of its employees.

s
s
the economy of the country. (a) The Minister said that the State was facing

.c
(d) The government is portraying a rosy picture of the

o
(b) an acute financial crunch and feared that

m
economy to gain people's confidence.
(c) the government might be compelled to take
(e) All the given statements are true.
(d) loans to pay the salaries of its employees
DIRECTIONS (Qs. 111-115): Which of the phrases given below (e) No error
the sentence should replace the word/phrase given in bold in 119. States dependent on oil and gas revenues/are growing
the sentence to make it grammatically correct? If the sentence is increasingly anxious about/the ripple effect that fading of
correct as it is given and no correction is required, select 'No prices/may have on their local economies,
correction required' as your answer. (a) States dependent on oil and gas revenues
111. People with persistent headaches are hence as likely to (b) are growing increasingly anxious about
develop bed's palsy even as research amount to other factors (c) the ripple effect that fading of prices
which could increase the risk of conditions like diabetes. (d) may have on their local economies
(a) accounting at (b) accounting towards (e) No error
(c) account for (d) accounted at 120. Though the year witnessed/a rise in crimes against women,/
(e) No correction required the police claimed that their changed images has given/
112. The actor and the producer has part ways due to differences confidence to women to come forward and lodge complaints.
over communication. (a) Though the year witnessed
(a) have parted ways (b) have been way parting (b) a rise in crimes against women,
(c) have parting away (d) have part ways (c) the police claimed that their changed images has given
(e) No correction required (d) confidence to women to come forward and lodge
113. The State is reeled from the impact of acute financial crisis complaints
around the world. (e) No error
(a) reeled at (b) reeling away 121. Although ghost pepper is no longer/the most hottiest chilli
(c) reel for (d) reeling from in the world, as India's speciest export/ it packs quite a punch.
(e) No correction required (a) Although ghost pepper is no longer
114. The reading club soon becoming a moving part for literature (b) the most hottiest chilli in the world
enthusiasts. (c) as India's speciest export
(a) soon became (b) becoming soon
(d) it packs quite a punch
(c) was became soon (d) soon is become
(e) No error
(e) No correction required
122. Migraines may doubles/the risk of a/nervous system
115. Having been confined to their homes for a week after a
leopard was sighted, the villagers grew restless and condition that/causes facial paralysis.
frustrated. (a) Migraines may doubles
(a) As confine to (b) Have being confining at (b) the risk of a
(c) Been confining at (d) Being confined for (c) nervous system condition that
(e) No correction required (d) causes facial paralysis
(e) No error
DIRECTIONS (Qs. 116-125): Read the following sentences to 123. The police tactfully handed/ the farmers' agitation for/higher
find out whether there is any grammatical mistake/error in them. prices which is created/law and order problem in district
The error, if any, will be in one part of the sentence. Mark the part every year.
with the error as your answer. If there is no error, mark 'No error' as (a) The police tactfully handed
your answer. (Ignore the errors of punctuations If any) (b) the farmers’ agitation for
(c) higher prices which is created
116. The beautician listed out/a few quick tips that/one needs to
keep in mind/in order to get that perfect holiday radiant. (d) law and order problem in the district every year
(a) The beautician listed out (e) No error
(b) a few quick tips that 124. For the past one week,/people had stop working/out of their
(c) one needs to keep in mind houses,/fearing a sudden attack.
(d) in order to get that perfect holiday radiant (a) For the past one week,
(e) No error (b) people had stop working
117. After weeks of talks,/the two parties failed to resolving / (c) out of their houses
their differences and the makers eventually/decided to look (d) fearing a sudden attack
for an alternative. (e) No error
y
o
u
rs
551

m
IBPS : Specialist (I.T.) Officer Exam

a
h
125. Mock drills were/carried out by/security agencies at/various (a) fallen; negligible (b) risen; visible

b
locations in the city. (c) grown; create (d) soared; remote

o
o
(a) Mock drills were (b) carried out by (e) collapsed; much

b
133. As the country as a whole more than half the population

.w
(c) security agencies at
(d) varous locations in the city ......... regular electricity as connections to the national god

o
rd
(e) No error are ........... and generators are expensive.

p
(a) deficient; dependable (b) want; running

re
DIRECTIONS (Qs. 126-130): Rearrange the following six (c) lack; unenviable (d) short; faded

s
sentences A, B, C, D, E and F in the proper sequence to form a

s
(e) requiring; fumy

.c
meaningful paragraph, then answer the given questions. 134. It is .......... that 400 Indian passengers will want to fly in or

You might also like